You are on page 1of 561

Di erential

Calculus
With Sessionwise Theory & Exercises
Di erential
Calculus
With Sessionwise Theory & Exercises

Amit M. Agarwal
ARIHANT PRAKASHAN (Series), MEERUT
All Rights Reserved

© AUTHOR
No part of this publication may be re-produced, stored in a retrieval system or
by any means, electronic mechanical, photocopying, recording, scanning, web or
otherwise without the written permission of the publisher. Arihant has obtained
all the information in this book from the sources believed to be reliable and true.
However, Arihant or its editors or authors or illustrators don’t take any responsibility
for the absolute accuracy of any information published, and the damages or loss
suffered thereupon.
All disputes subject to Meerut (UP) jurisdiction only.

Administrative & Production Offices


Regd. Office
‘Ramchhaya’ 4577/15, Agarwal Road, Darya Ganj, New Delhi -110002
Tele: 011- 47630600, 43518550
Head Office
Kalindi, TP Nagar, Meerut (UP) - 250002 Tel: 0121-7156203, 7156204
Sales & Support Offices
Agra, Ahmedabad, Bengaluru, Bareilly, Chennai, Delhi, Guwahati,
Hyderabad, Jaipur, Jhansi, Kolkata, Lucknow, Nagpur & Pune.

ISBN : 978-93-25298-65-1

PO No : TXT-XX-XXXXXXX-X-XX
Published by Arihant Publications (India) Ltd.
For further information about the books published by Arihant, log on to
www.arihantbooks.com or e-mail at info@arihantbooks.com
Follow us on
PREFACE

“YOU CAN DO ANYTHING IF YOU SET YOUR MIND TO IT, I TEACH CALCULUS
TO JEE ASPIRANTS BUT BELIEVE THE MOST IMPORTANT FORMULA IS
COURAGE + DREAMS = SUCCESS”

Iit is a matter of great pride and honour for me to have received such an overwhelming response to
the previous editions of this book from the readers. In a way, this has inspired me to revise this book
thoroughly as per the changed pattern of JEE Main & Advanced. I have tried to make the contents
more relevant as per the needs of students, many topics have been re-written, a lot of new problems
of new types have been added in etcetc. All possible efforts are made to remove all the printing
errors that had crept in previous editions. The book is now in such a shape that the students would
feel at ease while going through the problems, which will in turn clear their concepts too.
A Summary of changes that have been made in Revised & Enlarged Edition
— Theory has been completely updated so as to accommodate all the changes made in JEE Syllabus &
Pattern in recent years.
— The most important point about this new edition is, now the whole text matter of each chapter has
been divided into small sessions with exercise in each session. In this way the reader will be able to go
through the whole chapter in a systematic way.
— Just after completion of theory, Solved Examples of all JEE types have been given, providing the
students a complete understanding of all the formats of JEE questions & the level of difficulty of
questions generally asked in JEE.
— Along with exercises given with each session, a complete cumulative exercises have been given at the
end of each chapter so as to give the students complete practice for JEE along with the assessment of
knowledge that they have gained with the study of the chapter.
— Last 10 Years questions asked in JEE Main & Adv, IIT-JEE & AIEEE have been covered in all
the chapters.
However I have made the best efforts and put my all calculus teaching experience in revising this
book. Still I am looking forward to get the valuable suggestions and criticism from my own
fraternity i.e. the fraternity of JEE teachers.
I would also like to motivate the students to send their suggestions or the changes that they want
to be incorporated in this book. All the suggestions given by you all will be kept in prime focus at
the time of next revision of the book.

Amit M. Agarwal
CONTENTS
1. ESSENTIAL MATHEMATICAL TOOLS 1-24
LEARNING PART Session 4
Session 1 — Quadratic Expression
— Basic Definitions — Non-Negative Function

Session 2 PRACTICE PART


— Intervals — Chapter Exercises
— Modulus or Absolute Value Function
Session 3
— Number Line Rule
— Wavy Curve Method

2. DIFFERENTIATION 25-96
LEARNING PART Session 6
Session 1 — Logarithmic Differentiation
— Geometrical Meaning of the Derivative — Differentiation of Infinite Series
— Derivative of f (x) from the First Principle or ab- Session 7
Initio Method — Differentiation of a Function w.r.t. Another
— Rules of Differentiation Function
Session 2 Session 8
— Chain Rule — Higher Derivatives of a Function
Session 3 Session 9
— Differentiation of Implicit Functions — Differentiation of a function given in the form
Session 4 of a Determinant
— Differentiation of Inverse Trigonometric Session 10
Functions — Derivation form inverse function
— Graphical Approach for Differentiation of
Inverse PRACTICE PART
— JEE Type Examples
Session 5
— Chapter Exercises
— Differentiation of a function in
Parametric Form
3. FUNCTIONS 97-200
LEARNING PART Session 7
Session 1 — Periodic Functions
— Functions
Session 8
Session 2 — Mapping Functions
— Domain
Session 9
— Algebraic Functions — Identical Functions
Session 3 Session 10
— Transcendental Functions — Composite Functions
Session 4 Session 11
— Piecewise Functions — Inverse of a Function
Session 5 Session 12
— Range — Miscellaneous Problems of Functions
Session 6 PRACTICE PART
— Odd and Even Functions
— JEE Type Examples
— Chapter Exercises

4. GRAPHICAL TRANSFORMATIONS 201-244


LEARNING PART — f (x) Transforms to ïf (ïxï), i.e.
Session 1 f (x) ®ïf (ïxï)ï
— When f (x) Vertically transforms to f (x) ± a — To draw the graph of ïyï=f (x) when
y=f (x) is given
Session 2
— When f (x) horizontally transforms to Session 4
f (x±a), where a is any positive constant — To draw y=f ([x]) when the graph of
When f (x) transforms to a f (x) or —1 y=f (x) is given
— af (x)
— When f (x) transforms to f (ax) or f (x/a) — When f (x) transforms to y =f ({x}) where
— When f (x) Transforms to f (–x) {×} denotes fractional part of x, i.e. {x} =
— When f (x) Transforms to –f (x) x–[x] or f (x)® f ({x})
— When f (x) and g (x) are two functions and
Session 3 are transformed to their sum
— To draw y =÷ f (x)÷ , when y = f (x) is given 1
— When f (x) transforms to f (x)® —— =h (x)
— To draw y = f (÷ x÷), when f (x) is given f (x)
— To draw the graph for y=f (x) × sin x when PRACTICE PART
graph of y = f (x) is given — JEE Type Examples
— When f (x) and g (x) are given, then find h(x) — Chapter Exercises
= max (f (x), g (x)) or h(x) = min (f (x), g (x))

5. LIMITS 245-310
LEARNING PART Session 4
Session 1 — Miscellaneous Form
— Definition of Limits Session 5
— Indeterminant Form — Left Hand and Right Hand Limit
— L’Hospital’s Rule Session 6
— Evaluation of Limit — Use of Standard Theorems/Results
Session 2 — Use of Newton-Leibnitz’s Formula in Evaluating
— Trigonometric Limits the Limit
Session 3
PRACTICE PART
— Logarithmic Limits
— JEE Type Examples
— Exponential Limits
— Chapter Exercises

6. CONTINUITY AND DIFFERENTIABILITY 311-396


LEARNING PART Session 5
Session 1 — Intermediate Value Theorem
— Continuous Function Session 6
— Continuity of a Function at a Point — Differentiability : Meaning of Derivative
Session 2 Session 7
— Continuity in an Interval or Continuity at — Differentiability in an Interval
End Points
Session 3 PRACTICE PART
— Discontinuity of a Function — JEE Type Examples
— Chapter Exercises
Session 4
— Theorems on Continuity
— Continuity of Composite Function
7. dy/dx AS A RATE MEASURER & TANGENTS, NORMALS 397-458
LEARNING PART Session 4
Session 1 — Angle of Intersection of Two Curves
— Derivative as the Rate of Change — Length of Tangent, Subtangent, Normal and
— Velocity and Acceleration in Rectilinear Motion Subnormal

Session 2 Session 5
— Differential and Approximation — Rolle’s Theorem

— Geometrical Meaning of Dx, Dy, dx and dt — Lagrange’s Mean Value Theorem

Session 3 Session 6
— Slope of Tangent and Normal — Application of Cubic Functions

— Equation of Tangent PRACTICE PART


— Equation of Normal — JEE Type Examples
— Chapter Exercises

8. MONOTONICITY, MAXIMA AND MINIMA 459-550


LEARNING PART — Methods of Finding Extrema of Continuous
Session 1 Functions
— Convexity/Concavity and Point of Inflection
— Monotonicity
— Concept of Global Maximum/Minimum
Session 2
Session 5
— Critical Points
— Maxima and Minima of Discontinuous
Session 3 Functions
— Comparison of Functions Using Calculus
Session 4 PRACTICE PART
— Introduction to Maxima and Minima — JEE Type Examples
— Chapter Exercises
SYLLABUS

JEE MAIN
Limit, Continuity and Differentiability
Real valued functions, algebra of functions, polynomials, rational, trigonometric,
logarithmic and exponential functions, inverse functions. Graphs of simple functions.
Limits, continuity and differentiability.
Differentiation of the sum, difference, product and quotient of two functions.
Differentiation of trigonometric, inverse trigonometric, logarithmic exponential,
composite and implicit functions derivatives of order upto two. Rolle's and Lagrange's
Mean Value Theorems. Applications of derivatives: Rate of change of quantities,
monotonic - increasing and decreasing functions, Maxima and minima of functions of
one variable, tangents and normals.

JEE ADVANCED
Differential Calculus
Real valued functions of a real variable, into, onto and one-to-one functions, sum,
difference, product and quotient of two functions, composite functions, absolute
value, polynomial, rational, trigonometric, exponential and logarithmic functions.
Limit and continuity of a function, limit and continuity of the sum, difference, product
and quotient of two functions, l'Hospital rule of evaluation of limits of functions.
Even and odd functions, inverse of a function, continuity of composite functions,
intermediate value property of continuous functions.
Derivative of a function, derivative of the sum, difference, product and quotient of
two functions, chain rule, derivatives of polynomial, rational, trigonometric, inverse
trigonometric, exponential and logarithmic functions.
Derivatives of implicit functions, derivatives up to order two, geometrical
interpretation of the derivative, tangents and normals, increasing and decreasing
functions, maximum and minimum values of a function, applications of Rolle's
Theorem and Lagrange's Mean Value Theorem.
CHAPTER

01
Essential
Mathematical Tools
Learning Part
Session 1
● Basic Definitions
Session 2
● Intervals
● Modulus or Absolute Value Function
Session 3
● Number Line Rule
● Wavy Curve Method
Session 4
● Quadratic Expression
● Non-negative Functions

Practice Part
● Chapter Exercises

Arihant on Your Mobile !


Exercises with this #L
symbol can be practised on your mobile. See title inside to activate for free.
Session 1
Basic Definitions
(i) Natural Numbers The set of numbers {1, 2, 3, 4, ....... } is (vi) Prime Numbers A counting number is called a
called natural numbers and it is denoted by N . prime number when it has exactly two factors,
i.e. N = {1, 2, 3, 4, ....... } 1 and itself.
(ii) Whole Numbers The set of numbers {0, 1, 2, 3, 4, ......} is e.g. 2, 3, 5, 7, 11, 13, 17,K etc
called whole numbers and it is denoted by W. Remarks
i.e.W = {0, 1, 2, 3, 4, ....... } (a) 2 is the only even number which is prime.
(iii) Integers The set of numbers (b) A prime is always greater than 1.
{......, – 3, – 2, – 1, 0, 1, 2, 3, ...... } is called integers and it is How to test a number is prime or not
denoted by I or Z , Let given number be p, then
i.e. I (or Z ) = {....., – 4, – 3, – 2, – 1, 0, 1, 2, 3, 4, ...... } Find whole number x such that x > p .
Take all prime numbers less than or equal to x.
where we represent; If none of these divides with p exactly, then p
(a) Positive integers by I + = {1, 2, 3, 4, ..... } = natural is a prime otherwise p is non-prime.
numbers. e.g. Let p = 193; clearly, 14 > 193
Prime numbers upto 14 are 2, 3, 5, 7, 11, 13.
(b) Negative integers by I – = {.......,– 4,– 3,– 2,– 1} No one of these divides 193 exactly.
(c) Non-negative integers {0, 1, 2, 3, 4, ..... } = whole Hence, 193 is a prime number.
numbers (vii) Co-prime Numbers Two natural numbers are
(d) Non-positive integers {.......,– 3, – 2, – 1, 0 } said to be co-primes, if their HCF is 1.
a e.g. (7, 9), (15, 16) are called co-prime numbers.
(iv) Rational Numbers All the numbers of the form ,
b Remark
where a and b are integers and b ¹ 0 are called rational Co-prime numbers may or may not be prime.
numbers and their set is denoted by Q.
(viii) Twin Prime A prime number that is either 2 less
ìa ü or 2 more than another prime number, i.e. a twin
i.e. Q = í : a, b Î I and b ¹ 0 and HCF of a,b is 1ý .
î b þ prime is a prime that has a prime gap of two.
e.g. (3, 5), (5, 7), (11,13), (17, 19), (29, 31),K are
Remarks called twin prime pairs.
(a) Every integer is a rational number as it can be written as
a (ix) Composite Numbers Composite numbers are
(where b = 1)
b non-prime natural numbers. They must have atleast
(b) All recurring decimals are rational numbers. one factor apart from 1 and itself.
1 e.g. 4, 6, 8, 9,K are called composite numbers.
For example, = 0 . 3333 .....
3
(v) Irrational Numbers Those values which neither Remarks
terminate nor could be expressed as recurring decimals are (a) Composite numbers can be both even or odd.
a (b) 1 is neither prime nor composite number.
called irrational numbers, i.e. they can’t be expressed as
b (x) Real Numbers The set which contains both
form and are denoted by Q C (i.e. complement of Q). rational and irrational numbers is called the real
1 –3 2 1 numbers and is denoted by R.
e.g. 2 , 1 + 2 , , , , 3, 1 + 3, ± , p , K etc.
2 2 2 3 i.e. R = Q È QC

Remark R = { x : x Î Q or x Î Q C }
The set of rational and irrational numbers cannot be expressed in
roster form.
Chap 01 Essential Mathematical Tools 3

_ _
ì 5 3 7 1 1 1 ü –π –√2 √2 π
ï....., – 2, – 1, 0, 1, 2, 3, ....., , , , , , , ï
\ R =í 6 4 9 3 7 5 ý –∞ –4 –3 –2 –1 0 1 2 3 4 ∞
ïî .... ., 2 , 3 , p, ......ïþ Figure 1.1
(ii) The sum, difference, product and quotient of two rational
Remarks numbers is a rational number.
(i) As from above definitions; (iii) The sum, difference, product and quotient of two irrational
N Ì W Ì I Ì Q Ì R, numbers need not be irrational.
It could be shown that real numbers can be expressed on (iv) The sum, difference, product and quotient of a non-zero
number line with respect to origin as rational and an irrational number is always irrational.

Exercise for Session 1


n Directions (Q. Nos. 1 to 6) State whether following statements are True/False.

1. All the rational numbers are irrational also.

2. All the integers are irrational also.

3. Irrational numbers are real numbers also.

4. Zero is a natural number.

5. Sum of two natural numbers is a rational number.

6. A positive integer is a natural number also.

n
Directions (Q. Nos. 7 to 10) These questions have only one option correct.

7. Sum of two rational numbers is


(a) rational (b) irrational
(c) Both (a) and (b) (d) None of these

8. Sum of two irrational numbers is


(a) rational (b) irrational
(c) real (d) None of these

9. Product of two rational numbers is


(a) always rational (b) rational or irrational
(c) always irrational (d) None of these

10. If a is an irrational number which is divisible by b, then the number b


(a) must be rational (b) must be irrational
(c) may be rational or irrational (d) None of these
Session 2
Intervals, Modulus or Absolute
Value Function
Intervals i.e. If a £ x £ b Þ x Î[ a, b ]

The set of numbers between any two real numbers is


a b
called intervals. The types of interval discussed below.
Figure 1.5
(i) Open-Open Interval If a and b are real numbers
and a < b, then the set of all real numbers x such that y Example 1 Solve 2x + 1 > 3.
a < x < b is called open-open interval. It is denoted by
Sol. Here, 2x + 1 > 3 or 2x > 2 or x > 1, i.e. x Î (1, ¥ )
(a, b ) or ]a, b [.
i.e. If a < x < b Þ x Î ] a, b [ or x Î( a, b ) This solution can be graphed on a real line as;

–∞ 1 +∞
a b
Figure 1.2
y Example 2 Solve –2 < 2x – 1 < 2 .
e.g. 1< x<2 Sol. Here, – 2 < 2x – 1 < 2
Þ x Î(1, 2 ) or x Î ]1, 2 [ (using above definition)
or –1 < 2x < 3
Note At ± ¥ brackets are always open.
1 3 æ 1 3ö
or – <x< , i.e. x Î ç– , ÷
(ii) Open-Closed Interval If a and b are real numbers 2 2 è 2 2ø
and a < b, then the set of all real numbers x such that
This solution can be graphed on a real line as;
a < x £ b is called open-closed interval. Here, a is
excluded and b is included.
i.e. If a < x £ b –∞ –1/2 3/2 +∞
– 1< x < 3
Þ x Î ] a, b ] or x Î( a, b ] 2 2

a b y Example 3 Solve the following inequations.


Figure 1.3 3 (x – 2) 5 (2 – x ) 2x – 3 4x
(iii) Closed-Open Interval If a and b are real numbers
(i) ³ (ii) +9³3 +
5 3 4 3
and a < b, then the set of all real ‘x’ such that 3 ( x – 2) 5 ( 2 – x )
a £ x < b is called closed-open interval. Sol. (i) We have, ³
5 3
Here, a is included and b is excluded. Þ 3 (3x – 6) ³ 5 (10 – 5x )
i.e. If a £ x <b Þ 9 x – 18 ³ 50 – 25x
Þ x Î[a, b [ Þ 34 x ³ 68 Þ x ³2
or x Î[ a, b ) Hence, the solution set is x Î[2, ¥ ) and graphically it
could be shown as
a b
Figure 1.4 –∞ 2 +∞
2x – 3 4x
(iv) Closed-Closed Interval If a and b are real numbers (ii) We have, +9 ³3+
and a < b, then the set of all real ‘x’ such that 4 3
a £ x £ b is called closed-closed interval. Here, a and b Þ
2x – 3 4 x
– ³ 3 –9
both are included. 4 3
Chap 01 Essential Mathematical Tools 5

Þ
3 (2x – 3) – 16x
³ –6 Þ
–10 x – 9
³ –6 |– 2 | = 2, |– 1 . 3291 | = 1 . 3291 ..... etc.
12 12 or it is the distance defined with respect to origin, as
Þ –10x – 9 ³ – 72 Þ –10x ³ – 63 | x | =1 means distance covered is one unit on right hand
As we know the inequality sign changes, if multiplied side or left hand side of origin shown as:
by (–ve).
63
\ 10x £ 63 or x £ –1 0 1
10
Figure 1.6
Hence, the solution set of the given inequation is
æ 63 ù \ | x | =1 Þ x = ± 1
ç– ¥, ú . Graphically it could be shown as
è 10 û Again, | x | <1 means distance covered is less than one unit
on right hand side or left hand side of origin shown as:
–∞ 63/10 +∞
–1 0 1
y Example 4 Solve for x.
Figure 1.7
4 6
£ 3£ , ( x > 0)
x+1 x+1 Similarly, | x | >1 means distance covered is more than one
Sol. Consider the first inequality unit on right hand side or left hand side of origin shown as:
4
£ 3 Þ 4 £ 3( x + 1)
x +1 –1 0 1
Þ 4 £ 3x + 3 Figure 1.8
Þ 4 - 3 £ 3x Þ 3x ³ 1
1
Þ x³ ...(i) Graphical Representation
3
Again, 3£
6
Þ 3( x + 1) £ 6 of Modulus
x +1
As we define,
Þ 3x + 3 £ 6 Þ 3x £ 6 - 3
ì x, x³0
Þ 3x £ 3 y = | x | =í
î– x, x <0
Þ x £1 ...(ii)
Combining the results (i) and (ii), we have This behaviour is due to two straight lines represented by
1 é1 ù modulus.
£ x £ 1, i.e. x Î ê , 1ú
3 ë3 û For plotting the graph of the modulus function, we put
é1 ù x 0 1 2 –1 –2
Hence, the solution set of the given in equations is ê , 1ú.
ë3 û y 0 1 2 1 2
Graphically, it could be shown as
y = –x Y
y=x

–∞ 0 1/3 1 ∞
(–2, 2) 2
(2, 2)
1
Modulus or Absolute (–1, 1) (1, 1)
X
Value Function –2 –1 0 1 2

ì x, x ³ 0
The function defined by f ( x ) = x = í is called Figure 1.9
î- x , x < 0
Now, we consider the following cases
modulus function. This always gives a positive result.
Case I When x ³0
As we know modulus means numerical value,
Equation of the straight line, passing through (0, 0)
i.e. | 3 | = |– 3 | = 3, and (1, 1) is
6 Textbook of Differential Calculus

y – y1 y2 – y1 y – 0 1– 0 (ii) If | x | = – 5
= , i.e. =
x – x1 x2 – x1 x – 0 1– 0 Þ x has no solution.
Þ y = x, when x ³0 As | x | is always positive or zero, it can never be
negative.
Case II When x £ 0
\ RHS < LHS
Equation of the straight line passing through (–1, 1) and or given relation has no solution.
(–2, 2)
Aliter
y – y1 y2 – y1
= Same as in (i), plotting the graph of y = | x | and
x – x1 x2 – x1 y = - 5.
y–1 2–1 The two graphs do not intersect.
i.e. = Þ –y + 1 = x + 1 or y = – x
x + 1 –2 + 1 \ No solution.
Y
when x £ 0.
Thus for every modulus function it exhibits two values,
which could be shown graphically. X
O
ì ( x – 1), x ³ 1
Similarly, y = | x – 1| = í
î–( x – 1), x £ 1
y = –5

(iii) | x | < 5
Remarks
(a) Every modulus function exhibits two values which are It means that x is the number, which is at distance less
represented by +ve and –ve, signs but it only gives the than 5 from 0.
positive outcomes. So, students shouldn’t get confused by Hence, –5 < x < 5
+ ve or –ve, signs as these signs are in different intervals, but
the outcomes are positive. Aliter | x | < 5. Plotting the graph of we have y = | x |
(b) Modulus function is never negative, thus|x| ³ 0 for any and y = 5.
real x and|x| </ 0. Y
y = |x|

y Example 5 Explain the following : y=5


(i) | x | =5 (ii) | x | = – 5
(iii) | x | <5 (iv) | x | < – 5
X
(v) | x | > – 5 (vi) | x | >5 –5 0 5

Sol. (i) If | x | = 5
We see that, | x | < 5, when -5 < x < 5
Þ x = ± 5, which means, x is at a distance of 5 units
(iv) | x | < – 5
from 0, which is certainly 5 and – 5.
which shows no solution.
Aliter
As LHS is non-negative and RHS is negative or | x | < – 5
| x | = 5.
does not possess any solution.
Here, students are advised to consider two different
Aliter | x | < - 5.
functions, as
Plotting the graph of y = | x | and y = - 5.
y = | x | and y = 5.
We see that, | x | < - 5 is not possible as | x | > - 5, for
Now, we plot graph of these two equations.
all x Î R.
which intersect at two points, i.e. x = 5 and x = - 5. Y y = |x|
Y
y = |x|
y=5
X
0

X
–5 0 5 y = –5

\ | x | = 5 possess two solutions x = 5 and x = - 5. \ | x | < - 5 Þ No solution.


Chap 01 Essential Mathematical Tools 7

(v) | x | > – 5 We know, here LHS ³0 and RHS <0 (iv) If a < 0, then
So, LHS > RHS | x | £ a Þ No solution.
i.e. above statement is true for all real x. | x | ³ a Þ All real numbers solutions.
(as we know that non-negative number is always
(v) | x + y | = | x | + | y | Û (x ³ 0 and y ³ 0) or (x £ 0 and
greater than negative).
y £ 0) Û xy ³ 0.
Aliter
(vi) | x - y | = | x | - | y | Û ( x ³ 0, y ³ 0 and | x | ³ | y | )
Here, | x | > - 5
Plotting the graph of y = | x | and y = - 5. or ( x £ 0, y £ 0 and | x | ³ | y | )
We see that, | x | > - 5, for all x Îreal number. (vii) | x ± y | £ | x | + | y | (viii) | x ± y | ³ | x | - | y |

Y y = |x| y Example 6 Solve for x, where


(i) f (x ) = | x | ³ 0 (ii) f (x ) = | x | > 0
Sol. (i) f ( x ) = | x | ³ 0
X
O As we know modulus is non-negative quantity.
(i.e. It is always greater than equal to zero)
y = –5 \ x Î R is the required solution.
Aliter f ( x ) = | x | ³ 0.
(vi) | x | > 5
Plotting two graphs y = | x | and y = 0.
It means that x is the number which is at distance
greater than 5 from 0. Hence, x <–5 or x > 5 From graph | x | ³ 0, for all x Î R.
Y
Aliter y = |x|
|x | > 5
Plotting the graph of y = | x | and y = 5. We have
Y
y = |x| X
O y=0
y=5
(ii) f ( x ) = | x | > 0
Here equal sign is absent, so we have to exclude those
X value of x for which | x | = 0.
–5 0 5
\ x Î R except x = 0 or x Î R – { 0 } is the required
solution.
We see that, x < - 5 or x > 5.
Aliter f ( x ) = | x | > 0.
Plotting two graphs y = | x | and y = 0.
Generalized Results Y

(i) For any real number x, we have x = | x | 2 2 y = |x|

(ii) For any real number x, we have x 2 = | x |


(iii) If a < 0, then
X
(a) x 2 £ a 2 Û | x | £ a Û - a £ x £ a O y=0

(b) x 2 < a 2 Û | x | < a Û - a < x < a From graph | x | > 0, for all x Î R except 0.
(c) x 2 ³ a 2 Û | x | ³ a Û x £ - a or x ³ a \ x Î R - {0} .
(d) x 2 > a 2 Û | x | > a Û x < - a or x > a y Example 7 Solve | x – 3 | < 5 .
(e) a £ x £ b Û a £ | x | £ b
2 2 2
Sol. | x – 3 | < 5
Û x Î[ -b, - a ] È [a, b ] Þ –5 < x – 3 < 5
(f) a 2 < x 2 < b 2 Û a < | x | < b Þ –5 + 3 < x < 5 + 3
Û x Î ( -b, - a ) È (a, b ) Þ –2 < x < 8 or x Î ]– 2, 8[ or x Î(–2, 8)
8 Textbook of Differential Calculus

Aliter For | x - 3 | < 5. y Example 10 Solve | x – 1 | £ 5, | x | ³ 2.


Plotting two graphs y = | x - 3 | and y = 5
Sol. Here, | x –1| £ 5 and | x | ³ 2
y = |x – 3|
i.e. (–5 £ x – 1 £ 5)
5
y=5 Þ (– 4 £ x £ 6) …(i)
Similarly, (x £–2 or x ³ 2)
3
Þ (x £ – 2 or x ³ 2) ...(ii)
From Eqs. (i) and (ii) could be graphically shown as
–2 0 3 8

From graph, -2 < x < 8 ; \ x Î ( -2, 8). –4 –2 2 4 6

y Example 8 Solve | x – 1 | £ 2 . 2
–2
Sol. | x – 1 | £ 2
Thus, the shaded portion, i.e. common to both Eqs. (i) and
Þ – 2 £ x – 1 £ 2 Þ –2 + 1 £ x £ 2 + 1 (ii) is the required region x Î [– 4, – 2] È[2, 6].
Þ –1 £ x £ 3 or x Î[–1, 3] Aliter Here, | x - 1 | £ 5 and | x | ³ 2.
Take (y = | x - 1|, y = 5) and (y = | x |, y = 2) on graph and
Aliter For | x - 1 | £ 2. take common.
Plotting two graphs y = | x - 1 | and y = 2 Y
Y y = |x–1| x| –1
|
|
x
y= |
2
y=2
5 y=
4
1 3
2
X 1
–1 0 1 3
X′ X
–6 –5 –4 –3 –2 –1 O
1 2 3 4 5 6
From graph, -1 £ x £ 3 –1

\ x Î [ -1, 3] –2
–3
y Example 9 Solve 1 £ | x – 1 | £ 3 . –4
–5
Sol. Here, 1 £ | x–1| £ 3 Y′
Þ – 3 £ ( x – 1) £ – 1 or 1 £ ( x –1) £ 3
From the graph, - 4 £ x £ -2
[Qa £ | x | £ b Þ x Î [ -b,-a ] È [b,a ]]
or 2£ x £6
i.e. the distance covered between 1 unit to 3 units.
\ x Î [ - 4, - 2] È [2, 6]
Þ –2 £ x £ 0 or 2 £ x £ 4
2
Hence, the solution set of the given inequation is y Example 11 Solve > 1, x ¹ 4 .
x Î [–2, 0] È [2, 4 ]. x–4
Aliter Here, 1 £ | x - 1 | £ 3 2
Sol. We have, > 1, where x ¹ 4
Plotting three graphs y = 1, y = | x - 1 | and y = 3. x–4
–1
| ...(i)
|x 2 é a |a| ù
y= Þ >1 = and | 2| = 2ú
y=3 êQ
|x – 4| ë b |b | û
Þ 2>|x–4|
y=1
1 Þ | x – 4 | <2
–2 0 1 2 4 Þ –2 < x – 4 < 2
Þ 2< x<6
From graph, -2 £ x £ 0 or 2 £ x £ 4. \ x Î(2, 6) , but x ¹ 4 [from Eq. (i)]
\ x Î [ -2, 0] È [2, 4 ]
Chap 01 Essential Mathematical Tools 9

Hence, the solution set of the given inequation is Case II When 1 £ x £ 2


x Î (2, 4 ) È ( 4, 6). i.e. | x – 1 | +| x – 2 | ³ 4
½ 2 ½ Þ ( x – 1) – ( x – 2) ³ 4
Aliter ½ ½ > 1, x ¹ 4 Þ | x - 4 | < 2
½ x - 4½ Þ 1 ³ 4, which is meaningless.
Plotting the graph of y = | x - 4 | and y = 2 \ No solution for x Î[1, 2]. ...(ii)
Y
Case III When 2 < x < ¥
4
i.e. | x – 1 | +| x – 2 | ³ 4
y=2 Þ ( x – 1) + ( x – 2) ³ 4
2
Þ 2x – 3 ³ 4
0
X Þ x ³ 7/2
2 4 6
But 2< x < ¥
From graph, 2 < x < 6, x ¹ 4
x Î (2, 6) - { 4 } é7 ö
i.e. \ Solution set is ê , ¥ ÷. ...(iii)
ë2 ø
Note
Students should always remember that they have to compare From Eqs. (i), (ii) and (iii), we get
the solution set with the initial condition. æ 1ù é7 ö
x Î ç– ¥, – ú È ê , ¥ ÷
è 2û ë 2 ø
y Example 12 Solve | x – 1 | + | x – 2 | ³ 4 .
Aliter
Sol. On the LHS of the given inequation, we have two
modulus, so we should define each modulus i.e. by Plotting two graphs y = | x - 1 | + | x - 2 | and y = 4 shown
equating it to zero. below.
ì ( x – 1), x ³ 1 Y
i.e. | x –1 | = í
î–( x –1), x < 1
y = |x – 1| + |x – 2|

ì ( x – 2), x ³ 2 y=4
and | x –2 | = í
î–( x –2), x < 2 y = 3 – 2x
Thus, it gives three cases : y = 2x – 3
1
Case I When – ¥ < x < 1 (1, 1) (2, 1)
i.e. | x – 1 | +| x – 2 | ³ 4
Þ –( x – 1) – ( x – 2) ³ 4 0
X
–1 1 2 7/2
Þ –2x + 3 ³ 4 Þ –2x ³ 1 2
1
Þ x£– ...(i) From graph, x £ -
1
or x ³
7
2 2 2
But –¥ < x<1 æ 1ù é7 ö
æ 1ù \ x Î ç - ¥, - ú È ê , ¥ ÷
\ Solution set is x Î ç–¥, – ú. è 2û ë 2 ø
è 2û

Exercise for Session 2


n
Directions (Q. Nos. 1 to 5) Solve the following inequalities for real values of x.

1. | x - 1| < 2 2. | x -3| > 5

3. 0 < | x - 1| < 3 4. | x - 1| + | 2x - 3| = | 3x - 4 |
x -3
5. £1
x 2 -4
Session 3
Number Line Rule, Wavy Curve Method
Number Line Rule y Example 15 Find the values of x for which
It is used to solve algebraic inequalities using following steps: (2x – 1)( x – 1) 2 ( x – 2) 3
f (x ) = > 0.
(i) Factorize numerator as well as denominator. ( x – 4 )4
(ii) Now, check the coefficients of x and make them (2x – 1)( x – 1)2 ( x – 2)3
positive. Sol. f ( x ) = , which gives x ¹ 4 ...(i)
( x – 4 )4
(iii) Put only odd power factors in numerator and As denominator ¹ 0 and x ¹ 1, as at x = 1, f ( x ) has even
denominator and put them equal to zero separately powers.
and find the value of x. Putting zero to (2x – 1) and ( x – 2)3 as they have odd powers
(As for polynomial function only numerator = 0, and neglecting ( x – 1)2 and ( x – 4 )4 on number line as
denominator ¹0). shown in figure.
(iv) Plot these points on number line in increasing order. + +
1/2 – 2
(v) Start number line from right to left taking sign of f ( x ).
(vi) Check your answer so that it should not contain a which shows f ( x ) > 0 when x < 1/ 2 or x > 2 but except for 4
and 1.
point for which f ( x ) doesn’t exist.
\ x Î (–¥, 1/ 2) È (2, ¥ ) –{ 4 }
y Example 13 Find the interval in which f ( x ) is positive
y Example 16 Find the value of x for which
or negative: f ( x ) = ( x – 1)( x – 2)( x – 3).
Sol. Here, f ( x ) = ( x – 1)( x – 2)( x – 3) has all factors with odd ( x – 2) 2 (1 – x )( x – 3) 3 ( x – 4 ) 2
f (x ) = £ 0.
powers, so put them as zero. ( x + 1)
i.e. x – 1 = 0, x – 2 = 0, x – 3 = 0, we get x = 1, 2, 3 ( x – 2)2 (1 – x )( x – 3)3 ( x – 4 )2
Sol. Here, f ( x ) =
[using step (iii)] ( x + 1)
Using steps (iv) and (v), plotting on number line, we get ( x – 2)2 ( x – 1)( x – 3)3 ( x – 4 )2
or f ( x) = – , ( x ¹ – 1) ...(i)
( x + 1)
1 2 3 Putting zero to ( x – 1), ( x – 3)3 , ( x + 1) as having odd
f ( x ) > 0 when 1 < x < 2 and x > 3 powers and neglecting ( x – 2)2 , ( x – 4 )2 , we get
f ( x ) < 0 when x < 1 and 2 < x < 3
+ +
( x – 1)(2 – x ) –1 – 1 3 –
y Example 14 Solve f ( x ) = ³ 0.
( x – 3) f ( x ) £ 0 when –1 < x £ 1 or 3 £ x < ¥ or f ( x ) = 0 at x = 2
( x – 1)(2 – x ) or x Î (–1, 1] È [3, ¥ ) È {2} [using Eq. (i) as x ¹–1]
Sol. Here, f ( x) = ³0
( x – 3) | x |– 1
( x – 1)( x – 2) y Example 17 Solve ³ 0; x ÎR, x ¹ ± 2
or f ( x) = – , which gives | x |– 2
( x – 3)
| x|–1 y–1
x – 3 ¹ 0 or x ¹3 ...(i) Sol. We have, ³0 Þ ³ 0 ; where y = | x |
| x |– 2 y–2
Using number line rule as shown in the figure,
Þ y £ 1 or y > 2 using number line rule
+ – +
1 2 3
1 2
which shows f ( x ) ³ 0 when x £ 1 or 2 £ x < 3 Þ | x| £ 1 or | x| > 2
i.e. x Î (–¥, 1] È [2, 3). (as x ¹ 3) Þ (–1 £ x £ 1) or (x < –2 or x > 2)
Chap 01 Essential Mathematical Tools 11

Þ x Î [–1, 1] È (–¥, – 2) È(2, ¥ )


Hence, the solution set is
Wavy Curve Method
x Î (–¥, – 2) È [–1, 1] È (2, ¥ ). The method of intervals (or wavy curve) is used for
solving inequalities of the form :
–1
y Example 18 Solve ³ 1, where x ÎR, x ¹ ± 2. ( x - a 1 ) n1 ( x - a 2 ) n2 . . . ( x - a k ) nk
| x |– 2 f (x ) = >0
mp
–1 ( x - b 1 ) m1 ( x - b 2 ) m 2 . . . ( x - b p )
Sol. We have, ³1
| x |–2 ( < 0, £ 0 or ³ 0 )
–1 –1–(| x | – 2) where n1 , n2 , ......, nk and m 1 , m 2 , ......, m p are natural
Þ –1 ³ 0 Þ ³0
| x|–2 | x|–2 numbers.
1–| x | –(| x | – 1) a 1 , a 2 , ......, a k ; b 1 , b 2 , ......, b p are any real numbers such
Þ ³0 Þ ³0
| x|–2 (| x | – 2) that a i ¹ b j , where i = 1, 2, 3, ..., k and j = 1, 2, 3, ..., p
Using number line rule, It consists of the following statements :
– + – 1. All zeros of the function f ( x ) (the values of x
1 2 corresponding to numerator = 0) contained on the left
hand side of the inequality should be marked on the
Þ 1 £ | x | < 2 Þ x Î (–2, – 1] È [1, 2)
number line with inked (black) circles.
{Q a £ | x | < b Û x Î (–b, – a] È [ a, b )}
2. All points of discontinuities of the function f ( x ) (the
Hence, the solution set is (–2, – 1] È [1, 2).
values of x corresponding to denominator = 0)
| x + 3| + x contained on the left hand side of the inequality
y Example 19 Solve > 1. should be marked on the number line with uninked
x+2
(white) circles.
| x + 3| + x | x + 3 | + x – x –2
Sol. Here, –1 > 0 Þ >0 3. Check the value of f ( x ) for any real number greater
x+2 x+2
than the rightmost marked number on the number line.
| x + 3|–2
Þ >0 ...(i) 4. From right to left, beginning above the number line
x+2
(in case of value of f ( x ) is positive in step (iii)
Now, two cases arise : otherwise from below the number line), a wavy curve
Case I When x + 3 ³ 0, i.e. x ³ -3 ...(ii) should be drawn to pass through all the marked
x + 3 –2 x+1 points so that when it passes through a simple point,
Þ >0 Þ >0
x+2 x+2 the curve intersects the number line and when
passing through a double point, the curve remains
Þ x Î (– ¥, – 2) È (–1, ¥ ) using number line rule as shown
in the figure.
located on one side of the number line.
+ – + 5. The appropriate intervals are chosen in accordance
–2 –1 with the sign of inequality (the function f ( x ) is
But x ³ –3 [from Eq. (ii)] positive whenever the curve is situated above the
number line, it is negative if the curve is found below
Þ x Î [–3, – 2) È (–1, ¥ ) ...(a)
the number line). Their union just represents the
Case II When x + 3 < 0, i.e. x < -3 ...(iii) solution of the inequality.
–( x + 3)–2 –( x + 5)
Þ >0 Þ >0 Remarks
x+2 ( x + 2) (a) Points of discontinuity will never be include in the answers.
Þ x Î(–5, – 2) using number line rule as shown in the figure. (b) If asked to find the intervals where f ( x ) is non-negative or
– + – non-positive, then make the intervals closed, corresponding
to the roots of the numerator and let it remain open
–5 –2
corresponding to the roots of denominator.
But x < –3 [from Eq. (iii)]
( x - 1) 3 ( x + 2) 4 ( x - 3) 5 ( x + 6 )
\ x Î(–5, – 3) ...(b) y Example 20 Let f ( x ) = .
x 2 ( x - 7 )3
Thus, from Eqs. (a) and (b), we have
Solve the following inequalities :
x Î [–3, – 2) È (–1, ¥ ) È (–5, – 3)
(i) f (x ) > 0 (ii) f (x ) ³ 0
Þ x Î (–5, – 2) È (–1, ¥ )
(iii) f (x ) < 0 (iv) f (x ) £ 0
12 Textbook of Differential Calculus

( x - 1) 3 ( x + 2) 4 ( x - 3) 5 ( x + 6) Hence, solution of
Sol. Given, f ( x ) =
x 2 ( x - 7 )3 æ p ö æ p p ö æ 5p 4 p ö
(i) f ( x ) > 0 is x Î ç0, ÷ È ç , ÷ È ç , ÷
Put numerator = 0 è 6ø è3 2ø è 6 3 ø
Þ ( x - 1) 3 ( x + 2) 4 ( x - 3) 5 ( x + 6) = 0 æ 3p ö
È ç , 2p ÷ - {e , 3}
Þ x = 1, - 2, 3, - 6 è 2 ø
Again, put denominator = 0 æ p ù é p p ö é 5p 4 p ù
(ii) f ( x ) ³ 0 is x Î ç0, ú È ê , ÷ È ê ,
Þ x 2 ( x - 7 )3 = 0 è 6 û ë 3 2 ø ë 6 3 úû
Þ x = 0, 7 æ 3p ö
È ç , 2p ÷ - {3}
We mark on the number line zeros of the function: è 2 ø
1, - 2, 3 and - 6 (with black circles) and the points of
æ p p ö æ p 5p ö æ 4 p 3p ö
discontinuities 0 and 7 (with white circles). Isolate the (iii) f ( x ) < 0 is x Î ç , ÷ È ç , ÷ È ç , ÷ - {2}
double points: –2 and 0 and draw the curve of signs. è6 3ø è2 6 ø è 3 2 ø
é p p ù æ p 5p ù é 4 p 3p ö
(iv) f ( x ) £ 0 is x Î ê , ú È ç , ú È ê , ÷ È {e } - {2}
+ 0 + – + ë6 3û è2 6 û ë 3 2 ø
–6 – – – 1 3 7
–2 y Example 22 Let
3
æ 3ö
(cos x + |cos x | ) ç sin x - ÷ (tan x - 1) 5
From the graph, we get è 2ø
f (x ) =
(i) if f ( x ) > 0, then x Î ( -¥, - 6) È (1, 3) È (7, ¥ ) (cos x - 2) 2 (tan x - 3 ) 3
(ii) if f ( x ) ³ 0, then x Î ( -¥, - 6] È { -2} È [1, 3] È (7, ¥ ) æ -p p ö
Find the interval of x Î ç , ÷ for which
(iii) if f ( x ) < 0, then x Î ( -6, - 2) È ( -2, 0) È (0, 1) È (3, 7 ) è 2 2ø
(iv) if f ( x ) £ 0, then x Î [ - 6, 0) È (0, 1] È [3, 7 )
(i) f (x ) > 0 (ii) f (x ) < 0
y Example 21 Let æ -p p ö
Sol. For x Î ç , ÷
è 2 2ø
æ 1ö
ç sin x - ÷ (ln x - 1) ( x - 2)(tan x - 3 )
2
è 2ø cos x > 0 for all x Þ | cos x | = cos x
f (x ) = Þ cos x + | cos x | = 2 cos x
(e x - e 2 )( x - 3) 2 × cos x 3
æ 3ö
2 cos x çsin x - ÷ (tan x - 1)5
Solve the following inequalities for x Î[0, 2p ] : è 2ø
\ f (x ) =
(i) f (x ) > 0 (ii) f (x ) ³ 0 (cos x - 2)2 (tan x - 3 )3
(iii) f (x ) < 0 (iv) f (x ) £ 0 p
Critical points cos x = 0 Þ x = (not in domain)
p 3p 2
Sol. Clearly, x ¹ 2, 3,
, and f ( x ) = 0
2 2 +
p p 5p 4p
for x = , , , e, –π/2 π/4 π/3 π/2
6 3 6 3 –

π/3 5π/6 4π/3 2π 3
0 π/6 sin x = (not possible)
π/2 2 e 3 3π/2 2
p
Now, sign of f ( x ) will not change around x = 2, e , 3. tan x = 1 Þ x = Þ cos x = 2 (not possible)
4
Then, for f ( x ) > 0 p
tan x = 3 Þ x = ×
æ 1ö 3
çsin x - ÷ (tan x - 3 ) > 0
è 2ø æp pö
Then, f ( x ) > 0 Þ x Î ç , ÷
è4 3ø
æ p ö æ p p ö æ 5p 4 p ö
Þ x Î ç0, ÷ È ç , ÷ È ç , ÷
è 6ø è3 2ø è 6 3 ø æ -p p ö æ p p ö
and f (x ) < 0 Þ x Î ç , ÷ È ç , ÷
æ 3p ö è 2 4ø è3 2ø
È ç , 2p ÷ - {e , 3}
è 2 ø
Chap 01 Essential Mathematical Tools 13

Exercise for Session 3


n Directions (Q. Nos. 1 to 5) Solve the following inequalities.
x - 2 2x - 3 2x - 1
1. > 2. >0
x + 2 4x - 1 2x + 3x 2 + x
3

4x ( x - 1)( x + 1)( x + 4)( x + 6)


3. ³1 4. >0
x2 +3 7x 2 + 8x + 4
x
5. £1
x - 5x + 9
2

6. Solution of inequality | x - 1| < 0 is


(a) x = 0 (b) x = 1
(c) x ¹ 1 (d) No solution
7. Solution of inequality x 2 + x + | x | + 1 £ 0 is
(a) (1, 2) (b) (0, 1)
(c) No solution (d) None of these
8. Solution of inequality | x + 3 | > | 2x - 1| is

(a) æç - , 4ö÷
2
(b) (4, ¥)
è 3 ø

(c) æç - , 1ö÷
2
(d) None of these
è 3 ø
1
9. Solution of inequality x + < 4 is
x
(a) (2 - 3, 2 + 3 ) È (-2 - 3, - 2 + 3) (b) R - (2 - 3, 2 + 3)
(c) R - (-2 - 3, 2 + 3 ) (d) None of these
10. The solution of | x 2 + 3x | + x 2 - 2 ³ 0 is
(a) (- ¥, 1) (b) (0, 1)
(c) æç -¥, - ù È é , ¥ö÷
2 1
(d) None of these
è 3 ûú êë 2 ø

11. The solution of | x | - 1 < | 1 - x | , x Î R is


(a) (-1, 1) (b) (0, ¥)
(c) (-1, ¥) (d) None of these
12. The solution of 2x + 2 | x | ³ 2 2 is
(a) (-¥, log2 ( 2 + 1)) (b) (0, ¥)
(c) æç , log2 ( 2 - 1)ö÷ (d) (-¥, log2 ( 2 - 1)] È é , ¥ö÷
1 1
è2 ø êë 2 ø
Session 4
Quadratic Expression, Non-negative Functions
Quadratic Expression Û f ( x ) ³ 0, " x Î R
The expression ax 2 + bx + c is said to be a real quadratic In this case the parabola touches the X-axis and
expression in x, where a, b, c are real and a ¹ 0. Let remains concave upwards.
f ( x ) = ax 2 + bx + c , where a, b, c Î R (a ¹ 0 ). f ( x ) can be (iii) If a > 0 and b 2 - 4ac > 0.
rewritten as Let f ( x ) = 0 has two real roots a and b (a < b).
ìïæ Then, f ( x ) > 0, " x Î ( - ¥, a ) È (b, ¥),
4ac - b 2 üï éæ D ù
2 2
bö bö
f ( x ) = a íç x + ÷ + ý = a êç x + ÷ - ú f ( x ) < 0, " x Î (a, b) and f ( x ) = 0 for x Î {a, b}.
ïîè 2a ø 4a 2 ïþ êë è 2a ø 4a 2 úû
y=f(x)
where D = b 2 - 4ac is the discriminant of the quadratic –b/2a
X′ α X
expression. β
Therefore, y = f ( x ) represents a parabola whose axis is
æ b -D ö In this case the parabola cuts the X-axis at two points
parallel to the Y-axis, with vertex at A ç - , ÷. a and b and remains concave upwards.
è 2a 4a ø
(iv) If a < 0 and b 2 - 4ac < 0 Û f ( x ) < 0, " x Î R

Graph for Quadratic Expressions X′


–b/2a
X

That if a > 0, the parabola will be concave upwards and if


y=f(x)
a < 0, the parabola will be concave downwards and it
depends on the sign of b 2 - 4ac that the parabola cuts the
X-axis at two points (b 2 - 4ac > 0 ), touches the X-axis In this case the parabola remains concave downwards
and always below the X-axis.
(b 2 - 4ac = 0 ) or never intersects with the X-axis
(b 2 - 4ac < 0 ). (v) If a < 0 and b 2 - 4ac = 0.
This gives rise to the following cases: –b/2a
X′ X
(i) If a > 0 and b 2 - 4ac < 0.
y=f(x) = ax2+bx+c y=f(x)

Û f ( x ) £ 0, " x Î R
In this case the parabola touches the X-axis and
X′ X remains concave downwards.
–b/2a
(vi) If a < 0 and b 2 - 4ac > 0.
Û f ( x ) > 0, " x Î R
In this case the parabola always remains concave X′ X
α β
upwards and above the X-axis. –b/2a
y=f(x)
(ii) If a > 0 and b 2 - 4ac = 0.
Let f ( x ) = 0 have two real roots a and b (a < b).
y=f(x) = ax2+bx+c
Then, f ( x ) < 0, " x Î ( - ¥, a ) È (b, ¥),
f ( x ) > 0, " x Î (a, b) and f ( x ) = 0 for x Î {a, b}.
X′ X In this case the parabola cuts the X-axis at two points
–b/2a
a and b and remains concave downwards.
Chap 01 Essential Mathematical Tools 15

Quick and Important Results for y Example 25 Solve ( x + 1) 2 + ( x 2 + 3x + 2) 2 = 0.


Characteristic Expression
1. The expression ax 2 + bx + c will be at same sign for all real Sol. Here, ( x + 1)2 + ( x 2 + 3x + 2)2 = 0 if and only if each term
values of x, iff D < 0. is zero simultaneously,
2. ax 2 + bx + c will always be positive iff D < 0 and a > 0. ( x + 1) = 0 and ( x 2 + 3x + 2) = 0 Þ ( x + 1)( x + 2) = 0
3. ax 2 + bx + c will always be negative iff D < 0 and a < 0.
i.e. x = –1 and x = – 1, – 2
4. If D > 0, then sign of the expression between the roots will be
opposite to that of a. \ The common solution is x = –1.
-b Hence, solution of the above equation is x = –1.
5. If a > 0, then minima of f ( x ) occurs at x = and if a < 0,
2a
-b
then maxima of f ( x ) occurs at x = and maximum or y Example 26 Solve | x + 1 | + x – 1 = 0.
2a
-Dö
minimum value of f ( x ) will be given by æç ÷. Sol. Here, | x + 1 | + x – 1 = 0, where each term is
è 4a ø
non-negative.
6. If f ( x ) = 0 has two distinct real roots, then a × f ( d ) < 0 if and
only if d lies between the roots and a × f ( d ) > 0 if and only if d \ | x + 1 | = 0 and x–1 = 0
lies outside the roots. should be zero simultaneously.
y Example 23 Find a for which 3x + ax + 3 > 0 , " x ÎR.
2 i.e. x = –1 and x = 1, which is not possible.
\ There is no x for which each term is zero simultaneously.
Sol. Here, 3x 2 + ax + 3 > 0, " x Î R Hence, there is no solution.
Þ D < 0 [Q if f ( x ) > 0 and a > 0, then D < 0]
y Example 27 Solve
Þ (a ) – 4 (3)(3) < 0
2
Þ a 2 – 36 < 0
Þ(a – 6)(a + 6) < 0, using number line rule as shown in figure, | x 2 – 1 | + ( x – 1) 2 + x 2 – 3x + 2 = 0.
+ + Sol. Here, each of the term is non-negative, thus each term
–6 6 must be zero simultaneously.
which shows –6 < a < 6 or a Î(–6, 6) i.e. ( x 2 – 1) = 0, ( x – 1)2 = 0 and x 2 – 3x + 2 = 0

y Example 24 Find a for which ax 2 + x – 1 < 0, " x ÎR Þ x = ± 1 , x = 1 and x = 1, 2


? The common solution is x = 1.
Therefore, x = 1 is the solution of above equation.
Sol. Here, ax 2 + x – 1 < 0, " x Î R
Þ a <0 and D < 0 [Qa < 0, D < 0, so f ( x ) < 0] y Example 28 Let f ( x ) = x and g ( x ) = | x | be two
Þ a <0 and 1 + 4a < 0 real-valued functions, f ( x ) be a function satisfying the
Þ a <0 and a < –1/ 4 condition;
\ a Î (– ¥, – 1 / 4 )
[ f ( x ) – f ( x )] 2 + [ f ( x ) – g ( x )] 2 = 0. Then, find f ( x ).
Sol. Here, [ f ( x ) – f ( x )]2 + [ f ( x ) – g ( x )]2 = 0 is only possible,
Non-negative Functions if f ( x ) – f ( x ) = 0 and f ( x ) – g ( x ) = 0
The sum of several non-negative terms is zero if and only Þ f ( x ) = f ( x ) = g ( x ) or f ( x ) = x = | x |,
if each term is zero. which is only possible, if x is non-negative.
i.e. a2 + b2 + c2 = 0 Therefore, f ( x ) = x , " x Î [0, ¥ )
if a =b =c

Exercise for Session 4


1. Find all values of ‘m’ for which (2m - 3)x 2 + 2mx + 4 < 0 for all real x.

2. If ax 2 - bx + 5 = 0 does not have two distinct real roots, then find the minimum value of 5a + b .

3. If a, b, c ÎR, a ¹ 0 and the quadratic equation, ax 2 + bx + c = 0 has no real root, then show that a (a + b + c ) > 0.
2
x -1
4. If x, y Î[0, 10], then find the number of solutions ( x , y ) of the inequation 3sec × 9y 2 - 6y + 2 £ 1.
16 Textbook of Differential Calculus

#L Essential Mathematical Tools Exercise 1 :


Subjective Questions
n Directions (Q. Nos. 1 to 11) Solve each of the following 13. Solve the equation
system of equations :
2x – 1 + 3x – 2 = 4 x – 3 + 5x – 4 .
1. For a < 0 , determine all solutions of the equation
x 2 – 2a | x – a | – 3a 2 = 0. 14. If x , y and z are three real numbers such that
2. Solve | x + 4 x + 3 | + 2x + 5 = 0.
2 x + y + z = 4 and x 2 + y 2 + z 2 = 6, then show that each
3. Solve | x 2 – 3x – 4 | = 9 –| x 2 – 1 |. é2 ù
of x , y and z lies in the closed interval ê , 2ú .
ë3 û
4. Solve 2| x + 1| – 2 x = | 2 x – 1 | + 1.
15. If {(a + 1) (b – 1) + (b + 1) (a – 1)} a + (a – 1) (b – 1) = 0
5. Find the set of all real ‘ a’ such that 5a 2 – 3a – 2, a 2 + a – 2
and a (a + 1) (b + 1) – (a – 1) (b – 1) = 0
and 2a 2 + a – 1 are the lengths of the sides of a triangle?
ì a + 1 b + 1ü
6. Solve ( x + 3) 5 – ( x – 1) 5 ³ 244. Also, let A = í , ý
î a – 1 b – 1þ
7. Solve || x – 2 |– 1 | ³ 3.
ì 2a 2b ü
3x 2 – 7 x + 8 and B = í , ý. If A Ç B ¹ f , then find all the
8. Solve 1 £ £ 2. î a + 1 b + 1þ
x2 +1
permissible values of the parameter ‘a’.
2x 1
9. Let f ( x ) = and g ( x ) =
×
x –1 ( x – 1) 2
2x + 5x + 2
2 x +1 16. Solve + | 1– x | = +1
Find the set of real values of x for which f ( x ) > g ( x ). 3 + 2x – 8x 2 | 3 + 2x – 8x 2 |
10. For x Î R, || x || is defined as follows; (2 | x | - 2)
17. Let f ( x ) = ( x 2 - 2 | x | ) (2 | x | - 2) - 9 ×
ì x + 1, 0 £ x < 2 x 2 - 2|x |
|| x || = í
î| x – 4 |, 2 £ x Solve the following inequalities
Then, solve the equation, || x || + x = || x || + x .
2 2
(i) f ( x ) > 0 (ii) f ( x ) ³ 0
11. Solve the inequality | x - 1 | + | 2 - x | > 3 + x . (iii) f ( x ) < 0 (iv) f ( x ) £ 0
½ |x | ½ ½³ 1 .
12. Solve the equation 18. Solve the inequality½1 -
x + 12y + y + 12x = 33, x + y = 23.
2 2 ½ 1 + | x |½ 2

Essential Mathematical Tools Exercise 2 :


More Than One Correct Option Type Questions

19. If cos x - y 2 - y - x 2 - 1 ³ 0, then 21. If | ax 2 + bx + c | £ 1 for all x is [0, 1],


(a) y ³ 1 (b) x Î R (c) y = 1 (d) x = 0 then
(a) |a | £ 8
20. If (sin a )x 2 - 2x + b ³ 2 for all real values of x £ 1 and
(b) |b | £ 8
æ pö æp ö
a Î ç0, ÷ È ç , p ÷, then the possible real values of b is/are (c) |c | £ 1
è 2ø è2 ø
(d) | a | + | b | + | c | £ 17
(a) 2 (b) 3 (c) 4 (d) 5
Chap 01 Essential Mathematical Tools 17

Essential Mathematical Tools Exercise 3 :


Passage Based Questions
Passage I 30. The number of ordered pairs ( P, Q ) so that the number
(Q. Nos. 22 to 24) ‘N’ is divisible by 44, is
Let f ( x ) = ax 2 + bx + c ; a, b, c Î R (a) 2 (b) 3 (c) 4 (d) 5

It is given | f ( x ) | £ 1, " | x | £ 1 . Passage IV


Now, answer the following questions. (Q. Nos. 31 to 35)
8
22. The possible value of | a + c | , if a 2 + 2b 2 is maximum, is Consider the nine digit number n = 7 3 a 4 9 6 1 b 0.
3
given by 31. If p is the number of all possible distinct values of
(a) 1 (b) 0 (c) 2 (d) 3 (a - b ), then p is equal to
8
23. The possible value of | a + b | , if a 2 + 2b 2 is maximum, is (a) 17 (b) 18 (c) 19 (d) 20
3
given by 32. If q is the number of all possible values of b for which
(a) 1 (b) 0 (c) 2 (d) 3 the given number is divisible by 8, then q is equal to
8 (a) 2 (b) 3 (c) 4 (d) 5
24. The possible maximum value of a 2 + 2b 2 is given by
3 33. The number of ordered pairs (a , b ) for which the given
32 2 16
(a) 32 (b) (c) (d) number is divisible by 88, is
3 3 3
(a) 1 (b) 2 (c) 3 (d) 4
Passage II 34. The number of possible values of (a + b ) for which the
(Q. Nos. 25 to 27) given number is divisible by 6, is
Consider the equation | 2x | - | x - 4 | = x + 4. (a) 3 (b) 4 (c) 6 (d) 7

25. The least integer satisfying the equation, is 35. The number of possible values of b for which i N = 1
(a) – 4 (b) 4 (c) 5 (d) – 5 (where i = -1), is
(a) 2 (b) 3 (c) 4 (d) 5
26. Total number of prime numbers less than 20 satisfying
the equation, is
(a) 3 (b) 4 (c) 5 (d) 6
Passage V
(Q. Nos. 36 to 38)
27. If P = greatest composite number less than 34 satisfying The set of integers can be classified into k classes, according to the
the given equation, then P 2007 has the digit on its units remainder obtained when they are divided by k (where k is a fixed
place as natural number). The classification enables is solving even some
(a) 8 (b) 1 more difficult problems of number theory e.g.
(c) 7 (d) 0 (i) even, odd classification is based on whether remainder is
0 or 1 when divided by 2.
Passage III (ii) when divided by 3, the remainder may be 0, 1, 2. Thus,
(Q. Nos. 28 to 30) there are three classes.
Consider a number N = 2 1 P 5 3 Q 4.
36. The remainder obtained, when the square of an integer
28. The number of ordered pairs ( P, Q ) so that the number is divided by 3, is
‘N’ is divisible by 9, is (a) 0, 1 (b) 1, 2 (c) 0, 2 (d) 0, 1, 2
(a) 11 (b) 12
(c) 10 (d) 8
37. n + n + 1 is never divisible by
2

(a) 2 (b) 3
29. The number of values of Q so that the number ‘N’ is (c) 111 (d) None of these
divisible by 8, is
(a) 4 (b) 3
38. If n is odd, n 5 - n is not divisible by
(c) 2 (d) 6 (a) 16 (b) 15 (c) 240 (d) 720
18 Textbook of Differential Calculus

Essential Mathematical Tools Exercise 4 :


Single Integer Answer Type Questions
39. The number of solutions of the equation | x - 1 |- | 2x - 5 | ³ 4 41. The number of solutions of the system of equation
x + 2y = 6 and | x - 3 | = y is/are
40. The number of integral solution of the equation | x 2 - 7 | £ 9
are

Answers
Exercise for Session 1 Chapter Exercises
1. False 2. False 3. True 4. False 5. True 1. {(1 – 2)a, (–1 + 6 )a} 2. {–4 , – 1 – 3}
6. True 7. (a) 8. (c) 9. (a) 10. (b) 3. {–2, 2} 4. {–2} È [ 0, ¥ )
æ 3 + 57 5 + 17 ö
Exercise for Session 2 5. ç <a< ÷ 6. (– ¥ , – 2 ] È [ 0, ¥ )
è 8 4 ø
1. x Î ( - 1, 3)
æ 2 1ö
2. x Î ( - ¥, - 2) È (8, ¥) 7. (– ¥ , – 2 ] È [ 6, ¥ ) 8. [1, 6 ] 9. x Î (–2, – 1) È ç – ,– ÷
è 3 2ø
3. x Î (-2, 1) È (1, 4)
10. x ={0, 2} 11. (- ¥, 0) È (6, ¥)
4. x Î ( - ¥, 1] È é , ¥ö÷
3
êë 2 ø 12. (13, 10) and (10, 13) 13. x = 1
æ -1 - 29 ù é 1 - 5 1 + 5 ù é -1 + 29 ö 14. x, y, z Î é , 2ù
2
5. x Îç - ¥, úÈê 2 , 2 úÈê , ¥÷ êë 3 úû
è 2 û ë û ë 2 ø
15. a Î {-1, 1 + i , 1 - i}
Exercise for Session 3 ì 1 ± 129 3 ± 73 ü
16. x Î í0, 2, , ý
î 16 16 þ
1. x Î ( - ¥, - 2) È æç , 1ö÷ È (4, ¥)
1
è4 ø 17. (i) x Î (-¥, - 3) È (-2, - 1) È (1, 2) È (3, ¥)
-1 (ii) x Î (-¥, - 3] È (-2, - 1] È [1, 2) È [ 3, ¥)
2. x Î ( - ¥, - 1) È æç , 0ö÷ È æç , ¥ö÷
1
è 2 ø è2 ø (iii) x Î ( - 3, - 2) È (-1, 0) È (0,1) È (2, 3)
(iv) x Î[ - 3, - 2) È [ -1, 0) È (0,1] È (2, 3]
3. x Î[1, 3]
18. [ -1, 1]
4. x Î[ - ¥, - 6) È ( - 4, - 1) È (1, 0) 5. x Î R
19. (c,d)
6. (d) 7. (c) 8. (a) 9. (a) 10. (c) 11. (d) 20. (b,c,d)
12. (d) 21. (a,b,c,d) 22. (a) 23. (c) 24. (b)
25. (a) 26. (d) 27. (c) 28. (a) 29. (b) 30. (c)
Exercise for Session 4 31. (c) 32. (a) 33. (b) 34. (d) 35. (d) 36. (a)
1. m Îf 2. (-1) 4. (4) 37. (c) 38. (d) 39. (0) 40. (9) 41. (2)
Þ x = - 1 ± 3, as x Î [ -3, - 1 ]

Solutions \ Neglecting x = - 1 + 3
\ x Î { - 4, - 1 - 3 }.
3. | x - 3x - 4 | + | x 2 - 1 | = 9
2

Þ | x + 1 | {| x - 4 | + | x - 1 | } = 9
Þ x = -1 - 3 …(ii)

ì( x - a ), x ³ a x+1
1. For a < 0, | x - a | = í .
î(a - x ), x £ a
+ x–1
+ + x–4
Case I x ³ a +
+ +
x 2 - 2a( x - a ) - 3a 2 = 0 –1 1 4

Þ x 2 - 2ax - a 2 = 0
Þ x 2 - 2ax + a 2 = 2a 2
Case I x ³ 4
Þ ( x - a ) 2 = 2a 2 Þ x = ± 2a + a
(x + 1) { x - 4 + x - 1} = 9
Þ x = a(1 + 2 ), a(1 - 2 )
Þ ( x + 1 )(2 x - 5 ) = 9
Since, x ³ a and a < 0 Þ 2 x 2 - 3 x - 14 = 0
\ Neglecting {a(1 + 2 )} Þ x = a(1 - 2 )
7
Case II x £ a Þ x = , -2
2
Þ x 2 + 2a( x - a ) - 3a 2 = 0 Þ x 2 + 2ax = 5a 2 [neglecting both as x ³ 4]
Þ ( x + a ) 2 = 6a 2 Þ x = - a ± 6a Case II 1 < x < 4
Þ ( x + 1 ){ 4 - x + x - 1 } = 9
\ x = a( 6 - 1 ), - a( 6 + 1 )
Þ (x + 1) × 3 = 9
Since, x £ a and a < 0
Þ x = 2 Î (1, 4 ) …(i)
\ Neglecting { - a( 6 + 1 )} Þ x = a( 6 - 1 )
Case III - 1 < x < 1
Hence, x Î {a(1 - 2 ), a( 6 - 1 )}. ( x + 1 ){ 4 - x + 1 - x } = 9
2. | x + 4x + 3 | + 2x + 5 = 0
2 Þ ( x + 1 ) (5 - 2 x ) = 9
Take two cases, i.e.| x 2 + 4 x + 3 | = ± ( x 2 + 4 x + 3 ) Þ 2 x 2 - 3 x + 4 = 0, having imaginary roots.

Case I x 2 + 4x + 3 ³ 0 \ No solution.
Case IV x £ - 1
( x + 3 )( x + 1 ) ³ 0
- ( x + 1 ){ 4 - x + 1 - x } = 9
7
( x + 1 )(2 x - 5 ) = 9 Þ x = - 2, ×
– – + 2
–3 –1 As x £ - 1,
Þ x Ï ( -3, - 1 ) 7
\ x = - 2 , neglecting x = . …(ii)
or x Î ( - ¥, - 3 ] È [ -1, ¥ ) 2
Þ x 2 + 4x + 3 + 2x + 5 = 0 Thus, from Eqs. (i) and (ii), we get x Î { -2, 2}.
Þ x 2 + 6x + 8 = 0 4. 2 | x + 1 | - 2 x = | 2 x - 1 | + 1
Þ ( x + 4 )( x + 2 ) = 0 Case I x³0
Þ x = - 2, - 4. 2x + 1 - 2x = 2x - 1 + 1 Þ 2x × 2 = 2 × 2x
But x Ï ( -3, - 1 ) i.e. true for all x ³ 0 …(i)
\ Neglecting x = -2 Case II -1 £ x £ 0
Þ x = -4 …(i) 2+ x + 1 - 2x = 1 - 2x + 1
Case II x + 4 x + 3 £ 0
2
Þ 2x + 1 = 2
Þ x=0 …(ii)
– Case III x £ - 1
–3 –1 Þ 2-x - 1 - 2x = 1 - 2x + 1
( x + 3 )( x + 1 ) £ 0 Þ 2-x - 1 = 2
Þ x Î [ -3, - 1 ] Þ x = -2 …(iii)
Þ x 2 + 4x + 3 - 2x - 5 = 0 Thus, from Eqs. (i), (ii) and (iii), we get x Î { -2 } È [ 0, ¥ ).
Þ x 2 + 2x - 2 = 0
20 Textbook of Differential Calculus

5. If 5a 2 - 3a - 2, a 2 + a - 2, 2 a 2 + a - 1 are lengths of sides of Þ y 4 + 8y 2 - 9 ³ 0


triangle, then sum of any two sides must be greater than third Þ (y 2 + 9 )(y 2 - 1 ) ³ 0
side.
Case I (5a 2 - 3a - 2 ) + (a 2 + a - 2 ) > 2a 2 + a - 1 Þ y2 ³1

Þ 6a 2 - 2a - 4 > 2a 2 + a - 1 i.e. y £ - 1 or y ³ 1
Þ x + 1 £ - 1 or x + 1 ³ 1
Þ 4a 2 - 3a - 3 > 0 Þ x Î ( - ¥, - 2 ] È [ 0, ¥ )
ì æ 3 + 57 öü ì æ 3 - 57 öü 7. || x - 2 | - 1 | ³ 3
Þ ía - ç ÷ ý ía - ç ÷ý > 0
î è 8 ø þî è 8 øþ Þ | x - 2 | - 1 £ -3 or | x - 2 | - 1 ³ 3
+ – + Þ | x -2| £ -2 or | x - 2 | ³ 4
Þ No solution or x - 2 £ - 4 or x - 2 ³ 4
3 – 57 3 + 57
8 8
Þ x £ -2 or x ³ 6
Þ x Î ( - ¥, - 2 ] È [6, ¥ )
æ 3 - 57 ö æ 3 + 57 ö
Þ a Î ç - ¥, ÷ Èç , ¥÷ …(i) 3x 2 - 7x + 8
è 8 ø è 8 ø 8. 1 £ £2
x2 + 1
Case II (a 2 + a - 2 ) + (2a 2 + a - 1 ) > 5a 2 - 3a - 2 Þ 2 x 2 - 7 x + 7 ³ 0 and x 2 - 7 x + 6 £ 0
Þ 3a + 2a - 3 > 5a - 3a - 2
2 2
Þ x Î R and x Î[1, 6 ]
Þ 2a 2 - 5a + 1 < 0 \ x Î[1, 6 ]
2x 1
ì æ 5 + 17 öü ì æ 5 - 17 öü 9. f ( x ) > g( x ) Þ >
Þ ía - ç ÷ ý ía - ç ÷ý < 0 2x 2 + 5x + 2 x + 1
î è 4 øþ î è 4 øþ
2x 1
Þ - >0
+ – + (2 x + 1 )( x + 2 ) ( x + 1 )
5 – 17 5 + 17 2 x 2 + 2 x - (2 x 2 + 5 x + 2 )
4 4 Þ >0
(2 x + 1 )( x + 2 )( x + 1 )
æ 5 - 17 5 + 17 ö - (3 x + 2 )
Þ a Îç , ÷ …(ii) Þ >0
è 4 4 ø (2 x + 1 )( x + 2 )( x + 1 )
Case III (5a 2 - 3a - 2 ) + (2a 2 + a - 1 ) > (a 2 + a - 2 ) – + – + –
–2 –1 – 2/3 – 1/2
Þ 7a 2 - 2a - 3 > a 2 + a - 2
Þ 6a 2 - 3a - 1 > 0 Þ x Î ( -2, - 1 ) È ( -2 / 3, - 1 / 2 )
ì æ 3 + 33 öü ì æ 3 - 33 öü 10. || x | | + x = || x || + x 2 .
2

Þ ía - ç ÷ ý ía - ç ÷ý > 0 Case I When 0 £ x < 2


î è 12 ø þî è 12 øþ
Þ (x + 1)2 + x = (x + 1) + x 2
+ – +
Þ x 2 + 2x + 1 + x = x 2 + x + 1
3 – 33 3 + 33
12
Þ 2x = 0
12
Þ x=0 …(i)
æ 3 - 33 ö æ 3 + 33 ö Case II When x ³ 2
Þ a Î ç -¥, ÷ Èç , ¥÷ …(iii)
è 12 ø è 12 ø Þ (x - 4)2 + x = | x - 4 | + x 2
From Eqs. (i), (ii) and (iii), we get Now, if 2 £ x £ 4
æ 3 + 57 5 + 17 ö
a Îç , ÷ Þ ( x 2 - 8 x + 16 ) + x = 4 - x + x 2
è 8 4 ø
Þ 6 x = 12
6. ( x + 3) 5 - ( x - 1) 5 ³ 244 Þ x =2 …(ii)
(x + 3) + (x - 1) Again, if x³4
Let y = Þ y =x+1
2 Þ (x - 4)2 + x = x - 4 + x 2
\ (y + 2 ) 5 - (y - 2 ) 5 ³ 244 Þ x 2 - 8 x + 16 + x = x 2 + x - 4
Þ 2 { 5C1 × y 4 × 2 + 5C 3 × y 2 × 2 3 + 5C 5 × 2 5 } ³ 244 Þ 8 x = 20
5
Þ 2 {10y 4 + 80y 2 + 32 } ³ 244 Þ x= ,
2
Þ 4 {5y 4 + 40y 2 + 16 } ³ 244 but x ³ 4.
Þ 5y 4 + 40y 2 + 16 ³ 61 \ Only two solutions x = 0 and 2.
Chap 01 Essential Mathematical Tools 21

11. The points x = 1 and x = 2 divide the number axis into three 14. Here, x 2 + y 2 + ( 4 – x – y ) 2 = 6 (Q z = 4 - x - y )
intervals as follows : x + (y – 4 ) x + (y + 5 – 4y ) = 0
2 2

Since, x is real.
+ve
– ∞ –ve ∞ Þ D³0
1
Þ (y – 4 ) 2 – 4 (y 2 – 4y + 5 ) ³ 0
Þ – 3y 2 + 8y – 4 ³ 0
–∞ ∞
+ve 2 –ve Þ – (3y – 2 ) (y – 2 ) ³ 0
We solve the inequality on each intervals. – + –
If x < 1, then x - 1 < 0 and 2 - x > 0. 2/3 2
\ |x - 1 | + | 2 - x | > 3 - x 2
Þ £y £2
Þ 1 - x + 2 - x >3 + x 3
Þ 3 - 2x > 3 + x é2 ù
Similarly, we can show that x, y and z Î ê , 2 ú .
Þ x<0 …(i) ë3 û
If 1 £ x £ 2, then x - 1 ³ 0 and 2 - x ³ 0, we have a+1 b+1 1 ( a + 1 )(b + 1 ) 1
15. Here, + =– and =
x -1 + 2 - x >3 + x a–1 b–1 a ( a – 1 )(b – 1 ) a
Þ 1 > 3 + x Þ x < -2 æ 1ö 1
Now, the quadratic x 2 – ç– ÷ x + = 0 has roots
The system of inequalities obtained has no solution, for è aø a
1 £ x £ 2. If x > 2, then x - 1 > 0 and 2 - x < 0, we have æa + 1 b + 1ö
ç and ÷
x -1 + x -2 >3 + x è a–1 b–1ø
Þ 2x - 3 > 3 + x Þ x > 6 Þ ax + x + 1 = 0
2
…(i)
Combining the solutions obtained on all parts of the domain of 2a x a+1 1
Let x= Þ a= Þ =
admissible values of the given inequality, we get the solution a+1 2– x a–1 x –1
set ( - ¥, 0 ) Î (6, ¥ ).
1
Now, replacing x by in Eq. (i), we get a quadratic
12. Here, x 2 + 12(23 – x ) + (23 – x ) 2 + 12x = 33 (Q y = 23 - x ) x –1
Þ x 2 – 12 x + 276 + x 2 – 34 x + 529 = 33 2a 2b
whose roots are and .
a+1 b+1
Let a = x 2 – 12 x + 276
a 1
+ +1=0
and b = x 2 – 34 x + 529 …(i) (x – 1)2 (x – 1)
Þ a + b = 33 and a – b = 11(2 x – 23 )
2 2
Þ x2 – x + a = 0 ...(ii)
2 x – 23
a –b = Hence, Eqs. (i) and (ii) are the two quadratics whose roots are
3 elements of set ‘ A’ and set ‘ B’, respectively.
x + 38
\ a= …(ii) \ They must have a root in common as A Ç B ¹ f
3
2 Case I Both roots are common.
æ x + 38 ö
Þ x 2 – 12 x + 276 = ç ÷ [using Eqs. (i) and (ii)] Þ
a
=
1
=
1
è 3 ø
1 –1 a
Þ 8 x 2 – 184 x + 1040 = 0 Þ x = 13, 10 Þ a = – 1 and a 2 = 1 Þ a = – 1
Þ y = 10, 13 Case II For a common root,
\ (13, 10) and (10, 13) are the required solutions. x2 x 1
= =
13. Here, u + v = p + q ...(i) a + 1 1 – a 2 –(a + 1 )
Where u = 2 x – 1 , v = 3 x – 2 , p = 4 x – 3, Þ a =1 ± i
q = 5x – 4 \ a Î {– 1, 1 + i , 1 – i }
\ u2 – v 2 = 1 – x x –1 (x – 1)2
16. Here, + | 1 – x | = +1
p 2 – q 2 = 1 – x Þ u2 – v 2 = p 2 – q 2 3 + 2x – 8x 2 | 3 + 2x – 8x 2 |
x –1
Þ u – v = p – q …(ii) {using Eq. (i), u + v = p + q } Let a = ; b = ( x – 1 ),
\ From Eqs. (i) and (ii), we get 3 + 2x – 8x 2
2u = 2 p Þ 2 x – 1 = 4 x – 3 then | a | + | b | = | ab | + 1
or x = 1, which clearly satisfies Þ | ab | – | a | – | b | + 1 = 0
2x – 1 + 3x – 2 = 4x – 3 + 5x – 4 Þ (| a |– 1 ) (| b |– 1 ) = 0
\ | a | =1 or | b | = 1
\ x = 1 is the required solution.
22 Textbook of Differential Calculus

When | a | =1 We solve the first system :


x –1 ì½ x ½ 1 ì 1 1
Þ =1 ï½1 - ½ ³ ï ³
3 + 2x – 8x 2 í½ 1 + x½ 2 Û í1 + x 2
ï x ³0 ïî x ³ 0
x –1 x –1 î
= 1 or =–1
3 + 2x – 8x 2
3 + 2x – 8x 2 ì2 - 1 - x ì1-x
ï ³ 0 ï
Û í 1 + x Û í 1+ x Û 0 £ x £1
Þ x – 1 = 3 + 2x – 8x 2
ïî x ³ 0 ïî x ³ 0
or x – 1 = –3 – 2x + 8x 2
Again, we solve the second system. Its first inequality is
Þ 8 x 2 – x – 4 = 0 or 8 x 2 – 3 x – 2 = 0 ½ 1 ½ 1
equivalent to the inequality½ ½³ .
1 ± 1 + 128 3 ± 9 + 64 ½1 - x½ 2
Þ x= or x =
16 16 If x < 0, then 1 - x > 0 and consequently the second system is
1 ± 129 3 ± 73 equivalent to the system
Þ x= or x = ...(i)
16 16 ì 1 1 ì2 - 1 + x
ï ³ ï ³ 0
Again, when | b | = 1, | x – 1 | = 1 í1 - x 2 Û í 1-x
Þ x – 1 = 1 or x – 1 = – 1 ïî x £0 ïî x £ 0
Þ x = 0 or x = 2 …(ii) ì1 + x
ï ³0
From Eqs. (i) and (ii), solution set is Û í1 - x Û -1 £ x < 0
ì 1 ± 129 3 ± 73 ü îï x £0
x Î í 0, 2, , ý
î 16 16 þ Thus, the set of all solutions of the original inequality consists
of the numbers belonging to the interval [ -1, 1 ] .
(2 | x | - 2 )
17. We have, f ( x ) = ( x 2 - 2 | x | )(2 | x | - 2) - 9 ×
x2 - 2 |x | 19. Given, cos x - y 2 - y - x 2 - 1 ³ 0 ...(i)
ì 9 ü Clearly, y - x - 1 is defined when y - x - 1 ³ 0
2 2
= (2 | x | - 2 ) í x 2 - 2 | x | - 2 ý
î x - 2 | x |þ or y ³ x 2 + 1. So, minimum value of y is 1.
ì (x 2 - 2 | x | )2 - 9ü From Eq. (i), we have cos x - y 2 ³ y - x 2 - 1
= (2 | x | - 2 ) í ý
î x - 2 |x | þ
2
where cos x - y 2 £ 0 [as when cos x is maximum ( = 1 ) and y 2
(2 | x | - 2 ) ( x 2 - 2 | x | + 3 )( x 2 - 2 | x | - 3 )
= is minimum ( = 1 ). So, cos x - y 2 is maximum.]
x2 - 2 |x |
(2 | x | - 2 ) {(| x | - 1 ) 2 + 2 } {(| x | + 1 )(| x | - 3 )} Also, y - x2 -1 ³ 0
=
x2 - 2 |x | Hence, cos x - y 2 = y - x 2 - 1 = 0
Taking, N r = 0 Þ | x | = 1, 3 Þ y = 1 and cos x = 1, y = x 2 + 1
Þ x = ± 1, ± 3 and Dr = 0 Þ x = 0, y = 1
Þ | x |(| x | - 2 ) = 0 20. Given, (sin a ) x 2 - 2x + b ³ 2
Þ 0, ± 2 Let f ( x ) = (sin a ) x 2 - 2 x + b - 2
We mark these roots on a number line : 1
Abscissa of the vertex is given by x = >1
sin a
+ + +
–∞ – –2 – – – ∞ The graph of f ( x ) = (sin a ) x 2 - 2 x + b - 2, "x £ 1 is shown in
–3 –1 1 2 3
0
the figure.
Y
From the wavy curve method, we have
(i) f ( x ) > 0 Þ x Î ( - ¥, - 3 ) È ( -2, - 1 ) È (1, 2 ) È (3, ¥ )
(ii) f ( x ) ³ 0 Þ x Î ( - ¥, - 3 ] È ( -2, - 1 ] È [1, 2 ) È [3, ¥ )
(iii) f ( x ) < 0 Þ x Î ( -3, - 2 ) È ( -1, 0 ) È ( 0, 1 ) È (2, 3 )
(iv) f ( x ) £ 0 Þ x Î [ -3, - 2 ) È [ -1, 0 ) È ( 0, 1 ] È (2, 3 ]
18. The domain of admissible values of this inequality consists of X′ X
O 1 1
Y′ x=
all the real numbers. The inequality is equivalent to the sin a
collection of two systems : Therefore, minimum of f ( x ) = (sin a ) x 2 - 2 x + b - 2 must be
ì½ x ½ 1 ì½ -x ½ 1 greater than zero but minimum is at x = 1.
ï½1 - ½ ³ ï½1 - ½ ³
í½ 1 + x½ 2 , í½ 1 - x½ 2 i.e. sin a - 2 + b - 2 ³ 0,
ï x ³0 ï £ ì pü
î î x 0
Þ b ³ 4 - sin a, a Î ( 0, p ) - í ý
î 2þ
Chap 01 Essential Mathematical Tools 23

21. Given, | ax 2 + bx + c| £ 1, "x Î [ 0,1] ...(i) 26. Prime numbers less than 20 satisfying the equation are 5, 7, 11,
1 13, 17, 19 i.e. 6 prime number.
Putting successively x = 0, 1 and in Eq. (i), we get
2 27. P = 33, P 2007 = (33) 2007 = number has 7 at its units place.
-1 £ c £ 1 ...(ii)
Sol. (Q. Nos. 28-30)
-1 £ a + b + c £ 1 ...(iii)
28. Sum of digits = P + Q + 15
-4 £ a + 2b + 4c £ 4 ...(iv)
From Eqs. (ii), (iii) and (iv), we get N is divisible by 9, if P + Q + 15 = 18, 27
| b | £ 8 and | a | £ 8 Þ P +Q =3 …(i)
Þ | a | + | b | + | c | £ 17 or P +Q = 12 …(ii)
Sol. (Q. Nos. 22-24) P = 0, 1, 2, 3, 4, 5, 6, 7, 8, 9
P = 0, Q = 3ü
| f (1 ) - f ( 0 ) | £ 2 (Q if | u | £ 1, | v | £ 1, then | u - v | £ 2 )
Þ | a + b| £ 2 ìQ f (1 ) = a + b + cü P = 1, Q = 2ïï
í ý
From Eq. (i), we get ý Number of ordered pairs is 4.
Þ (a + b ) 2 £ 4 P = 2, Q = 1ï
î and f ( 0 ) = c þ
P = 3, Q = 0þï
Also, | f ( -1 ) - f ( 0 )| £ 2
Þ | a - b| £ 2 [Q f ( -1 ) = a - b + c and f ( 0 ) = c ] From Eq. (ii), we get
Þ (a - b ) 2 £ 4 P = 3, Q = 9ü
P = 4, Q = 8ï
Now, 4a 2 + 3b 2 = 2(a + b ) 2 + 2(a - b ) 2 - b 2 ïï
. . . . . . . . ý Number of ordered pairs is 7.
£ 2 ´ 4 + 2 ´ 4 - b 2 £ 16 - b 2 £ 16
P = 8, Q = 4ï
Þ Maximum value of 4a 2 + 3b 2 = 16, when b = 0 ï
P = 9, Q = 3ïþ
Þ |a + b | + |a - b | = 4 Þ |a | = 2
Total number of ordered pairs is 11.
Also, | f (1 ) - f ( 0 )| = | a + c - c | = | a| = 2
Þ | a + c| = | c| = 1 29. N is divisible by 8, if Q = 0, 4, 8
\ The possible values of (a, b, c ) are (2, 0, - 1 ) or ( -2, 0,1 ) Number of values of Q is 3.
8 2 2 30. SO = P + 9
Also, a 2 + 2b 2 = ( 4a 2 + 3b 2 ) £ (16 )
3 3 3 SE = Q + 6
8a 2 SO - S E = P - Q + 3
22. | a + c| = 1, when + 2b 2 is maximum. N is divisible is 11, if
3
P - Q + 3 = 0, 11
8a 2 P - Q = -3
23. | a + b| = 2, when + 2b 2 is maximum. …(i)
3 or P -Q =8 …(ii)
8a 2 32 N is divisible by 4, if
24. The maximum value of + 2b 2 is .
3 3 Q = 0, 2, 4, 6, 8
Sol. (Q. Nos. 25-27) From Eq. (i), Q = 0, P = - 3 (not possible)
25. Given, | 2x | - | x - 4 | = x + 4 Q = 2, P = - 1 (not possible)
2x Q = 4, P = 1
+ x– 4 Q = 6, P = 1
+ + Q = 8, P = 5
– –
0 4 \ Number of ordered pairs is 3.
– From Eq. (ii), Q = 0 P = 8
Q = 2 P = 10 (not possible)
Case I When, -¥ < x £ 0 Similarly, Q ¹ 4, 6, 8
-2 x - 4 + x = x + 4 \ Number of ordered pairs is 1.
Þ -2 x = 8 Þ x = -4 \ Total number of ordered pairs, so that the number N is
Case II When, 0 < x < 4 divisible by 44 is 4.
2x - (4 - x ) = x + 4 Sol. (Q. Nos. 31-35)
Þ 2x = 8 Þ x=4 31. If a = 0, then the possible values of a - b are
Case III When, 4 £ x < ¥ { 0, - 1, - 2, - 3, . . . , - 9 }
2x - x + 4 = x + 4 If a = 1 , then the possible values of a - b are
Þ x + 4 = x + 4 Þ x ÎR {1, 0, - 1, - 2, - 3, . . . , - 8 }
Solution of the given equation is { - 4 } È [ 4, ¥ ). If a = 2 , then the possible values of a - b are
\ Least integer = - 4. {2, 1, 0, - 1, - 2, - 3, . . . , - 7 } and so on
24 Textbook of Differential Calculus

If a = 9, then the possible values of a - b are 38. n = odd


{9, 8, 7, 6, 5, 4, 3, 2, 1, 0 } Let n = 2k + 1
Thus, in all the values of a - b are n 5 - n = n(n 4 - 1 ) = n(n - 1 )(n + 1 )(n 2 + 1 )
{ -9, - 8, - 7, - 6, - 5, - 4, - 3, - 2, - 1, 0, 1, 2, 3, . . . , 9 } = (2k + 1 )(2k )(2k + 2 )( 4k 2 + 4k + 2 )
\Total number of values of a - b is 19.
= 8 (k ) (k + 1 ) (2k + 1 ) (2k 2 + 2k + 1 )
\ p = 19
which is divisible by 15, 16 and 240 but not divisible by 720
32. Number N is divisible by 8, if 100 + 10 b is divisible by 8.
When n = 5 Þn(n 4 - 1 )
\ Possible values of b are 2, 6.
\ q =2 = 5 (5 4 - 1 ) = 5 (625 - 1 ) = 5 ´ 624 = 3120,
33. N = 7 3 a 4 9 6 1 b 0 which is not divisible by 720.
N is divisible by 88, if N is divisible by 8 as well as 11. 39. Given, | x - 1 | - |2x - 5 | ³ 4
N is divisible by 8, then b = 2, 6 x– 1
For divisibility by 11, + 2x – 5
SO = 17 + a , S E = 13 + b +
+
SO - S E = (17 + a ) - (13 + b ) = a - b + 4 –
1

5
a - b + 4 be the integer lying between –5 and 13. 2

But, a - b + 4 is divisible by 11, so a - b + 4 = 0, 11
Case I a -b + 4 = 0
when b = 2, a = - 2 (rejected) Case I When, x < 1
when b = 6, a = 2 1 - x - (5 - 2 x ) ³ 4 Þ x - 4 ³ 4 Þ x ³ 8
Case II a - b + 4 = 11 when a = 9, b = 2 but x <1
when b = 6, a = 13 (rejected) Þ No solution
\ ( a, b ) = (9, 2) (2, 6) Case II 1 £ x £5/2
\ Number of ordered pairs are 2. x - 1 + 2x - 5 ³ 4
Þ 3 x - 6 ³ 4 Þ 3 x ³ 10
34. N is divisible by 6, if it is divisible is 2 and 3.
10
N = 7 3 a 4 9 6 1 b 0 is divisible by 2 for all values of a and b. Þ x³ but 1 £ x £ 5 / 2
3
30 £ (30 + a + b ) Î I £ 48
Þ No such x exist.
But 30 + a + b is divisible by 3, if
Case III x ³5/2
30 + a + b = 30, 33, 36, 39, 42, 45, 48
Þ x - 1 - (2 x - 5 ) ³ 4 Þ x - 1 - 2 x + 5 ³ 4
a + b = 0, 3, 6, 9, 12, 15, 18
Þ -x + 4 ³ 4 Þ x£0
Number of possible values of ( a + b ) is equal to 7.
Þ No such x exist.
35. i N = 1 Þ N must be divisible by 4.
\ Number of solution is 0.
Þ b = 0, 2, 4, 6, 8
40. Given, | x 2 - 7| £ 9
Number of possible values of b is 5.
Þ -9 £ x 2 - 7 £ 9 Þ -2 £ x 2 £ 16
Sol. (Q. Nos. 36-38)
Þ 0 £ x 2 £ 16 Þ 0 £ | x| £ 4
36. Let n = 3k + 4, r = 0, 1, 2
n 2 = 9k 2 + 6kr + r 2 = 3m + r 2 Þ x Î [ -4, 4 ]
Hence, number of integral solution 9.
r 2 = 0, 1, 4
41. System of equations are x + 2y = 6 and | x - 3 | = y
\ Remainder = 0 or 1.
37. (a) n 2 + n + 1 is an odd number. Case I When x ³ 3, x + 2y = 6 and x - 3 = y
Solving these equations, we get x = 4,y = 1
(b) when n = 1, n 2 + n + 1 is divisible by 3.
Case II When x < 3, x + 2y = 6 and 3 - x = y
(c) 111 = 10 2 + 10 + 1 \ when n = 10
Solving these equations, we get x = 0,y = 3
n 2 + n + 1 is divisible by 111.
Hence, two solutions are ( 0,3 ) and ( 4, 1 ).
CHAPTER

02
Differentiation
Learning Part
Session 1
● Geometrical Meaning of the Derivative

● Derivative of f ( x ) from the First Principle or ab-Initio Method

● Rules of Differentiation

Session 2
● Chain Rule

Session 3
● Differentiation of Implicit Functions

Session 4
● Differentiation of Inverse Trigonometric Functions

● Graphical Approach for Differentiation of Inverse Trigonometric Functions

Session 5
● Differentiation of a Function in Parametric Form

Session 6
● Logarithmic Differentiation

● Differentiation of Infinite Series

Session 7
● Differentiation of a Function w.r.t. Another Function

Session 8
● Higher Derivatives of a Function

Session 9
● Differentiation of a Function given in the form of a Determinant

Session 10
● Derivative of an Inverse Function

Practice Part
● JEE Type Examples ● Chapter Exercises

Arihant on Your Mobile !


Exercises with this #L
symbol can be practised on your mobile. See title inside to activate for free.
Session 1
Geometrical Meaning of the Derivative,
Derivative of f(x) from the First Principle
or ab-Initio Method, Rules of Differentiation
Derivative is the instantaneous rate of change of a which means that the value of the derivative f ¢ ( x ) for
function with respect to dependent variable. Also, the a given value of x is equal to the tangent of the angle
derivative is the slope of tangent to a curve at a point. formed by the tangent line to the graph of the
function y = f ( x ) at the point P ( x , y ) with the
positive X-axis.
Geometrical Meaning of
the Derivative Relation between dy/dx and dx/dy
Let us consider a function y = f ( x ) in a rectangular Let x and y be two variables connected by a relation of the
coordinate system. We also consider a point P ( x , y ) on the form f ( x , y ) = 0.
curve. If a point corresponding to an increased value of the
argument x + D x is considered, its ordinate value is given Let D x be a small change in x and Dy be a small change in
byy + Dy = f ( x + D x ). y, then
dy Dy dx Dx
In figure, the point ( x + D x , y + D y ) is represented by A. = lim and = lim
dx D x ® 0 D x dy Dy ® 0 D y
A (x +∆ x, y + ∆y)
Y Dy D x
Now, × =1
∆y D x Dy
, y)
P (x
æ Dy D x ö
Þ lim ç × ÷ =1
y Dx ®0 è D x Dy ø
Ψ
X′
∆x X Dy Dx
O x Þ lim × lim =1 [Q D x ® 0 Û Dy ® 0]
Dx ®0 D x Dy ® 0 Dy
Y′
dy dx
Figure 2.1 Þ × =1
Here, PA is the secant to the curve. dx dy
Now, as D x ® 0, Dy ® 0 Þ PA ® 0 So,
dy
=
1
(i.e. the distance PA tends to zero or to a single point P) dx dx /dy
Þ lim (slope of chord PA) ® (slope of tangent at P).
Dx ®0
or tan y = lim tan q
D y dy
Dx ®0 Derivatives of f ( x ) from
Þ lim
D x ®0 Dx
=
dx
or f ¢ (x ) the First Principle
or ab-Initio Method or Delta
Y Tangent
Method or By Definition
P If f ( x ) is differentiable function, then
Dy f ( x + Dx ) - f ( x ) dy
lim = lim = f ¢ (x ) =
θ Dx ® 0 D x Dx ® 0 Dx dx
X
O
f (x + h) - f (x )
Figure 2.2 Simply, f ¢ ( x ) = lim
h ®0 h
Chap 02 Differentiation 27

y Example 1 Differentiate the following functions w.r.t. d f (x + h) - f (x ) sin( x + h ) - sin x


\ f ( x ) = lim = lim
x using first principle. dx h ® 0 h h ® 0 h
(i) f (x ) = tan x (ii) f (x )=e 2 x sin( x + h ) - sin x
= lim
2
h ® 0 h ( sin( x + h )) + sin x
(iii) e x (iv) sinx
[rationalising the numerator]
Sol. (i) Let f ( x ) = tan x
æh ö æ 2x + h ö
2 sin ç ÷ cos ç ÷
Then, f ( x + h ) = tan( x + h ) è2ø è 2 ø
d f (x +h )- f (x ) = lim
h ® 0 h ( sin( x + h + sin x )
\ (tan x ) = lim
dx h ®0 h
tan( x + h ) - tan( x ) é æC + D ö æC - D öù
= lim êQ sin C - sin D = 2 cos çè 2 ÷ø sin çè 2 ÷ø ú
h ®0 h ë û
tan( x + h - x )×[1 + tan x × tan( x + h )] sin h / 2 cos ( x + h / 2)
= lim = lim lim
h ®0 h / 2 ® 0 h / 2 h ® 0 sin ( x + h ) + sin x
h
[Q tan A - tan B = tan( A - B )×(1 + tan A tan B )] [as h ® 0 Þ h / 2 ® 0]
cos x cos x
tan h é tan h ù = 1× =
= lim ×(1 + tan 2 x ) êQ lim = 1ú 2 sin x 2 sin x
h ®0 h ë h ® 0 h û
= sec 2 x
\
d
(tan x ) = sec 2 x
Differential Coefficients
dx of Standard Functions
(ii) Let f ( x ) = e 2 x Þ f ( x + h ) = e 2 ( x +h ) d d
d f (x +h )- f (x )
(i) (constant ) = 0 (ii) ( x n ) = nx n – 1
\ ( f ( x )) = lim dx dx
dx h ®0 h d x d x
d 2x e 2 (x + h ) - e 2 x (e 2h - 1)
(iii) (| x | ) = (iv) (e ) = e x
Þ (e ) = lim = lim e 2 x × dx | x| dx
dx h ®0 h h ®0 h
d d 1
æ e 2h
- 1 ö é e x
-1 ù (v) (a x ) = a x log a (vi) (log e | x | ) =
= e 2 x × ç lim ÷ ´ 2 = 2e
2x
êQ lim = 1ú dx dx x
èh ®0 2h ø ë x ®0 x û d 1 d
(vii) (log a | x | ) = (viii) (sin x ) = cos x
d 2x dx x log e a dx
\ (e ) = 2e 2x
dx d d
2
+ h )2 (ix) (cos x ) = – sin x (x) (tan x ) = sec 2 x
(iii) Let f (x ) = e x Þ f ( x + h ) = e (x dx dx
+ h )2 2
d d
d f (x + h) - f (x ) e (x - ex (xi) (cot x ) = – cosec2 x (xii) (secx ) = sec x tan x
\ f ( x ) = lim = lim
dx h ®0 h h ®0 h dx dx
x 2 + h 2 + 2hx
-e x2 d
= lim
e (xiii) (cosec x ) = – cosec x cot x
h ®0 h dx
æ e h 2 + 2hx - 1 ö
d 1
x2 h 2 + 2hx (sin–1 x ) = , –1 < x < 1 or | x | < 1
2
e ×e - ex x2 (xiv)
= lim = lim e ç ÷ dx
h ®0 h ®0 ç ÷ 1– x 2
h è h ø
d –1
2 æ e h 2 + 2hx - 1 ö æ h 2 + 2hx ö (xv) (cos –1 x ) = , –1 < x < 1 or | x | < 1
= ex lim ç ÷´ç ÷ dx 1– x 2
h ® 0 ç h 2 + 2hx ÷
è ø è h ø
d 1
(xvi) (tan–1 x ) = , –¥ < x < ¥ or x Î R
2 æet - 1ö dx 1+ x2
= e x lim ç ÷ lim (h + 2x ), where t = h + 2hx
2
t ® 0è t ø h ®0 d 1
é ù
(xvii) (sec –1 x ) = , | x | >1 or x Î R –[–1,1]
2 ex - 1 dx
= e x ´ 1 ´ lim (h + 2x ) êQ lim = 1ú | x | x2 –1
h ®0
2 2 ë x ®0 x û d –1
= e x ´ 2x = 2xe x (xviii) (cot –1 x ) = , –¥ < x < ¥ or x Î R
(iv) Let f ( x ) = sin x
dx 1+ x2
d –1
Þ f ( x + h ) = sin( x + h ) (xix) (cosec –1 x ) = , | x | > 1 or x Î R –[–1,1]
dx | x | x2 –1
28 Textbook of Differential Calculus

Þ y = log10 x + loge 10 + 2
Rules for Differentiation On differentiating, we get
Here, we will discuss some rules which are related to dy d d d
= (log10 x ) + (loge 10) + ( 2)
differentiation. dx dx dx dx
1
= +0+0
Differentiation of Sum and x loge 10
Difference of Functions =
1
When a function is expressed as sum or difference of two x loge 10
or more functions then, to find the derivative of function,
sin x
we need to differentiate each term seperately. i.e. y Example 5 If y = x –1/ 2 + log 5 x + + 2x ,
cos x
If y = u ( x ) ± v ( x ) ± w ( x ) ± ... , then
dy
dy du ( x ) dv ( x ) dw ( x ) then find .
= ± ± ± ... dx
dx dx dx dx
Sol. Here, y = x –1/ 2 + log 5 x + tan x + 2x
is known as term-by-term differentiation.
On differentiating, we get
dy
y Example 2 If y = sin x + e , then find .
x
dy d d d d x
dx = ( x )–1/ 2 + (log 5 x ) + tan x + (2 )
dx dx dx dx dx
Sol. Here, y = sin x + e x 1 1
= – ( x )–1/ 2 –1 + + sec 2 x + 2x log 2
On differentiating both sides w.r.t. x, we get 2 x loge 5
dy d 1 1
= (sin x + e x ) = – x –3 / 2 + + sec 2 x + 2x log 2
dx dx 2 x loge 5
dy d d x
Þ = sin x + e
dx dx dx

Þ
dy
= cos x + e x
Differentiation of a Function
dx Multiplied with a Constant
y Example 3 If y = x 2 + sin –1 x + log e x , If a function multiplied by a constant, then the derivative
dy of constant times a function is the constant times the
then find . derivative of the function. i.e.
dx
If y = k f ( x ), then on differentiating, we get
Sol. Here, y = x 2 + sin –1 x + loge x
dy d
On differentiating, we get =k × f (x )
dx dx
dy d d d
= (x 2 ) + (sin –1 x ) + (loge x ) dy
dx dx dx dx y Example 6 If y = m 2 sec - 1 x , then find .
dy 1 1 dx
Þ = 2( x ) 2 – 1 + +
dx 1– x2 x Sol. Here, y = m 2 sec - 1 x
dy 1 1 On differentiating, we get
\ = 2x + +
dx 2 x
1– x dy d
= m 2 sec -1x
dx dx
y Example 4 If y = log 10 x + log e 10 + log x x + log 10 10 , 1
= m2 ´
dy | x | x2 - 1
then find .
dx
m2
=
Sol. Here, y = log10 x + loge 10 + log x x + log10 10 | x | x2 - 1
Þ y = log10 x + loge 10 + 1 + 1 [Q loga a = 1]
Chap 02 Differentiation 29

y Example 7 If y = log x 3 + 3 sin –1 x + kx 2 ,


dy
Differentiation of Product of
then find .
dx More than Two Functions
(i) If 3 functions are involved, then remember
Sol. Here, y = log x 3 + 3 sin –1 x + kx 2
D { f ( x ) × g( x ) × h ( x )} = f ( x ) × g( x ) × h ¢ ( x )
On differentiating, we get
dy d d d + f ( x ) × g ¢ ( x ) × h( x ) + f ¢ ( x ) × g( x ) × h( x )
= [log x 3 ] + [3 sin –1 x ] + [kx 2 ]
dx dx dx dx (ii) The result can be generalised to product of n terms as
d d
= 3 [log x ] + 3 (sin –1 x ) + k
d
(x 2 ) D { f 1 ( x ) f 2 ( x ) f 3 ( x ) ¼ f n ( x )}
dx dx dx
= [ f 1¢( x ) × f 2 ( x ) × f 3 ( x ) ¼ f n ( x )]
[Q log x n = n log x ]
1 1 3 3 + [ f 1 ( x ) × f 2¢( x ) f 3 ( x ) ¼ f n ( x )]
= 3× + 3× + k ( 2x ) = + + 2kx + [ f 1 ( x ) f 2 ( x ) f 3¢ ( x ) ¼ f n ( x )] +¼
x 1– x 2 x 1 - x2
+ [ f 1 ( x ) f 2 ( x ) f 3 ( x ) ¼ f n ¢ ( x )]

Product Rule y Example 10 Let f , g and h be differentiable


If u ( x ) and v ( x ) are two differentiable functions, then functions. If f (0) = 1, g (0) = 2, h(0) = 3 and the
u ( x ) × v ( x ) is also differentiable. derivative of their pairwise product at x = 0 are
If y = u ( x ) × v ( x ), then
( fg )¢ (0) = 6 ; ( gh )¢ (0) = 4 and (hf )¢ (0) = 5, then
dy ì d ü ìd ü
= í u ( x )ý × v ( x ) + u ( x ) × í v ( x )ý. compute the value of ( fgh )¢ (0).
dx îdx þ îdx þ
which is known as product rule. ( fg )¢ h + ( gh )¢ f + (hf )¢ g
Sol. We know that, ( fgh )¢ =
2
i.e. derivative of product of two functions
é( fg )¢ (0) × h (0) + ( gh )¢ (0) × f (0) ù
= (first function) ´ (derivative of second function) ê + (hf )¢ (0) × g (0) úû
+ (second function) ´ (derivative of first function) \ ( fgh )¢ (0) = ë
2
dy
y Example 8 If y = e x sin x , then find . ( 6) × ( 3) + ( 4 ) × ( 1) + ( 5) × ( 2)
= = 16
dx 2
Sol. Here, y = e x sin x
y Example 11 If y = (1 + x ) (1 + x 2 ) (1 + x 4 ) K (1 + x 2n ) ,
On differentiating, we get dy
dy ì d x ü ìd ü then find at x = 0.
= í (e )ý × sin x + e x × í (sin x )ý dx
dx î dx þ î dx þ
Sol. Here, y = (1 + x ) (1 + x 2 ) (1 + x 4 ) K (1 + x 2n )
= e x × sin x + e x × cos x = e x (sin x + cos x )
On differentiating, we get
dy dy
y Example 9 If y = e x tan x + x × log e x , then find . = (1) (1 + x 2 ) (1 + x 4 ) K (1 + x 2n )
dx dx
Sol. Here, y = e x tan x + x × loge x + (1 + x ) (2x ) (1 + x 4 ) K (1 + x 2n )
On differentiating, we get + (1 + x ) (1 + x 2 ) ( 4 x 3 ) K (1 + x 2n )
dy d x d +¼+ (1 + x ) (1 + x 2 ) (1 + x 4 ) (1 + x 8 )
= (e tan x ) + ( x log x )
dx dx dx
K (1 + x 2n - 1 ) ´ 2n × x 2n - 1
ìd ü ìd ü ìd ü
= í e x ý × tan x + e x × í tan x ý + í x ý × log x æ dy ö
î dx þ î dx þ î dx þ Now, ç ÷ =1
ìd ü è dx ø x
+ x ×í log x ý =0
î dx þ
1
= e x × tan x + e x × sec 2 x + 1 × log x + x ×
dy
x Quotient Rule
Hence, = e x (tan x + sec 2 x ) + (log x + 1)
dx If u ( x ) and v ( x ) are two differentiable functions such that
u (x )
v ( x ) ¹ 0, then is also differentiable.
v (x )
30 Textbook of Differential Calculus

u (x ) (a 3 - 3ax 2 ) (3a 2 - 3x 2 ) - (3a 2 x - x 3 ) ( - 6ax )


If y = , then =
v (x ) (a 3 - 3ax 2 )2
ìd ü ìd ü
í u ( x )ý × v ( x ) – í v ( x )ý × u ( x ) 3ax 4 + 6a 3 x 2 + 3a 5
dy îdx þ îdx þ =
= (a 3 - 3ax 2 )2
dx 2
{ v (x ) }
is known as the quotient rule of differentiation. e x – tan x dy
y Example 15 If y = , then find .
Reader’s can learn the given formula in following ways x + cot x
n dx
Derivative of quotient of two functions
e x – tan x
(denominator) ´ (derivative of numerator) Sol. Here, y=
x n + cot x
- (numerator) ´ derivative of denominator
= On differentiating, we get
(denominator) 2
é n d x ù
ê( x + cot x ) dx (e – tan x ) – (e – tan x ) ú
x
dy x
y Example 12 If y = 2 , then find × ê
d
ú
x +1 dx ê ( x n + cot x )ú
x dy ë dx û
Sol. Here, y = =
x +1 2 dx ( x n + cot x )2
On differentiating, we get
é(e x – sec 2 x )( x n + cot x ) – (e x – tan x ) ù
d d 2
( x 2 + 1) ( x ) - x ( x + 1) ê ú
dy
= dx dx dy êë (nx n – 1 – cosec 2 x ) úû
\ =
dx ( x + 1)
2 2
dx ( x n + cot x )2
( x 2 + 1)×1 - x ×(2x ) 1- x 2
= = log x
( x + 1)
2 2
(1 + x 2 )2 y Example 16 If y = + e x sin x + log 5 x ,
x
x sin x dy dy
y Example 13 If y = , then find . then find .
log e x dx dx
log x
Sol. Here, y =
x sin x Sol. Here, y = + e x sin x + log 5 x
loge x x

On differentiating, we get On differentiating, we get


d d dy d æ log x ö d x d
loge x ( x sin x ) - x sin x (loge x ) = ç ÷+ (e sin x ) + (log 5 x )
dy
= dx dx dx dx è x ø dx dx
dx (loge x )2
æ1ö ìd ü ìd ü
loge x ( x cos x + sin x ) - x sin x ç ÷ í (log x )ý × x – log x í x ý
èx ø =î þ î dx þ
dx
= 2
(loge x ) 2 x
loge x ( x cos x + sin x ) - sin x ìd ü ìd ü 1
= + í e x ý × sin x + e x × í sin x ý +
(loge x ) 2 î dx þ î dx þ x loge5

1
3a 2 x - x 3
dy × x – log x × 1
1
y Example 14 If y = 3 , then find . = x + e x sin x + e x × cos x +
a - 3ax 2 dx x 2
x loge 5

3a 2 x - x 3 dy æ 1 – log x ö x 1
Sol. Here, y= Hence, =ç ÷ + e (sin x + cos x ) +
a - 3ax
3 2 dx è x 2 ø x loge 5
On differentiating, we get
Remark
é 3 d d ù
(a -3ax 2 ) (3a 2 x - x 3 ) - (3a 2 x - x 3 ) (a 3 - 3ax 2 )ú While applying the quotient rule, think twice and check whether
dy ëê dx dx û your function could be simplified prior to differentiation.
=
dx (a 3 - 3ax 2 )2 (See Examples 17 and 18).
Chap 02 Differentiation 31

x4 + x2 +1
dy Þ y=
(sec x + tan x ) - (sec 2 x - tan 2 x )
y Example 17 If y = 2 and = ax + b , then -sec x + tan x + 1
x + x+1 dx
find a and b . (Qsec 2 x - tan 2 x = 1)
x4 + x2 + 1 (sec x + tan x ){1 - sec x + tan x }
Sol. Here, y = Þ y=
x + x +1
2 tan x - sec x + 1
dy
x 4 + 2x 2 + 1 - x 2 ( x 2 + 1) 2 - x 2 Þ y = sec x + tan x Þ = sec x tan x + sec 2 x
Þ y= = dx
x2 + x + 1 ( x 2 + x + 1)
æ dy ö
\ ç ÷ = 2 +2
( x 2 + x + 1)( x 2 - x + 1) è dx ø at x p
\ y= =
( x 2 + x + 1) 4

[Qa 2 - b 2 = (a + b )(a - b )] tan -1 x - cot -1 x æ dy ö


y Example 19 If y = -1 -1
, then find ç ÷
Þ y = x2 - x + 1 Þ
dy
= 2x - 1 tan x + cot x è dx ø
dx
at x = -1.
é dy ù
Þ ax + b = 2x - 1 êëQ dx = ax + b úû tan -1 x - cot -1 x p
Sol. Here, y= -1 -1
, as tan -1 x + cot -1 x =
tan x + cot x 2
\ a = 2, b = - 1
-1
tan x - ( p / 2 - tan -1 x )
sec x + tan x - 1 dy \ y=
y Example 18 If y = , then find ( p / 2)
( - sec x ) + tan x + 1 dx pù dy 2 é 2 ù
p 2 é -1
Þ y= êë2 tan x - 2 úû Þ =
at x = . p dx p êë 1 + x 2 úû
4
sec x + tan x - 1 æ dy ö 2
Sol. Here, y= \ ç ÷ =
-sec x + tan x + 1 è dx ø x = -1 p

Exercise for Session 1


n
Directions (Q. Nos. 1 to 9) Differentiate the following functions w.r.t. x.

1 - cos 2x
1. e x log a + e a log x + e a log a 2.
1 + cos 2x

3. log3 x + 3 loge x + 2 tan x

4. | x | + a 0 x n + a1 x n - 1 + a 2 x n - 2 + . . . + a n - 1x + a n

æ x + 1ö -1 æ x - 1ö
5. sec-1 ç ÷ + sin ç ÷
è x - 1ø è x + 1ø

2x cot x
6. x n loga x e x 7.
x
sin x - x cos x
8.
x sin x + cos x
x x 2 x 3 x4 dy
9. If y = 1 + + + + + . . . ¥, then find the value of .
1! 2 ! 3! 4! dx
x4 x 3 x4
10. Find the values of ‘x’ for which the rate of change of + - x is more than the rate of change of .
4 3 4
Session 2
Chain Rule (Derivative of Composite Function)

Chain Rule Sol. Here, y = e (tan x )


–1 3

(Derivative of Composite Function) On differentiating, we get


–1 3
dy d {e (tan x ) } d {(tan –1 x )3 } d (tan –1 x )
If u ( x ) and v ( x ) are differentiable functions, then uov ( x ) = × ×
or u[v ( x )] is also differentiable. dx d {(tan –1 x )3 } d (tan –1 x ) dx

If y [uov ( x )] = [u { v ( x )}] , then –1


x )3 1
[by chain rule]
= e (tan × 3 (tan –1 x )2 ×
dy du { v ( x )} d 1+ x 2
= ´ v (x ) Aliter
dx d { v ( x )} dx -1
x )3
Here, y = e (tan
is known as chain rule. Or
dy dy du Put (tan -1 x ) = v and (tan -1 x )3 = u
If y = f (u ) and u = g( x ), then = ×
dx du dx Þ u = v3
The chain rule can be extended as follows \ y = e u , u = v 3 and v = tan -1 x
If y [uovow ( x )] = u [ v { w ( x )}] , then On differentiating, we get
dy du [v { w ( x )}] d v { w ( x )} d w ( x ) dy dy du dv
= ´ ´ = × ×
dx d v { w ( x )} d w (x ) dx dx du dv dx
dy d u d (v )3 d 1
dy = e ´ ´ tan -1 x = e u × 3v 2 ×
y Example 20 If y = log (sin x ), then find . dx du dv dx 1 + x2
dx -1 1
Sol. Here, y = log (sin x ) = e (tan x )3
× (3 tan -1 x )2 ×
1 + x2
On differentiating, we get
dy d [log (sin x )] d
= ´ (sin x ) =
1
´ cos x y Example 22 If y = log e (tan –1 1 + x 2 ) , then
dx d (sin x ) dx sin x dy
[by chain rule] find .
dy dx
Hence, = cot x
dx Sol. Here, y = loge (tan –1 1 + x 2 )
Aliter
On differentiating, we get
Here, y = log (sin x )
dy d [ loge (tan –1 1 + x 2 )] d (tan –1 1 + x 2 )
Put sin x = u , we have, y = log u where u = sin x = ×
On differentiating, we get
dx d (tan –1 1 + x 2 ) d( 1 + x2 )
dy dy du
= × d ( 1 + x 2 ) d (1 + x 2 )
[by chain rule] × ×
dx du dx d (1 + x 2 ) dx
dy d d
= (log u ) × (sin x ) =
1
×
1
×
1
× 2x
dx dx dx -1
tan ( 1+ x ) 1+( 1+ x )2 2 2
2 1 + x2
1
= ´ cos x x
u =
-1
dy 1 tan ( 1 + x ) (2 + x 2 ) 1 + x 2
2

= ´ cos x = cot x [Qu = sin x ]


dx sin x
–1 dy Aliter
x )3
y Example 21 If y = e (tan , then find . Here, y = loge (tan - 1 1 + x 2 )
dx
Chap 02 Differentiation 33

Put 1 + x 2 = w , 1 + x 2 = v From now and onwards, we apply chain rule directly.

Þ v = w and tan - 1 1 + x 2 = u y Example 24 Differentiate the following w.r.t. x.


ì æx2 öü æ x + x 2 -a2 ö
Þ u = tan - 1 v (i) log ísin ç - 1÷ ý (ii) log e ç ÷
ç x - x 2 -a2 ÷
\ y = log u , u = tan - 1 v î è 3 øþ è ø
v = w and w = 1 + x 2 ìï æx2 öüï
Sol. (i) Let y = log ísin ç - 1÷ý
On differentiating, we get è 3 øïþ
îï
dy dy du dv dw
= × × × On differentiating, we get
dx du dv dw dx
dy 1 1
d d d d = ´
= (log u ) ´ tan - 1 v × x× (1 + x 2 ) dx ïì æx 2 öïü æ x2 ö
du dv dw dx 2 log ísin ç - 1÷ý sin ç - 1÷
1 1 1 ïî è 3 øïþ è 3 ø
= ´ ´ ´ ( 2x )
u 1+v 2
2 w æx2 ö 2x
´ cos ç - 1÷ ´
=
1
´
1
´
1
´ ( 2x ) è 3 ø 3
tan -1
1+ x 2 2 + x2 2 1 + x2 æx2 ö
x cot ç - 1÷
1 1 x è 3 ø
= ´ ´ =
tan -1
1+ x 2 2 + x2 1 + x2 ìï æx2 öüï
3 log ísin ç - 1÷ý
x ïî è 3 øïþ
=
(tan –1
1 + x )(2 + x 2 ) 1 + x 2
2 æ x + x 2 - a2 ö
(ii) Let y = loge ç ÷
ç x - x 2 - a2 ÷
dy è ø
y Example 23 If y = e ax × cos (bx + c ), then find .
dx Þ y = log ( x + x 2 - a 2 ) - log ( x - x 2 - a 2 )
Sol. Here, y = e ax × cos (bx + c ) é a ù
On differentiating, we get êëQ log b = log a - log b úû
dy ì d ax ü ìd ü On differentiating, we get
= í (e )ý × cos (bx + c ) + e ax × í {cos (bx + c )}ý
dx î dx þ î dx þ dy æç 1 öæ
÷ ç1 + 2x ö
÷
=
d (e ax ) d (ax ) d cos (bx + c ) d (bx + c ) dx çè x + x 2 - a 2 ÷ø çè 2 x 2 - a 2 ÷ø
= × × cos (bx + c ) + e ax × ×
d (ax ) dx d (bx + c ) dx éæ öæ öù
1 2x
= e ax × a × cos (bx + c ) + e ax × {–sin (bx + c )} × b - êç ÷ ç1 - ÷ú
ç 2 ÷ ç
êë è x - x - a ø è
2
2 x 2 - a2 ÷ú
øû
= ae ax cos (bx + c ) – be ax sin (bx + c ) 1 1 2
= + =
x -a
2 2
x -a
2 2
x - a2
2

Remark

Exercise for Session 2


n
Directions (Q. Nos. 1 to 19) Differentiate the following w.r.t. x.
1. (x 2 + x + 1)4 2. x2 + x + 1
1
3. sin3 x 4.
a2 - x 2

æ a + b cos x ö
5. ex sin x
6. sin-1 ç ÷ ,b > a > 1
è b + a cos x ø
x
7. ee 8. log ( x + a 2 + x 2 )
34 Textbook of Differential Calculus

æ a + b sin x ö 1 + sin x
9. log ç ÷ 10. log
è a - b sin x ø 1 - sin x

e x + log x
11. 12. sin (m sin-1 x ), | x | < 1
sin 3x
-1
x )2 cos-1 ( 1 - x 2 )
13. a(sin ,| x | < 1 14. e ,| x | < 1
x sin-1 x
15. + log 1 - x 2 , | x | < 1 16. log10 x + logx 10 + logx x + log10 10
1- x 2

a2 + x 2 + a2 - x 2
53 - x + (3 - x 2 )5
2
17. 18.
a2 + x 2 - a2 - x 2

19. 4 + 4 + 4 + x2

Multiple Choice Questions


20. The differential coefficient of f (loge x ) w.r.t. x, where f ( x ) = loge x , is
x 1 1
(a) (b) loge x (c) (d) None of these
loge x x x loge x

21. If f ( x ) = loge | x |, x ¹ 0, then f ¢( x ) is equal to


1 1 1
(a) (b) (c) - (d) None of these
|x | x x

22. Let f ( x ) = sin x , g( x ) = x 2 and h( x ) = log x . If F ( x ) = h(f ( g( x ))), then F ¢( x ) is


(a) 2x cot x 2 (b) 2 cosec3 x (c) -2 cosec2 x (d) None of these
æpö
23. If f ( x ) = cos x × cos 2x × cos 4x × cos 8x × cos 16x , then f ¢ ç ÷ is
è4ø
1
(a) 2 (b) (c) 1 (d) None of these
2
æ 3x + 4 ö dy
24. If y = f ç ÷ and f ¢ ( x ) = tan x , then
2
is equal to
è 5x + 6 ø dx
2
æ 3x + 4 ö 1 æ 3 tan x 2 + 3 ö
(a) -2 tan ç ÷ × (b) f çç ÷÷ tan x 2
è 5x + 6 ø (5x + 6)2 è 5 tan x + 6 ø
2

æ 3x + 4 ö
(c) 2x tan ç ÷ (d) tan x 2
è 5x + 6 ø

dy 2p
25. If y = | cos x | + | sin x |, then at x = is
dx 3
1 1
(a) ( 3 + 1) (b) 2 ( 3 - 1) (c) ( 3 – 1) (d) None of these
2 2
æ pö
26. If f ¢ ( x ) = sin x + sin 4x × cos x , then f ¢ ç2x 2 + ÷ is
è 2ø
(a) 4x {cos (2x 2 ) - sin 8x 2 × sin 2x 2 } (b) 4x {cos (2x 2 ) + sin 8x 2 × cos 2x 2 }
(c) {cos (2x 2 ) - sin 8x 2 × sin 2x 2 } (d) None of these
dy
27. If f ¢ ( x ) = 2x 2 - 1 and y = f ( x 2 ), then at x = 1, is
dx
(a) 1 (b) –1 (c) –2 (d) 2
Session 3
Differentiation of Implicit Functions

Differentiation of Implicit Differentiating both sides, we get

Functions d
dx
(x 2 ) +
d
dx
(y 2 ) +
d
dx
( xy ) =
d
dx
( 2)
In previous rules, we have studied functions in which
y was only expressed in terms of x without complication dy ì d x ü ìd ü
Þ 2x + 2y + í ýy + x í yý = 0
called explicit functions. dx î dx þ î dx þ
But, if the relation between the variables x and y are given dy dy
Þ 2x + 2y + 1 ×y + x × =0
by an equation and this equation is not immediately dx dx
solvable for y, then y is called an implicit function of x. dy
Þ (2y + x ) = – ( 2x + y )
Implicit functions are given by f ( x , y ) = 0. dx
dy dy ( 2x + y )
In this case, to find , we differentiate both sides of the Þ =–
dx dx (2y + x )
given relation with respect to x and collect the terms Aliter
dy Put f = x 2 + y 2 + xy – 2
containing to the left hand side and transpose the
dx ¶f

= ¶x
other terms to the right hand side. Now, divide both sides dy
Now, ...(i)
dy dx ¶f
by coefficient of .
dx ¶y
¶f
where, = 2x + 0 + y – 0 = 2x + y
¶x
Shortcut for Differentiation ¶f
and = 2y + x
of Implicit Functions ¶y
(Only to be Applied while Solving Objective MCQs) Substituting in Eq. (i), we get
d – ¶f / ¶x dy ( 2x + y )
For implicit function put; { f (x , y ) } = , =–
dx ¶f / ¶y dx (2y + x )
¶f y Example 26 If y = x cos y + y cos x ,
where is a partial differentiation of given function
¶x
dy
w.r.t. x (i.e. differentiating f w.r.t. x keeping y as then find ×
¶f dx
constant) and means partial differentiation of given
¶y Sol. Here, y = x cos y + y cos x
function w.r.t. y (i.e. differentiating f w.r.t. y keeping x Differentiating both the sides, we get
as constant). In simple words, we write this method as dy ì d ü ìd ü
= í ( x )ý cos y + x í (cos y )ý
for implicit functions dx î dx þ î dx þ
Differentiation of function w.r.t. x ìd ü ìd ü
+ y í (cos x )ý + í (y )ý × cos x
dy keeping y as constant î dx þ î dx þ
=
dx Differentiation of function w. r. t . y dy dy dy
Þ = 1 × cos y + x (–sin y ) + y (–sin x ) + (cos x )
keeping x as constant dx dx dx
dy dy
Þ (1+ x sin y –cos x ) = cos y –y sin x
y Example 25 If x 2 + y 2 + xy = 2, then find . dx
dx dy cos y –y sin x
Sol. Here, x 2 + y 2 + xy = 2 \ =
dx 1 + x sin y – cos x
36 Textbook of Differential Calculus

Aliter y Example 28 If 1 – x 6 + 1 – y 6 = a 3 ( x 3 – y 3 ), then


¶f
Let f = x cos y + y cos x – y Þ = cos y – y sin x
¶x dy x 2 1– y 6
¶f prove that = .
and = – x sin y + cos x –1 dx y 2 1– x 6
¶y
æ ¶f ö Sol. Here, 1 – x 6 + 1 – y 6 = a 3 ( x 3 – y 3 )
–ç ÷
dy è ¶x ø – (cos y – y sin x ) Let x 3 = sin q, y 3 = sin f , then we get
\ = =
dx æ ¶f ö (–x sin y + cos x – 1)
ç ÷ 1 – sin 2 q + 1 – sin 2 f = a 3 (sin q – sin f)
è ¶y ø
dy cos y – y sin x Þ cos q + cos f = a 3 (sin q – sin f )
or =
dx 1 + x sin y – cos x æq + fö æq – fö 3 é æq + fö æq – föù
Þ 2 cos ç ÷ × cos ç ÷ =a ê2 cos ç ÷ sin ç ÷
è 2 ø è 2 ø ë è 2 ø è 2 ø úû
y Example 27 If y 1 – x 2 + x 1 – y 2 = 1, when | x | < 1 æq – fö 3 æq – fö æq – fö 3
Þ cos ç ÷ = a sin ç ÷ Þ cot ç ÷ =a
dy è 2 ø è 2 ø è 2 ø
and | y | < 1, then find .
dx Þ q – f = 2 cot –1(a 3 )
Sol. Here, y 1 – x + x 1 – y = 1, | x | < 1 and | y | < 1
2 2
or sin –1 x 3 – sin –1 y 3 = 2 cot –1(a 3 )
Let x = sin q , y = sin f , then é æC + D ö æC - D ö ù
sin f 1 – sin q + sin q 1 – sin f = 1
2 2 êQ cos C + cos D = 2 cos çè 2 ÷ø cos çè 2 ÷ø ú
ê ú
Þ sin f cos q + sin q cos f = 1 ê æC + D ö æC - D öú
ê and sin C - sin D = 2 cos çè 2 ÷ø sin çè 2 ÷ø ú
Þ sin (q + f ) = 1 ë û
Þ q + f = sin –1(1) Differentiating both the sides, we get
1 1 dy
Þ sin x + sin y = sin (1)
–1 –1 –1
× 3x 2 – × 3y 2 × =0
6 6 dx
1– x 1–y
éQsin q = x Þ q = sin -1 x ù
ê -1
ú dy x 2 1 – y 6
êë and sin f = y Þ f = sin y úû Hence, =
dx y 2 1 – x 6
Differentiating both the sides, we get
1 1 dy
+ × =0
1– x 2
1–y 2 dx y Example 29 If y = sin x + sin x + sin x + ... ¥ ,
dy
Hence,
dy
=–
1–y2 where sin x > 0, then find .
dx 1– x2 dx
Aliter Sol. We have, y = sin x + sin x + sin x + ... ¥
Let f = y 1 – x 2 + x 1 – y 2 – 1 Þ y = (sin x ) + y
¶f – xy ¶f xy Þ y 2 = sin x + y
Þ = + 1–y 2 and = 1– x 2 –
¶x 1– x 2 ¶ y 1–y2 Differentiating both the sides, we get
dy dy
æ ¶f ö 2y = cos x +
–ç ÷ dx dx
dy è ¶x ø [ 1– x2 1 – y 2 – xy ] / 1 – x 2
\ = =– dy dy cos x
dx æ ¶f ö [ 1– x2 1 – y 2 – xy ] / 1 – y 2 Þ (2y - 1) = cos x Þ =
ç ÷ dx dx 2y - 1
è ¶y ø
dy 1–y2 Remark
Þ =– dy æ ¶f / ¶x ö
dx 1– x2 In above examples, it is advised to use = –ç ÷
dx è ¶f / ¶y ø
Chap 02 Differentiation 37

Exercise for Session 3


æyö dy x + y
1. If log ( x 2 + y 2 ) = 2 tan-1 ç ÷ , then show that = .
èxø dx x - y
dy -1
2. If x 1 + y + y 1 + x = 0, then prove that = .
dx ( x + 1)2
æ x2 - y2 ö dy y
3. If cos -1 ç 2 ÷ = tan-1 a, then prove that = .
èx + y2ø dx x

dy sin2(a + y )
4. If sin y = x sin (a + y ), then prove that = .
dx sin a
1 1 dy 1
5. If x 2 + y 2 = t - and x 4 + y 4 = t 2 + 2 , then prove that = .
t t dx x 3y

Multiple Choice Questions


dy
6. If sin( xy ) + cos( xy ) = 0, then is
dx
y y
(a) (b) -
x x
x x
(c) - (d)
y y
dy
7. If ax 2 + 2hxy + by 2 = 0, then is
dx
y x
(a) (b)
x y
x
(c) - (d) None of these
y
dy
8. If x 2 × e y + 2xye x + 13 = 0, then is
dx
-2xe y - x - 2y (x + 1) 2xe x - y + 2y (x + 1)
(a) (b)
x (xe y - x + 2) x (xe y - x + 2)
-2xe x - y + 2y (x + 1)
(c) (d) None of these
x (xe y - x + 2)

9. If log( x + y ) = 2xy , then y ¢(0) is


(a) 1 (b) -1 (c) 2 (d) 0
dy
10. If x loge y + y loge x = 5, then is
dx
y æ x log y + y ö x æ x log y + y ö
(a) - ç ÷ (b) - ç ÷
x è x + y log x ø y è x - y log x ø
y æ x log y - y ö
(c) - ç ÷ (d) None of these
x è x + y log x ø
Session 4
Differentiation of Inverse
Trigonometric Functions

Differentiation of Inverse Trigonometric Functions


Inverse trigonometric functions are continuous functions, their domain and range are shown in the table given below.
S. No. Function Domain Range Principal value branch
-1
[–1, 1] é p pù p p
sin x - £ y £ , where y = sin -1 x
êë - 2 , 2 úû
1.
2 2
2. cos-1 x [–1, 1] [ 0, p ] 0 £ y £ p , where y = cos-1 x
3. tan -1 x R æ p pö p p
ç- , ÷ - < y < , where y = tan -1 x
è 2 2ø 2 2

cosec-1 x (- ¥ , - 1] È [1, ¥ ) é p pù p p
- £ y £ , where y = cosec-1 x , y ¹ 0
êë - 2 , 2 úû - {0}
4.
2 2
sec-1 x (- ¥ , - 1] È [1, ¥ ) ì pü p
5. [ 0, p ] - í ý 0 £ y £ p , where y = sec-1 x , y ¹
î 2þ 2
6. cot -1 x R (0, p ) 0 < y < p , where y = cot -1 x

To differentiate inverse trigonometric functions, we need to 3 tan x - tan 3 x


know some basic trigonometric identities and substitution, (viii) tan 3 x =
1 - 3 tan2 x
they can help you finding the differentiation of inverse = tan (60° - A ) × tan A × tan (60° + A )
trigonometric functions. sin 2 n A
(ix) cos A × cos 2 A × cos 2 2 A . . . cos 2 n - 1 A =
Useful Trigonometric Identities 2 n sin A
Remark
(i) sin 2 x = 2 sin x cos x If no branch of an inverse trigonometric function is mentioned,
then it means that the principal value branch of the function
(ii) cos 2 x = 2 cos x - 1, cos 2 x = 1 - 2 sin x
2 2
have to be taken.
2 tan x Substitutions for Inverse Trigonometric Functions
(iii) sin 2 x =
1 + tan x2
S.No. Expression Substitutions
1 - tan x2
1. a -x
2 2 x = asinq or acos q
(iv) cos 2 x =
1 + tan x2
2. a2 + x 2 x = a tanq or acot q
2 tan x p 3. x 2 - a2 x = asec q or acosec q
(v) tan 2 x = , where x ¹ (2n + 1)
1 - tan x2 4 4. a+ x a- x x = acos q or acos2q
or
a- x a+ x
(vi) sin 3 x = 3 sin x - 4 sin x 3
5. a- x x-b x = acos 2 q - bsin2 q
( a - x )( x - b) or or
= 4 sin (60° - A ) × sin A × sin (60° + A ) x-b a- x
(vii) cos 3 x = 4 cos 3 x - 3 cos x 6. ( x - a)( x - b) or
x-a
or
x-b x = asec 2 q + b tan2 q
x-b a- x
= 4 cos (60° - A ) × cos A × cos (60° + A )
7. 2ax - x 2 x = a( 1- cos q)
Chap 02 Differentiation 39

y Example 30 If y = sec -1 ( 1 + x 2 ), when - 1 < x < 1, æ 1 + x 2 – 1ö


y Example 33 If y = tan –1 ç ÷ , then find dy
dy ç x ÷ dx
then find . è ø
dx
when – 1 £ x £ 1.
Sol. Here, y = sec -1( 1 + x 2 ),
æ 1 + x 2 – 1ö
put x = tan q Sol. Here, y = tan –1 ç ÷, let x = tan q
ç x ÷
Þ y = sec -1( sec 2 q ) è ø

= sec -1 (sec q ) = q = tan -1 x æ 1 + tan 2 q – 1 ö


Þ y = tan –1 ç ÷
ç tan q ÷
dy 1 è ø
\ =
dx 1 + x 2 æ sec q – 1 ö
Þ y = tan –1 ç ÷
è tan q ø
1- x dy
y Example 31 If y = tan -1 , then find , when æ 2q ö
1+ x dx æ 1 – cos q ö ç 2sin
2
÷
Þ y = tan ç –1
÷ = tan ç
–1
÷
- 1 < x < 1. è sin q ø q q
ç 2sin cos ÷
è 2 2ø
1- x
Sol. Here, y = tan -1 , put x = cosq æ qö q
1+ x Þ y = tan –1 ç tan ÷ =
è 2ø 2
1 - cos q
Þ y = tan -1 Þ
1
y = tan –1 x Þ
dy
=
1
1 + cos q 2 dx 2 (1 + x 2 )
2sin 2 q / 2
Þ y = tan -1 dy æ a cos x - b sin x ö
2 cos 2 q / 2 y Example 34 Find , if y = tan - 1 ç ÷,
dx è b cos x + a sin x ø
éQ- 1 < cosq < 1 Þ 0 < q < p ù
ê ú
p p a
Þ y = tan -1 tan q / 2 q p where - < x < and tan x > - 1.
ê\0 < < ú 2 2 b
ë 2 2 û
æ a cos x - b sin x ö
\ y = tan -1 (tan q / 2) Sol. Here, y = tan - 1 ç ÷
è b cos x + a sin x ø
[Q tan -1(tan x ) = x, - p / 2 < x < p / 2] æ a ö
q 1 -1
ç - tan x ÷
Þ y= Þ y = cos -1 x Þ y = tan ç b ÷
2 2 ç 1 + a tan x ÷
dy 1 è b ø
\ =-
dx 2 1- x 2 æa ö æa ö
= tan - 1 ç ÷ - tan - 1(tan x ) = tan - 1 ç ÷ - x
èb ø èb ø
1 – cos x dy dy
y Example 32 If y = tan –1 , then find , Þ =0-1= -1
1 + cos x dx dx
p dy
when 0 < x < . y Example 35 Find for the function
2 dx
1 – cos x æ 5x + 12 1 - x 2 ö
Sol. Here, y = tan –1 y = sin - 1 ç ÷.
1 + cos x ç 13 ÷
è ø
2 sin 2 x/ 2
Þ y = tan –1 = tan –1 tan 2 x / 2 æ 5x + 12 1 - x 2 ö
2 cos 2 x/ 2 Sol. Here, y = sin - 1 ç ÷
ç 13 ÷
è ø
Þ y = tan –1 (tan x/ 2)
Put x = sin q, 5 = r cos f and 12 = r sin f
x é -1 p pù
Þ y= êëQ tan (tan x ) = x , - 2 < x < 2 úû Þ r = (r cos f )2 + (r sin f )2
2

Hence,
dy 1
= = (5)2 + (12)2 = 25 + 144 = 169 = 13
dx 2 æ 12 ö
f = tan - 1 ç ÷
è5ø
40 Textbook of Differential Calculus

é r cos f sin q + r sin f cos q ù y Example 37 Sketch the graph for y = sin -1 (sin x ) and
\ y = sin - 1 ê úû
ë 13 dy
Þ y = sin - 1 (sin(q + f )) = q + f
hence find .
dx
æ 12 ö Sol. As, y = sin -1 (sin x ) is periodic with period 2p.
Þ y = sin - 1 x + tan - 1 ç ÷
è5ø
\ To draw this graph we should draw the graph for one interval
é -1 - 1 æ 12 ö ù of length 2p and repeat it for entire values of x.
êQq = sin x and f = tan çè 5 ÷ø ú
ë û As we know,
dy 1 ì p p
\ = - £x£
dx 1 - x2 -1 ïx , 2 2
sin (sin x ) = í
p p æ p 3p ö
dy a-x ï( p - x ), - £ p - x < ç i.e. £ x £ ÷
y Example 36 Find , for y = tan - 1 , î 2 2 è 2 2 ø
dx a+ x
ì p p
where - a < x < a. ï x, - £x£
-1 2 2
or sin (sin x ) = í
a-x p 3p
Sol. Here, y = tan - 1 , where - a < x < a ïp - x , £x£
a+x î 2 2
x which is defined for the interval of length 2p, plotted as;
Put x = a cos q Þ q = cos - 1
a Y
ì a - a cos q ü Repeated curve
π/2
Main curve Repeated curve y = π/2
\ y = tan - 1 í ý
î a + a cos q þ


y=
x
x+

y=

x–
y=

π–

y=
ì 1 - cos q ü X′ π π 3π
X
y=

y = tan - 1 í π

y=
Þ O

x
–π


î 1 + cos q þ
2 2
–x

–x
–π/2 y = –π/2
ì qü
Þ y = tan - 1 í tan 2 ý Y′
î 2þ
Thus, the graph for y = sin -1 (sin x ), is a straight line up and a
q
Þ y = tan -1 tan straight line down with slopes 1 and –1 respectively lying between
2 é p pù
Also, - a < x < a Þ - 1 < cos q < 1 êë - 2 , 2 úû .
q æ pö ì æ 1ö æ 1ö
Þ q Î(0, p) Þ Î ç0, ÷ 1, ç2n - ÷ p < x < ç2n + ÷p
2 è 2ø ï è 2 ø è 2ø
ï
q æ qö q dy ï æ 1ö æ 3ö
\ y = tan - 1 tan = tan - 1 ç tan ÷ = Thus, = í – 1, ç2n + ÷ p < x < ç2n + ÷p
2 è 2ø 2 dx ï è 2ø è 2ø
1 x ïdoes not exist, ì p ü
Þ y= cos - 1 x = í(2n + 1 ) ; n Î I ý
2 a ïî î 2 þ
dy 1 1 d æx ö 1
\ =- ´ ç ÷=- Remark
2 dx è a ø
dx 2 x 2 a - x2
2
1- 2 Students are advised to learn the definition of sin–1 (sin x ) as
a
ì x + 2p, 5p 3p
ï – £x£–
2 2
ï 3p p
Graphical Approach for ï – p – x,
ï

2
£x£–
2
ï p p
Differentiation of Inverse y = sin–1 (sin x ) = í x,
ï
– £x£
2
p
2
3p
ï p – x, £x£
Trigonometric Functions ï
ï
2
3p
2
5p
In this section, we are going to introduce, graphs of ïî x – 2p 2
£x£
2
... and so on
some important inverse trigonometric functions
which are highly useful in differential calculus.
Chap 02 Differentiation 41

y Example 38 Sketch the graph for y = cos -1 (cos x ) and y Example 40 Sketch the graphs for
hence find
dy (i) y = sin (sin -1 x ) (ii) y = cos (cos -1 x )
.
dx (iii) y = tan (tan -1 x ) (iv) y = cosec (cosec -1 x )
-1
Sol. As, y = cos (cos x) is periodic with a period 2p.
(v) y = sec (sec -1 x )
\ To draw this graph, we should draw the graph for one inter- dy
val of length 2p and repeat it for entire values of x of length (vi) y = cot (cot -1 x ) and hence find
2p. dx
ìx , 0£ x £ p Sol. As we know, all the above mentioned six curves are
As we know, cos -1 (cos x ) = í non-periodic, but have restricted domains and
î2 p - x , 0 £ 2 p - x £ p ranges.
ìx , 0£ x £ p So, we shall first define each curve for its domain
or cos -1 (cos x ) = í
î2 p - x , p £ x £ 2 p and range and then sketch the curves.
Thus, it has been defined for 0 < x < 2 p that has length 2p. (i) Sketch for the curve y = sin (sin -1 x )
So, its graph could be plotted as We know that, domain x Î [ -1, 1] , i.e. -1 £ x £ 1
Y
y=π and range y = x Þ y Î [ -1, 1].
(π, π)
2π Hence, we should sketch y = sin (sin -1 x ) only


–2


–x –x
when x Î [ -1, 1] and y = x . So, its graph could be
x
π

x+

–x
x–
x+


x

X′ plotted as shown in the figure.


–2π –π 0 π 2π 3π 4π X
Y′ Y
Thus, the curve y = cos –1(cos x ) and hence y=1
1 –1 x)
n
ìdoes not exist, x = {np, n Î I } (si
dy ï sin
= í 1, 2np < x < (2n + 1)p y=
X′ X
dx ï –1 0 1
î - 1, (2n + 1)p < x < (2n + 2)p
y = –1
y Example 39 Sketch the graph for y = tan -1 (tan x ) and –1

dy x = –1 Y′ x=1
hence find × -1
dx Thus, y = sin (sin x ) = x , - 1 £ x £ 1.
-1
Sol. As, y = tan (tan x ) is periodic with period p. dy
Þ = 1, - 1 < x < 1.
\ To draw this graph we should draw the graph for one dx
interval of length p and repeat it for entire values of x. (ii) Sketch for the curve y = cos (cos -1 x )
ì p pü
As we know, tan -1(tan x ) = í x , - < x < ý We know that, domain x Î [ -1, 1] , i.e. -1 £ x £ 1
î 2 2þ
and range y = x Þ y Î [ -1, 1] .
p p
Thus, it has been defined for - < x < that has length p. So, Hence, we should sketch y = cos (cos -1 x ) = x
2 2
its graph could be plotted as only when x Î [ -1, 1]. So, its graph could be
plotted as shown in the figure.
Y
y = π/2 Y
x
X′ X y=1
–3π/2 –π –π/2 0 π/2 π 3π/2 2π 5π/2 1
y = –π/2 –1 x
)
Y′ 0 s
X′ s (co X
–1 co 1
Thus, the curve for y = tan -1(tan x ), where y is not defined for y=
p
x Î (2n + 1) . y = –1
2 –1

ì æ 1ö æ 1ö x = –1 Y′ x=1
dy ï
1, ç2n - ÷ p < x < ç2n + ÷ p
and hence =í è 2ø è 2ø
dx ï p Thus, y = cos(cos -1 x ) = x , - 1 £ x £ 1
does not exit, x = (2n + 1)
î 2 Þ
dy
= 1, - 1 < x < 1.
dx
42 Textbook of Differential Calculus

(iii) Sketch for the curve y = tan (tan -1 x ) (v) Sketch for the curve y = sec (sec-1 x )
We know that, domain x Î R, i.e. - ¥ < x < ¥ We know that, domain Î R - ( -1, 1)
and range y = x Þ y Î R. (i.e. - ¥ < x £ - 1 or 1 £ x < ¥)
Hence, we should sketch y = tan (tan -1 x ) = x , and range y = x
" x Î R. Þ y Î R - ( -1, 1)
So, its graph could be plotted as shown in the figure. Hence, we should sketch y = sec (sec -1 x ) = x , only
Y
x) when
–1
(ta
n x Î ( - ¥, – 1] È [1, ¥ )
n
ta
1 So, its graph could be plotted as shown in the figure.
y=

x
Y =
X′ –1 x)
–1 (0, 0) 1 X c
se
c(
se
–1 y=
y=1
(1, 1)
Y′
X′ X
Thus, y = tan (tan -1 x ) = x, x Î R (0, 0)

dy
Þ = 1, x Î R (–1, –1)
y = –1
dx
(iv) Sketch for the curve y = cosec (cosec-1 x )
Y′
x = –1 x=1
We know that, domain Î R - ( -1, 1)
(i.e. –¥ < x £ - 1 or 1 £ x < ¥) Thus, y = sec (sec -1x ) = x , | x | ³ 1
and range y = x Þ y Î R - ( -1, 1). dy
Þ = 1, | x | > 1
Hence, we should sketch dx
y = cosec ( cosec -1x ) = x only when (vi) Sketch for the curve y = cot (cot -1 x )
x Î ( - ¥, - 1] È [1, ¥ ). We know that, domain Î R (i.e. – ¥ < x < ¥)
So, its graph could be plotted as shown in the figure. and range y = x
x
Y
–1 x)
= Þ y = R.
c
se Hence, we should sketch
co
c(
se y = cot (cot –1 x ) = x , " x Î R.
co
y= y=1 So, its graph could be plotted as shown in the figure.
1
x
=
X′ X Y –1 x)
–1 0 1 ot
t (c
co
y = –1 1 y=
–1

X′ X
–1 0 1

Y′ –1
x = –1 x=1
Y′
Thus, y = cosec ( cosec -1x ) = x , | x | ³ 1 -1
Thus, y = cos ( cot x ) = x , x Î R
dy
Þ = 1, | x | > 1 dy
dx Þ = 1, x Î R.
dx
Chap 02 Differentiation 43

Exercise for Session 4


æ 1 - cos x ö dy
1. If y = tan-1 ç ÷, then is
è sin x ø dx
ì 1, x ÎR - {np}; n ÎI
ï ìï 1, x ÎR - {np}; n ÎI
(a) í 2 (b) í 2
1
ï- , x = {np}; n ÎI ïîdoes not exist, x = {np}; n ÎI
î 2
ì- 1, x ÎR - {np}; n ÎI
ï
(c) í 2 (d) None of these
îïdoes not exist, x = {np}; n ÎI

æ x - x -1 ö dy
2. If y = cos -1 ç -1
÷ , then is
èx + x ø dx
ì 2 , x>0
ì 2 , x>0 ï 1+ x 2
ïï 1 + x 2 ï
(a) í (b) ídoes not exist, x = 0
2
ï- , x<0 ï -2
ïî 1 + x 2 ï 1+ x 2 , x<0
î
ì 2 , x<0
ï 1+ x 2
ï
(c) ídoes not exist, x = 0 (d) None of these
ï -2
ï 1+ x 2 , x>0
î
ì 1 + sin x + 1 - sin x ü dy
3. If y = tan-1 í ý, then is
î 1 + sin x - 1 - sin x þ dx
ì1 x x ì 1 x x
ï , cos > sin ï- , cos > sin
ï 2 2 2 ï 2 2 2
ï ï x x
(a) í- 1, x
cos < sin
x (b) í 1, cos < sin
ï 2 2 2 ï2 2 2
ï ï
ïîdoes not exist, x = {np};n Î integer ïîdoes not exist, x = {np};n Î integer
ì- 1, cos x ³ sin x
ï
(c) í 2 2 2 (d) None of these
1 x x
ï , cos < sin
î2 2 2
dy
4. If y = cot -1 (cot x ), then is
dx
(a) 1, x ÎR (b) 1, x ÎR - {np}
ì1 , x ÎR - {np}
(c) í (d) None of these
î does not exist, x Î {n p}; n Î integer

5. Sketch the graph for


æ 1- x 2 ö
(i) y = sin-1 æç
2x ö
÷ (ii) y = cos-1 çç 2÷
÷
è 1+ x 2 ø è 1+ x ø
æ 3x - x 3 ö
(iii) y = tan-1 æç
2x ö
÷ (iv) y = tan-1 çç 2 ÷
÷
è 1- x 2 ø è 1- 3x ø
(v) y = sin-1 (3x - 4x 3 ) (vi) y = cos-1 (4x 3 - 3x )
Session 5
Differentiation of a Function in
Parametric Form

Differentiation of a Function dx
= a (1 – cos q )
in Parametric Form and
dq
y = a (1 – cos q )
Until now we have differentiate the variable y w.r.t. x, but
On differentiating both sides, we get
what can we do if the variable x and y are depend on third
dy
variable ‘t’ (say) i.e. x = g(t ), y = f (t ). In such case x and y = a (sin q )
dq
are called parametric functions and t is called parameter.
dy q q
dy 2 sin cos
In this case to find , we first obtain the relationship dy dq a sin q 2 2 = cot q
So, = = =
dx dx dx a (1 – cos q ) 2q 2
2 sin
between x and y by eliminating the parameter t, then we dq 2
differentiate w.r.t. x. But always, it is not possible to -1 -1
eliminate t, then we use the following formula y Example 43 If x = a sin t , y = a cos t
,a>0
dy dy y
dy dt f ¢ (t ) and - 1 < t < 1. Show that =- .
= = dx x
dx dx g ¢ (t ) -1 -1
Sol. Given, x = a sin t
and y = a cos t
dt
1
dy dx 1 sin - 1 t - d sin - 1 t
y Example 41 If x = e –t 2 Þ = (a ) 2 (a )
and y = tan (2t + 1), then find .
–1
dt 2 dt
dx 1
2 dx 1 sin - 1 t - -1 1 x loge a
Sol. Here, x = e –t Þ = (a ) 2 × a sin t
loge a ´ =
On differentiating both sides, we get
dt 2 1-t 2
2 1 - t2
dx 2 dy 1 cos -1 t -1/ 2 d cos -1 t
= e –t × (–2t ) and y = tan –1(2t + 1) Again, = (a ) (a )
dt dt 2 dt
On differentiating both sides, we get
1 -1 -1 æ 1 ö -y log a
dy 1 = (a cos t )-1/ 2 × a cos t loge a ´ ç ÷= e
= ( 2) 2 ç 1 - t2 ÷ 2 1 - t2
dt 1 + (2t + 1)2 è ø
dy
dy 2 dy dt y
dy dt 1 + ( 4 t 2 + 4 t + 1) So, = =-
\ = = dx dx x
dx dx 2t dt
– 2
dt et Aliter
t2
dy –e -1 -1 -1
t + cos - 1 t
Hence, = We have, xy = a sin t
× a cos t
= a sin = ap / 2
dx 2t (2t + 2t + 1)
2
é pù
Þ x 2y 2 = a p/ 2 -1 -1
êëQsin x + cos x = 2 úû
y Example 42 If x = a (q – sin q ) , y = a (1 – cos q ) ,
dy On differentiating, we get
then find ×
dx 2xy 2 + 2x 2y
dy
=0 Þ
dy
=-
y
Sol. Here, x = a (q – sin q ) dx dx x

On differentiating both the sides, we get


Chap 02 Differentiation 45

y Example 44 If x = a æç cos t + log tan 2 t ö÷


1 æ ö
dx ç 1 ÷
è 2 ø Þ =a ç - sin t + ÷
dt ç t t
dy p 2 sin cos ÷
and y = a sin t, then find at t = . è 2 2ø
dx 4 æ
dx 1 ö a cos 2 t dy
æ 1 ö Þ = a ç - sin t + ÷= and = a cos t
Sol. Here, x = a ç cos t + log tan 2 t ÷ and y = a sin t dt è sin t ø sin t dt
è 2 ø
dy dy dx a cos t
On differentiating both equations w.r.t. t, we get So, = / = = tan t
dx dt dt a cos 2 t
æ ö
dx ç 1 2 t 1÷ sin t
= a ç - sin t + sec ´ ÷ æ dy ö p
dt ç t 2 2÷ Þ ç ÷ = tan = 1
tan è dx øt = p
è 2 ø 4
4

Exercise for Session 5


dy æ 3q ö
1. If x = 2 cos q - cos 2 q and y = 2 sin q - sin 2 q, prove that = tan ç ÷ ×
dx è2ø
dy - y log x
2. If x = e cos 2t and y = e sin 2t , then prove that = ×
dx x log y
dy 1 2p
3. If x = cos t and y = sin t , then prove that = , at t = ×
dx 3 3
æ 1ö æ 1ö dy x
4. If x = a çt + ÷ and y = a çt - ÷ , then prove that = ×
è tø è tø dx y
æ 2t ö æ 2t ö dy
5. If x = sin-1 ç ÷ and y = tan-1 ç ÷ , then prove that = 1.
è1+ t ø
2
è1- t ø
2
dx
dy p
6. If x = a sec3 q and y = a tan3 q, then find at q = .
dx 3
dy dx
7. Let y = x 3 - 8x + 7 and x = f (t ). If = 2 and x = 3 at t = 0, then find the value of at t = 0.
dt dt
Session 6
Logarithmic Differentiation,
Differentiation of Infinite Series
Logarithmic Differentiation On differentiating both sides, we get
So far, we have discussed derivatives of the functions of 1 dy æ1ö
= sin x × ç ÷ + log x × (cos x )
y dx èx ø
the form ( f ( x )) n , n f ( x ) and n n , where f ( x ) is a function of
x and n is a constant. In this section, we will mainly be dy é sin x ù
Þ = yê + (cos x ) log x ú
discussing derivatives of the functions of the form dx ë x û
( f ( x )) g ( x ) , where f ( x ) and g( x ) are functions of x or dy é sin x ù
f ( x ) f 2 ( x )K \ = x sin x ê + (cos x ) log x ú
f ( x )s( x )h( x )K or 1 dx ë x û
g 1 ( x ) g 2 ( x )K Aliter
To find the derivatives of these types of functions, we Here, y = x sin x could also be differentiated by using
proceed as follows definition;
d d
Let y = ( f ( x )) g ( x ) (variable)variable = (variable)constant
dx dx
Taking logarithm on both sides, we have d
+ (constant)variable
log y = g( x ) log { f ( x )} dx
dy d d
Differentiating w.r.t. x, we get i.e. = ( x )sin x + ( x )sin x
1 dy 1 d f (x ) d g( x ) dx dx dx
× = g( x ) × × + log { f ( x )} ×
y dx f (x ) dx dx variable constant constant variable

dy é g( x ) d f ( x ) d g( x ) ù = sin x ( x ) sin x – 1
+ (x ) sin x
× (log x ) × cos x
\ =y ê × + log { f ( x )} ×
dx úû
sin x
(x )
dx ë f (x ) dx = (sin x ) + ( x )sin x × log x × cos x
x
dy ì g (x ) d f (x ) d g (x )ü ì sin x ü
or = ( f ( x )) g ( x ) í × + log { f ( x )} × ý = x sin x í + cos x × log x ý
dx î f (x ) dx dx þ î x þ
dy
y Example 46 If x y × y x = 1 , then find .
Logarithmic Differentiation dx
Sol. Taking log on both the sides, y log x + x log y = log 1
(For Quickly Solving Objective MCQs)
Differentiating both the sides, we get
If y = { f ( x )} f ( x ) , then
d ìd ü ìd ü
y× (log x ) + í y ý × log x + í x ý × log y
dy dx î dx þ î dx þ
= Differentiation of y taking f ( x ) as constant
dx ìd ü
+ Differentiation of y taking f( x ) as constant. +xí log y ý = 0
or when we have to differentiate the function of the form î dx þ
y = (Variable) variable , take log on both sides and then Þ y×
1
+ log x ×
dy 1 dy
+ 1 × log y + x × × =0
differentiate. x dx y dx

dy é x ù dy éy ù
y Example 45 If y = x sin x , then find . Þ ê log x + y ú dx = – ê x + log y ú
dx ë û ë û
Sol. y = x sin x , taking log on both sides, we get dy (y + x log y ) y
Þ =– ×
log y = sin x log x dx ( x + y log x ) x
Chap 02 Differentiation 47

dy ì 1 1 1ü
y Example 47 If y cot x + (tan –1 x ) y = 1, then find . = (1 × 2 × 3 ×. . . × n )í1 + + + . . . + ý
dx î 2 3 nþ
Sol. Let u = y cot x and v = (tan –1 x )y , we get ì 1 1 1ü
= (n )! í1 + + + . . . + ý
u +v = 1 ...(i) î 2 3 nþ
Taking log on both sides for u and v, we get 100
y Example 49 If f ( x ) = P ( x - n )n(101 - n ) , then find
log u = cot x log y and log v = y log (tan x ) –1
n =1
On differentiating both sides, we get f (101)
.
1 du 1 dy f ¢ (101)
× = cot x × + log y × (– cosec 2 x )
u dx y dx 100
Sol. f ( x ) = P ( x - n )n (101 - n )
du é cot x dy ù n =1
Þ = y cot x ê × – ( cosec 2 x ) log y ú ...(ii) ì ì 100 üü
dx ë y dx û Þ log f ( x ) = ín (101 - n ) logí P ( x - n )ýý
î în = 1 þþ
Again, differentiating log v = y log (tan –1 x ), we get
{Here, P changes to S when taken log}
1 dv 1 1 dy 100
× =y× × + log (tan –1 x ) ×
v dx tan x 1 + x
–1 2
dx Þ log f ( x ) = å n(101 - n ) log ( x - n )
n =1
dv é y dy ù Differentiating both sides, we get
Þ = (tan –1 x )y ê + log (tan –1 x ) × ú
ë (1 + x ) tan x
2 –1
dx dx û f ¢ ( x ) 100 1
...(iii) = å n (101 - n ) ×
du dv f ( x ) n =1 x -n
From Eq. (i), we get + =0
dx dx f ¢ (101) 100 n (101 - n ) 100

ì cot x dy ü \ =å = å n = 5050
Þ y cot x í × – ( cosec 2 x ) log y ý f (101) n =1 (101 - n ) n =1
î y dx þ f (101) 1
y é y dy ù Þ =
+ (tan x ) ê
–1
+ log (tan –1 x ) × ú = 0 f ¢ (101) 5050
ë ( 1 + x 2
) tan –1
x dx û
ìy cot x
× cot x ü dy
Þ í + (tan –1 x )y × log (tan –1 x )ý
î y þ dx Differentiation of Infinite Series
ì (tan –1 x )y × y ü
= íy cot x × ( cosec 2 x ) log y – ý When the value of y is given as an infinite series, then we
î (1 + x 2 ) tan –1 x þ use the fact that, if one term is deleted from an infinite
series, it remains unaffected, to replace all terms except
ì (tan –1 x )y – 1 × y ü first term by y. Thus, we convert it into a finite series or
(y cot x × cosec 2 x × log y ) – í ý
dy î 1 + x2 þ function.
Hence, = cot x – 1
dx (y × cot x ) + {(tan –1 x )y × log (tan –1 x )}
y Example 50 If y = log x + log x + log x + ¼ ¥
dy
Derivative of the Product of Finite then find
dx
.
Number of Functions Sol. We have, y = log x + log x + log x + ¼ ¥
y Example 48 If f ( x ) = ( x + 1)( x + 2)( x + 3) ...( x + n ),
Then y = log x + y
then find f ¢(0).
Þ y 2 = log x + y
Sol. Here f ( x ) = ( x + 1)( x + 2)( x + 3) . . . ( x + n )
Differentiating both sides, we get
log f ( x ) = log {( x + 1)( x + 2) . . . ( x + n )}
dy 1 dy
Differentiating both sides w.r.t. x, we get 2y × = +
1 1 1 1 1 dx x dx
f ¢( x ) = + + + ... +
f (x ) x +1 x +2 x +3 x +n Þ
dy
(2y - 1) =
1
dx x
ì 1 1 1ü
\ f ¢ (0) = f (0) í1 + + + . . . + ý dy 1
î 2 3 nþ Þ =
dx x (2y - 1)
48 Textbook of Differential Calculus

Exercise for Session 6


n Directions (Q. Nos. 1 to 10) Differentiate the following functions w.r.t. x.
x
1. xx 2. x x
3. xx
4. ( x x )x 5. (x x ) x 6. (cos x )x
-1
7. (sin x )cos x 8. (sin x )cos x
9. cos ( x x )

10. log ( x x + cosec2 x )


dy
11. If y = (sin x )tan x + (cos x )sec x , then find .
dx
-y dy log x
12. If x y = e x , then prove that =
dx (1 + log x )2
dy
13. If x y + y x = 2, then find .
dx
dy
14. If (cos x )y = (sin y )x , then find .
dx
dy sin2 (a + y )
15. If x sin (a + y ) + sin a × cos (a + y ) = 0. Prove that = .
dx sin a
dy sin x
16. If y = cos x + cos x + cos x + . . . ¥ , then prove that = .
dx 1 - 2y
(tan x ). . . ¥ dy p
17. If y = (tan x )(tan x ) , then prove that = 2 at x = ×
dx 4
ex ex xe
18. If y = e x + xe + e x , then prove that
dy ex ìe x ü ex ì1 ü xe
+ e x × log x ý + x e × e e × í + e x × log x ý + e x × x x × x e - 1{1 + e log x }.
x x e
= ex × x e ×í
dx î x þ î x þ

æa x - aö -1 æ a 2 - a1 ö -1 æ a 3 - a 2 ö
æ
-1 a n - a n - 1
ö dy
19. If y = tan-1 ç 1 ÷ + tan ç ÷ + tan ç ÷ + . . . + tan ç ÷ - tan-1 (a n ), then find .
è a1a + x ø è 1 + a1a 2 ø è 1 + a 2a 3 ø è 1 + a na n - 1 ø dx
Session 7
Differentiation of a Function
w.r.t. Another Function
Differentiation of a Function Sol. Let
æ 1 - x2 ö
u = tan - 1 ç ÷
w.r.t. Another Function ç
è x ÷
ø
Suppose it is required to differentiate a function f ( x ) with and v = cos - 1 (2x 1 - x 2 )
respect to another function g( x ). Let y = f ( x ) and
z = g ( x ). Here, both functions are different but both are in æ 1 ö
Putting x = sin q, then x Î ç , 1÷
è 2 ø
same variable x.
1 p p
Then, to find the derivative of f ( x ) with respect to g( x ) or Þ < sin q < 1 Þ <q <
2 4 2
dy
derivative of g( x ) with respect to f ( x ), i.e. to find or æ 1 - x2 ö æ 1 - sin 2 q ö
dz Now, u = tan - 1 ç ÷ Þ u = tan -1 ç ÷
dz ç x ÷ ç sin q ÷
. We firstly differentiate both functions f ( x ) and g( x ) è ø è ø
dy
æ cos q ö
with respect to x separately and then put these values in Þ u = tan - 1 ç -1
÷ Þ u = tan (cot q )
è sin q ø
the following formulae:
ì æp öü
dy dy /dx dz dz /dx Þ u = tan - 1 í tan ç - q ÷ý
= or = î è 2 øþ
dz dz /dx dy dy /dx
é p p p pù
æ 2x ö êëQ 4 < q < 2 Þ 0 < 2 - q < 4 úû
y Example 51 Differentiate sin - 1 ç ÷ w.r.t.
è1+ x ø
2
p p
-1
tan x, - 1 < x < 1. Þ u = - q Þ u = - sin - 1 x
-1æ ö
2x 2 2
Sol. Let u = sin ç 2÷
and v = tan - 1 x
è1 + x ø du 1
Þ =- ...(i)
Putting x = tan q Þ q = tan - 1 x
dx 1 - x2
æ 2 tan q ö Again, v = cos - 1 (2x 1 - x 2 )
Þ u = sin - 1 ç ÷
è 1 + tan 2 q ø p
Þ v= - sin - 1 (2x 1 - x 2 )
é 2 tan q ù 2
Þ u = sin - 1 (sin 2q) êQsin 2q = ú
ë 1 + tan 2 q û p
Þ v = - sin -1(sin 2 q ), where x = sinq
2
Þ u = 2q Þ u = 2 tan - 1 x
du 2 p p
Þ = ...(i) Þ v = - sin -1(sin ( p - 2q) ) Þ v = - ( p - 2q)
dx 1 + x 2 2 2
Again, v = tan - 1 x Þ
dv
=
1 é p p pù
êëQ 4 < q < 2 Þ 0 < p - 2q > 2 úû
...(ii)
dx 1 + x 2
2
p p
du du / dx 1 + x 2 Þ v =- + 2q Þ v = - + 2 sin - 1 x
Now, = = =2 2 2
dv dv / dx 1
dv 2
1 + x2 Þ = ...(ii)
dx 1 - x2
æ 1 ö
y Example 52 If x Î ç , 1÷. Differentiate 1
è 2 ø du -
æ 1- x2 ö \
du dx
= =
1 - x2
=-
1
tan - 1 ç ÷ w.r.t. cos - 1 (2x 1 - x 2 ) . dv dv 2 2
ç x ÷
è ø dx 1- x 2
50 Textbook of Differential Calculus

æ 1 ö dy
= 3x 2 tan x 3 [Q f ¢ ( x ) = tan x ]
y Example 53 Find the derivative of sec -1 ç 2 ÷ dx
è 2x - 1 ø dz
and z = g(x 5 ) Þ = g ¢ ( x 5 ) × 5x 4
1 dx
w.r.t. 1 - x 2 at x = . dz
2 = 5x 4 × sec x 5 [Q g ¢ ( x ) = sec x ]
dx
æ 1 ö
Sol. Let u = sec -1 ç 2 ÷ and v = 1 - x
2
dy 3x 2 tan x 3 3 tan x 3
è 2x - 1 ø Now, = = ´
dz 5x 4 sec x 5 5x 2 sec x 5
Put x = cos q dy
\ u = sec - 1 (sec 2q) = 2q 3 tan x 3 3
\ lim dz = lim 3 5
=
x ®0 x x ® 0 5x sec x 5
and v = 1 - cos 2 q = sin q

Þ
du
=2
y Example 55 Find the derivative of f (tan x ) w.r.t.
dq p
dv
g (sec x ) at x = , where f ¢ (1) = 2 and g ¢ ( 2 ) = 4.
and = cos q 4
dq Sol. Let u = f (tan x ) and v = g (sec x )
du du / d q 2 2
Þ = = = du
dv dv / d q cos q x Þ = f ¢ (tan x ) × sec 2 x
dx
æ du ö 2 dv
Now, ç ÷ = =4 and = g ¢ (sec x ) × sec x × tan x
è dv ø at x =
1 1 dx
2
2
du du /dx f ¢ (tan x ) sec 2 x
Þ = =
y Example 54 If y = f ( x 3 ), z = g ( x 5 ), f ¢ ( x ) = tan x and dv dv /dx g ¢ (sec x ) sec x tan x
dy / dz æ pö
g ¢ ( x ) = sec x , then find the value of lim . f ¢ ç tan ÷
x ®0 x æ du ö è 4ø f ¢ (1) 2 2 2 1
Þ ç ÷ = = = =
è dv ø x p æ pö p g ¢( 2) 4 2
dy = g ¢ çsec ÷ sin
Sol. Here, y = f ( x 3 ) Þ = f ¢ ( x 3 ) × 3x 2 4
è 4ø 4
dx

Exercise for Session 7


2x 2x
1. Find the derivative of tan- 1 w.r.t. sin- 1 , if
1- x 2 1+ x 2
(i) x Î (- 1, 1) (ii) x Î (1, ¥)
(iii) x Î (- ¥, - 1)
-1
2. Differentiate x sin x
w.r.t. sin-1 x.

3. Differentiate sin-1 (2ax 1 - a 2x 2 ) w.r.t. 1 - a 2x 2 .

4. Differentiate log sin x w.r.t. cos x

ìï 1 + x 2 - 1 - x 2 üï
5. Differentiate tan- 1 í -1 2
ý w.r.t. cos x .
ïî 1 + x 2 + 1 - x 2 ïþ

6. Differentiate x x w.r.t. x log x .

7. Differentiate sin- 1 (4x 1 - 4x 2 ) w.r.t. 1 - 4x 2 , if

(i) x Î æç -
1 ö
(ii) x Î æç
1 1 1ö
, ÷ , ÷
è 2 2 2 2ø è2 2 2ø

(iii) x Î æç - , -
1 1 ö
÷
è 2 2 2ø
Session 8
Higher Derivatives of a Function
Higher Derivatives Þ (sec x ) y ¢ = cos(sin x ) …(ii)

of a Function Again differentiating w.r.t. x, we get


(sec x )y ¢¢ + (sec x tan x )y ¢ = - sin(sin x )× cos x
dy Þ y ¢¢ + (tan x )y ¢ = - sin(sin x )× cos 2 x
If y = f ( x ), then the derivative of w.r.t. x is called the
dx
Using Eq. (i), we get y = sin(sin x )
d 2y
second derivative of y w.r.t. x and it is denoted by . Þ y ¢¢ + (tan x )y ¢ + (cos 2 x )y = 0
dx 2
d 2y d æ dy ö d 3 y d æ d 2y ö y Example 58 If (a + bx )e y / x = x , show that
Also, = ç ÷ ; = ç ÷ and so on.
dx 2 dx è dx ø dx 3 dx è dx 2 ø x 3 y ¢¢ = ( xy ¢ - y ) 2 .
x
d ny Sol. Here, (a + bx )e y / x = x Þ e y / x =
The nth order derivative of y w.r.t. x is denoted by . If a + bx
dx n
Taking logarithm on both sides, we have
dy d 2 y d 3 y d ny
y = f ( x ), then other notations for , , , ¼, y æ x ö
dx dx 2 dx 3 dx n = log ç ÷ …(i)
x è a + bx ø
are y 1 , y 2 , y 3 , ¼, y n or y ¢ , y ² , y ¢¢¢, ¼, y or
n
y
f ¢ ( x ), f ² ( x ), f ¢¢¢( x ) ,¼, f n ( x ). Þ = log x - log(a + bx )
x
Differentiating both sides, we get
Remark xy ¢ - y × 1 1 b
If y is a function of x, given parametrically by x = f( t ) y = y ( t ), = -
x 2
x a + bx
then
d2y d æ dy ö d æ y ¢(t ) ö d × æ y ¢(t ) ö dt Þ xy ¢-y =
ax
= ae y / x
= ç ÷= ç ÷= ç ÷´ [from Eq. (i)]
dx 2
dx è dx ø dx è f ¢(t ) ø dt è f ¢(t ) ø dx (a + bx )
Again taking logarithm on both sides, we get
y Example 56 If y = x 3 log e x , then find y ¢¢ and y ¢¢¢. y
log( xy ¢ - y ) = log a + × log e
1 x
Sol. Here, y = x 3 loge x Þ y ¢ = x 3 × + 3x 2 × loge x
x Again differentiating both sides w.r.t. x, we get
y ¢ = x 2 + 3x 2 × loge x xy ¢¢ + y ¢ - y ¢ ( xy ¢ - y ×1)
= 0+
Again differentiating both sides, we get xy ¢ - y x2
1 Þ x 3y ¢¢ = ( xy ¢ - y )2 .
y ¢¢ = 2x + 6x × loge x + 3x 2 ×
x
y Example 59 If x = a (t + sin t ) and y = a (1 – cos t ),
y ¢¢ = 5x + 6x × loge x
Differentiating again, d 2y
then find ×
y ¢¢¢ = 5 + 6 × log x + 6x ×
1 dx 2
x Sol. Here, x = a (t + sin t ) and y = a (1 – cos t )
y ¢¢¢ = 11 + 6log x Differentiating both sides w.r.t. t, we get
dx dy
y Example 57 If y = sin (sin x ), then prove that = a (1 + cos t ) and = a (sin t )
dt dt
y ¢¢+ (tan x ) y ¢+ y cos 2 x = 0. t
2 sin × cos
t
dy a sin t 2 2 æt ö
Sol. Here, y = sin(sin x ) …(i) \ = = = tan ç ÷
dx a (1 + cos t ) 2t è2ø
2 cos
Þ y ¢ = cos(sin x )× cos x 2
52 Textbook of Differential Calculus

Again, differentiating both sides, we get and y = e t × cos t


d 2y
= sec2 æç ö÷ × × = sec2 æç ö÷ ×
t 1 dt 1 t 1 On differentiating w.r.t. t, we get
dx 2 è 2 ø 2 dx 2 è 2 ø a (1 + cos t ) dy
= e t × (–sin t ) + e t × (cos t ) =e t (cos t – sin t )
2 æt ö dt
sec ç ÷ dy
1 è2ø
=
× dy dt e t (cos t – sin t ) y – x
2a æ So, = = =
tö dx dx e t (cos t + sin t ) y + x
2 ç cos 2 ÷
è 2ø dt
d 2y 1 æt ö dy y – x
Hence, = × sec 4 ç ÷ or = ...(i)
dx 2
4a è2ø dx y + x
On differentiating both sides w.r.t. x, we get
y Example 60 Show that the function y = f ( x ) defined
ì dy ü ì dy ü
by the parametric equations x = e t × sin t, y = e t × cos t, (y + x ) × í – 1ý –(y – x )í + 1ý
d 2y î dx þ î dx þ
=
satisfies the relation y ¢¢ ( x + y ) 2 = 2 ( xy ¢ – y ). dx 2 (y + x )2
Sol. Here, x = e t ×sin t (y + x ) × (y ¢ – 1)–(y – x )(y ¢ + 1)
or y ¢¢ =
On differentiating w.r.t. t, we get (y + x )2
dx
= e t × cos t + e t × sin t = e t (sin t + cos t ) Therefore, ( x + y )2 × y ¢ ¢ = 2( xy ¢ – y )
dt

Exercise for Session 8


2
d 2y 1 æ dy ö y
1. If y = x x , then prove that - ç ÷ - = 0.
dx 2 y è dx ø x
d 2y dy
2. If y = A cos (log x ) + B sin (log x ), then prove that x 2 +x + y = 0.
dx 2 dx
æ x ö 2
3d y æ dy ö
2
3. If y = x log ç ÷ , then prove that x = ç x - y ÷ .
è a + bx ø dx 2 è dx ø
d 2y dy
4. If y = log ( x + x 2 + a 2 ), then prove that ( x 2 + a 2 ) 2
+x = 0.
dx dx
5. If y = ( x + x 2 + 1)m, then prove that ( x 2 + 1)y 2 + xy1 - m 2y = 0.
d 2y
6. If x = at 2, y = 2at , then find .
dx 2
d 2y
7. If x = a cos 3 q, y = a sin3 q, then find .
dx 2
æ1 ö d 2y dy
8. If x = tan ç log y ÷ , then show that (1 + x 2 ) = (a - 2x ) .
èa ø dx 2 dx
d 2y dy
9. If x = a cos q + b sin q and y = a sin q - b cos q, then prove that y 2 -x + y = 0.
dx 2 dx
ax + b
10. If y = , then prove that 2y1y 3 = 3 ( y 2 )2.
cx + d
d 2y f1f 2 - f2f1
11. If x = f (t ) and y = f (t ), prove that = , where suffixes denote differentiation w.r.t. ‘t ’.
dx 2 f13
12. If x = sin t , y = sin Kt , then show that (1 - x 2 )y 2 - xy1 + K 2y = 0.
x +b
13. If = a tan-1 (a loge y ),a > 0, then prove that yy ¢¢ - yy ¢ log y = ( y ¢ )2.
2
14. Let f ( x ) be a polynomial function of degree 2 and f ( x ) > 0 for all x Î R. If g( x ) = f ( x ) + f ¢ ( x ) + f ¢ ¢ ( x ), then for
any x show that g( x ) > 0.
Session 9
Differentiation of a Function Given
in the Form of a Determinant

Differentiation of a Function Given 1 2x 3x 2 x x2 x3 x x2 x3


in the Form of a Determinant or f ¢ ( x ) = 1 2x 3x 2 + 0 2 6x + 1 2x 3x 2
0 2 6x 0 2 6x 0 0 6
To find the derivative of a determinant, we differentiate
the row (or columns) keeping other rows (or columns) As we know, if any two rows or columns are equal, then
unchanged, i.e. value of the determinant is zero.
u(x ) v( x ) w ( x ) x x2 x3
If y = p ( x ) q ( x ) r ( x ) , then \ f ¢( x ) = 0 + 0 + 1 2x 3x 2 = 6 ( 2x 2 – x 2 )
l( x ) m( x ) g (x ) 0 0 6
Therefore, f ¢( x ) = 6 x 2
u ¢ (x ) v ¢ (x ) w ¢ (x ) u(x ) v( x ) w (x )
dy
= p (x ) q (x ) r (x ) + p ¢ (x ) q ¢ (x ) r ¢ (x ) x + a2 ab ac
dx
l( x ) m( x ) g( x ) l( x ) m( x ) g( x ) y Example 62 If f ( x ) = ab x +b2 bc ,
u ( x ) v( x ) w ( x ) ac bc x+c 2

+ p (x ) q (x ) r (x ) then find f ¢( x ) .
l¢ ( x ) m ¢ ( x ) g ¢ ( x ) x + a2 ab ac
Sol. We have, f ( x ) = ab x +b 2
bc
The differentiation can also be done column-wise.
ac bc x + c2
x x2 x3 1 0 0 x + a 2 ab ac
y Example 61 If f ( x ) = 1 2x 3x 2 , then find \ f ¢ ( x ) = ab x + b 2 bc + 0 1 0
0 2 6x ac bc x + c2 ac bc x + c2

f ¢( x ). x + a2 ab ac
+ ab x + b bc
2
x x2 x3
0 0 1
Sol. Here, f (x ) = 1 2x 3x 2
0 2 6x = {( x + b 2 ) ( x + c 2 ) – b 2c 2 } + {( x + a 2 ) ( x + c 2 ) – a 2c 2 }
On differentiating, we get + {( x + a 2 ) ( x + b 2 ) – a 2b 2 }
d d 2 d 3 = 3x 2 + 2x ( a 2 + b 2 + c 2 )
(x ) (x ) (x )
dx dx dx
f ¢ (x ) = 1 2x 3x 2 x3 sin x cos x
0 2 6x y Example 63 Let f ( x ) = 6 –1 0 , where p
p p2 p3
x x2 x3 x x2 x3
+
d d d
( 3x 2 ) + 1 3x 2
d3
dx
( 1)
dx
( 2x )
dx
2x is constant. Then, find [ f ( x )] at x = 0.
d d d dx 3 [IIT JEE 1997]
0 2 6x ( 0) ( 2) ( 6x )
dx dx dx
54 Textbook of Differential Calculus

x 3 sin x cos x æ np ö
yn = an cos ç + ax ÷
Sol. Given, f ( x ) = 6 –1 0 è 2 ø
p p2 p3 y y1 y 2
\ The determinant D( x ) = y 3 y 4 y 5
On differentiating, we get
d 3 d d y6 y7 y8
(x ) (sin x ) (cos x )
d dx dx dx æp ö æ 2p ö
[ f ( x )] = (–1) cos ax a cos ç + ax ÷ a 2 cos ç + ax ÷
dx 6 0 è2 ø è 2 ø
p p2 p3 æ 3p ö p p
D( x ) = a 3 cos ç + ax ÷ æ 4 ö æ 5 ö
a 4 cos ç + ax ÷ a 5 cos ç + ax ÷
è 2 ø è 2 ø è 2 ø
x 3 sin x cos x x 3 sin x cos x æ 6p ö æ7p ö æ 8p ö
a 6 cos ç + ax ÷ a 7 cos ç + ax ÷ a 8 cos ç + ax ÷
+ 0 0 0 + 6 –1 0 è 2 ø è 2 ø è 2 ø
p p2 p3 0 0 0
cos (ax ) –a sin (ax ) –a 2 cos (ax )
3x 2 cos x –sin x D( x ) = a 3 ´ a 6 sin (ax ) a cos (ax ) –a 2 sin (ax )
d
So, [ f ( x )] = 6 –1 0 –cos (ax ) a sin(ax ) a 2 cos (ax )
dx
p p2 p3
D( x ) = a 9 ´ (0) { R1 ® – R 3 }
Again differentiating, we get Hence, D( x ) = 0
6x –sin x –cos x
d2 (1 + x )a (1 + 2x ) b 1
[ f ( x )] = 6 –1 0 + Remaining
dx 2 y Example 65 If f ( x ) =
p p 2
p 3
1 (1 + x )a (1 + 2x ) b ,
two determinants as zero. (1 + 2x ) b 1 (1 + x )a
Again differentiating, we get
then find
6 –cos x sin x
d3 (i) constant term (ii) coefficient of x.
[ f ( x )] = 6 –1 0 + Remaining two
dx 3 2 (1 + x ) a
(1 + 2x )b 1
p p p3
determinants as zero Sol. Here, f (x ) = 1 (1 + x )a (1 + 2x )b
6 –1 0 (1 + 2x )b 1 (1 + x )a
3
d
At x = 0, [ f ( x )] = 6 –1 0 =0 = A + Bx + Cx 2 + K ...(i)
dx 3
p p2 p3 Putting x = 0, we get
1 1 1
æ d3 ö
\ ç 3 [ f ( x )]÷ =0 (i.e. independent of p) f (0) = 1 1 1 = A + B(0) + C (0)2 + K
è dx ø at x =0 1 1 1
y Example 64 If y = cos ax, prove that Þ A =0
y y1 y2 Again, differentiating Eq. (i) w.r.t. x, we get
dr a(1 + x )a – 1 2b(1 + 2x )b – 1
y3 y4 y 5 = 0, where y r = r y . 0
dx f ¢( x ) = 1 (1 + x )a
(1 + 2x )b
y6 y7 y8
(1 + 2x )b 1 (1 + x )a
Sol. Given, y = cos (ax )
æp ö (1 + x )a (1 + 2x )b 1
Then, y1 = – a sin (ax ) = a cos ç + ax ÷
è2 ø + 0 a (1 + x )a –1
2b (1 + 2x )b – 1
æ 2p ö ( 1 + 2x )b
1 (1 + x )a
y 2 = – a 2 cos (ax ) = a 2 cos ç + ax ÷
è 2 ø
(1 + x )a (1 + 2x )b 1
æ 3p ö
y 3 = + a sin (ax ) = a cos ç
3
+ ax ÷ 3
+ 1 (1 + x ) a
(1 + 2x )b = B + 2Cx + K
è 2 ø
2b (1 + 2x )b –1 0 a (1 + x )a – 1
..............................................................
..............................................................
Chap 02 Differentiation 55

a 2b 0 1 1 1 1 1 1 Differentiating both the sides w.r.t. x, we get


f ¢ ( 0) = 1 1 1 + 0 a 2b + 1 1 1 = B a1b1 (1 + x )a1b1 – 1 (1 + x )a1b2 (1 + x )a1b3
1 1 1 1 1 1 2b 0 a a 2b1(1 + x ) a 2b1 – 1
(1 + x ) a 2b 2
(1 + x )a2b3
Þ B=0 ...(ii) a 3b1(1 + x ) a 3b1 – 1
(1 + x )a3b2 (1 + x )a3b3
\Coefficient of constant term = coefficient of x = 0. (1 + x )a1b1 a1b 2 (1 + x )a1b2 – 1 (1 + x )a1b3
+ (1 + x )a2b1 a 2b 2 (1 + x )a2b2 – 1 (1 + x )a2b3
y Example 66 If ai , b i Î N for i = 1, 2 , 3, then coefficient
(1 + x )a3b1 a 3b 2 (1 + x )a3b2 – 1 (1 + x )a3b3
of x in the determinant;
(1 + x )a1b1 (1 + x )a1b2 a1b 3 (1 + x )a1b3 – 1
(1 + x )a 1b1 (1 + x )a 1b2 (1 + x )a 1b3
+ (1 + x )a2b1 (1 + x )a2b2 a 2b 3 (1 + x )a2b3 – 1
(1 + x )a 2 b1 (1 + x )a 2 b2 (1 + x )a 2 b3 (1 + x )a3b1 (1 + x )a 3b 2
a 3b 3 (1 + x )a3b3 – 1
(1 + x )a 3 b1 (1 + x )a 3 b2 (1 + x )a 3 b3
= B + 2Cx + K
Putting x = 0,
Sol. Here,
a1b1 1 1 1 a1b 2 1 1 1 a1b 3
(1 + x )a1b1 (1 + x )a1b2 (1 + x )a1b3
B = a 2b1 1 1 + 1 a 2b 2 1 + 1 1 a 2b 3
(1 + x )a2b1 (1 + x )a2b2 (1 + x )a2b3 = A + Bx + Cx 2 + K
a 3b1 1 1 1 a 3b 2 1 1 1 a 3b 3
(1 + x )a3b1 (1 + x )a3b2 (1 + x )a3b3
Þ B=0

Exercise for Session 9


f (x ) g( x ) h( x )
1. If f ( x ), g( x ) and h( x ) are three polynomials of degree 2, then prove that f( x ) = f ¢ ( x ) g ¢ ( x ) h ¢ ( x )
f ¢ ¢(x ) g ¢ ¢(x ) h ¢ ¢(x )
is a constant polynomial.
f g h
2. If f , g and h are differentiable functions of x and D = ( x f )¢ ( x g )¢ ( x h )¢ .
( x 2f )¢ ¢ ( x 2g )¢ ¢ ( x 2h )¢ ¢
f g h
Prove that D¢ = f¢ g¢ h¢ .
( x f ¢ ¢ )¢ ( x g ¢ ¢ )¢ ( x h ¢ ¢ )¢
3 3 3

px
dy sin 3
3. Find at x = - 1, when (sin y ) 2 + sec-1 (2x ) + 2x tan {loge ( x + 2)} = 0.
dx 2
56 Textbook of Differential Calculus

Session 10
Derivative of an Inverse Function
Derivative of an
Inverse Function when f ( x ) = 2, then x = 1
f
Theorem If the inverse functions f and g are defined by
y = f ( x ) and x = g(y ) and if f ¢ ( x ) exists and f ¢ ( x ) ¹ 0, x y
1 f'
then g ¢ (y ) = . This result can also be written as, if
f ¢( x ) g
dy dy g ¢ ( 2) × f ¢ ( 1) = 1 [but f ¢ (1) = 8]
exists and ¹ 0, then
dx dx \ g ¢ ( 2) = 1 / 8
dx 1 dy dx dy 1 é dx ù y Example 68 Let f ( x ) = exp ( x 3 + x 2 + x ) for any real
= or . = 1 or = ê Q ¹ 0ú
dy dy dx dy dx dx ë dy û number x and let g be the inverse function for f. The
dx dy value of g ¢(e 3 ) is
y Example 67 If y = f ( x ) = x 3 + x 5 and g is the 1 1 1
(a) 3
(b) (c) (d) 6
6e 6 34e39
inverse of f, then find g ¢(2) (means dx /dy when 3
+ x2 + x
y = 2 ). Sol. Let y = ex ,
Sol. Here, y = x + x dy
= e x + x + x × ( 3x 2 + 2x + 1)
3 5 3 2
On differentiating,
dx
On differentiation, we get dx 1
dy dx 1 Þ g ¢ (y ) = = 3 2
= 3x 2 + 5x 4 Þ g ¢ ( y ) = = 2 dy e x + x + x × [3x 2 + 2x + 1]
dx dy 3x + 5x 4 y = e3
e3 = ex + x + x
3 2
when y = 2, then 2 = x + x 3 5
Þ x =1 Þ
dx ù 1 1 Þ x3 + x2 + x - 3 = 0
\ g ¢ ( 2) = = = Þ ( x - 1)( x 2 + 2x + 3) = 0
dy úû x =1 3 + 5 8
y =2 Þ x =1 [Q x 2 + 2x + 3 > 0]
y = f ( x ), x = g (y ) 1
Aliter \ g ¢ (e 3 ) = 3
6e
( gof )( x ) = x
Hence, (a) is the correct option.
g ¢ [ f ( x )] f ¢ ( x ) = 1

Exercise for Session 10


1. The function f ( x ) = e x + x , being differentiable and one to one, has a differentiable increase f - 1( x ). Find
d
(f - 1( x )) at point f (log 2).
dx
2. Let g( x ) be the inverse of an invertible function f ( x ) which is differentiable for all real x, then find g ¢¢(f ( x )).
3. Let g( x ) be the inverse of an invertible function f ( x ) which is differentiable at x = c, then g ¢(f (c )) equals
1
(a) f ¢ (c ) (b) (c) f (c ) (d) None of these
f ¢ (c )
4. If f ( x ) = x + tan x and f is inverse of g, then g ¢( x ) equals
1 1 1
(a) (b) (c) (d) None of these
1 + [g (x ) - x ]2 2 - [g (x ) - x ]2 2 + [g (x ) - x ]2
JEE Type Solved Examples :
Single Option Correct Type Questions

l Ex. 1 Let f be a twice differentiable function such that Now, two cases arise either f (1 ) = 0 or 1
f ¢ ¢ ( x ) = – f ( x ) and f ¢ ( x ) = g ( x ). If Case I If f (1 ) = 0
h( x ) = { f ( x )} 2 + { g ( x )} 2 , where h(5 ) = 11. Find h(10 ). Þ f (x + 1) = f (x ) [using Eq. (iii)]
Þ f (1 ) = f (2 ) = f (3 ) = .... = 0
(a) 0 (b) 9
\ f ( x ) is identically zero. (which is not possible)
(c) 11 (d) None of these
Case II If f (1 ) = 1
Sol. (c) Given, h( x ) = { f ( x )} 2 + { g( x )} 2
Þ f (x + 1) = f (x ) + 1 [using Eq. (iii)]
On differentiating both the sides w.r.t. x, we get
\ f (2 ) = f (1 ) + 1 = 1 + 1 = 2
h ¢ ( x ) = 2 f ( x ) × f ¢( x ) + 2 g ( x ) × g ¢( x ) ...(i) f (3 ) = f (2 ) + 1 = 2 + 1 = 3
Now, f ¢( x ) = g ( x ). f ( 4 ) = f (3 ) + 1 = 3 + 1 = 4
Then, f ¢¢( x ) = g ¢( x ) f (5 ) = f ( 4 ) + 1 = 4 + 1 = 5
Þ – f ( x ) = g ¢( x ) ...(ii)
. . Proceeding in same way, we get
[ . f ¢¢( x ) = – f ( x ), given]
f ( x ) = x and f ¢( x ) = 1 Þ f ¢(10 ) = 1
From Eqs. (i) and (ii), we get
h ¢( x ) = 2 f ( x ) × g ( x ) + 2 g( x ) × {– f ( x )} So, f (5 ) = 5 and f ¢(10 ) = 1
[using f ¢( x ) = g( x ) and g ¢( x ) = – f ( x )] æ x + y ö f ( x ) + f (y )
\ h ¢( x ) = 0
l Ex. 3 Let f ç ÷= and f ¢ (0 ) = a and
è 2 ø 2
é d ù
So, h( x ) must be constant. êëQ dx (constant) = 0úû f (0 ) = b. Find f ¢ ¢ ( x ) (where y is independent of x), when
But h(5 ) = 11 f ( x ) is differentiable.
h( x ) = 11 (a) 0 (b) 1
So, h(10 ) = 11 (c) a (d) None of these
æ x + y ö f ( x ) + f (y )
l Ex. 2 Let f ( x ) be a real valued differentiable function Sol. (a) Here f ç ÷= , this holds for any real x, y
è 2 ø 2
not identically zero such that dy
and y is independent of x. i.e. =0
f ( x + y 2n + 1 ) = f ( x ) + { f (y )} 2n + 1 , n Î N and x , y are any dx
real numbers and f ¢ (0 ) ³ 0. Find the values of f (5 ) and On differentiating w.r.t. x, we get
f ¢ (10 ). æx + yö 1æ dy ö 1 ì dy ü
f ¢ç ÷ × ç1 + ÷ = í f ¢( x ) + f ¢(y ) × ý
(a) 3, 2 (b) 0, 1 è 2 ø 2è dx ø 2 î dx þ
(c) 1, 5 (d) 5, 1 1 æx + yö 1 é dy ù
\ f ¢ç ÷ = f ¢( x ) as = 0ú
Sol. (d) Here,f ( x + y 2n + 1 ) = f ( x ) + { f (y )} 2n + 1 ...(i) 2 è 2 ø 2 ëê dx û
Putting x = 0, y = 0, we get æx + yö
Þ f ¢ç ÷ = f ¢( x ) ...(i)
f ( 0 ) = f ( 0 ) + { f ( 0 )} 2n + 1 Þ f ( 0 ) = 0 è 2 ø
f ¢( 0 ) ³ 0 [given] Taking x = 0 and y = x in Eq. (i), we get
f (x ) – f (0) f (x ) æ0 + xö æxö
Þ lim ³ 0 Þ lim ³0 f ¢ç ÷ = f ¢( 0 ) Þ f ¢ ç ÷ = a [Q f ¢( 0 ) = a, given]
x®0 x–0 x®0 x è 2 ø è2ø
Now, if x >0 Þ f (x ) ³ 0 ...(ii) æxö æxö
which shows f ç ÷ = a ç ÷ +c [using integration]
è2ø è2ø
Putting x = 0, y = 1 in Eq. (i), we get \ f ( x ) = ax + c
f (1 ) = f ( 0 ) + { f (1 )} 2n + 1 or f (1 ) [1 – { f (1 )} 2n ] = 0 Let x=0
\ f (1 ) = 0 or 1 [using Eq. (ii)] Þ f (0) = b = c
Putting y = 1 in Eq. (i), for all real x. \ f ( x ) = ax + b
f ( x + 1 ) = f ( x ) + { f (1 )} 2n + 1 ...(iii) On differentiating,
f ¢( x ) = a and f ¢¢( x ) = 0
58 Textbook of Differential Calculus

(a - x ) a - x - (b - x ) x - b dy l Ex. 6 Let f ( x ) = x + sin x , suppose g denotes the inverse


l Ex. 4 If y = , then find
a - x + x -b dx æp 1 ö
function of f . Then, find the value of g ¢ ç + ÷.
whenever defined. è4 2ø
x - (a + b ) 2x + (a + b ) (a) 2 + 2 (b) 2 - 2
(a) (b)
(a - x )( x - b ) a - x (x - b ) (c) 2 - 2 (d) 2 2
2 x - (a + b ) Sol. (c) Here, f ( x ) = x + sin x
(c) (d) None of these
2 (a - x )( x - b ) dy dx 1
\ = 1 + cos x Þ g ¢(y ) = =
(a - x ) 3/2 + ( x - b ) 3/2 dx dy 1 + cos x
Sol. (c) Here, y = , use
(a - x )1/2 + ( x - b )1/2 p 1 p
where, y = + = x + sin x Þ x =
a 3 + b 3 = (a + b )(a 2 - ab + b 2 ) 4 2 4
é {(a - x )1/2 + ( x - b )1/2 }{(a - x ) ù æp 1 ö 1 2
ê ú \ g¢ ç + ÷= = =2- 2
è4 2 ø 1 2+1
ê - (a - x )( x - b ) + ( x - b )}úû 1+
\ y =ë 2
{(a - x )1/2 + ( x - b )1/2 }
Þ y = (a - b ) - (ax + bx - ab - x 2 )
l Ex. 7 Let e f ( x ) = log x . If g ( x ) is the inverse of f ( x ),
then find g ¢ ( x ).
dy (a + b - 2 x )
\ =- x x
+x
dx 2 (a - x )( x - b ) (a) e e (b) e x (c) e e (d) None of these
-1
dy 2 x - (a + b ) Sol. (c) Let f ( x ) = y Þ x = f (y ) = g(y )
Þ =
dx 2 (a - x )( x - b ) y dx y y
+ y
Þ x = ee Þ = ee × e y = ee = g ¢(y )
dy
y ¢¢
l Ex. 5 If x 2 + y 2 = R 2 (R > 0 ), then K = . \ g ¢( x ) = e e
x
+x
(1 + (y ¢ ) 2 ) 3
Find K in terms of R. l Ex. 8 If f ( x ) = 4 x 3 - 6 x 2 cos 2a + 3 x × sin 2a × sin 6a
1 1
(a) (b) - + log ( 2a - a 2 ) , then
R R
æ 1ö æ 1ö
(a) f ¢ ç ÷ < 0 (b) f ¢ ç ÷ £ 0
1
(c) R (d)
2R è 2ø è 2ø
Sol. (b) Here, x 2 + y 2 = R 2 æ 1ö
(c) f ¢ ç ÷ > 0 (d) None of these
On differentiating w.r.t. x, we get è 2ø
2 x + 2yy ¢ = 0 Þ x + yy ¢ = 0 Sol. (c) Here, f ( x ) = 4 x 3 - 6 x 2 cos 2a + 3 x sin 2a × sin 6a
x
Þ y¢=- …(i) + log (2a - a 2 ) for f ( x ) to exists.
y
Again, differentiating both the sides, log (2a - a 2 ) ³ 0 Þ (2a - a 2 ) ³ e 0
1 + yy ¢¢ + (y ¢ ) 2 = 0 i.e. 2a - a 2 ³ 1 or a 2 - 2a + 1 £ 0 Þ(a - 1 ) 2 £ 0
(1 + (y ¢ ) 2 ) which is only possible, if (a - 1 ) 2 = 0, i.e. a = 1.
\ y ¢¢ = - …(ii)
y
y ¢¢ (1 + (y ¢ ) 2 )
Now, K = =-
(1 + (y ¢ ) 2 ) 3 y × (1 + (y ¢ ) 2 ) 3/2 \ f ( x ) = 4 x 3 - 6 x 2 cos 2 + 3 x sin 2 × sin 6

=-
1
=-
1
[from Eq. (i)] Þ f ¢( x ) = 12 x 2 - 12 x cos 2 + 3 sin 2 sin 6
y 1 + (y ¢ ) 2
x 2
æ1ö
y 1+ Þ f ¢ ç ÷ = 3 - 6 cos 2 + 3 sin 2 sin 6
y2 è2ø
1 1 æ1ö
=- =- = 3(1 + sin 2 sin 6 ) - 6 cos 2 Þ f ¢ ç ÷ > 0
x +y
2 2 R è2ø

1 Note
\ K =-
R cos 2 < 0 and 1 + sin 2 sin 6 > 0
Chap 02 Differentiation 59

l Ex. 9 Suppose, f ( x ) = e ax + e bx , where a ¹ b and Þ (a 2 - 2a - 15 ) eax + (b 2 - 2b - 15 )ebx = 0, for all x.


f ¢¢( x ) - 2 f ¢ ( x ) - 15 f ( x ) = 0 for all x Î R. Then, find ab. Þ a 2 - 2a - 15 = 0 and b 2 - 2b - 15 = 0
(a) 15 (b) –15 (c) 10 (d) 16 Þ (a - 5 )(a + 3 ) = 0
Sol. (b) f ( x ) = eax + ebx and (b - 5 )(b + 3 ) = 0
Þ f ¢( x ) = aeax + bebx , f ¢¢( x ) = a 2eax + b 2ebx Þ a = 5 or – 3
and b = 5 or – 3
\ f ¢¢( x ) - 2 f ¢( x ) - 15 f ( x ) = 0
But a ¹ b, hence, a = 5, b = - 3
Þ 2 ax
{a e + b 2ebx } - 2 {aeax + bebx }
or a = - 3, b = 5 Þ ab = - 15
- 15 {eax + ebx } = 0, for all x.

JEE Type Solved Examples :


More than One Correct Option Type Questions
l Ex. 10 The functions u = e x × sin x and v = e x × cos x On differentiating w.r.t. x, we get
satisfy the equation xe yy ¢ + e y + e yy ¢ = 0
du dv du dv xy ¢ + y ¢ + 1 = 0 [Qe y ¹ 0] …(ii)
(a) v -u = u 2 + v 2 (b) v +u = u2 + v2
dx dx dx dx 1
Þ y¢ = -
du dv du dv x+1
(c) + = 2v (d) + = 2u
dx dx dx dx Þ y ¢ = - ey [from Eq. (i)]
Sol. (a, c ) We have, u = e sin x, v = e cos x
x x
Þ y¢ + e = 0
y

Differentiating both the equations w.r.t. x, we get


From Eq. (ii), we have
du
= e x (sin x + cos x ) …(i) xy ¢ + 1 = - y ¢ Þ xy ¢ + 1 = e y
dx
dv dy
and = e x (cos x - sin x ) …(ii) l Ex. 12 If x p × y q = ( x + y ) p + q , then is
dx dx
du dv
Now, + = 2e x cos x [using Eqs. (i) and (ii)] (a) independent of p
dx dx
(b) independent of q
du dv
Þ + = 2v (c) dependent both p and q
dx dx
y
du dv (d)
Also, = u + v and =v -u x
dx dx
du dv Sol. (a, b, d ) We have, x p × y q = ( x + y ) p + q
\ v = uv + v 2 and u = uv - u 2
dx dx On taking log both sides, we get
Þ v
du
-u
dv
= u2 + v 2 p log x + q log y = ( p + q ) log ( x + y )
dx dx On differentiating w.r.t. x, we get
p q dy p + q æ dy ö
l Ex. 11 If y + log (1 + x ) = 0, then which of the following + = ç1 + ÷
x y dx x + y è dx ø
is true?
dy æ q p + q ö p + q p
(a) e y = xy ¢ + 1 (b) y ¢ = -
1 Þ ç - ÷= -
( x + 1) dx è y x + y ø x + y x
(c) y ¢ + e y = 0 (d) y ¢ = e y Þ
dy y
=
dx x
Sol. (a, b, c) We have, y + loge (1 + x ) = 0
Þ y = - log (1 + x ) Þ y = log(1 + x ) -1 l Ex. 13 Two functions f and g have first and second
1
Þ e =
y
…(i) derivatives at x = 0 and satisfy the relations f (0 ) =
2
,
1+x g (0 )
Þ xe y + e y = 1 f ¢ (0 ) = 2 g ¢ (0 ) = 4 g (0 ), g ¢¢(0 ) = 5 f ¢¢(0 ) = 6 f (0 ) = 3 Then,
60 Textbook of Differential Calculus

f (x ) 15 1
(a) If h ( x ) = , then h ¢ (0) = k¢( 0 ) = ´ 4 ´1 + 0 + 0 =2
g(x ) 4 2
g ¢( x ) g ¢( 0 ) 1
(b) If k ( x ) = f ( x ) × g ( x ) × sin x , then k ¢ (0) = 2 (c) lim = =
x ® 0 f ¢( x ) f ¢( 0 ) 2
g ¢( x ) 1
(c) lim =
x ® 0 f ¢( x ) 2
l Ex. 14 If f ( x ) = | x 2 - 3 | x | + 2 |, then which of the
(d) None of the above
2 following is/are true?
Sol. (a, b, c) We have, f ( 0 ) = ,
g( 0 ) (a) f ¢ ( x ) = 2x - 3 for x Î (0, 1) È ( 2, ¥ )
(b) f ¢ ( x ) = 2x + 3 for x Î ( - ¥, - 2) È ( - 1, 0)
f ¢( 0 ) = 2 g ¢( 0 ) = 4 g ( 0 )
(c) f ¢ ( x ) = - 2x - 3 for x Î ( - 2, - 1)
g ¢¢( 0 ) = 5 f ¢¢( 0 ) = 6 f ( 0 ) = 3
(d) None of the above
Now, on solving these equations, we get
1 Sol. (a, b, c ) We have,
f ( 0 ) = , g( 0 ) = 4, f ¢( 0 ) = 16, g ¢( 0 ) = 8
2 ì | x 2 - 3 x + 2|, x ³ 0
f ( x ) = | x 2 - 3| x| + 2| = í 2
3
f ¢¢( 0 ) = , g ¢¢( 0 ) = 3 î| x + 3 x + 2|, x < 0
5
ì x 2 - 3 x + 2, x 2 - 3 x + 2 ³ 0, x ³ 0
f (x ) ï
(a) We have, h( x ) = , ï - x + 3 x - 2,
2
x 2 - 3 x + 2 < 0, x ³ 0
g( x ) Þ f (x ) = í 2
g ( x ) f ¢( x ) - g ¢( x ) f ¢( x ) ï x + 3 x + 2, x 2 + 3 x + 2 ³ 0, x < 0
h ¢( x ) = ïî - x 2 - 3 x - 2, x 2 + 3 x + 2 < 0, x < 0
[ g( x )]2
g ( 0 ) f ¢( 0 ) - g ¢( 0 ) f ¢( 0 ) ì x 2 - 3 x + 2, x Î [ 0, 1 ] È (2, ¥ )
\ h ¢( 0 ) = ï
ï - x + 3 x + 2, x Î (1, 2 )
2
[ g( 0 )]2 Þ f (x ) = í 2
1 ï x + 3 x + 2, x Î ( - ¥ , - 2 ] È [ - 1, 0 )
4 ´ 16 - 8 ´ ïî - x 2 - 3 x + 2,
2 = 15 x Î ( - 2, - 1 )
=
(4)2 4 ì 2 x - 3, x Î ( 0, 1 ) È (2, ¥ )
(b) k( x ) = f ( x ) × g( x ) × sin x ï - 2 x + 3, x Î (1, 2 )
ï
Þ f ¢( x ) = í
k ¢( x ) = f ( x ) g( x ) cos x + f ( x ) sin x × g ¢( x ) + g( x )sin xf ¢( x ) 2 x + 3, x Î ( - ¥ , - 2) È ( - 1, 0 )
ï
k ¢( 0 ) = f ( 0 ) g( 0 ) cos 0 + f ( 0 )sin 0 g ¢( 0 ) + g( 0 )sin 0 f ¢( 0 ) ïî - 2 x - 3, x Î ( - 2, - 1 )

JEE Type Solved Examples :


Statements I and II Type Questions
n
Directions (Ex. Nos. 15-17) This section is based on x x
Sol. (b) We have, f ( x ) = Þ f (x ) =
Statement I and Statement II. Select the correct answer x2 - 1 ( x + 1 )( x - 1 )
from the codes given below.
1é 1 1 ù
(a) Both Statement I and Statement II are correct and Þ f (x ) = ê + ú
Statement II is the correct explanation of Statement I 2 ë x + 1 x - 1û
(b) Both Statement I and Statement II are correct but 1é 1 1 ù
Þ f ¢( x ) = ê - - ú
Statement II is not the correct explanation of Statement I 2 ë (x + 1)2 (x - 1)2 û
(c) Statement I is correct but Statement II is incorrect
( - 1 )( - 2 ) é 1 1 ù
(d) Statement II is correct but Statement I is incorrect Þ f ¢¢( x ) = ê + ú
ë (x + 1) (x - 1)3 û
3
2
x
l Ex. 15 Consider f ( x ) = and g ( x ) = f ¢¢( x ) M M
x -1 2
( - 1 )n n ! é 1 1 ù
Statement I Graph of g ( x ) is concave up for x >1 . Þ f n (x ) = ê n+1
+ n +1ú
2 ë (x + 1) (x - 1) û
Statement II
dn ( - 1) n n ! ì 1 1 ü Now, g( x ) = f ¢¢( x )
f (x ) = í + ý n ÎN
n 2 î( x + 1 ) n +
(x - 1 ) n + g ¢( x ) = f ¢¢¢( x )
þ
1 1
dx
Þ g ¢¢( x ) = f ¢¢¢¢( x )
Chap 02 Differentiation 61

(- 1)4 4! é 1 1 ù Þ f ( x ) = f ( x + 4 ) = f ( x + 8 ) … f ( x + 4000 )
Þ g ¢¢( x ) = ê + ú
2 ë (x + 1)
5
(x - 1)5 û Þ f ¢( x ) = f ¢( x + 4000 )
Hence, both the statements are true and Statement II is the
4! é 1 1 ù
Þ g ¢¢( x ) = ê + ú correct explanation of Statement I.
2 ë (x + 1)5 (x - 1)5 û
\ g ¢¢(1 ) > 0 l Ex. 17 Statement I Let f ( x ) = x[ x ] and [× ] denotes
\Graph of g( x ) is concave up for x > 1. greatest integer function, when x is not an integral, then rule
Hence, both statements are correct but Statement II is not the for f ¢ ( x ) is given by [ x ].
correct explanation of Statement I.
Statement II f ¢ ( x ) does not exist for any x Î Z .
l Ex. 16 Statement I If differentiable function f ( x )
ì - x, - 1 £ x < 0
satisfies the relation f ( x ) + f ( x - 2 ) = 0, " x Î R, and if ï 0, 0 £ x <1
æd ö æd ö ï
= 6, then ç =6 Sol. (a) f ( x ) = x[ x ] = í x, 1 £ x <2
ç f (x )÷ f (x )÷
è dx ø x =a è dx ø a + 4000 ï
2 x , 2 £ x <3
ï
Statement II f ( x ) is a periodic function with period 4. î M M
Sol. (a) We have, f (x ) + f (x - 2) = 0 …(i) ì - 1, - 1 < x < 0
ï 0, 0 < x <1
Replace x by x - 2 in Eq. (i), we have ï
Þ f ¢( x ) = í 1, 1 < x <2
f (x - 2) + f (x - 4) = 0 …(ii)
ï
From Eqs. (i) and (ii), ï 2, 2 < x <3
î M
f (x ) - f (x - 4) = 0
M
…(iii)
Replace x by x + 4 in Eq. (iii), we get Þ f ¢( x ) = [ x ]
f (x + 4) - f (x ) = 0 Hence, both statements are true and Statement II is the correct
explanation of Statement I.
Þ f (x + 4) = f (x )

JEE Type Solved Examples :


Passage Based Questions

Passage I
(Ex. Nos. 18 to 20) Since, f ¢¢( x ) × f ¢( x ) = f ( x )
A non-zero polynomial with real coefficients has the property Þ (n - 1 ) + (n - 2 ) = n
that f ¢¢ ( x ) × f ¢ ( x ) = f ( x ). Then, Þ 2n - 3 = n or n = 3
\ Degree of f ( x ) = 3
l Ex. 18 The leading coefficient of f ( x ), is Hence, f ( x ) = ax 3 + bx 2 + cx + d
(a) 1/6 (b) 1/9 Þ f ¢( x ) = 3ax 2 + 2bx + c
(c) 1/12 (d) 1/18 Þ f ¢¢( x ) = 6ax + 2b
f ¢¢¢( x ) = 6a
l Ex. 19 The degree of f ( x ), is
As, f ¢¢( x ) × f ¢( x ) = f ( x )
Þ (6ax + 2b )(3ax 2 + 2bx + c ) = (ax 3 + bx 2 + cx + d )
(a) 2 (b) 3
(c) 4 (d) 5
On comparing coefficients of x 3, we get
l Ex. 20 The value of f ¢¢¢( x ), is 1
18a 2 = a Þ a = 0,
(a) 1/18 (b) 1/3 18
1
(c) 1/9 (d) 1/6 \ a= [Qa ¹ 0]
18
Sol. (Ex. Nos. 18 to 20)
1 1
Let degree of f ( x ) = n The leading coefficient of f ( x ) = and f ¢¢¢( x ) = 6a = .
18 3
\ Degree of f ¢( x ) = n - 1, degree of f ¢¢( x ) = n - 2
18. (d) 19. (b) 20. (b)
62 Textbook of Differential Calculus

Passage II Passage III


(Ex. Nos. 21 to 22) (Ex. Nos. 23 to 24)
The ends A and B of a rod of length 5 are sliding along the curve The figure of y = P ( x ) = ax 5 + bx 4 + cx 3 + dx 2 + ex + f is
y = 2x 2 . Let x A and x B be the x-coordinate of the ends. Then, given below
Y
l Ex. 21 The moment when A is at (0, 0 ) and B is
dx (–2, 2)
at (1, 2). The derivative B , is
dx A (0, 1)
1 1
(a) (b)
9 7 X′ 0 X
–3 –2 –1 1/2 1 2
1
(c) (d) None of these –1
5 (1/2,–1)

l Ex. 22 The moment when A is at (1, 2) and B is Y′


dx
at (0, 0). The derivative B , is l Ex. 23 If P ¢¢ ( x ) has real roots a, b, g.
dx A
Then, [a ] + [b] + [ g ] is
(a) 16 (b) 8 (c) 9 (d) 2
(a) -2 (b) -3
Sol. (Ex. Nos. 21 to 22) (c) -1 (d) 0
We have, y = 2 x 2 1
Sol. (b) Here, P ¢( x ) = 0 at x = -2, -1, 0, .
( AB ) 2 = ( x B - x A ) 2 + (2 x B2 - 2 x A2 ) 2 = 5 2
Þ ( x B - x A ) 2 + 4 ( x B2 - x A2 ) 2 = 5 æ 1ö
Þ P ¢¢( x ) will have real roots belongs to ( -2,-1 ), ( -1,0 ), ç 0, ÷ .
dx B è 2ø
Differentiating w.r.t. x A and denoting =D
dx A [Q between any two roots of a function, atleast
one root of its derivative exist]
2 ( x B - x A )( D - 1 ) + 8 ( x B2 - x A2 ) × {2 x B D - 2 x A } = 0 …(i)
\ [ a ] + [b ] + [ g ] = -3
Now, when A( 0, 0 ) and B(1, 2 ) Þ x A = 0, x B = 1
Then, Eq. (i) reduces to l Ex. 24 The minimum number of real roots of equation
2 (1 - 0 ) ( D - 1 ) + 8 (1 - 0 ) {2 D - 0 } = 0 (P ¢¢ ( x )) 2 + P ¢ ( x ) × P ¢¢¢( x ) = 0
Þ 2 D - 2 + 16 D = 0 (a) 5
1 dx B 1
Þ D= Þ = (b) 7
9 dx A 9 (c) 6
Again, when A(1, 2 ) and B( 0, 0 ) Þ x A = 1, x B = 0 (d) 4
Eq. (i) reduces to Sol. (c) Let f ( x ) = ( P ¢¢( x )) 2 + P ¢( x ) × P ¢¢¢( x )
2 ( 0 - 1 )( D - 1 ) + 8 ( 0 - 1 ) { 0 - 2 } = 0 d
Þ -2 D + 2 + 16 = 0 Þ D = 9 Þ f (x ) = ( P ¢( x ) × P ¢¢( x ))
dx
dx B
\ =9 Since, P ¢( x ) has 4 real roots.
dx A
So, P ¢¢( x ) has 3 real roots.
21. (a) 22. (c) Þ f ( x ) has 6 real roots.
Chap 02 Differentiation 63

JEE Type Solved Examples :


Matching Type Questions
l Ex. 25 Match the Column I with Column II and mark the l Ex. 26 Match the Column I with Column II and mark the
correct option from the codes given below. correct option from the codes given below.
Column I Column II Column I Column II
If f ¢ (x ) = 3x + 6 and y = f (x ) then at
2 3 p. -2 (i) -1
If y = cos (cos x ), then y¢ at x = 5 is p. -1
(i) dy equal to
x = 1, is
dx (ii) For the function q. 0
-1 æ p xö
(ii) If f is a differentiable function such that q. f (x ) = logetan ç + ÷ , if
f (xy) = f (x ) + f ( y) : x, y Î R, then è 4 2ø
f (e) + f (1 / e) is dy
= sec x + p, then p is equal to
(iii) If f is a twice differentiable function such r. 0 dx
that f ¢¢(x ) = - f (x ) and f ¢ (x ) = g (x ). If æ1 + xö r. 1/2
h(x ) = [ f (x )]2 + [ g (x )]2 and h(5) = 9, then (iii) The derivative of tan -1 ç ÷ at x = - 1
è1- xø
h(10) is
is
y = tan -1 (cot x ) + cot -1 (tan x ), s. 9
log | x | s. 1
(iv) p dy (iv) The derivative of at x = - 1is
< x < p , then is x
2 dx
Codes Codes
i ii iii iv i ii iii iv i ii iii iv i ii iii iv
(a) s r s p (b) p q r s (a) p q r s (b) q p r s
(c) s r q p (d) r s p q
(c) q p r r (d) s p q q
Sol. (a) (i) We have, y = cos-1(cos x )
Sol. (a) (i) Q y = f (x )
3
ì x, 0£x£p dy ì 1, 0£x£p
dy y =í Þ =í
\ = f ¢( x 3 ) × 3 x 2 î 2 p - x , p £ x £ 2 p dx î - 1, p £ x £ 2p
dx
æ dy ö
æ dy ö Q 5 Î[ p , 2 p ] \ ç ÷ = -1
ç ÷ = f ¢(1 ) × 3 = 9 è dx ø x = 5
è dx ø x = 1
æp xö
(ii) Q f ( xy ) = f ( x ) + f (y ) (ii) We have, f ( x ) = log tan ç + ÷
è 4 2ø
Put x = y = 1,
f (1 ) = f (1 ) + f (1 )
1 æp xö 1
f ¢( x ) = ´ sec 2 ç + ÷ ´
æ p x ö è 4 2ø 2
\ f (1 ) = 0 tan ç + ÷
è 4 2ø
Also, f (1 ) = f (e ) + f (1 / e )
1
i.e. x = e, y =
1 f ¢( x ) =
æp xö æp xö
e 2 sin ç + ÷ cos ç + ÷
è 4 2ø è 4 2ø
\ f (e ) + f (1 / e ) = 0
1 1
(iii) Q f ¢¢( x ) = - f ( x ) and g( x ) = f ¢( x ) f ¢( x ) = = = sec x
æp ö cos x
\ g ¢( x ) = f ¢¢( x ) = - f ( x ) …(i) sin ç + x ÷
è2 ø
Also, h( x ) = [ f ( x )]2 + [ g( x )]2 \ p=0
\ h ¢( x ) = 2 f ( x ) f ¢( x ) + 2 g ( x ) g ¢( x ) æ1 + x ö
(iii) Let y = tan -1 ç -1
÷ Þ y = tan (1 ) + tan x
-1

= 2 f ( x ) g( x ) - 2 g( x ) f ( x ) = 0 è1 - x ø
\ h( x ) = c, " x Î R dy 1 æ dy ö 1
\ = Þç ÷ =
Here, h(5 ) = 9 Þ h(10 ) = 9 dx 1 + x 2 è dx ø x = - 1 2
p dy - log | x|
(iv)Q y = tan (cot x ) + cot -1(tan x ), < x < p
-1
(iv) Let y =
log | x|
Þ =
1
+ 2
2 x dx x2 x
dy - cosec 2x ( - 1 )sec 2 x æ dy ö - log 1 1
\ = + = -1 -1 = -2 ç ÷ = + =1
dx 1 + cot 2 x 1 + tan 2 x è dx ø x = - 1 1 1
Hence, (i) ® s; (ii) ® r; (iii) ® s; (iv) ® p Hence, (i) ® p; (ii) ® q; (iii) ® r; (iv) ® s
64 Textbook of Differential Calculus

JEE Type Solved Examples :


Single Integer Answer Type Questions

Ex. 27 If y = ( x - sin x ) + ( x - sin x ) +..., é æa + b öù


Sol. (4) Let g( x ) = f ¢( x )sin(pf ( x )) = 2 ê x – ç
l
÷ sin ( p f ( x ))
ë è 2 ø úû
2
dx
then - 2 p = ……… . g ¢( x ) = p ( f ¢( x )) 2 cos( p f ( x )) + sin ( p f ( x )) f ¢¢ ( x ) = 0
dy x=
p æa + b ö
g(a ) = 0 = g(b ) = g ç ÷ = g( a ) = g(b )
2 è 2 ø
Sol. (3) Here, y = ( x - sin x ) + ( x - sin x ) + .... ¥
\ g ¢( x ) must have atleast one root in the intervals ( a, a ),
So, y = ( x - sin x ) + y æ a + b ö æa + b ö
ç a, ÷, ç , b ÷ and (b, b). i.e. minimum 4 roots.
è 2 øè 2 ø
\ y 2 = x - sin x + y
Differentiating, we get x
Ex. 29 Let f ( x ) = ò e (1+t ) dt and g ( x ) = f (h( x )),
2
l
dy dy -2
2y = 1 - cos x + …(i)
dx dx where h( x ) is defined for all x Î R. If g ¢ ( 2 ) = e 4 and h¢ ( 2 ) = 1,
p p 1 p 3
At x = , y 2 -y = -1 Þ y 2 -y + = - then absolute value of sum of all possible values of h( 2 ), is
2 2 4 2 4
2
2p - 3
…...... .
æ 1ö
Þ çy - ÷ = Þ (2y - 1 ) = ± 2 p - 3 x 2
è 2ø 4 Sol. (2) Here, f ( x ) = ò e (1+ t ) dt
-2
dy
From Eq. (i), we get (2y - 1 ) = 1 - cos x h (x ) 2
dx g(x ) = ò-2 e (1+t ) dt

(1+h ( x ) )2
\
dy
=
1
Þ
dx 2
- 2p = 3 Þ g ¢( x ) = h ¢( x ) ×e [by Leibnitz Rule]
dx x=
p 2p - 3 dy x=
p
( 1 + h ( 2) ) 2
2 2 Þ g ¢(2 ) = h ¢(2 ) × e
2
Þ e 4 = 1 × e (1+ h ( 2)) [given g ¢(2 ) = e 4 and h¢(2 ) = 1)]
l Ex. 28 If f ( x ) = ( x - a )( x - b ) for a , b Î R. Then, the
Þ (1 + h(2 )) 2 = 4 Þ h(2 ) = -3, 1
minimum number of roots of equation
p ( f ¢ ( x )) 2 cos (pf ( x )) + sin( pf ( x )) f ¢¢ ( x ) = 0 in (a, b), \ Absolute value of sum of all possible values of
h(2 ) = - 3 + 1 = 2.
where f (a ) = 3 = f (b), is ......... (where a < a < b < b).

Subjective Type Questions


ìp ü æ pö æpö
2/ 3
p æ pö
l Ex. 30 If f ( x ) = cos í [ x ] – x 3 ý,1 < x < 2 and [ x ] = the \ f ¢ ç 3 ÷ = 3 ç ÷ × cos = 0 Þ f ¢ ç 3 ÷ = 0
î2 þ è 2ø è2ø 2 è 2ø
æ pö
greatest integer £ x, then find f ¢ ç 3 ÷ .
è 2ø
l Ex. 31 If u = f ( x 3 ), v = g ( x 2 ), f ¢ ( x ) = cos x and
du
p g ¢ ( x ) = sin x , then find .
Sol. As we know, 1 < 3 <2 dv
2
p Sol. Here, u = f (x 3 )
ìp ü
\ If x = 3 Þ [ x ] =1, so f ( x ) = cos í – x 3ý
2 î2 þ du d
Þ = f ¢( x 3 ) × ( x 3 ) = {cos( x 3 )} ×3 x 2 = 3 x 2 × cos x 3
dx dx
é p ù
f ( x ) = sin x 3 ê at x = 3 Î (1, 2 ) ú and v = g( x 2 )
ë 2 û
dv d
Þ f ¢( x ) = cos x 3 × 3 x 2 Þ = g ¢( x 2 ) × ( x 2 ) = {sin x 2 } ×(2 x ) = 2 x ×sin x 2
dx dx
Chap 02 Differentiation 65

du sin x
du dx 3 x 2 × cos x 3
l Ex. 35 If y = , prove that
\ = = cos x
dv dv 2 x × sin x 2 1+
sin x
dx 1+
Þ
du 3
= x × cos x 3 × cosec x 2
1 + cos x ... ¥
dv 2 dy (1 + y ) cos x + sin x
= .
dx 1 + 2y + cos x – sin x
l Ex. 32 Find a , b, c and d, where
sin x (1 + y ) sin x
f ( x ) = (ax + b ) cos x + (cx + d ) sin x and f ¢ ( x ) = x cos x is Sol. Given function is y = =
cos x 1 + y + cos x
identity in x. 1+
1+y
Sol. Here, f ¢( x ) = x cos x
Þy + y 2 + y cos x = (1 + y ) sin x
Þa cos x – (ax + b ) sin x + c sin x + (cx + d ) cos x º x cos x
On differentiating both the sides w.r.t. x, we get
or (a + cx + d ) cos x + (–ax – b + c ) sin x º x cos x + 0 × sin x
dy dy dy
Þ a + d + cx = x and –ax – b + c = 0 + 2y + y (–sin x ) + cos x ×
dx dx dx
which is again identity in x. dy
Þ a + d = 0, c = 1, – a = 0, – b + c = 0 = (1 + y ) cos x + × sin x
dx
Þ a = 0, b = 1, c = 1, d = 0 dy
Þ {1 + 2y + cos x – sin x } = (1 + y ) cos x + y sin x
dx
l Ex. 33 If f ( x ) = x 3 + x 2 f ¢ (1 ) + xf ¢¢( 2 ) + f ¢¢¢(3 ) for all dy (1 + y ) cos x + y sin x
Þ =
x Î R. Then, find f ( x ) independent of f ¢ (1 ), f ¢¢( 2 ) and dx (1 + 2y + cos x – sin x )
f ¢¢¢(3 ).
Sol. Here, f ( x ) = x 3 + x 2 f ¢(1 ) + x f ¢¢(2 ) + f ¢¢¢(3 ) x dy
l Ex. 36 If y = , then find .
Put f ¢(1 ) = a, f ¢¢(2 ) = b, f ¢¢¢(3 ) = c ...(i) 3
x dx
x+
\ f ( x ) = x 3 + ax 2 + bx + c 3
x
x+
Þ f ¢( x ) = 3 x + 2ax + b
2
or f ¢(1 ) = 3 + 2a + b ...(ii) x + x + ....¥3

Þ f ¢¢( x ) = 6 x + 2a or f ¢¢(2 ) = 12 + 2a ...(iii) x


Sol. y =
Þ f ¢¢¢( x ) = 6 or f ¢¢¢(3 ) = 6 ...(iv) 3
x
x+
From Eqs. (i) and (iv), c =6 x 3
x+
From Eqs. (i), (ii) and (iii), we have a = – 5, b = 2 x + x + .... ¥
3

\ f (x ) = x 3 – 5x 2 + 2x + 6 x
Þ y =
x
x+ × x – 2/ 3
l Ex. 34 Let f ( x ) = x + xg ¢ (1 ) + g ¢¢ ( 2 ) and
2
x 3
x+
g ( x ) = f (1 ) × x 2 + xf ¢ ( x ) + f ¢¢ ( x ), then find f ( x ) and g ( x ) . x + 3 x + ....¥
x
Sol. Here, put g ¢(1 ) = a, g ¢¢ (2 ) = b ...(i) Þ y =
x + y × x –2/ 3
Then, f ( x ) = x 2 + ax + b, f (1 ) = 1 + a + b
Þ y { x 5/ 3 + y } = x 5/ 3
Þ f ¢( x ) = 2 x + a, f ¢¢ ( x ) = 2
\ g( x ) = (1 + a + b ) x 2 + (2 x + a ) × x + 2 or x 5/ 3y + y 2 = x 5/ 3

= x 2 (3 + a + b ) + ax + 2 Differentiating both the sides w.r.t. x, we get

Þ g ¢( x ) = 2 x (3 + a + b ) + a x 5/ 3 ×
dy 5 2/ 3
+ × x × y + 2y
dy 5 2/ 3
= ×x
Hence, g ¢(1 ) = 2 (3 + a + b ) + a ...(ii) dx 3 dx 3
g ¢¢(2 ) = 2 (3 + a + b ) ...(iii) dy 5 2/ 3 5 2/ 3
Þ ( x 5/ 3 + 2y ) = x – x y
From Eqs. (i), (ii) and (iii), we have dx 3 3
a = 2 (3 + a + b ) + a and b = 2 (3 + a + b ) 5 2/ 3
i.e. 3 + a + b = 0 and b + 2a + 6 = 0 x (1 – y )
dy 3
or =
Hence, b = 0 and a = – 3 dx ( x 5/ 3 + 2y )
So, f ( x ) = x 2 – 3 x and g( x ) = – 3 x + 2
66 Textbook of Differential Calculus

l Ex. 37 If y = tan –1
1
+ tan –1
1 l Ex. 39 Find the sum of the series
x +x + 1
2
x + 3x + 3
2 sin x + 3 sin 3 x + 5 sin 5 x + ...+ ( 2k – 1) sin ( 2k – 1) x
1 (using calculus).
+ tan –1 +... to n terms, show that
Sol. Let S = cos x + cos 3 x + cos 5 x + ......+ cos(2k – 1 ) x
x + 5x + 7
2

dy 1 1 Here, the angles are in AP whose first term is x and common


= – . difference is 2x.
dx ( x + n ) +1 x + 1
2 2
æ k × 2x ö
sin ç ÷
Sol. Given, y = tan –1
1
+ tan –1 2
1 è 2 ø ì x (k – 1 )2 x ü sin kx
\ S= × cos í + ý= × cos kx
x2 + x + 1 x + 3x + 3 æ 2x ö î1 2 þ sin x
sin ç ÷
1 è2 ø
+ tan –1 + ... to n terms
x 2 + 5x + 7 or {cos x + cos3 x + cos5 x + ... + cos(2k – 1 ) x } =
sin 2kx
...(i)
2 sin x
–1 ì 1 ü –1 ì 1 ü
= tan í ý + tan í ý
î 1 + x ( x + 1 ) þ î 1 + ( x + 1 )( x + 2 ) þ On differentiating Eq. (i) w.r.t. x, we get

ì 1 ü – {sin x + 3 sin 3 x + 5 sin 5 x + ... + (2k – 1 ) sin (2k – 1 ) x }


+ tan –1 í ý
î 1 + ( x + 2 )( x + 3 )þ 1 ì 2k (cos 2kx ) × sin x – (sin 2kx ) × cos x ü
= í ý
2î sin 2 x þ
ì 1 ü
+ ... + tan –1 í ý \ [sin x + 3 sin 3 x + 5 sin 5 x + ...+ (2k – 1 ) sin (2k – 1 ) x ]
î 1 + { x + (n – 1 )} { x + n } þ
1 é
=– k {sin (2k + 1 ) x – sin (2k – 1 ) x }
ì (x + 1) – x ü
= tan –1 í –1 ì ( x + 2 ) – ( x + 1 ) ü 2 sin x êë
2
ý + tan í ý
î 1 + ( x + 1 ) x þ î 1 + ( x + 2 )( x + 1 )þ 1 ù
– {sin (2k + 1 ) x + sin (2k – 1 ) x } ú
ì (x + 3) – (x + 2)ü 2 û
–1 ì ( x + n ) – ( x + n – 1 ) ü
+ tan –1 í ý + ...+ tan í ý =
1
[(2k + 1 ) sin (2k – 1 ) x – (2k – 1 ) sin (2k + 1 ) x ]
î 1 + ( x + 3 )( x + 2 ) þ î 1 + ( x + n )( x + n – 1 )þ 4 sin 2 x
\ y = {tan –1( x + 1 ) – tan –1( x )} + {tan –1( x + 2 ) n

– tan ( x + 1 )} + {tan ( x + 3 )– tan ( x + 2 )}


–1 –1 –1 l Ex. 40 Find the sum of series å r x r –1 , using calculus.
r =1
+ ... + {tan –1( x + n ) – tan –1( x + n – 1 )}
Sol. Let S = 1 + x + x 2 + x 3 + x 4 + ... + xn
y = tan –1( x + n ) – tan –1( x )
which is a geometric progression.
On differentiating both the sides w.r.t. x, we get
1 (1 – xn + 1 )
dy
=
1

1 \ S = 1 + x + x 2 + x 3 + K + xn =
dx 1 + ( x + n ) 2 1 + x 2 1– x
On differentiating both the sides, we get
l Ex. 38 If f (q ) = cos q 1 × cos q 2 × cos q 3 ... cos q n , show 0 + 1 + 2 x + 3 x 2 + 4 x 3 + K + nxn – 1
ì f ¢ (q ) ü (1 – x ) × [– (n + 1 ) xn ]–(1 – xn + 1 ) × (–1 )
that {tan q 1 + tan q 2 + tan q 3 + ... + tan q n } = – í ý, =
(1 – x ) 2
î f (q ) þ
n
dq 1 dq 2 dq 1
where = = ... = n =1 . \ år xr –1 = (1 – x )2 × {1 – (n + 1) xn + n × xn + 1 }
dq dq dq r =1

Sol. f ( q) = cos q1 × cos q2 × cos q3 ... cos qn


l Ex. 41 Use calculus to find the sum of
Taking log on both the sides,
é 1 2n ù
log f ( q) = log(cos q1 ) + log(cos q2 ) + ... + log(cos qn ) ê +
2
+
4
+K + n ú.
On differentiating both the sides w.r.t. q, we get êë x + 1 x 2 + 1 x 4 + 1 x 2 + 1úû
1 1 1
× f ¢( q ) = × (–sin q1 ) + × (–sin q2 )
f ( q) cos q1 cos q2 Sol. We know that, (1 – x )(1 + x ) = (1 – x 2 )
1 2
+ ... + × (–sin qn ) (1 – x )(1 + x )(1 + x 2 ) = (1 – x 4 ) = (1 – x 2 )
cos qn 3
(1 – x )(1 + x )(1 + x 2 )(1 + x 4 ) = (1 – x 8 ) = (1 – x 2 )
ì f ¢( q) ü
Hence, (tan q1 + tan q2 + ... + tan qn ) = – í ý n n+1

î f ( q) þ (1 – x )(1 + x )(1 + x 2 )(1 + x 4 ) ... (1 + x 2 ) = (1 – x 2 ) ...(i)


Chap 02 Differentiation 67

Taking log on both the sides of Eq. (i), we get fn – 1 ( x )


n n+1
l Ex. 42 If f n ( x ) = e for all n Î N and f 0 ( x ) = x ,
log {(1 – x )(1 + x )(1 + x 2 )(1 + x 4 ) ... (1 + x 2 )} = log(1 – x 2 ) d
then find { f n ( x )}.
or log(1 – x ) + log(1 + x ) + log(1 + x 2 ) + log(1 + x 4 ) dx
n n+1 fn – 1 ( x )
+ ...+ log (1 + x 2 ) = log(1 – x 2 ) Sol. Here, fn ( x ) = e ...(i)
On differentiating the above equation, we get d f (x ) d
Þ { fn ( x )} = e n – 1 × { fn – 1( x )}
2n – 1 dx dx
1(–1 ) 1 ×1 1 (2 x ) 1 × ( 4 x ) 1 ×2 × x
3 n
+ + + +K+ d d
1 – x 1 + x 1 + x2 1 + x4 1+ x2
n
or { fn ( x )} = fn ( x ) { fn – 1( x )}
n+1
dx dx
–1 × 2n + 1 × x 2 –1
[using Eq. (i)] ...(ii)
= n+1
1– x 2 Replacing n by n – 1 in Eq. (ii), we have
d d
n { fn – 1( x )} = fn – 1 ( x ) × { fn – 2( x )} ...(iii)
1 2x 4x 3 2n × x 2 – 1 dx dx
or + + +K+
1+x 1+x 2
1+x 4
1+ x2
n
\ From Eqs. (ii) and (iii), we get
n+1 d d
1 2n + 1 × x 2 –1
{ fn ( x )} = fn ( x ) × fn – 1( x ) × { fn – 2( x )} ...(iv)
= – n+1 dx dx
1– x 1– x 2 d d
Similarly, { fn – 2( x )} = fn – 2 ( x ) × { fn – 3( x )}
Multiplying both the sides by x, dx dx
é x n
ù d d
2x 2 4x 4 2n × x 2 { f1( x )} = f1( x ) { f 0( x )} …(v)
ê + + + ... + ú dx dx
êë 1 + x 1 + x 1 + x4
2 n
1 + x2 úû From Eqs. (iv) and (v), we get
n+1
x 2n + 1 × x 2 d ìd ü
= – { fn ( x )} = fn ( x ) fn – 1( x ) × fn – 2( x ) ... f 2( x ) × f1( x ) í f 0( x )ý
1– x 1 – x 2n + 1 dx î dx þ
d
(x + 1) – 1 2 (x 2 + 1) – 2 4 (x 4 + 1) – 4 Þ { fn ( x )} = fn ( x ) fn – 1( x ) × fn – 2( x ) ... f 2( x ) × f1( x ) × 1
or + + dx
1+x 1 + x2 1 + x4
é. . d ù
n
2n ( x 2 + 1 ) – 2n x x2
n+1
êë . f 0( x ) = x Þ dx { f 0( x )} = 1 úû
+ ... + n = – 2n + 1 × n+1
1 + x2 1– x 1 – x2
n+1 d 2y 24y
x 2n + 1 × x 2 l Ex. 43 If y 3 – y = 2 x , then prove that =– .
or [(1 + 2 + 2 2 + ... + 2n )– P ] = – n+1 dx 2
(3y 2 – 1) 3
1– x 1 – x2
2n æ 1 ö d 2y dy 1
where, P =
1
+
2
+
4
+ ... + Hence, show that ç x 2 – ÷ +x× = y.
1+x 1+x 2
1+x 4
1 + x2
n è 27 ø dx 2 dx 9
n+1
2n + 1 – 1 x 2n + 1 × x 2 Sol. Given, y 3 – y = 2x ...(i)
Then, P = – + n+1
2–1 1– x 1 – x2 On differentiating both the sides w.r.t. x, we get
n+1 dy dy 2
x 2n + 1 × x 2 (3y 2 – 1 ) = 2 or = ...(ii)
Þ P = 2n + 1 – 1 – + n+1
dx dx 3y 2 – 1
1– x 1 – x2
Again, differentiating w.r.t. x,
é x2
n+1
ù é x ù dy
Þ P = 2n + 1 ê1 + ú – ê1 + ú d 2y
– 2(6y ) ×
êë
n+1
ú = dx = –12y × 2
û ë
1– x 2 1 – xû [from Eq. (ii)]
dx 2 (3y 2 – 1 ) 2 (3y 2 – 1 ) 2 (3y 2 – 1 )
é 1 ù é 1 ù
Þ P = 2n + 1 ê ú–ê ú \
d 2y
=–
24y
2n + 1 ...(iii)
êë 1 – x úû ë 1 – x û dx 2
(3y 2 – 1 ) 3
ìï 1 2n üï 1 ö d 2y
+
2
+
4
+ ... + æ dy
Hence, í n ý Now, LHS = ç x 2 – ÷ 2 + x ×
ïî 1 + x 1 + x
2
1+x 4
1 + x 2 ïþ è 27 ø dx dx
æ y 2 (y 2 – 1 ) 2 1 ö æ –24y ö y (y 2 – 1 )
ïì 2
n+1 ü 2
ï ì 1 ü =ç – ÷ç 2 ÷+ × 2
=í ý –í ý è 4 27 ø è (3y – 1 ) 3 ø 2 (3y – 1 )
2n + 1
ïî 1 – x ïþ î 1 – x þ
[from Eqs. (i), (ii) and (iii)]
68 Textbook of Differential Calculus

æ 27y 2 (y 2 – 1 ) 2 – 4 ö –24y y (y 2 – 1 ) a1 a2x


=ç ÷ + l Ex. 45 If y = 1 + +
è 108 ø (3y 2 – 1 ) 3 (3y 2 – 1 ) (x – a1 ) (x – a1 ) (x – a 2 )
y é – 54y 2 (y 2 – 1 ) 2 + 8 9 (y 2 – 1 ) ù a3 x 2
= ê + ú + + ... to (n +1) terms, show that
9ë (3y 2 – 1 ) 3 (3y 2 – 1 ) û (x – a1 ) (x – a 2 ) (x – a3 )
Let (3y 2 – 1 ) = a dy y é x x x x ù
= ên – - K–
y é –2 (1 + a ) ( a – 2 ) 2 + 8 3 ( a – 1 ) ù
x – a n úû
– – .
Then, LHS = ê + ú dx x ë x – a1 x – a 2 x – a3
9 ë a3 a û
a1 a 2x
y é –2 a 3 + 6 a 2 + 3 a 3 – 6 a 2 ù Sol. Here, y = 1 + +
= ( x – a1 ) ( x – a1 ) ( x – a 2 )
ê ú
9 ë a3 û a3 x 2
y + + K to (n + 1) terms
= = RHS ( x – a1 ) ( x – a 2 )( x – a 3 )
9
x a 2x
æ 1 ö d 2y dy y Þ y = +
Hence, ç x 2 – ÷ 2 + x × = ( x – a1 ) ( x – a1 ) ( x – a 2 )
è ø
27 dx dx 9
a 3x 2
+ + Kto (n ) terms
l Ex. 44 If 2 x = y 1/ 5 + y –1/ 5 , then express y as an explicit ( x – a1 ) ( x – a 2 )( x – a 3 )

d 2y dy [adding first two terms]


function of x and prove that ( x 2 – 1) +x× = 25 y . x2 a 3x 2
dx 2 dx Þ y = +
( x – a1 ) ( x – a 2 ) ( x – a1 ) ( x – a 2 )( x – a 3 )
Sol. We have, (y 1/ 5 + y –1/ 5 ) 2 = (y 1/ 5 – y –1/ 5 ) 2 + 4
+ ... to (n – 1 ) terms
Then, (y 1/ 5 – y –1/ 5 ) 2 = 4 x 2 – 4 [Q 2 x = y 1/ 5 + y –1/ 5]
[again adding first two terms]
\ y 1/ 5 – y – 1/ 5 = 2 x 2 – 1 …(i) Proceeding in the same way, we get
and y 1/ 5
+y – 1/ 5
= 2x [given]...(ii) xn
y = ...(i)
( x – a1 ) ( x – a 2 )( x – a 3 ) K( x – an )
Adding Eqs. (i) and (ii), we get
2y 1/ 5 = 2 x + 2 x 2 – 1 Taking logarithm on both the sides of Eq. (i), we have
log y = n log x – log( x – a1 )– log( x – a 2 )– log ( x – a 3 )
\ y 1/ 5
=x+ 2
x –1
– K– log ( x – an )
or y = (x + x 2 – 1 )5 ...(iii) On differentiating both the sides w.r.t. x, we get
On differentiating both the sides w.r.t. x, we get 1 dy n 1 1 1
× = – – – ... –
dy ìï 1 × 2 x üï y dx x ( x – a1 ) ( x – a 2 ) ( x – an )
= 5 ( x + x 2 – 1 ) 4 × í1 + ý
dy y é x x x ù
îï 2 x 2 – 1 þï
dx
or = n– – – ... –
dx x êë ( x – a1 ) ( x – a 2 ) ( x – an ) úû
dy 5 ( x + x – 1 )
2 5
Þ =
dx x2 – 1 l Ex. 46 If for all x , y the function f is defined by
dy 5y
Þ = [using Eq. (iii)] f ( x ) + f (y ) + f ( x ) × f (y ) =1 and f ( x ) > 0.
dx x2 – 1
2 Then, show f ¢ ( x ) = 0, when f ( x ) is differentiable.
æ dy ö
Þ ( x 2 – 1 ) ç ÷ = 25y 2 ...(iv) Sol. Here, f ( x ) + f (y ) + f ( x ) × f (y ) = 1 ...(i)
è dx ø
Put x = y = 0, we get
Again, differentiating both the sides w.r.t. x, we get
2 2 f ( 0 ) + { f ( 0 )} 2 = 1
æ dy ö dy d 2y dy
2 x ç ÷ + ( x 2 – 1 ) × 2 × 2 = 50y ×
è dx ø dx dx dx Þ { f ( 0 )} 2 + 2 f ( 0 ) – 1 = 0
dy –2 ± 4 + 4
On dividing by 2 on both the sides, f (0) = = – 1 – 2 and –1 + 2
dx 2
dy d 2y é dy ù As f (0) > 0 Þ f (0) = 2 –1
x× + ( x 2 – 1 ) 2 = 25y êëQ dx ¹ 0 úû
dx dx [neglecting f ( 0 ) = – 1 – 2 as f ( 0 ) is positive]
Chap 02 Differentiation 69

Again, putting y = x in Eq. (i), Sol. Let f ( x ) = lx 2 + mx + n


2 f ( x ) + { f ( x )} = 1
2
Then, f ¢( x ) = 2lx + m
On differentiating w.r.t. x, we have Also, f (1 ) = f (–1 )
2 f ¢( x ) + 2 f ( x ) f ¢( x ) = 0 Þ l + m + n = l–m + n
2 f ¢( x ) {1 + f ( x )} = 0 Þ m=0
Þ f ¢( x ) = 0 \ f ¢( x ) = 2lx
Because f (x ) > 0 Þ f ¢(a1 ) = 2la1, f ¢(a 2 ) = 2la 2
and f ¢(a 3 ) = 2la 3
Therefore, f ¢( x ) = 0 when f ( x ) > 0
As, a1, a 2, a 3 are in AP. Therefore, f ¢(a1 ), f ¢(a 2 ), f ¢(a 3 ) are in
æ 2x – 1 ö AP.
l Ex. 47 If y = f ç 2 ÷ and f ¢ ( x ) = sin x 2 , then find
è x + 1ø æ1ö
l Ex. 49 If 5 f ( x ) + 3 f ç ÷ = x + 2 and y = x f ( x ), then
dy èxø
. dy
dx find at x =1.
dy d æ 2x – 1 ö dx
Sol. Here, = f ç ÷ æ1ö
dx dx è x 2 + 1 ø Sol. Here, 5 f ( x ) + 3 f ç ÷ = x + 2 ...(i)
èxø
æ 2x – 1 ö æ 2x – 1 ö
df ç 2 ÷ d ç 2 ÷ 1 æ1ö 1
è x + 1ø è x + 1ø Put x = , we get 5 f ç ÷ + 3 f ( x ) = + 2 ...(ii)
= × x èxø x
æ 2x – 1 ö dx
d ç 2 ÷ Solving Eqs. (i) and (ii), we get
è x + 1ø 3
16 f ( x ) = 5 x – + 4 ...(iii)
æ 2 x – 1 ö ì ( x 2 + 1 ) × (2 ) – (2 x – 1 ) × (2 x )ü x
= f ¢ç 2 ÷ ×í ý
è x + 1ø î (x 2 + 1)2 þ Þ y = x f (x )
1ì 3 ü
2
æ 2x – 1 ö 2 { x 2 + 1 – 2x 2 + x } Þ y = x × í5 x – + 4ý
= sin ç 2 ÷ × 16 î x þ
è x + 1ø (x 2 + 1)2 1
2
Þ y = { 5x – 3 + 4x }
2

æ 2 x – 1 ö 2 (1 + x – x 2 ) 16
= sin ç 2 ÷ × dy 1
è x + 1 ø (x 2 + 1)2 or = { 10 x + 4 }
dx 16
2
dy 2 (1 + x – x 2 ) æ 2x – 1 ö dy
Þ = ×sin ç 2 ÷ Now, at x = 1,
dx (x 2 + 1)2 è x + 1ø dx
æ dy ö 10 + 4 14 7
ç ÷ = = =
è dx ø at x = 1 16 16 8
l Ex 48 Let f ( x ) be a polynomial function of second
degree. If f (1) = f (–1) and a 1 , a 2 , a 3 are in AP, then show æ dy ö 7
\ ç ÷ =
that f ¢ (a 1 ), f ¢ (a 2 ), f ¢ (a 3 ) are in AP. è dx ø at x = 1 8
Differentiation Exercise 1 :
Single Option Correct Type Questions
sec x - tan x dy x dy
1. If y = , then equals 7. If y = , then equals
sec x + tan x dx x dx
a+
(a) 2sec x (sec x - tan x ) (b) - 2 sec x (sec x - tan x ) 2 x
b+
(c) 2 sec x(sec x + tan x ) 2
(d) - 2 sec x (sec x + tan x ) 2 x
a+
b +¼¥
1+ x 2 + x 4 dy
2. If y = and = ax + b, then (a)
a
(b)
b
1+ x + x 2 dx ab + 2ay ab + 2by
(a) a =2, b =1 (b) a = -2, b =1 a b
(c) (d)
(c) a =2, b = -1 (d) a = -2, b = -1 ab + 2by ab + 2ay
3. Which of the following could be the sketch of graph of 2 dy
d 8. If y = x x , then equals
y = ( x log x )? dx
dx (a) 2 log x × x 2
2
Y Y (b) (2 log x + 1 ) × x x
2
+1
(c) (log x + 1 ) × x x
(a) 1 (b) 2
+1
(d) x x ×(log(ex 2 ))
X X dy
0 0 1 9. If x 1 + y + y 1 + x = 0, then equals
dx
Y Y 1 -1
(c) (d) (a) (b)
(1 + x ) 2 (1 + x ) 2
-1 1
0
X
0 e
X (c) + (d) None of these
1/e (1 + x ) (1 + x ) 2

dy
10. If x 2 e y + 2xye x + 13 = 0, then equals
4. Let f ( x ) = x + 3 log( x - 2) and g ( x ) = x + 5 log( x - 1), dx
then the set of x satisfying the inequality f ¢ ( x ) < g ¢ ( x ) - 2 xe y - x + 2y ( x - 1 ) é 2 xe y - x + 2y ( x + 1 ) ù
(a) (b) - ê ú
is x ( xe y - x + 2 ) ë x ( xe
y -x
+ 2) û
(a) (2, 7 / 2 ) (b) (1, 2 ) È (7 / 2, ¥ ) 2 xe x - y + 2y ( x - 1 )
(c) (2, ¥ ) (d) (7 / 2, ¥ ) (c) (d) None of these
x ( xe y - x + 2 )
æx 2 -y 2 ö dy
5. If cos -1 ç ÷ = log a, then is y +¼¥ dy
èx +y ø
2 2 dx 11. If x = e y + e ; x > 0, then equals
dx
x y y x 1+ x
(a) - (b) - (c) (d) (a)
x
(b)
y x x y 1+ x x
6. If f ( x ) = (| x | )| sin x | , then f ¢ ( - p / 4 ) is equal to 1-x 1
(c) (d)
x x
æpö
1/ 2
æ 2 4 2 2ö
(a) ç ÷ ×ç log - ÷ x 10
è 4ø è 2 p p ø 12. Let g be the inverse function of f and f ¢ ( x ) = .
æ 2
1+ x 2
4 2 2ö
1/ 2
æpö
(b) ç ÷ ×ç log + ÷ If g (2) = a, then g ¢ (2) is equal to
è 4ø è 2 p p ø
5 1+a2
æpö
1/ 2
æ 2 p 2 2ö (a) (b)
(c) ç ÷ ×ç log - ÷ 210 a10
è 4ø è 2 4 p ø a10 1 + a10
(c) (d)
æpö
1/ 2
æ 2 p 2 2ö 1+a2 a2
(d) ç ÷ ×ç log + ÷
è 4ø è 2 4 p ø
Chap 02 Differentiation 71

13. If f and g are the functions whose graphs are as shown, 20. Let g ( x ) = loge f ( x ), where f ( x ) is twice differentiable
let u( x ) = f ( g ( x )); w ( x ) = g ( g ( x )) positive function on (0, ¥) such that f ( x + 1) = x f ( x ), for
Y
N = 1, 2, 3, …, then
æ 1ö æ1ö
5 f (2,4) g ¢¢ ç N + ÷ - g ¢¢ ç ÷ equals
è 2 ø è2ø
4
3)

ì 1 1 ü
(1 ,

3 1
(a) - 4 í1 + + + ¼+ 2ý
2 (6,3) î 9 25 (2 N - 1 ) þ
1 (2,0)
ì 1 1 1 ü
X′
–1 0 1 2 3 4 5 6
X (b) 4 í1 + + + ¼+ 2ý
î 9 25 (2 N - 1 ) þ
ì 1 1 1 ü
Y′ (c) - 4 í1 + + + ¼+ 2ý
î 9 25 (2 N + 1 ) þ
Then, the value of u ¢ (1) + w ¢ (1) is
1 3 5 ì 1 1 1 ü
(a) - (b) - (c) - (d) does not exist (d) 4 í1 + + + ¼+ 2ý
2 2 4 î 9 25 (2 N + 1 ) þ
14. If f ¢ ( x ) = g ( x ) and g ¢ ( x ) = - f ( x ) for all real x and -1
21. If the function f ( x ) = x 3 + e x / 2 and g ( x ) = f ( x ), then
f (5) = 2 = f ¢ (5), then f (10) + g (10) equals
2 2
the value of g ¢ (1) is
(a) 2 (b) 4 (a) 1 (b) 2
(c) 8 (d) None of these (c) 3 (d) e
x2 x3 xn
15. If f ( x ) = x + + +¼+ , then
1! 2! (n - 1)! æ æ sin q ö ö p p
22. Let f (q) = sin çç tan -1 çç ÷÷ ÷, where - < q < .
÷
f (0) + f ¢ (0) + f ¢¢(0) + ¼+ f (0) is equal to è cos 2 q ø ø
n
è 4 4
n (n + 1 ) (n 2 + 1 ) d
(a) (b) Then, the value of × f (q ), is
2 2 d (tan q)
2
æ n + 1) ö
n ( n (n + 1 )(2n + 1 ) (a) 1 (b) 2
(c) ç ÷ (d)
è 2 ø 6 (c) 3 (d) 4
16. If y = ( fofof )( x ) and f (0) = 0,f ¢ (0) = 2 , then y ¢ (0) is equal dy p
to 23. If y = log sin x (tan x ), then at x = is equal to
dx 4
(a) 6 (b) 7 (c) 8 (d) 9 4
(a) (b) - 4 log 2
17. If y 2 = p ( x ) is a polynomial of degree 3, then log 2
d æ 3 d 2y ö -4
2 çy × ÷ is equal to (c) (d) None of these
log 2
dx è dx 2 ø
(a) p ¢¢¢( x ) × p ¢( x ) (b) p ¢¢( x ) × p ¢¢¢( x ) x æ 1 ö dy
(c) p( x ) × p ¢¢¢( x ) (d) None of these 24. If y = å tan -1 çè 1 + r + r 2 ÷ø , then dx is equal to
r =1
18. If y = f ( x ) and x = g (y ) are inverse functions of each 1 1
other, then (a) (b)
1 + x2 1 + (1 + x ) 2
1 f ¢¢( x )
(a) g ¢¢(y ) = (b) g ¢¢(y ) = - (c) 0 (d) None of these
f ¢( x ) ( f ¢( x )) 3
f ¢( x ) æ sin a sin x ö
(c) g ¢¢(y ) = - (d) None of these 25. If y = sin -1 ç ÷ , then y ¢ (0) is equal to
( f ¢¢( x )) 3 è 1 - cos a sin x ø
dy d 2y (a) 1 (b) 2 tan a
19. If y is a function of x, then = 0. If x is a 2
+y × æ1ö
dx dx (c) ç ÷ tan a (d) sin a
è2ø
function of y, then the equation becomes
d 2x d 2x æ dx ö
3 æ x x - x -x ö
(a)
dx
- x× =0 (b) +y ç ÷ =0 26. If f ( x ) = cot -1 ç ÷ , then f ¢ (1) equals
dy 2 dy dy 2 è dy ø è 2 ø
2 2
d 2x æ dx ö d 2x æ dx ö (a) - 1 (b) 1
(c) -y ç ÷ = 0 (d) -x ç ÷ =0
dy 2
è dy ø dy 2
è dy ø (c) loge 2 (d) - loge 2
72 Textbook of Differential Calculus

27. The function f ( x ) = e x + x , being differentiable and 35. If f and g are differentiable functions such that g ¢ (a ) = 2
one-one, has a differentiable inverse f -1
( x ). The value and g (a ) = b and if fog is an identity function, then f ¢ (b )
has the value equal to
d
of ( f -1 ) at the point f (log 2) is (a) 2/3 (b) 1
dx (c) 0 (d) 1/2
1 1
(a) (b) 36. The derivative of the function,
ln 2 3
ì 1 ü
(c)
1
(d) None of these f ( x ) = cos -1 í (2 cos x - 3 sin x )ý
4 î 13 þ
28. If f ¢¢ ( x ) = - f ( x ) , g ( x ) = f ¢ ( x ) , -1 ì 1 ü
+ sin í (2 cos x + 3 sin x )ý
æ æ x öö
2
æ æ x öö
2 î 13 þ
F ( x ) = ç f ç ÷ ÷ + ç g ç ÷ ÷ and F (5) = 5, then F (10) is 3
è è 2 øø è è 2 øø w.r.t. 1 + x at x = is
2
4
equal to 3 5
(a) (b)
(a) 5 (b) 10 (c) 0 (d) 15 2 2
29. If x = sec q - cos q and y = secn q - cos n q, then 10
(c) (d) 0
2 3
æ dy ö
( x 2 + 4 ) ç ÷ is equal to 37. If f ( x ) = x + 2 2x - 4 + x - 2 2x - 4 , then the value
è dx ø
(a) n 2 (y 2 - 4 ) (b) n 2 ( 4 - y 2 )
of 10 f ¢ (102 + ), is
(c) n 2 (y 2 + 4 ) (d) None of these (a) –1 (b) 0
(c) 1 (d) does not exist
30. If x = f (t ) cos t - f ¢ (t ) sin t and d 2y 2
2 2 38. Let y = ln (1 + cos x ) 2 , then the value of +
æ dx ö æ dy ö dx 2 e y /2
y = f (t ) sin t + f ¢ (t ) cos t , then ç ÷ + ç ÷ is
è dt ø è dt ø equals
equal to 2
(a) 0 (b)
(a) f (t ) - f ¢¢(t ) (b) [ f (t ) - f ¢¢(t )] 2 1 + cos x
4 -4
(c) [ f (t ) + f ¢¢(t )] 2
(d) None of these (c) (d)
(1 + cos x ) (1 + cos x ) 2
d 3y
31. If y = at 2 + 2bt + c and t = ax 2 + 2bx + c , then is a + a2 - x 2 + x
dx 3 39. If f ( x ) = , where a > 0 and x < a, then
equal to a2 - x 2 + a - x
(a) 24a 2(at + b ) (b) 24a(ax + b ) 2 f ¢ (0) has the value equal to
(c) 24a(at + b ) 2 (d) 24a 2(ax + b ) (a) a (b) a
32. Differential coefficient of 1 1
1 1 1 (c) (d)
a a
æ l +m ön -l æ m +n öl -m æ n +l öm -n
çx m -n ÷
× çx n -l ÷
× çx l -m ÷
w.r.t. x, is 40. Let u( x ) and v ( x ) be differentiable functions such that
ç ÷ ç ÷ ç ÷
è ø è ø è ø u( x ) u¢ (x ) æ u( x ) ö ¢ p +q
= 7. If = p and ç ÷ = q , then has the
(a) 1 (b) 0 (c) –1 (d) x lmn v( x ) v ¢ (x ) èv( x ) ø p -q
-2
33. If y = ( A + Bx )e mx
+ (m - 1) x
e , then value equal to
(a) 1 (b) 0
d 2y dy
- 2m + m 2 y is equal to (c) 7 (d) –7
dx 2 dx
(a) e x (b) emx (c) e -mx (d) e (1 - m ) x 41. If f ( x ) = | loge | x || , then f ¢ ( x ) equals
1
x3 (a) ,x¹0
34. If f ( x ) = - + x 2 sin 1.5 a - x sin a × sin 2a - 5 |x |
3 1 1
sin –1 (a 2 - 8a + 17 ), then (b) for | x | > 1 and - for | x | < 1
x x
(a) f ( x ) is not defined at x = sin 8 1 1
(c) - for | x | > 1 and for | x | < 1
(b) f ¢(sin 8 ) > 0 x x
(c) f ¢( x ) is not defined at x = sin 8 1 1
(d) for x > 0 and - for x < 0
(d) f ¢(sin 8 ) < 0 x x
Chap 02 Differentiation 73

42. If f ( x ) is given by, f ( x ) = (cos x + i sin x ) (a) f ( x ) (b) 0


(c) f ( x ) f ¢(nx ) (d) None of these
(cos 3x + i sin 3x ) K (cos (2n - 1) x + i sin (2n - 1) x ),
then f ¢¢ ( x ) is equal to
48. If y = f ( x ) is an odd differentiable function defined on
3
(a) n f ( x ) (b) -n f ( x )
4 ( - ¥, ¥ ) such that f ¢ (3) = - 2, then f ¢ ( -3) equals
(a) 4 (b) 2
(c) -n 2 f ( x ) (d) n 4 f ( x )
(c) –2 (d) 0
43. Let f ( x ) = x n , n being a non-negative integer. The value
dy
of n for which the equality f ¢ ( x + y ) = f ¢ ( x ) + f ¢ (y ) is 49. If y = x + y + x + y + . . . ¥ , then is equal to
dx
valid for all x , y > 0, is
y +x y3 - x
(a) 0, 1 (b) 1, 2 (a) (b)
(c) 2, 4 (d) None of these y 2 - 2x 2y 2 - 2 xy - 1
p y3 + x
44. If f ( x ) = sin ìí [x ] - x 2 üý for 2 < x < 3 and [x ] denotes (c)
2y 2 - x
(d) None of these
î3 þ
p
the greatest integer less than or equal to x, 50. If f ( x ) = cos x - sin x , then f ¢ æç ö÷ is equal to
then f ¢ ( p / 3 ) is equal to è4ø
(a) 2 (b) - 2
(a) p /3 (b) - p /3
(c) 0 (d) None of these
(c) - p (d) None of these
51. Let f ( x ) = x 2 + xg ¢ (1) + g ¢ ¢ (2) and
45. The functions u = e sin x , v = e x cos x satisfy the
x
g ( x ) = x 2 + xf ¢ (2) + f ¢ ¢ (3). Then,
equation
du dv d 2u (a) f ¢(1 ) = 4 + f ¢(2 )
(a) v -u = u2 + v 2 (b) = 2v
dx dx dx 2 (b) g ¢(2 ) = 8 + g ¢(1 )
d v 2
(c) g ¢¢(2 ) + f ¢¢(3 ) = 4
(c) = - 2u (d) All of these
dx 2 (d) All of the above
46. If f ( x ) = log x {ln ( x )}, then f ¢ ( x ) at x = e , is 52. If f ( x ) = x n , then the value of
(a) e (b) - e f ¢(1) f ¢¢(1) f ¢¢¢(1) f ¢¢¢ ¢(1)
2 -1
f (1) - + - +
(c) e (d) e 1! 2! 3! 4!
47. Let f be a differentiable function satisfying ( -1)n f n (1)
-K+ is
[ f ( x )] = f (nx ) for all x Î R.
n
n!
(a) 1 (b) 2n
Then, f ¢ ( x ) f (nx ) equals
(c) 2n - 1 (d) 0

Differentiation Exercise 2 :
More than One Option Correct Type Questions
53. If y + log(1 + x ) = 0, which of the following is true? 55. If g is the inverse of f and f ( x ) = x 2 + 3x - 3, ( x > 0) ,
(a) e y = xy ¢ - 1 (b) y ¢ = -
1 then g ¢ (1) equals
(x + 1) 1 1 f ¢(1 )
(a) (b) - 1 (c) (d) -
(c) y ¢ + e y = 0 (d) y ¢ = e y 2 g(1 ) + 3 5 ( f (1 )) 2
x
54. If y = 2 3 , then y ¢ equals 56. If x 3 - 2x 2 y 2 + 5x + y - 5 = 0 and y(1) = 1, then
(a) 3 × log 3 × log 2
x
4 1
(a) y ¢(1 ) = (b) y ¢¢(1 ) = -
(b) y × (log 2 y ) × log 3 × log 2 3 3
x
(c) 2 3 × 3 x × log 6 (c) y ¢¢(1 ) = - 8
22
(d) y ¢(1 ) =
2
x 27 3
(d) 2 3 × 3 x × log 3 × log 2
74 Textbook of Differential Calculus

57. Let y = x + x + x +¼ ¥ , then dy / dx equals dy


61. If 2 x + 2 y = 2 x + y , then is equal to
dx
1 x
(a) (b) 2y 1
2y - 1 x + 2y (a) - (b)
1 y 2x 1 - 2x
(c) (d) 2 x (1 - 2 y )
1 - 4x 2x + y (c) 1 - 2 y (d)
2 y (2 x - 1 )
ln (ln x ) dy
58. If y = x (ln x ) , then is equal to 62. For the function y = f ( x ) = ( x 2 + bx + c )e x , which of
dx
y the following holds?
(a) (ln x ln x - 1 + 2 ln x ln (ln x ))
x (a) If f ( x ) > 0 for all real x Þ/ f ¢( x ) > 0
y
(b) (ln x ) ln (ln x ) (2 ln (ln x ) + 1 ) (b) If f ( x ) > 0 for all real x Þ f ¢( x ) > 0
x (c) If f ¢( x ) > 0 for all real x Þ f ( x ) > 0
y
(c) ((ln x ) 2 + 2 ln x (ln x )) (d) If f ¢( x ) > 0 for all real x Þ
/ f (x ) > 0
x ln x
dy
y ln y 63. If y + x + y - x = c , where c ¹ 0, then has the
(d) × (2 ln (ln x ) + 1 ) dx
x ln x
value equal to
59. Which of the following functions are not derivable at 2x x
(a) (b)
x =0? c2 y + y 2 - x2
æ 2x + 1 ö y - y 2 - x2 c2
(a) f ( x ) = sin -1 2 x 1 - x 2 (b) g( x ) = sin -1 ç ÷ (c) (d)
è 1 + 4x ø x 2y
æ 1 - x2 ö dy
(c) h( x ) = sin -1 ç ÷ (d) k( x ) = sin -1 (cos x ) 64. If y = tan x tan 2x tan 3x , then has the value equal to
è1 + x2 ø dx
(a) 3 sec 2 3 x tan x tan 2 x + sec 2 x tan 2 x tan 3 x
x -2 x -1
60. Let f ( x ) = × x , then + 2 sec 2 2 x tan 3 x tan x
x -1 -1 (b) 2y ( cosec 2 x + 2 cosec 4 x + 3 cosec 6 x )
(a) f ¢(10 ) = 1 (b) f ¢(3/2 ) = - 1 (c) 3 sec 2 3 x - 2 sec 2 2 x - sec 2 x
(c) domain of f ( x ) is x ³ 1 (d) None of these (d) sec 2 x + 2 sec 2 2 x + 3 sec 2 3 x

Differentiation Exercise 3 :
Statements I and II Type Questions
n
Directions (Q. Nos. 65 to 74) This section is based on Statement I and Statement II. Select the correct answer from the
codes given below.
(a) Both Statement I and Statement II are correct and Statement II is the correct explanation of Statement I
(b) Both Statement I and Statement II are correct but Statement II is not the correct explanation of Statement I
(c) Statement I is correct but Statement II is incorrect
(d) Statement II is correct but Statement I is incorrect
65. Consider f ( x ) =
x 67. Statement I If f (0) = a, f ¢ (0) = b, g(0) = 0, ( fog )¢ (0) = c ,
x -1 2
c
then g ¢ (0) = .
Statement I Graph of f ( x ) is concave up for x > 1. b
Statement II If f ( x ) is concave up, then f ¢¢ ( x ) > 0 . Statement II ( f ( g ( x ))¢ = f ¢ ( g ( x )) × g ¢ ( x ), for all n.
æ 2x ö 68. Let f and g be real valued functions defined on interval
66. If f ( x ) = sin -1 ç ÷ , then ( -1 , 1) such that g ¢¢( x ) is continuous, g(0) = 0, g ¢ (0) = 0,
è1 + x 2 ø
2 g ¢¢ (0) = 0 and f ( x ) = g ( x ) sin x .
Statement I The value of f ¢ (2) = - . Statement I lim ( g ( x ) cot x - g (0) cosec x ) = f ¢¢(0).
5 x ®0
æ 2x ö -2
Statement II f ( x ) = sin -1 ç ÷= , for x > 1. Statement II f ¢ (0) = g ¢ (0).
è1 + x ø 1 + x 2
2
Chap 02 Differentiation 75

dy 3 d ([x ]) ì 0, xÎ/ Integer


69. Statement I If y = sin -1 (3x - 4 x 3 ), then = Statement II =í
dx 1- x2 dx îdoes not exist , x Î Integer.
-1 1 72. Statement I If f ( x ) is a continuous function defined
only when £x< .
2 2 from R to Q and f (5) = 3, then differential coefficient of
-1
Statement II sin (3x - 4 x ) 3 f ( x ) w.r.t. x will be 0.
ì -1 1 Statement II Differentiation of constant function is
ï - p - 3 sin x , - 1 £ x £ - 2 always zero.
ïï 1 1 æ 2x ö
= í 3 sin -1 x , - £x£ 73. Statement I Derivative of sin -1 ç ÷ w.r.t.
ï 2 2 è1 + x ø 2
ï p - 3 sin x ,
- 1
1
£ x £ 1. æ1 - x 2 ö
ïî 2 cos -1 ç ÷ is 1 for 0 < x < 1.
æ1 - x 2 ö è1 + x 2 ø
70. If y = cos -1 ç ÷ , then
è1 + x 2 ø æ 2x ö æ1 - x 2 ö
Statement II sin -1 ç ÷ = cos -1 ç ÷
dy 2 è1 + x 2 ø è1 + x 2 ø
Statement I = for x Î R .
dx 1 + x 2 for - 1 £ x £ 1.
æ 1 - x 2 ö ì 2 tan -1 x , x ³ 0 74. Consider function f ( x ) satisfies the relation
Statement II cos -1 ç ÷ =í
è 1 + x 2 ø î - 2 tan -1 x , x < 0. f ( x + y 3 ) = f ( x ) + f (y 3 ), " x , y Î R and differentiable
71. If y = x + [x ] , then for all x.
dy Statement I If f ¢ (2) = a, then f ¢ ( - 2) = a.
Statement I = 1 for all x Î R.
dx Statement II f ( x ) is an odd function.

Differentiation Exercise 4 :
Passage Based Questions
Passage I (Q. Nos. 75 to 77) Passage II (Q. Nos. 78 to 80)
f (x + y ) - f (x ) f ( y ) - 1 Left hand derivative and right hand derivative of a
Let = + xy, for all x, y Î R, f ( x ) is
2 2 function f ( x ) at a point x = a are defined as
differentiable and f ¢ ( 0) = 1. Let g ( x ) be a derivable function at f ( a ) - f ( a - h)
x = 0 and follows the functional rule f ¢ ( a - ) = lim
h ® 0+ h
æ x + y ö g (x ) + g ( y )
gç ÷= ; k Î R , k ¹ 0, 2
f ( a + h) - f ( a )
è k ø k = lim
+
and g ¢ ( 0) - lg ¢ ( 0) ¹ 0. h ®0 h
75. Domain of log( f ( x )), is f ( a + h) - f ( a )
and f ¢ ( a + ) = lim
+ +
(a) R (b) R - { 0 } h ®0 h
-
(c) R (d) R f ( a ) - f ( a + h)
= lim
76. Range of y = log 3 / 4 ( f ( x )) is h ®0 + h
(a) ( -¥, 1 ] (b) [3 / 4, ¥ ) f (a ) - f (x )
(c) ( -¥, ¥ ) (d) R = lim respectively.
h ®0 + a-x
77. If the graphs of y = f ( x ) and y = g ( x ) intersect in
Let f be a twice differentiable function. We also know
coincident points then l can take values
that derivative of an even function is odd function and
(a) -3 (b) 1
(c) -1 (d) 4
derivative of an odd function is even function.
76 Textbook of Differential Calculus

78. If f is odd, then which of the following is left hand 84. If u = f ( x ), v = g ( x ), then the value of D * æç ö÷ is
u
derivative of f at x = - a? èv ø
f (a - h ) - f (a )
(a) lim u (D v ) - v (D u )
2 * 2 *
u ( D v ) - v ( D *u )
*
h ® 0- -h (a) (b)
v4 v2
f (h - a ) - f (a )
(b) lim v ( D u ) - u 2( D *v )
2 *
v ( D u ) - u ( D *v )
*
h ® 0- h (c) (d)
f (a ) + f (a - h ) v4 v2
(c) lim
h ® 0+ -h Passage V
f (a ) - f (a - h ) (Q. Nos. 85 to 87)
(d) lim
h ® 0- -h
A curve is represented parametrically by the equations
79. If f is even function, then which of the following is right x = e t cos t and y = e t sin t, where t is a parameter. Then,
hand derivative of f ¢ at x = a?
85. The relation between the parameter ‘t’ and the angle a
f ¢(a ) + f ¢( -a + h )
(a) lim between the tangent to the given curve and the X -axis is
h ® 0- h
given by, ‘t’ equals
f ¢(a ) + f ¢( -a - h )
(b) lim p p
h ® 0+ h (a) -a (b) +a
2 4
- f ¢( - a ) + f ¢( -a - h )
(c) lim p p
h ® 0- -h (c) a - (d) -a
4 4
f ¢(a ) + f ¢( -a + h )
(d) lim
-h
d 2y
h ® 0+ 86. The value of at the point, where t = 0, is
f ( - x ) - f ( -x - h ) dx 2
80. The statement lim (a) 1 (b) 2
h ®0 h
f (x ) - f (x - h) (c) –2 (d) 3
= lim implies that for all x Î R,
h ®0 -h
p
(a) f is odd 87. If F (t ) = ò ( x + y ) dt , then the value of F æç ö÷ - F (0) is
è ø 2
(b) f is even
(c) f is neither even nor odd (a) 1 (b) –1
(d) Nothing can be concluded (c) e p/2 (d) 0

Passage III Passage VI


(Q. Nos. 81 to 82) (Q. Nos. 88 to 90)

If f ( x ) = sin -1 (3x - 4x 3 ). Then answer the following Equation x - 1 = 0, n > 1, n Î N has roots 1, a1 , a 2 , …, a n .
n

88. The value of (1 - a1 ) (1 - a 2 ) . . . (1 - an ), is


81. The value of f ¢ (0) is
n2
(a) - 3 (b) 3 (a) (b) n
(c) 2 (d) - 2 2
(c) ( -1 )n n (d) None of these
æ 1 ö
82. The value of f ¢ ç ÷ is n
1
è 2ø 89. The value of å2 - a , is
(a) - 3 (b) 3 r =1 r
(c) -3 2 (d) 3 2 2n -1(n - 2 ) + 1 2n (n - 2 ) + 1
(a) (b)
2n - 1 2n - 1
Passage IV (Q. Nos. 83 to 84) 2n -1(n - 1 ) - 1
(c) (d) None of these
Let the derivative of f ( x ) be defined as 2n - 1
f 2 ( x + h) - f 2 ( x ) n
1
D * f ( x ) = lim
h ®0 h
, where f 2 ( x ) = { f ( x )}2 . 90. The value of å1 - a , is
r =1 r

83. If u = f ( x ), v = g ( x ), then the value of D * (u × v ) is n n(n - 1 )


(a) (b)
4 2
(a) ( D *u )v + ( D *v ) u (b) u 2 ( D *v ) + v 2( D *u ) n -1
(c) (d) None of these
(c) D u + D v
* *
(d) uv D * (u + v ) 2
Chap 02 Differentiation 77

Differentiation Exercise 5 :
Matching Type Questions
91. Match the entries between following two columns. 92. Match the following.
Column I Column II Column I Column II

(A) y = f (x ) be given by x = t 5 - 5t 3 - 20t + 7 (p) 0 æ 2x ö


(A) y = sin -1 ç ÷ , then (p) for x < 0
and y = 4 t 3 - 3t 2 - 18t + 3, then è 1 + x2 ø
dy dy 2
-5 ´ at t = 1 =-
dx dx 1 + x2
(B) P (x ) be a polynomial of degree 4 with (q) –2
æ 1 ö÷
P (2) = - 1, P ¢ (2) = 0, P¢¢ (2) = 2, y = cos-1 ç
(B)
ç 1 + x2 ÷
, then (q) for x > 1
P¢¢¢ (2) = - 12 and P¢¢¢¢ (2) = 24 , then P¢¢(3) è ø
is equal to
dy 1
=-
dy dx 1 + x2
1 1 + y4
(C) y= , then (r) 2 dy (r) for x < - 1
x dx (C) y = | e| x | - e | , then >0
dx
1 + x4
(D) u = log | 2x | , v = | tan -1 x | , then (s) for
æ 2x + 3 y ö 2 f (x ) + 3 f ( y) -1 < x < 0
(D) fç ÷= and (s) –1 du
>2
è 5 ø 5 dv
f ¢ (0) = p and f (0) = q, then f ¢¢(0)

Differentiation Exercise 6 :
Single Integer Answer Type Questions
93. If f ( x ) = x 3 + x 2 f ¢ (1) + xf ¢ ¢ (2) + f ¢ ¢ ¢ (3) for all x Î R. ( x - a) 4 ( x - a) 3 1
Then, f (1) is .......... . 96. If f ( x ) = ( x - b ) 4
( x - b) 3
1 and
æ ö
2x + 2 (x - c ) 4
(x - c ) 3
1
94. Let f ( x ) = sin -1 ç ÷,
ç 4 x 2 + 8x + 13 ÷ ( x - a) 4
( x - a) 2
1
è ø
d (tan f ( x )) 1 f ¢ ( x ) = l ( x - b) 4 ( x - b) 2 1 , then value of l is …. .
then the value of , when x = , is ………… . (x - c ) 4
(x - c ) 2
1
d (tan -1 x ) 2

95. Let x , x 1 , x 2 , x 3 , x 4 , ¼ , x 8 be 9 real zeroes, of the 97. Let P( x ) be a polynomial of degree 4 such that
P (1) = P (3) = P (5) = P (7 ) = 0. If the real number x ¹ 1, 3, 5
polynomial P( x ) = x 10
+ ax + bx + c , where a, b, c Î R. If
2
p
is such that P( x ) = 0 can be expressed as x = , where p
p q
the value of Q( x 1 ) = , where p and q are coprime to
q and q are relatively prime, then ( p - 8q ) is ................ .
each other. If Q( x ) = ( x - x 2 ) ( x - x 3 ) ¼ ( x - x 8 ) and 1 1
98. If x 2 + y 2 = t - and x 4 + y 4 = t 2 + 2 ,
1 t t
x 1 = , then the value of q - p is ................. . æ dy ö
2 then ç ÷ is ……… .
è dx ø (1, 1)
78 Textbook of Differential Calculus

99. If x 2 + y 2 + z 2 - 2xyz = 1, then the value of 103. Suppose, A =


dy
of x 2 + y 2 = 4 at ( 2, 2 ), B =
dy
of
dx dy dz dx dx
+ + is equal to ……… . dy
sin y + sin x = sin x × sin y at ( p, p ) and C = of
1- x 2
1-y 2
1-z2 dx
xy + 1
2e + e e - e - e = e
xy x y x y
at (1, 1), then
100. If y is twice differentiable function of x, then the ( A - B - C ) has the value equal to ……… .
d 2y
dy 104. A function is represented parametrically by the
expression (1 - x 2 ) × -x
+ y by means of the 1+t
3
2
dx 3 2 dy æ dy ö
dx equations x = 3 ; y = 2 + , then - x ×ç ÷
d 2y t 2t t dx è dx ø
transformation x = sin t in terms of t is 2 + ly. has the absolute value equal to ……… .
dt
Thus, l is ……… . 105. Suppose, the function f ( x ) - f (2x ) has the derivative 5
æ 1 ö at x = 1 and derivative 7 at x = 2. The derivative of the
101. The derivative of f ( x ) = cos -1 ç (2 cos x - 3 sin x )÷
è 3 ø function f ( x ) - f ( 4 x ) at x = 1, has the value 10 + l, then
the value of l is equal to ……… .
ì æ 1 öü
+ ísin -1 ç (2 cos x + 3 sin x )÷ý
î è 3 øþ 106. If x + y = 3e 2 , then D( x y ) vanishes when x equals to
1
w.r.t. 1 + x at x =
2
is ……… . le 2 , then the value of l is equal to ……… .
3
107. Let h( x ) be differentiable for all x and let
102. Suppose f ( x ) = e ax + e bx , where a ¹ b and
f ( x ) = (kx + e x )h( x ), where k is some constant.
f ¢ ¢ ( x ) - 2 f ¢ ( x ) - 15 f ( x ) = 0 for all x. Then, the value of If h(0) = 5, h¢ (0) = -2 and f ¢ (0) = 18, then the value of k is
a + b is equal to ……… . equal to ……… .

Complex Number Exercise 7 :


Questions Asked in Previous 10 Years’ Exams
(i) JEE Advanced & IIT-JEE
108. For x Î R, f ( x ) = | log 2 - sin x | and g ( x ) = f ( f ( x )), then [One Correct Option, 2016 Adv.]
(a) g is not differentiable at x = 0 (b) g ¢( 0 ) = cos (log 2 )
(c) g ¢( 0 ) = - cos (log 2 ) (d) g is differentiable at x = 0 and g ¢( 0 ) = - sin (log 2 )
109. Let f : R ® R, g : R ® R and h : R ® R be differentiable functions such that f ( x ) = x 3 + 3x + 2, g ( f ( x )) = x and
h ( g ( g ( x ))) = x for all x Î R. Then, [More than One Correct Option, 2016 Adv.]
1
(a) g ¢(2 ) = (b) h ¢(1 ) = 666 (c) h( 0 ) = 16 (d) h( g(3 )) = 36
15

(ii) JEE Main & AIEEE


æ 6x x ö
110. If for x Î æç0, ö÷ , the derivative of tan -1 ç
1
÷ is x × g ( x ), then g ( x ) equals
è 4ø 3
è1 - 9x ø [2017 JEE Main]
3 9 3x x 3x
(a) (b) (c) (d)
1 + 9x 3 1 + 9x 3 1 - 9x 3 1 - 9x 3

111. Let g ( x ) = log f ( x ), where f ( x ) is a twice differentiable positive function on (0, ¥ ) such that f ( x + 1) = x f ( x ). Then, for
æ 1ö æ1ö
N = 1, 2, 3,..., g ¢¢ ç N + ÷ - g ¢¢ ç ÷ is equal to
è 2 ø è2ø [2008 AIEEE]
ì 1 1 1 ü ì 1 1 1 ü
(a) - 4 í1 + + + ... + 2ý
(b) 4 í1 + + + ... + 2ý
î 9 25 (2 N - 1 ) þ î 9 25 (2 N - 1 ) þ
ì 1 1 1 ü ì 1 1 1 ü
(c) - 4 í1 + + + ... + 2ý
(d) 4 í1 + + + ... + 2ý
î 9 25 (2 N + 1 ) þ î 9 25 (2 N + 1 ) þ
Chap 02 Differentiation 79

d 2x 114. If y is a function of x and log ( x + y ) = 2xy, then the


112. equals
dy 2 [2007 AIEEE] value of y ¢ (0) is [2004 AIEEE]
-1 -1 -3 (a) 1
æd y ö 2 æ d y ö æ dy ö
2
(a) ç 2 ÷ (b) - ç 2 ÷ ç ÷ (b) -1
è dx ø è dx ø è dx ø
(c) 2
æ d 2y ö æ dy ö - 2 æ d 2y ö æ dy ö - 3 (d) 0
(c) ç 2 ÷ ç ÷ (d) - ç 2 ÷ ç ÷
è dx ø è dx ø è dx ø è dx ø
115. If x 2 + y 2 = 1 , then [2000 AIEEE]
113. If f ¢¢ ( x ) = - f ( x ), where f ( x ) is a continuous double (a) yy ¢¢ - 2 (y ¢ ) 2 + 1 = 0
differentiable function and g ( x ) = f ¢ ( x ). (b) yy ¢¢ + (y ¢ ) 2 + 1 = 0
2 2
ì æ x öü ì æ x öü (c) yy ¢¢ + (y ¢ ) 2 - 1 = 0
If F ( x ) = í f ç ÷ý + í g ç ÷ý and F(5) = 5, then
è 2 øþ è 2 øþ
F(10) is î î [2006 AIEEE] (d) yy ¢¢ + 2 (y ¢ ) 2 + 1 = 0
(a) 0 (b) 5 (c) 10 (d) 25

Answers
Exercise for Session 1 (sin 3 x) æç ex + ö÷ - 3 (ex + log x) cos 3x
1
a-1 tan x è xø
1. a log a + ax
x
2. sec2 x 11.
| tan x | sin 2 3x
1 3
3. + + 2 sec2 x 12.
m
cos (m sin -1 x)
x loge 3 x 1 - x2
x
4. + na0xn - 1 + (n - 1)a1xn - 2 + (n - 2)a2xn - 3 + . . . + an - 1 1
2 log a × sin -1 x -1
x) 2
|x| 13. × a(sin
5. 0 1- x 2

ì 1 ü
6. ex xn - 1 ín log a x + + x loga xý cos-1 1 - x2 1 sin -1 x
14. e × 15.
î log e a þ (1 - x2 )3/ 2
1 - x2
2x ì cot x ü
ílog 2 cot x - cosec x -
2
7. ý 1 loge 10
x î 2x þ 16. -
x loge 10 x(loge x)2
x2
8. 9. y
(x sin x + cos x)2
2
17. -2x {5 3 - x × log e 5 + 5 (3 - x2 )4}
10. x Î (- ¥, - 1) È (1, ¥)
-2a2 ìï a2 üï
Exercise for Session 2 18. í1 + ý
x3 ïî a - x ïþ
4 4

1. 4 (x2 + x + 1)3 × (2x + 1)


x
1 19.
2. × (2x + 1)
2 x2 + x + 1 4 4+ 4+ 4+ x 2
4+ 4 + x2 4 + x2
x
3. 3 sin 2 x cos x 4. 20. (c) 21. (b) 22. (a) 23. (a)
(a2 - x2 )3/ 2 24. (a) 25. (c) 26. (a) 27. (d)
b2 - a2
5. ex sin x
(x cos x + sin x) 6. -
b + a cos x Exercise for Session 3
x 1 6. (b) 7. (a) 8. (a) 9. (a) 10. (a)
7. ee × ex 8.
a2 + x2
Exercise for Session 4
2ab cos x
9.
a2 - b2 sin 2 x 1. (b) 2. (c) 3. (b) 4. (c)
10. sec x
80 Textbook of Differential Calculus

5. (i) y = sin -1 ç
æ 2x ö (vi) y = cos-1 (4x3 - 3x)
÷
è 1 + x2 ø Y
Y

π/2 π
y= y=π
–1 x
–π

y=
2 )
ta n ta π/2

3c
n –1
2 (0,

os
y= x
–1 x y=π/2
X′ y=

–1x
X s
– π2 –1 0 1 co

3
tan –1

2x–
x
X′ X
–π/2 –1 –1/2 0 1/2 1
Y′
æ 1 - x2 ö Y′
-1
(ii) y = cos çç 2÷
÷
è1 + x ø Exercise for Session 5
Y
3 2
π 6. 7.
2 19
y=
–2 –1 x
tan –
1 2 tan Exercise for Session 6
x y=
X′ X 1. xx (1 + log x)
0
Y′ x æ log x + 1 ö
2. x ç ÷
è2 x xø
(iii) y = tan -1 æç
2x ö
÷ × xx × ìí(1 + log x) × log x + üý
x 1
è 1 - x2 ø 3. xx
Y î xþ
2
4. x × xx × (2 log x + 1)
1
x+ ì æ 2 x + 1ö ü
–1 x –1 x
2 ta
n
2 ta
n 5. x íç 2
÷ + log xý
y =π
– y= î è 2x ø þ
X′ X 6. (cos x)x (log cos x - x tan x)
0
–1 1 n– x
1
ta æ
–2 cos2 x ö
y=π 7. (sin x)cos x çç - sin x log sin x + ÷
è sin x ÷ø
-1 æ log (sin x) ö÷

Y′ 8. (sin x)cos cos-1 x cot x -
ç 1 - x2 ÷ø
æ 3x - x3 ö è
(iv) y = tan -1 çç 2÷
÷
è 1 - 3x ø 9. - xx (sin xx ) × { 1 + log x}
Y 1
10. {xx (1 + log x) - 2 cosec2 x cot x}
xx + cosec2 x
π/2 y= 2 tan–1x
11. (sin x)tan × {sec2 x (log sin x) + 1}
x

+ (cos x) × {sec x tan x × log (cos x) - sec x tan x}


sec x

–1/√3 1/√3
X′ X æ yx y - 1 + yx log y ö
–√3 0 √3 13. - çç ÷
x-1 ÷
è x log x + xy ø
y

log sin y + y tan x


–π/2 14.
log cos x - x cot y
Y′ a
19.
(v) y = sin -1 (3x - 4x3 ) a2 + x2
Y
Exercise for Session 7
π/2
1. (i) 1 (ii) – 1 (iii) – 1
x
n –1
y=

y=

-1 æ ö
i
–3s 1/2 1 - x2
–π

y=π 2. xsin x ç log x + sin -1 x÷


π–

ç ÷
3sin
–3

X′ X
–1 –√3 –1/2 0 √3/2 è x ø
sin

–1 x

— 2
2 3. - 4. - 2 cos x × cot x cosec x
–1 x

ax
–π/2 1 1 1 1
5. - 6. xx 7. (i) - (ii) (iii)
Y′ 2 x x x
Chap 02 Differentiation 81

Exercise for Session 8 29. (c) 30. (c) 31. (d) 32. (b)
33. (a) 34. (d) 35. (d) 36. (c) 37. (c)
1 1
6. - 7. sec4 q cosec q 38. (a) 39. (d) 40. (a) 41. (b) 42. (b)
2at 3 3a 43. (b) 44. (b) 45. (d) 46. (d) 47. (c)
48. (c) 49. (d) 50. (d) 51. (d) 52. (d)
Exercise for Session 9 53. (b, c) 54. (b, d) 55. (a, c) 56. (a, c) 57. (a, d)
58. (b, d) 59. (b, c, d) 60. (a, b) 61. (a, b, c, d)
3
3. 62. (a, c) 63. (a, b) 64. (a, b, c) 65. (a)
p p2 - 3 66. (a) 67. (a) 68. (b) 69. (a)
70. (d) 71. (d) 72. (a) 73. (c)
Exercise for Session 10 74. (a) 75. (c) 76. (a) 77. (c)
78. (a) 79. (a) 80. (b) 81. (b)
1 f ¢¢(x)
1. 2. - 3. (b) 4. (c) 82. (c) 83. (b) 84. (c) 85. (c)
3 ( f ¢ (x))3 86. (b) 87. (c) 88. (b) 89. (a)
90. (c)
Chapter Exercises 91. (A) ® (q), (B) ® (r), (C) ® (s), (D) ® (p)
1. (b) 2. (c) 3. (c) 4. (d) 5. (c) 92. (A) ® (q, r), (B) ® (p), (C) ® (q, s), (D) ® (q, r)
6. (a) 7. (d) 8. (d) 9. (b) 93. (4) 94. (1) 95. (1) 96. (3) 97. (1) 98. (1)
10. (b) 11. (c) 12. (b) 13. (b) 14. (c) 99. (0) 100. (1) 101. (2) 102. (2) 103. (1) 104. (-1)
15. (a) 16. (c) 17. (c) 18. (b) 19. (c) 105. (9) 106. (1) 107. (3) 108. (b) 109. (b, c) 110. (c)
20. (a) 21. (b) 22. (a) 23. (c) 24. (b) 111. (a) 112. (d) 113. (b) 114. (a) 115. (b)
25. (d) 26. (a) 27. (b) 28. (a)
æ x 2 -y 2 ö
cos-1 ç 2 ÷ = log a

Solutions
5. Here,
è x +y 2 ø
æ 1 - (y / x ) 2 ö
Þ cos-1 ç ÷ = log a
è 1 + (y / x ) ø
y
Put = tan q
x
sec x - tan x sec x - tan x æ 1 - tan 2 q ö
1. Here, y = × Þ cos-1 ç
sec x + tan x sec x - tan x ÷ = log a
è 1 + tan 2 q ø
\ y = (sec x - tan x ) 2
Þ cos-1(cos2q) = log a
dy
Þ = 2(sec x - tan x ) × (sec x tan x - sec 2 x ) Þ 2q = loge a
dx
ö
-1 æ y
= -2 sec x (sec x - tan x ) 2 Þ 2 tan ç ÷ = log a
èxø
1+ x + x
2
(1 + x ) - x
4 2 2 2
æ log a ö
2. Here, y = = Þ
y
= tan ç ÷
1+ x + x 2
(1 + x + x 2 ) x è 2 ø
(1 + x + x 2 )(1 - x + x 2 ) On differentiating both sides, we get
= = (1 - x + x 2 )
(1 + x + x 2 ) dy
x - y ×1
dy dx =0
\ = 2 x - 1 = ax + b x2
dx
dy
Þ a = 2, b = -1 Þ x -y = 0
d 1 dx
3. y = ( x log x ) = x × + log x ×1 dy y
dx x Þ =
dx x
Þ y = 1 + loge x
sin x -p
\y = 0, when loge x = -1 Þ x =
1 6. Here, f ( x ) = (| x| ) as x ®
e 4
Thus, y = 1 + loge x can be shown as Þ f ( x ) = ( - x ) - sin x
Taking logarithm on both sides, we get
Y
log( f ( x )) = ( - sin x ) log( - x )
On differentiating both sides, we get
X 1 1
0 1/e × f ¢( x ) = ( - sin x ) + log( - x ) ×( - cos x )
f (x ) x
p
At x = - ,
3 x +1 5 x+4 4
4. Here, f ¢( x ) = 1 + = and g ¢( x ) = 1 + = é 1 ù
x -2 x -2 x -1 x -1 -
æ pö æ pö ê 2 æ p ö æ 1 öú
Since, f ¢( x ) < g ¢( x ) f ¢ç - ÷ = f ç - ÷ ê + log ç ÷ × ç - ÷ ú
è 4ø è 4ø p è 4ø è 2ø
x +1 x + 4 ê ú
Þ < ë 4 û
x -2 x -1 1
x +1 x + 4 æpö é 2
2 4 2 2ù
or - <0 =ç ÷ ê log - ú
x -2 x -1 è 4ø ë 2 p p û
(x 2 -1) -(x 2 + 2x -8) x x(b + y )
Þ <0 7. Here, y = =
( x - 1 )( x - 2 ) a+
x ab + ay + x
-2x + 7 2x -7 b +y
Þ <0 Þ >0
( x - 1 )( x - 2 ) ( x - 1 )( x - 2 ) Þ aby + ay 2 + xy = bx + xy
− + − + Þ aby + ay 2 = bx
−∞ ∞
1 2 7/2 On differentiating w.r.t. x, we get
\ x Î(1, 2 ) È(7 / 2, ¥ ), since log( x -2 ) exists, when x >2. dy dy
ab + 2ay =b
æ7 ö dx dx
Þ x Î ç , ¥÷ dy b
è2 ø \ =
dx a (b + 2y )
Chap 02 Differentiation 83

8. Taking logarithm on both sides, we get Þ w ¢(1 ) = g ¢( g(1 )) × g ¢(1 ) = g ¢(3 ) × g ¢(1 )
log y = x 2 log x 3 9
= ×( - 3 ) = -
On differentiating both sides, we get 4 4
3 9 6 3
1 dy 1 \ u ¢(1 ) + w ¢(1 ) = - = - = -
× = x 2 × + 2 x ×(log x ) 4 4 4 2
y dx x
dy 2
14. Let h( x ) = f 2( x ) + g 2( x )
Þ = x x ×( x + 2 x × log x ) On differentiating both sides, we get
dx
2
+1 2
+1 h ¢( x ) = 2 f ( x ) × f ¢( x ) + 2 g ( x ) × g ¢( x )
= xx ×(1 + 2 log x ) = x x ×(log(e x 2 ))
= - 2 g ¢( x ) × f ¢( x ) + 2 f ¢( x ) × g ¢( x ) = 0
9. Here, x 1 + y = - y 1 + x \ h( x ) is constant.
On squaring both sides, we get Thus, h(10 ) = f 2(10 ) + g 2(10 ) = h (5 )
x 2 (1 + y ) = y 2(1 + x ) \ f 2(10 ) + g 2(10 ) = f 2(5 ) + g 2(5 ) = f 2(5 ) + ( f ¢(5 )) 2 = 4 + 4 = 8
Þ x + x y = y +y x
2 2 2 2
15. Here, f ( 0) = 0
Þ ( x - y )( x + y ) + xy ( x - y ) = 0 f ¢( 0 ) = 1
Þ ( x - y )( x + y + xy ) = 0 Þ x + y + xy = 0 f ¢¢( 0 ) = 2
-x f ¢¢¢( 0 ) = 3
Þ y =
(1 + x ) M M
dy é (1 + x ) ×1 - x ×1 ù dy -1 f n (0) = n
\ =-ê ú Þ =
dx ë (1 + x ) 2
û dx (1 + x )2 \ f ( 0 ) + f ¢( 0 ) + f ¢¢ ( 0 ) + f ¢¢¢( 0 ) + ¼+ f n ( 0 ) = 1 + 2 + 3 + ¼+ n
10. Here, x 2e y + 2xye x + 13 = 0 n (n + 1 )
=
2
On differentiating both sides, we get dy
dy é dy ù 16. Here, = f ¢( f ( f ( x ))) × f ¢( f ( x )) × f ¢( x )
x 2 ×e y × + 2 x ×e y + 2 ê xe x × + ye x + yx ×e x ú = 0 dx
dx ë dx û \ y ¢( 0 ) = f ¢( f ( f ( 0 ))) × f ¢( f ( 0 )) × f ¢( 0 )
dy = f ¢( f ( 0 )) × f ¢( 0 ) ×2 = f ¢( 0 ) × f ¢( 0 ) × 2 = 2 ´ 2 ´ 2 = 8
Þ × x ( xe + 2e ) + 2 ( xe + ye + xye ) = 0
y x y x x
dx
17. Here, y 2 = p( x )
dy 2 [ xe y + ye x ( x + 1 )] Differentiating both sides, we get 2y ×y ¢= p( x )
Þ =-
dx x ( xe y + 2e x ) Again, differentiating both sides, we get
dy é ( xe y - x + y ( x + 1 )) ù 2yy ¢¢+ 2(y ¢ ) 2 = p ¢¢( x )
or = -2 ê y -x ú
dx ë x ( xe + 2) û On multiplying by y 2, we get
y+ x
11. Here, x = e . Taking log on both sides, we get 2y 3 ×y ¢¢+ 2 (y ×y ¢ ) 2 = y 2p ¢¢( x )
log x = (y + x ) log e p ¢( x ) é p ¢( x ) ù
On differentiating w.r.t. x, we get Q y ×y ¢= êëQ y × y ¢= 2 úû
2
1 dy dy 1 1-x
= +1 Þ = -1 = ( p ¢( x )) 2
x dx dx x x Þ 2y 3 ×y ¢¢+ 2 × = y 2p ¢¢( x )
4
12. g( f ( x )) = x, as g is the inverse of f .
1
Þ g ¢( f ( x )) × f ¢( x ) = 1 Þ 2y 3 ×y ¢¢+ ×( p ¢( x )) 2 = p ¢¢( x ) ×y 2
2
1+ x2 Again, differentiating both sides, we get
Þ g ¢( f ( x )) = 10 , as g(2 ) = a
x d æ d 2y ö 1
Put x = a 2 × çy 3 × 2 ÷ + ×2 p ¢( x ) × p ¢¢( x ) = p ¢¢¢( x )y 2 + 2y ×y ¢ p ¢¢( x )
dx è dx ø 2
1+a2
Þ g ¢( f (a )) = 10 Þ f (a ) =2 d æ 3 d 2y ö
a Þ 2× çy × ÷ = p ¢¢¢( x ) × p( x ) + 2y ×y ¢×p ¢¢( x ) - p ¢( x ) × p ¢¢( x )
dx è dx 2 ø
1+a2 [Q2y ×y ¢= p ¢( x )]
\ g ¢(2 ) = 10
a = p ¢¢¢( x ) × p( x ) + p ¢( x ) × p ¢¢( x ) - p ¢( x ) × p ¢¢( x )
13. Here, u ¢( x ) = f ¢( g( x )) × g ¢( x ) = p ¢¢¢( x ) × p( x )
Þ u ¢(1 ) = f ¢( g(1 )) × g ¢(1 ) = f ¢(3 ) × g ¢(1 ) 18. As,
dy
= f ¢( x ) and
dx
= g ¢(y )
æ 1ö 3 dx dy
= ç - ÷ ×( - 3 ) =
è 4ø 4 1
Þ g ¢(y ) = …(i)
and w ¢( x ) = g ¢( g( x )) × g ¢( x ) f ¢( x )
84 Textbook of Differential Calculus

On differentiating w.r.t. y, we get æ sin q ö


22. Let f ( q) = sin a, where a = tan -1 ç ÷
d æ 1 ö d æ 1 ö dx è cos 2 q ø
g ¢¢(y ) = ç ÷= ç ÷×
dy è f ¢( x ) ø dx è f ¢( x ) ø dy sin q
f ¢¢( x ) dx Þ tan a =
=- × cos2 q
( f ¢( x )) 2 dy sin q
\ sin a = = tan q [Q q Î( - p / 4, p / 4 )]
f ¢¢( x ) 1 f ¢¢( x ) cos q
=- × =-
( f ¢( x )) f ¢( x )
2
( f ¢( x )) 3 d ( f ( q))
Þ f ( q) = tan q \ =1
f ¢¢( x ) d (tan q)
\ g ¢¢(y ) = -
( f ¢( x )) 3 log tan x
23. We have, y =
19. As discussed in Q.18, logsin x
d 2x d 2y / dx 2 æ sec 2 x ö
= - (logsin x ) ç ÷ - (log tan x )(cot x )
dy 2 æ dy ö
3
dy è tan x ø
ç ÷ Þ =
è dx ø dx (logsin x ) 2
d 2y d 2x æ dy ö
3 é dy ù -4
Þ = - 2 ×ç ÷ …(i) \ êë dx úû = [on simplification]
dx 2
dy è dx ø x = p/ 4 log 2
d 2y dy x
æ 1 ö x
é (r + 1 ) - r ù
Now,
dx 2
+y ×
dx
=0 24. We have, y = å tan-1 çè 1 + r + r 2 ÷ø = å tan-1 ê 1 + (r + 1) r ú
r =1 r =1 ë û
3
d 2x æ dy ö æ dy ö x
Þ - 2 ×ç ÷ + y ç ÷ = 0
dy è dx ø è dx ø = å[tan-1(r + 1) - tan-1 r ]
r =1
2
d 2x æ dx ö
Þ -y ç ÷ = 0 = [tan -1 2 - tan -1 1 + tan -1 3 - tan -1 2 + K + tan -1 x
dy 2 è dy ø
- tan -1( x - 1 ) + tan -1( x + 1 ) - tan -1 x ]
20. g( x + 1) = log( f ( x + 1)) = log x + log( f ( x ))
= [tan -1( x + 1 ) - tan -1 1 ]
Þ g( x + 1 ) - g( x ) = log x
dy 1
On differentiating both sides, we get \ =
dx 1 + ( x + 1 ) 2
1
g ¢( x + 1 ) - g ¢( x ) = æ sin a sin x ö
x 25. We have, y = sin -1 ç ÷
Again, differentiating both sides, we get è 1 - cos a sin x ø
1
g ¢¢( x + 1 ) - g ¢¢( x ) = - 2 …(i) dy 1 d æ sin a sin x ö
x Þ = × ç ÷
dx sin a sin x
2 2 dx è 1 - cos a sin x ø
æ 1ö æ1ö 1-
\ g ¢¢ ç1 + ÷ - g ¢¢ ç ÷ = - 4 (1 - cos a sin x ) 2
è 2ø è2ø
æ 1ö æ 1ö 4 dy (1 - cos a sin x ) sin a
g ¢¢ ç2 + ÷ - g ¢¢ç1 + ÷ = - Þ = ×
è 2ø è 2ø 9 dx 1 + cos2 a sin x - 2 cos a sin x cos a
2

æ 1ö æ 1ö -4
g ¢¢ ç3 + ÷ - g ¢¢ ç2 + ÷ = d æ 1 ö
è 2ø è 2 ø 25 ç- ÷ tan a (1 - cos a sin x )
dy dx è 1 - cos a sin x ø
M M M Þ =-
æ 1 ö æ 1 ö 4 dx { 1 + cos2 a sin 2 x - 2 sin x cos a }
g ¢¢ ç N + ÷ - g ¢¢ ç N - ÷ = -
è 2 ø è 2 ø (2 N - 1 ) 2 é - cos a cos x ù
ê 2ú
sin a cos x
On adding, we get
Þ
dy
= ë (1 - cos a sin x ) û
æ 1ö æ1ö æ 1 1 1 ö dx (1 - cos a sin x ) [1 + cos2 a sin 2 x - 2 cos a sin x ]
g ¢¢ ç N + ÷ - g ¢¢ç ÷ = - 4 ç1 + + + ¼+ ÷
è 2ø è2ø è 9 25 (2 N - 1 ) 2 ø
é dy ù
\ ê ú = sin a Þ y ¢( 0 ) = sin a
21. Here, f ( x ) = x 3 + e x /2 and g( x ) = f -1( x ) ë dx û x = 0
Þ g ¢( f ( x )) × f ¢( x ) = 1
æ x x - x -x ö
Putting f ( x ) = 1 Þ x 3 + e x /2 = 1 Þ x = 0 26. We have, f ( x ) = cot -1 ç ÷
1 è 2 ø
\ g ¢(1 ) × f ¢( 0 ) = 1, f ¢( x ) = 3 x 2 + e x /2
2 1 d æ x x - x -x ö
Þ f ¢( x ) = - × ç ÷
1 æ x x - x -x ö
2
dx è 2 ø
Þ g ¢(1 ) × = 1 Þ g ¢(1 ) = 2
2 1+ç ÷
è 2 ø
Chap 02 Differentiation 85

-2 d æ dx ö
2
æ dy ö
2
Þ f ¢( x ) = × (x x - x -x ) \ ç ÷ + ç ÷ = [ f ( t ) + f ¢¢( t )]
2
4 + ( x - x -x ) 2 dx
x
è dt ø è dt ø
-2 d
Þ f ¢( x ) = x -x 2
× (e x log x - e - x log x ) d 3y
( x + x ) dx 31. Note that in y highest degree of x is 4 and therefore is a
dx 3
-2 linear function of x, which is satisfied only in (d).
Þ f ¢( x ) =
(x x + x -x )2 [l 2 - m 2 + m 2 - n 2 + n 2 - l 2 ]
32. Exponent of x =
ì x log x d d ü (l - m )(m - n )(n - l )
íe × ( x log x ) - e - x log x × ( - x log x )ý
î dx dx þ Þ y = x0 = 1 Þ y = 1 Þ y ¢ = 0
-2
Þ f ¢( x ) = x { x x (1 + log x ) + x - x (1 + log x )} 33. y = ( A + Bx )emx + (m - 1) -2 × e x
(x + x -x )2
Þy × e -mx = ( A + Bx ) + (m - 1 ) -2 × e (1 - m ) x
-2(1 + log x ) x -2 (1 + log x )
Þ f ¢( x ) = x × (x + x -x ) =
(x + x -x )2 xx + x - x Þ e -mx × y1 - mye -mx = B - (m - 1 ) -1 × e -(m - 1) x
-2 Þ e -mx × y 2 - y1e -mx × m - m[e -mx × y1 - ye -mx × m ] = e - (m - 1) x
\ f ¢(1 ) = = -1
(1 + 1 ) Þ e -mx × y 2 - 2my1e -mx + m 2y × e -mx = e - (m - 1) x
27. f ( g( x )) = x Þ f ¢( g( x )) g ¢( x ) = 1 \ y 2 - 2my1 + m 2y = e x
Þ (eg (x )
+ 1 ) g ¢( x ) = 1 x3
34. We have, f ( x ) = - + x 2 sin 6 - x sin 4 × sin 8
Þ (e g ( f (log 2))
+ 1 ) g ¢( f (log 2 )) = 1 3
Þ + 1 ) g ¢( f (log 2 )) = 1
(e log 2 - 5 sin -1((a - 4 ) 2 + 1 ) (a = 4 )
1 Þf ¢( x ) = - x + 2 x sin 6 - sin 4 sin 8
2
Þ g ¢( f (log 2 )) =
3 Þ f ¢(sin 8 ) = - sin 2 8 + 2 sin 6 sin 8 - sin 4 sin 8
é æxö æxö æ x ö æ x öù = sin 8 [ - sin 8 + 2 sin 6 - sin 4 ]
28. F ¢( x ) = ê f ç ÷ × f ¢ ç ÷ + g ç ÷ g ¢ ç ÷ ú sin 8
ë è2ø è2ø è 2 ø è 2 øû = - sin 8 [sin 8 + sin 4 - 2 sin 6 ]
Here, g( x ) = f ¢( x ) and g ¢( x ) = f ¢¢( x ) = - f ( x ) = - sin 8 [2 sin 6 cos 2 - 2 sin 6 ] sin 4 sin 6
æxö æxö æxö æxö = 2 sin 8 sin 6 [1 - cos 2 ] < 0
So, F ¢( x ) = f ç ÷ g ç ÷ - f ç ÷ g ç ÷ = 0
è2ø è2ø è2ø è2ø
35. f ( g( x )) = x Þ f ¢( g( x )) × g ¢( x ) = 1; f ¢( g(a )) × g ¢(a ) = 1; f ¢(b ) × 2 = 1
\ F ( x ) is a constant function. Hence, F (10 ) = 5 1
Þ f ¢(b ) =
29. We have, x = sec q - cos q and y = secn q - cosn q 2
2 3 3
Þ
dx
= sec q tan q + sin q = tan q (sec q + cos q) 36. Put cos f = ; sin f = ; tan f =
dq 13 13 2
dy y = cos-1 {cos ( x + f )} + sin -1 {cos ( x - f )}
and = n [secn q tan q + cosn - 1 qsin q] p
dq = cos-1 {cos ( x + f )} + - cos-1 {cos ( f - x )}
2
= n tan q (secn q + cosn q) p
=x+f+ -f+x
dy 2
dy dq n tan q (secn q + cosn q) p
\ = = y = 2x + ; z = 1 + x 2
dx dx tan q (sec q + cos q) 2
dq dy
dy dx 2 1 + x
2
2
2 (sec q - cos q) + 4 n 2(y 2 + 4 ) = =
n n 2
æ dy ö Now,
Þ ç ÷ =n × = dz dz x
è dx ø (sec q - cos q) + 4
2
(x 2 + 4)
dx
2
æ dy ö æ dy ö 10
Þ ( x 2 + 4 ) ç ÷ = n 2(y 2 + 4 ) \ ç ÷ =
è dx ø è dz ø x = 3 3
4
30. We have, x = f (t ) cost - f ¢(t )sin t
37. f ( x ) = x + 2 2x - 4 + x - 2 2x - 4
and y = f (t )sin t + f ¢(t ) cost
dx \ f (x ) = ( x - 2 + 2 )2 + ( x - 2 - 2 )2
Þ = - f ( t )sin t + f ¢( t ) cost - f ¢( t ) cost - f ¢¢( t )sin t
dt =| x -2 + 2 | + | x -2 - 2 |
= - [ f ( t ) + f ¢¢( t )]sin t
For x - 2 to exist, x ³ 2
dy
and = f ( t ) cost + f ¢( t )sin t - f ¢( t )sin t + f ¢¢( t ) cost
dt Also, x -2 + 2 > 0 [true]
= [ f ( t ) + f ¢¢( t )]cost But x - 2 - 2 ³ 0 only, if x ³ 4 < 0 only, if x < 4
86 Textbook of Differential Calculus

Now, f ( x ) becomes For 0 < x < 1, we have


ìï x - 2 + 2 - x - 2 + 2, for 2 £ x < 4 f ( x ) = | log | x || = - log x
f (x ) = í 1
ïî x - 2 + 2 + x - 2 - 2, for x ³ 4 f ¢( x ) = -
x
ìï2 2, for 2 £ x < 4 For -1 < x < 0, we have
\ f (x ) = í
ïî2 x - 2, for 4 £ x < ¥ f ( x ) = - log ( - x )
1
f ¢( x ) = -
Q f is continuous in [2, 4 ) È [ 4, ¥ ). x
ì 0, 2£x<4 ì 1
\
ï
f ¢( x ) = í 1 ï , |x | > 1
,4£x<¥ Hence, f ¢( x ) = í x
ï x -2 1
ï- , | x | < 1
î
î x
1 1
\ f ¢(102 + ) = = 42. We have, f ( x ) = cos { x + 3x + . . . + (2n - 1) x }
102 - 2 10
+ i sin { x + 3 x + 5 x + . . . + (2n - 1 ) x }
\ 10 f ¢ (102 + ) = 1
Þ f ( x ) = cos n x + i sin n 2x
2

-2 sin x
38. Here, y = 2 ln (1 + cos x ) Þ y1 = Þ f ¢( x ) = - n 2(sin n 2 x ) + n 2(i cos n 2 x )
1 + cos x
Þ f ¢¢( x ) = - n 4 cos n 2 x - n 4i sin n 2 x
é (1 + cos x ) cos x - sin x ( - sin x ) ù
y2 = -2 ê ú Þ f ¢¢( x ) = - n 4 (cos n 2 x + i sin n 2 x )
ë (1 + cos x ) 2 û
Þ f ¢¢( x ) = - n 4 f ( x )
é cos x + 1 ù -2
= -2 ê 2ú
= 43. We have, f ( x ) = xn
ë (1 + cos x ) û (1 + cos x )
Þ f ( x + y ) = ( x + y )n Þ f ¢( x + y ) = n( x + y )n - 1
- ln (1 + cos x ) 2
\ 2e - y /2 = 2 × e 2 =
2 Also, f ¢( x ) = nxn - 1 and f ¢(y ) = ny n - 1
(1 + cos x )
\ f ¢( x + y ) = f ¢( x ) + f ¢(y )
\ y2 +
2
=0 Þ n ( x + y )n - 1 = n × xn - 1 + n × y n - 1
e y /2
Þ ( x + y )n - 1 = xn - 1 + y n - 1 …(i)
(a + x ) + a - x × a + x
39. y = For n - 1 > 1, we find that LHS of Eq. (i) is greater than the
(a - x ) + a - x × a + x RHS. So, we must have n - 1 £ 1, i.e. n - 1 = 0 or n - 1 = 1.
(a + x ) ( a + x + a - x ) æ a + x ö
1/ 2 Hence, n = 1 or n = 2
y = =ç ÷
a - x ( a + x + a - x) èa - x ø 44. For 2 < x < 3, we have [ x ] = 2
æ 2p ö
dy 1 a - x æ (a - x ) + (a + x ) ö 1 a-x 2a \ f ( x ) = sin ç - x2÷
= ç ÷= ´ è 3 ø
dx 2 a+x è (a - x ) 2 ø 2 a + x (a - x ) 2
æ 2p ö
æ dy ö 1 Þ f ¢( x ) = - 2 x cos ç - x2÷
\ ç ÷ = è 3 ø
è dx ø x = 0 a
Þ f ¢( p / 3 ) = - 2 p / 3 cos p / 3 = - p / 3
40. We have, u( x ) = 7v( x )
45. We have, u = e x sin x
Þ u ¢( x ) = 7v ¢( x ) Þ p = 7
du
u( x ) æ u( x ) ö ¢ Þ = e x sin x + e x cos x = u + v
Again, =7 Þ ç ÷ =0 dx
v(x ) è v(x ) ø
and v = e x cos x
Þ q=0
dv
p+q 7+0 Þ = e x cos x + e x sin x = v - u
Now, = =1 dx
p -q 7 - 0
du dv
41. For x > 1, we have f ( x ) = | log | x || = log x \ v -u = v(u + v ) - u(v - u ) = u 2 + v 2
dx dx
1
Þ f ¢( x ) = d 2u du dv
x = + = u + v + v - u = 2v
For x < - 1, we have dx 2 dx dx
f ( x ) = | log | x || = log ( - x ) d 2v dv du
1 and = - = (v - u ) - (v + u ) = - 2u
Þ f ¢( x ) = dx 2 dx dx
x
Chap 02 Differentiation 87

ln {ln ( x )} æ pö æ pö
46. We have, f ( x ) = log x {ln ( x )} = \ ç LHD at x = ÷ ¹ ç RHD at x = ÷
ln ( x ) è 4ø è 4ø
ln ( x ) ×
1 1
× - ln {ln ( x )}
1 æpö
Thus, f ¢ ç ÷ doesn’t exist.
ln ( x ) x x 1 - ln {ln ( x )} è 4ø
\ f ¢( x ) = =
{ln ( x )} 2 x { ln ( x ) } 2
51. We have, f ( x ) = x 2 + xg ¢(1 ) + g ¢¢(2 )
1 - ln {ln (e )} 1 - ln (1 ) 1
Þ f ¢(e ) = = = or e -1 and g( x ) = x 2 + xf ¢(2 ) + f ¢¢(3 )
e {ln (e )} 2 e e
Þ f ¢( x ) = 2 x + g ¢(1 )
47. We have, [ f ( x )] n = f (nx ) for all x
and g ¢( x ) = 2 x + f ¢(2 ) …(i)
Þ n[ f ( x )] n - 1 f ¢( x ) = nf ¢(nx ) Putting x = 1 in Eq (i), we get
Þ n[ f ( x )] n f ¢( x ) = nf ( x ) f ¢(nx ) f ¢(1 ) = 2 + g ¢(1 )
[multiplying both sides by f ( x )] and g ¢(1 ) = 2 + f ¢(2 )
Þ nf (nx ) f ¢( x ) = nf ( x ) f ¢(nx ) [Q [ f ( x )]n = f (nx )] Þ f ¢(1 ) = 4 + f ¢(2 )
Þ f (nx ) f ¢( x ) = f ( x ) f ¢(nx ) Putting x = 2 in Eq. (i), we get

48. Since, f ( x ) is an odd differentiable function defined on R. f ¢(2 ) = 4 + g ¢(1 )


Therefore, f ( - x ) = - f ( x ) for all x Î R and g ¢(2 ) = 4 + f ¢(2 )
Differentiating both the sides w.r.t. x, we get Þ g ¢(2 ) = 4 + 4 + g ¢(1 ) = 8 + g ¢(1 )
- f ¢( - x ) = - f ¢( x ) for all x Î R Differentiating Eq. (i) w.r.t. x, we get
Þ f ¢( - x ) - f ¢( x ) = 0 for all x Î R f ¢¢( x ) = 2
Þ f ¢( -3 ) = f ¢(3 ) = - 2
and g ¢¢( x ) = 2 for all x
Aliter
We know that the derivative of a differentiable odd function is Þ f ¢¢(3 ) = 2
an even function. Therefore, f ¢( x ) is an even function. and g ¢¢(2 ) = 2
Hence, f ¢( -3 ) = f ¢(3 ) = - 2 Þ g ¢¢(2 ) + f ¢¢(3 ) = 2 + 2 = 4

52. We have, f (x ) = xn
49. We have, y = x+ y + x + y + ... ¥ f r ( x ) = n(n - 1 )(n - 2 ) . . . {n - (r - 1 )} xn - r

Þ y2 = x + y + x + y + ... ¥ n!
Þ f r (x ) = xn - r
(n - r )!
Þ y 2 = x + y + y Þ (y 2 - x ) 2 = 2y
n!
Differentiating both the sides w.r.t. x, we get Þ f r (1 ) =
(n - r )!
æ dy ö dy
2 (y 2 - x ) ç2y - 1÷ = 2 f ¢(1 ) f ¢¢(1 ) f ¢¢¢(1 ) ( -1 )n f n (1 )
è dx ø dx \ f (1 ) - + - + ... +
1! 2! 3! n!
dy y2 - x
Þ = 3 n
f r (1 )
dx 2y - 2 xy - 1 = å(-1)r r!
, where f 0(1 ) = f (1 )
r=0
50. We have, f ( x ) = cos x - sin x
n n
n!
ì p = å(-1)r (n - r )! r ! = r S= 0 (-1)r nCr = 0
ï cos x - sin x, for 0 < x £ x r=0
Þ f (x ) = í
p p
ïsin x - cos x, for < x < 53. We have, y + log(1 + x ) = 0
î 4 2
dy 1
pö ì d Þ + =0
æ ü dx (1 + x )
Clearly, ç LHD at x = ÷ = í (cos x - sin x )ý
è 4 ø î dx þ at x=
p
1
4 Þ y¢ = - …(i)
= ( - sin x - cos x ) =- 2 (x + 1)
p
x=
4 Also, log(1 + x ) = - y
æ pö ì d ü Þ 1 + x = e - y Þ 1 = e - y × ( -y ¢ )
and ç RHD at x = ÷ = í (sin x - cos x )ý
è 4 ø î dx þ at p
x=
4
Þ ey = - y ¢
= (cos x + sin x ) p = 2 or y ¢ + ey = 0
x=
4
88 Textbook of Differential Calculus

x
54. As, y = 2 3 On differentiating w.r.t. x, we get
dy dy
dy x æd ö 2y × =1 +
Þ = 2 3 × log 2 × ç (3 x ) ÷ dx dx
dx è dx ø
dy 1
x Þ =
= 2 3 × log 2 × 3 x × log 3 dx 2y - 1
x x
= 2 3 × 3 x × log 2 × log 3 …(i) Also, y = +1
y
3x
Also, y = 2 and log 2 y = 3 x
dy
y ×1 - x ×
\ Eq. (i) becomes, Þ
dy
= dx
dy dx y2
= y × (log 2 y ) × log 3 × log 2
dx dy
Þ (y 2 + x ) =y
55. Here, f ( x ) = x 2 + 3x - 3 dx
dy y
g( f ( x )) = x Þ = 2 [Qy 2 = x + y ]
dx y + x
Þ g ¢( f ( x )) × f ¢( x ) = 1, f ¢( x ) = 2 x + 3
dy y y
Þ g ¢( f ( x )) =
1 Þ = =
2x + 3 dx ( x + y ) + x 2 x + y
ln (ln x )
1 58. We have, y = x (ln x )
\ g ¢(1 ) = …(i)
2 g(1 ) + 3 Þ ln y = (ln x ) ln (ln x ) ln x …(i)
-1
and g (x ) = f (x ) Þ ln (ln y ) = ln (ln x ) × ln (ln x ) + ln (ln x )
When f ( x ) = 1, 1 1 dy 2 ln (ln x ) 1 1
Þ × × = × +
then g -1( x ) = 1 Þ x = g(1 ) ln y y dx ln x x x ln x
Also, f (x ) = 1 2 ln (ln x ) + 1
=
Þ 1 = x + 3x - 3
2 x ln x

Þ x + 3x - 4 = 0
2 dy y ln y
\ = × (2 ln (ln x ) + 1 )
dx x ln x
Þ ( x + 4 )( x - 1 ) = 0
Substituting the value of ln y from Eq. (i), we get
Þ x =1 [as x > 0]
dy y
1 1 = (ln x ) ln (ln x ) (2 ln (ln x ) + 1)
\ g ¢(1 ) = = dx x
2(1 ) + 3 5
59. (a) We have, y = sin –1 2x 1 – x 2
56. Here, x 3 - 2x 2y 2 + 5x + y - 5 = 0
On differentiating, we get dy 2 – 4x 2
Þ =
3 x 2 - 4 xy 2 - 4 x 2y dy / dx + 5 + dy / dx = 0 dx 1 – 4x 2 + 4x 4
3 x 2 - 4 xy 2 + 5 Clearly, it is not defined at x =
1
Þ y ¢= .
4 x 2y - 1 2
3-4+5 4 1
\ y ¢(1 ) = = Not derivable at x = , check with x = sin q or direct
4 -1 3 2
differentiation.
1 (1 + 4 x )(2 x + 1 ) ln 2 - 2 x + 1 × 4 x × ln 4
Also, y ¢¢( x ) (b) g ¢( x ) = ×
æ 2 × 2x ö
2 (1 + 4 x ) 2
[(6 x - 4y 2 - 8 xyy ¢)]( 4 x 2y - 1 ) - (8 xy + 4 x 2y ¢ )(3 x 2 - 4 xy 2 + 5 )] 1-ç ÷
=
( 4 x 2y - 1 ) 2 è 1 + 2 2x ø

æ 4ö æ 4ö 1 + 2 2x 2 x + 1 ln 2 (1 – 4 x )
ç6 - 4 - 8 × ÷ × ( 4 - 1 ) - ç8 + 4 × ÷ × (3 - 4 + 5 ) = ´
è 3ø è 3ø 1+4 2x
– 2 ×2 2x (1 + 4 x ) 2
Þ y ¢¢ (1 ) =
(4 - 1)2 2 x +1
ln 2(1 – 4 x )
22 =
= -8 1 – 2 2x (1 + 4 x )
27
57. Here, y = x + y Clearly, it is not defined, when 1 – 2 x = 0
i.e. at x = 0
Þ y2 = x +y
Þ Not derivable at x = 0
Chap 02 Differentiation 89

æ 1 - x2 ö p -1 æ 1 - x ö
2 Substituting this in Eq. (i), we get
(c) h( x ) = sin -1 ç ÷ = - cos ç ÷
è1 + x2 ø 2 è1 + x2 ø dy 2y
=- x
dx 2
ìp -1
ï 2 - 2 tan x, x ³ 0 62. f ( x ) = ( x 2 + bx + c )e x

p \ f ¢( x ) = { x 2 + (b + 2 ) x + (b + c )}e x
ï + 2 tan -1 x, x < 0
î2 f ( x ) > 0 iff D = b 2 - 4c < 0
ì -2
ï 1 + x2 , x>0 Now, f ¢( x ) > 0 iff D ¢ = (b + 2 ) 2 - 4(b + c )
ï =D+4<0
\ h ¢( x ) = í not derivable, x = 0
ï 2 Thus, for f ¢( x ) > 0, D + 4 < 0 holds.
ï 1 + x2 , x<0
î Þ D<0
\ Not differentiable at x = 0. Þ f (x ) > 0
p 63. We have, y + x + y – x = c
(d) k( x ) = sin -1 (cos x ) = - cos-1(cos x )
2 On differentiating w.r.t. x, we get
and graph for cos-1 (cos x ), is shown as dy æ 1 1 ö 1 1
ç + ÷= –
Y dx è y + x y -xø y–x y +x

π (π, π) dy y +x– y–x



Þ =
–x x –x
dx ( y + x + y – x )
dy (y + x ) – (y – x ) 2x
X = = 2
− 2π −π 0 π 2π dx ( y + x + y – x ) 2
c
dy x
\ =
\ k( x ) is not differentiable at x = 0. dx y + y 2 - x 2

( x - 1 )2 + 1 - 2 x - 1 dy ( y + x – y – x ) y - y 2 - x2
2
60. f ( x ) = ×x Now, = =
x -1 -1 dx (y + x ) - (y – x ) x

| x -1 -1| ì - x , if x Î [1, 2 ) 64. y = tan x × tan 2x × tan 3x …(i)


= ×x =í Differentiating both sides, we get
x -1 -1 î x , if x Î (2, ¥ ) dy
ì–1, if x Î [1, 2 ) = 3 sec 2 3 x × tan x × tan 2 x + sec 2 x × tan 2 x × tan 3 x
Þ f ¢( x ) = í dx
î 1, if x Î (2, ¥ ) + 2 sec 2 2 x × tan x × tan 3 x …(ii)
¢ æ3ö Taking log on both sides of Eq. (i), we get
\ f ¢(10 ) = 1 and f ç ÷ = – 1
è2ø log y = log tan x + log tan 2 x + log tan 3 x
61. 2 x + 2 y = 2 x + y Differentiating both sides, we get
1 dy
Differentiating both sides, we get × = 2 cosec 2 x + 4 cosec 4 x + 6 cosec 6 x
y dx
dy æ dy ö
2 x log 2 + 2 y log 2 = 2 x + y × log 2 ç1 + ÷
dx è dx ø \
dy
= 2y ( cosec 2 x + 2 cosec 4 x + 3 cosec 6 x ) …(iii)
dy dx
Þ log 2 × 2 y (2 x - 1 ) = 2 x (1 - 2 y ) log 2 tan 2 x + tan x
dx tan (3 x ) = tan (2 x + x ) =
dy 2 x (1 - 2 y ) 1 - tan 2 x tan x
Þ = …(i)
dx 2 y (2 x - 1 ) Þ tan 3 x × tan 2 x × tan x = tan 3 x - tan 2 x - tan x
Also, 2 = 2 (2 - 1 )
x y x
Differentiating both sides, we get
dy dy
\ = (1 - 2 y ) …(ii) = 3 sec 2 3 x - 2 sec 2 2 x - sec 2 x
dx dx
when 2 y = 2 x (2 y - 1 ) [from Eq. (i)] x 1 é (x + 1) + (x - 1) ù
65. Here, f (x ) = = ê ú
dy -1 1 (x - 1) (x + 1) 2 ë (x - 1) (x + 1) û
Þ = x = …(iii)
dx 2 - 1 1 - 2 x
1é 1 1 ù
f (x ) = ê + ú
As, 2 x = 2 y (2 x - 1 ) and 2 y = 2 x (2 y - 1 ) 2 ë x - 1 x + 1û
2 y 2 x (1 - 2 y )
\ - = 1é 1 1 ù
2 x 2 y (2 x - 1 ) Þ f ¢( x ) = ê - - ú
2 ë (x - 1) 2
(x + 1)2 û
90 Textbook of Differential Calculus

1é 2 2 ù f (5 ) = 3
f ¢¢ ( x ) = ê + ú > 0, for all x > 1
2 ë (x - 1) 3
(x + 1)3 û f (x ) = 3
f ¢( x ) = 0
\ f ( x ) is concave up.
So, Statements I and II both are correct and Statement II is the
p p
66. Let x = tan q, where - < q < . correct explanation of Statement I.
2 2
2x æ 1 - x2 ö
Þ f ( x ) = sin -1(sin 2q) 73. Let f ( x ) = sin -1 , g( x ) = cos-1 ç ÷
1 + x2 è1 + x2 ø
ì p
ï p - 2 q, 2
< 2q < p
ì p - 2 tan -1 x , x >1 ì 2 tan -1 x, -1 £ x £1
ïï p p ï ï -1
\ f (x ) =í 2q , - £ 2 q £ = í 2 tan -1 x , -1 £ x £ 1 f ( x ) = í p - 2 tan x, x >1
ï 2 2 ï ï - p - 2 tan -1 x, < -1
- p - 2 tan -1 x , x < - 1 î x
ï -p - 2q , -p <2q < - p î
îï 2 ì 2 tan -1 x, x ³ 0
g( x ) = í -1
ì -2 î - 2 tan x, x < 0
ï , x >1
ï1 + x
2
ì 2
ï 2 -2 -2 ï , -1 £ x £1
\ f ¢( x ) = í , -1 £ x £ 1 Þ f ¢(2 ) = = 1 + x2
1 + x 2
1 + 4 5 ï
ï ï 2
ï -2 , f ¢( x ) = í - , x >1
x < -1 ï 1+x
2
ï1 + x 2
î ï- 2 , x < -1
67. As, ( f ( g( x )) ¢ = f ¢( g( x )) × g ¢( x ) ï 1 + x2
î
( f ( g( 0 ))) ¢ = f ¢( g( 0 )) × g ¢( 0 ) = c ì 2
c ïï 1 + x 2 , x ³ 0
Þ f ¢( 0 ) × g ¢( 0 ) = c Þ g ¢( 0 ) = g ¢( x ) = í
b 2
ï- , x<0
g( x ) cos x - g( 0 ) g ¢( x ) cos x - g( x ) sin x ïî 1 + x 2
68. lim = lim =0
x® 0 sin x x ® 0 cos x
f ¢( x )
Now, f ( x ) = g( x ) sin x = 1, 0 < x < 1
g ¢( x )
f ¢( x ) = g( x ) cos x + g ¢( x ) sin x
\ f ¢( 0 ) = 0 \ f ¢( x ) ¹ g ¢( x ), - 1 £ x £ 1
Statement I is correct and Statement II is incorrect.
On differentiating both sides, we get
f ¢¢( x ) = g ¢( x ) cos x - g( x ) sin x + g ¢¢( x ) sin x + g ¢( x ) cos x 74. Given, f ( x + y 3 ) = f ( x ) + f (y 3 ), " x, y Î R
= g ¢¢( x ) sin x + 2 g ¢( x ) cos x - g( x ) sin x Put x = y = 0, we get
Also, f ¢( 0 ) = g ¢( 0 ) f (0 + 0) = f (0) + f (0) Þ f (0) = 0
\Both statements are correct but Statement II is not the
correct explanation of Statement I. Now, put y = - x1/3, we get
69. Statement II is correct. f (0) = f (x ) + f (- x )
-1 1 -1 1 Þ f (x ) + f (- x ) = 0
\ y = sin (3 x - 4 x ) = 3 sin x, when - £ x £
3
2 2 Þ f ( x ) is an odd function.
dy 3 -1 1
Þ = only when £x£ Þ f ¢( x ) is an even function.
dx 1 - x2 2 2
Þ f ¢(2 ) = f ¢( -2 ) = a
\Both statements are true and Statement II is the correct
explanation of Statement I. Sol. (Q. Nos. 75 to 77)
70. Clearly, Statement I is false. Consider, f ( x + y ) - f ( x ) = f (y ) - 1 + 2 xy
æ 1 - x ö ì 2 tan x , x ³ 0
2 -1
cos-1 ç Þ f (0 + 0) - f (0) = f (0) - 1 + 0
÷ =í
è 1 + x 2 ø î - 2 tan -1 x , x < 0 Þ f ( 0 ) = 1 and f ¢( 0 ) = 1 [given]
\Statement II is correct but Statement I is incorrect. f (x + h ) - f (x ) é f (h ) - 1 2 xh ù
Also, f ¢( x ) = lim = lim ê +
71. Clearly, Statement II is true. h ®0 h h ®0 ë h h úû
dy ì 1, xÎ/ Integer f ¢( x ) = f ¢( 0 ) + 2 x
\ =í
dx î does not exist , x Î Integer \ f ¢( x ) = 2 x + 1
Thus, Statement I is incorrect but Statement II is correct. Integrating both sides, f ( x ) = x 2 + x + c
72. We have, f ( x ) is a continuous function f : R ® Q. As, f (0) = 1 Þ c = 1
f : R ® 0, iff f ( x ) is constant function. \ f (x ) = x 2 + x + 1
Chap 02 Differentiation 91

Thus, domain of log( f ( x )) = log( x 2 + x + 1 ) is x Î R Sol. (Q. Nos. 83 to 84)


Range of x 2 + x + 1 ³ 3 / 4 f 2( x + h ) × g 2( x + h ) - f 2( x ) × g 2( x )
D * (u × v ) = lim
\ Range of log 3/4 ( x 2 + x + 1 ) £ 1 h®0 h
Þ Range Î ( -¥, 1 ] f 2( x + h ) × g 2( x + h ) - f 2( x + h ) g 2( x )
g( 0 ) + g( 0 ) + f 2( x + h ) g 2( x ) - f 2( x ) g 2( x )
and g( 0 ) = Þ 2 g ( 0 ) = kg ( 0 ) = lim
k h®0 h
Þ g( 0 ) = 0 [Q k ¹ 2 ] æ g ( x + h ) - g 2( x ) ö
2

æ x + hö = lim f 2( x + h ) × ç ÷
gç ÷ - g( x ) h®0 è h ø
è 1 ø g( x ) + g(h ) - g( x )
g ¢( x ) = lim = lim æ f 2( x + h ) - f 2( x ) ö
h® 0 h h® 0 h + g 2( x ) ç lim ÷
g(h ) - g( 0 ) èh ® 0 h ø
= lim = g ¢( 0 ) = l
h® 0 h = f 2( x ) × ( D *v ) + g 2( x ) × ( D *u )
\ g ( x ) = lx + c = u 2( D *v ) + v 2( D *u )
As g( 0 ) = 0 Þ c = 0
f 2( x + h ) f 2( x )
Þ g ( x ) = lx -
æuö g 2( x + h ) g 2( x )
Þ lx = x 2 + x + 1 Þ x 2 + x(1 - l ) + 1 = 0 Also, D * ç ÷ = lim
èv ø h ® 0 h
Q D=0
f 2( x + h ) × g 2( x ) - g 2( x + h ) × f 2( x )
\ (1 - l ) 2 - 4 = 0 Þ l = 3, - 1 = lim
h®0 g 2( x + h ) × g 2( x ) × h
75. (c) 76. (a) 77. (c)
é f 2( x + h ) g 2( x ) - f 2( x ) g 2( x ) ù
Sol. (Q. Nos. 78 to 80) ê ú
êë + f ( x ) g ( x ) - g ( x + h ) f ( x ) úû
2 2 2 2
f (a - h ) - f (a ) = lim
LHD = lim h®0 g 2( x + h ) × g 2( x ) × h
- h
h®0

- f (a - h ) + f (a ) f (a - h ) - f (a ) é æ f 2( x + h ) - f 2( x ) ö 2 ù
= lim = lim êç ÷ × g (x ) ú
h ® 0- h h ® 0- -h êè h ø ú
f ¢(a ) + f ¢( -a + h ) f ¢(a ) + f ¢(h - a ) ê æ ö ú
RHD = lim = lim ê - f 2( x ) × ç lim g 2
( x + h ) - g 2
( x )
h ® 0- h h ® 0- h ÷ú
êë èh ® 0 h ø úû
f (-x ) - f (- x - h ) f (- x - h ) - f (- x ) = lim
Also, lim = lim h®0 g 2( x + h ) × g 2( x )
h ® 0- h h ® 0- -h
g 2( x ) × ( D *u ) - f 2( x ) × ( D *v )
= f ¢( - x ) …(i) =
f (x ) - f (x - h ) v 2 ×v 2
and lim = - f ¢( x ) …(ii)
h®0 -h v 2 × ( D *u ) - u 2( D *v )
=
From Eqs. (i) and (ii), f ¢( x ) is odd function and hence f ( x ) is v4
even function. 83. (b) 84. (c)
78. (a) 79. (a) 80. (b) dy
85. y = et sin t Þ = et [cos t + sin t ]
Sol. (Q. Nos. 81 to 82) dt
dx
Since, f ( x ) = sin -1(3 x - 4 x 3 ) x = et cos t Þ = et [cos t - sin t ]
dt
ì -1 1 dy cos t + sin t
ï - p - 3 sin x , - 1 £ x £ - 2 \ = = tan a
ïï dx cos t - sin t
1 1
= í 3 sin -1 x , - £x£ æp ö
ï 2 2 Þ tan ç + t ÷ = tan a
1 è4 ø
ï p - 3 sin -1 x , £ x £1
ïî 2 æp ö p
Þ ç + t÷ = a Þ t = a -
è4 ø 4
ì 3 1 1
ï , - £x£
ï 1-x
2 2 2 æp ö
\ f ¢( x ) = í sec 2 ç + t ÷
3 1 d 2y è4 ø
ï- , £ |x| £ 1 86. = t
ïî 1 - x 2 2 dx 2
e (cos t - sin t )
æ d 2y ö
æ 1 ö \ ç 2÷ =2
Þ f ¢( 0 ) = 3 and f ¢ ç ÷ = - 3 2
è 2ø è dx ø t = 0
81. (b) 82. (c)
92 Textbook of Differential Calculus

87. F (t ) = ò et (cos t + sin t ) dt = et sin t + C dy dy /dt 12t 2 - 6t - 18


91. (A) = = 4
dx dx /dt 5t - 15t 2 - 20
æpö
F ç ÷ - F ( 0 ) = (e p /2 + C ) - 0 = e p /2 12 - 6 - 18 2
è2ø é dy ù é dy ù
Þ ê ú = = Þ ê– 5 =–2
ë dx û t = 1 5 - 15 - 20 5 ë dx úû t = 1
Sol. (Q. Nos. 88 to 90)
88. Since, 1, a1, a 2, ……, an are roots of xn - 1 = 0. (B) Let us take
P ( x ) = a( x - 2 ) 4 + b ( x - 2 ) 3 + c ( x - 2 ) 2 + d ( x - 2 ) + e
\ xn - 1 = ( x - 1 )( x - a1 )( x - a 2 ) . . . ( x - an ) …(i)
-1 = P (2 ) = e
x -1
n
0 = P ¢(2 ) = d
Þ = ( x - a1 )( x - a 2 ) . . . ( x - an )
x -1 2 = P ¢¢(2 ) = 2c
xn - 1 Þ c =1
Þ lim = lim [( x - a1 )( x - a 2 ) . . . ( x - an )]
x ®1 x - 1 x ®1 -12 = P ¢¢¢(2 ) = 6b
Þ (1 - a1 )(1 - a 2 ) . . . (1 - an ) = n Þ b = -2
89. From Eq. (i), log ( xn - 1) = log ( x - 1) + log ( x - a1 ) P (2 ) = 24 a = 24 Þ a = 1
iv

+ . . . + log ( x - an ) Thus, P ¢¢( x ) = 12( x - 2 ) 2 - 12( x - 2 ) + 2


Differentiating w.r.t. x, we get Þ P ¢¢(3 ) = 12 - 12(1 ) + 2 = 2
nxn - 1 1 1 1 1 æ 1 ö 1 + x4 é 1ù
= + + + ... + (1 + y 4 ) = ç1 + 4 ÷ = êëQ y = x úû
…(ii) (C) Here,
xn - 1 x - 1 x - a1 x - a 2 x - an è x ø x2
Putting x = 2 in Eq. (ii), we get 1 + y4 1
n2n - 1 1 1 1 Þ = …(i)
=1 + + + ... + 1 + x4 x2
xn - 1 2 - a1 2 - a 2 2 - an
1
But y =
1 1 1 n2n - 1 x
Þ + + ... + = n -1
2 - a1 2 - a 2 2 - an 2 - 1 dy 1
\ =- 2 …(ii)
n -1 n -1 dx x
n2 -2 + 1 2
n
(n - 2 ) + 1
= = From Eqs. (i) and (ii), we get
2 -1
n
2n - 1
1 + y4 dy dy dx
nxn - 1 1 1 1 1 =- Þ + =0
90. From Eq. (ii), n - = + + ... + 1+x 4 dx (1 + y )4
(1 + x 4 )
x - 1 x - 1 x - a1 x - a 2 x - an
dy
nxn - 1 - 1(1 + x + x 2 + . . . + xn - 1 )
Þ 1 + y4
xn - 1 Þ =–1
dx
1 1 1
= + + ... + 1 + x4
x - a1 x - a 2 x - an
(D) Obviously, f ( x ) is a linear function.
nxn - 1 - 1(1 + x + x 2 + . . . + xn - 1 )
Þ lim Also, from f ¢( 0 ) = p and f ( 0 ) = q,
x ®1 xn - 1
f ( x ) = px + q Þ f ¢¢( 0 ) = 0
æ 1 1 1 ö 92. (A) We know that,
= lim ç + + .. + ÷
x ® 1 è x - a1 x - a2 x - an ø ì -1 æ 2 x ö
ï sin ç ÷, if - 1 £ x £ 1
n(n - 1 ) x n-2
- {1 + 2 x + . . . + (n - 1 ) x n-2
} ï è1 + x2 ø
Þ lim
x ®1 nx n -1 ïï æ 2x ö
2 tan -1 x = í p - sin -1 ç ÷, if x > 1
1 1 1 ï è1 + x2 ø
= + + ... +
1 - a1 1 - a 2 1 - an ï -1 æ 2 x ö
ï - p - sin ç ÷, if x < - 1
[applying L’Hospital’s rule on LHS] ïî è1 + x2 ø
n(n - 1 ) - {1 + 2 + . . . + (n - 1 )}
Þ Þ
dy
=-
2
, if x < - 1 or x > 1
n dx 1 + x2
1 1 1
= + + ... + æ 1 ö ì tan -1 x, x ³ 0
1 - a1 1 - a 2 1 - an (B) cos-1 ç ÷=
n -1 ç 1 + x 2 ÷ í - tan -1 x, x < 0
1 1 1 è ø î
Þ + + ... + =
1 - a1 1 - a 2 1 - an 2 dy 1
Þ =- , if x < 0
dx 1 + x2
Chap 02 Differentiation 93

ì | ex - e | , x ³ 0 95. P ( x ) = ( x - x1 ) 3 ( x - x 2 ) ( x - x 3 ) ¼ ( x - x 8 )
(C) y = | e| x | - e | = í - x
î| e - e | , x < 0 Þ P ( x ) = ( x - x1 ) 3 × Q ( x )
ì e x - e, x ³ 1 P (x )
ï \ Q (x ) =
ïe -e , 0 £ x <1
x
( x - x1 ) 3
=í -x
ïe - e , - 1 £ x < 0 As, Q ( x ) is polynomial.
ïîe - x - e, x < - 1 \ Q ( x ) must be continuous at x = x1.
Þ
dy
> 0, if x > 1 or -1 < x < 0 \ Q ( x1 ) = lim Q ( x )
x ® x1
dx
ì 1 x10 + ax 2 + bx + c
, x>0 = lim
du 1 dv ïï 1 + x 2 x ® x1 ( x - x1 ) 3
(D) = and =í
dx x dx ï - 1 , x < 0 By L’ Hospital’s rule, we have
ïî 1 + x 2 10 ´ 9 ´ 8 ´ x17
Q ( x1 ) =
ì 1+x 2
3 ´2 ´1
du ïï , x>0
Þ =í x 7
dv ï 1 + x 2 æ1ö
10 ´ 9 ´ 8 ´ ç ÷
ïî - x , x < 0 \
æ1ö
Qç ÷=
è2ø
=
15 p
=
è2ø 3 ´2 ´1 16 q
1 + x2 1
Now, we know that = x + > 2, if x > 1 and Þ q - p =1
x x
du
< -2, if x < - 1 Þ > 2, if x < - 1 or x > 1 4(x - a )3 (x - a )3 1
dv
96. Here, f ¢( x ) = 4 ( x - b ) 3 ( x - b ) 3 1
93. Here, f ( x ) = x 3 + x 2 f ¢(1 ) + xf ¢¢(2 ) + f ¢¢¢(3 )
4(x - c )3 (x - c )3 1
Put f ¢(1 ) = a, f ¢¢(2 ) = b, f ¢¢¢(3 ) = c
Þ f ( x ) = x 3 + ax 2 + bx + c (x - a )4 3(x - a )2 1 (x - a )4 (x - a )3 0
+ (x - b )4 3(x - b ) 1 + (x - b )4
2
(x - b )3 0
f ¢( x ) = 3 x 2 + 2ax + b
(x - c )4 3(x - c )2 1 (x - c )4 (x - c )3 0
Þ f ¢(1 ) = 3 + 2a + b or a = 3 + 2a + b
Þ a + b = -3 …(i) (x - a )4 (x - a )2 1
f ¢¢ = 6 x + 2a Þ f ¢¢(2 ) = 12 + 2a = 0 + 3 (x - b )4 (x - b )2 1 + 0
\ b = 12 + 2a Þ 2a - b = - 12 …(ii) (x - c )4 (x - c )2 1
f ¢¢¢( x ) = 6 Þ c = 6 …(iii)
Þ l =3
From Eqs. (i) and (ii), we get
a = - 5, b = 2 97. Let P ( x ) = a( x - 1)( x - 3)( x - 5)( x - l)
\ f (1 ) = 1 + a + b + c Þ log P ( x ) = log a + log( x - 1 ) + log( x - 3 )
=1 -5 + 2 + 6 = 4 + log( x - 5 ) + log( x - 7 )
æ æ öö Differentiating both sides, we get
2x + 2
94. Let y = tan çsin -1 çç ÷ ÷ and z = tan -1 x.
÷
1
× P ¢( x ) =
1
+
1
+
1
+
1
ç è 4 x + 8 x + 13 ø ÷ø
2
è P (x ) x -1 x -3 x -5 x - l
æ æ 2x + 2 öö As, P ¢(7 ) = 0 [Q P ( x ) = 0 Þ P ¢( x ) = 0 Þ P ¢(7 ) = 0]
Þ y = tan çsin -1 ç ÷÷ 1 1 1 1
ç ç (2 x + 2 ) 2 + 9 ÷ ÷ Þ 0= + + +
è è øø 6 4 2 7-l
æ æ 2x + 3 ö ö 2x + 3 89 p
= tan ç tan -1 ç ÷ = Þ l= =
è è 3 ø ÷ø 3 11 q

Þ
dy 2
= and
dz
=
1 \ p - 8q = 1
dx 3 dx 1 + x 2 1
98. We have, x2 + y 2 = t -
dy t
dy dx 2 2
\ = = (1 + x 2 ) æ 1ö 1
Þ (x 2 + y 2 )2 = çt - ÷ = t 2 + 2 - 2
dx dz 3 è tø t
dx
Þ x 4 + y 4 + 2 x 2y 2 = x 4 + y 4 - 2
æ dy ö 2æ 1ö 2 3
Þ ç ÷ = ç1 + ÷ = ´ = 1 Þ 2 x 2y 2 = -2
è dz ø x = 1 3è 2ø 3 2
2
Þ y 2 = -x - 2
94 Textbook of Differential Calculus

Differentiating w.r.t. x, we get 2


101. Put cos f =
dy 13
2y × = +2 x - 3
dx 3
and sin f =
dy 1 13
\ =+ 3
dx xy Þ y = cos-1 {cos( x + f )} + sin -1 {cos( x - f )}
æ dy ö p
Þ ç ÷ =1 =x+f+ -f+x
è dx ø (1,1) 2
p
99. We have, x 2 + y 2 + z 2 - 2 xyz = 1 …(i) y = 2x +
2
Þ d ( x + y + z - 2 xyz ) = 0
2 2 2
and z = 1 + x2
Þ 2 x dx + 2y dy + 2z dz - 2( xy dz + yz dx + zx dy ) = 0 1
æ2 1 + x2 ö 2 1+
dy 2 1 + x
2
Þ ( x - yz ) dx + (y - zx ) dy + (z - xy ) dz = 0 \ = =ç ÷ = 3 =4
ç ÷ 1
dx dy dz x è x øx = 1
Þ + 3
(y - zx ) (z - xy ) ( x - yz ) (z - xy ) 3
dz
+ =0 102. Here, f ¢¢( x ) - 2 f ¢( x ) - 15 f ( x ) = 0
( x - yz ) (y - zx )
Now, (y - zx ) 2 (z - xy ) 2 = (y 2 - 2 xyz + z 2x 2 ) Þ (a 2 - 2a - 15 ) eax + (b 2 - 2b - 15 ) ebx = 0
(z 2 - 2 xyz + x 2y 2 ) Þ a 2 - 2a - 15 = 0 and b 2 - 2b - 15 = 0
= (1 - x 2 - z 2 + z 2x 2 ) (1 - x 2 - y 2 + x 2y 2 ) [using Eq. (i)] Þ a = (5 or - 3) and (b = 5 or - 3)
= (1 - x ) (1 - z ) (1 - x ) (1 - y )
2 2 2 2 Since, a ¹ b
\ a + b = 2 or - 2
= (1 - x 2 ) 2 (1 - y 2 ) (1 - z 2 )
Þ |a + b | = 2
\ (y - zx ) (z - xy ) = (1 - x 2 ) × (1 - y 2 ) (1 - z 2 ) x
103. A : 2x + 2yy ¢ = 0 Þ y ¢ = -
Similarly, ( x - yz )(z - xy ) = (1 - y 2 ) × (1 - x 2 ) (1 - z 2 ) y
\ y ¢( 2 ) = - 1
and ( x - yz ) (y - zx ) = (1 - z 2 ) × (1 - x 2( (1 - y 2 )
Substituting these values in Eq. (i), we get B : cos y × y ¢ + cos x = sin x × cos y × y ¢ + sin y × cos x
when x =y = p
dx dy dz
+ + =0
1-x 2
1 -y 2
1 - z2 Þ -y ¢ - 1 = 0 + 0
Þ y ¢( p ) = - 1
100. We have, x = sin t or t = sin -1 x
dy dy dt dy 1 C : 2e ( xy ¢ + y ) + e e y ¢ + e ye x - e x - e yy ¢
xy x y

Now, = × = ×
dx dt dx dt 1 - x2 = e × e xy ( xy ¢ + y )

Þ
dy dy
1 - x2
= At x = 1, y = 1
dx dt 2e(y ¢ + 1 ) + e 2y ¢ + e 2 - e - ey ¢ = e 2(y ¢ + 1 )
Differentiating w.r.t. x, we get
d 2y 1 ( -2 x ) dy d æ dy ö Þ ey ¢ + e = 0 Þ y ¢ = - 1
1 - x2 × 2 + × = ç ÷
dx 2 1 - x 2 dx dx è dt ø Hence, A - B - C = 1
2
d y x dy d y dt 2 dx 3 2 æ 3 + 2t ö
Þ 1 - x2 × - = 2× 104. =- 4 - 3 =-ç 4 ÷
2 dt t t è t ø
dx 1 - x dx dt dx
2

d 2y d 2y
dy æ3 2ö æ 3 + 2t ö
dy 1 = - ç 3 + 2÷ = - ç 3 ÷
Þ (1 - x 2 ) 2
-x = 1 - x2 × 2 × dt èt t ø è t ø
dx dx dt 1`- x 2
dy dy / dt
d 2y dy d 2y \ = =t
\ (1 - x 2 )2
-x = dx dx / dt
dx dx dt 2
æ1 + t ö
Adding y on both sides, we get = t - ç 3 ÷ × t3 = -1
è t ø
d 2y dx d 2y
(1 - x 2) 2 - x +y = 2 +y 3
dx dy dx du æ dy ö
Now, -x×ç ÷
dx è dx ø
\ l =1
Chap 02 Differentiation 95

105. We have, y = f ( x ) - f (2x ) When f ( x ) = 2, then


Þ y ¢ = f ¢( x ) - 2 f ¢(2 x ) x 3 + 3x + 2 = 2

y ¢(1 ) = f ¢(1 ) - 2 f ¢(2 ) = 5 …(i) Þ x=0


i.e. when x = 0, then f ( x ) = 2
and y ¢(2 ) = f ¢(2 ) - 2 f ¢( 4 ) = 7 …(ii)
1
Now, let y = f (x ) - f (4x ) \ g ¢( f ( x )) = 2 at (0, 2)
3x + 3
Þ y ¢ = f ¢( x ) - 4 f ¢( 4 x )
1
Þ y ¢(1 ) = f ¢(1 ) - 4 f ¢( 4 ) …(iii) Þ g ¢(2 ) =
3
Substituting the value of f ¢(2 ) = 7 + 2 f ¢( 4 ) in Eq. (i), \ Option (a) is incorrect.
f ¢(1 ) - 2 [7 + 2 f ¢( 4 )] = 5 Now, h( g( g( x ))) = x
Þ f ¢(1 ) - 4 f ¢( 4 ) = 19 Þ h( g( g( f ( x ))) = f ( x )
Þ h( g( x )) = f ( x ) …(ii)
Hence, 19 = 10 + l
As g( f ( x )) = x
Þ l =9
\ h( g(3 )) = f (3 )
106. y = 3e - x
2
= 3 3 + 3(3 ) + 2 = 38
2
-x \ Option (d) is incorrect.
Let x y = x 3e
2 From Eq. (ii),
-x
Þ f ( x ) = x 3e h( g( x )) = f ( x )
Þ ln ( f ( x )) = (3e 2 - x ) ln x Þ h( g( f ( x ))) = f ( f ( x ))
1 3e - x 2 Þ h( x ) = f ( f ( x )) …(iii)
Þ × f ¢( x ) = - ln x
f (x ) x [using g( f ( x )) = x]
\ f ¢( x ) = 0 Þ h ¢( x ) = f ¢( f ( x )) × f ¢( x ) …(iv)
Þ 3e - x = x ln x
2
Putting x = 1, we get
Þ 3e = x(1 + ln x )
2 h ¢(1 ) = f ¢( f (1 )) × f ¢(1 )
Þ x =e 2
[by verification] = (3 ´ 36 + 3 ) ´ (6 ) = 111 ´ 6 = 666

Hence, l =1 \ Option (b) is correct.


Putting x = 0 in Eq. (iii), we get
107. f ¢( x ) = (kx + e )h ¢( x ) + h( x )(k + e x )
x
h( 0 ) = f ( f ( 0 ))
\ f ¢( 0 ) = h ¢( 0 ) + h( 0 )(k + 1 )
= f (2 ) = 8 + 6 + 2 = 16
Þ 18 = - 2 + 5 (k + 1 )
\ Option (c) is correct.
Þ k =3 æ 6x x ö æ 1ö
110. Let y = tan -1 ç ÷ , where x Î ç 0, ÷
108. We have, f ( x ) = log 2 - sin x è 1 - 9x 3 ø è 4ø
and g( x ) = f ( f ( x )), x Î R æ 2 . (3 x 3/2 ) ö
Note that, for x ® 0, log 2 > sin x = tan -1 ç ÷ = 2 tan -1 (3 x 3/2 )
è 1 - (3 x 3/2 ) 2 ø
\ f ( x ) = log 2 - sin x
æ 3ö
Þ g( x ) = log 2 - sin ( f ( x )) As 3 x 3/2 Î ç 0, ÷
è 8ø
= log 2 - sin (log 2 - sin x )
dy 1 3
Clearly, g( x ) is differentiable at x = 0 as sin x is differentiable. \ =2 ´ ´ 3 ´ ´ x 1/ 2
dx 1 + 9x 3 2
Now, g ¢( x ) = - cos (log 2 - sin x ) ( - cos x )
9
= cos x × cos (log 2 - sin x ) = x
1 + 9x 3
\ g ¢( 0 ) = 1 × cos (log 2)
9
109. As, g( f ( x )) = x \ g( x ) =
1 + 9x 3
Thus, g( x ) is inverse of f ( x ).
111. Since, f ( x ) = e g ( x )
Þ g( f ( x )) = x
Þ g ¢( f ( x )) × f ¢( x ) = 1 Þ e g ( x + 1) = f ( x + 1 ) = xf ( x ) = xe g ( x )
1 and g( x + 1 ) = log x + g( x )
\ g ¢( f ( x )) = [where, f ¢( x ) = 3 x 2 + 3] …(i)
f ¢( x ) i.e. g( x + 1 ) - g( x ) = log x ...(i)
96 Textbook of Differential Calculus

1
Replacing x by x - , we get æ æ x öö æxö 1
Þ F ¢ (x ) = 2 ç f ç ÷ ÷ × f ¢ ç ÷ ×
2 è è 2 øø è2ø 2
æ 1ö æ 1ö æ 1ö
g ç x + ÷ - g ç x - ÷ = log ç x - ÷ æ æ x öö æxö 1
è 2ø è 2ø è 2ø + 2 ç g ç ÷ ÷ × g ¢ ç ÷ × = 0 [from Eq.(i)]
è è 2 øø è2ø 2
= log(2 x - 1 ) - log 2
-4 \ F ( x ) is constant Þ F (10 ) = F (5 ) = 5
æ 1 ö æ 1 ö
\ g ¢¢ ç x + ÷ - g ¢¢ ç x - ÷ = ...(ii) 114. Given that, log ( x + y ) = 2xy
è 2 ø è 2 ø (2 x - 1 ) 2 …(i)

On substituting x = 1,2, 3,..., N in Eq. (ii) and adding, we get \ At x = 0

æ 1ö æ1ö ì 1 1 1 ü Þ log (y ) = 0
g ¢¢ ç N + ÷ - g ¢¢ ç ÷ = - 4 í1 + + + ... + 2ý
.
è 2ø è2ø î 9 25 (2 N - 1 ) þ Þ y =1
-1 dy
dx 1 æ dy ö \ To find at ( 0, 1 )
112. Since, = =ç ÷ dx
dy dy / dx è dx ø
On differentiating Eq. (i) w.r.t. x, we get
-1
d æ dx ö d æ dy ö dx 1 æ dy ö dy
⇒ ç ÷= ç ÷ ç1 + ÷ = 2x + 2y × 1
dy è dy ø dx è dx ø dy x+y è dx ø dx
æ d 2y ö æ dy ö - 2 æ dx ö dy 2y ( x + y ) - 1
d 2x Þ =
⇒ = - ç 2÷ç ÷ ç ÷ dx 1 - 2 ( x + y ) x
dy 2 è dx ø è dx ø è dy ø
æ dy ö
Þ ç ÷ =1
æ d 2y ö æ dy ö -3 è dx ø ( 0, 1)
= - ç 2÷ç ÷
è dx ø è dx ø
115. Given, x 2 + y 2 = 1
113. Since, f ¢¢ ( x ) = - f ( x ) On differentiating w.r.t. x, we get
d 2 x + 2yy ¢ = 0
Þ { f ¢( x )} = - f ( x )
dx Þ x + yy ¢ = 0
Þ g ¢ (x ) = - f (x ) [Q g( x ) = f ¢( x ), given] …(i) Again, differentiating w.r.t. x, we get
ì æ x öü ì
2
æ x öü
2
1 + y ¢y ¢ + yy ¢¢= 0
Also, F (x ) = í f ç ÷ý + í g ç ÷ý
î è 2 øþ î è 2 øþ \ 1 + (y ¢ ) 2 + yy ¢¢ = 0
CHAPTER

03
Functions
Learning Part
Session 1 Session 7
● Function ● Periodic Functions
Session 2 Session 8
● Domain ● Mapping of Functions
● Algebraic Functions
Session 9
Session 3 ● Identical Functions
● Transcendental Functions
Session 10
Session 4
● Composite Functions
● Piecewise Functions
Session 11
Session 5
● Range
● Inverse of a Function

Session 6 Session 12
● Odd and Even Functions ● Miscellaneous Problems on Functions

Practice Part
● JEE Type Examples
● Chapter Exercises

Arihant on Your Mobile !


Exercises with this #L
symbol can be practised on your mobile. See title inside to activate for free.
Session 1
Function
Function Sol. From the given figure, we conclude that
Let A and B be two non-empty sets and f be a relation A ® Domain of f = { p, q, r, s }
which associates each element of set A with unique B ® Codomain of f = {1, 2, 3, 4, 5}
element of set B, f is called a function from A to B. Range of f = {1, 2, 3, 4}
Or
Remark
If for each element in a set A there is assigned a unique We should always remember that “the range is a subset of
element of set B, we call such an assignment a function codomain”.
f : A ® B. “f is a function from A to B”.
If a Î A, the element in B which is assigned to ‘a’ is called
the image of ‘a’ and denoted by f (a).
Number of Functions
Here, set A is called the domain of f and B is called the (or Mapping) from A to B
codomain of f . The set of elements of B, which are the Let A = { x 1 , x 2 , x 3 , ... , x m } [i.e.m elements]
images of the elements of set A is called the range of f . and B = {y 1 , y 2 , y 3 ,..., y n } [i.e. n elements]
e.g. Let A = { a, b, c , d }, B = { 1, 2, 3, 4, 5 }. Here, f (a ) = 2, Then, each element x i 's (i = 1, 2, K, m ) of domain
f (b ) = 3, f (c ) = 5, f (d ) = 1 given by corresponds n images.
f:A B f:A B
1 x1 y1
a
2 x2 y2
b ... ...
3
... ...
c
4 xm yn
d
5
Figure 3.2
Figure 3.1
i.e. x 1 can take n images.
i.e. A ® Domain of f = { a, b, c , d } x 2 can take n images.
B ® Codomain of f = { 1, 2, 3, 4, 5 } ......................................
Range of f = { 1, 2, 3, 5 } ......................................
An element of A (i.e. Domain) cannot associate with more x m can take n images.
than one element in B.
Thus, total number of functions from A to B
y Example 1 In the given figure, find the domain, = n ´ n ´ . . . m times = n m
codomain and range. i.e. (Number of elements in codomain) number of elements in domain .
A f B
y Example 2 The number of functions f : {1, 2, 3, ..., n}
p 1
q 2 ® {2016, 2017}, where n Î N , which satisfy the
r 3 condition f (1) + f (2) + ... + f (n ) is an odd number are
s 4 (a) 2n (b) n × 2n -1
5
(c) 2n -1 (d) n !
Chap 03 Functions 99

Sol. f:A B Sol. We have, y = f ( x ) = x 3 ," x Î R.


1 Y
2 1
3 2016
-
- 2017
- X′ X
n x=–1 O x=1

We can send 1, 2, 3, ..., (n - 1) any where in 2n -1 ways, the –1


value f (n ) can uniquely determined according to sum of Y′
f (1) + f (2) + ... + f (n - 1) is even or odd. Here, all the straight lines drawn parallel to Y -axis cut
i.e. if { f (1) + f (2) + ... + f (n - 1)} Îeven, f (n ) = 2017 y = x 3 only at one point. Thus, y = f ( x ) forms a mapping.
and if { f (1) + f (2) + ... + f (n - 1)} Îodd, f (n ) = 2016 x2 y
2

\ Number of functions = 2 n -1 y Example 4 Find whether 2 + 2 = 1, a > b , forms a


mapping or not. a b
Graphical Representation of Functions x2 y2 b 2
Sol. We have, 2
+ 2
= 1, i.e. y = ± a – x 2 , which
a b a
The graph of y = f ( x ), where f is a function, consists of
represents a horizontal ellipse.
points whose coordinates (x , y ) satisfy y = f ( x ). Also, it is
Y
such that lines drawn parallel to the Y-axis do not
intersect the curve at more than one point.
Vertical line test for f to be a function If graph of a
X′ X
function is cut a line parallel to Y-axis at more than one O
point, then it does not form a function.

y Example 3 Find whether f ( x )= x 3 forms a mapping Y′


or not. Here, straight lines drawn parallel to Y-axis meet the curve
b 2
at more than one points. Thus, f ( x ) = ± a – x 2 does not
form a mapping. a

Exercise for Session 1


1. Which of the following graphs are graphs of a function?

Y Y
Y

(i) X (ii) X (iii) X


O O O

Y Y Y

(vi) O X
(iv) X (v) X
O O
100 Textbook of Differential Calculus

Y Y

(vii) O (viii) X
X O

Y
Y

(ix) X (x) X
O O

2. For which of the following, y can be a function of x, ( x Î R, y Î R ) ?


(i) (x - h )2 + (y - k )2 = r 2 (ii) y 2 = 4ax
(iii) x4 = y 2 (iv) x 6 = y 3
(v) 3y = (log x )2

3. Let g( x ) be a function defined on [ - 1, 1]. If the area of equilateral triangle with two of its vertices at (0, 0) and
3
( x , g( x )), is sq unit, the function g( x ) may be
4
(a) g (x ) = ± 1- x 2 (b) g (x ) = 1- x 2
(c) g (x ) = - 1 - x 2 (d) g (x ) = 1+ x 2

4. Represent diagramatically all possible functions from {a, b } to {1, 2 }.

5. The number of functions from f : {a1, a 2, ¼, a10} ® {b1, b 2, ¼, b 5} is


(a) 105 (b) 510
10 !
(c) (d) 5!
5!
Session 2
Domain, Algebraic Functions
Domain Þ
5x - 1
£ 0 and 7 - x - 7 ¹ 0
The domain of y = f ( x ) is the set of all real x for which 7-x - 7
f ( x ) is defined (real). Now, 5x - 1 = 0 Þ x = 0 and 7 - x - 7 = 0 Þ x = -1
Rules for Finding Domain – + –
–1 0
(i) If domain of y = f ( x ) and y = g ( x ) are D 1 and D 2
respectively, then the domain of f ( x ) ± g( x ) or 5x - 1
The sign of
f ( x ) × g ( x ) is D 1 Ç D 2 . 7-x - 7
f (x ) Hence, from the sign scheme x Î ( -¥, - 1) È [0, ¥ ).
(ii) Domain of is D 1 Ç D 2 – { x : g( x ) = 0 }.
g( x ) \ Domain of f ( x ) is ( -¥,-1) È [0, ¥ ).

y Example 5 Find the domain of the following


functions. Classification of Functions
(i) y = 5x - 3 (ii) y = 3 5x - 3 Functions are classified by the type of mathematical equation
1 1 which represent their relationship.
(iii) y = (iv) y =
(x - 1) (x - 2) 3 (x - 1)
Functions
Sol. (i) Here, y = 5x - 3 is defined, if
5x - 3 ³ 0 1. Algebraic 2. Transcendental 3. Piecewise
3 é3 ö
Þ x³ Þ x Î ê , ¥ ÷. Monomial Exponential Modulus
ë5 ø
5 Polynomial Constant Logarithmic Signum
Identity Greatest integer
Rational Trigonometric
é3 ö
Thus, domain is ê , ¥ ÷. Irrational Inverse Fractional part
ë5 ø trigonometric Least integer
(ii) Here, y = 3 5x - 3 is defined, if
5x - 3 Î R Þ x Î R Algebraic Functions
Thus, domain is R . A function f is called an algebraic function, if it can be
1 constructed using algebraic operations such as addition,
(iii) Here, y = is defined, if
( x - 1)( x - 2) subtraction, multiplication, division and taking roots, starting
( x - 1)( x - 2) ¹ 0 with polynomials.
i.e. if x ¹ 1, 2 e.g. f (x ) = 1 + x ,
Thus, domain is R - {1, 2}. x 3 - 16 x
1 g( x ) = + ( x - 2) 3 x -1
(iv) Here, y = is defined, if x+ x
3 x -1

Different types of algebraic functions are described below


x - 1 ¹ 0Þx ¹ 1
Thus, domain is R - {1}. (i) Monomial Function
1 - 5x A function of the form y = ax n , where a is constant and n is a
y Example 6 Find the domain of f ( x ) = .
7 -x - 7 non-negative integer, called monomial function.
1 - 5x -x
e.g. y = x 2 , y = 2 x , y = -x , ……… .
Sol. For f ( x ) to be defined, ³ 0 and 7 -7 ¹0
7 -x - 7
102 Textbook of Differential Calculus

(ii) Polynomial Function (iv) Graph of y = x ( x - 1) ( x - 2) is shown as


Y
A function y = f ( x ) = a 0 x n + a 1 x n – 1 + K + a n , where y=x(x–1)(x–2)
a 0 , a 1 , a 2 , ..., a n are real constants and n is a non-negative
integer, then f ( x ) is called a polynomial function. If
a 0 ¹ 0, then n is the degree of polynomial function. e.g.
X
Expression Degree O 1 2

f ( x ) = x1920 + 5 x1919 + 6 x polynomial of degree 1920


g (x ) = x + 3x + 3
2
polynomial of degree 2
\ Domain; x Î R and Range; y Î R.
h (x ) = 7 = 7x 0
polynomial of degree 0
Constant Function
A polynomial of degree one with no constant term is
If the range of a function f consists of only one number,
called an odd linear function, i.e. f ( x ) = ax , a ¹ 0 then f is called a constant function. The graph of a
In case f ( x ) = 0, it is constant function for which degree is constant function is as shown below.
not defined. A polynomial of odd degree has its range
Y
( - ¥, ¥) but a polynomial of even degree has a range
which is always subset of R. f(x) = c
Note: All polynomial functions are algebraic functions but
not the converse. X′ X
O
y Example 7 Draw the graph of polynomial functions Y′
and find their domain and range. Figure 3.3
(i) y = x + 1 (ii) y = x 2
Thus, domain is R and range is {c }.
(iii) y = x 3 + 1 (iv) y = x (x - 1)(x - 2)
Sol. (i) Graph of y = ( x + 1) is shown as
Identity Function
Y
The function y = f ( x ) = x for all x Î R is called an identity
y=x+1
function on R.
Y
y=x
X′ X
O

X′ X
Y′ O
From graph domain is x Î R and range is y Î R.
(ii) Graph of y = x 2 is shown as
Y Y′
Figure 3.4

X′ X
Here, domain of identity function is the set of real
O number. (As here for all values of x , f ( x ) exists.)
Y′
Thus, domain is R and range is R.
From graph domain is x Î R and range is y Î [0, ¥ ).
(iii) Graph of y = x 3 + 1 is shown as (iii) Rational Function
Y y=x3+1 It is defined as the ratio of two polynomials.
Let P ( x ) = a 0 x n + a 1 x n – 1 + K + a n
1 and Q ( x ) = b 0 x m + b 1 x m – 1 + K + b m .
X′ X P (x )
–1 O
Then, f ( x ) = is a rational function provided
Q (x )
Y′ Q ( x ) ¹ 0.
\ Domain; x Î R and Range; y Î R.
Chap 03 Functions 103

Here, we can say that domain of f ( x ) is all real numbers e.g. f ( x ) = x 1 /2 ,


except the numbers where denominator is zero
x2 + x
[i.e. Domain (f ) = R - { x : Q ( x ) = 0 }]. e.g. f (x ) = , etc.
x 2 – 3x + 5 1 + 3x 2
e.g. f ( x ) = ; Domain is R – {1, 2 }
( x – 1)( x – 2 )
An Important Result
1
e.g. f ( x ) = ; As 2 – cos 3 x ¹ 0. [Q–1 £ cos 3 x £ 1] There are two polynomial functions, satisfying the relation;
2 – cos 3 x
f ( x ) × f æç ö÷ = f ( x ) + f æç ö÷ .
1 1
\ Domain of f ( x ) is R. èx ø èx ø
They are
(iv) Irrational Function (i) f ( x ) = 1 + x n and (ii) f ( x ) = 1 - x n,
An algebraic function having non-integral rational where n is a positive integer.
exponent is called irrational function. For proof, refer Example 38 in Jee Type Solved Examples.

Exercise for Session 2


n
Directions (Q. Nos. 1 to 6) Find the domain of the following.

1. f ( x ) = x 2 - 5x + 6

2x + 1
2. f (x ) =
x - 3x 2 + 2x
3

3. f (x ) = 1 - 1 - 1 - x 2

4. f ( x ) = x 12 - x 9 + x 4 - x + 1

5. f ( x ) = 16 - x C2x -1 + 20 - 3 x
P4x – 5
x 2 + 4x
6. f (x ) = C2x 2 + 3
Session 3
Transcendental Functions
Transcendental Functions Properties of Logarithmic Functions
The functions which are not algebraic are called 1. log e (ab ) = log e a + log e b [a , b > 0 ]
transcendental functions. Exponential, logarithmic, æa ö
2. log e ç ÷ = log e a – log e b [a , b > 0 ]
trigonometric and inverse trigonometric functions are èb ø
transcendental functions.
3. log e a m = m log e a [a > 0, m Î R ]

(i) Exponential Functions 4. log a a = 1 [a > 0, a ¹ 1]


1
The function f ( x ) = a x , a > 0, a ¹ 1, a Î constant is said to 5. log bm a = log b a [a, b > 0, b ¹ 1 and m Î R – { 0 }]
m
be an exponential function. 1
6. log b a = [a, b > 0 and a, b ¹1
If 0 < a < 1 If a > 1 log a b
Y log m a
Y
Session- 7. log b a = [a, b, m > 0 and b, m ¹ 1]
log m b
1 1
8. a loga m = m [a, m > 0 and a ¹ 1]
X′ O X X′ O X
9. a logc b = b logc a [a, b, c > 0 and c ¹ 1]
ìx > y , if m > 1
Y′ Y′ 10. If log m x > log m y Þ í
Figure 3.5 Figure 3.6 îx < y , if 0 < m < 1
[m, x , y > 0, m ¹ 1]
In the given graphs, none of them intersect the X-axis.
11. log m a = b Þ a = m b
Thus, we can say that they exist for all x Î R and their
corresponding values of y are greater than zero. [m, a > 0, m ¹ 1, b Î R]
Thus, domain is R and range is ] 0, ¥[. ì a >m , b
if m >1
12. log m a > b Þ í
îa < m ,
b
if 0 <m < 1
(ii) Logarithmic Functions ìï a < m b , if m >1
13. log m a < b Þ í
The function f ( x ) = log a x ; ( x , a > 0 ) and a ¹ 1 is a îï a > m ,
b
if 0 <m < 1
logarithmic function. Thus, domain of a logarithmic
function is the set of all positive numbers and their range y Example 8 Find the domain of a single-valued
is the set R of real numbers. function y = f ( x ) given by the equation 10 x + 10 y = 10.
If 0 < a < 1 If a > 1 Sol. Given, 10x + 10y = 10
Y Y Þ 10y = 10 – 10x
Þ y = log10 (10 – 10x ) [as am = b Þ m = loga b]
X′ X′ Now, 101 – 10x > 0
O 1 X O 1 X
Þ 101 > 10x Þ 1> x
Y′ Y′
Therefore, domain of the single-valued function
Figure 3.7 Figure 3.8 y = f ( x ) is x < 1 or x Î ( - ¥, 1)
Chap 03 Functions 105

y Example 9 Find the domain of y Example 11 Find the domain of f ( x ) = log 10 (1 + x 3 ) .


1 Sol. f ( x ) = log10 (1 + x 3 ) exists,
f (x ) = ×
log 1/ 2 ( x 2 - 7 x + 13) if 1+ x 3 > 0
1 Þ (1 + x )(1– x + x 2 ) > 0
Sol. f ( x ) = exists, if
log1/ 2 ( x 2 - 7 x + 13) [where (1 - x + x 2 ) is always positive as D <0 and a >0]
log1/ 2 ( x 2 - 7 x + 13) > 0 So, 1 + x > 0 or x >–1 or x Î ] – 1, ¥ [
Þ ( x –7 x + 13) < 1
2
...(i) Thus, domain of above function f ( x ) is ]– 1, ¥ [ .

and x 2 – 7 x + 13 > 0 ...(ii) y Example 12 Find the domain of


Considering Eq. (ii), x –7 x + 13 > 0, we get
2 f ( x ) = log 10log 10 (1 + x 3 ) .
æ 2 49 ö 49 Sol. f ( x ) = log10log10 (1 + x 3 ) exists,
ç x – 7 x + ÷ + 13 – >0
è 4 ø 4 if log10 (1 + x 3 ) > 0 and 1 + x 3 > 0
2
æ 7ö 3 Þ 1 + x 3 > 100 and 1 + x 3 > 0
Þ çx – ÷ + > 0
è 2ø 4 Þ 1 + x 3 > 1 and 1 + x 3 > 0
2
æ 7ö Þ 1 + x3 > 1Þ x3 > 0
which is true for all x Î R. [As ç x – ÷ ³ 0 for all x.] ...(iii)
è 2ø Þ x Î (0, ¥ )
Now, considering Eq. (i), we get Thus, domain of above function f ( x ) is (0, ¥ ).
x 2 – 7 x + 12 < 0
y Example 13 Find the domain of
Þ ( x – 3) ( x – 4 ) < 0
f (x ) = log 10 {log 10 log 10 log 10 x } .
Þ 3< x < 4 ...(iv)
Sol. f ( x ) exists, if {log10 log10 log10 x } > 0 and x >0
Combining Eqs. (iii) and (iv), we get
Domain of f ( x ) is ] 3, 4 [. Þ log10 log10 x > 10 0
and x >0
Þ log10 log10 x > 1 and x >0
y Example 10 Find the domain of function Þ log10 x > 101 and x >0
1 Þ x > 1010 x >0
f (x ) = + ( x + 2). and
log 10 (1 – x ) Thus, f ( x ) exists, if x Î (10 , ¥ ).
10

1
Sol. f ( x ) = + x +2 Hence, domain of above function f ( x ) is (1010, ¥ ) .
log10 (1 – x )
(As we know, loga x is defined when x, a > 0 and a ¹ 1, y Example 14 Find the domain for
also loga 1 = 0) æ x–1 ö
Thus, log10 (1 – x ) exists when f ( x ) = log 0. 4 ç ÷.
è x + 5ø
1– x > 0 ...(i)
1 æ x –1 ö
Also, exists when Sol. f ( x ) = log 0. 4 ç ÷ exists, if
log10 (1 – x ) è x +5ø
1 – x > 0 and 1 - x ¹ 1 …(ii) æ x –1 ö æ x –1 ö
log 0. 4 ç ÷ ³0 and ç ÷ >0
Þ x < 1 and x ¹0 ...(iii) è x +5ø è x +5ø
Now, we have x + 2, which exists when x + 2 ³ 0 x –1 x –1
Þ £ (0.4 )0 and >0
or x ³ –2 ...(iv) x +5 x +5
1 x –1 x –1
Thus, f ( x ) = + x + 2 exists when Eqs. (iii) and Þ £1 and >0
log10 (1 – x ) x +5 x +5
(iv) both hold true. x –1 x –1
Þ –1 £ 0 and >0
Þ –2 £ x < 1 and x ¹ 0 x +5 x +5
Þ x Î [–2, 0) È (0,1) Þ
–6
£0 and
x –1
>0
Thus, domain of f ( x ) is [ -2, 0) È (0, 1). x +5 x +5
106 Textbook of Differential Calculus

x –1
Þ x +5>0 and >0 Cosine Function f ( x ) = cos x
x +5 Y
Þ x >–5 and x –1 > 0 [as x + 5 > 0] (0, 1)
y=1
Þ x >–5 and x >1
Thus, domain f ( x ) is (1, ¥ ). X′ X
π O π

2 2
y Example 15 Find the domain of
æ 2 log 10 x + 1 ö (–π, –1) (π, –1)
y = –1
f ( x ) = log 100x ç ÷
è –x ø Y′
Figure 3.10
Sol. f ( x ) exists, if 100x > 0 ...(i)
Here, domain is R and range is [– 1, 1].
100x ¹ 1 ...(ii)
2 log10 x + 1
>0 ...(iii)
Tangent Function f ( x ) = tan x
–x Y
2 log10 x + 1 < 0 [as x > 0 from Eq. (i)]
1
log10 x < –
2 –π π
X′ π π X
3π 3π
x < (10) –1/ 2
...(iv) –
2

2
O 2
2
Now, from Eqs. (i), (ii) and (iv), we get
x Î (0, 10– 2 ) È (10– 2 , 10– 1/ 2 ). Y′
Thus, domain of f ( x ) is (0, 10-2 ) È(10-2 , 10-½ ). Figure 3.11

y Example 16 Find the domain of definition of ì(2n + 1) p ü


Here, domain is R – í , n Î Z ý and range is R.
log 2 ( x + 3) î 2 þ
f (x ) = 2 ×
x + 3x + 2 [IIT JEE 2001] Cosecant Function f ( x ) = cosec x
log 2 ( x + 3)
log 2 ( x + 3) Y
Sol. Here, f ( x ) = 2 = exists, if
x + 3x + 2 ( x + 1)( x + 2)
y=1
x + 3 > 0 Þ x >–3
Numerator ...(i)
(–32π , 1( ( π2 , 1(
and denominator ( x + 1)( x + 2) ¹ 0 X′ X
O
Þ x ¹– 1,– 2 ...(ii) – π , –1 ( 32π,–1(
( 2 (
Thus, from Eqs. (i) and (ii), we get domain of f ( x ) is
y = –1
(–3, ¥ ) - {–1,– 2 }.

x = –2π x=–π Y′ x=π x = 2π


(iii) Trigonometric Functions Figure 3.12

Sine Function f ( x ) = sin x Here, domain is R – { np, n Î Z } and range is R – (– 1, 1).


Y Secant Function f ( x ) = sec x
(– 32π , 1( ( 2π , 1( Y
y=1
y=1
(–2π,1) (0,1) (2 π,1)
X′ –π π X′ X
O X O
(–π,–1) (π,–1)
y = –1
y = –1
3 π , –1
– π ,–1
( ( ( ( π
2 Y′ 2 x = 3π x = π Y′ x = x=

2 2 2 2
Figure 3.9
Figure 3.13
Here, domain is R and range is [–1, 1]. p
Here, domain is R – {(2n + 1) , n Î Z } and range is
R – (–1, 1). 2
Chap 03 Functions 107

Cotangent Function f ( x ) = cot x Inverse Trigonometric Functions


Y We know that trigonometric functions are many-one in
their actual domain. Hence, for inverse functions to get
defined, the actual domain of trigonometric functions
must be restricted to make the function one-one.
X′
3 0 O 3 0
X y = sin –1 x Y
y = sin–1 x
2
0
2
0 π
2
Since, y = sin x is not 2 y=x
one-one. Therefore, it is 1
y = sin x
not invertible. Let us
X′ X
x=–2 x=– Y′ x= x =2 consider the function –π –1 O 1 π
2
2
Figure 3.14 é -p p ù
y = sin x , x Î ê , ú –1
ë 2 2û
Here, domain is R – {np, n Î Z } and range is R. –π
2
and y Î[ -1, 1] whose Y′
y Example 17 Find domain for graph is the portion of
Figure 3.15
f ( x ) = cos (sin x ). the sin x curve in the
Sol. f ( x ) = cos(sin x ) is defined, if
é -p p ù
interval ê , ú , which is strictly increasing. Hence,
ë 2 2û
cos(sin x ) ³0 ...(i)
one-one and its inverse is y = sin -1 x .
As, we know –1 £ sin x £ 1 for all x
é p pù
Here, domain is [– 1, 1]; range is ê– , ú .
1 ë 2 2û

y = cos –1 x
y = cos–1 x Y

π –1 O 1 π π
2 2 y=x
y = cos x π/2
1

X′ X
–π –1 O 1 π
\ cos(sin x ) ³0, for all x (using graph of y = cos x ) –1
y = cos x
i.e. x ÎR
π
Thus, domain f ( x )is R. Y′

y Example 18 The function f ( x ) is defined in [0, 1]. Find Figure 3.16

the domain of f (tan x ). Here, domain is [– 1, 1] ; range is [0, p ].


Sol. Here, f ( x ) is defined in [0, 1].
y = tan –1 x y=tanx
Þ x Î[0, 1], i.e. we can substitute only those values of x, Y
which lie in [0, 1]. π y=x
2
For f (tan x ) to be defined, we must have
y=tan–1x
0 £ tan x £ 1 [as x is replaced by tan x]
X′ X
π π
p O
i.e. np £ x £ np + [in general] 2 2
4
π
p 2
or 0£ x £ [in particular] Y′
4
Figure 3.17
é pù
Thus, domain for f (tan x ) is ênp, np + ú. æ p pö
ë 4û Here, domain is R; range is ç – , ÷ .
è 2 2ø
108 Textbook of Differential Calculus

y = cot –1 x Y
y=x æx2 ö
y Example 19 Find the domain for f ( x ) = sin –1 ç ÷ .
è 2 ø
π
y = cot–1 x
æx2 ö
π Sol. f ( x ) = sin –1 ç ÷ is defined, if
2 è 2 ø
X′ X
O π x2
2 –1 £ £ 1 or –2 £ x 2 £ 2
2
Y′ y = cot x Þ 0 £ x2 £ 2 [as x 2 cannot be negative]
Þ – 2£x£ 2
Figure 3.18
Therefore, domain of f ( x ) is [ - 2, 2 ].
Here, domain is R ; range is (0, p ).
y Example 20 Find the domain for
y = sec –1x
x= π æ æ x 2 öö
2
y= x y = sin –1 çç log 2 ç ÷ ÷÷ .
Y è è 2 øø
y= π
x2
y = sec–1 x Sol. For y to be defined, >0
y= π 2
1 2
...(i)
X′ X æx2 ö
–1 O 1 π and – 1 £ log 2 ç ÷ £ 1 ...(ii)
è 2 ø
–1
From Eq. (i), we have x Î R – { 0 } ...(iii)
From Eq. (ii), we have
Y′ x =1
x2
Figure 3.19 2–1 £ £ 21 Þ 1 £ x 2 £ 4
2
Here, domain is R – (– 1, 1); range is [0, p ]– { p / 2 }. Þ –2 £ x £ – 1 or 1 £ x £ 2 ...(iv)

y = cosec x –1 Thus, from Eqs. (iii) and (iv), we get


x Î [– 2, – 1] È [1, 2]
Y y = cosec x
æ 1+ x 2 ö
y Example 21 Find the domain for f ( x ) = sin –1 çç ÷÷ .
π
è 2x ø
2 x æ1+ x 2 ö
y= Sol. f ( x ) = sin –1 ç ÷ is defined, if
1 è 2x ø
π y = cosec–1 x 1+ x 2 1+ x 2
2 –1 –1 £ £ 1 or £1
X′ X 2x 2x
O 1 π
2
Þ | 1 + x 2 | £ | 2x |
–1
Þ 1 + x 2 £| 2 x | [as 1 + x 2 > 0]
π Þ x2 - 2| x | + 1 £ 0
2
Þ | x |2 - 2 | x | + 1 £ 0 [as x 2 =| x | 2 ]
Y′ Þ (| x |–1)2 £ 0
Figure 3.20 But (| x |–1)2 is either always positive or zero.
Thus, (| x |–1)2 = 0
é p pù
Here, domain is R – (– 1, 1); range is ê– , ú – { 0 }.
ë 2 2û Þ | x | =1 Þ x = ±1
Thus, domain for f ( x ) is {– 1, 1}.
Chap 03 Functions 109

y Example 22 Find the domain for y = sin –1 (log 2 x ). 3


log| sin x | ( x 2 – 8x + 23)– >0
log 2 |sin x |
Sol. y is defined, if x > 0, ...(i)
Þ log| sin x | ( x 2 – 8x + 23)– 3 log| sin x | 2 > 0
– 1 £ log 2 x £ 1
æ x 2 – 8x + 23 ö
Þ 2–1 £ x £ 2 Þ log | sin x | ç ÷ >0
1 è 8 ø
Þ £ x £2 ...(ii)
2 x 2 – 8x + 23
Þ < |sin x | 0
and sin –1 (log 2 x ) ³ 0 ...(iii) 8
Þ log 2 x ³ 0 and |sin x | ¹ 0, 1
Þ x ³ 20 Þ x 2 – 8x + 23 < 8 and |sin x | ¹ 0, 1

Þ x ³1 ...(iv) Þ x 2 – 8x + 15 < 0 and sin x ¹ 0, ± 1


From Eqs. (i), (ii) and (iv), we get p
Þ ( x – 3) ( x – 5) < 0 and x ¹ np, (2n + 1)
1£ x £2 2
Hence, domain is 1 £ x £ 2 or x Î[1, 2]. 3p
Þ x Î(3, 5) and x ¹ p, [using number line rule]
2
y Example 23 Find the domain of the function, æ 3p ö æ 3p ö
Þ x Î (3, p ) È ç p, ÷ È ç , 5÷
ì 3 ü è 2 ø è 2 ø
f ( x ) = log ílog |sin x | ( x 2 – 8 x + 23)– ý.
î log 2|sin x | þ æ 3p ö æ 3p ö
Thus, domain of f ( x ) is (3, p ) È ç p , ÷ È ç , 5÷.
è 2 ø è 2 ø
Sol. Here, f ( x ) is defined, if

Exercise for Session 3


n
Directions (Q. Nos. 1 to 10) Find the domain of the following.

1. f ( x ) = log10 ( x - 4 + 6 - x )

æ 5x - x 2 ö
2. f ( x ) = log1/ 2 ç ÷
è 4 ø

æ 3x - x 2 ö
3. f ( x ) = log0. 3 ç ÷
è x -1 ø

log0. 3 ( x - 1)
4. f (x ) =
x 2 - 2x - 8

5. f ( x ) = log10 (1 - log10 ( x 2 - 5x + 16))

6. f ( x ) = sin | x | + sin-1 (tan x ) + sin (sin-1 x )


-1
(log 4 x 2 )
7. f ( x ) = e cos
æ 3 - 2x ö
8. f ( x ) = sin-1 ç ÷ + 3-x
è 5 ø
log2x 3
9. f (x ) =
cos -1 (2x - 1)

10. f ( x ) = log10 log2 log2/ p (tan-1 x )-1


Session 4
Piecewise Functions
Piecewise Functions (x) | x - y | = | x | + | y |, iff xy £ 0
(xi) | x | £ a, (a is +ve) Þ - a £ x £ a or x Î[ - a, a ]
(i) Absolute Value Function (xii) | x | ³ a, (a is +ve) Þ x £ - a or x ³ a
(or Modulus Function) or x Î ( - ¥, - a ] È [a, ¥)
The modulus function is defined as
(xiii)| x | £ a, (a is –ve) Þ No solution or x Î f
ì x, x ³ 0
y =| x | =í . (xiv) | x | ³ a, (a is –ve) Þ x Î Real number or x Î ( - ¥, ¥)
î– x , x < 0 (xv) a £ | x | £ b, where a and b are +ve.
It is the numerical value of x .
Þ - b £ x £ - a or a £ x £ b or x Î[ - b, - a ] È [a, b ]
Y
y = –x
y=x y Example 24 Solve for x.
|x|-1
135° (i) (| x | - 1)( | x | - 2) £ 0 (ii) ³0
45° |x|-2
X′ X
O (iii) | x - 3| + | 4 - x | = 1 (iv) | x | + | x + 4 | = 4
Y′ Sol. (i) ( | x | - 1) ( | x | - 2) £ 0
Figure 3.21 Using number line rule for | x | , we get
+ – +
Geometrical Interpretation of 1 2
Modulus of a Function
Þ 1£|x |£2
Geometrically, | x | represents the distance of the point
P ( x , 0 ) or Q ( -x , 0 ) from origin. i.e. x Î [ -2, - 1] È [1, 2]
|x |-1
|x| |x| (ii) ³ 0. Using number line rule for | x | , we get
| x | -2
X′ X + – +
Q (– x,0) O (0,0) P (x,0)
1 2
Figure 3.22
Þ | x | £ 1 or | x | > 2
i.e. x Î [ -1, 1] È ( - ¥, - 2) È (2, ¥ )
i.e. d (O , P ) = ( x - 0 ) 2 + (0 - 0 ) 2 = x 2 = | x |
(iii) | x - 3 | + | 4 - x | = 1
d (O , Q ) = ( -x - 0 ) 2 + (0 - 0 ) 2 = x 2 = | x | As we know, | x | + | y | = | x + y |, iff xy ³ 0
\ ( x - 3)( 4 - x ) ³ 0 or - ( x - 3)( x - 4 ) ³ 0
Properties of Modulus Using number line rule, we get
(i) | x |2 = x 2 (ii) x2 =| x | – + –
3 4
(iii) || x || = | - x | = | x | (iv) | x | = max { -x , x }
Þ x Î[3, 4 ]
a +b ½ a - b½
(v) -| x | = min { -x , x } (vi) max (a, b ) = +½ ½ (iv) | x | + | x + 4 | = 4
2 ½ 2 ½ As we know, | x | + | y | = | x - y |, iff xy £ 0
a +b ½ a - b½
(vii) min (a, b ) = -½ ½ \ x(x + 4) £ 0
2 ½ 2 ½ Using number line rule, we get
(viii)| x + y | £ | x | + | y | + – +
(ix) | x + y | = | x | + | y |, iff xy ³ 0 –4 0
Þ x Î [ - 4, 0]
Chap 03 Functions 111

y Example 25 Solve | x 2 - 1 + sin x | = | x 2 - 1 | + | sin x |, y Example 29 Domain of the function


1
where x Î [ -2p, 2p ]. f (x ) = is
Sol. Let f ( x ) = x 2 - 1 and g ( x ) = sin x
4 x - | x - 10x + 9 |
2

We know, | f ( x ) + g ( x ) | = | f ( x ) | + | g ( x ) | iff (a) (7 - 40, 7 + 40 ) (b) (0, 7 + 40 )


f ( x ) × g ( x ) ³ 0. (c) (7 - 40, ¥ ) (d) None of these
\ ( x 2 - 1) × sin x ³ 0 on [ -2p, 2p ]. 1
Sol. Here, f ( x ) = would exists, if
Using number line rule, we get 4 x - | x - 10x + 9 |
2

+ + + 4 x - | x 2 - 10x + 9 | > 0
–2π –π – –1 0 – 1 π – 2π
i.e. | x 2 - 10x + 9 | < 4 x ,
Þ x Î [ -2p, - p ] È [ -1, 0] È [1, p ] where
ì x 2 - 10x + 9, x £ 1 or x ³ 9ü
½ x ½ ½+ | x | = x
2
| x 2 - 10x + 9 | = í ý
y Example 26 Solve ½ × î - ( x - 10x + 9 ), 1 < x < 9
2
þ
½x - 1½ | x - 1|
Þ Two cases
x
Sol. Let f ( x ) = and g ( x ) = x Case I When x £ 1 or x ³ 9
x -1
\ x 2 - 10x + 9 < 4 x
x x2
Then, f (x ) + g(x ) = +x= Þ x 2 - 14 x + 9 < 0 Þ ( x - 7 )2 < 40
x -1 x -1
We know, | f ( x ) | + | g ( x ) | = | f ( x ) + g ( x ) | , Þ x Î (7 - 40, 7 + 40 ) [but x £ 1 or x ³ 9]
iff f (x )× g(x ) ³ 0 Þ x Î (7 - 40, 1] È [9, 7 + 40 ) …(i)
x x 2
Case II When 1 < x < 9
\ ×x ³0 Þ ³0
x -1 x -1 - x 2 + 10x - 9 < 4 x Þ x 2 - 6x + 9 > 0
+ Þ ( x - 3)2 > 0, which is always true except at x = 3
0 1
\ x Î (1, 9 ) - { 3} …(ii)
Þ x Î {0} È (1, ¥ )
From Eqs. (i) and (ii), domain of f ( x ) is
1
y Example 27 Find domain for y = . (7 - 40, 7 + 40 ) - { 3}
|x|– x
Hence, (d) is the correct answer.
Sol. y is defined, if (| x | – x ) > 0 Þ | x | > x , which is true for
negative x only. y Example 30 The domain of the function
Hence, domain Î (– ¥, 0).
f ( x ) = |sin -1 (sin x )| - cos -1 (cos x ) in [0, 2p ] is
y Example 28 Find domain for
é p ù é 3p ù
æ 1 – 2| x | ö (a) ê0, ú È ê , 2p ú (b) [ p,2p ]
y = cos –1 ç ÷ + log |x – 1| x . ë 2û ë 2 û
è 3 ø
ì pü ì p 3p ü
(c) [0, p ] - í ý (d) [0, 2p ] - í , ý
æ1 – 2 |x |ö 1 – 2 |x | î 2þ î2 2 þ
÷ exists, if – 1 £ £1
–1
Sol. Here, cos ç
è 3 ø 3
Sol. As, | sin -1 (sin x )| could be sketched, as
Þ – 3 £ 1 – 2 | x | £ 3 Þ –4 £ – 2 | x | £ 2
Y
Þ 2 ³ |x | ³ – 1 Þ –2 £ x £ 2 ...(i)
Also, log| x – 1| x exists, if π
x > 0, | x – 1| > 0 and | x - 1| ¹ 1 2
π–


π

x
x > 0, x Î R – { 1 } and x ¹ 0, 2
x–

...(ii)
x

–x

Hence, from Eqs. (i) and (ii), we get X


x Î(0, 1) È (1, 2) O π π 3π 2π
2 2
112 Textbook of Differential Calculus

and cos -1 (cos x ) could be sketched as ì -1, if x 2 + 1 < 0


ï
Y Sol. (i) f ( x ) = sgn ( x + 1) = í 0,
2
if x 2 + 1 = 0
ï 1, if x 2 + 1 > 0
î
x


Y

–x
(0, 1) f (x) = sgn (x 2 + 1)
X
O π 2π X′ X
O

Y′
\ | sin -1 (sin x )| > cos -1 (cos x ) is not possible.
f ( x ) = 1, x Î R
Only equality holds as
(ii) f ( x ) = sgn (loge x )
Y
ì 1, loge x > 0 ì 1, x >1
x) ï ï
c os = í -1, loge x < 0 \ f ( x ) = í -1, 0< x <1
–1 (
ï 0, loge x = 0 ï 0, x =1
c os î î
Graph for, f ( x ) = sgn (loge x ) is
X Y
O π π 3π 2π
2 2 1

Thus, | sin -1 (sin x )| = cos -1 (cos x ) X


O 1
é p ù é 3p ù
When x Î ê0, ú È ê , 2p ú –1
ë 2 û ë 2 û
é p ù é 3p ù (iii) f ( x ) = sgn (sin x )
So, domain for f ( x ) is ê0, ú È ê , 2p ú.
ë 2û ë 2 û ì 1, sin x > 0 ì 1, 2np < x < (2n + 1)p
ï ï
Hence, (a) is the correct answer. f ( x ) = í -1, sin x < 0 = í -1, (2n + 1)p < x < (2n + 2)p
ï 0, sin x = 0 ïî 0, x = np
î

(ii) Signum Function Y

Signum function is denoted by sgn (x) and it is defined by 1

ì| x | x ì 1, if x > 0 –2π –π O π
X
ï or , if x ¹ 0 ï 2π 3π
y = sgn ( x ) = í x |x | = í–1, if x < 0 –1
ïî 0, if x = 0 ï 0, if x = 0
î
Y (iv) f ( x ) = sgn (cos x )
y = 1, x > 0 ì 1, cos x > 0
ï
f ( x ) = í -1, cos x < 0
ï 0, cos x = 0
X î
O
ì 1, 2np - p/2 < x < 2np + p/2
ï
= í -1, 2np + p/2 < x < 2np + 3p/2
y = –1, x< 0 ï 0,
î x = (2n + 1)p/2
Figure 3.23 Y
1
y Example 31 Sketch the graph of
(i) f ( x ) = sgn ( x 2 + 1). (ii) f ( x ) = sgn (log e x ). –5π –3π –π π 3π 5π 7π
X
O
2 2 2 2 2 2 2
(iii) f ( x ) = sgn (sin x ). (iv) f ( x ) = sgn (cos x ). –1
Chap 03 Functions 113

(ix) [ x ] > n Û x ³ n + 1, n Î Integer


(iii) Greatest Integer Function
(x) [ x ] £ n Û x < n + 1, n Î Integer
or Step Function
[ x ] indicates the integral part of x, which is nearest and (xi) [ x ] < n Û x < n, n Î Integer
smaller to x . It is also known as floor of x.
é x ù é x + 1ù
[x] = n (xii) [ x ] = ê ú + ê ú
ë2û ë 2 û
é n + 1ù é n + 2 ù é n + 4 ù
ú +ê ú +ê ú + . . . = n,
n n+1 (xiii) ê
x
ë 2 û ë 4 û ë 8 û
Figure 3.24
n Î Natural number
Thus, [2 .3202 ] = 2, [0 . 23 ] = 0, [ 5 ] = 5,
[– 8 . 0725 ] = – 9, [– 0 . 6 ] = – 1 (xiv)[ x ] + [y ] £ [ x + y ] £ [ x ] + [y ] + 1
In general, n £ x <n +1 [n Î Integer] é 1ù é 2ù
(xv) [ x ] + ê x + ú + ê x + ú +
Þ [x ] = n ë n û ë n û
Here, f ( x ) = [ x ] could be expressed graphically as é n - 1ù
. . . + êx + = [nx ] , n Î N
ë n úû
x [ x]
Y
0 £ x <1 0 3 y Example 32 Find domain for, f ( x ) = cos -1 [ x ] .
1 £ x <2 1 2
Sol. As, y = cos -1 x exists, when -1 £ x £ 1.
2 £ x <3 2 1
\ f ( x ) = cos -1 [ x ] exists, when -1 £ [ x ] £ 1 Þ -1 £ x < 2
–1 £ x < 0 –1 X′
0 1
X
–2 –1 2 3
–2 £ x <–1 –2 –1 or x Î [ -1, 2)
–2
y Example 33 Find the value of
–3
Y′ é 3ù é 3 1 ù é 3 2 ù é 3 99 ù
Figure 3.25 êë 4 úû + êë 4 + 100úû + êë 4 + 100úû + K + êë 4 + 100úû .

(· ) darkened circle represents value is taken, ( o ) é3ù é3 1 ù é 3 24 ù é 3 25 ù


Sol. ê ú + ê + +K+ ê + + +
represents value is neglected. ë 4 û ë 4 100 úû ë 4 100 úû êë 4 100 úû
é 3 99 ù
+ ... + ê +
Properties of the Greatest ë 4 100 úû
Integer Functions Þ . ] + ...+ [0.99 ] + [10
[075 . ] Þ 75
. ] + ...+ [174
1442443 1442443
(i) [ x ] £ x < [ x ] + 1 25 terms are each 75terms are each
equal to zero equal to 1
(ii) x - 1 < [ x ] < x
Aliter Apply property (xv)
(iii) I £ x < I + 1 Þ [ x ] = I , where I Î Integer 3
Here, x = and n = 100
4
(iv) [[ x ]] = [ x ]
é ù é
3 3 1 ù é 3 99 ù é 3 ù
\ê ú+ê + ú + ... + ê + = ´ 100ú = 75
ì 0, if x Î Integer
(v) [ x ] + [ -x ] = í ë 4 û ë 4 100 û ë 4 100 úû êë 4 û
î-1, if x Ï Integer y Example 34 Given y = 2 [ x ] + 3 and y = 3 [ x – 2] + 5,
ì -x , if x Î Integer find the value of [ x + y ] .
i.e. [ -x ] = í
î-1 - [ x ], if x Ï Integer Sol. We have, 2[ x ] + 3 = 3 [ x – 2] + 5
ì 2x , if x Î Integer Þ 2[ x ] + 3 = 3[ x ]–6 + 5
(vi) [ x ] - [ -x ] = í Þ [x ] = 4 Þ 4 £ x < 5
î2 [ x ] + 1, if x Ï Integer
or x=4+f [f ® fraction]
(vii) [ x ± n ] = [ x ] ± n, n Î Integer and y = 2[ x ] + 3 = 11
(viii)[ x ] ³ n Û x ³ n, n Î Integer Hence, [ x + y ] = [ 4 + f + 11] = [15 + f ] = 15
114 Textbook of Differential Calculus

y Example 35 Find domain for 1


Sol. f ( x ) = exists, if
æ p ö [ x ]– x
f ( x ) = [sin x ] cos ç ÷.
è [ x – 1] ø [ x ]– x > 0
æ p ö i.e. [x ] > x
Sol. [sin x ] is always defined. cos ç ÷ is also defined But from definition of greatest integral function, we know
è [ x – 1] ø
except when [ x – 1] = 0 Þ 0 £ x – 1 < 1 [x ]£ x [as x = [ x ] + { x }]
Þ 1£ x <2 Thus, it is not possible that [ x ] > x .
Hence, domain of f ( x ) is R – [1, 2). Hence, domain f ( x ) = f
y Example 39 If domain for y = f ( x ) is [– 3, 2] , find
y Example 36 The domain of the function
log 4 ( 5 - [ x - 1] - [ x ] 2 ) the domain of g ( x ) = f {|[ x ]| }.
f (x ) = is Sol. Here, f ( x ) is defined in [– 3, 2].
x2 + x -2
Þ x Î[– 3, 2].
(where [ x ] denotes greatest integer function)
(i.e. we can substitute only those values of x, which lie
Sol. For domain of f ( x ), between [–3, 2] ).
For g ( x ) = f {|[ x ]| }to be defined, we must have
5 - [ x - 1] - [ x ]2 > 0
– 3 £ | [ x ]| £ 2
and x2 + x -2 ¹ 0 Þ 0 £ | [ x ]| £ 2 [as | x | ³ 0 for all x ]
Þ ( x + 2)( x - 1) ¹ 0 Þ – 2 £ [x ] £ 2 [as | x | £ a Þ – a £ x £ a]
Þ x ¹ 1, - 2 Þ –2 £ x < 3
[by definition of greatest integral function]
Now, 5 - [ x ] + 1 - [ x ]2 > 0
Hence, domain of g ( x ) is [– 2, 3 [ or [– 2, 3).
Þ [ x ]2 + [ x ] - 6 < 0
Þ ([ x ] + 3)([ x ] - 2) < 0 y Example 40 Find the domain of function
1
Þ - 3 < [x ] < 2 f (x ) = , where [ × ] denotes the
[ | x - 1| ] + [ | 7 - x | ] - 6
Þ -2 £ x < 2
greatest integer function.
Q x ¹ 1, - 2
Sol. f ( x ) is defined when
\ Domain Î ( -2, 1) È (1, 2)
[| x - 1 | ] + [ | 7 - x | ] - 6 ¹ 0
y Example 37 Let [ n + 1 ] = [ n + l ] , where
2 2 ì[1 - x ] + [ 7 - x ] ¹ 6 , when x £1 ...(i)
ï
Þ í [ x – 1] + [ 7 - x ] ¹ 6 , when 1 £ x £ 7 ...(ii)
n, l Î N . Show that l can have 2n different values.
ï [ x – 1] + [ x - 7 ] ¹ 6 , when x ³ 7 ...(iii)
Sol. We have, n 2 + 1 = (n + 1)2 – 2n < (n + 1)2 ; n Î N î
[1 - x ] + [ 7 - x ] ¹ 6
i.e. n2 + 1 < n + 1
1 + [ - x ] + 7 + [ - x ] ¹ 6 Þ 2 [ - x ] ¹ –2
Þ n < n2 + 1 < n + 1 Þ [ - x ] ¹ –1
Þ [ n2 + 1] = n Þ x Ï(0, 1] ...(a)
From Eq. (ii), we have
\ [ n2 + l ] = n [ x - 1] + [ 7 - x ] ¹ 6
Þ n < n2 + l < n + 1 Þ [x ] - 1 + 7 + [- x ] ¹ 6
Þ [x ] + [- x ] ¹ 0
or n 2 < ( n 2 + l ) < ( n + 1) 2
Þ x Ï Integer
Þ 0 < l < 2n + 1 Þ x Ï {1, 2, 3, 4, 5, 6, 7} ...(b)
Thus, l can take 2n different values. From Eq. (iii), we have
1 [ x - 1] + [ x - 7 ] ¹ 6
y Example 38 f ( x ) = , where [ × ] denotes the
[ x ]– x Þ [x ] - 1 + [x ] - 7 ¹ 6
Þ 2 [ x ] ¹ 14 Þ [ x ] ¹7
greatest integer function less than or equals to x.
Þ x Ï[ 7, 8) ...(c)
Then, find the domain of f ( x ).
Chap 03 Functions 115

Hence, froms (a), (b) and (c), we get Sol. Given that, [1 + sin x ] + [1 - cos x ] = sin x
Domain f ( x ) is R – {(0, 1) È { 1, 2, 3, 4, 5, 6, 7 } È (7, 8)}. Þ 1 + [sin x ] + 1 + [ - cos x ] = sin x
y Example 41 If the function f ( x ) = [3. 5 + b sin x ] Þ 2 + [sin x ] + [ - cos x ] = sin x

(where [ × ] denotes the greatest Þ 2 + [ - cos x ] = {sin x }


integer function) is an even function, the complete set Here, LHS is 1, 2 or 3, but RHS Î[0, 1)
of values of b is \ No solution. Hence, (d) is the correct answer.
(a) (- 0.5, 0.5) (b) [- 0.5, 0.5]
(c) (0, 1 ) (d) [ -1, 1] (iv) Fractional Part Function
Sol. We have f ( x ) = [35
. + b sin x ] For f ( x ) to be an even y = { x }, { x } indicates fractional part of x .
funcion. Let x = I + f , where I =[ x ]
3 < 35
. + b sin x < 4, " x Î R and f ={x }
Þ - 05. < b < 05
. Þ b Î ( - 05 . , 05
. )
Then, y ={x }
Hence, (a) is the correct answer.
= x –[ x ]
y Example 42 The domain of the function Here, y = { x } could be expressed graphically, as
1
f ( x ) = log 3 log 1/ 3 ( x 2 + 10x + 25) +
[x ] + 5 x {x }
Y
(where [ × ] denotes the greatest integer function) is 0 £ x <1 x

(a) (- 4 , - 3) (b) (- 6, - 5) 1 £ x <2 x –1 1


X′ X
(c) (- 6, - 4) (d) None of these 2 £ x <3 x –2 –2 –1 O 1 2 3

Sol. Here, log 3 log1/ 3 ( x 2 + 10x + 25) is defined, when –1 £ x < 0 x+1
–2 £ x <–1 x+2 Y′
I. x 2 + 10x + 25 > 0 , i.e. ( x + 5)2 > 0 Þ x ¹ - 5 …(i)
II. log1/ 3 ( x 2 + 10x + 25) > 0 Properties of Fractional Part of x Figure 3.26
(i) { x } = x, if 0 £ x < 1.
Þ x 2 + 10x + 25 < 1 (ii) { x } = 0, if x Îinteger.
or x 2 + 10x + 24 < 0 (iii) {– x } = 1 – { x }, if x Ïinteger.
or ( x + 6)( x + 4 ) < 0 (iv) { x ± integer } = { x }.
+ +
y Example 44 If { x } and [ x ] represent fractional
–6 –4

Þ x Î ( - 6, - 4 ) …(ii)
and integral parts of x respectively, then find the
2000
{x + r }
From Eqs. (i) and (ii), we get
x Î ( - 6, - 5) È ( - 5, - 4 ) …(iii)
value of [ x ] +
å 2000 .
r =1
III. [x ] + 5 ¹ 0 2000
{x + r } {x }
2000

Þ [x ] ¹ - 5 Sol. Clearly, [ x ] + å 2000 = [x ] + å 2000


r =1 r =1
Þ x Ï [ -5, - 4 ) …(iv)
[Q we know that { x + r } = { x }, if r Î Integer]
From Eqs. (iii) and (iv), we get
é {x } {x } ù
Domain of f ( x ) is ( - 6, - 5). = [x ] + ê + + ... + upto 2000 times ú
ë 2000 2000 û
Hence, (b) is the correct answer.
2000 { x }
= [x ] +
y Example 43 Let [ x ] be the greatest integer less than 2000
or equal to x. = [x ] + {x }
Then, the equation sin x = [1 + sin x ] + [1 - cos x ] has =x
é p pù ép ù 2000
{x + r }
(a) one solution in ê - (b) one solution in ê , p ú
ë 2
, ú
2û ë2 û
Thus, [x ] + å =x
r = 1 2000
(c) one solution in R (d) no solution in R
116 Textbook of Differential Calculus

y Example 45 Solve 4{ x } = x + [ x ]. (v) Least Integer Function


Sol. We know, x = [ x ] + { x }
y = ( x ) = [ x ],[ x ] or ( x ) indicates the integral part of x
2[ x ]
\ 4 { x } = [x ] + { x } + [x ] Þ { x } = ...(i) which is the nearest and greater to x .
3
But 0 £ {x } < 1
It is known as ceiling of x.
2[ x ] [x] = n ( x )= [ x ] = n +1
So, 0£ <1
3
3
Þ 0 £ [ x ] < , therefore [ x ] = 0 or 1 n x n+1
2 Figure 3.27
2
If [ x ] = 1, then { x } = [from Eq. (i)] Thus, [2 . 3203 ] = 3, (0 . 23 ) = 1, (– 8 . 0725 ) = – 8, (– 0.6 ) = 0
3
Thus, x=
5 In general, n < x £ n + 1 (n Î Integer)
3 Þ (x ) = n + 1 = [x ]
If [ x ] = 0, { x } = 0 so, x = 0 Here, f ( x ) = ( x ) = [ x ] can be expressed graphically as
ì 5ü
Thus, solutions of 4 { x } = x + [ x ] are x Î í0, ý × x [ x] = ( x )
Y
î 3þ 3
–1 < x £ 0 0 2
y Example 46 Prove that [ x ] + [ y ] £ [ x + y ] , where 0 < x £1 1 –3 –2 –1 1
X
x = [ x ] + { x } and y = [ y ] + { y }, [× ] represents greatest 1 < x £2 2 O 1 2 3
–1
integer function and {× } represents fractional part 2 < x £3 3 –2
function. –2 < x £–1 –1 –3

Sol. Here, x + y = [ x ] + [y ] + { x } + { y } (· ) represents value is taken.


Figure 3.28
( o ) represents value is neglected.
\ [ x + y ] = [[ x ] + [y ] + { x } + { y }]
= [ x ] + [y ] + [{ x } + { y }]
Properties of Least Integer Function
[using [ x + I ] = [ x ] + I , I ÎInteger]
(i) ( x) = x = [ x] holds, if x is Integer.
Þ [ x + y ] ³ [ x ] + [y ]
(ii) ( x + I) = [ x + I] = ( x ) + I, I Î Integer.
Remark (iii) Greatest integer function [ x ] converts
This could be generalised for n terms as x = I + f into I, while ( x ) converts to I + 1.
[ x1 ] + [ x2 ] + ... + [ xn ] £ [ x1 + x2 + ... + xn ].
(iv) x = ( x ) + { x } - 1,
y Example 47 Find the number of solutions of ì - ( x ), if x Î I
(v) ( - x ) = í
î - ( ) + 1, if x ÏI
|[ x ] – 2x | = 4, where [ x ] is the greatest integer £ x. x
ì 2 ( x ), if x Î I
Sol. Let x = I + f , I Î integer, f Î fractional part (vi) ( x ) - ( - x ) = í
[i.e. 0 £ f < 1] î 2 ( x ) - 1, if x Ï I
Then, |[ x ] – 2x | = 4 (vii)( x ) ³ n Û x > n - 1, n Î I
Þ |[ I + f ] – 2 ( I + f )| = 4 (viii)( x ) > n Û x > n, n Î I
Þ | I – 2I – 2 f | = 4 Þ | I + 2 f | = 4, (ix)( x ) £ n Û x £ n, n ÎI
æ 1 ö ( x ) < n Û x < n - 1, n ÎI
which is possible, only if ç f = or 0÷ (x)
è 2 ø æ (x ) ö æ x ö
1 (xi) ç ÷ = ç ÷ , n ÎN
If f = , è n ø è nø
2
æ n + 1ö æ n + 2 ö æ n + 4 ö
| I + 1| = 4 Þ I + 1 = ± 4 (xii) ç ÷+ç ÷+ç ÷ + . . . = 2 ( n - 1), n Î N
1 è 2 ø è 4 ø è 8 ø
So, I = 3, – 5 and f =
2 ì [ x ], x ÎI
(xiii) ( x ) = í
If f = 0, | I | = 4 î [ x ] + 1, x ÏI
So, I = ± 4 and f = 0 æ 1ö æ 2ö æ n - 1ö
ì 7 9ü (xiv) ( x ) + ç x + ÷ + ç x + ÷ + . . . + çx + ÷ = ( nx ) + n - 1, n Î N
Thus, number of solutions are x = í ± 4, , – ý , è nø è nø è n ø
î 2 2þ
i.e. 4 solutions.
Chap 03 Functions 117

y Example 48 Find the solution set of Sol. Let x = I + f , where I Î integer, f Î fractional part such
( x ) 2 + ( x + 1) 2 = 25, where ( x ) is the least integer that 0 £ f < 1.
\ [ x ]2 + ( x )2 > 25
greater than equals to x.
Þ [ I + f ]2 + ( I + f )2 > 25
Sol. Let x = I + f , where I (integer) and f (fractional part)
such that 0 < f < 1 . Þ I 2 + {I + 1}2 > 25
Then, ( I + f )2 + ( I + f + 1)2 = 25 Þ I 2 + I 2 + 2I + 1 > 25
Þ { I + 1} 2 + { I + 2} 2 = 25 Þ 2I 2 + 2I – 24 > 0
Þ I 2 + 2I + 1 + I 2 + 4 I + 4 = 25 Þ I 2 + I – 12 > 0
Þ 2I 2 + 6I + 5 – 25 = 0 Þ ( I + 4 )( I – 3) > 0
Þ 2I + 6I – 20 = 0
2
+ +
So, I = 2, – 5 –4 – 3
Thus, x = 2 + f ,– 5 + f
\ I < –4
where 0 < f < 1
or I >3
Þ 2 < 2 + f < 3, – 5 < – 5 + f < – 4 ...(i)
Here, x=I +f
Again, let x =I
So, x < –4 + f
Then, x + ( x + 1)2 = 25
2
or x >3+ f …(i)
Þ x = 3, - 4 ...(ii)
Now, let x = I , then x + x > 25
2 2
From Eqs. (i) and (ii), we get x Î ( - 5, - 4 ] È (2, 3]
Þ x 2 > 12.5
y Example 49 If [ x ] = the greatest integer less than Þ x £ –4 or x ³ 4
or equal to x and ( x ) = the least integer greater Form Eqs. (i) and (ii), we get x Î ( - ¥, - 4 ] È [ 4, ¥ )
than or equal to x and [ x ] 2 + ( x ) 2 > 25, then x
belongs to ........... .

Exercise for Session 4


n
Directions (Q. Nos. 1 to 18) Find the domain of the following.
1. f (x ) = x 2 - | x | - 2

2. f (x ) = 2 - | x | + 1 + | x |

3. f ( x ) = loge | loge x |
æ 2 - 3 [x ]ö
4. f ( x ) = sin-1 ç ÷ , which [ × ] denotes the greatest integer function.
è 4 ø

5. f ( x ) = log ( x - [ x ]), where [ × ] denotes the greatest integer function.


1
6. f (x ) = , where [ × ] denotes the greatest integer function.
[x ] - [x ] - 6
2

7. f ( x ) = cosec-1 [1 + sin2 x ], where [ × ] denotes the greatest integer function.


118 Textbook of Differential Calculus

|x |
8. f ( x ) = cos -1 log [ x ] , where [ × ] denotes the greatest integer function.
x
x -1
9. f (x ) = , where { × } denotes the fractional part function.
x - 2 {x }

æ [x ] ö
10. f ( x ) = sin-1 ç ÷ , where [ × ] and { × } denotes the greatest integer and fractional part function.
è {x } ø

11. f ( x ) = sin-1 [2x 2 - 3], where [ × ] denotes the greatest integer function.
é æ x 2 öù
12. f ( x ) = sin-1 êlog2 ç ÷ ú , where [ × ] denotes the greatest integer function.
êë è 2 ø úû

13. f ( x ) = 2 {x } 2 - 3 {x } + 1, x Î [ -1, 1], where { × } denotes the fractional part function.


1
14. f (x ) = , where [ × ] denotes the greatest integer function.
[| x - 2 | ] + [| x - 10 | ] - 8

15. If a function is defined, as g( x ) = | sin x | + sin x , f ( x ) = sin x + cos x , 0 £ x £ p, then find the domain for
f ( x ) = logf ( x ) g( x ).
1
16. Solve the equations, y = [sin x + [sin x + [sin x ]]] and [ y + [ y ]] = 2 cos x , where [ × ] denotes the greatest
3
integer function.
é x ù é x + 1ù
17. If [ x ] = ê ú + ê , where [ × ] denotes the greatest integer function and n be positive integer, then show that
ë 2 û ë 2 úû
é n + 1ù é n + 2 ù é n + 4 ù é n + 8 ù
êë 2 úû + êë 4 úû + êë 8 úû + êë 16 úû + . . . = n.

18. Find the integral solutions to the equation [ x ] [ y ] = x + y . Show that all the non-integral solutions lie on exactly
two lines. Determine these lines.
Session 5
Range

Range y Example 51 Find the range for f ( x ) =


ex
The range of a function is the set of all possible values that 1+ [x ]
can be produced by that function. when x ³ 0.
e.g. Let f ( x ) = x 2 Sol. Here, f ( x ) is defined for all x ³ 0. Also, f ( x ) is an
increasing function in [0, ¥ ).
No matter what value we substitute for x , f ( x ) will always
Thus, range = [ f (0), f ( ¥ ))
be positive. Therefore, we would say that the range of this
function is the set of all positive real numbers. Hence, range = [1, ¥ )

y Example 52 Find the domain and range of the


Rules for Finding the Range function y = log e ( 3x 2 – 4 x + 5).
First of all find the domain of given function y = f ( x ) Sol. y is defined, if 3x 2 – 4 x + 5 > 0
(i) If domain contains finite number of points, range is Where D = 16 – 4 (3)(5) = – 44 < 0
the set of corresponding f ( x ) values.
and coefficient of x 2 = 3 > 0
(ii) If domain is R or R – {some finite points},
Hence, (3x 2 – 4 x + 5) > 0, " x Î R
express x in terms of y. From this, find y for x to be
defined. Thus, domain Î R

(i.e. Find the values of y for which x exists.) Now, from y = loge (3x 2 – 4 x + 5) ,

(iii) If domain is a finite interval, then find the least and we have, 3x 2 – 4 x + 5 = e y or 3x 2 – 4 x + (5 – e y ) = 0
the greatest values for range, using monotonicity. Since, x is real, therefore discriminant ³ 0.
e.g. If f ( x ) is defined on [a, b ] , then i.e. (– 4 )2 – 4(3)(5 – e y ) ³ 0 Þ 12e y ³ 44
Step 1 Put, f ¢ ( x ) = 0 Þ x = a 1 , a 2 ,K, a n Î (a, b ) 11 æ 11 ö
So, e y ³ . Thus, y ³ log ç ÷
Step 2 Find, {f (a ), f (a 1 ), f (a 2 ), … , f (a n ), f (b )} 3 è3ø
The greatest and least values gives the range of f ( x ). é æ 11 ö ö
Hence, range is ê log ç ÷, ¥ ÷.
ë è3ø ø
x – [x ]
y Example 50 Find the range for y = . Aliter Since, log is an increasing function and3x 2 – 4 x + 5
1– [x ] + x 2
is minimum at x = , i.e. loge (3x 2 – 4 x + 5) is minimum at
x – [x ] {x } 3
Sol. Here, y = =
1 + x – [ x ] 1 + {x } 2 11 æ 11 ö
x = and minimum value ofy = loge , i.e. ç loge £ y < ¥ ÷ .
Thus, domain is R. 3 3 è 3 ø
é æ 11 ö ö
{x } Thus, range is ê loge ç ÷ , ¥ ÷ .
Now, from y = , ë è3ø ø
1 + {x }

we have, y + y {x } = { x } Þ { x } =
y y Example 53 Find the range of y = x – 1 + 5 – x .
1 –y
Sol. Here, domain x ³ 1 and x £ 5, i.e. x Î[1, 5].
y 1
Here, 0 £ { x } < 1, so 0 £ <1 Þ 0£y < dy 1 1
1–y 2 Now, check monotonicity, = –
dx 2 x – 1 2 5 – x
é 1ö
Hence, range = ê0, ÷. dy
ë 2ø When = 0, we get x = 3
dx
120 Textbook of Differential Calculus

+
– or x Î [ - 2, 2 ]
1 3 5
min max min Q 0 £ | 1 - x 2 | £ 1, " x Î [ - 2, 2 ]
Thus, y min at x = 1 or x = 5 p
y min at x = 1 is 2; \ 0 £ cos -1 |1 - x 2 | £
2
y min at x = 5 is 2. Hence, (a) is the correct answer.
Thus, minimum value of y = 2.
Also, y max at x = 3 is 2 2. y Example 56 If x, y and z are real such that
Hence, range = [2, 2 2 ] x + y + z = 4, x 2 + y 2 + z 2 = 6, x belongs to
(a) (- 1, 1) (b) [0 , 2]
Remarks
(i) When y is minimum at two or more points, the smallest value (c) [ 2, 3 ] (d) é 2 , 2ù
êë 3 úû
amongst them is taken.
(ii) When y is maximum at two or more points, the largest value Sol. Eliminating z,
amongst them is taken.
x 2 + y 2 + ( 4 - ( x + y ))2 = 6

y Example 54 Find the range of Þ y 2 + ( x - 4 )y + x 2 - 4 x + 5 = 0

log 3{log 1/ 2 ( x 2 + 4 x + 4 )}. Þ D ³0


Þ ( x - 4 ) 2 - 4 ( x 2 - 4 x + 5) ³ 0
Sol. Firstly, finding domain
log 3 {log1/ 2 ( x 2 + 4 x + 4 )} exists, if Þ 3x 2 - 8x + 4 £ 0
Þ ( x - 2)(3x - 2) £ 0
log1/ 2 ( x 2 + 4 x + 4 ) > 0
+ – +
0
æ1ö
Þ x + 4x + 4 < ç ÷
2 2/3 2
è2ø
\ x Î[2 / 3, 2]
[using loga x > b Þ x < ab , if 0 < a < 1] Hence, (d) is the correct answer.
Þ x 2 + 4x + 4 < 1
y Example 57 The range of the function
Þ x 2 + 4x + 3 < 0 1 1
Þ ( x + 1) ( x + 3) < 0 Þ –3 < x < – 1 ...(i)
f (x ) = + is
| sin x | | cos x |
+ – +
(a) [ 2 2 , ¥) (b) ( 2 , 2 2) (c) (0 , 2 2) (d) (2 2 , 4)
–3 –1
1 1
Sol. f ( x ) = +
and x 2 + 4x + 4 > 0 Þ ( x + 2)2 > 0, | sin x | | cos x |
which is always true except for x = – 2. ...(ii) Using AM ³ GM, we get
From Eqs. (i) and (ii), we have 1 1
+ 1/ 2
x Î (– 3, – 2) È (– 2, – 1) | sin x | | cos x | æ 1 ö
³ç ÷
Thus, domain is (– 3, – 2) È (– 2, – 1) 2 è | sin x | | cos x | ø
Now, we find out the range. 1 1
Since, 0 < log1/ 2 ( x 2 + 4 x + 4 ) < ¥, " x Î domain y Þ + ³ 2 (2 | cosec 2x | )1/ 2
| sin x | | cos x |
Þ – ¥ < log 3 {log1/ 2 ( x 2 + 4 x + 4 )} < ¥ [where, | cosec 2x | ³ 1]
Thus, range (y ) is R. 1 1
Þ + ³2 2
| sin x | | cos x |
y Example 55 Range of the function
\ Range of f ( x ) is [2 2, ¥ ).
f ( x ) = (cos -1 | 1 - x 2 | ) is
Hence, (a) is the correct answer.
é pù é pù æp ö
(a) ê0 , (b) ê0 , (c) (0 , p) (d) ç , p ÷
ë 2 úû ë 3 úû è2 ø y Example 58 If z = x + iy and x 2 + y 2 = 16, then the
Sol. Here, f ( x ) = cos -1 | 1 - x 2 | would exists, only when range of || x | - | y || is
|1 - x2 | £ 1 (a) [0 , 4 ] (b) [0 , 2]
Þ -1 £ 1 - x £ 1 Þ 2 ³ x ³ 0
2 2 (c) [ 2, 4 ] (d) None of these
Chap 03 Functions 121

Sol. Let x = 4 cos q , y = 4 sin q , then Sol. When x = n - 1, f ( x ) = (n - 1)2 - (n - 1)2 = 0


| 4 | cos q | - 4 | sin q || = 4 || cos q | - | sin q || When n - 1 < x < n,
= 4 1 - 2 | cos q || sin q | [x ] = n - 1
= 4 1 - | sin 2q | and ( n - 1) 2 £ [ x 2 ] £ n 2 - 1
\ Range is [0, 4 ]. Hence, (a) is the correct answer. \ 0 £ [ x 2 ] - [ x ]2 £ n 2 - 1 - (n - 1)2
Þ 0 £ f ( x ) £ 2n - 2, but f ( x ) has to be an integer.
y Example 59 The range of
1 x+1 The set of values of f ( x ) is { 0, 1, 2, …, 2n - 2 }.
f ( x ) = (sin -1 x + tan -1 x ) + 2 is Hence, (d) is the correct answer.
p x + 2x + 5
é 3 1ù é 5 3ù y Example 62 Range of the function
(a) ê - , ú (b) ê - ,
ë 4 5û ë 4 4 úû
f ( x ) = | sin -1 | sin x || - cos -1 | cos x |, is
é 3 5ù é 3 ù
(c) ê - , ú (d) ê - ,1 é
ë 4 4û ë 4 úû pù
(a) {0} (b) ê0 , ú (c) [0 , p ] (d) None of these
1 1 ë 2û
Sol. Here, f ( x ) = (sin -1 x + tan -1 x ) +
p ( x + 1) +
4 Sol. We know that, | sin -1| sin x || = cos -1 | cos x | ,
( x + 1) " x Î domain
= g ( x ) + h ( x ),
where domain of g ( x ) is [ -1, 1] \ f ( x ) = | sin -1 | sin x || - cos -1 | cos x | = 0, " x Î domain
3 \ Range of f ( x ) is {0}.
\ Maximum value of g ( x ) = g (1) =
4 Hence, (a) is the correct answer.
3
and minimum value of g ( x ) = g ( -1) = -
4 y Example 63 The number of values of y in [ -2p, 2p ]
4
Also, maximum value of h ( x ) occurs, when ( x + 1) +
is minimum at x = 1.
( x + 1) satisfying the equation | sin 2x | + | cos 2x | = | sin y | is
é 3 ù (a) 3 (b) 4 (c) 5 (d) 6
Þ Range of f ( x ) is ê - , 1ú. Sol. Here, 1 £ | sin 2x | + | cos 2x | £ 2 and | sin y | £ 1.
ë 4 û
Hence, (d) is the correct answer. So, solution is possible only when | sin y | = 1.
p 3p
y Example 60 The range of the function Þ sin y = ± 1 Þ y = ± , ±
2 2
f ( x ) = sin 2 x - 5 sin x - 6 is \ Number of values of y is 4.
é 49 ù Hence, (b) is the correct answer.
(a) [ -10 , 0 ] (b) [-1, 1] (c) [0 , p ] (d) ê - ,0
ë 4 úû
y Example 64 Let f ( x ) = cot -1 ( x 2 - 4 x + 5), then range
Sol. Here, f ( x ) = sin 2 x - 5 sin x - 6
æ 25 ö 25 of f ( x ) is equal to
= çsin 2 x - 5 sin x + ÷ - 6 - æ pö æ pù
è 4ø 4 (a) ç0 , ÷ (b) ç0 , ú
2 è 2ø è 4û
æ 5ö 49
= çsin x - ÷ - …(i) é pö
è 2ø 4 (c) ê0 , ÷ (d) None of these
2 ë 4ø
9 æ 5ö 49
where £ çsin x - ÷ £ …(ii) Sol. Here, x 2 - 4 x + 5 Þ ( x - 2)2 + 1 ³ 1
4 è 2ø 4
\ 1 £ x 2 - 4x + 5 < ¥
From Eqs. (i) and (ii), we get -10 £ f ( x ) £ 0
p
Þ Range of f ( x ) is [ -10, 0]. Þ 0 < cot -1 ( x 2 - 4 x + 5) £
4
Hence, (a) is the correct answer.
[using, x 1 < x 2 Þ cot -1 x 1 > cot -1 x 2 ,
y Example 61 If f ( x ) = [ x ] - [ x ] , where [×] denotes
2 2
since cot -1 x is decreasing]
the greatest integer function and x Î [0, n ], n Î N , then
æ pù
the number of elements in the range of f ( x ) is Þ Range of f ( x ) is ç0, ú.
è 4û
(a) (2n + 1) (b) 4n - 3 (c) 3n - 3 (d) 2n - 1
Hence, (b) is the correct answer.
122 Textbook of Differential Calculus

1
To Find Range for Case II If 0 < y £ 1. Put z =
y
Rational Expressions Þ 1£z < ¥
ax + bx + c
2
Now, from Eq. (i), we get
Let f ( x ) =
px 2 + qx + r z 2 + 4z - 4a ³ 0 holds for 1 £ z < ¥

Step 1 Write the given function as a equation Þ (z + 2)2 - 4 - 4a ³ 0 holds for 1 £ z < ¥
ax 2 + bx + c Consider, ( z + 2) 2 - 4 - 4a ³ 0
y=
px 2 + qx + r Þ ( z + 2) 2 ³ 4 + 4a
Step 2 Rewrite the above equation for x in standard form Þ z + 2 ³ 4 + 4a ; 4 + 4a ³ 0
x 2 ( py - a ) + x (qy - b ) + ry - c = 0 a ³ -1 ...(ii)
Step 3 Find the discriminant to the above equation but z ³1
\ 9 ³ 4 + 4a
D = (qy - b ) 2 - 4 ( py - a )(ry - c )
Þ 5 ³ 4a [as (u + 2)2 ³ 9 for u = 1]
D = q 2 y 2 - 2qyb + b 2 - 4 [ pry 2 - ( pc + ar )y + ac ]
5
Þ a£ ,a ¹ - 1 ...(iii)
D = y 2 (q 2 - 4 pr ) + ( 4 pc + 4ar - 2bq )y + b 2 - 4ac 4
-B ± B 2 - 4 AC æ 5ù
Step 4 Find y 1 , y 2 = \ From Eqs. (i), (ii) and (iii), we get a Î ç -1, ú.
2A è 4û
where, A = q 2 - 4 pr , B = 4 pc + 4ar - 2bq , C = b 2 - 4ac .
y Example 67 Find the range of the function
Step 5 Let y 1 < y 2 , then range of f ( x ) is [y 1 , y 2 ] if A < 0 and sin 2 x + sin x - 1
A > 0, then range of f ( x ) is R - (y 1 , y 2 ). f (x ) = .
sin 2 x - sin x + 2
x 2 + 14 x + 9
y Example 65 Find the range of f ( x ) = , sin 2 x + sin x - 1
x 2 + 2x + 3 Sol. Let y=
sin 2 x - sin x + 2
where x ÎR . t2 + t - 1
Let t = sin x Þ -1 £ t £ 1 and y =
Sol. Here, A = 4 - 12 = - 8, B = 12 + 36 - 56 = - 8, C = 160 t2 - t + 2
- B ± B 2 - 4 AC -1 ± 1 + 80 Þ (y - 1) t 2 - (y + 1) t + (2y + 1) = 0 …(i)
Now, = = - 5, 4 and
2A -2 3 - 2 11 3 + 2 11
Since t is real, £y £
here A < 0 7 7
\ Range is [ - 5, 4 ] Case I If both roots of Eq. (i) are greater than 1, i.e. t 1 > 1
and t 2 > 1.
y Example 66 For what real values of a does the
Þ t1 + t 2 > 2
x+1
range of f ( x ) = contains the interval [0, 1] ? and (t 1 - 1)(t 2 - 1) > 0
a+ x2 y +1
Þ >2
x +1 y -1
Sol. Let y =
a + x2 2y + 1 y + 1
and - +1>0
Þ y (a + x 2 ) = x + 1 y -1 y -1
Þ yx 2 - x + (ay - 1) = 0 has real roots for every y Î[0, 1]. Þ 1<y <3
\ 1 - 4y (ay - 1) ³ 0 1
and y > 1 or y <
2
Þ 1 - 4ay 2 + 4y ³ 0 holds for 0 £ y £ 1 …(i)
\ y Î(1, 3) …(ii)
Case I If y = 0
Case II If t 1 < - 1 and t 2 < - 1
Here, y = 0 is assumed at x = - 1 for any a ¹ - 1.
Þ t 1 + t 2 < - 2 and (t 1 + 1)(t 2 + 1) > 0
x +1
For a = - 1, y = 2 is undefined for x = - 1. y +1 2y + 1 y + 1
x -1 Þ + 2 < 0 and + +1>0
y -1 y -1 y -1
Chap 03 Functions 123

1 1 2y + 1 y + 1 2y + 1 y + 1
Þ < y < 1 and y > 1 or y < - Þ + + 1 < 0 and - +1<0
3 4 y -1 y -1 y -1 y -1
Þ y Îf …(iii) 1 1 1
Þ - < y < 1 and <y <1 Þ <y <1
Case III If t 1 < - 1 and t 2 > - 1, t 1 < 1 and t 2 > 1 4 2 2
Þ (t 1 + 1)(t 2 + 1) < 0 and (t 1 - 1)(t 2 - 1) < 0 é 3 - 2 11 1 ù
\ Rf = ê , ú È {1}
ë 7 2û

Exercise for Session 5


n Directions (Q. Nos. 1 to 25) Find the range of the following.
x
1. f (x ) = 9 – x 2 2. f (x ) =
1+ x 2
3. f ( x ) = sin x + cos x + 3 4. f ( x ) = | x – 1| + | x – 2 |, –1 £ x £ 3
x2 –2
5. f ( x ) = log3 (5 + 4x – x 2 ) 6. f (x ) =
x2 –3
x 2 + 2x + 3
7. f (x ) = 8. f ( x ) = | x – 1| + | x – 2 | + | x - 3 |
x
9. f ( x ) = log [ x –1] sin x , where [×] denotes the greatest integer function.
|x |
10. f ( x ) = cos –1 log [ x ] , where [×] denotes the greatest integer function.
x

11. f (x ) = [sin 2x ] – [cos 2x ], where [×] denotes the greatest integer function.

é 1ù é 1ù
12. f ( x ) = sin–1ê x 2 + ú + cos –1ê x 2 – ú , where [×] denotes the greatest integer function.
ë 2 û ë 2 û
æ x2 ö
13. f ( x ) = sin–1( x 2 + x + 1) 14. f ( x ) = cos –1ç ÷
è 1+ x 2 ø
x –1
15. f ( x ) = log (cos (sin x )) 16. f (x ) =
x – 2x + 3
2

sin x cos x tan ( p [ x 2 – x ])


17. f (x ) = – 18. f (x ) =
1+ tan x 2
1+ cot x
2 1+ sin (cos x )
x
e
19. f (x ) = ,x ³0
[ x + 1]

20. f ( x ) = [|sin x | + |cos x | ] , where [×] denotes the greatest integer function.

pæ p ö
21. f ( x ) = - x 2 + 4 x - 3 + sin çsin ( x - 1)÷
2è 2 ø

22. Find the image of the following sets under the mapping f ( x ) = x 4 - 8x 3 + 22x 2 - 24x + 10 (i) ( - ¥, 1) (ii) [1, 2].
é 1ù
23. Find the domain and range of f ( x ) = log êcos | x | + ú , where [×] denotes the greatest integer function.
ë 2û

24. Find the domain and range of f ( x ) = sin-1 (log [ x ]) + log (sin-1 [ x ]), where [×] denotes the greatest integer function.

25. Find the domain and range of f ( x ) = [log (sin-1 x 2 + 3x + 2 )], where [×] denotes the greatest integer function.
Session 6
Odd and Even Functions
Odd and Even Functions Properties of Odd and Even Functions
(i) Product of two odd functions or two even functions is an even
Odd Functions function.
A function f ( x ) is said to be an odd function, if (ii) Product of odd and even function is an odd function.
f ( - x ) = - f ( x ) for all x . Graph of an odd function is (iii) Every function y = f ( x ) can be expressed as the sum of an
symmetrical in opposite quadrants, i.e. the curve in first even and odd function.
quadrant is identical to the curve in the third quadrant and (iv) The derivative of an odd function is an even function and
the curve in second quadrant is identical to the curve in derivative of an even function is an odd function.
fourth quadrant. Some graphs which are symmetrical in (v) A function which is even or odd, when squared becomes an
opposite quadrants (or about origin) are even function.
(vi) The only function which is both even and odd is f ( x ) = 0, i.e.
Y Y zero function.
y=x y = x3
y Example 68 If f is an even function, then find the
X′ X X′ X real values of x satisfying the equation
O O
æ x + 1ö
f (x ) = f ç ÷. [IIT JEE 1996, 2001]
è x + 2ø
Y′ Y′
Sol. Since, f ( x ) is even, so f (–x ) = f ( x )
Figure 3.29 Figure 3.30
x +1
Thus, x=
Y x +2
y = sin x x +1
or –x =
x +2
X′ O X Þ x 2 + 2x = x + 1
or –x 2 – 2 x = x + 1
Y′
Þ x2 + x –1 = 0
Figure 3.31
or –x 2 – 3 x – 1 = 0
Even Functions
–1 ± 5
A function f ( x ) is said to be an even, if f (–x ) = f ( x ) for all Þ x=
2
x . The graph is always symmetrical about Y -axis, i.e. the
–3 ± 5
graph on left hand side of Y-axis is the mirror image of the or x=
curve on its right hand side. 2
ì– 1 + 5 – 1 – 5 – 3 + 5 – 3 – 5ü
Some graphs which are symmetrical about Y -axis are Thus, x Îí , , , ý
î 2 2 2 2 þ
Y Y
y=|x| y Example 69 Find out whether the given function
y=x2
is even, odd or neither even nor odd,
135° ì x|x| , x £ –1
ï
X′ X X′
45° where f ( x ) = í[1 + x ] + [1 – x ] , –1 < x < 1
X
O O ï –x| x| , x ³1
î
Y′ Y′ where | | and [ ] represent the modulus and greatest
Figure 3.32 Figure 3.33 integer functions.
Chap 03 Functions 125

Sol. The given function can be written as y Example 70 Find whether the given function is even
ì –x 2 , x £ –1 x (sin x + tan x )
ï
f ( x ) = í2 + [ x ] + [–x ] , –1 < x < 1 or odd function, where f ( x ) = , when
é x + πù 1
ï –x 2 , x ³1 ê π ú–2
î ë û
[using definition of modulus function and the x ¹ n p, where [ × ] denotes the greatest integer function.
properties of greatest integer functions] x (sin x + tan x ) x (sin x + tan x )
Sol. f ( x ) = =
ì –x 2 , x £ –1 éx + pù 1 éx ù 1
ï êë p úû 2 – êë p úû + 1 – 2
ïï2 – 1 + 0 , –1 < x < 0
Þ f (x ) = í 2 , x =0 x (sin x + tan x )
ï2 + 0 – 1 f (x ) =
, 0< x <1 éx ù
ï
ïî –x 2 , x ³1 êë p úû + 0.5

ì–x 2 –x (sin (–x ) + tan(–x ))


, x £ –1 Now, f (–x ) =
ï é xù
ïï 1 , –1 < x < 0 êë– p úû + 0.5
f (x ) = í 2 , x =0
ï 1 ì x (sin x + tan x )
, 0< x <1 ïï , x ¹ np
ï 2 f (–x ) = í –1 – é ù + 0.5
x
ïî–x x ³1 Þ
, êë p úû
ï
which is clearly even as ïî 0 , x = np
f (–x ) = f ( x ). x (sin x + tan x )
if So, f (–x ) = – or [Q x ¹ np ]
éx ù
Thus, f ( x ) is even.
êë p úû + 0 . 5

\ f (–x ) = – f ( x ). Hence, f ( x ) is an odd function (if x ¹ n p).

Exercise for Session 6


1. Determine whether the following functions are even or odd.
æ a x + 1ö
(i) f ( x ) = log ( x + 1+ x 2 ) (ii) f ( x ) = x ç x ÷
è a – 1ø

(iii) f ( x ) = sin x + cos x (iv) f ( x ) = x 2 –| x |


æ 1– x ö
(v) f ( x ) = log ç ÷ (vi) f ( x ) = {(sgn x )sg n x } n; n is an odd integer.
è 1+ x ø

(vii) f ( x ) = sgn ( x ) + x 2 (viii) f ( x + y ) + f ( x – y ) = 2f ( x ) × f ( y ); where f (0) ¹ 0 and x , y Î R.

2. Determine whether function; f ( x ) = (–1)[ x ] is even, odd or neither of two (where [×] denotes the greatest integer
function).
ì x | x |, 0 £ x < 1
3. A function defined for all real numbers is defined for x ³ 0 as follows f ( x ) = í .
î2 x , x ³1
How is f defined for x £ 0, if (i) f is even? (ii) f is odd?
2x (sin x + tan x )
4. Show that the function f ( x ) = is symmetric about origin.
é x + 21p ù
2ê úû - 41
ë p
éx2ù
5. If f : [ -20, 20] ® R defined by f ( x ) = ê ú sin x + cos x is an even function, find the set of values of ‘a’
ë a û
(where [ × ] denotes the greatest integer function).
Session 7
Periodic Functions
Periodic Functions Graphically f ( x ) = x – [ x ] = { x }
Definition A function f ( x ) is said to be periodic Y
function, if there exists a positive real number, T such that
1
f ( x + T ) = f ( x ), " x Î Dom ( f ). Then, f ( x ) is periodic y=1
with period T, where T is least positive value.
X
Graphically If the graph repeats at fixed interval, the –3 –2 –1 O 1 2 3
function is said to be periodic and its period is the width
of that interval.
Clearly, from the given graph, the function repeats itself at
y Example 71 Prove that sin x is periodic and an interval of 1 unit. Thus, period of f ( x ) = 1.
find its period.
Sol. Let f ( x ) = sin x and T > 0, then f ( x ) is periodic, if
Remark
For those functions whose periods are not deducted by graphs,
f ( x + T ) = f ( x ). they can be judged by inspection method.
Þ sin ( x + T ) = sin ( x ), " x Î R
Þ T = 2p, 4 p, 6p, .... y Example 73 Let f ( x ) be periodic and k be a positive
But period of f ( x ) is smallest positive real number. real number such that f ( x + k ) + f ( x ) = 0 for all x ÎR.
Thus, period of f ( x ) is 2p. Prove that f ( x ) is a periodic with period 2k.
Aliter f ( x ) = sin x could be expressed graphically as Sol. We have,
shown in figure. f ( x + k ) + f ( x ) = 0, " x Î R
Þ f ( x + k ) = – f ( x ), " x Î R , put x = x + k
π
1 2 ,1 y=1 Þ f ( x + 2k ) = – f ( x + k ), " x Î R
[as f ( x + k ) = – f ( x ) ]
X′ X Þ f ( x + 2k ) = f ( x ), " x Î R
O π 2π
which clearly shows that f ( x ) is periodic with period 2k.
π ,–1 –1 3π,–1 y=–1
2 2 Some Standard Results on
Periodic Functions
Here, graph repeats at an interval of 2p. Functions Periods
Thus, f ( x ) is periodic with period 2p. n
(i) sin x, cos x n
p, if n is even.
n n 2p , if n is odd or fraction.
y Example 72 Prove that f ( x ) = x – [ x ] is periodic sec x, cosec x

function. Also, find its period. (ii) tannx , cotn x p, n is even or odd.
Sol. Let T > 0. (iii) |sin x |, | cos x |, | tan x | p
Then, f ( x + T ) = f ( x ), " x Î R | cot x |, | sec x |, | cosec x |
Þ ( x + T ) – [ x + T ] = x – [ x ], " x Î R (iv) x – [ x ] 1
Þ [x + T ] – [x ] = T , " x Î R (v) Algebraic functions Period doesn’t exist.
Þ T = 1, 2, 3, 4, K e.g. x , x , x + 5, ... etc.
2 3

[since, subtraction of two integers]


(vi) f ( x ) = constant Periodic with no fundamental
The smallest value of T satisfying f ( x + T ) = f ( x ) is 1. period.
Thus, it is periodic with period 1.
Chap 03 Functions 127

y Example 74 Find periods for Periodicity of Constant Function


(i) cos 4 x . (ii) sin 3 x . (iii) cos x . (iv) cos x.
The LCM rule is not applicable, if function reduces to
Sol. (i) cos 4 x has a period p as n is even. constant.
(ii) sin 3 x has a period 2p as n is odd.
e.g. f ( x ) = sin2 x + cos 2 x .
(iii) cos x is not periodic, as for no value of T ,
f ( x + T ) = f ( x ) Þ cos x + T = cos ( x ) Since, period of sin2 x and cos 2 x are p.
Thus, there exists no value of T for which f ( x + T ) = f ( x ). 1 p
\ Period of f ( x ) = LCM { p, p } = , which is not correct.
Hence, cos x is not periodic. 2 2
(iv) f ( x ) = cos x has the period 2p as n is in fraction. Whereas, sin2 x + cos 2 x = 1 is a constant function and
Aliter f ( x + T ) = f ( x ) Þ cos ( x + T ) = cos ( x ) period is undetermined.
Þ T = 2p, 4 p, ...
But T is the least positive value, hence f ( x ) is periodic y Example 75 Find the period, if f ( x ) = sin x + { x },
with period 2p. where { x } is fractional part of x .
Sol. Here, sin x is periodic with period 2p and { x } is periodic
Properties of Periodic Functions with period 1. Thus, LCM of 2p and 1 Þ Does not exist.
(i) If f ( x ) is periodic with period T, then Hence, f ( x ) is not periodic.
(a) c × f ( x ) is periodic with period T
y Example 76 Find period of f ( x ) = tan 3x + sin æç ö÷ .
x
(b) f ( x + c ) is periodic with period T
è 3ø
(c) f ( x ) ± c is periodic with period T, where c is any constant.
We know, sin x has period 2p. p
Sol. Period for tan 3x is .
Then, f ( x ) = 5 (sin x ) + 4 is also periodic with period 2p. 3
i.e. ‘‘If constant is added, subtracted, multiplied or divided in x 3
Period for sin is 2p ´ =| 6p |
a periodic function, its period remains the same.’’ 3 1
T
(ii) If f ( x ) is periodic with period T, then kf ( cx + d ) has period , p
| c| Thus, LCM of
i.e. period is only affected by coefficient of x, 3
where, k, c, d Î constant. 6p 6p
ì æ p öü 2p and Þ
We know, f ( x ) = í 7 sin ç 2x + ÷ý – 12 has the period = p ; as 1 1
î è 9 ø þ | 2|
Hence, f ( x ) is periodic with period 6p.
sin x is periodic with period 2p.
(iii) If f 1 ( x ), f 2 ( x ) are periodic functions with periods T1, T2 y Example 77 Find the period of
respectively, then h ( x ) = f 1 ( x ) + f 2 ( x ) has period x x x
ìLCM of { T1, T2 }, if h(x ) is not an even function. f ( x ) = sin x + tan + sin 2 + tan 3 + K
ï 2 2 2
OR
ï x x
=í1 + sin n – 1 + tan n .
ï 2 LCM of { T1, T2 }, if f1(x ) and f 2 (x ) are complementary 2 2
ï æ xö
îpair-wise comparable functions. Sol. We have, Period of çsin x + tan ÷ is 2p,
è 2ø
While taking LCM we should always remember
æ a c e ö LCM of ( a, c, e ) æ x xö
çsin 2 + tan 3 ÷ is 2 p,
3
(a) LCM of ç , , ÷ = è
è b d f ø HCF of (b, d, f ) 2 2 ø
LCM of ( 2p , p , p ) 2p ...........................................
e.g. LCM of = =
HCF of ( 3, 6, 12) 3 ...........................................
æ 2p p p ö 2p æ x xö n
\ LCM of ç , , ÷ = çsin n – 1 + tan n ÷ is 2 p.
è 3 6 12 ø 3 è 2 2 ø
(b) LCM of rational with rational is possible. Thus, LCM of {2p, 23 p, ..., 2n p } = 2n p.
LCM of irrational with irrational is possible.
But LCM of rational and irrational is not possible. Hence, the period of f ( x ) is 2n p.
e.g. LCM of ( 2p , 1, 6 p ) is not possible, as 2p , 6 p Îirrational
and 1 Îrational.
128 Textbook of Differential Calculus

y Example 78 Find the period of f ( x ) = |sin x | + |cos x|. y Example 83 Period of the function
Sol. |sin x | has period p and | cos x | has period p. f ( x ) = sin (sin ( px )) + e { 3x } , where {.} denotes the
Here, f ( x ) is an even function and sin x , cos x are fractional part of x is
complementary. (a) 1 (b) 2
1 p (c) 3 (d) None of these
Thus, period of f ( x ) = { LCM of p and p } =
2 2 2p
Sol. As, sin ( px ) has period = =2
p p
Thus, period for f ( x ) is ×
2 \ sin (sin ( px )) has period 2
y Example 79 Find the period of 1
and e { 3 x } has period ×
f ( x ) = sin 4 x + cos 4 x . 3
ì 1ü
Sol. sin 4 x and cos 4 x both has a period p. [as n is even] \ Period of f ( x ) = sin (sin ( px )) + e { 3 x } is LCM of í2, ý = 2
î 3þ
But f ( x ) is an even function and sin x and cos x are
complementary. Hence, (b) is the correct answer.
1 p
Hence, f ( x ) has period = {LCM of p, p } = y Example 84 sin ax + cos ax and | cos x | + | sin x | are
2 2
p
periodic functions of same fundamental period, if ‘a ’
Thus, period of f ( x ) is . equals
2
(a) 0 (b) 1 (c) 2 (d) 4
y Example 80 Find the period of p
Sol. Fundamental period of | sin x | + | cos x | is ×
f ( x ) = cos (cos x ) + cos (sin x ) . 2
Sol. Here, cos (cos x ) has period p; as it is even, also 2p
Fundamental period of (sin ax + cos ax ) is ×
cos (sin x ) has period p; as it is even. a
1 \ a=4
Thus, period of f ( x ) = {LCM of p and p}
2 Hence, (d) is the correct answer.
p
Hence, period of f ( x ) = y Example 85 Let f ( x ) = sin x + cos ( 4 - a 2 ) x . Then,
2
the integral values of ‘a’ for which f ( x ) is a periodic
y Example 81 Find the period of f ( x ) = cos –1 (cos x ) . function, are given by
Sol. Here, f ( x ) = cos –1(cos x ); f ( x + T ) = f ( x ) (a) {2, - 2} (b) (-2, 2]
(c) [ -2, 2] (d) None of these
Þ cos –1 {cos ( x + T )} = cos –1 {cos ( x )}
Sol. f ( x ) will be periodic, if 4 - a 2 is a rational which is
Þ T = 2p, 4 p, 6p,...
only possible when ( 4 - a ) is a perfect square.
2
But, T is the least positive value. Hence, T = 2p or period is 2p.
Aliter f ( x ) = cos (cos x ) has period 2p, since cos x has
–1 Þ a = 0, 2, - 2 or a Î { -2, 0, 2}
period 2p. Hence, (d) is the correct answer.
(i.e. In composition of function ( fog ) or ( gof ) are periodic, if
ì -1 + sin K 1 px , x is rational.
g ( x ) and f ( x ) are periodic, respectively.) y Example 86 Let f ( x ) = í
î1 + cos K 2 px , x is irrational.
y Example 82 The period of
If f ( x ) is a periodic function, then
f ( x ) = cos (| sin x | - | cos x | ) is
(a) p (b) 2p (a) either K 1 , K 2 Îrational or K 1 , K 2 Îirrational
p (b) K 1 , K 2 Î rational only
(c) (d) None of these
2 (c) K 1 , K 2 Î irrational only
Sol. As, cos q is even and | sin x | - | cos x | has the period p. K
(d) K 1 , K 2 Î irrational such that 1 is rational
p K2
\ cos (| sin x | - | cos x | ) has period .
2 Sol. Range of -1 + sin K 1px is [ -2, 0] and range of
(i.e. Half the period of g ( x ), if f ( x ) is even in fog). 1 + cos K 2 px is [0, 2].
Hence, (c) is the correct answer. Þ If g ( x ) = - 1 + sin K 1px
Chap 03 Functions 129

Þ g ( x + T1 ) = - 1 + sin K 1px , where T1 is a period of 1 1


\ >| 1 or >| 1
1 + sin K 1 px 2+m 3+m
Þ T1 is rational Þ m Ï ( -2, - 1) È ( -3, - 2)
Þ Period of f ( x ) is rational. Hence, (c) is the correct answer.
Þ K 1 and K 2 are rational.
y Example 88 Let f ( x ) be a periodic function with
Hence, (b) is the correct answer.
æ 2ö x
px period 3 and f ç - ÷ = 7 and g ( x ) = ò f (t + n ) dt,
y Example 87 If f ( x ) = tan 2 è 3ø 0
n - 5n + 82
æ ö
7
where n = 3K , K Î N . Then, g ¢ ç ÷ is equal to
+ cot (n + m )px ; (n Î N , m Î Q ) is a periodic function è 3ø
with 2 as its fundamental period, then m can’t 2
belong to (a) - (b) 7
3
(a) (- ¥ , - 2) È (-1, ¥) (b) (- ¥ , - 3) È (-2, ¥) 7
(c) -7 (d)
æ 5ö æ 5 ö 3
(c) (-2, - 1) È (-3 , - 2) (d) ç - 3 , - ÷ È ç - , - 2÷
è 2ø è 2 ø Sol. g ¢( x ) = f ( x + n ) = f ( x + (n - 3) + 3)
Sol. Period is LCM of n 2 - 5n + 8 and
1
× = f ( x + n - 3)
n +m .....................
Þ n 2 - 5n + 8 = 1 or n 2 - 5n + 8 = 2 .....................
= f ( x + 3) = f ( x )
Þ n 2 - 5n + 7 = 0 or n 2 - 5n + 6 = 0
Þ g ¢( x ) = f ( x )
Since, n Î N \ n = 2, 3
æ7 ö æ7 ö æ 2 ö æ 2ö
æ 1 ö 1 Þ g ¢ ç ÷ = f ç ÷ = f ç - + 3÷ = f ç - ÷ = 7
and ç ÷ K 1 = 2, (K 1 Î I ) Þ >| 1 è3ø è3ø è 3 ø è 3ø
èn + m ø n +m
Hence, (b) is the correct answer.

Exercise for Session 7


1. Find the periods of following functions.
1 ì |sin x | |cos x |ü
(i) f ( x ) = [sin 3x ] + |cos 6x | (ii) f ( x ) = í + ý
2 î cos x sin x þ
4
px + x – [ x ] + cos2 px px px
(iii) f ( x ) = e cos (iv) f ( x ) = 3 sin + 4 cos
3 4
px px
(v) f ( x ) = cos 3x + sin 3px (vi) f ( x ) = sin – cos
n! (n + 1) !
(vii) f ( x ) = x – [ x – b ] (viii) f ( x ) = eln(sin x ) + tan3 x – cosec (3x – 5)

2. Find the period of the real-valued function satisfying f ( x ) + f ( x + 4) = f ( x + 2) + f ( x + 6).

3. Check whether the function defined by


f ( x + l ) = 1 + 2f ( x ) - f 2( x ), " x Î R is periodic. If yes, find its period.

4. Let f ( x + p ) = 1 + {2 - 3 f ( x ) + 3 (f ( x ))2 - (f ( x ))3}1/ 3, " x Î R, where p > 0, prove that f ( x ) is periodic.


99
5. Let f ( x ) be a function such that ; f ( x - 1) + f ( x + 1) = 3 f ( x ), " x Î R. If f (5) = 100, find å f (5 + 12r ) .
r =0
Session 8
Mapping of Functions
Mapping of Functions
a a p
As discussed earlier, a function exists only if, “to every
element in domain there exists unique image in the b q
p b
codomain”.
i.e. To every element of A there exists one and only one c c r
element of B.
This is written as f : A ® B and is read as f maps from Figure 3.35 Figure 3.36
A to B and this correspondence is denoted by y = f ( x ).
From definition, it follows that there may exist some Method to Check One-one Mapping
elements in B, which may not have any corresponding Method 1. Theoretically If f ( x ) = f (y ) Þ x = y , then
element in set A. f ( x ) is one-one.
But there should not be any x left (element of A) for which Method 2. Graphically A function is one-one iff no line
there is no element in set B. parallel to X-axis meets the graph of function at more
There are four types of mappings defined as than one point.

y Example 89 Let f : [ - p / 2, p / 2] ® [ -1, 1] where


1. One-one Mapping or Injective f ( x ) = sin x. Find whether f ( x ) is one-one or not.
or Monomorphic Sol. Here, f : [ - p / 2, p / 2] ® [ -1,1] indicates that
A function f : A ® B is said to be one-one mapping or é p pù
domain Îê– , ú and codomain Î[–1, 1] .
injective, if different elements of A have different images ë 2 2û
in B. Y
1
Thus, no two elements of set A can have the same f
image.
–π/2
Verbal Description Let us consider set A = {1, 3, 5 } and O
X
π/2
B = {3, 7, 11, 15 }, where f : A ® B and f ( x ) = 2 x + 1, then
here every element in domain possess distinct images in
–1
codomain.
Thus, the graph of f ( x ) = sin x should be plotted in
1 3
[ -p / 2, p / 2] .
3 7 Which is clearly not intersected at more than one point by
any straight line parallel to X -axis.
5 11 Thus, f ( x ) is one-one.
15
A B
Method 3. By Calculus For checking whether f ( x ) is
one-one, find whether function is only increasing or only
Figure 3.34
decreasing in its domain. If yes, then one-one.
Thus, f ( x ) is one-one or injective. i.e. if f ¢ ( x ) ³ 0, " x Î domain
From above definition, following mappings are not or if f ¢ ( x ) £ 0, " x Î domain, then one-one.
one-one.
Remark
(i) f : A ® B (ii) f : A ® B Students are advised to use the graphical or calculus method for
finding one-one.
Chap 03 Functions 131

Verbal Description Let f : A ® B and g : X ® Y be two


Number of One-one Mapping functions represented by
If A and B are finite sets having m and n elements, then
f :A B g : X Y
number of one-one function from A to B.
f : A B x1 y1
x1 y1
x1 y1 x2 y2 x2 y2
x2 y2
x3 y3 x3 y3
x4
x4 y4
xm yn
Figure 3.38 Figure 3.39

Figure 3.37 Clearly, f and g both are many-one as there are two
elements x 3 , x 4 which correspond to the same image.
Here, x 1 can take n images, i.e. f ( x 3 ) = f ( x 4 ) = y 3 . Thus, many-one.
x 2 can take (n - 1) images,
Method to Check Many-one
x 3 can take (n - 2 ) images, They are same as for one-one because, if mapping is not
................................................... one-one it is many-one.
...................................................
x m can take (n - m + 1) images. y Example 91 Show f : R ® R defined by f ( x ) = x 2 + x
Thus, number of mapping Þ n (n - 1) (n - 2 ) ...(n - m + 1) for all x ÎR is many-one.
ì n Pm , if n ³ m Sol. By graph
Þ í Y
î 0 , if n < m
1
y Example 90 f ( x ) = x 3 + 3x 2 + 4 x + b sin x
+ c cos x , " x ÎR is a one-one function, the value of
b 2 + c 2 is –1 O
X

(a) ³ 1 (b) ³ 2 (c) £ 1 (d) None of these


Sol. Here, f ( x ) = x + 3x + 4 x + b sin x + c cos x
3 2
–1

Þ f ¢ ( x ) = 3x + 6x + 4 + b cos x - c sin x
2

Now, for f ( x ) to be one-one, the only possibility is


f ¢ ( x ) ³ 0, " x Î R f ( x ) = x 2 + x can be represented graphically, as shown in
figure,
Þ 3x 2 + 6x + 4 + b cos x - c sin x ³ 0, " x Î R
where the straight line parallel to X -axis meets the curve at
Þ 3x 2 + 6x + 4 ³ c sin x - b cos x , " x Î R two points (i.e. more than one point).
Þ 3x 2 + 6x + 4 ³ b 2 + c 2 , " x Î R Thus, it is not one-one it is many-one.
Aliter By calculus,
Þ b 2 + c 2 £ 3( x 2 + 2x + 1) + 1, " x Î R
f (x ) = x 2 + x
Þ b + c £ 3( x + 1) + 1, " x Î R
2 2 2
Þ f ¢ ( x ) = 2x + 1
Þ b + c £ 1, " x Î R Þ b + c £ 1 , " x Î R
2 2 2 2 where f ¢( x ) >0
1
Hence, (c) is the correct answer. if x >-
2
and f ¢( x ) <0
2. Many-one Mapping 1
if x<-
A mapping f : A ® B is said to be many-one function, if 2
two or more elements of set A have the same image in B. which shows f ( x ) is neither increasing nor decreasing
In other words; f : A ® B is a many-one function, if it is i.e. not monotonic. Hence, many-one.
not one-one function.
132 Textbook of Differential Calculus

3. Onto Mapping or Surjective = n m - {Number of injective functions}

A function f : A ® B is an onto function, if such that each ½ n ½


= n m -½ È Ai½, …(i)
element of B is the f image of atleast one element in A. It ½ i = 1 ½
is expressed as f : A ® B. n n

Here, range of f = codomain. where È Ai = å | Ai | - S Ai Ç A j + . . .


i =1 i, j
i =1
i.e. f (A) = B
= n × (n - 1)m - n C 2 (n - 2)m + . . . + ( -1)n - 1 × n C n - 1 (1)m …(ii)
Method to show onto or surjective
\ Number of onto functions
Find the range of y = f ( x ) and show range of f ( x ) =
codomain of f ( x ). = n m - {n × (n - 1)m - n C 2 (n - 2)m +. . .+ ( -1)n - 1 × n C n - 1 (1)m }
n

Remark = å ( -1)n - r n
Cr ×r m
r =1
If range = codomain, then f ( x ) is onto. Any polynomial of odd
degree has range all real numbers and is onto for f : R ® R.
p p æ x 2 - aö
y Example 93 If f :R ® é , ö÷ , f ( x ) = sin -1 ç 2 ÷
y Example 92 Show f :R ® R defined by êë 6 2 ø è x +1ø
f ( x ) = ( x - 1)( x - 2)( x - 3) is surjective but not injective is an onto function, the set of values of ‘a’ is
Sol. We have, ì 1ü é 1 ö
(a) í - ý (b) ê - , - 1÷
f ( x ) = ( x - 1)( x - 2)( x - 3) î 2þ ë 2 ø
f ( 1) = f ( 2) = f ( 3) = 0
(c) (-1, ¥) (d) None of these
Hence, it is not injective, it is many-one function.
Sol. Here, f ( x ) is onto.
Now, f ( x ) is a polynomial of degree 3, i.e. odd.
p æx2 - aö p 1 x2 - a
Hence, f ( x ) is surjective. \ £ sin -1 ç 2 ÷< Þ £ 2 <1
6 è x + 1ø 2 2 x +1
1 ( a + 1)
Number of Onto Functions Þ £1- 2 < 1, " x Î R Þ a + 1 > 0 Þ a Î ( -1, ¥ )
2 x +1
If A and B are two sets having m and n elements Hence, (c) is the correct answer.
respectively, such that 1 £ n £ m, then number of onto
functions from A to B is
4. Into Mapping
Coefficient of x m in m ! (e x – 1) n
A function f : A ® B is an into function, if there exists an
Þ Coefficient of x m in element in B having no pre-image in A.
n
m !{ n C 0 e nx – n C 1 e (n – 1 ) x + K + (–1) r C r e rx In other words, f : A ® B is into function, if it is not onto
n function (mapping).
+ K + (–1) n C n }
n y Example 94 Show f :R ® R defined by
= å (–1) n – r n C r r m f ( x ) = x 2 + 4 x + 5 is into.
r =1
Sol. We have, f (x ) = x 2 + 4x + 5
Or
f ( x ) = ( x + 2) 2 + 1
Consider the set of all possible functions from A to B,
Since, the codomain of f is R but the range of f is [1, ¥ ).
i.e. { f : A ® B }. Hence, f is into.
Now, let us define a subset Q such that
Ai = { f Î Q | i Ï Range of f }, i Î n One-one Onto Mapping or Bijective
n
i.e. È Ai = { f is injective } A function is one-one onto or bijective, if it is both
i =1 one-one and onto. A function is bijective if and only if
To find number of onto functions every possible image is mapped to by exactly one
argument.
= Total number of functions
- Number of injective functions The function f : A ® B is bijective iff for all y Î B, there is
a unique x Î A such that f ( x ) = y .
Chap 03 Functions 133

y Example 95 Let A = { x : - 1 £ x £ 1 } = B and a mapping (iv) f ( x ) = [ x ]


f : A ® B. For each of the following functions from A to Graph shows that f ( x ) is many-one, as the straight
line parallel to X -axis meets at more than one points.
B, find whether it is surjective or bijective.
Y
(i) f (x) = |x | (ii) f (x) = x |x | (iii) f (x)= x 3 2
px
(iv) f (x) = [ x ] (v) f (x) = sin 1
2 –2 –1
X′ X
O
Sol. (i) f ( x ) = | x | –1
1 3
Y
–2
Y′

Here, range f ( x ) Î { -1, 0, 1}


which shows into as range Í codomain.
X′ X Hence, many-one-into.
–1 O 1
px
Y′ (v) f ( x ) = sin
2
Which shows many-one, as the straight line is parallel
Graph shows f ( x ) is one-one and onto as range
to X -axis cuts at two points. Here, range for
= codomain.
f ( x )Î[0,1]. Which is clearly a subset of codomain.
Y
i.e. [0,1]Í [ -1,1]
1
Thus, into. Hence, function is many-one-into.
\ Neither injective nor surjective. X′ X
–1 O 1
(ii) f ( x ) = x | x |
The graph shows that f ( x ) is one-one, as the straight –1
line parallel to X -axis cuts only at one point. Y′
Y Therefore, f ( x ) is bijective.

1 y Example 96 The function f :R ® R defined as

X′
O
X f ( x ) = log e ( x 2 +1+ x + x 2 + 1 - x ) is
–1 1
(a) one-one and onto both
–1
(b) one-one but not onto
Y′ (c) onto but not one-one
(d) Neither one-one nor onto
Here, range f ( x )Î[ -1, 1]
Sol. (d) Here, f ( x ) = log æç x 2 + 1 - x ö÷
2
Thus, range = codomain x2 +1 + x +
2 è ø
Hence, onto. Therefore, f ( x ) is one-one onto or 2
= log æç x 2 + 1 + x + x 2 + 1 - x ö÷
(bijective). 1
2 è ø
(iii) f ( x )= x 3
= log é x 2 + 1 + x + x 2 + 1 - x + 2 ( x 2 + 1 )2 - x 2 ù
1
Graph shows f ( x ) is one-one onto (i.e. bijective). (as
2 êë úû
explained above)
1 é ù
Y = log ê2 x 2 + 1 + 2 x 2 + 1 - x 2 ú
2 ë û
1
1
= log[2 x 2 + 1 + 2]
X′ X 2
–1 O 1
Now, f ( - x ) = f ( x )
–1 \ f ( x ) is even function.
Y′ Hence, neither one-one nor onto.
134 Textbook of Differential Calculus

Quick Review y Example 97 If X = {1, 2, 3, 4, 5} and Y = {a, b , c , d , e , f }


Let X = { x 1, x 2 , x 3 , . . . , x n } [i.e. n elements] and f : X ® Y , find the total number of
and Y = { y 1, y 2 , y 3 ,. . . , yr } [i.e. r elements] (i) functions (ii) one to one functions
f:X Y (iii) many-one functions (iv) constant functions
x1 y1
f (x) (v) onto functions (vi) into functions
x2 y2
Sol. (i) Total number of functions = 65 = 7776
(ii) Total number of one to one functions
xn yr = 6C 5 × 5! = 6! = 720
Figure 3.40 (iii) Total number of many-one functions = 65 - 6! = 7056
(a) Total number of functions = r n (iv) Total number of constant functions = 6
= (Number of elements in codomain) number of elements in domain (v) Total number of onto functions = 0 (as r > n )
ì r C × n!, r ³ n (vi) Total number of into functions = 65 = 7776
(b) Total number of one to one functions = í n
î 0, r <n
y Example 98 Find the number of surjections from A
ìr n - rC n × n!, r ³n to B, where A = {1, 2, 3, 4 }, B = { a,b }.
(c) Total number of many-one functions = í [IIT JEE 2000]
î r n, r <n 2
Sol. Number of surjections from A to B = å (–1)2 – r 2 C r (r )4
(d) Total number of constant functions = r r =1
(e) Total number of onto functions = (–1)2 – 1 2 C 1(1)4 + (–1)2 – 2 2 C 2 (2)4 = - 2 + 16 = 14
ì r n - r C1 ( r - 1) n + r C2 ( r - 2) n - r C3 ( r - 3 ) n + . . ., r<n
ï Therefore, number of onto mappings from A to B = 14.
=í r !, r=n
ï 0, r>n
Aliter Total number of mapping from A to B is 24 of which
î two functions f ( x ) =a for all x ÎA and g ( x ) = b for all x ÎA
(f) Total number of into functions are not surjective.
ì r C ( r - 1) n - r C2 ( r - 2) n + r C3 ( r - 3 ) n - ... , r £ n
=í 1 Thus, total number of surjections from
î rn , r>n
A to B = 24 - 2 =14.

Exercise for Session 8


1. There are exactly two distinct linear functions, which map [ -1,1] onto [0, 3]. Find the point of intersection of the
two functions.
2. Let f be one-one function with domain {x , y , z } and range {1, 2, 3}. It is given that exactly one of the following
statement is true and remaining two are false.
f ( x ) = 1, f ( y ) ¹ 1, f (z ) ¹ 2. Determine f -1(1).
x -2
3. Let A = R - {3}, B = R - {1} and f : A ® B defined by f ( x ) = . Is ‘f ’ bijective? Give reasons.
x -3
x2
4. Let f : R ® R defined by f ( x ) = . Prove that f is neither injective nor surjective.
1+ x 2
x2
5. If the function f : R ® A, given by f ( x ) = is surjection, find A.
x +1
2

ex - e-| x |
6. If the function f : R ® A, given by f ( x ) = is surjection, find A.
ex + e | x |

7. Let f ( x ) = ax 3 + bx 2 + cx + d sin x . Find the condition that f ( x ) is always one-one function.


æ pö
8. Let f : X ® Y be a function defined by f ( x ) = a sin ç x + ÷ + b cos x + c. If f is both one-one and onto, find the
è 4ø
sets X and Y.
Session 9
Identical (or Equal) Functions

Identical (or Equal) Functions (xviii) f ( x ) =


1
1
, g( x ) =
x
1+ x
(NI)
Two functions f and g are said to be identical (or equal) 1+
x
functions, if -1
(xix) f ( x ) = e ln sec x
, g( x ) = sec -1 x (NI)
(i) the domain of f = the domain of g,
(ii) the range of f = the range of g, and Identical, if x Î ( - ¥, - 1] È [1, ¥)
(iii) f ( x ) = g ( x ), " x Î domain. (xx) F ( x ) = ( fog )( x ), G ( x ) = ( gof )( x ), where f ( x ) = e x ,
g( x ) = ln x (NI)
Examples of Equal or Identical Functions
(i) f ( x ) = ln x 2 , g( x ) = 2 ln x (NI) y Example 99 If f ( x ) = log x 2 25 and g ( x ) = log x 5,
1 then f ( x ) = g ( x ) holds, now find the interval for x.
f ( x ) = cosec x , g( x ) = (I)
sin x
Sol. Domain of f ÎR - { ±1, 0 }
(ii) f ( x ) = cot (cot -1 x ), g( x ) = x (I) Domain of g Î(0, ¥ ) - { 1 }
f ( x ) = tan x , g( x ) =
1
(NI) For f ( x ) = g ( x ),
cot x domain of f = domain of g.
(iii) f ( x ) = sin -1 (3 x - 4 x 3 ), g( x ) = 3 sin -1 x (NI) i.e., f ( x ) = g ( x ), if x Î(0, ¥ ) - {1}

(iv) f ( x ) = sgn ( x + 1), g( x ) = sin x + cos x


2 2 2
(I) y Example 100 Let A = { 1, 2 }, B = { 3, 6 } and f : A ® B
(v) f ( x ) = tan2 x × sin2 x , g( x ) = tan2 x - sin2 x (I) given by f ( x ) = x 2 + 2 and g : A ® B given by g ( x ) = 3x .
Find whether they equal or not.
(vi) f ( x ) = sec 2 x - tan2 x , g( x ) = 1 (NI)
Sol. f (1) = 3, f (2) = 6
1
(vii) f ( x ) = log x e, g( x ) = (I) g (1) = 3, g (2) = 6
log e x which shows f ( x ) and g ( x ) have same domains and range,
(viii) f ( x ) = tan (cot -1 x ), g( x ) = cot (tan -1 x ) (I) thus f = g .

(ix) f ( x ) = x 2 - 1, g( x ) = x - 1 × x + 1 (NI) y Example 101 Which pair of functions is identical?


(a) sin -1 (sin x) and sin (sin -1 x)
(x) f ( x ) = tan x × cot x , g( x ) = sin x × cosec x (NI)
x (b) log e e x , e log e x
(xi) f ( x ) = e ln e , g( x ) = e x (I)
(c) log e x 2 , 2 log e x
1 - cos 2 x
(xii) f ( x ) = , g( x ) = sin x (NI) (d) None of the above
2
é p pù
Sol. Here, (a) sin -1 (sin x ) is defined for x Î ê - , ú ,
(xiii) f ( x ) = x , g( x ) = ( x )
2 2
(NI) ë 2 2û
(xiv) f ( x ) = log ( x + 2 ) + log ( x - 3 ), while sin (sin -1 x ) is defined only for x Î [ -1, 1].

g( x ) = log ( x 2 - x - 6 ) (NI) (b) loge e x is defined for all x,

(xv) f ( x ) = x | x |, g( x ) = x 2 sgn x (I) while e log e x


is defined for x > 0.

x 2n - 1 (c) loge x is defined for all x Î R - {0}, while 2 log e x is


2

(xvi) f ( x ) = lim , g( x ) = sgn (| x | - 1) (I) defined for x > 0.


n®¥ x 2n + 1
\ None is identical.
(xvii) f ( x ) = sin (sin -1 x ), g( x ) = cos (cos -1 x ) (I) Hence, (d) is the correct answer.
Exercise for Session 9
n Directions (Q. Nos. 1 to 10) Which of the following are identical (or equal) functions?
2
1. f ( x ) = ln e x , g( x ) = eln x
1
2. f ( x ) = sec x , g( x ) =
cos x
p
3. f ( x ) = sec-1 x + cosec-1 x , g( x ) =
2
4. f ( x ) = cot 2 x × cos 2 x , g( x ) = cot 2 x - cos 2 x

5. f ( x ) = sgn (cot -1 x ), g( x ) = sgn ( x 2 - 4x + 5)


1
6. f ( x ) = loge x , g( x ) =
logx e

7. f ( x ) = 1 - x 2 , g( x ) = 1 - x × 1 + x
1
8. f (x ) = , g( x ) = x -2
|x |

9. f ( x ) = [{x }], g( x ) = {[ x ]} [Note that f ( x ) and g( x ) are constant functions]


-1
10. f ( x ) = eln cot x
, g( x ) = cot -1 x
Session 10
Composite Functions
Composite Functions q : I0 ® Q g (x ) =
1
x
Let us consider two functions, f : X ® Y1 and g : Y1 ® Y .
h: Q ® R h( x ) = e 1/ x
We define function h : X ® Y , such that,
(hog) of = ho (gof) = e 2x
h ( x ) = g ( f ( x )) = ( gof )( x ).
(iv) The composite of two bijections is a bijection, i.e. if f and g are two
g bijections such that gof is defined, then gof is also a bijection.
f
Proof f : A ® B and g : B ® C be two bijections. Then, gof exists
such that gof : A ® C .
We have to prove that gof is one-one and onto.
x f (x) g(f(x)) One-One Let a 1, a 2 Î A such that ( gof )( a 1 ) = ( gof )( a 2 ), then
( gof )( a 1 ) = ( gof ) ( a 2 ) Þ g[ f ( a 1 )] = g[ f ( a 2 )]
X Y Þ f (a1 ) = f (a2 ) [Q g is one-one]
Y1
Þ a1 = a2 [Q f is one-one]
h=gof \ gof is also one-one function.
Onto Let c Î C , then c Î C
Figure 3.41
Þ $ b Î B such that g(b ) = c [Q g is onto]
To obtain h ( x ), we first take f -image of an element x Î X and b Î B Þ $ a Î A such that f ( a ) = b [Q f is onto]
so that f ( x ) ÎY1 , which is the domain of g ( x ). Then, we Therefore, we see that
take g-image of f ( x ), i.e. g ( f ( x )) which would be an c Î C Þ $ a Î A such that ( gof )( a ) = g[ f ( a )] = g( g) = c
element of Y. i.e. Every element of C is the gof image of some element of A. As
such gof is an onto function. Hence, gof being one-one and
The diagram below shows the steps to be taken onto is a bijection.
(v) f is even, g is even Þfog is even function.
x f (x) f (x) g {f(x)} (vi) f is odd, g is odd Þ fog is odd function.
(vii) f is even, g is odd Þ fog is even function.
f g
(viii) f is odd, g is even Þ fog is even function.
Figure 3.42
y Example 102 Let f : A ® B and g :B ® C be
The function h defined in the diagram is called
composition of f and g, and is denoted by ( gof ). Clearly, functions and gof : A ® C . Which of the following
domain ( gof ) = {X : x Î domain ( f ), f ( x ) Î domain ( g )}.
statements is true?
(a) If gof is one-one, then f and g both are one-one
Similarly, we can write, ( fog )( x ) = f { g( x )}
(b) If gof is one-one, then f is one-one
and domain (fog) = { x : x Î domain ( g), g( x ) Î domain f }
(c) If gof is a bijection, then f is one-one and g is onto
In general, fog ¹ gof .
(d) If f and g are both one-one, then gof is one-one
Sol. (a) As shown gof is one-one, but g is many-one.
Properties of Composite Functions f g
(i) It should be noted that gof exists, iff the range of f Í domain of g. A B C
Similarly, fog exists; iff the range of gÍ domain of f . 3
a 4
(ii) Composite of functions is not commutative, i.e. gof ¹ fog.
1
(iii) Composite of functions is associative, i.e. if f, g and h are three b 5
functions such that fo ( foh) and ( fog) oh are defined, then 2
fo ( goh) = ( fog) oh.
e.g. Associativity f : (N ) ® I 0 f ( x ) = 2x Þ (a) is not correct.
138 Textbook of Differential Calculus

(b) If gof is one-one, then f is also one-one, ì2 + x , 0£ x £1


ï
if f is many-one, then gof cannot be one-one. = í2 – x , 1< x £2
ï4 – x , 2< x £3
f g î
A B C
Aliter f ( x ) can be expressed graphically as shown in figure
a
below;
1 4
Y
b 2 5

3
x
gof 1+
2 (x) =
f
Þ (c) and (d) are obviously true.
1 f(x) = 3–x
Hence, (b), (c) and (d) are correct answers.

y Example 103 If f :R ® R, f ( x ) = x 2 and g :R ® R; O 1 2 3 X


x=2
g ( x ) = 2x + 1. Find fog and gof, also show fog ¹ gof .
Sol. Q ( gof )( x ) = g { f ( x )} = g { x 2 }
when
( gof )( x ) = 2x 2 + 1 and 0 £ f ( x ) < 1; 2 < x £ 3 where f ( x ) = 3 – x .
( fog )( x ) = f { g ( x )} = f (2x + 1) 1 £ f ( x ) £ 2; 0 £ x £ 1 where f ( x ) = 1 + x .
( fog )( x ) = (2x + 1)2 2 < f ( x ) £ 3;1 £ x £ 2 where f ( x ) = 1 + x .
where (2x + 1) ¹ (2x + 1) . Therefore, ( gof ) ¹ ( fog ).
2 2
ì1 + f ( x ), 0 £ f ( x ) £ 2
Thus, ( fof )( x ) = í
î 3 – f ( x ), 2 < f ( x ) £ 3
y Example 104 Let g ( x ) = 1 + x – [ x ]
ì1 + f ( x ), 0 £ f ( x ) < 1
ì–1, x < 0 ï
ï ( fof )( x ) = í1 + f ( x ), 1 £ f ( x ) £ 2
and f ( x ) = í 0, x = 0. ï 3 – f ( x ), 2 < f ( x ) £ 3
î
ï 1, x > 0
î ì 1 + (3 – x ), 2 < x £ 3
Then, for all x, find f ( g ( x )) . [IIT JEE 2001] ï
( fof )( x ) = í1 + (1 + x ), 0 £ x £ 1
Sol. Here, g ( x ) = 1 + x –[ x ] ï 3 – (1 + x ), 1 < x £ 2
î
Þ g ( x ) = 1 + {x } [as x –[ x ] = { x }]
ì4 – x , 2 < x £ 3
i.e. g ( x ) ³ 1. ï
( fof )( x ) = í2 + x , 0 £ x £ 1
So, f ( g ( x )) = 1. Since, f ( x )= 1 for all x >0.
ï2 – x , 1 < x £ 2
Thus, f ( g ( x )) = 1, for all x ÎR. î
ì2 + x , 0 £ x £ 1
ì1 + x , 0 £ x £ 2 ï
y Example 105 Let f ( x ) = í , ( fof )( x ) = í 2 – x , 1 < x £ 2
î3 – x , 2 < x £ 3 ï4 – x , 2 < x £ 3
î
find ( fof )( x ) .
ì f (1 + x ) ,
Sol. Clearly, fof ( x ) = f { f ( x )} = í
0£ x £2 ì x + 1, x £1
y Example 106 Let f ( x ) = í
2< x £3
î f (3 – x ) , î2x + 1 , 1 < x £ 2
ì f (1 + x ), 0£ x £1 ì x 2, – 1 £ x < 2
ï and g (x ) = í .
= í f (1 + x ), 1£ x £2
ï f (3 – x ),
îx + 2 , 2 £ x £ 3
î 2< x £3
Find ( fog ).
ì1 + (1 + x ) , 0 £ x £ 1 ì g ( x ) + 1, g (x ) £ 1
ï Sol. f ( g ( x )) = í
= í 3 – (1 + x ) , 1 < x £ 2
î2g ( x ) + 1 , 1 < g ( x ) £ 2
ï 1 + (3 – x ) , 2 < x £ 3
î
Chap 03 Functions 139

Here, g ( x ) becomes the variable that means we should draw ì æx ö


ï3 , x ³0
the graph. It is clear that g ( x ) £ 1; " x Î [– 1, 1] \ f ( g ( x )) = í çè 3 ÷ø
ïî x , x <0
Y
5 Þ f ( g ( x )) = x , " x Î R
2
x+ \ h( x ) = x
4
Þ sin -1 (h (h (h . . . h ( x ) . . . ))) = sin -1 x
3
\ Domain of sin -1 (h (h (h (h . . . h ( x ) ... )))) is [-1, 1].
2 x2 Hence, (a) is the correct answer.
1
y Example 108 A function f : R ® R satisfies
X
–2 –1 0 1 2 3 sin x cos y ( f (2x + 2y ) - f (2x - 2y ))
= cos x sin y ( f (2x + 2y ) + f (2x - 2y )).
and 1 < g ( x ) £ 2; " x Î (1, 2 ].
1
ìx 2 + 1, - 1 £ x < 1 If f ¢ (0) = , then
\ f ( g ( x )) = í 2 2
î2x + 1 , 1 < x £ 2 (a) f ¢¢( x ) = f ( x ) = 0 (b) 4 f ¢¢( x ) + f ( x ) = 0
1 (c) f ¢¢( x ) + f ( x ) = 0 (d) 4 f ¢¢( x ) - f ( x ) = 0
y Example 107 If f ( x ) = 2x + | x | , g ( x ) = (2x - | x | ) and
3 f (2x + 2y ) sin ( x + y )
Sol. We have, =
h( x ) = f ( g ( x )), domain of sin -1 (h(h(h(h ... h( x ) ...)))) is f (2x - 2y ) sin ( x - y )
14444244443
f (a ) f (b )
n times Þ = =K
a b
é 1ù é 1 ù sin sin
(a) [ -1, 1] (b) ê -1, - ú È ê , 1ú 2 2
ë 2û ë 2 û
x
é 1ù é 1 ù Þ f ( x ) = K sin
(c) ê -1, - (d) ê , 1ú 2
ë 2 úû ë2 û K x
Þ f ¢( x ) = cos
ì2x + x , x ³ 0 ì3x , x ³ 0 2 2
Sol. Since, f (x ) = í =í
î2x - x , x < 0 î x , x < 0 and f ¢¢( x ) =
-K
sin
x
4 2
1 ì2x – x , x ³ 0 ìï , x ³ 0
x
and g(x ) = í = í3 Þ 4 f ¢¢( x ) + f ( x ) = 0
3 î2x + x , x < 0 ï x , x < 0
î Hence, (b) is the correct answer.

Exercise for Session 10


1. Consider the real-valued function satisfying 2f (sin x ) + f (cos x ) = x . Find the domain and range of f ( x ).

2. If f ( x ) is defined in [– 3, 2], find the domain of definition of f ([| x | ]) and f ([2x + 3]).
ì x - 1, -1 £ x £ 0
3. f (x ) = í 2 and g( x ) = sin x . Find h( x ) = f (| g( x )| ) + | f ( g( x ))|.
îx , 0 < x £1

ì - 1, -2 £ x £ 0
4. Let f ( x ) be defined on [ -2, 2] and is given by f ( x ) = í and g( x ) = f (| x | ) + | f ( x ) |, find g ( x ).
î x - 1, 0 £ x £2

ì x 2, -1 £ x < 2 ì x + 1, x £ 1
5. Let two functions are defined as g( x ) = í and f ( x ) = í , find gof .
î x + 2, 2 £ x £ 3 î2x + 1, 1 < x £ 2
Session 11
Inverse of a Function
Inverse of a Function y Example 109 If f ( x ) = 3x – 5, find f –1 ( x ).
Let f : A ® B be a one-one and onto function, then there Sol. Here, f ( x ) =3x –5
exists a unique function, g : B ® A such that
which is clearly bijective as it is linear in x.
f ( x ) = y Û g(y ) = x , " x Î A and y Î B.
Now, let f ( x ) = y Þ y = 3 x –5
Then, g is said to be inverse of f .
y +5
Thus, g = f -1 : B ® A = {( f ( x ), x )|( x , f ( x )) Î f } Þ x=
3
Let us consider a one-one function with domain A and y + 5
Þ f –1 ( y ) = [as f ( x ) = y Þ x = f –1
(y ) ]
range B. 3
Where, A = {1, 2, 3, 4 } and B = {2, 4, 6, 8 } and f : A ® B is x +5
Therefore, f –1 ( x ) =
given by f ( x ) =2 x , then write f and f -1 as a set of 3
ordered pairs.
y Example 110 If f : [1, ¥ ) ® [2, ¥ ) is given by
A B 1
f
f ( x ) = x + , find f –1 ( x ), (assume bijection).
1 2 x [IIT JEE 2001, 2002]
2 4 Sol. Let y = f ( x )
3 6 x 2 +1
\ y= Þ x 2 - xy + 1 = 0
4 8
x
y ± y 2 –4
Þ x=
Figure 3.43 2
y ± y 2 –4
B A Þ f –1
(y ) = [as f ( x ) = y Þ x = f –1
(y ) ]
f –1 2
2 1 x ± x 2 –4
Þ f –1
(x ) =
4 2 2
6 3 Since, range of inverse function is [1, ¥ ), therefore
neglecting the negative sign, we have
8 4
x + x 2 –4
f –1
(x ) =
Figure 3.44
2

Here, member y Î B arises from one and only one member y Example 111 Let f ( x ) = x 3 + 3 be bijective, then
x Î A. find its inverse.
So, f = {(1, 2 ) (2, 4 ) (3, 6 ) ( 4, 8 )} Sol. Let y = x3 + 3 [ i.e. y = f ( x )]
-1
and f = {(2, 1) ( 4, 2 ) (6, 3 ) (8, 4 )} Þ x =y -3
3

In above function, Þ x = (y – 3)1/ 3


-1
Domain of f = { 1, 2, 3, 4 } = Range of f . Þ f –1
(y ) = (y – 3)1/ 3 [y = f ( x ) Þ f –1
(y ) = x ]

Range of f = { 2, 4, 6, 8 } = Domain of f -1
. Þ f –1
( x ) = ( x – 3) 1/ 3

which represents for a function to have its inverse, it must Thus, f –1


( x ) = ( x - 3)1/ 3
be one-one onto or bijective. when f ( x ) = x 3 + 3 is bijective.
Chap 03 Functions 141

y Example 112 Find the inverse of the function, y Example 113 Let f : R ® R be defined by
(assuming onto). e x - e -x
f (x ) = . Is f ( x ) invertible? If so, find its
y = log a ( x + x 2 + 1 ), (a > 1). 2
inverse.
Sol. We have, y = loga ( x + x 2 + 1 )
Sol. Let us check for invertibility of f ( x ) .
Since, x 2 + 1 > |x |
e x + e -x
(a) One-one Here, f ¢( x ) =
\It is defined for all x. 2
Now, y = loga ( x + x 2 + 1 ), e 2x + 1
Þ f ¢( x ) = , which is strictly increasing as
which is strictly increasing when a > 1. 2e x
Thus, one-one. Also, given that f ( x ) is onto. e 2x > 0 for all x. Thus, it is one-one.
[where y = f ( x )] (b) Onto Let y = f ( x )
Hence, the given function is invertible. e x - e -x
Þ y= , where y is strictly monotonic.
Now, y = loga ( x + x 2 + 1 ) 2
Hence, range of f ( x ) = ( f (–¥ ), f ( ¥ ))
Þ ay = x + x 2 + 1 and a –y = x 2 + 1 – x Þ Range of f ( x ) = ( - ¥, ¥ )
1 So, range of f ( x ) = codomain
Þ x = (a y - a - y )
2 Hence, f ( x ) is one-one and onto.
-1
Hence, the inverse in the form y = f ( x ) is, -1 e 2x - 1
(c) To find f y=
1 x 2e x
y= (a - a - x )
2 2y ± 4y 2 + 4
Þ e 2 x - 2e x y - 1 = 0 Þ e x =
2
Þ x = log (y ± y 2 + 1 )
Graphical Representation
Þ f –1
(y ) = log (y ± y 2 + 1 )
of Invertible Functions
Let (h, k ) be a point on the graph of the function f . Then, [as f ( x ) = y Þ x = f –1
(y )]
(k, h ) is the corresponding point on the graph of inverse of f – 1 (x )
Since, e is always positive.
f , i.e. g. So, neglecting the negative sign.
The line segment joining the points (h, k ) and (k, h ) is Hence, f -1
( x ) = log ( x + x 2 + 1)
bisected at right angle by the line y = x .

Y
y Example 114 Let f :[1 / 2, ¥ ) ® [ 3 / 4, ¥ ), where
(k, h) (object) f ( x ) = x 2 - x + 1. Find the inverse of f ( x ). Hence, solve
A
1 3
the equation x 2 - x + 1 = + x - ×
2 4
(h, k) (image)
B Sol. (a) f ( x ) = x 2 - x + 1
X 2
(0, 0) æ 1ö 3
Þ f ( x ) = ç x – ÷ + , which is clearly one-one and
è 2ø 4
onto in given domain and codomain.
Figure 3.45
Thus, its inverse can be obtained.
So, that the two points play object-image role in the line æ 1ö 3
2

y = x as plane mirror. Let y = çx – ÷ +


è 2ø 4
It follows that the graph of y = f ( x ) and its inverse written 1 3 1 3
in form y = g( x ) are symmetrical about the line y = x . Þ x– =± y– Þ x = ± y–
2 4 2 4
The graphs y = f ( x ) and y = f -1 ( x ), if they intersect then 1 3
Þ f –1
(y ) = + y– [f ( x ) = y Þ x = f –1
(y )]
they meet on the line y = x only. Hence, the solutions of 2 4
f ( x ) = f -1 ( x ) are also the solutions of f ( x ) = x . [neglecting - ve sign as x is always +ve]
142 Textbook of Differential Calculus

1 3 (iii) The inverse of a bijection is also a bijection.


Þ f –1
(x ) = + x–
2 4 Proof Let f : A ® B be a bijection and g : B ® A be
1 3 its inverse. We have to show that g is one-one and
(b) To solve x - x + 1 = + x – , since solution of
2
2 4 onto.
f ( x ) = f – 1( x ) are solutions of f ( x ) = x .
One-one Let g(b 1 ) = a 1 and
i.e. f (x ) = x Þ x 2 - x + 1 = x
g(b 2 ) = a 2 : a 1 , a 2 Î A and b 1 , b 2 Î B
Þ x 2 - 2x + 1 = 0 Þ ( x - 1) 2 = 0
Then, g(b 1 ) = g(b 2 )
\ x = 1 is the required solution. Þ a1 = a2
Þ f (a 1 ) = f (a 2 )
Properties of Inverse Functions Þ b1 = b2 [Q f is a bijection]
(i) The inverse of a bijection is unique. [Qg (b1 ) = a1 Þ b1 = f (a1 ), g (b 2 ) = a 2 Þ b 2 = f (a 2 )]
Proof Let f : A ® B be a bijection. If possible let which proves that g is one-one.
g : B ® A and h : B ® A be two inverse functions of f . Onto Again, if a Î A, then
Also, let a 1 , a 2 Î A and b Î B such that g(b ) = a 1 and
h(b ) = a 2 , then g(b ) = a 1 Þ f (a 1 ) = b a Î A Þ $ b Î B such that f (a ) = b
[by definition of f ]
h(b ) = a 2 Þ f (a 2 ) = b
Þ $ b Î B such that g(b ) = a [Q f (a ) = b Þ a = g(b )]
But, since f is one-one, so f (a 1 ) = f (a 2 ) Þ a 1 = a 2
which proves that g is onto.
Þ g(b ) = h(b ), " b Î B
Hence, g is also a bijection.
(ii) If f : A ® B is a bijection and g : B ® A is the inverse
of f , thenfog = I B and gof = I A , where I A and I B are (iv) If f and g are two bijections f : A ® B, g : B ® C,
identity functions on the sets A and B, respectively. then the inverse of gof exists and ( gof ) -1 = f -1 og -1 .
Proof Since, f : A ® B and g : B ® C are two
Remarks bijections.
(a) The graphs of f and g are the mirror images of each other
in the line y = x. As shown, in the figure given below a point \ gof : A ® C is also a bijection.
( x ¢, y ¢ ) corresponding to y = x 2( x ³ 0 ) changes to ( y ¢, x ¢ ) [by theorem, the composite of two
corresponding to y = + x , the changed form of x = y . bijections is a bijection]
Y X As such gof has an inverse function ( gof ) -1 : C ® A.
x= y We have to show that ( gof ) -1 = f -1
og -1 .
y = x2 f g

A B C
O X O Y
Figure 3.46 Figure 3.47 gof
Y Figure 3.49

Now, let a Î A, b Î B,c Î C such that


y= x
f (a ) = b and g(b ) = c
x

( gof )(a ) = g[ f (a )] = g(b ) = c


y=

y = x2 So,
-1
Now, f (a ) = b Þ a = f (b ) …(i)
O X
Figure 3.48 g(b ) = c Þ b = g -1 (c ) …(ii)
-1
1
(b) If f ( x ) has its own inverse as in f ( x ) = , then f ( x ) = f -1( x ) ( gof )(a ) = c Þ a = ( gof ) (c ) …(iii)
x
-1 -1 -1 -1
will have infinite solutions but f ( x ) = f -1( x ) = x will have Also, ( f og )(c ) = f [ g (c )] [by definition]
only one solution.
Chap 03 Functions 143

=f -1
(b ) [by Eq. (ii)] y +2
Again, let ( gof )x = y = 3e x - 2 Þ e x =
3
=a [by Eq. (i)]
æy + 2ö
-1 Þ x = log ç ÷
= ( gof ) (c ) [by Eq. (iii)] è 3 ø

\ ( gof ) -1 = f -1
og -1 , æy + 2ö
Þ ( gof )-1 y = log ç ÷
è 3 ø
which proves the theorem.
æ x + 2ö
Þ ( gof )-1 x = log ç ÷
y Example 115 Let g ( x ) be the inverse of f ( x ) and è 3 ø
1 x +2
f ¢ (x ) = . Find g ¢( x ) in terms of g ( x ). and domain of ( gof )–1 is > 0.
3
1+ x 3
Hence, domain of ( gof )-1 is x > –2, i.e. x Î(–2, ¥ ).
Sol. We know, if g ( x ) is inverse of f ( x ).
Þ g { f ( x )} = x Þ g ¢ { f ( x )} × f ¢ ( x ) = 1 y Example 118 If f ( x ) = ax + b and the equation
Þ g ¢ { f ( x )} =
1
= 1 + x 3 Þ g ¢ { f ( g ( x ))} = 1 + ( g ( x ))3 f ( x ) = f -1 ( x ) be satisfied by every real value of x, then
f ¢( x )
(a) a = 2, b = - 1 (b) a = - 1, b Î R
Þ g ¢ ( x ) = 1 + ( g ( x )) 3 [Qf ( g ( x )) = x ]
(c) a = 1, b Î R (d) a = 1, b = - 1
Sol. If f ( x ) = ax + b
y Example 116 If f :R ® R is defined by f ( x ) = x 2 + 1,
-1 x b
Þ f (x ) = -
find the value of f –1 (17 ) and f –1 (– 3). a a
-1
Sol. f ( x ) = x 2 + 1; f –1
(17 ) Þ f ( x ) = 17 Þ x 2 + 1 = 17 Since, f ( x ) = f ( x ), " x Î R
Þ x=±4 and f –1
(– 3) Þ
1 b
= a and b = - Þ a = - 1 and b Î R
Þ f (x ) = – 3 Þ x + 1 = – 3
2 a a
Hence, (b) is the correct answer.
Þ x2 = – 4 [which is not possible]
-1 -1
Hence, f (17 ) = ± 4 and f ( - 3) = f y Example 119 If g ( x ) is the inverse of f ( x ) and
f ¢ ( x ) = sin x , then g ¢( x ) is equal to
y Example 117 If the function f and g are defined as (a) sin (g(x)) (b) cosec (g(x))
f ( x ) = e x and g ( x ) = 3x - 2, where f :R ® R and
(c) tan (g(x)) (d) None of these
g :R ® R, find the function fog and gof. Also, find the -1
Sol. Given, g ( x ) = f (x )
-1 -1
domain of ( fog ) and ( gof ) . So, x = f ( g ( x ))
Sol. ( fog ) ( x ) = f { g ( x )} On differentiating w.r.t. ‘x’, we get 1 = f ¢ ( g ( x )) × g ¢ ( x )
Þ f { g ( x )} = f (3x - 2) Therefore, g ¢ ( x ) =
1
=
1
Þ f { g ( x )} = e 3 x -2
...(i) f ¢ ( g ( x )) sin ( g ( x ))
and ( gof ) ( x ) = g { f ( x ) } \ g ¢ ( x ) = cosec ( g ( x ))
Þ g { f ( x )} = g {e } x Hence, (b) is the correct answer.

Þ g { f ( x )} = 3e x - 2 ...(ii) y Example 120 If A and B are the points of


-1
For finding ( fog ) and ( gof ) . -1 intersection of y = f ( x ) and y = f -1 ( x ), then
Let ( fog ) ( x ) = y = e 3 x -2 (a) A and B necessarity lie on the line y = x
(b) A and B must be coincident
log y + 2
Þ 3x - 2 = log y Þ x= (c) slope of line AB may be –1
3
log y + 2 log x + 2 (d) None of the above
Þ ( fog )-1 y = and ( fog )-1 x =
3 3 Sol. If solution of f ( x ) = f -1( x ) doesn’t lie on y = x , then they
and domain of ( fog )-1 is x >0, i.e. x Î (0, ¥ ). must be of the form (a , b ) and (b, a ).
\ Slope of line AB may be -1. Hence, (c) is the correct answer.
144 Textbook of Differential Calculus

General Results (iii) f ( x + y ) = f ( x ) × f (y ) Þ f ( x ) = a kx .


If x , y are independent variables, then (iv) f ( x + y ) = f ( x ) = f (y ) Î f ( x ) = k, where k is
(i) f ( xy ) = f ( x ) + f (y ) Þ f ( x ) = k ln x or f ( x ) = 0. constant.
(ii) f ( xy ) = f ( x ) × f (y ) Þ f ( x ) = x n , n Î R. (v) f ( x ) takes rational values for all x Þ f ( x ) is a
constant function.

Exercise for Session 11


1. Find the inverse of the following functions
æxö
(i) f ( x ) = sin-1 ç ÷ , x Î [ - 3, 3]
è3ø

(ii) f ( x ) = loge ( x 2 + 3x + 1), x Î [1, 3]


(iii) f ( x ) = 5 loge x , x > 0

(iv) f ( x ) = loge ( x + x 2 + 1)

ìx, x <1
ï
(v) f ( x ) = í x 2, 1£ x £ 4
ï8 x , x > 4
î
- 1)
2. If the function f : [1, ¥ ) ® [1, ¥ ) is defined by f ( x ) = 2x ( x , then find f -1( x ).
Session 12
Miscellaneous Problems on Functions

Miscellaneous Problems Þ
æ 1ö æ
f ç1 - ÷ + f ç1 -
x ö æ 1ö
÷ = 1 + ç1 - ÷
è xø è x - 1ø è xø
On Functions
æ 1ö æ -1 ö 1
We should consider certain examples to make the concept Þ f ç1 - ÷ + f ç ÷ = 2-
è xø è x - 1ø x
clear.
æ 1ö æ 1 ö 1
or f ç1 - ÷ + f ç ÷ = 2- ...(ii)
y Example 121 For x ÎR, the function f ( x ) satisfies è xø è1 - x ø x
2 f ( x ) + f (1 - x ) = x 2 . The value of f (4 ) is equal to æ 1ö 1
Also, f (z ) + f ç1 - ÷ = 1 + z , put z =
13 43 23 è zø 1- x
(a) (b) (c) (d) None of these
3 3 3 æ 1 ö 1
f ç ÷ + f (1 - (1 - x ) = 1 +
Sol. We have, 2 f ( x ) + f (1 - x ) = x 2
…(i) è1 - x ø 1- x
In Eq. (i) x is replaced by (1 - x ), we get æ 1 ö 1
Þ f ç ÷ + f (x ) = 1 + ...(iii)
2 f (1 - x ) + f ( x ) = (1 - x ) 2
…(ii) è1 - x ø 1- x
On solving Eqs. (i) and (ii), we get On subtracting Eq. (ii) from Eq. (iii), we get
\ 2 f ( x ) + f (1 - x ) = x } ´ 2
2 æ 1ö 1 1
f ( x ) - f ç1 - ÷ = + -1 ...(iv)
è x ø 1- x x
Þ +
-2 f (1 - x ) - f ( x ) = -(1 - x )
2

From Eqs. (i) and (iv), we get


Þ 3 f ( x ) = 2x 2 - ( 1 - x ) 2 2f (x ) =
1 1
+ +x
1 2 1- x x
Þ f (x ) = { x + 2x - 1} 1 1 3
3 \ 2 f ( 2) = + +2 =
1 23 -1 2 2
\ f ( 4 ) = {16 + 8 - 1} = 3
3 3 Þ f ( 2) = Þ 4 f ( 2) = 3
4
y Example 122 Let f ( x ) = ax 7 + bx 3 + cx - 5, where
y Example 124 Let f ( x ) = max { x , x 2 }. Then, find
a, b and c are constants. If f ( -7 ) = 7, find f (7 ). equivalent definition of f ( x ).
Sol. As, f ( x ) = ax 7 + bx 3 + cx - 5 Sol. Note These type of questions, where f ( x ) are either maxi-
mum or minimum should be solved graphically for better
\ f ( - x ) = - ax 7 - bx 3 - cx - 5
representation.
Þ f ( x ) + f ( - x ) = - 10 Y
Put x = 7, f (7 ) + f ( -7 ) = - 10 Þ f (7 ) = - 17 y=x2
2 y=x

1 neglecting
y Example 123 If f ( x )+ f æç 1 - ö÷ = 1 + x for x ÎR -{0, 1}.
1
è xø –2 –1 O 1 2
X

The value of 4 f (2) is equal to –1


g neglecting
tin
æ 1ö g lec
Sol. Here, f ( x ) + f ç1 - ÷ = 1 + x ...(i) ne –2
è xø
æ 1ö 1 Let f 1( x ) = x and f 2 ( x ) = x 2
Þ f (y ) + f ç1 - ÷ = 1 + y , let y = 1 -
è yø x
146 Textbook of Differential Calculus

y = 2π
Now, draw graph for f 1( x ) = x and f 2 ( x ) = x 2 .
Here, neglecting the graph, below point of intersection. y = 3π/2
Since, we want to find the maximum of two functions f 1( x )
and f 2 ( x ). y = 3π/4
ì x 2 , x £ 0 or x ³ 1
\ f (x ) = í y=1
îx , 0£ x £1
0
y Example 125 Let
1–2π 1– 3π 1– 3π 1 2 1+ 3π 3π 1+2π
f ( x ) = max { 1 + sin x , 1 - cos x , 1}, " x Î [0, 2p ] 2 4 4
1+
2

and g ( x ) = max { 1, | x – 1| }, " x ÎR. Determine f { g ( x )} ì 1– x, x £0


ï
and g { f ( x )} in terms of x. \ g ( x ) = í 1, 0< x £2
ï x – 1, x >2
î
Sol. Here, f ( x ) = max {1 + sin x , 1 – cos x , 1}.
Graphically it can be shown as ì 1 – f ( x ), f (x ) £ 0
ï
Now, g { f ( x )} = í 1, 0 < f (x ) £ 2
Y ï f ( x )– 1, f (x ) > 2
y = 1 + sin x î
y = 1 – cos x Since, f ( x ) Î [1, 2], " x Î [0, 2p ] ; g ( f ( x )) = 1, " x Î [0, 2p ]
y=2 3p
ì
y ï 1 + sin { g ( x )}, 0 £ g(x ) £
4
=1 y=1 ïï 3p 3p
Also, f { g ( x )} = í1 – cos { g ( x )}, < g(x ) £
4 2
ï
ï 1, 3 p < g ( x ) £ 2p
X
0 π 3π π 3π 2π ïî 2
2 4 2 1 – 2p £ x < 1 – 3p / 2
ì1,
ï1 – cos (1 – x ), 1 – 3p/ 2 £ x < 1 – 3p/ 4
ì 3p ï
ï 1 + sin x , 0 £ x £ 4 ï1 + sin (1 – x ), 1 – 3p/ 4 £ x £ 0
ïï 3p 3p ï
f ( x ) = í 1 – cos x , <x£ [using above graph] Þ f ( g ( x )) = í1 + sin 1, 0< x £2
ï 4 2 ï1 + sin ( x – 1), 2 < x £ 1 + 3p/ 4
ï 3 p ï
1, < x £ 2p ï1 – cos ( x – 1), 1 + 3p/ 4 < x £ 1 + 3p/ 2
ïî 2
ï1, 1 + 3p / 2 < x £ 2p + 1
î
Again, g ( x ) = max {1, | x – 1| }, graphically it can be shown as

Exercise for Session 12


æ 1 ö
1. For x Î R - {1}, the function f ( x ) satisfies f ( x ) + 2f ç ÷ = x . Find f (2).
è1- x ø

2. Let f ( x ) and g( x ) be functions which take integers as arguments. Let f ( x + y ) = f ( x ) + g( y ) + 8 for all integers
x and y . Let f ( x ) = x for all negative integers x and let g (8) = 17. Find f (0).
3. The function f : R ® R satisfies the condition mf ( x - 1) + nf ( - x ) = 2 | x | + 1. If f ( -2) = 5 and f (1) = 1, find (m + n ).

4. The equivalent definition of f ( x ) = max { x 2, (1 – x )2, 2x (1 – x )}, where 0 £ x £ 1.


JEE Type Solved Examples :
Single Option Correct Type Questions

a 2k x 2k + a 2k -1 x 2k -1 + . . . + a 1 x + a 0 Sol. (b)
l Ex. 1 Let f ( x ) = , Y
2k -1
b 2k x + b 2k -1 x
2k
+ . . . + b1 x + b 0
where k is a positive integer, a i , bi Î R and a 2k ¹ 0, 3
b 2k ¹ 0 such that b 2k x 2k + b 2k -1 x 2k -1 + . . . + b1 x + b 0 = 0 (8, 9)
has no real roots, then
(a) f ( x ) must be one to one (h, h)
9 h
(b) a 2k x 2k + a 2k -1 x 2k -1 + . . . + a1x + a 0 = 0 h
must have real roots
X
(c) f ( x ) must be many to one O
8 10
(d) Nothing can be said about the above options
Sol. (c) f ( x ) is continuous, " x Î R. Here, ( x - h )2 + (y - h )2 = h 2 passes through (8, 9).
a
lim f ( x ) = lim f ( x ) = 2k (8 - h )2 + (9 - h )2 = h 2
x ®- ¥ x ®¥ b 2k
\ f ( x ) is many to one. Þ h 2 - 34h + 145 = 0
Þ (h - 5) (h - 29 ) = 0
æ æ p ö ö log 10 6 - 1
10 ç sin ç x + ÷ ÷ = Þ h = 5, neglecting h = 29
l Ex. 2 If log , the value of
è è 4 øø 2 \ r =5
log 10 (sin x ) + log 10 (cos x ) is Area of the largest circle = p(5)2 = 25p.
(a) –1 (b) –2 (c) 2 (d) 1
l Ex. 4 Suppose that the temperature T at every point
æ sin x + cos x ö æ6ö
Sol. (a) 2 log10 ç ÷ = log10 ç ÷ ( x , y ) in the cartesian plane is given by the formula
è 2 ø è 10 ø
T = 1 - x 2 + 2y 2 . The correct statement about the maximum
æ 1 + 2 sin x cos x ö æ6ö and minimum temperature along the line x + y =1 is
Þ log10 ç ÷ = log10 ç ÷
è 2 ø è 10 ø
(a) Minimum is –1. There is no maximum
1 3
Þ + sin x cos x = (b) Maximum is –1. There is no minimum
2 5 (c) Maximum is 0. Minimum is – 1
1
Þ sin x cos x = (d) There is neither a maximum nor a minimum along the
10 line
\ log10 (sin x ) + log10 (cos x ) = - 1 Sol. (a) T = 1 - x 2 + 2y 2 , where x + y = 1
l Ex. 3 The diagram shows the dimensions of the floor of T = 1 - x 2 + 2 (1 - x )2
an L-shaped room. (All the angles are right angles). The area = 1 - x 2 + 2 ( 1 + x 2 - 2x )
of the largest circle that can be drawn on the floor of this
T = x 2 - 4x + 3
room is
= ( x - 2) 2 - 1
3 10 \ T max = doesn’t exist
and T min = - 1
12
9
l Ex. 5 The domain of the function
18
f ( x ) = max {sin x , cos x } is ( - ¥, ¥). The range of f ( x ) is
é 1 ù é 1 1 ù
(a) ê - (b) ê -
2 úû 2 2 úû
(a) 16p (b) 25p ,1 ,
ë ë
81p 145p
(c) (d) (c) [0, 1] (d) [– 1, 1]
4 4
148 Textbook of Differential Calculus

Sol. (a) Here, f ( x ) = max {sin x , cos x } Now, remaining element can be mapped only in 3 ways.
\ Total = 4 C 3 × 3 = 12 ways …(iii)
Y
Case IV Range contains all 4 elements f (1) = 1, f (2) = 2,
1 f (3) = 3, f ( 4 ) = 4.
1/ 2 f

X′ X
O π 2π 1 1
–1 2 2
2 3 3
–1
4 4
Y′
é 1 ù \ Only 1 way.
From graph, range of f ( x ) Î ê - ,1 .
ë 2 úû \ Total ways = 4 + 24 + 12 + 1 = 41
l Ex. 6 Let a function f be defined as l Ex. 7 Area bounded by the relation [ 2 x ] + [y ] = 5,
f : {1 , 2 , 3 , 4 } ® {1 , 2 , 3 , 4 }. If f satisfy f ( f ( x )) = f ( x ), x , y > 0 is (where [ × ] represent greatest integer function)
" x Î {1, 2, 3, 4 }, the number of such functions is
(a) 2 (b) 3 (c) 4 (d) 5
[2x ] + [y ] = 5
(a) 10 (b) 40 (c) 41 (d) 31
Sol. (b) Here,
Sol. (c) Here, f ( f ( x )) = f ( x ), let f ( x ) = y
Let us consider [2x ] = 0 Þ 0 £ 2x < 1 and [y ] = 5
Þ f (y ) = y i.e. x Î[0, 1 / 2) and y Î[5, 6).
Case I Range contains exactly one element. Similarly, we can consider [2x ] = 1, 2, 3, 4 and 5.
It can be done in 4 C 1 ways. when, [2x ] = 1 Þ 1 / 2 £ x < 1 and y Î[ 4, 5)
[2x ] = 2 Þ 1 £ x < 3 / 2 and y Î[3, 4 )
f
[2x ] = 3 Þ 3 / 2 £ x < 2 and y Î[2, 3)
5
1 [2x ] = 4 Þ 2 £ x < and y Î[1, 2)
2 1
2
[2x ] = 5 Þ 5 / 2 £ x < 3 and y Î[0, 1)
3
which can be shown as
4
Y
Say, f (1) = 1 remaining 3 elements 2, 3, 4 can be mapped 6
only in one way Þ Total = 4 C 1 × 1 = 4 …(i)
5
Case II Range contains two elements, this can be done in 4
4
C 2 ways.
3
f
2
1 1 1
2 2
0 X
3 1/2 1 3/2 2 5/2 3

4 \ From the graph, area is 3 sq units.


Say, f (1) = 1, f (2) = 2 l Ex. 8 If the integers a, b, c, d are in arithmetic progres-
Remaining 2 elements, i.e. 3 and 4 each can be mapped in sion and a < b < c < d and d = a 2 + b 2 + c 2 , the value of
2 ways. (a + 10b + 100c + 1000d ) is
\ Total = 4 C 2 ´ 2 ´ 2 = 24 …(ii) (a) 2008 (b) 2010 (c) 2099 (d) 2016
Case III Range contains exactly 3 elements which can be Sol. (c) Let a = x - t , b = x , c = x + t and d = x + 2t .
done in 4 C 3 = 4 ways. Say, f (1) = 1, f (2) = 2, f (3) = 3.
Where, x Î I and t > 0 and t is an integer (as a < b < c < d ).
f
Now, d = a2 + b2 + c 2
1 1 Þ x + 2t = ( x - t )2 + x 2 + ( x + t )2
2 2 Þ x + 2t = 3x 2 + 2t 2
3 3
Þ x (1 - 3x ) = 2t (t - 1) …(i)
4
as t > 0 and t Î I , hence t ³ 1 Þ RHS ³ 0
Chap 03 Functions 149

\ LHS = x (1 - 3x ) ³ 0 Þ x Î[0, 1 / 3] Sol. (a) Here, f (n ) = (sum of digits of natural number n) 2


The only integer is zero Þ x = 0 \ f (2011) = (2 + 0 + 1 + 1)2 = 16
From Eq. (i), t = 0 or t = 1
f 2 (2011) = f ( f (2011)) = f (16) = (1 + 6)2 = 49
But t > 0 Þt = 1
\ a = -1, b = 0, c = 1, d = 2 f 3 (2011) = f ( 49 ) = ( 4 + 9 )2 = 169
Þ a + 10b + 100c + 1000d = 2099 f 4 (2011) = f (169 ) = (1 + 6 + 9 )2 = 256
l Ex. 9 Let f (n ) denotes the square of the sum of the digits f 5 (2011) = f (256) = (2 + 5 + 6)2 = 169

of natural number n, where f 2 (n ) denotes f ( f (n )), f 3 (n ) f 6 (2011) = f (169 ) = (1 + 6 + 9 )2 = 256


denotes f ( f ( f (n ))) and so on. Then, the value of 2n +1
Thus, f 2n
(2011) = 256 and f (2011) = 169 n ³ 2
f 2017 ( 2011 ) - f 2016 ( 2011 )
, is f 2017
(2011) - f 2016
(2011) 169 - 256
f 2017 ( 2011 ) - f 2018 ( 2011 ) Þ = =1
f 2017
(2011) - f 2018
(2011) 169 - 256
(a) 1 (b) 3 (c) 5 (d) 7

JEE Type Solved Examples :


More Than One Correct Option Type Questions
4 4 4 (c) f ¢( 2) - f ¢( -2) = 0
l Ex. 10 If å a i2 x 2 - 2 å a i a i +1 x + å a i2+1 £ 0, where (d) f ¢(3 ) + f ¢( -3 ) = 0
i =1 i =1 i =1
Sol. (b, d) Given, 2f ( x ) × f (y ) = f ( x + y ) + f ( x - y )
a i > 0 and all are distinct. Then,
At x = y = 0,
(a) a1 + a 5 > 2a 3 (b) a1a 5 = a 3
2 f ( 0) × f ( 0) = 2 f ( 0)
5
Þ 2 f ( 0) × ( f ( 0) - 1) = 0
2 1 1
(c) > + (d) Õ ai = a 35
a1a 4 a1 a 4 i =1
Þ f ( 0) = 1 [as f (0) ¹ 0] …(i)
Sol. (a, b, d) (a12 + a 22 + a 32 + a 42 ) x 2 - 2 (a1a 2 + a 2a 3 + a 3a 4 + a 4a 5 ) x At x = 0,
+ (a 22 + a 32 + a 42 + a 52 ) £ 0 2 f (0) × f (y ) = f (y ) + f ( -y ) [using Eq. (i), f (0) = 1]
Þ (a12 x 2 - 2a1a 2 x + a 22 ) + (a 22 x 2 - 2a 2a 3 x + a 32 ) + ... Þ 2f (y ) = f (y ) + f ( -y )
+ (a 42 x 2 - 2a 4a 5 x + a 52 ) £ 0 \ f (y ) = f ( -y ) Þ f ( x ) is even function.
Þ (a1x - a 2 )2 + (a 2 x - a 3 )2 + ... + (a 4 x - a 5 )2 £ 0 We know, if f ( x ) is even, then f ¢( x ) is odd.
Þ f ¢( - x ) = - f ¢( x )
\ (a1x - a 2 )2 + (a 2 x - a 3 )2 + ... + (a 4 x - a 5 )2 = 0
\ f ¢(1) = - f ¢( -1) and f ¢(3) = - f ¢( -3)
a2 a3 a4 a5
\ x= = = =
a1 a 2 a 3 a 4 l Ex. 12 If the equation x 2 + 4 + 3 cos (ax + b ) = 2 x has
Þ a1, a 2 , a 3 , a 4 , a 5 are in GP. atleast one solution where a, b Î[0, 5 ], the value of (a + b )
a1 + a 5 equal to
\ > a1a 5 Þ a1 + a 5 > 2a 3 . (Q a1a 5 = a 3 )
2 (a) 5p (b) 3p (c) 2p (d) p
2 1 1
HM of a1 and a 4 < GM of a1 and a 4 Þ < + Sol. (b, d) x - 2x + 4 = - 3 cos(ax + b )
2
a1a 4 a1 a 4
( x - 1)2 + 3 = - 3 cos(ax + b )
a1 × a 2 × a 3 × a 4 × a 5 = a 35 . [Qa1a 5 = a 32 , a 2 × a 4 = a 32 ]
Here, LHS ³ 3 and RHS £ 3. Thus, the above equation have
solution if LHS = RHS = 3
l Ex. 11 If f ( x - y ), f ( x ) × f (y ), f ( x + y ) are in AP for Þ x = 1 and cos(ax + b ) = - 1
all x , y Î R and f (0 ) ¹ 0, then Hence, cos (a + b ) = - 1\ a + b = p, 3p, 5p
(a) f ¢( x ) is an even function But, 0 £ a + b £ 10 Þ a + b = p or 3p
(b) f ¢(1) + f ¢( -1) = 0 Hence, (b) and (d) are the correct answers.
150 Textbook of Differential Calculus

l Ex. 13 Which of the following functions have the same (c) h ( x ) = cot -1 (cot x ) is shown as,
graph? Y
(a) f ( x ) = loge e x
(b) g ( x ) = x sgn x
π
2 x
(c) h ( x ) = cot -1(cot x ) (d) k ( x ) = lim tan-1(nx )
n ®¥ p X
–2π –π 0 π 2π 3π
Sol. (a,b,d) (a) f ( x ) = loge e = x × loge e = x shown as,
x

Y which is not same as f ( x ) and g ( x ).


f (x) 2| x |
(d) k ( x ) = lim × tan -1 (nx )
n ®¥ p
X′ X ì 2x p
ï × , x >0
O p 2 ìx , x > 0
ïï 2x p ï
Y′ = í- × - , x < 0 = íx , x < 0
ï p 2 ï
ï 0, x = 0 î 0, x = 0
ì x
x ¹0
ï| x | × , x ¹ 0 ìx , ïî
(b) g ( x ) = | x | sgn x = í |x| =í
x =0
x =0 î
ïî 0,
0, \ k ( x ) = x , " x.
\ g ( x ) = x , which is same as f ( x ). Hence, (a), (b) and (d) are the correct answers.

JEE Type Solved Examples :


Statements I and II Type Questions
n
Directions (Q. Nos. 14 to 15) For the following questions, 4 sin 2 q cos q - cos q + sin q
choose the correct answers from the codes (a), (b), (c) and Sol. (a) We have, f (q ) =
4 (cos q + sin q )
(d) defined as follows
2 sin q ((sin q + cos q )2 - 1) - (cos q + sin q ) + 2 sin q
(a) Statement I is true, Statement II is also true; =
Statement II is the correct explanation of Statement I 4 (cos q + sin q )
1 1
= sin q (sin q + cos q ) -
(b) Statement I is true, Statement II is also true;
Statement II is not the correct explanation of 2 4
Statement I
1 é 2 2ù
= (sin 2 q - cos 2 q ) Î ê - ,
(c) Statement I is true, Statement II is false 4 ë 4 4 úû
(d) Statement I is false, Statement II is true 2
l Ex. 15 Consider two functions f ( x ) = 1 + e cot x
sin 2 q cos q 1 æp ö 1 - cos 2 x
l Ex. 14 Let f (q ) = - tan ç - q ÷ , and g ( x ) = 2 | sin x | - 1 + ×
(sin q + cos q ) 4 è4 ø 1 + sin 4 x
ì pü Statement I The solutions of the equation f ( x ) = g ( x ) is
" q Î R - ínp - ý, n Î I .
î 4þ p
given by x = ( 2n + 1) , " n Î I.
Statement I The largest and smallest value of f (q ) differ 2
1 Statement II If f ( x ) ³ k and g ( x ) £ k (where k Î R), then
by . solutions of the equation f ( x ) = g ( x ) is the solution corre-
2
sponding to the equation f ( x ) = k .
Statement II a sin x + b cos x + c Î[c - a 2 + b 2 , 2
Sol. (c) LHS = 1 + e cot x
³2
c + a + b ], " x Î R, where a , b, c Î R.
2 2
As, 2 | sin x | - 1 £ 1
Chap 03 Functions 151

1 - cos 2x 2 sin 2 x 2 p
and = = £1 Þ ,n ÎI
x = (2n + 1)
1 + sin x 4
1 + sin x 4 1 2
+ sin 2 x
sin 2 x \ Statement I is correct.
1 - cos 2x Statement II is not always correct because solution of the
\ RHS = 2 | sin x | - 1 + £2 equation f ( x ) = g ( x ) will be solutions corresponding to
1 + sin 4 x f ( x ) = g ( x ) = k in the domain of f ( x ) and g ( x ) both.
Equation will satisfy, if LHS = RHS = 2 which is possible Hence, (c) is the correct answer.
when cot 2 x = 0 and | sin x | = 1.

JEE Type Solved Examples :


Passage Based Questions
Passage I é5 ö
Þ (a - 5)(26a - 5) £ 0 \ a Î ê , 5÷
(Ex. Nos. 16 to 18) ë 26 ø
Let a m (m =1, 2, . . ., p ) be the possible integral values of a for If a = 5, 5bx + (b + 5) = 0 is not satisfied for b = 0.
which the graphs of \ am Î {1, 2, 3, 4 }; t r = (r - 1)(r - 2)(r - 3)(r - 4 )
f ( x ) = ax 2 + 2bx + b 1 n
Sn = å(r - 4 )(r - 3)(r - 2)(r - 1)(r - (r - 5))
and g ( x ) = 5 x 2 - 3bx - a 5 r =1
1 n
meet at some points for all real values of b.
p n
= å[(r - 4 )(r - 3) (r - 2)(r - 1)r
5 r =1
Let t r = P (r - a m ) and Sn =
m =1
åtr , n ÎN. - (r - 5) (r - 4 )(r - 3)(r - 2)(r - 1)]
r =1
1
= n (n - 1)(n - 2)(n - 3)(n - 4 )
l Ex. 16 The minimum possible value of a is 5
1 5 3 2 Sn = 0
(a) (b) (c) (d)
5 26 28 43 Þ n = 1, 2, 3, 4
\ Sum of values of n for which Sn vanishes is
l Ex.17 The sum of values of n for which Sn vanishes is 1 + 2 + 3 + 4 = 10 [n = 0 rejected]
¥
1 1 n
( r - 1) - ( r - 4 )
åt lim å
(a) 8 (b) 9
=
(c) 10 (d) 15 r =5 r 3 n ® ¥ r = 5 (r - 4 )(r - 3)(r - 2)(r - 1)
¥ 1 n
é 1 1 ù
l Ex. 18 The value of å
1
is equal to = lim
3 n ®¥
å ê (r - 4 )(r - 3)(r - 2) - (r - 3)(r - 2)(r - 1) ú
tr r =5 ë û
r =5
1 é1 1 ù 1
(a)
1
(b)
1
(c)
1
(d)
1 = lim ê 6 - (n - 3)(n - 2)(n - 1) ú = 18
n ®¥ 3 ë û
3 6 15 18

n
Sol. (Q. Nos. 16 to 18) Passage II
16. (a) 17. (c) 18. (d) (Ex. Nos. 19 to 21)
ax 2 + 2bx + b = 5x 2 - 3bx - a Let w be non-real fifth root of 3 and x = w 3 + w 4 . If
Þ (a - 5)x 2 + 5bx + (b + a ) = 0 x 5 = f ( x ), where f ( x ) is real quadratic polynomial, with
roots a and b, (a, b ÎC ), then determine f ( x ) and answer
If a ¹ 5, then since x Î R,
the following questions.
D = 25b 2 - 4(b + a )(a - 5) ³ 0, " b Î R
Þ 25b 2 - 4(a - 5)b - 4a(a - 5) ³ 0, " b Î R
l Ex. 19 Every term of the sequence { f (n )}, n Î N is
divisible by
\ 16(a - 5)2 + 16(25)a (a - 5) £ 0
(a) 12 (b) 18
Þ 16(a - 5)(a - 5 + 25a ) £ 0
(c) 24 (d) 27
152 Textbook of Differential Calculus

l Ex. 20 Which of the following is not true? 1 1 3 æ 15 ö


D = AB(OM ) = × ×ç ÷
(a) a + b = -3 (b) ab = 12 / 5 2 2 5 è 10 ø
(c) |a - b | = 3 / 5 (d) |a | + |b | = 2 3 / 5
Y

l Ex. 21 If a and b are represented by points A and B in A(–15/10, √15/10)

argand plane, then circumradius of DOAB, where O is origin, is


(a) 4 / 5 (b) 8 / 5 (c) 16 / 5 (d) 32 / 5 X
M O(0, 0)
n Sol. (Q. Nos. 19 to 21)
19. (b) 20. (d) 21. (a)
B(–15/10, –√15/10)
Here, x = w3 +w4
Þ x 5 = (w 2 )5 (w + w 2 )5 12 3 12
= (w ) [w + 5w × w + 10w × w + 10w × w
5 2 5 4 2 3 4 2 6 OA × AB × OB 5 5 5 =4
\ Circumradius, R = =
+ 5w × w 8 + w 10 ] 4D 1 3 æ 15 ö 5
4× × ç ÷
2 5 è 10 ø
Put w 5 = 3 and w 10 = 9
Now, x 5 = 9 [3 + 5 × (3w ) + 10 × (3w 2 ) + 10 × 3w 3 + 5 × (3w 4 ) + 32 ] Passage III
= 9 [12 + 15(w + w 2 ) + 15(w 2 + w 3 ) + 15(w 3 + w 4 )] (Ex. Nos. 22 to 24)
é æ x ö æxö ù Let A = {1, 2, 3, 4, 5 } and B = { -2, - 1 , 0, 1 , 2, 3, 4, 5 }.
= 9 ê12 + 15 ç 2 ÷ + 15 × ç ÷ + 15x ú
ë è w ø è w ø û
é æ 1 1ö ù
l Ex. 22 Increasing function from A to B is
= 9 ê12 + 15x ç 2 + ÷ + 15x ú
ë è w w ø û
(a) 120 (b) 72 (c) 60 (d) 56

é w +w2 w2 ù l Ex. 23 Non-decreasing functions from A to B is


= 9 ê12 + 15x × 3
× 2 + 15x ú
ë w w û (a) 216 (b) 540 (c) 792 (d) 840
é (w + w )
3 4 ù
= 9 ê12 + 15x × 5
+ 15x ú l Ex. 24 Onto functions from A to A such that
ë w û f (i ) ¹ i for all i, is
é x ù
= 9 ê12 + 15x × + 15x ú (a) 44 (b) 120 (c) 56 (d) 76
ë 3 û
n
Sol. (Q. Nos. 22 to 24)
= 9 [12 + 15x + 5x 2 ] = f ( x )
22. (d) The desired number is 8 C 5 = 56
Now, f ( x ) = 9 (5x 2 + 15x + 12)
23. (c) Here, for non-decreasing functions from A to B, is
\ f (x ) = 0 8
C 1 × 1 + 8 C 2 × 4 + 8 C 3 × 6 + 8 C 4 × 4 + 8 C 5 × 1 = 792
- 15 ± -15 - 15 ± i 15
Þ x= = = a, b Explanation for case 8 C 2 , say two elements of set B are
10 10
selected in 8 C 2 is { -1, 0}.
(i) For sequence { f (n )},
f (n ) = 9(5n 2 + 15n + 12) = 9 [5n (n + 1) + 10n + 12] Now, x1 + x 2 = 5 [ x 1 ³ 1, x 2 ³ 1]
where, x 1 denotes number of elements of set A maps to -1
= 9 ´ an even number and x 2 denotes number of elements of set A maps to 0.
\Every term of { f (n )} is always divisible by 18.
\ Total number of solutions is 4 C 1.
12 2i 15 3
(ii) Clearly, a + b = -3, ab = , a - b = = Similarly, explanation for case 8 C 3 say three elements
5 10 5
selected in 8 C 3 is { -1, 0, 1}.
æ 2 2ö
ç æ 15 ö æ 15 ö ÷ 3 Now, x 1 + x 2 + x 3 = 5 ( x 1 ³ 1, x 2 ³ 1, x 3 ³ 1)
But a + b = +ç ÷ ÷=4
ç çè 10 ÷ø
2
è 10 ø 5 \ Total number of solutions is C 2 etc. 4
è ø
(iii) In argand plane, 24. (a) Onto functions from A to A such that f (i ) ¹ i for all i.
æ 1 1 1 1 1ö
OA = OB =
12
and AB = 2
15
=
3 Þ 5! ç1 - + - + - ÷ = 44
è 1! 2! 3! 4 ! 5! ø
5 10 5
and area of DOAB,
Chap 03 Functions 153

JEE Type Solved Examples :


Matching Type Questions
lEx. 25 Match the statements of Column I with values of Therefore, no value of ri can be 49 or larger factor of
Column II. 2009 otherwise their sum is larger than 56.
As 56 = 41 + 7 + 7 + 1
Column I Column II
Therefore, r1 = 41; r 2 = r 3 = 7
(A) Let f (x ) be a function on (- ¥ , ¥ ) and (p) 4
f (x + 2) = f (x - 2). If f (x ) = 0 has only three and r4 = 1
real roots in [ 0, 4 ] and one of them is 4, then the Hence, n=4
number of real roots of f (x ) = 0 in (- 8, 10 ] is (C) 2xy = 2009 - 3y yields 2xy + 3y = 2009
(B) Let r1 , r2 , r3 , . . . , rn be n positive integers, not (q) 5 2009
Þ y=
necessarily distinct, such that 2x + 3
(x + r1 )(x + r2 ) . . . (x + rn ) = x n + 56x n - 1 + . . . So, 2x + 3 must be the factors of 2009.
+ 2009. The possible value of n is
Since, 2009 = 7 2 × 41
(C) If x and y are positive integers and (r) 8
Thus, 2x + 3 can be one of the factors 7, 49, 41, 287
2xy = 2009 - 3 y, then the number of ordered
and 2009.
pairs of (x , y) is
or 2x + 3 = 7; 2x + 3 = 49 ; 2x + 3 = 41;
(D) If x , y Î R , satisfying the equation (s) 9
2x + 3 = 287
(x - 4 )2 y2
+ = 1, then the difference between and 2x + 3 = 2009.
4 9
the largest and smallest value of the expression Hence, x = 2, 23, 19, 142 and 1003
x2 y2 Þ Ordered pairs are (2, 287), (23, 41), (19, 49),
+ is (142, 7), (1003, 1).
4 9
(D) Let x - 4 = 2 cos q
Sol. (A) ® (s), (B) ® (p), (C) ® (q), (D) ® (r)
Þ x = 2 cos q + 4
(A) Given, f ( x + 2) = f ( x - 2), domain is R x ® x + 2
and y = 3 sin q
f (x + 4) = f (x )
Y
Þ f is periodic with period 4.
Put x = 0, f ( 4 ) = f (0)
But f ( 4 ) = 0
Þ f ( 0) = 0
Þ x = 0, is also the root. X
O 1 (6, 0)
Since, in [0, 4 ], f has only 3 roots.
æ px ö (2, 0)
So, f ( x ) is the form of sin ç ÷ fulfilling all
è 2 ø
conditions given in the question, hence the 3rd root in
[0, 4 ] is 2 as f (2) = 0. x 2 y 2 (2 cos q + 4 )2
Now, z= + = + sin 2 q
\The number of real roots in ( - 8, 10] is 9 and the 4 9 4
roots are ( - 6, - 4, - 2, 0, 2, 4, 6, 8, 10). 4 cos 2 q + 16 + 16 cos q + 4 sin 2 q 20 + 16 cos q
= =
(B) (r1 + r 2 + . . . + rn ) = 56 4 4
and r1r 2 . . . rn = 2009 = 7 2 × 41 z = 5 + 4 cos q
[using theory of equation] Hence, z max - z min = (9 - 1) = 8
As 2009 = 7 2 × 41
JEE Type Solved Examples :
Single Integer Answer Type Questions
l Ex. 26 Let f ( x ) = sin 23 x - cos 22 x and 9
l Ex. 28 If f ( x ) = 3 - 1, the value of ‘a’
log 2 (3 - 2 x )
g ( x ) = 1 + tan -1 | x |.
1

which satisfies f -1 ( 2a - 4 ) = , is
2 1
The number of values of x in interval [ -10 p, 20 p ] satisfying 2
the equation f ( x ) = sgn ( g ( x )), is 5a. Then, a is equal to …… . 1
-1
1 Sol. (3) Given, f (2a - 4 ) =
Sol. (3) g ( x ) = tan -1 | x | + 1 2
2 æ1ö
Þ sgn ( g ( x )) = 1 \ f ç ÷ = 2a - 4
è2ø
Þ sin 23 x - cos 22 x = 1
1 9
Put x = in f ( x ) = 3 -1
Þ sin 23 x = 1 + cos 22 x 2 log 2 (3 - 2x )
which is possible, if sin x = 1 and cos x = 0 9
p \ 2a - 4 = 3 -1
x = 2np + log 2 2
2
p Þ 2a - 4 = 3 9 - 1
Hence, -10p £ 2np + £ 20p
2 Þ 2a - 4 = 2
21 39 Þ 2a = 6
Þ - £ 2n £
2 2 \ a=3
21 39
Þ - £n £ l Ex. 29 Let f be defined on the natural numbers as
4 4
follow:
Þ -5£n £9
f (1 ) = 1 and for n >1, f (n ) = f [ f (n -1 )] + f [n - f (n - 1 )],
Hence, number of values of x = 15 Þ a = 3.
1 20
the value of å f (r ) is
l Ex. 27 Consider the function g ( x ) defined as 30 r =1

g( x ) × ( x ( 2
2008
- 1)
- 1 ) = ( x + 1 )( x 2 + 1 )( x 4 + 1 ) Sol. (7) Here, f ( 1) = 1
2007 f (2) = f [ f (1)] + f [2 - f (1)], using f (1) = 1
. . . (x 2 + 1 ) - 1 the value of g( 2 ) equals …… .
f (2) = f (1) + f (1) = 2.
2007
( x - 1)( x + 1)( x 2 + 1)( x 4 + 1) . . . ( x 2 + 1) f (3) = f [ f (z )] + f [3 - f (2)]
Sol. (2) RHS = -1
( x - 1)
= f ( 2) + f ( 1) = 2 + 1 = 3
2 2007
( x - 1)( x + 1) . . . ( x
2 2
+ 1) f (n ) = n
= -1 Thus,
( x - 1)
1 20 1
2 2
( x 2 - 1)( x 2 + 1) . . . ( x 2
2007
+ 1)
\ å
30 r =1
f (r ) = [1 + 2 + 3 + ... + 20]
30
= -1
( x - 1) 1 20 (20 + 1)
= ´ =7
2 2008 30 2
2008
- 1) (x - 1) - ( x - 1)
Hence, g ( x ) × ( x (2 - 1) =
( x - 1) l Ex. 30 If a , b, c are real roots of the cubic equation
2 2008 - 1
x (x - 1) f ( x ) = 0 such that ( x -1 ) 2 is a factor of f ( x ) + 2 and
=
( x - 1)
( x +1 ) 2 is a factor of f ( x ) - 2 , then | ab + bc + ca | is equal to
x
Þ g(x ) = Sol. (3) Let f ( x ) = Ax 3 + Bx 2 + Cx + D
x -1
Since, ( x - 1)2 is a factor of f ( x ) + 2 .
\ g(2) = 2
Þ f ( 1) + 2 = 0
Chap 03 Functions 155

Þ A + B +C + D +2 = 0 …(i) \ f ( x ) can be plotted as Y


Also, ( x + 1) is a factor of f ( x ) - 2 .
2
e 2
So, clearly from graph , k > ,k Î I
Þ f ( - 1) - 2 = 0 4
e2/4
Þ -A + B - C + D - 2 = 0 …(ii) Hence, k min =2
X
From Eq. (i), D = -A - B - C - 2 O
l Ex. 32 The value of expression
\ f ( x ) = Ax 3 + Bx 2 + Cx - A - B - C - 2
Þ f ( x ) + 2 = A ( x 3 - 1) + B( x 2 - 1) + C ( x - 1) 1 + 2 1 + 3 1 + 4 1 + 5 ... ¥ is
Since, ( x - 1)2 is factor of f ( x ) + 2.
Sol. (3) As, we can write
Þ f ¢ ( 1) = 0 Þ 3A + 2B + C = 0 …(iii)
Similarly, ( x + 1)2 is factor of f ( x ) - 2. n = n 2 = 1 + (n 2 - 1) = 1 + (n - 1)(n + 1)

Þ f ¢ ( - 1) = 0 Þ 3A - 2B + C = 0 …(iv) = 1 + ( n - 1) × ( n + 1) 2 .
From Eqs. (i), (ii), (iii) and (iv), A = 1, C = -3, B = D = 0.
= 1 + ( n - 1) × 1 + ( n + 1) 2 - 1
\ f ( x ) = x 3 - 3x
Hence, | ab + bc + ca | = 3 = 1 + ( n - 1) × 1 + ( n ) × ( n + 2) .

l Ex. 31 The minimum value of k (k Î I ) for which the = 1 + ( n - 1) × 1 + n × ( n + 2) 2


equation e x = kx 2 has exactly three real solutions, is
= 1 + ( n - 1) × 1 + n × 1 + ( n + 2) 2 - 1
ex
Sol. (2) Here, e x = kx 2 Þ =k …(i)
x2 = 1 + (n - 1) × 1 + n × 1 + (n + 1)(n + 3)
x
e
Let f (x ) =
x2 = 1 + (n - 1) × 1 + n × 1 + (n + 1) × (n + 3)2 … and
( x - 2)e x
f ¢( x ) = so on.
x3
\ f ( x ) is increasing on ( -¥,0 ) È [2, ¥ ) and decreasing on ( 0,2 ) . \ 1 + 2 × 1 + 3 × 1 + 4 × 1 + 5 ....... ¥ = 3.

Subjective Type Questions


l Ex. 33 Let a sequence x 1 , x 2 , x 3 , ... of complex numbers l Ex. 34 If a , b, c , d , e are positive real numbers, such that
be defined by x 1 = 0, x n + 1 = x n2 – i for all n >1, where a + b + c + d + e = 8 and a 2 + b 2 + c 2 + d 2 + e 2 = 16, find
i 2 = – 1. Find the distance of x 2000 from x 1997 in the complex
the range of e.
plane.
Sol. As we know,
Sol. Given, x1 = 0
2
Then, x 2 = 02 – i = – i æa + b + c + d ö a2 + b2 + c 2 + d 2
ç ÷ £ …(i)
è 4 ø 4
x 3 = (–i )2 – i = – 1 – i = –(1 + i )
[using Tchebycheff’s Inequality]
x 4 = [– (1 + i )]2 – i = 2i – i = i
where a+b+c +d +e =8
x 5 = (i )2 – i = – 1 – i = x 3
and a + b 2 + c 2 + d 2 + e 2 = 16
2
\ x 6 = x 4 and hence x 7 = x 5 and so on.
\ x 2n = i , x 2n + 1 = –(1 + i ) From Eq (i), reduces to
2
\ x 2000 = i = (0, 1) and æ8 – e ö 16 – e 2
x 1997 = – 1 – i = (– 1, – 1) in the complex plane. ç ÷ £
è 4 ø 4
So, the distance between x 2000 and x 1997 is 1 + 4 = 5. Þ 64 + e 2 – 16e £ 4 (16 – e 2 )
156 Textbook of Differential Calculus

Þ 5e 2 – 16e £ 0 But, if [ x ] = 1, then given equation


Þ e (5e – 16) £ 0 {x } = x …(iii)
16 which is true only when x Î[0, 1)
Þ 0£e £ [using number line rule] and [ x ] = 1 Þ x Î[1, 2) …(iv)
5
+ – + From Eqs. (iii) and (iv) no value of x,
0 16 when [ x ] = 1 …(v)
5
[x ]
é 16 ù Now, from Eq. (ii), {x } = ,
Thus, e Î ê0, ú. [ x ]– 1
ë 5û
when [x ] ¹ 1
lEx. 35 Find the set of all solutions of the equation Again, as we know { x } Î [0, 1)
2 – 2 y – 1 – 1 = 2 y – 1 +1 .
|y|
\ 0£
[x ]
<1
[IIT JEE 1997] [ x ]– 1
Sol. Here, 2| y | – 2y –1 –1 = 2y –1 + 1 [x ] [x ]
i.e. < 1 and ³0
[ x ]– 1 [ x ]– 1
We know, how to define modulus, we have three cases as
1
Case I y < 0 i.e. < 0 and {[ x ] £ 0 or [ x ] > 1}
[ x ]– 1
ì as when y < 0
i.e. [x ] < 1 and {[ x ] £ 0 or [ x ] > 1}
Þ 2 –y
+ (2 y –1
–1 ) = 2 y –1
+ 1; ïï| y | = - y
í [using number line rule]
Þ 2–y = 21 ï
ïî and 2 –1 = –(2 –1)
y –1 y –1 i.e. [x ] £ 0
[x ]
\ x = [x ] + { x } Þ x = [x ] +
Hence, y = –1, which is true when y <0. …(i) [ x ]–1
Case II 0 £ y < 1 [ x ]2
Þ x= , [where x takes values less than 1]
ì as when 0 £ y < 1 [ x ]– 1
Þ 2 + (2 –1) = 2 + 1; ïï| y | = y
y y –1 y –1
[ x ]2
í \ x= , [where x < 1]
Þ 2y = 2 ï [ x ]– 1
ïî and 2 –1 = –(2 –1)
y –1 y –1

[ x ]2
Þ y = 1, which shows no solution as, Þ x= , [where [ x ] is any non-positive integer]
[ x ]– 1
0 £ y <1 …(ii)
Case III y ³1 l Ex. 37 Find all possible values of x satisfying
2y –(2y –1 –1) = 2y –1 + 1 ì as when y ³ 0 [x ] [ x – 2 ] 8 { x } + 12
Þ =
ïï [x – 2]

[x ] [ x – 2 ][ x ]
Þ 2 =2
y y –1
+2 y –1
; í| y | = y
Þ 2 = 2×2
y y –1 ï (where [ ] denotes the greatest integer function and { } is the
ïî and 2 –1 = (2 –1)
y –1 y –1
fractional part).
Þ 2y = 2y , which is an identity, therefore it is true, " y ³1. [ x ] [ x – 2] 8 { x } + 12
Sol. Here, – =
…(iii) [ x – 2] [ x ] [ x – 2] [ x ]
Hence, from Eqs. (i), (ii) and (iii) the solution set is [ x ]2 –[ x – 2]2 8 { x } + 12
{ y :y ³ 1 È y = –1}. Þ =
[ x – 2] [ x ] [ x ] [ x – 2]

Ex. 36 Solve the equation [ x ] { x } = x , where [ ] and { } ([ x ]–[ x – 2]) ([ x ] + [ x – 2]) 8 { x } + 12


l
Þ =
[ x – 2] [ x ] [ x ] [ x – 2]
denote the greatest integer function and fractional part,
respectively. Þ ([ x ]–[ x – 2]) ([ x ] + [ x – 2]) = 8 { x } + 12
Sol. We know that, x = [x ] + { x } …(i) Þ ([ x ]–[ x ] + 2) ([ x ] + [ x ]– 2) = 8 { x } + 12
Thus, we have [ x ]{ x } = [ x ] + { x } [Q[ x + I ] = [ x ] + I ]
[x ] Þ 4 ([ x ]– 1) = 8 { x } + 12
Þ {x } = …(ii)
[ x ]– 1 Þ [ x ]– 4 = 2 { x } …(i)
Here, in Eq. (ii), [ x ] ¹ 1 Now, as we know 0 £ { x } < 1 Þ 0 £ 2 { x } < 2
Chap 03 Functions 157

Þ 0 £ [ x ]– 4 < 2 Þ 4 £ [ x ] < 6 Hence, f (6) = 63 + 1 = 217


Þ [ x ] = 4, 5 Aliter Let f ( x ) = a 0 x n + a1x n – 1 + a 2 x n – 2
if [ x ] = 4 Þ 2 { x } = [ x ] – 4ü + K + an – 1x + an
ý …(ii) æ1ö æ1ö
becomes { x } = 0 þ Then, f ( x ) × f ç ÷ = f ( x ) + f ç ÷
èx ø èx ø
and if [ x ] = 5 Þ 2 { x } = [ x ]– 4ü æa ö
ï a
Þ (a 0 x n + a1x n – 1 + K + an ) ç 0n + n 1– 1 + K + an ÷
becomes { x } =
1 ý …(iii)
èx ø
ïþ x
2
Thus, from Eqs. (ii) and (iii), we have
æa a ö
x = [x ] + { x } = (a 0 x n + a1x n – 1 + K + an ) + ç 0n + n 1– 1 + K + an ÷
èx x ø
i.e. x = 4 +0 = 4 [using Eq. (ii)]
On comparing the coefficient of x n , we have
1 11
and x = 5+ = [using Eq. (iii)] a 0an = a 0 Þ an = 1 [as a 0 ¹ 0]
2 2
n -1
Comparing the coefficient of x , we have
ì 11ü
Þ x Î í 4, ý a 0an – 1 + an a1 = a1
î 2þ
Þ a 0an – 1 + a1 = a1 [as an = 1]
l Ex. 38 If f ( x ) is a polynomial function satisfying Þ a 0an – 1 = 0
æ1ö æ1ö Þ an – 1 = 0 [as a 0 ¹ 0]
f ( x ) × f ç ÷ = f ( x ) + f ç ÷ and f ( 4 ) = 65 . Find f (6 ).
èxø èxø Similarly, an – 1 = an – 2 = ...... = a1 = 0
and a0 = ± 1
æ1ö æ1ö
Sol. Here, f (x )× f ç ÷ = f (x ) + f ç ÷ \ f (x ) = ± xn + 1
èx ø èx ø
æ1ö æ1ö Þ f ( 4 ) = ± 4n + 1
Þ f (x ) × f ç ÷ – f (x ) = f ç ÷
èx ø èx ø Þ 4n + 1 = 65 Þ 4n = 64
Þ f (x ) =
f (1/x )
…(i) Þ n =3
f (1/x ) – 1 So, f (x ) = x 3 + 1
æ1ö æ1ö f (6) = 63 + 1 = 217
Also, f (x )× f ç ÷ = f (x ) + f ç ÷ Hence,
èx ø èx ø
æ1ö æ1ö l Ex. 39 If f ( x ) satisfies the relation,
Þ f (x )× f ç ÷ – f ç ÷ = f (x )
èx ø èx ø f ( x + y ) = f ( x ) + f (y ) for all x , y Î R and f (1) = 5, then
m
æ1ö f (x )
Þ fç ÷=
è x ø f (x ) – 1
…(ii) find å f (n ). Also, prove that f ( x ) is odd.
n =1
On multiplying Eqs. (i) and (ii), we get
Sol. Here, f ( x + y ) = f ( x ) + f (y ), put x = r - 1, y = 1
æ1ö f (1/x ) × f ( x )
f (x )× f ç ÷ = f ( r ) = f ( r – 1) + f ( 1) …(i)
è x ø { f (1/x ) – 1} { f ( x ) – 1}
\ f ( r ) = f ( r – 1) + 5
é æ1ö ù
Þ ê f çè x ÷ø – 1ú [ f ( x ) – 1] = 1 …(iii) Þ f ( r ) = { f ( r – 2) + 5 } + 5 [using definition]
ë û
Þ f ( r ) = f ( r – 2) + 2 × ( 5)
Since, f ( x ) is a polynomial function, so { f ( x ) – 1} and Þ f ( r ) = f ( r – 3) + 3 × ( 5)
ì æ1ö ü 1 ………………………………
í f ç ÷ – 1ý are reciprocals of each other. Also, x and are
î è x ø þ x ……………………………….
reciprocals of each other. Þ f (r ) = f {r – (r – 1)} + (r – 1) × 5
Thus, Eq. (iii) can hold only when Þ f ( r ) = f ( 1) + ( r – 1) × 5
f (x ) – 1 = ± xn , where n Î R Þ f ( r ) = 5 + ( r – 1) × 5
Þ f (r ) = 5r
\ f ( x ) = ± x n + 1, but f ( 4 ) = 65 m m
Þ ± 4 + 1 = 65 Þ 4 = 64
n n \ å f (n ) = å(5n ) = 5[1 + 2 + 3 + K + m ]
n =1 n =1
Þ 4n = 4 3 Þ n = 3 [Q 4n > 0 ] 5m (m + 1)
=
So, f (x ) = x 3 + 1 2
158 Textbook of Differential Calculus

m
5m (m + 1) æ 997 ö æ 999 ö
Hence, å f (n ) = 2
…(ii) Þ f ç ÷+f ç
è 1996 ø
÷ =1
è 1996 ø
n =1

Now, putting x = 0, y = 0 in the given function, we have æ 998 ö æ 998 ö


Þ f ç ÷+f ç ÷ =1
è 1996 ø è 1996 ø
f ( 0 + 0) = f ( 0) + f ( 0)
\ f ( 0) = 0 æ 998 ö 1
or f ç ÷=
Also, putting (– x ) for (y ) in the given function, è 1996 ø 2
f ( x – x ) = f ( x ) + f (– x ) On adding all the above expressions, we get
Þ f (0) = f ( x ) + f (– x ) æ 1 ö æ 2 ö æ 1995 ö
f ç ÷+f ç ÷ +K+ f ç ÷
Þ 0 = f ( x ) + f (– x ) è 1996 ø è 1996 ø è 1996 ø
Þ f (– x ) = – f ( x ) …(iii) 1
= (1 + 1 + 1 + K + to 997 times ) +
m
5m (m + 1) 2
Thus, å f (n ) = and f ( x ) is odd. 1
n =1 2 = 997 +
2
= 997.5
9x
l Ex. 40 Let f ( x ) = . Show that f ( x ) + f (1 – x ) = 1
9x + 3 l Ex. 41 ABCD is a square of side a. A line parallel to the
and hence evaluate diagonal BD at a distance x from the vertex A cuts the two
æ 1 ö æ 2 ö æ 3 ö æ 1995 ö adjacent sides. Express the area of the segment of the square
f ç ÷+f ç ÷+ f ç ÷ +K+ f ç ÷.
è 1996 ø è 1996 ø è 1996 ø è 1996 ø with A at a vertex, as a function of x.
Sol. There are two different situations,
9x
Sol. Given, f (x ) = …(i) (i) when x = AP £ OA ,
9x + 3
a
91 – x i.e. x£
Þ f (1 – x ) = 2
9 1–x
+3
(ii) when x = AP > OA ,
9
a
9x 9 i.e. x > but x £ 2a
Þ f (1 – x ) = = 2
9
+3 9 + 3× 9 x
9x D C D F C
x
9 –
a
f (1 – x ) = …(ii) √2
3 (3 + 9 x )
F P
On adding Eqs. (i) and (ii), we get O a a O
P E
9x 9
f ( x ) + f (1 – x ) = + x
9x + 3 3 (3 + 9 x )
3 ×9 x + 9 3 ( 9 x + 3) A E B A B
= = a a
3 ( 9 x + 3) 3 ( 9 x + 3)
1
\ f ( x ) + f (1 – x ) = 1 …(iii) Case I ar ( D AEF ) =
x × 2x = x 2
2
1 2 3 998
Now, putting x = , , , K, in Eq. (iii), we Case II ar ( ABEFDA ) = ar ( ABCD ) – ar ( DCEF )
1996 1996 1996 1996 1
get = a 2 – ( 2a - x )2( 2a – x )
2
æ 1 ö æ 1995 ö
fç ÷+fç ÷ =1 ar ( ABEFDA ) = 2 2 ax – x 2 – a 2
è 1996 ø è 1996 ø
æ 2 ö æ 1994 ö ì a
Þ fç ÷+fç ÷ =1 ï x 2, 0£ x £
è 1996 ø è 1996 ø \ f (x ) = í 2
a
æ 3 ö æ 1993 ö ï2 2 ax – x 2 – a 2 , < x £ 2a
Þ fç ÷+fç ÷ =1 î 2
è 1996 ø è 1996 ø
……………………………………
……………………………………
Chap 03 Functions 159

l Ex. 42 A function R ® R is defined by Þ f (8y , – 8x ) = f {8(– 8x ), – 8(8y )} [using Eq. (ii)]


a x 2 + 6x - 8 Þ f ( x , y ) = f (2x + 2y , 2y – 2x ) = f (8y , – 8x )
f (x ) = . Find the interval of values of a for
a + 6x - 8x 2 = f (–64 x , –64y )

which f is onto. Is the function one-one for a = 3? Justify Þ f ( x , y ) = f (–64 x , – 64y ) …(iii)
your answer. [IIT JEE 1996] Þ f (–64 x , – 64y ) = f (64 ´ 64 x , 64 ´ 64y )
Sol. Since, f : R ¾® R is an onto mapping. = f (212 x , 212 y )
\ Range of f = R Þ f ( x , y ) = f (212 x , 212 y ) [using Eq. (iii)]
a x 2 + 6x - 8 12 + x
Þ assumes all real values of x. Þ f ( 2 , 0) = f ( 2 × 2 , 0) = f ( 2
x 12 x
, 0) …(iv)
a + 6x - 8x 2
Given, g ( x , 0) = f (2x ,0)
a x 2 + 6x - 8
Let y=
a + 6x - 8x 2 Þ g ( x ,0) = f (2x , 0) = f (212 + x , 0) [using Eq. (iv)]
Then, y assumes all real values for real values of x. Þ g ( x , 0) = g ( x + 12, 0)
Þ ay + 6xy - 8x 2y = ax 2 + 6x - 8, " y ÎR Hence, g ( x ) is periodic with period 12.
Þ x 2 (a + 8y ) + 6x (1 - y ) - (8 + a y ) = 0, " y ÎR l Ex. 44 Solve the equation
We know, above equation assumes all real values.
10 ( x + 1)(3 x + 4 ) – 2 × 10 ( x + 1) ( x + 2 ) = 10 1 – x – x .
2

So, D ³0
Þ 36 (1 - y )2 + 4 (a + 8y ) (8 + ay ) ³ 0 Sol. Given equation is
2
+ 7x + 4 2
+ 3x + 2 10
Þ 4 [9 (1 - 2y + y ) + (8a + a y + 64y + 8ay )] ³ 0
2 2 2
103 x – 2 × 10x =
x + x2
Þ [9 - 18y + 9y 2 + 8a + a 2y + 64y + 8ay 2 ] ³ 0 10
4x 2 + 8x + 4 2x 2 + 4x + 2
Þ [y 2 (8a + 9 ) + y (a 2 + 46) + (8a + 9 )] ³ 0 Þ 10 – 2 × 10 = 10
2 2
+ 2 x + 1) + 2 x + 1)
We know, if ax 2 + bx + c ³ 0, " x and a > 0 Þ D £ 0 Þ 104 (x – 2 × 102 (x = 10
So, (a + 46) - 4 (8a + 9 ) (8a + 9 ) £ 0 and (8a + 9 ) > 0
2 2 + 1) 2
2 ( x + 1) 2
Þ 104 (x – 2 × 10 = 10
Þ (a + 46) - [2 (8a + 9 )] £ 0
2 2 2
and a > - 9/8 Þ {10 2 ( x + 1) 2 2
} – 2 × {10 2 ( x + 1) 2
} = 10 …(i)
Þ (a 2 + 46 - 16a - 18) (a 2 + 46 + 16a + 18) £ 0 + 1) 2
Let 102 (x =y …(ii)
and a > - 9/8
Þ y – 2y = 10 Þ y – 2y – 10 = 0
2 2
(a - 16 a + 28) (a + 16 a + 64 ) £ 0 and a > -9 /8
2 2

2 ± 4 + 40
Þ (a - 14 ) (a - 2) (a + 8)2 £ 0 and a > - 9 / 8 Þ y=
2
+ +
2 – 14 Þ y = 1 ± 11
Þ y = 1 + 11 [neglecting –ve sign as y > 0]
a Î [2, 14 ] È {–8 } and a > - 9 / 8
2
Thus, a Î[2, 14 ] Þ 102 (x + 1)
= 1 + 11
Hence, f is onto when a Î[2, 14 ] .
Þ 2 ( x + 1) = log10 (1 + 11 )
2
3x 2 + 6x - 8
Again, if a = 3, we have, f ( x ) = . 1
3 + 6x - 8x 2 Þ ( x + 1)2 = log10 (1 + 11 )
2
Clearly, f (1) = f (–1), which shows f ( x ) is not one-one.
1
Þ x +1= ± log10 (1 + 11 )
l Ex. 43 Let f ( x , y ) be a periodic function satisfying the 2
condition f ( x , y ) = f (( 2 x + 2y ), ( 2y - 2 x )), " x , y Î R. Now, 1
Hence, x = –1 ± log10 (1 + 11 )
define a function g by g ( x , 0 ) = f ( 2 x , 0 ). Then, prove that 2
g ( x ) is a periodic function and find its period.
Sol. We have f ( x , y ) = f (2x + 2y , 2y – 2x ) …(i) l Ex. 45 Find the real solution of
Þ f (2x + 2y , 2y – 2x ) = f {2(2x + 2y ) + 2(2y – 2x ), [ x ] + [ 5 x ] + [10 x ] + [ 20 x ] = 36K + 35, K Îinteger, where [× ]
2(2y – 2x ) – 2(2x + 2y )} [using Eq. (i)] denotes the greatest integral function.
Þ f ( x , y ) = f (2x + 2y , 2y – 2x ) = f (8y , – 8x ) Sol. Let x = a + r , where a Î integer and r Î[0, 1). Then, there
are four cases
Þ f ( x , y ) = f (8y ,– 8x ) …(ii)
160 Textbook of Differential Calculus

1 1 1 1900 f (1990) 2000 f (1990)


(i) 0 £ r < (ii) £ r< Þ < < Þ 21.1 < < 22.2
20 20 10 90 90 90 90
1
£ r<
1 1
(iv) £ r < 1 é f (1990) ù
(iii) \ êë 90 úû = 21,22 …(i)
10 5 5
1 éxù é f (x )ù
Case I If 0 £ r < × Given, x – f ( x ) = 19 ê ú –90 ê .
20 ë 19 û ë 90 úû
\ [ x ] + [ 5x ] + [10x ] + [20x ] = 36K + 35 Now, there are two cases arise.
Þ [a + r ] + [ 5a + 5r ] + [10a + 10r ] + [20a + 20r ] = 36K + 35 é f (1990) ù
Þ a + 5a + 10a + 20a = 36K + 35
Case I êë 90 úû = 21
[by definition of the greatest integer function] é 1990 ù é f (1990) ù
Þ 36a = 36K + 35, We have, 1990 – f (1990) = 19 ê ú –90 ê
ë 19 û ë 90 úû
which is not possible for any values of a and K as they
1990 - f (1990) = 19 ×(104 ) - 90 × (21)
belongs to integers. …(i)
Þ f (1990) = 1904 … (ii)
1 1
Case II If £r < × é f (1990) ù
20 10 Case II êë 90 úû = 22
\ [ x ] + [5x ] + [10x ] + [20x ] = 36K + 35
Þ [a + r ] + [5a + 5r ] + [10a + 10r ] + [20a + 20r ] é 1990 ù é f (1990) ù
We have, 1990 – f (1990) = 19 ê ú –90 ê
= 36K + 35 ë 19 û ë 90 úû
Þ a + 5a + 10a + 20a + 1 = 36K + 35 Þ f (1990) = 1994 … (iii)
Þ 36a + 1 = 36K + 35 From Eqs. (ii) and (iii), we have f (1990) = 1904 or 1994.
Þ 36a = 36K + 34,
which is again not possible, " a, K Î integer. …(ii)
l Ex. 47 Solve the system of equations;
1 1 | x 2 – 2 x | + y = 1, x 2 + | y | = 1.
Case III If £r < ×
10 5 Sol. Here, | x 2 – 2x | + y = 1 and x 2 + | y | = 1 gives four cases as
\ [ x ] + [5x ] + [10x ] + [20x ] = 36K + 35 Case I x 2 – 2x ³ 0 and y ³ 0 …(i)
Þ [a + r ] + [5a + 5r ] + [10a + 10r ] + [20a + 20r ] = 36K + 35
Equations are x – 2x + y = 1 and
2
x +y = 1
2
Þ a + 5a + 10a + 1 + 20a + 2 = 36K + 35
Þ 1 – 2x = 1 Þ x = 0 and y = 1 …(ii)
Þ 36a + 3 = 36K + 35
From Eq. (i), x 2 – 2x ³ 0 Þ x £ 0 x ³ 2 and y ³ 0
Þ 36a = 36K + 32,
Thus, x = 0 and y = 1 is the only solution when
which shows no value of a and K can be given. …(iii)
1 x 2 – 2x ³ 0 and y ³0
Case IV If £ r < 1.
5 Case II x 2 – 2x ³ 0 and y <0
\ [ x ] + [5x ] + [10x ] + [20x ] = 36K + 35 Þ ( x £ 0 or x ³ 2) and (y < 0) …(iii)
Þ [a + r ] + [5a + 5r ] + [10a + 10r ] + [20a + 20r ] = 36 K + 35 \ Equations are x 2 – 2x + y = 1 and x 2 – y = 1.
Þ a + 5a + 1 + 10a + 2 + 20a + 4 = 36K + 35 Adding these equations, we get
Þ 36a + 7 = 36K + 35 2x 2 – 2x = 2
Þ 36a = 36K + 28 1± 5 1± 5
or x= and y = …(iv)
Again, no solution is possible. …(iv) 2 2
Hence, from Eqs. (i), (ii), (iii), (iv) there exists no real value 1– 5 1– 5
which possesses solution. From Eqs. (iii) and (iv), x = and y =
2 2
l Ex. 46 Let f : N ® N be a function such that Case III x 2 – 2x £ 0 and y ³ 0.

éxù é f (x )ù Þ ( 0 £ x £ 2) and ( y ³ 0) …(v)


(i) x – f ( x ) = 19 ê ú – 90 ê ú , " x Î N, where [× ]
ë19 û ë 90 û Equations are –x + 2x + y = 1 and
2

denotes the greatest integer function. x 2 + y = 1, subtracting, we get


(ii) 1900 < f (1990 ) < 2000. –2x 2 + 2x = 0 Þ x = 0, 1
Find all the possible values of f (1990). i.e. x = 0, y = 1 or x = 1, y = 0 …(vi)
Sol. Since, 1900 < f (1990) < 2000 which satisfy Eq. (v).
Chap 03 Functions 161

Case IV x 2 – 2x £ 0 and y £ 0. l Ex. 49 If p and q are positive integers, f is a function


Þ (0 £ x £ 2) and ( y £ 0) …(vii) defined for positive numbers and attains only positive values
\ Equations are –x 2 + 2x + y = 1 and x 2 – y = 1, such that f ( x f (y )) = x p y q , then prove that q = p 2 .
adding, we get 2x = 2 Þ x = 1 Sol. For x =
1
, we have
and y = 0 which satisfies Eq. (vii). f (y )
Thus, solutions are æ 1ö 1
f çx × ÷ = × yq
æ 1– 5 ö è x ø f (y )p
( x = 0, y = 1) ( x = 1, y = 0) and ç x = y = ÷.
è 2 ø yq
Þ f ( 1) =
{ f (y )} p
l Ex. 48 Let f and g be real-valued functions such that
y q/p
f ( x + y ) + f ( x – y ) = 2 f ( x ) × g (y ), " x , y Î R. Prove that, if Þ f (y ) =
f ( x ) is not identically zero and | f ( x )| £1, " x Î R, then { f (1)}1/p
| g (y )| £1, " y Î R. For y =1, we have f (1) = 1
Sol. Let maximum value of f ( x ) be M. \ f (y ) = y q/p
Þ max | f ( x )| = M , where 0 < M £ 1 …(i) or f ( x ) = x q/p …(i)
[since, f is not identically zero and | f ( x )| £ 1, " x Î R ]
Hence, f ( x × y q/p ) = x p × y q
Now, f ( x + y ) + f ( x – y ) = 2 f ( x ) × g (y )
Let y q/p = z
Þ | 2f ( x )| × | g (y )| = | f ( x + y ) + f ( x – y )|
Þ 2 | f ( x )| × | g (y )| £ | f ( x + y )| + | f ( x – y )| Þ y = z p/q

[as | a + b | £ | a | + | b | ] Þ f ( x ×z ) = x p ×z p
Þ 2 | f ( x )| × | g (y )| £ M + M or f (x ) = x p …(ii)
[using Eq. (i), i.e. max | f ( x )| = M ] From Eqs. (i) and (ii), we have x q/p
=x p

Þ | g (y )| £ 1 , " y Î R q
Þ = p or q = p 2
Thus, if f ( x ) is not identically zero and | f ( x )| £ 1, " x Î R, p
then | g (y )| £ 1, " y Î R.
162 Textbook of Differential Calculus

Functions Exercise 1 :
Single Option Correct Type Questions
ì x , for 0 £ x £ 1 f 2 ( x ) = f 1 ( - x ), for all x 7. If f ( x ) = 2x 3 + 7 x - 9, then f -1
( 4 ) is
1. f 1 ( x ) = ïí 1, for x > 1 and f 3 ( x ) = - f 2 ( x ), for all x . (a) 1 (b) 2 (c) 1/3 (d) non-existent
ï 0, otherwise f 4 ( x ) = f 3 ( -x ), for all x
î 8. The range of the function
2
+2
Which of the following is necessarily true? e x × log x × 5 x × ( x 2 - 7 x + 10)
(a) f 4 ( x ) = f1( x ), for all x (b) f1( x ) = - f 3( - x ), for all x f (x ) = is
2x 2 - 11x + 12
(c) f 2( - x ) = f 4 ( x ), for all x (d) f1( x ) + f 3( x ) = 0, for all x
æ3 ö æ3 ö
2. Which one of the following is an odd function? (a) ( - ¥, ¥ ) (b) [ 0, ¥ ) (c) ç , ¥ ÷ (d) ç , 4 ÷
è2 ø è2 ø
Y
10
Y
9. If x = cos -1 (cos 4 ) and y = sin -1 (sin 3), then which of the
10
following holds?
(a) X′ X (b) X′ X (a) x - y = 1 (b) x + y + 1 = 0
0 10 –10 0 10
(c) x + 2y = 2 (d) x + y = 3 p - 7
2/ 3
–10 –10 æ 2 sin x + sin 2x 1 - cos x ö
Y′ Y′ 10. Let f ( x ) = ç × ÷ ; x Î R.
Y è 2 cos x + sin 2x 1 - sin x ø
Y
10
Consider the following statements.
0
I. Domain of f is R.
(c) X′ 0 X (d) X′ X II. Range of f is R.
–10 10 –10 –5 5 10
p
–10 III. Domain of f is R - ( 4n - 1) , n Î I .
Y′ Y′ 2
p
8 8 4 4 IV. Domain of f is R - ( 4n + 1) , n Î I .
3. If f ( x ) = + and g ( x ) = + , 2
1- x 1+ x f (sin x ) f (cos x )
Which one of the following is correct?
the g ( x ) is periodic with period (a) I and II (b) II and III
p 3p
(a) (b) p (c) (d) 2p (c) III and IV (d) II, III and IV
2 2
11. If f ( x ) = e sin(x - [ x ]) cos px , where [x ] denotes the greatest
f (x )
4. Let f be a function defined by f ( xy ) = for all integer function, then f ( x ) is
y
(a) non-periodic
positive real numbers x and y. If f (30) = 20, the value of (b) periodic with no fundamental period
f ( 40) is (c) periodic with period 2
(a) 15 (b) 20 (c) 40 (d) 60 (d) periodic with period p
{e| x | sgn x } [e| x| sgn x ]
5. Let f ( x ) = e and g ( x ) = e , x Î R, where 12. The range of the function,
{x } and [x ] denote fractional part and greatest integer f ( x ) = cot -1 (log 0. 5 ( x 4 - 2x 2 + 3)) is
function, respectively. Also, h( x ) = log( f ( x )) + log( g ( x )), (a) ( 0, p)
æ 3p ù
(b) ç 0,
é 3p ö
(c) ê , p ÷
é p 3p ù
(d) ê ,
then for all real x, h( x ) is è 4 úû ë 4 ø ë 2 4 úû
(a) an odd function é ù 1 1
(b) an even function 13. Range of f ( x ) = ê ú+ , where [×]
êë log( x + e ) úû
2
(c) neither odd nor even function 1+ x2
(d) both odd as well as even function denotes greatest integer function, is
6. Which of the following function is surjective but not æ e + 1ö
(a) ç 0, ÷ È {2 } (b) ( 0, 1 )
injective? è e ø
(a) f : R ® R , f ( x ) = x 4 + 2 x 3 - x 2 + 1 (c) ( 0, 1 ] È {2 } (d) [ 0, 1 ) È {2 }
(b) f : R ® R , f ( x ) = x 3 + x + 1 14. The period of the function f ( x ) = sin ( x + 3 - [x + 3]),
(c) f : R ® R + , f ( x ) = 1 + x 2 where [×] denotes greatest integer function, is
(d) f : R ® R , f ( x ) = x 3 + 2 x 2 - x + 1 (a) 2 p + 3 (b) 2p (c) 1 (d) 4
Chap 03 Functions 163

15. Which one of the following functions best represent the 22. The area between the curve 2{y } = [x ] + 1 , 0 £ y < 1 ,
graph as shown below? where {×} and [×] are the fractional part and greatest
Y integer functions, respectively and the X-axis, is
1 3
(0, 1) (a) (b) 1 (c) 0 (d)
2 2
X′
O
X 23. If f ( x ) = sin -1 x and g ( x ) = [sin (cos x )] + [cos (sin x )],
then range of f ( g ( x )) is (where, [×] denotes greatest
integer function)
Y′
ì -p p ü ì -p ü
1 1 (a) í , ý (b) í ,0ý
(a) f ( x ) = (b) f ( x ) = î 2 2þ î 2 þ
1 + x2 1 + |x|
ì pü ì p pü
(c) í 0, ý (d) í - ,0, ý
(c) f ( x ) = e - | x | (d) f ( x ) = a| x | , a > 1 î 2þ î 2 2þ
16. The solution set for [x ] { x } = 1, where {x } and [x ] denote 24. The number of solutions of the equation
fractional part and greatest integer functions, is e 2 x + e x - 2 = [{x 2 + 10x + 11}] is
+ +
(a) R - ( 0, 1 ) (b) R - {1 } (where, {x } denotes fractional part of x and [x ] denotes
ì 1 ü ì 1 ü greatest integer function)
(c) ím + ; m Î I - { 0 }ý (d) ím + ; m Î N - {1 }ý
î m þ î m þ (a) 0 (b) 1 (c) 2 (d) 3
17. The domain of definition of function 1
25. Total number of values of x, of the form , n Î N in the
n
f ( x ) = log( x 2 - 5x - 24 - x - 2), is é1 1ù
interval x Î ê , ú, which satisfy the equation
(a) ( - ¥, -3 ] (b) ( - ¥ - 3 ] È [8, ¥ ) ë 25 10 û
æ - 28 ö
(c) ç -¥, ÷ (d) None of these {x } + {2x } + ... + {12x } = 78x (where, {×} represents
è 9 ø
fractional part function), is
18. If f ( x ) is a function f : R ® R , we say f ( x ) has property (a) 12 (b) 13
(c) 14 (d) 15
I. If f ( f ( x )) = x for all real numbers x.
II. If f ( - f ( x )) = - x for all real numbers x. 26. The sum of the maximum and minimum values of the
1
How many linear functions, have both property I and II? functionf ( x ) = is
(a) 0 (b) 2 1 + (2 cos x - 4 sin x ) 2
(c) 3 (d) Infinite 22 21 22 21
x rx (a) (b) (c) (d)
19. Let f ( x ) = and g ( x ) = . Let S be the set of all 21 20 20 11
1+ x 1- x 27. Let f : X ® Y , f ( x ) = sin x + cos x + 2 2 be invertible,
real numbers r, such that f ( g ( x )) = g ( f ( x )) for infinitely then X ® Y is/are
many real numbers x. The number of elements in set S is
é p 5p ù
(a) 1 (b) 2 (a) ê , ú ® [ 2,3 2 ]
(c) 3 (d) 5 ë4 4 û

20. Let f ( x ) be linear functions with the properties that é p 3p ù


(b) ê - , ú ® [ 2, 3 2 ]
ë 4 4 û
f (1) £ f (2) , f (3) ³ f ( 4 ) and f (5) = 5. Which one of the
following statements is true? é 3p 3p ù
(c) ê - , ú ® [ 2, - 3 2 ]
(a) f ( 0 ) < 0 (b) f ( 0 ) = 0 ë 4 4 û
(c) f (1 ) < f ( 0 ) < f ( - 1 ) (d) f ( 0 ) = 5 é 3p p ù
(d) ê - ,- ú ® [ 2, 3 2 ]
21. Suppose R is relation whose graph is symmetric to both ë 4 4û
X -axis andY -axis and that the point (1, 2) is on the graph 28. The range of values of a, so that all the roots of the
of R. Which one of the following is not necessarily on equation 2x 3 - 3x 2 - 12x + a = 0 are real and distinct,
the graph of R? belongs to
(a) ( -1, 2 ) (b) (1, - 2 ) (a) (7, 20) (b) (–7, 20)
(c) ( - 1, - 2 ) (d) (2, 1 ) (c) (–20, 7) (d) (–7, 7)
164 Textbook of Differential Calculus

29. If f ( x ) is continuous such that | f ( x ) | £ 1, " x Î R and 36. If a function satisfies f ( x + 1) + f ( x - 1) = 2 f ( x ), then
e f (x ) - e | f (x ) | period of f ( x ) can be
g( x ) = , then range of g ( x ) is (a) 2 (b) 4 (c) 6 (d) 8
e f (x )
+e | f (x ) |

é e2 + 1ù 37. If x and a are real, then the inequation


(a) [0, 1] (b) ê 0, 2 ú log 2 x + log x 2 + 2 cos a £ 0
ë e - 1û
(a) has no solution
é e2 - 1 ù é 1 - e2 ù
(c) ê 0, 2 ú (d) ê , 0ú (b) has exactly two solutions
ë e + 1û ë1 + e
2
û (c) is satisfied for any real a and any real x in (0, 1)
(d) is satisfied for any real a and any real x in (1, ¥ )
30. Let f ( x ) = | x | - {x } , where {×} denotes the fractional
|x|
38. The range of values of ‘a’ such that æç ö÷
1
part of x and X , Y and its domain and range respectively, = x 2 - a is
then è2ø
(a) f : X ® Y : y = f ( x ) is one-one function satisfied for maximum number of values of ‘x’
æ 1ù é1 ö (a) ( - ¥, - 1 ) (b) ( - ¥, ¥ ) (c) ( -1, 1 ) (d) ( -1, ¥ )
(b) X Î ç - ¥, - ú È [ 0, ¥ ) and Y Î ê , ¥ ÷
è 2û ë2 ø 39. Let f : R ® R be a function defined by f ( x ) = {| cos x | },
æ 1ù where {x } represents fractional part of x. Let S be the set
(c) X Î ç - ¥, - ú È [ 0, ¥ ) and Y Î [ 0, ¥ )
è 2û containing all real values x lying in the interval [0, 2p ]
(d) None of the above for which f ( x ) ¹ | cos x |. The number of elements in the
31. If the graphs of the functions y = ln x and y = ax set S is
(a) 0 (b) 1 (c) 3 (d) infinite
intersect at exactly two points, then a must be
æ1 ö 40. The domain of the function
(a) ( 0, e ) (b) ç , 0 ÷
èe ø f ( x ) = log sin x + cos x ( | cos x | + cos x ), 0 £ x £ p is
æ 1ö æ pö
(c) ç 0, ÷ (d) None of these (a) ( 0, p ) (b) ç 0, ÷
è eø è 2ø
32. A quadratic polynomial maps from [-2, 3] onto [0, 3] and æ pö
(c) ç 0, ÷ (d) None of these
touches X -axis at x = 3, then the polynomial is è 3ø
3 2 3 2 41. If f ( x ) = ( x 2 + 2 ax + a 2 - 1)1/ 4 has its domain and
(a) ( x - 6 x + 16 ) (b) (x - 6x + 9)
16 25
3 2 3 2
range such that their union is set of real numbers, then
(c) ( x - 6 x + 16 ) (d) (x - 6x + 9) a satisfies
25 16
(a) -1 < a < 1 (b) a £ - 1
3 (c) a ³ 1 (d) a £ 1
33. The range of the function y = 2 { x } - { x } - 2
4 42. Let f : (e , ¥ ) ® R be a function defined by
(where, { × } denotes the fractional part) is
f ( x ) = log (log (log x )), the base of the logarithm being e.
é 1 1ù é 1ö
(a) ê - , ú (b) ê 0, ÷ Then,
ë 4 4û ë 2ø
é 1ù é 1 1ù (a) f is one-one and onto
(c) ê 0, ú (d) ê , ú (b) f is one-one but not onto
ë 4û ë 4 2û
(c) f is onto but not one-one
34. Let f ( x ) be a fourth differentiable function such that (d) the range of f is equal to its domain
f (2x 2 - 1) = 2xf ( x ), " x Î R, then f iv (0) is equal to 43. The expression x 2 - 4 px + q 2 > 0 for all real x and also
(where, f iv
(0) represents fourth derivative of f ( x ) at x +r
r 2 + p 2 < qr , the range of f ( x ) = is
x = 0) x + qx + p 2
2

(a) 0 (b) 1 ép qù
(c) – 1 (d) Data insufficient (a) ê , ú (b) ( 0, ¥ )
ë 2r 2r û
35. Number of solutions of the equation [y + [y ]] = 2 cos x is (c) ( - ¥, 0 ) (d) ( - ¥, ¥ )
1 x - lx - 3x 2 + 3lx
4 3
(where, y = [sin x + [sin x + [sin x ]]] and [×] denotes 44. Let f ( x ) = . If range of f ( x ) is the
3 x -l
the greatest integer function) set of entire real numbers, the true set in which l lies is
(a) 1 (b) 2 (a) [ -2, 2 ] (b) [ 0, 4 ]
(c) 3 (d) None of these (c) (1, 3 ) (d) None of these
Chap 03 Functions 165

45. Let a = 31/ 224 + 1 and for all n ³ 3, 53. Let f ( x ) be a polynomial with real coefficients such that
let f (n ) = C 0 a
n n -1
- C 1a
n n -2
+ C 2a
n n -3 f ( x ) = f ¢ ( x ) ´ f ¢ ¢ ¢ ( x ). If f ( x ) = 0 is satisfied x = 1, 2, 3
only, then the value of f ¢ (1) f ¢ (2) f ¢ (3) is
+ ... + ( -1)n -1 ×n C n -1 × a 0 .
(a) positive (b) negative
If the value of f (2016) + f (2017 ) = 3 K , the value of K is (c) 0 (d) Inadequate data
(a) 6 (b) 8 54. Let A = {1, 2, 3, 4, 5} and f : A ® A be an into function
(c) 9 (d) 10 such that f (i ) ¹ i, " i Î A, then number of such functions
46. The area bounded by f ( x ) = sin -1 (sin x ) and f are
2 (a) 1024 (b) 904
p p2 æ pö (c) 980 (d) None of these
g( x ) = - - ç x - ÷ is
2 2 è 2ø 55. If functions f : {1, 2, . . . , n } ® {1995, 1996} satisfying
p3 p2 f (1) + f (2) + . . . + f (1996) = odd integer are formed, the
(a) sq units (b) sq units
8 8 number of such functions can be
p3 p2 (a) 2n (b) 2n/2 (c) n 2 (d) 2n - 1
(c) sq units (d) sq units
2 2
56. The range of y = sin 3 x - 6 sin 2 x + 11 sin x - 6 is
x + bx + 1
2
1 (a) [ - 24, 2 ] (b) [ - 24, 0 ]
47. If f : R ® R, f ( x ) = , (b > 1) and f ( x ) ,
x + 2x + b 2 f ( x) (c) [ 0, 24 ] (d) None of these
have the same bounded set as their range, the value of b 57. Let f ( x ) = x 2 - 2x and g ( x ) = f ( f ( x ) - 1) + f (5 - f ( x )),
is
then
(a) 2 3 - 2 (b) 2 3 + 2
(a) g( x ) < 0, "x Î R
(c) 2 2 - 2 (d) 2 2 + 2
(b) g( x ) < 0, for some x Î R
p [x ] p [x ] p [x ] g( x ) ³ 0, for some x Î R
48. The period of sin + cos + tan , where (c)
12 4 3 (d) g( x ) ³ 0, "x Î R
[x ] represents the greatest integer less than or equal to x 58. If f ( x ) and g ( x ) are non-periodic functions, then
is
(a) 12 (b) 4
h( x ) = f ( g ( x )) is
(c) 3 (d) 24 (a) non-periodic
(b) periodic
49. If f (2x + 3y, 2x - 7y ) = 20x , then f ( x , y ) equals (c) may be periodic
(a) 7 x - 3y (b) 7 x + 3y (d) always periodic, if domain of h( x ) is a proper subset of
(c) 3 x - 7y (d) x - y real numbers
50. The range of the function f ( x ) = x - 1 + 2 3 - x is 59. If f ( x ) is a real-valued function discontinuous at all
(a) [ 2, 2 2 ] (b) [ 2, 10 ] integral points lying in [0, n ] and if ( f ( x )) 2 = 1,
(c) [2 2, 10 ] (d) [1, 3 ] " x Î [0, n ], then number of functions f ( x ) are
51. The domain of the function (a) 2n + 1 (b) 6 ´ 3n (c) 2 ´ 3n - 1 (d) 3n + 1
f ( x ) = cos -1 (sec (cos -1 x )) 60. A function f from integers to integers is defined as
-1 -1
+ sin ( cosec (sin x )) is ìn + 3, n Î odd
(a) x Î R (b) x = 1, - 1 f (x ) = í
(c) -1 £ x £ 1 (d) x Î f
î n /2, n Î even
Suppose k Î odd and f ( f ( f (k ))) = 27, then the sum of
52. Let f ( x ) be a polynomial one-one function such that digits of k is
f ( x ) f (y ) + 2 = f ( x ) + f (y ) + f ( xy ), " x, y Î R - {0}, (a) 3 (b) 6 (c) 9 (d) 12
f (1) ¹ 1, f ¢ (1) = 3. sin ( p {x })
x x 61. If f : R ® R and f ( x ) = , where { } is a
Let g ( x ) = ( f ( x ) + 3) - ò f ( x ) dx , then x 4 + 3x 2 + 7
4 0
fractional part of x, then
(a) g( x ) = 0 has exactly one root for x Î( 0, 1 ) (a) f is injective
(b) g( x ) = 0 has exactly two roots for x Î( 0, 1 ) (b) f is not one-one and non-constant
(c) g( x ) ¹ 0, " x Î R - { 0 } (c) f is a surjective
(d) g( x ) = 0, " x Î R - { 0 } (d) f is a zero function
166 Textbook of Differential Calculus

62. Let f : R ® R and g : R ® R be two one-one and onto 69. The sum of the maximum and minimum values of
functions, such that they are the mirror images of each other function f ( x ) = sin -1 2x + cos -1 2x + sec -1 2x is
about the line y = a. If h ( x ) = f ( x ) + g ( x ), then h ( x ) is
p
(a) one-one and onto (a) p (b)
2
(b) only one-one and not onto 3p
(c) only onto but not one-one (c) 2p (d)
2
(d) None of the above
70. The complete set of values of ‘a’ for which the function
63. Domain of the function f ( x ), if 3 x + 3 f (x ) = minimum of
f ( x ) = tan -1 ( x 2 - 18x + a ) > 0, " x Î R, is
f(t ), where f (t ) = min {2t 3 - 15t 2 + 36t - 25, 2 + |sin t | } (a) (81, ¥ ) (b) [81, ¥ )
is (c) ( - ¥, 81 ) (d) ( - ¥, 81 ]
(a) ( - ¥, 1 ) 71. The domain of the function
(b) ( - ¥, log 3 e ) 1 1
(c) ( 0, log 3 2 ) f ( x ) = sin -1 + is
(d) ( - ¥, log 3 2 )
| x - 1|
2
sin 2 x + sin x + 1
64. Let x be the elements of the set (a) ( - ¥, ¥ )
(b) ( - ¥, - 2 ] È [ 2, ¥ )
A = {1,2,3,4,5,6,8,10,12,15,20,24,30,40,60,120} and
x 1 , x 2 , x 3 be positive integers and d be the number of (c) ( - ¥, - 2 ] È [ 2, ¥ ) È { 0 }
integral solutions of x 1 x 2 x 3 = x , then d is (d) None of the above
(a) 100 (b) 150 log (sin -1 x 2 + x + 1)
(c) 320 (d) 250 72. The domain of f ( x ) = is
log ( x 2 - x + 1)
65. If A > 0, c , d , u, v are non-zero constants and the graph of
(a) ( -1, 1 ) (b) ( -1, 0 ) È ( 0, 1 )
f ( x ) = Ax + c + d and g ( x ) = - Ax + u + v intersect
(c) ( -1, 0 ) È {1 } (d) None of these
exactly at two points (1, 4 ) and (3, 1), then the value of
u +c 73. The domain of f ( x ) = sin -1 (3x - 4 x 3 ) + cos -1 x is
equals
A equal to
(a) 4 (b) -4 é 3ù é 3ù é 1ù é 1ù
(c) 2 (d) -2 (a) ê -1, - ú È ê 0, ú (b) êë -1, - 2 úû È êë 0, 2 úû
ë 2 û ë 2 û
66. If f ( x ) = x 3 + 3x 2 + 4 x + a sin x + b cos x , "x Î R is a
é 1ù
one-one function, then the greatest value of (a 2 + b 2 ) is (c) ê 0, ú (d) None of these
ë 2û
(a) 1 (b) 2
(c) 2 (d) None of these 74. The domain of the function
67. If two roots of the equation f ( x ) = 6 4 x + 8 2 / 3 (x - 2 ) - 52 - 2 2 (x - 1) is
( p - 1)( x 2 + x + 1) 2 - ( p + 1)( x 4 + x 2 + 1) = 0 are real and (a) ( 0, 1 ) (b) [3, ¥ )
1- x æ æ 1 öö (c) [1, 0 ) (d) None of these
distinct and f ( x ) = , then f ( f ( x )) + f ç f ç ÷ ÷ is
1+ x è è x øø 75. The domain of derivative of the function
equal to f ( x ) = | sin -1 (2x 2 - 1) | is
(a) p (b) -p ì 1 ü
(c) 2p (d) -2p (a) ( -1, 1 ) (b) ( -1, 1 ) ~ í 0, ± ý
î 2þ
68. Let f ( x ) = x 13 + 2x 12 + 3x 11 +... + 13x + 14 and ì 1 ü
(c) ( -1, 1 ) ~ { 0 } (d) ( -1, 1 ) ~ í ± ý
2p 2p î 2þ
a = cos + i sin . If N = f (a ) f (a 2 ).... f (a 14 ), then
15 15 76. The range of a function
(a) number of divisors of N is 144
f ( x ) = tan -1 {log 5 / 4 (5x 2 - 8x + 4 )} is
(b) number of divisors of N is 196
æ -p p ö é -p pö
(c) number of divisors of N which are perfect squares (a) ç , ÷ (b) ê , ÷
è 4 2ø ë 4 2ø
of 49
æ -p p ù é -p pù
(d) number of divisors of N which are perfect square (c) ç , (d) ê ,
è 4 2 úû ë 4 2 úû
of 12
Functions Exercise 2 :
More Than One Option Correct Type Questions
77. Which of the following function(s) is/are transcendal? 86. Let f ( x ) = x - 1 + x - 2 + x - 3 + x - 4 , then
2 sin 3 x
(a) f ( x ) = 5 sin( x ) (b) f ( x ) = 2 (a) least value of f ( x ) is 4
x + 2x - 1 (b) least value is not attained at unique point
(c) f ( x ) = x 2 + 2 x + 1 (d) f ( x ) = ( x 2 + 3 ) × 2 x (c) the number of integral solution of f ( x ) = 4 is 2
f ( p - 1 ) + f (e )
x -2 x -1 (d) the value of is 1
78. Let f ( x ) = × x , then æ 12 ö
2fç ÷
x -1 -1 è5ø
(a) domain of f ( x ) is x ³ 1 (b) domain of f ( x ) is [1, ¥ ) - {2 } 87. Let A = {1, 2, 3, 4, 5}, B = {1, 2, 3, 4} and f : A ® B is a
æ3ö
(c) f ¢(10 ) = 1 (d) f ¢ ç ÷ = - 1 function, the
è2ø
(a) number of onto functions, if n( f ( A )) = 4 is 240
p p
79. f ( x ) = cos 2 x + cos 2 æç + x ö÷ - cos x × cos æç x + ö÷ is (b) number of onto functions, if n( f ( A )) = 3 is 600
è 3 ø è ø 3 (c) number of onto functions, if n( f ( A )) = 2 is 180
(a) an odd function (b) an even function (d) number of onto functions, if n( f ( A )) = 1 is 4
(d) f ( 0 ) = f (1 ) æ 1 ö
88. If 2 f ( x ) + x f æç ö÷ - 2 f ç 2 sin p æç x + ö÷ ÷
(c) a periodic function 1
80. If the following functions are defined from [-1, 1 ] to èx ø è è 4ø ø
[- 1, 1 ], identify these which are into.
æ px ö æpö
2 = 4 cos 2 ç ÷ + x cos ç ÷, " x Î R - {0}, which of the
(a) sin(sin -1 x ) (b) × sin -1(sin x ) è 2 ø èx ø
p
(c) sgn ( x ) × log(e x ) (d) x 3sgn ( x ) following statement(s) is/are true?
æ1ö
ì x 2 - 4 x + 3, x <3 (a) f (2 ) + f ç ÷ = 1 (b) f (2 ) + f (1 ) = 0
81. Let f ( x ) = í è2ø
î x - 4, x ³3 æ1ö æ1ö
(c) f (2 ) + f (1 ) = f ç ÷ (d) f (1 ) × f ç ÷ × f (2 ) = 1
ì x - 3, x<4 è2ø è2ø
and g ( x ) = í 2 , which one of the
î x + 2x + 2, x³4 89. If f ( x ) is a differentiable function satisfying the
following is/are true? condition f (100x ) = x + f (100x - 100), " x Î R and
(a) ( f + g )(3.5 ) = 0 (b) f ( g(3 )) = 3 f (100) = 1, then f (10 4 ) is
(c) f ( g(2 )) = 1 (d) ( f - g ) ( 4 ) = 0 100 100

82. If f ( x ) = x - 2ax + a(a + 1), f : [a, ¥ ) ® [a, ¥ ). If one of


2 (a) 5049 (b) år (c) år (d) 5050
-1 r =1 r=2
the solutions of the equation f ( x ) = f ( x ) is 5049, the
other may be 90. If [x ] denotes the greatest integer function then the
(a) 5051 (b) 5048 (c) 5052 (d) 5050 extreme values of the function
83. The function g defined by f ( x ) = [1 + sin x ] + [1 + sin 2x ] + ¼ + [1 + sin nx ], n Î I + ,
g ( x ) = sin a + cos a - 1; a = sin -1 { x }, where {×} denotes x Î(0, p ) are
fractional part function, is (a) (n - 1 ) (b) n (c) (n + 1 ) (d) (n + 2 )
(a) an even function (b) periodic function 91. Which of the following is/are periodic?
(c) odd function (d) neither even nor odd
ì 1, if x is rational
84. The graph of f : R ® R defined by y = f ( x ) is symmetric (a) f ( x ) = í
î 0, if x is irrational
with respect to x = a and x = b. Which of the following
ì x - [ x ], 2n £ x < 2n + 1
is true? ï
(b) f ( x ) = í 1 , where [×]
(a) f (2a - x ) = f ( x ) (b) f (2a + x ) = f ( - x ) ïî 2 , 2n + 1 £ x < 2n + 2
(c) f (2b + x ) = f ( - x ) (d) f is periodic
denotes the greatest integer function
é 2x ù
85. Let f be the continuous and differentiable function such ê ú
(c) f ( x ) = ( -1 ) ë p û , where [×] denotes the greatest
that f ( x ) = f (2 - x ), " x Î R and g ( x ) = f (1 + x ), then
integer function
(a) g( x ) is an odd function
æpxö
(b) g( x ) is an even function (d) f ( x ) = ax - [ax + a ] + tan ç ÷ , where [×] denotes
è 2 ø
(c) f ( x ) is symmetric about x = 1
the greatest integer function
(d) None of the above
168 Textbook of Differential Calculus

92. If f ( x ) is a polynomial of degree n , such that f (0) = 0, 97. Let f ( x ) be a real valued function such that f (0) = and
1
n f ( x + y ) = f ( x ) f (a - y ) + f (y ) f (a - x ), 2
f (1) = 1/2, … , f (n ) = , then the value of f (n + 1) is
n +1 " x , y Î R, then for some real a,
n (a) f ( x ) is a periodic function
(a) 1, when n is even (b) , when n is odd (b) f ( x ) is a constant function
n+2
1 cos x
n (c) f ( x ) = (d) f ( x ) =
(c) 1, when n is odd (d) , when n is even 2 2
n+2
98. If f ( g ( x )) is one-one function, then
93. Let f : R ® R be a function defined by (a) g( x ) must be one-one (b) f ( x ) must be one-one
f (x ) - 5 (c) f ( x ) may not be one-one (d) g( x ) may not be one-one
f ( x + 1) = , " x Î R. Then, which of the
f (x ) - 3 99. Which of the following functions have their range equal
following statement(s) is/are true? to R (the set of real numbers)?
(a) f (2008 ) = f (2004 ) (b) f (2006 ) = f (2010 ) (a) x sin x
(c) f (2006 ) = f (2002 ) (d) f (2006 ) = f (2018 ) [x ] æ p pö
(b) × x Î ç - × ÷ - { 0 }, where [×] denotes the
tan 2 x è 4 4ø
94. Let f ( x ) = 1 - x - x 3 . Then, the real values of x greatest integer function
satisfying the inequality, x
(c)
1 - f ( x ) - f 3 ( x ) > f (1 - 5x ), are sin x
(a) ( -2, 0 ) (b) ( 0, 2 ) (d) [ x ] +{ x }, where [×] and {×}, respectively denote the
(c) (2, ¥ ) (d) ( - ¥, - 2 ) greatest integer and fractional part functions

95. If a function satisfies 100. Which of the following pairs of function are identical?
-1
( x - y ) f ( x + y ) - ( x + y ) f ( x - y ) = 2( x y - y ), and g( x ) = sec -1 x
2 3
(a) f ( x ) = e ln sec x

" x , y Î R and f (1) = 2, then (b) f ( x ) = tan (tan -1 x ) and g( x ) = cot (cot -1 x )
(a) f ( x ) must be polynomial function (c) f ( x ) = sgn ( x ) and g( x ) = sgn (sgn ( x ))
(b) f (3 ) = 12 (d) f ( x ) = cot 2 x × cos2 x and g( x ) = cot 2 x - cos2 x
(c) f (0) = 0
(d) f ( x ) may not be differentiable 101. Let f : R ® R defined by f ( x ) = cos -1 ( - {- x }), where {x }
denotes fractional part of x. Then, which of the
96. If the fundamental period of function
following is/are correct?
f ( x ) = sin x + cos ( 4 - a 2 ) x is 4p, then the value of a (a) f is many one but not even function
is/are (b) Range of f contains two prime numbers
15 15 7 7 (c) f is non-periodic
(a) (b) - (c) (d) -
2 2 2 2 (d) Graph of f does not lie below X -axis

Functions Exercise 3 :
Statements I and II Type Questions
n
Directions (Q. Nos. 102 to 112) For the following Statement II The derivative of differentiable function
questions, choose the correct answers from the codes (a), (non-periodic) is non-periodic function.
(b), (c) and (d) defined as follows : 103. Statement I The maximum value of sin 2 x + sin ax
(a) Statement I is true, Statement II is also true; Statement II cannot be 2 (where a is positive rational number).
is the correct explanation of Statement I 2
Statement II is irrational.
(b) Statement I is true, Statement II is also true; Statement II a
is not the correct explanation of Statement I e | x | - e -x
104. Let f : R ® R be a function such that f ( x ) = x .
(c) Statement I is true, Statement II is false e + e -x
(d) Statement I is false, Statement II is true Statement I f ( x ) is into function.
102. Statement I The function f ( x ) = x sin x Statement II f ( x ) is many-one function and the
many-one function is not onto.
and f ¢ ( x ) = x cos x + sin x are both non-periodic.
Chap 03 Functions 169

105. Statement I The range of æ 1 ö


æp ö æp ö æ 2p ö
109. Statement I The range of log ç ÷ is ( - ¥, ¥ ).
f ( x ) = sin ç + x ÷ - sin ç - x ÷ - sin ç + x ÷ è1 + x ø 2
è5 ø è5 ø è5 ø
Statement II When 0 < x £ 1, log x Î ( - ¥, 0] .
æ 2p ö
+ sin ç - x ÷ is [–1, 1]. 110. Let f : X ® Y be a function defined by
è5 ø
æ pö
p 2p 1 f ( x ) = 2 sin ç x + ÷ - 2 cos x + c .
Statement II cos - cos = è 4ø
5 5 2
é p ù é 3p ù
106. Statement I The period of Statement I For set X , x Î ê0, ú È ê p, ú, f ( x ) is
1 1
f ( x ) = 2 cos ( x - p ) + 4 sin ( x - p ) is 3p. ë 2û ë 2 û
3 3 one-one function.
Statement II If T is the period of f ( x ), then the period é pù
T Statement II f ¢ ( x ) ³ 0, x Î ê0, ú
of f (ax + b ) is . ë 2û
|a|
111. Let f ( x ) = sin x
107. f is a function defined on the interval [-1, 1] such that
f (sin 2x ) = sin x + cos x . Statement I f is not a polynomial function.
é p pù Statement II nth derivative of f ( x ), w.r.t. x, is not a
Statement I If x Î ê - , ú , then f (tan 2 x ) = sec x zero function for any positive integer n.
ë 4 4û
Statement II f ( x ) = 1 + x , " x Î [-1, 1] 112. Statement I The function f : R ® R, given
108. Statement I The equation f ( x ) = 4 x 5 + 20x - 9 = 0 has f ( x ) = loga ( x + x 2 + 1 ), a > 0, a ¹ 1 is invertible.
only one real root. Statement II f is many-one and into.
Statement II f ¢ ( x ) = 20x 4 + 20 = 0 has no real root.

Functions Exercise 4 :
Passage Based Questions
Passage I (Q. Nos. 113 to 115) Passage III (Q. Nos. 118 to 120)
Let f : R ® R be a continuous function such that 1 é æ x öù +
Let f ( x ) = ê f ( xy ) + f ç ÷ ú for x , y Î R such that f (1) = 0;
æx ö æx ö 2 ë è y ø û
f ( x ) - 2f ç ÷ + f ç ÷ = x 2.
è2ø è4ø f ¢ (1) = 2.
113. f (3) is equal to 118. f ( x ) - f (y ) is equal to
(a) f ( 0 ) (b) 4 + f ( 0 ) (c) 9 + f ( 0 ) (d) 16 + f ( 0 ) æy ö æxö
(a) f ç ÷ (b) f ç ÷ (c) f (2 x ) (d) f (2y )
èxø èy ø
114. The equation f ( x ) - x - f (0) = 0 have exactly
(a) no solution (b) one solution 119. f ¢(3) is equal to
(c) two solutions (d) infinite solutions 1 2 1 1
(a) (b) (c) (d)
3 3 2 4
115. f ¢ (0) is equal to
120. f (e ) is equal to
(a) 0 (b) 1 (c) f ( 0 ) (d) - f ( 0 ) (a) 2 (b) 1 (c) 3 (d) 4
Passage II (Q. Nos. 116 to 117) Passage IV (Q. Nos. 121 to 123)
Consider the equation x + y - [ x ][y ] = 0, where [×] is the greatest If f : R ® R and f ( x ) = g ( x ) + h ( x ), where g ( x ) is a polynomial
integer function. and h ( x ) is a continuous and differentiable bounded function on
both sides, then f ( x ) is one-one, we need to differentiate f ( x ). If
116. The number of integral solutions to the equation is f ¢( x ) changes sign in domain of f , then f , if many-one else
(a) 0 (b) 1 one-one.
(c) 2 (d) None of these 121. If f : R ® R and f ( x ) = a1 x + a 3 x 3
117. Equation of one of the lines on which the non-integral + a 5 x 5 + . . . + a 2n + 1 x 2n + 1 - cot -1 x
solution of given equation lies, is where 0 < a1 < a 3 < . . . < a 2n + 1 , then the function f ( x ) is
(a) x + y = - 1 (b) x + y = 0 (a) one-one into (b) many-one onto
(c) x + y = 1 (d) x + y = 5 (c) one-one onto (d) many-one into
170 Textbook of Differential Calculus

x ( x 4 + 1)( x + 1) + x 4 + 2 130. Numbers of real roots of P( x ) = 0 is


122. If f : R ® R and f ( x ) = , then
x2 + x +1 (a) 1 (b) 3 (c) 5 (d) 2
f ( x ) is 131. The maximum value of y = P ¢ ¢ ( x ) can be obtained at x is
(a) one-one into (b) many-one onto
equal to
(c) one-one onto (d) many-one into 1 1
(a) - (b) 0 (c) (d) 1
123. If f : R ® R and f ( x ) = 2ax + sin 2 x , then the set of 3 3
values of a for which f ( x ) is one-one and onto is
132. The sum of pairwise product of all roots (real and
æ 1 1ö
(a) a Î ç - , ÷ (b) a Î ( -1, 1 ) complex) of P( x ) = 0 is
è 2 2ø
5 10
æ 1 1ö (a) - (b) - (c) 2 (d) – 5
(c) a Î R - ç - , ÷ (d) a Î R - ( -1, 1 ) 3 3
è 2 2ø
Passage VIII (Q. Nos. 133 to 135)
Passage V (Q. Nos. 124 to 126)
é1 ù é1 ù
Let g ( x ) = a 0 + a1x + a 2 x 2 + a 3 x 3 and f ( x ) = g ( x ), f ( x ) has its Consider a > 1 and f : ê ,a ú ® ê ,a ú be bijective function.
ë a û ëa û
non-zero local minimum and maximum values at - 3 and 3,
1 é1 ù
respectively. If a 3 Î the domain of the function Suppose that f -1( x ) = ; for all x Î ê ,a ú.
æ1 + x 2 ö f (x ) ëa û
h ( x ) = sin -1 ç ÷.
è 2x ø 133. f (1) is equal to
(a) 1 (b) 0
124. The value of a1 + a 2 is
(c) -1 (d) does’nt attain a unique value
(a) 30 (b) – 30 (c) 27 (d) – 27
134. Which of the following statements can be concluded
125. The value of a 0 is
about ( f ( x )) ?
(a) equal to 50 (b) greater than 54
é1 ù
(c) less than 54 (d) less than 50 (a) f ( x ) is discontinuous in ê , a ú
ëa û
126. f (10) is defined for é 1 ù
(a) a 0 > 830 (b) a 0 < 830 (b) f ( x ) is increasing in ê , a ú
ëa û
(c) a 0 = 830 (d) None of these
é1 ù
(c) f ( x ) is decreasing in ê , a ú
Passage VI (Q. Nos. 127 to 129) ëa û
4 2 (d) None of the above
Let f : [2, ¥ ) ® [1, ¥ ) defined by f ( x ) = 2x - 4 x and
ép ù sin x + 4 135. Which of the following statements can be concluded
g : ê , p ú ® A defined by g ( x ) = be two invertible about f ( f ( x )) ?
ë2 û sin x - 2
é1 ù
functions. (a) f ( f ( x )) is continuous in ê , a ú
ëa û
-1
127. f ( x ) is equal to é1 ù
(b) f ( f ( x )) is increasing in ê , a ú
(a) 2+ 4 - log 2 x (b) 2+ 4 + log 2 x ëa û
é1 ù
(c) 2 - 4 + log 2 x (d) None of these (c) f ( f ( x )) is decreasing in ê , a ú
ëa û
128. The set A is equal to (d) None of the above
(a) [ - 5, - 2 ] (b) [2, 5 ] (c) [ - 5, 2 ] (d) [ - 3, - 2 ]
-1 -1 Passage IX (Q. Nos. 136 to 137)
129. The domain of f g ( x ) is
é sin 1 ù Let f be a real valued function from N to N satisfying. The relation
(a) [ - 5, sin 1 ] (b) ê - 5,
ë 2 - sin 1 úû f (m + n ) = f (m ) + f (n ) for all m,n Î N .
é ( 4 + sin 1 ) ù é ( 4 + sin 1 ) ù 136. The range of f contains all the even numbers, the value
(c) ê - 5, - ú (d) ê - , - 2ú
ë 2 - sin 1 û ë 2 - sin 1 û of f (1) is
(a) 1 (b) 2 (c) 1 or 2 (d) 4
Passage VII (Q. Nos. 130 to 132) 137. If domain of f is first 3m natural numbers and if the
Let P ( x ) be polynomial of degree atmost 5 which leaves number of elements common in domain and range is m,
remainders – 1 and 1 upon division by ( x - 1)3 and ( x + 1)3 , then the value of f (1) is
respectively. (a) 2 (b) 3
(c) 6 (d) Can’t say
Chap 03 Functions 171

Functions Exercise 5 : Matching Type Questions


138. Match the statements of Column I with values of
Column II. Column I Column II
æ 2 tan x ö
Column I Column II (B) If f ç ÷
è 1 + tan 2 x ø
(A) sin (cos x ) has domain (p) x ÎR (q) x 2 + 2x
(cos 2 x + 1 )(sec 2 x + 2 tan x )
= ,
(B) ( cos (sin x ) ) -1 has domain (q) ì pü 2
R - ín p ± ý then f ( x ) is
î 6þ
(C) tan ( p sin x ) has domain (r) æ pö (C) If f ( x + y + 1 ) = ( f ( x ) + f (y )) 2
x Î ç np , np + ÷ (r) 1 + x
è 2ø
for all x, y Î R and f ( 0 ) = 1, then f ( x )
(D) ln (tan x ) has domain (s) é p pù is
x Î ê2np - , 2np + ú
ë 2 2û éxù
(D) If 4 < x < 5 and f ( x ) = ê ú + 2 x + 2,
139. Match the statements of Column I with values of ë 4û
(s) ( x + 1 ) 2
Column II. where [y ] is the greatest integer £ y,
then f -1( x ) is
Column I Column II
(A) | 4 sin x - 1 | < 5, x Î[ 0, p ], (p) é p ù é 3p ù 141. Match the statements of Column I with values of
the domain is êë 0, 4 úû È êë 4 , p úû Column II.
(B) 4 sin 2 x - 8 (q) é 3p ù Column I Column II
sin x + 3 £ 0 , [ 0, 2p ] , the êë 2 , 2 p úû È { 0 } æ pö (p) Defined for all
(A) f ( x ) = sin 2 x + sin 2 ç x + ÷
domain is è 3ø real ‘x’
| tan x| £ 1 and x Î[ 0, p ] , é 3p ö æ pö
(C) (r) + cos x cos ç x + ÷ then f ( x ) is
the domain is êë 0, 10 ÷ø è 3ø

(D) cos x - sin x ³ 1 and [ 0, 2p ] , (s) é p 5p ù (B) g( x ) = tan (e { x } ) + [ x + a ] - 5 - x, (q) Even function
the domain is êë 6 , 6 úû where [×] denotes the greatest
integer less than or equal to x, then
140. Match the statements of Column I with values of g( x ) is
Column II. x x (r) Odd function
(C) h( x ) = x + + 5, then h( x ) is
5 -1 2
Column I Column II
(D) k( x ) = 2 sin 2 x - cos 2 a (s) Periodic
ì x + 1, when x < 0
(A) If f ( x ) = í 2 , the x -3 + 4 sin a × sin x cos ( x + a ) function
î x - 1, when x ³ 0 (p)
2 + cos 2 ( x + a ), a Î R,
fof ( x ) for -1 £ x < 0 is then k( x ) is

Functions Exercise 6 : Single Integer Answer Type Questions


142. A function f : R ® R is defined by 144. Let f : N ® R be such that f (1) = 1 and
f ( x + y ) - k xy = f ( x ) + 2y , " x , y Î R and f (1) = 2 ;
2
f (1) + 2 f (2) + 3 f (3) + ¼ + nf (n ) = n(n + 1) f (n ), for n ³ 2,
f (2) = 8, where k is some real constant, then 1
then is ……… .
æ 1 ö 2010 f (2010)
f (x + y)× f ç ÷ is …………… .
èx + yø 2010 x + 163 2010
145. If f ( x ) = , x > 0 and x ¹ , the least value
165x - 2010 165
143. If f : R ® R satisfying
æ æ 4 öö
f ( x - f (y )) = f ( f (y )) + x f (y ) + f ( x ) - 1, for all of f ( f ( x )) + f ç f ç ÷ ÷ is ………… .
è è x øø
- f (10)
x , y Î R, then is ………… .
7
172 Textbook of Differential Calculus

146. If a , b, g Î R; a + b + g = 4 and a 2 + b 2 + g 2 = 6, the 159. If a + b = 3 - cos 4q and a - b = 4 sin 2 q, then ab is


number of integers lie in the exhaustive range of a is always less than or equal to …… .
……… . 160. Let ‘n’ be the number of elements in the domain set of
the functionf ( x ) =½ ln x + 4 x C 2 x 2 + 3½and ‘Y ’ be the
147. The number of linear functions satisfying 2

f ( x + f ( x )) = x + f ( x ), " x is ………… . ½ ½
148. If A = {1, 2, 3}, B = {1, 3, 5, 7, 9}, the ratio of number of global maximum value of f ( x ), then [n + [Y ]] is ………
(where [×] = greatest integer function).
one-one functions to the number of strictly monotonic
functions is ………… . 161. If f ( x ) is a function such that
149. If n( A ) = 4, n( B ) = 5 and number of functions from A to B f ( x - 1) + f ( x + 1) = 3 f ( x ) and f (5) = 10, then the sum
19
such that range contains exactly 3 elements is k,
k
is of digits of the value of å f (5 + 12r ) is .................... .
60 r=0
……… .
æx ö
150. If a and b are constants, such that 162. If 2f ( x ) = f ( xy ) + f ç ÷ for all positive values of x and
èy ø
f ( x ) = a sin x + bx cos x + 2x 2 and f (2) = 15, f ( -2) is
y, f (1) = 0 and f ¢ (1) = 1, then f (e ) is ............... .
………… .
-1
163. Let f be a function from the set of positive integers to
151. If the functions f ( x ) = x 5 + e x / 3 and g ( x ) = f ( x ), the
the set of real number such that f (1) = 1 and
value of g ¢ (1) is ……… . n

152. If f ( x ) = x - 12x + p ; p Î {1, 2, 3, …, 15} and for each ‘


3 å r f (r ) = n(n + 1) f (n ), " n ³ 2, the value of 2126
r =1
p’, the number of real roots of equation f ( x ) = 0 is f (1063) is ............... .
1
denoted by q, the Sq is equal to ……… . x4 +x2 +1
5 164. If f ( x ) = , the value of f ( wn )
x2 - x +1
153. Let f ( x ) denotes the number of zeroes in f ¢ ( x ). If
f (m ) - f (n ) = 3, the value of (where ‘w’ is the non-real root of the equation z 3 = 1 and
(m - n ) max - (m - n ) min ‘n’ is a multiple of 3), is ............... .
is ………… .
2 æ1 - x ö
165. If f 2 ( x ) × f ç ÷ = x , [x ¹ - 1, 1 and f ( x ) ¹ 0], the
3
æx 3 + y3 ö è1 + x ø
154. If x 2 + y 2 = 4, the maximum value of ç ÷ is
è x +y ø value of |[ f ( -2)]| (where [×] is the greatest integer
function), is .................... .
155. Let f (n ) denotes the square of the sum of the digits of
166. An odd function is symmetric about the vertical line
natural number n, where f 2 (n ) denotes f ( f (n )), f 3 (n ) ¥

denotes f ( f ( f (n ))) and so on. The value of x = a (a > 0) and if å [ f (1 + 4r )] r = 8, find the
r=0
f 2011
(2011) - f 2010
(2011) numerical value of 8 f (1) .
is ……… .
f 2013
(2011) - f 2012
(2011) e x - e -x 1+ x
167. Let = ln , then find x.
156. If [sin x ] + éê ùú + éê ùú =
x 2x 9x -x 1- x
, where [×] denotes the e x
+e
ë 2p û ë 5p û 10p
3x 2 + 9 x + 17
greatest integer function, the number of solutions in the 168. If the maximum value of f ( x ) = is 5k + 1,
interval (30, 40) is ……… . 3x 2 + 9 x + 7
1 1 1 the value of k is ............... .
157. The number of integral solutions of + = with x £ y
x y 6 169. The period of the function f ( x ) which satisfies the
is ‘a’. The value of ‘a - 6’ is ……… . relation f ( x ) + f ( x + 4 ) = f ( x + 2) + f ( x + 6) is ....……. .
158. If f ( x ) is a polynomial of degree 4 with leading 170. If a non-zero function f ( x ) is symmetrical about y = x ,
coefficient ‘1’ satisfying f (1) = 10, f (2) = 20 and f (3) = 30, then the value of p (constant) such that
æ f (12) + f ( - 8) ö f 2 ( x ) = ( f -1 ( x )) 2 - px × f ( x ) × f -1 ( x ) + 2x 2 f ( x ) for all
then ç ÷ is …… .
è 19840 ø x Î R + is .......…… .
Chap 03 Functions 173

3x 2 + mx + n 172. Let f ( x ) be a monotonic polynomial of (2m - 1) degree


171. Let f : R ® R and f ( x ) = . If the range of
x 2 +1 where m Î N , then the equation
m +n
2 2 f ( x ) + f (3x ) + f (5x ) +...+ f (2m - 1) x = (2m - 1) has
this function is [-4, 3], then the value of is.... ………… . roots.
4

Functions Exercise 7 : Subjective Type Questions


173. Let x be a real number. [x] denotes the greatest integer 1 n æ1 n ö
function, {x} denotes the fractional part and ( x ) denotes (i) åf –1
(xi ) = f ç å xi ÷
çn ÷
n i =1 è i =1 ø
the least integer function, then solve the following :
n n
(i) ( x ) 2 = [x ]2 + 2x (ii) å f –1 ( x i ) = å x i , where f –1 denotes the inverse
(ii) [2x ]– 2x = [x + 1] i =1 i =1

(iii)[x 2 ] + 2 [x ] = 3x , 0 £ x £ 2 of f . Find the AM of xi ’ s.


180. Let f ( x ) = x 2 – 2x , x Î R and
(iv)y = 4 –[x ]2 and [ y ] + y = 6
g ( x ) = f ( f ( x ) – 1) + f (5 – f ( x )), show that g ( x ) ³ 0,
(v) [x ] + | x – 2 | £ 0 and –1 £ x £ 3 " x Î R.
174. Let n be a positive integer and define 181. If f is a polynomial function satisfying
f (n ) = 1 ! + 2 ! + 3 ! + ...+ n !. Find polynomials P( x ) and
2 + f ( x ) × f (y ) = f ( x ) + f (y ) + f ( xy ), " x , y Î R and if
Q( x ) such that f (n + 2) = Q (n ) f (n ) + P(n ) f (n + 1) for
f (2) = 5 , find f ( f (2)).
all n ³ 1.
2n – 1 182. If a + b + c = abc , a, b and c Î R + , prove that
ax ær ö
175. If f ( x ) = x (a > 0), evaluate å 2 f ç ÷ . a + b + c ³ 3 3.
a + a r =1
è 2n ø
ì 1
x – [x ]– , if x Ï I
176. Find the domain of the function 183. Consider the function f ( x ) = ïí 2 ,
ì 3 ü ïî0, if x Î I
f ( x ) = log í log| sin x | ( x 2 – 8x + 23) – ý.
î log 2 |sin x | þ where [×] denotes the greatest integral function and I is
177. Let S (n ) denotes the number of ordered pairs ( x , y ) the set of integers. Find g ( x ) = max {x 2 , f ( x ), | x | };
1 1 1 –2 £ x £ 2 .
satisfying + = , where n > 1 and x , y, n Î N .
x y n 184. If f ( x ) is continuous function in [0, 2p ] and f (0) = f (2p ),
(i) Find the value of S(6). then prove that there exists a point c Î(0, p ) such that
(ii) Show that, if n is prime, then S (n ) = 3, always. f (c ) = f (c + p ).
1 1 1 185. Let g (t ) = | t – 1 | – | t | + | t + 1 |, " t Î R.
178. Solve + = { x } + , where [×] denotes the greatest
[x ] [2x ] 3 3 æ –3 ö
Find f ( x ) = max {g (t ) : – £ t £ x }, " x Î ç , ¥ ÷.
integer function and { } denotes fractional part of x. 2 è2 ø
179. Let f ( x ) = x 2 + 3x – 3, x ³ 0. n points x 1 , x 2 , ....., x n are 186. Find the integral solution for n 1n 2 = 2n 1 – n 2 , where
so chosen on the X -axis that n 1 , n 2 Îinteger.
174 Textbook of Differential Calculus

Functions Exercise 8 : Questions Asked in Previous 10 Year’s Exams


(i) JEE Advanced & IIT-JEE
187. Let f 1 : R ® R, f 2 :[0, ¥ ) ® R, f 3 : R ® R and (c) Statement I is true; Statement II is false.
f 4 : R ® [0, ¥ ) be defined by (d) Statement I is false; Statement II is true.
[Match Type Question, 2014 Adv.] Let F ( x ) be an indefinite integral of sin 2 x .
ì| x |, if x < 0 ìsin x , if x < 0 Statement I The function F ( x ) satisfies
f 1( x ) = í x ; f 2 (x ) = x 2 ; f 3 (x ) = í
î e , if x ³ 0 î x , if x ³ 0 F ( x + p ) = F ( x ) for all real x. Because
Statement II sin 2 ( x + p ) = sin 2 x , for all real x.
ì f 2 [ f 1 ( x )], if x < 0
and f 4 ( x ) = í [Assertion and Reason Type Question, 2007]
î f 2 [ f 1 ( x )] - 1, if x ³ 0 190. Match the conditons/expressions in Column I with
Column I Column II statement in Column II.
A. f 4 is p. onto but not one-one x 2 - 6x + 5
Let f ( x ) = 2 .
B. f 3 is q. neither continuous nor one-one x - 5x + 6 [Match Type Question, 2007]
C. f 2 of1, is r. differentiable but not one-one
D. f 2 is s. continuous and one-one Column I Column II

Codes A. If -1 < x < 1, then f ( x ) satisfies p. 0 < f ( x ) < 1


A B C D A B C D B. If 1 < x < 2 , then f ( x ) satisfies q. f ( x ) < 0
(a) r p s q (b) p r s q
(c) r p q s (d) p r q s C. If 3 < x < 5, then f ( x ) satisfies r. f ( x ) > 0
-1
188. If function f ( x ) = x 2 + e x / 2 and g ( x ) = f ( x ), then D. If x > 5, then f ( x ) satisfies s. f ( x ) < 1
the value of g ¢ (1) is [Integer Type Question, 2009]
191. Find the range of values of t for which
189. For the following questions, choose the correct answer
from the codes (a), (b), (c) and (d) defined as follows. 1 - 2x + 5x 2 é p pù
2 sin t = , t Î ê- , .
(a) Statement I is true, Statement II is also true; Statement II 3x - 2x - 1
2
ë 2 2 úû
is the correct explanation of Statement I. [Subjective Type Question, 2005]
(b) Statement I is true, Statement II is also true; Statement II
is not the correct explanation of Statement I.

(ii) JEE Main & AIEEE


1 (a) ± np , n Î { 0, 1, 2, K }
192. If f k ( x ) = (sin k x + cos k x ), where x Î R and k ³ 1,
k (b) ± np , n Î {1, 2, K }
then f 4 ( x ) - f 6 ( x ) is equal to [2014 JEE Main] (c) p /2 + 2np , n Î {...,- 2, - 1, 0, 1, 2, K }
(a) 1/6 (b) 1/3 (c) 1/4 (d) 1/12 (d) 2np, n Î {..., - 2, - 1, 0, 1, 2, K }
193. The function f : [0, 3] ® [1, 29 ], defined by b-x
195. Let f : (0, 1) ® R be defined by f ( x ) = , where b is
f ( x ) = 2x - 15 x + 36x + 1, is
3 2
[2012 AIEEE] 1 - bx
(a) one-one and onto a constant such that 0 < b < 1. Then, [2011 AIEEE]
(b) onto but not one-one (a) f is not invertible on (0, 1)
(c) one-one but not onto 1
(b) f ¹ f -1 on (0, 1) and f ¢(b ) =
(d) neither one-one nor onto f ¢( 0 )
194. Let f ( x ) = x 2 and g ( x ) = sin x for all x Î R. Then, the set 1
(c) f = f -1 on (0, 1) and f ¢(b ) =
f ¢( 0 )
of all x satisfying ( fogogof )( x ) = ( gogof )( x ) , where
( fog )( x ) = f ( g ( x )), is [2011 AIEEE] (d) f -1 is differentiable on (0, 1)
Chap 03 Functions 175

196. Let f be a real-valued function defined on the interval 205. Suppose f ( x ) = ( x + 1) 2 for x ³ - 1. If g ( x ) is the
x
( - 1, 1) such that e - x f ( x ) = 2 + ò t 4 + 1 × dt , " x function whose graph is reflection of the graph of f ( x )
0 with respect to the line y = x , then g ( x ) equals
-1
Î ( - 1, 1)and let f be the inverse function of f . Then, [2002 AIEEE]
-1
[f (2)]¢ is equal to [2010 AIEEE] (a) - x - 1, x ³ 0 (b)
1
, x > -1
(a) 1 (b) 1/3 (c) 1/2 (d) 1/e (x + 1)2
(c) x + 1 , x ³ - 1 (d) x - 1, x ³ 0
197. If X and Y are two non-empty sets, where f : X ® Y , is
1
function defined such that 206. If f : [1, ¥ ) ® [2, ¥ ) is given by f ( x ) = x + , then
f (C ) = { f ( x ) : x Î C } for C Í X and x
-1
-1 f ( x ) equals [2001 AIEEE]
f ( D ) = {x : f ( x ) Î D } for D Í Y ,
x+ x2 - 4 x
for any A Í Y and B Í Y , then [2005 AIEEE] (a) (b)
2 1 + x2
(a) f -1 { f ( A )} = A
x - x2 - 4
(b) f -1 { f ( A )} = A, only if f ( X ) = Y (c) (d) 1 + x2 - 4
2
(c) f { f -1( B )} = B, only if B Í f ( x )
207. Let f ( x ) = (1 + b 2 ) x 2 + 2bx + 1 and let m (b ) be the
(d) f { f -1( B )} = B
minimum value of f ( x ). As b varies, the range of m (b ) is
ì x , if x is rational ì 0, if x is rational
198. f ( x ) = í , g( x ) = í [2001 AIEEE]
î 0, if x is irrational î x , if x is irrational. é 1ù
(a) [0, 1] (b) ê 0, ú
Then, f - g is [2005 AIEEE] ë 2û
é1 ù
(a) one-one and into (b) neither one-one nor onto (c) ê , 1 ú (d) ( 0, 1 ]
(c) many one and onto (d) one-one and onto ë2 û
log 2 ( x + 3)
199. If f ( x ) = sin x + cos x , g ( x ) = x 2 - 1, then g { f ( x ) } is 208. The domain of definition of f ( x ) = is
( x 2 + 3x + 2)
invertible in the domain [2004 AIEEE]
[2001 AIEEE]
é pù é p pù é p pù
(a) ê 0, ú (b) ê - , ú (c) ê - , ú (d) [ 0 , p ] (a) R / { - 1, - 2 } (b) ( - 2, ¥ )
ë 2û ë 4 4û ë 2 2û
(c) R / { - 1, - 2, - 3 } (d) ( - 3, ¥ ) / { - 1, - 2 }
200. Domain of definition of the function
ax
p 209. Let f ( x ) = , x ¹ - 1. Then, for what value of a is
f ( x ) = sin -1 (2x ) + for real valued x, is x +1
6 [2003 AIEEE]
é 1 1ù é 1 1ù æ 1 1ö é 1 1ù
f [ f ( x )] = x ? [2001 AIEEE]
(a) ê - , ú (b) ê - , ú (c) ç - , ÷ (d) ê - , ú (a) 2 (b) - 2
ë 4 2û ë 2 2û è 2 9ø ë 4 4û
(c) 1 (d) -1
x2 + x +2
201. Range of the function f ( x ) = ; x Î R is ì -1, x < 0
x2 + x +1 [2003 AIEEE] 210. Let g ( x ) = 1 + x - [x ] and f ( x ) = ïí 0, x = 0 , then for
(a) (1, ¥) (b) (1, 11/7) ï 1, x > 0
(c) (1, 7/3] (d) (1, 7/5) î
x all x , f [g ( x )] is equal to [2001 AIEEE]
202. If f : [0, ¥ ) ® [0, ¥ ) and f ( x ) = , then f is (a) x (b) 1
1+ x [2003 AIEEE] (c) f ( x ) (d) g ( x )
(a) one-one and onto (b) one-one but not onto
(c) onto but not one-one (d) neither one-one nor onto 211. The domain of definition of the function y ( x ) is given by
203. Let function f : R ® R be defined by f ( x ) = 2 x + sin x the equation 2 x + 2 y = 2 , is [2000 AIEEE]
for x Î R . Then, f is [2002 AIEEE] (a) 0 < x £ 1
(a) one-to-one and onto (b) 0 £ x £ 1
(b) one-to-one but not onto (c) - ¥ < x £ 0
(c) onto but not one-to-one (d) -¥ < x < 1
(d) neither one-to-one nor onto
212. Let f (q ) = sin q (sin q + sin 3 q ). Then, f (q )
204. Let E = {1, 2, 3, 4 } and F = {1, 2}. Then, the number of onto [2000 AIEEE]
functions from E to F is [2001 AIEEE] (a) ³ 0, only when q ³ 0 (b) £ 0, for all real q
(a) 14 (b) 16 (c) 12 (d) 8 (c) ³ 0, for all real q (d) £ 0, only when q £ 0
Answers

Exercise for Session 1 9. (- ¥, 0] 10. ì
í ý
1. (i) not a function (ii) not a function (iii) function î2þ
(iv) function (v) not a function (vi) function 11. {0, 1} 12. {p}
p pù p
13. é , 14. æç 0, ù
(vii) function (viii) not a function (ix) function
15. {0}
(x) not a function. êë 3 2 úû è 2 úû
2. (i) not a function (ii) not a function (iii) not a function
16.
é -1 , 1 ù 17. [ - 1, 1]
(iv) is a function (v) not a function êë 2 2 2 2 úû
3. (a) 18. {0} 19. [1, ¥)
4. (a) f : A ® B (b) f : A ® B 20. {1} 21. [0, 2]
22. Image of (-¥, 1) under f is (1, ¥), Image of [1, 2]
α 1
α 1 under f is [1, 2]
β 2 23. Range of f (x) is {0}
β 2
é p p ù
and domain is È ê æç 2np - ö÷ , æç 2np + ö÷ ú
(c) f : A ® B (d) f : A ® B ëè 3ø è 3 øû
p
α 1 α 1 24. Range of f (x) is ì ü
ílog ý and domain is [1, 2)
î 2þ
β 2 β 2
25. Range of f (x) is the set of all non-positive integers and
5. (b) é- 3 - 5 ö æ - 3 + 5ù
domain is ê , - 2÷ È ç - 1, ú.
Exercise for Session 2 ë 2 ø è 2 û
1. (- ¥ , 2] È [ 3, ¥) 2. (- ¥, - 1 / 2] È (0, 1) È (2, ¥) Exercise for Session 6
3. [ -1,1] 4. (-¥, ¥) 1. (i) odd (ii) even (iii) neither even nor odd (iv) even
5. {2, 3} 6. {1, 2, 3} (v) odd (vi) odd (vii) neither even nor odd (viii) even
2. f is an even function when x Î integer f is an odd
Exercise for Session 3 function when x Ïinteger.
1. [ 4, 6] 2. (0,1] È [ 4,5) ì- 2x, x £ - 1 ì2x, x £ - 1
3. (i) f (x) = í (ii) f (x) = í
3. [1 - 2 , 0) È [1 + 2 , 3) 4. x Î[ 2, 4] î- x | x|, - 1 < x £ 0 îx | x|, - 1 < x £ 0
5. (2, 3) 6. [ -p / 4, p / 4] 5. a Î (400, ¥)
7. [ - 2, - 1 / 2] È [1 / 2, 2] 8. [ - 1, 3]
Exercise for Session 7
9. æç 0, ö÷ È æç ,1ö÷
1 1
10. f 1. (i) p / 3 (ii) 2p (iii) 1 (iv) 24 (v) Does not exist
è 2ø è2 ø
(vi) 2(n + 1)! (vii) 1 (viii) 2p
2. Period is 8.
Exercise for Session 4 3. f (x) is periodic with period 2l.
1. R - (-2, 2) 2. [ -2, 2] 4. f (x) is periodic with period 2 p.
99
3. (0, 1) È (1, ¥) 4. [ 0, 3)
5. f (x) is periodic with period 12 and S f (5 + 12r ) = 10000
5. R - I 6. (- ¥, - 2) È (4, ¥) r=0

7. R 8. (2, ¥) Exercise for Session 8


9. (- ¥, 0) È (0, 1] È [ 2, ¥) 1. x = 0 and y =
3
2. f -1 (1) = y
æ 5 ù é 5ö 2
10. (0, 1) 11. ç - ,-1ú È ê1, ÷ 3. As monotonic and range = Codomain Þ Bijective
è 2 û ë 2ø
5. A Î[ 0, 1)
13. é -1, - ù È é 0, ù È {1}
1 1
12. (- 8 , - 1] È [1, 8 )
6. A Î é 0, ö÷
êë 1
2 úû êë 2 úû êë 2 ø
14. R - (1, 2) È {2, 3, 4, 5, 6, 7, 8, 9, 10} È (10, 11)
7. b 2 < 3a (c - | d |)
ép pö
÷ p p
8. X Î é - - a, - aùú and Y Î[ c - r , c + r ], where
15. ê , 16. No solution
ë6 2ø
ëê2 2 û
18. Integral solutions are (2, 2) and (0, 0), all non-integral
-1 æ a + b 2 ö
solutions lie on exactly two lines x + y = 0 and x + y = 6. a = tan ç ÷ and r = a + 2ab + b
2 2
è a ø
Exercise for Session 5
1. [ 0, 3] 2. [ - 1 / 2, 1 / 2] Exercise for Session 9
3. [ 3 - 2 , 3 + 2] 4. [1, 5] 1. Not Identical 2. Identical 3. Not Identical
æ 4. Identical 5. Identical 6. Not Identical
6. ç - ¥, ù È (1, ¥)
2
5. (- ¥, log 3 9)
è 3 úû 7. Identical 8. Identical
9. Identical 10. Identical
7. R - (2 - 2 3 , 2 + 2 3 ) 8. [ 2, ¥)
Chap 03 Functions 177

1 x
(iii) f -1 (x) = x (iv) f -1 (x) = (e - e- x )
log 5 e
Exercise for Session 10 , x>0
-2 p p ù
1. Domain Î[ - 1, 1] and range Î é
2
êë 3 , 3 úû ì
ï x, x<1
2. Domain for f ([| x|]) Î (-3, 3) ï
(v) f -1 (x) = í x , 1 £ x £ 16
Domain for f ([ 2x + 3]) Î[ - 3, 0) ï x2 x > 16
ìïsin 2 x - sin x + 1, - 1 < x < 0 ï ,
3. h(x) = í î 64
ïî2 sin 2 x, 0< x£1 1 + 1 + 4 log2 x
2. f -1 (x) = ,x>0
ì - x, - 2 £ x £ 0 2
ï
4. g (x) = í 0, 0£ x£1 5. gof = {(x + 1)2 , - 2 £ x £ 1}
ï2(x - 1), 1 £ x £ 2 Exercise for Session 12 ì (1 - x)2 , 0£x£
1
î ï 3
2 4 ïï 1 2
Exercise for Session 11 1. 2. 17 3. 4. f (x) = í2x(1 - x), £x£
3 3 ï 3 3
-3+ 5 + 4e
x
ï x2 , 2
1. (i) f -1 (x) = 3 sin x (ii) f -1 (x) = £ x£1
2 ïî 3

Chapter Exercises
1. (b) 2. (d) 3. (a) 4. (a) 5. (a) 6. (d) 7. (a) 8. (a) 9. (d) 10. (c)
11. (c) 12. (c) 13. (d) 14. (c) 15. (c) 16. (d) 17. (a) 18. (b) 19. (b) 20. (d)
21. (d) 22. (a) 23. (d) 24. (b) 25. (b) 26. (a) 27. (a) 28. (b) 29. (d) 30. (c)
31. (c) 32. (b) 33. (c) 34. (a) 35. (d) 36. (d) 37. (c) 38. (d) 39. (c) 40. (d)
41. (b) 42. (a) 43. (d) 44. (a) 45. (c) 46. (a) 47. (a) 48. (d) 49. (b) 50. (b)
51. (b) 52. (d) 53. (c) 54. (c) 55. (d) 56. (b) 57. (d) 58. (c) 59. (c) 60. (b)
61. (b) 62. (d) 63. (d) 64. (c) 65. (b) 66. (a) 67. (a) 68. (b) 69. (c) 70. (a)
71. (c) 72. (d) 73. (a) 74. (b) 75. (b) 76. (b) 77. (a,b,d) 78. (b,c,d) 79. (b,c,d) 80. (b,c,d)
81. (a, b) 82. (b, d) 83. (a,b) 84. (a,b, c, d) 85. (b, c) 86. (a, b, c, d)
87. (a, b, c, d) 88. (a,b,c) 89. (b, d) 90. (b, c) 91. (a, b, c, d) 92. (c, d) 93. (a,b, c, d)
94. (a, c) 95. (a, b, c) 96. (a, b, c, d) 97. (a, b, c) 98. (a, c)
99. (a, d) 100. (b, c, d) 101. (a,b,d) 102. (c) 103. (b) 104. (c) 105. (a)
106. (d) 107. (a) 108. (a) 109. (d) 110. (d) 111. (a) 112. (c) 113. (d) 114. (c) 115. (a)
116. (c) 117. (b) 118. (b) 119. (b) 120. (a) 121. (c) 122. (d) 123. (d) 124. (c) 125. (b)
126. (d) 127. (b) 128. (a) 129. (c) 130. (a) 131. (c) 132. (b) 133. (a) 134. (b) 135. (b)
136. (c) 137. (a)
138. (A) ® (s), (B) ® (p), (C) ® (q), (D) ® (r) 139. (A) ® (r), (B) ® (s), (C) ® (p), (D) ® (q)
140. (A) ® (q), (B) ® (r), (C) ® (s), (D) ® (p) 141. (A) ® (p,q,s), (B) ® (s), (C) ® (q), (D) ® (p,q,r,s)

142. (4) 143. (7) 144. (2) 145. (4) 146. (2) 147. (2) 148. (3) 149. (6) 150. (1) 151. (3)
152. (9) 153. (9) 154. (6) 155. (1) 156. (1) 157. (4) 158. (1) 159. (1) 160. (5) 161. (2)
162. (1) 163. (2) 164. (3) 165. (2) 166. (7) 167. (0) 168. (8) 169. (8) 170. (2) 171. (4)
172. (1)
1 ì x 2, - 2 £ x £ -1
173. (i) 0, n + , where n Î Z ï
2 ï - x, -1 £ x £ -1 / 4 ì 3 / 2, - 3/ 2 < x £ - 1/ 2
ï 2 + x,
(ii) ìí- 1, - üý (iii) ìí0, 1, , üý ïï
1 4 5 1 1 ï -1 / 2 £ x < 0
183. g ( x ) = í x + , - £ x £ 0 185. f ( x ) = í
î 2þ î 3 3þ ï 2 4 ï 2, 0£ x£2
(iv) {1, - 1, ± 1 + k , where k is any positive proper fraction} ï x, 0 £ x £1 îï x, 2£x
ï 2 ï
(v) no solution 1£x£2
ïî x ,
174. P (x) = x + 3 and Q (x) = - x - 2 186. (- 3, 3), (- 2, 4), (0, 0) and (1, 1) 187. (d)
175. (2n - 1) 188. (2) 189. (d) 190. A ® p, B ® q, C ® q, D ® p
3p ö æ 3p ö p p 3p p
176. x Î (3, p ) È æç p , ÷ È ç , 5÷ 191. é - , - ùú È éê , ùú 192. (d) 193. (d)
è 2 ø è 2 ø ëê 2 10 û ë 10 2 û
177. (i) S (6) = 9 194. (a) 195. (a) 196. (b) 197. (c) 198. (d) 199. (b)
29 19 97 200. (a) 201. (c) 202. (b) 203. (a) 204. (a) 205. (d)
178. Possible solutions are , , .
12 6 24 206. (a) 207. (d) 208. (d) 209. (d)
n
179.
1
S xi = 1 181. f ( f (2)) = 26 210. (b) 211. (d) 212. (c)
n i=1
6. (a) f ( x ) = x 4 + 2x 3 - x 2 + 1

Solutions
A polynomial of degree even will always be into.
(b) f ( x ) = x 3 + x + 1
Þ f ¢( x ) = 3 x 2 + 1, i.e. injective as well as surjective.
(c) f ( x ) = 1 + x 2 , neither injective nor surjective as
range Î [1, ¥ ).
1. Here, (d) f ( x ) = x 3 + 2 x 2 - x + 1
Y Y
Þ f ¢( x ) = 3 x 2 + 4 x - 1
y=f1(x) y=f2(x)
1 1 Þ D>0
\ f ( x ) is surjective but not injective.
X′ X X′ X
0 1 –1 0 7. Here, f ( x ) is bijective, hence f -1( 4) exists when y = 4.
Y′ Y′ \ 2x 3 + 7x - 9 = 0
Y Y Þ (2 x 2 + 2 x + 9 ) ( x - 1 ) = 0
Þ x = 1 only, as 2 x 2 + 2 x + 9 = 0 has no other root.
1 2
X′ 0 X X′ 0
X e x × log x × 5 x + 2 × ( x 2 - 7 x + 10 )
–1 8. Here, f ( x ) =
–1 –1 2 x 2 - 11 x + 12
y=f3(x) y=f4(x)=f3(–x) 2
Y′ e x × log x × 5 x + 2 × ( x - 2 ) ( x - 5 )
Y′ =
(2 x - 3 ) ( x - 4 )
Þ f1( x ) = - f 3( - x ) 3
Note that at x = and x = 4, function is not defined and in
2. Only in option (d), the graph has a symmetry w.r.t. origin. 2
4 æ3 ö
3. Here, f (x ) = open interval ç , 4 ÷ , function is continuous.
è2 ø
1 - x2
2
4 e x × log x × 5 x + 2 × ( x - 2 ) ( x - 5 )
Þ f (sin x ) = \ lim
| cos x | x®
3+ (2 x - 3 ) ( x - 4 )
2
4
and f (cos x ) = ( + ve) ( + ve) ( - ve) ( - ve)
| sin x | = =-¥
( + ve) ( - ve)
\ g( x ) = | sin x | + | cos x | 2
e x × log x × 5 x + 2 × ( x - 2 ) ( x - 5 )
p and lim
\ Period of g( x ) = x ® 4- (2 x - 3 ) ( x - 4 )
2
( + ve) ( + ve) ( - ve)
f (1 ) = =¥
4. If x = 1, we see f (y ) = for all y ( + ve) ( - ve)
y
æ3 ö
Put y = 30 In the open interval ç , 4 ÷ , the function is continuous and
è2 ø
Þ f (1 ) = 30 × f (30 ) = 30(20 ) takes up all real values from ( - ¥, ¥ ).
= 600 …(i) Hence, range of the function is ( -¥, ¥ ).
Now, let y = 40 9. As, x = cos-1 (cos 4) = cos-1(cos(2p - 4)) = 2p - 4
f (1 ) 600
Þ f ( 40 ) = = = 15 and y = sin -1 (sin 3 )
40 40
= sin -1 (sin( p - 3 )) = p - 3
{e | x | sgn x } [e | x | sgn x ]
5. h( x ) = log( f ( x ) × g( x )) = log e ×e
\ x + y = 3p - 7
e | x | sgn x
= log e = e| x | sgn x æ 2 sin x + sin 2 x 1 - cos x ö
2/ 3
10. f ( x ) = ç × ÷
\ h( x ) = e |x|
sgn x è 2 cos x + sin 2 x 1 - sin x ø
ì e , x>0 x
æ 2 sin x × (1 + cos x ) (1 - cos x ) ö
2/ 3
ï =ç × ÷
= í 0, x=0 è 2 cos x × (1 + sin x ) (1 - sin x ) ø
ï -e - x , x < 0
î For domain, sin x ¹ ± 1
Þ h ( x ) + h ( - x ) = 0 for all x ì p pü
Þ x Î R - í( 4n - 1 ) , ( 4n + 1 ) ý
\ h( x ) is odd. î 2 2þ
Chap 03 Functions 179

æ (1 - cos2 x ) ö
2/ 3
Case II x 2 - 5 x - 24 ³ 0, x + 2 ³ 0
\ f ( x ) = ç tan x × ÷ = tan 2 x
è (1 - sin 2 x ) ø and x 2 - 5 x - 24 > ( x + 2 ) 2
Þ Range Î [ 0, ¥ ) Þ x Î ( - ¥, - 3 ] È [8, ¥ ) , x Î [ - 2, ¥ )
11. As, f ( x + 2) = e sin { x + 2} × cos p ( x + 2)
=e sin { x } × cos px
= f (x ) and x 2 - 5 x - 24 > x 2 + 4 x + 4
\ Periodic with period 2. Þ 9 x < – 28
-28
12. Here, y = x - 2x + 3 = ( x - 1) + 2 Þy min = 2
4 2 2 2
Þ x<
9
Þ log 0.5 ( x 4 - 2 x 2 + 3 ) £ log1/2 2 = - 1
\ No solution, i.e. x Î f. …(ii)
\ f ( x ) = cot -1 (log 0.5( x 4 - 2 x 2 + 3 )) ³ cot -1( -1 ) From Eqs. (i) and (ii), we get
é 3p ö x Î ( - ¥, - 3 ]
Þ Range Î ê , p ÷
ë 4 ø
18. Here, f ( f ( x )) = x
13. Here, loge ( x 2 + e ) ³ 1, for all x Î R Þ m(mx + b ) + b = x
\ 0<
1
£1 Þ m 2x + b(m + 1 ) = x
log( x 2 + e ) Þ m = ± 1, b = 0
é 1 ù ì 0, x ¹ 0 If f ( - f ( x )) = - x Þ - m(mx + b ) + b = - x
Þ ê ú =í
ë log( x + e ) û î 1 , x = 0 Þ - m 2x + b( -m + 1 ) = - x
2

ì 1 Þ m = ± 1 and b = 0
ï , x¹0
Þ f (x ) = í 1 + x 2 \Only 2 straight lines, i.e. y = ± x.
ï x=0 rx
î 2, 19. Here, f ( g( x )) = and g( f ( x )) = rx
Which is shown in figure as 1 + (r - 1 ) x
rx
Y If f ( g( x )) = g( f ( x )), then = rx
1 + (r - 1 ) x
2
Þ rx = rx(1 + (r - 1 ) x )
1
Þ r (r - 1 ) x 2 = 0
X′ X If this is to be true for infinitely many x, then
0 r (r - 1 ) = 0 Þr = 0, 1
20. Since, f is a linear function, so it has the form f ( x ) = mx + b
because f (1 ) £ f (2 ), we have m ³ 0. Similarly, f (3 ) £ f ( 4 )
Y′ Þm £ 0.
\ Range of f ( x ) Î [ 0, 1 ) È {2 }. Hence, m = 0 and f is constant function. Thus, f ( 0 ) = f (5 ) = 5
14. Let x + 3 = t 21. Suppose R is just a rectangle whose four vertices are
(1, 2 ), (1, - 2 ), ( - 1, - 2 ), ( - 1, 2 ).
\ f (t ) = sin({t }) Þ Period = 1
-| x| The X -axis and Y -axis symmetries in the problem are satisfied,
15. As, e can be shown by graphical transformation. but the point (2, 1 ) is not contained in R.
Y 22. Here, 2{y } = [ x ] + 1
(0, 1) Since, 0 £ y < 1
y=e– x \ {y } = y
X
X′
O
and 0 £ [ x ] + 1 < 2, i.e. -1 £ [ x ] < 1
When -1 £ x < 0 Þ y = 0
Y′
1
When 0 £ x < 1 Þy =
16. Here, [ x ] { x } = 1 2
1 1 1 Y
Þ { x} = or x - [ x ] = Þ x = [x ] +
[x ] [x ] [x ]
Obviously, x > 2 (0,1/2) y=1/2
1
\ x = m + , m Î N - {1 } X′ X
m –1 0 1
17. Here, x 2 - 5x - 24 > x + 2 is equivalent to the
collection of two system of inequations.
Y′
Case I x 2 - 5 x - 24 ³ 0 and x + 2 < 0
1
Þ ( x - 8 ) ( x + 3 ) ³ 0 and x < - 2 \ The required area = .
Þ x £ -3 …(i) 2
180 Textbook of Differential Calculus

23. Here, -1 £ cos x £ 1 So, the roots of f ¢( x ) = 0 are x = - 1, 2.


Þ - sin 1 £ sin(cos x ) £ sin 1 Y
Þ [sin(cos x )] = 0, - 1
Also, -1 £ sin x £ 1
Þ cos1 £ cos(sin x ) £ 1 2
X′ X
\ [cos(sin x )] = 0, 1 –1 0
ì -p p ü
\ f ( g( x )) has range í ,0, ý.
î 2 2þ
Y′
24. As we know, { f ( x )} Î [ 0, 1 )
Now, f ( x ) = 0 will have all real roots, if f ( -1 ) > 0 and
Þ [{ f ( x )}] = 0 f (2 ) < 0.
\ e 2x + e x - 2 = [{ x 2 + 10 x + 11 }] Þ -2 - 3 + 12 + a > 0 and 16 - 12 - 24 + a < 0
Þ e 2x + e x - 2 = 0 Þ -7 < a < 20
e f (x ) - e | f (x ) |
Þ (e x + 2 )(e x - 1 ) = 0 29. g( x ) = , - 1 £ f (x ) £ 1
e f (x ) + e | f (x ) |
\ e x = 1 is the only solution. For 0 £ f ( x ) £ 1,
Þ x=0 g( x ) = 0
Þ Number of solutions is 1. -1 £ f ( x ) < 0
25. We know that, { x } = x - [ x ]. e f (x ) - e - f (x ) e 2 f (x ) - 1 2
g( x ) = f ( x ) - f (x )
= 2 f (x ) = 1 - 2 f (x )
\ { x } + {2 x } + {3 x } + ...+ {12 x } = 78 x. e +e e +1 e +1
Þ x - [ x ] + 2 x - [2 x ] + 3 x - [3 x ] + ... + 12 x - [12 x ] = 78 x For -1 £ f ( x ) < 0,
Þ ( x + 2 x + 3 x + ...+12 x ) - ([ x ] + [2 x ]+ ...+ [12 x ]) = 78 x é 1 - e2 ö
g( x ) Î ê , 0÷
Þ [ x ] + [2 x ]+ ...+ [12 x ] = 0 ë1 + e
2
ø
\ 0 £ 12 x < 1
é 1 - e2 ù
1 1 1 For -1 £ f ( x ) < 1 , g( x ) Î ê , 0ú
Þ 0£x< , " £x£
ë1 + e
2
12 25 10 û
é1 1ö 30. f ( x ) = | x | - { x } ; | x | ³ { x }
\ Common values of x Î ê , ÷ .
ë 25 12 ø
Y
1
Since, x is of the form .
n
1 1 1 1 1
\ x = , , ,..., ,
25 24 23 14 13 X′ X
O
i.e. 13 solutions. Y′
26. - 20 £ 2 cos x - 4 sin x £ 20 æ 1ù
Þ X Î ç - ¥, - ú È [ 0, ¥ )
Þ 0 £ (2 cos x - 4 sin x ) 2 £ 20 è 2û
1 1 Þ Y Î [ 0, ¥ ) and f ( x ) is many-one.
min = = ; max = 1
1 + 20 21 31. Given, curves are y = ln x and y = ax.
22 Þ ln x = ax has exactly two solutions.
Þ M+m=
21 ln x ln x
Þ = a has exactly two solutions to find the range of .
æ pö x x
27. f ( x ) = 2 sin ç x + ÷ + 2 2 1
è 4ø x × - ln x
ln x dy x 1 - ln x
æ pö Let y = , x > 0; = =
or f ( x ) = 2 cos ç x - ÷ + 2 2 x dx x2 x2
è 4ø
y is increasing, if 1 - ln x > 0 or ln x < 1 Þ 0 < x < e
Þ Y = [ 2, 3 2 ]
æ 1ù ln x
é 3p p ù é p 5p ù Range of y Î ç - ¥, ú graph of y =
and X = ê- , ú or ê , è eû x
ë 4 4û ë 4 4 úû ln x
For exactly two solutions of =a
28. Let f ( x ) = 2x 3 - 3x 2 - 12x + a, then x
f ¢( x ) = 6 ( x 2 - x - 2 ) = 6 ( x + 1 )( x - 2 ) æ 1ö
Þ a Î ç 0, ÷
è eø
Chap 03 Functions 181

32. Let f ( x ) = ax 2 + bx + c as it touches X -axis at x = 3 In this case any a will satisfy, since 2 cos a can never be more
-b than 2.
Þ =3 Thus, the inequation is satisfied for any x in (0, 1) and for any a.
2a
log x log 2
Þ b = - 6a …(i) If x Î (1, ¥ ), then log 2 x > 0 Þ + >0
Also, 9a + 3b + c = 0 …(ii) log 2 log x
4a - 2b + c = 3 …(iii) The inequation cannot be satisfied unless
3 18 27 cos a = - 1 and x = 2
Þ a = ,b = - ,c =
25 25 25 i.e. log 2 x = 1
3 2 Option (d) is wrong, since in the last case there are infinite
Þ f (x ) = (x - 6x + 9)
25 solution.
| x|
3 æ1ö
33. y = 2 { x } - { x } 2 - 38. If we draw the graph of ç ÷ and x 2 - a, then the range of
4 è2ø
3 1 3 value of a will be ( -1, ¥ ).
Þ 2 {x } - {x } - ³ 0 Þ £ {x } £
2
4 2 2 Y y = x2 + 1
1
Þ £ {x } < 1 (Q 0 £ { x } < 1)
2 (0, 1)
3 1
\ 2 { x } - { x } 2 - is increasing for £ { x } < 1.
4 2
é 1ù X′ X
Þ Range = ê 0, ú
ë 4û
34. Replace x by - x
Y′
Þ x[ f ( x ) + f ( - x )] = 0 Maximum possible solution for ‘x’ is ‘2’.
Þ f ( x ) is an odd function. 39. f ( x ) ¹ | cos x | is true only when
Þ f iv ( x ) is also odd Þ f iv ( 0 ) = 0 | cos x | = 1 Þ x = 0, p , 2 p
35. [y + [y ]] = 2 cos x Þ [y ] = cos x é pö
40. | cos x | + cos x > 0 ® x Î ê 0, ÷
Þ
1
y = [sin x + [sin x + [sin x ]]] = [sin x ] ë 2ø
3 é pö
For x Î ê 0, ÷ , 1 £ sin x + cos x £ 2
Þ[sin x ] = cos x ë 2ø
Number of solutions in [ 0, 2p ] is 0. æ pö
But sin x + cos x ¹ 1 Þ x Î ç 0, ÷
Hence, total solution is 0. è 2ø
\Both are periodic with period 2p. æ pö
Now, for x Î ç 0, ÷ ,
36. By replacing x = x + 1 and x = x - 1, we get è 2ø
f (x + 2) + f (x ) = 2 f (x + 1) …(i) log sin x + cos x ( | cos x | + cos x ) ³ 0
f (x ) + f (x - 2) = 2 f (x - 1) …(ii) 1 æ pù
Þ cos x ³ Þ x Î ç 0, ú
From Eqs. (i) and (ii), gives 2 è 3û
f (x + 2) + f (x - 2) + 2 f (x ) 41. y = (( x + a ) 2 - 1)1/4 = [( x + a - 1)( x + a + 1)]1/4
= 2 [ f ( x + 1 ) + f ( x - 1 )] ( x + a - 1 )( x + a + 1 ) ³ 0
= 2 2 f (x ) x ³ 1 - a and x £ - 1 - a for a > 0
\ f (x + 2) + f (x - 2) = 0 For a < 0, x £ 1 - a, x ³ - 1 - a
On replacing x by x + 2, we get ( x + a ) £ 1 and ( x + a ) ³ - 1
f (x + 4) + f (x ) = 0 For a £ - 1, x £ 0 and range is [ 0, ¥ ).
f ( x + 8 ) = - f ( x + 4 ) = f ( x ), " x 42. f ( x ) = log (log (log x ))
\ f ( x ) is periodic with period 8. log x > 1 when x Î (e, ¥ )
37. The equation has meaning, if x > 0, x ¹ 1. \log (log x ) > 0 and hence, log (log (log x )) is well defined and
\ Domain = ( 0, 1 ) È (1, ¥ ) uniquely.
If x Î( 0, 1 ), then log 2 x < 0 It is evidently one-one. Since, the range of log x = R, f ( x ) is
log x log 2 one-one and onto.
and log 2 x + log x 2 = +
log 2 log x 43. x 2 - 4 px + q 2 > 0, " x Î R
= sum of a negative number £ - 2 Þ 4p 2 - q 2 < 0 …(i)
r + p < qr
2 2
…(ii)
182 Textbook of Differential Calculus

x+r æ pö
2
æ pö p2
2
Let y = Þ ç x - ÷ + çy - ÷ = , i.e. circle
x + qx + p 2
2
è 2ø è 2ø 2
Þ x 2y + x(qy - 1 ) + p 2y - r = 0 …(iii) and f ( x ) = sin -1(sin x ) is shown as
x is real,
Y
Þ (q 2 - 4 p 2 ) y 2 + y ( -2q + 4r ) + 1 > 0
π
From Eq. (i) Þ Coefficient of y 2 is a positive discriminant.
π/2 (π/2,π/2)
= ( 4r - 2q ) 2 - 4(q 2 - 4 p 2 ) π
— g(x)
= 16 (r 2 + p 2 - qr ) < 0 [from Eq. (ii)] √2
X′ X f(x) X′
Hence, Eq. (iii) is true for all real y or y Î ( - ¥, ¥ ). π/2 π
O Y′
44. Here,
x 4 - lx 3 - 3 x 2 + 3 lx Y′
f (x ) =
x -l q
\ Area of shaded part = ´ pr 2
( x 3 - 3 x )( x - l ) 360°
\ f (x ) =
(x - l) 90° p2 p3
= ´p ´ = sq units
Consider, g( x ) = x 3 - 3 x that can be shown as 360° 2 8
x 2 + bx + 1
Y 47. Let y = (b > 1 )
x 2 + 2x + b
2 Þ (y - 1 ) x 2 + (2y - b ) x + (by - 1 ) = 0
Þ D ³ 0 Þ ( 4 - 4b )y 2 + 4y + (b 2 - 4 ) ³ 0 …(i)
X′ X 1
–2 –√3 –1 0 1 √3 2 Since, f ( x ) and have the same bounded set as their range.
f (x )
1
–2 Thus, ( 4 - 4b )y 2 + 4y + (b 2 - 4 ) ³ 0 have roots a and .
a
Y′ \ Product of roots = 1
Now, the range of f ( x ) is the set of entire real numbers, if b2 - 4
l Î [ -2,2 ]. Þ = 1 Þ b 2 - 4 = 4 - 4b
4(1 - b )
Because, if l > 2 or l < -2, then range of f ( x ) Ï R .
or b 2 + 4b - 8 = 0
45. We know that,
-4 ± 16 + 32 -4 ± 4 3
(a - 1 )n = nC 0an - nC1an -1 + nC 2an - 2 b= = = 2 3 -2
2 2
-...+ ( -1 )n -1 × nCn -1 . a + ( -1 )n . nCn p [ x + 24 ] p
48. Since, sin = sin (24 + [ x ])
(a - 1 )n
12 12
\ = nC 0an -1 - nC1 × an - 2 + nC 2 × an - 3
a æ p [x ]ö p [x ]
= sin ç2 p + ÷ = sin
n
Cn è 2 ø 12
+ ...+ ( -1 )n -1 × nCn -1 + ( -1 )n ×
a p [x ]
The period of sin is 24.
(a - 1 ) - ( -1 )
n n 12
Hence, f (n ) = p [x ] p [x ]
a Similarly, period of cos is 8 and period of tan = 3.
4 3
(a - 1 ) 2016 - 1 (a - 1 ) 2017 + 1
Now, f (2016 ) + f (2017 ) = + Hence, the period of the given function = LCM of
a a
1
24, 8, 3 = 24.
(a - 1 ) 2016 [1 + a - 1 ]
= = (a - 1 ) 2016, where a = 3 224 + 1 49. Let f ( x, y ) = ax + by
a
1
Then, f (2 x + 3y , 2 x - 7y )
\ f (2016 ) + f (2017 ) = (3 224 ) 2016 = 3 9 = 3 K = a(2 x + 3y ) + b(2 x - 7y ) = 20 x [given]
\ 2a + 2b = 20 and 3a - 7b = 0
Þ K =9
\ a = 7 and b = 3
46. Here,
2
\ f ( x, y ) = 7 x + 3y
p p2 æ pö
g( x ) = - - çx - ÷ 50. Domain of f ( x ) is [1, 3] and the function is continuous.
2 2 è 2ø
1 1
2 2 f ¢( x ) = - =0
æ pö p2 æ pö 2 x -1 3-x
or çy - ÷ = - çx - ÷
è 2ø 2 è 2ø Þ 3 - x = 2 x -1
Chap 03 Functions 183

Þ 3 - x = 4x - 4 Þ x =
7 59. There are four possible functions defined in 0 £ x < 1, of them
5 2 are continuous and two are discontinuous, now for each of
7 the points (1, 2, . . ., n - 1 ), keep functions fixed from left of the
\Critical points in [1, 3] are 1, and 3. point, so there are 4 possible functions defined in between next
5
two consecutive integral points of them only one is continuous
f (1 ) = 2 2, f (3 ) = 2 and at last for x = n, there is only one possibility of
discontinuity of the function. So, total number of functions
æ7ö 2 8 = 2 ´ 3n - 1 ´ 1
and f ç ÷ = +2 = 10 , 2 2 being < 10, the range
è5ø 5 5
60. Q k Î odd
= [ 2, 10 ].
f (x ) = k + 3 [even]
51. f ( x ) = cos-1 (sec (cos-1 x )) + sin -1 ( cosec (sin -1 x )) k+3
f ( f (k )) =
Þ -1 £ sec (cos-1 x ) £ 1 2
-1 £ cosec (sin -1 x ) £ 1 k+3 k+3
and If Î odd Þ27 = +3
2 2
Þ sec (cos-1 x ) = ± 1
Þ k = 45 not possible.
and cosec (sin -1 x ) = ± 1 k+3
Now, let Î even
p p 2
Þ cos-1 x = 0, p and sin -1 x = , -
2 2 æk + 3ö k + 3
\ 27 = f ( f ( f (k ))) = f ç ÷=
Þ x = ± 1 and x = ± 1 è 2 ø 4
\ Domain is x = ± 1. \ k = 105
52. Put x = y = 1 Þ f (1) = 2 Verifying f ( f ( f (105 ))) = f ( f (108 )) = f (54 ) = 27
1 æ1ö æ1ö \ k = 105
Put y = Þ f (x ) + f ç ÷ = f (x ) f ç ÷
x èxø èxø Hence, sum of digits of k = 1 + 0 + 5 = 6
Þ f (x ) = x 3 + 1 sin( p { x })
61. f (x ) = 4
x + 3x 2 + 7
Þ g( x ) = 0, " x Î R - { 0 }
Here, f (1 / 2 ) = f ( - 1 / 2 )
53. f ( x ) = f ¢( x ) ´ f ¢¢( x ) is satisfied by only the polynomial of
degree 4. Clearly, f ( x ) is not one-one and also it is dependent on x.
Since, f ( x ) = 0 satisfies x = 1, 2, 3 only. It is clear one of the 62. Since, f ( x ) and g( x ) are one-one and onto and are also the
roots is twice repeated. mirror images of each other which respect to the line y = a. It
\ f ¢(1 ) f ¢(2 ) f ¢(3 ) = 0 clearly indicates that h( x ) = f ( x ) + g( x ) will be a constant
function and will always be equal to 2a.
54. Total number of functions for which f (i ) ¹ i = 4 5 and number
63. Let g(t ) = 2t 3 - 15t 2 + 36t - 25
of onto functions in which f (i ) ¹ i = 44.
\Required number of functions = 980 g ¢(t ) = 6t 2 - 30t + 36 = 6(t 2 - 5t + 6 )
55. We can send 1, 2, . . . , n - 1 anywhere and the value of f (n ) will = 6(t - 2 )(t - 3 ) = 0 Þ t = 2, 3
then be uniquely determined. For 2 £ t £ 3,
56. Put sin x = t, g(t ) min = g(3 ) = 2 ´ 27 - 15 ´ 9 + 36 ´ 3 - 25 = 2
Also, 2 + | sin t | ³ 2
y = t 3 - 6t 2 + 11t - 6, -1 £ t £ 1
Hence, minimum f(t ) = 2
f ( -1 ) = - 24, f (1 ) = 0
\ 3x + 3 f (x ) = 2
57. g( x ) = f ( x 2 - 2x - 1) + f (5 - x 2 + 2x )
Þ 3 f (x ) = 2 - 3x Q 3 f (x ) > 0
= 2 x 4 - 8 x 3 - 4 x 2 + 24 x + 18
t=2 t=3
g ¢( x ) = 8 x 3 - 24 x 2 - 8 x + 24
–∞ +ve –ve +ve ∞
g ¢( x ) = 0 Þ x = - 1, 1, 3
We observe that, Þ 2 - 3x > 0 Þ 3 x < 2 Þ x < log 3 2
g( x ) ³ min { g( -1 ), g(1 ), g(3 )} = 0
\ x Î ( - ¥, log 3 2 )
\ g( x ) ³ 0, " x Î R
64. Here, x1 x 2 x 3 = x and x be the element of A.
e-| x |
58. Let f ( x ) = [ x ], g( x ) = 120
2 \ x1 x 2 x 3 =
x4
ée-|x | ù
Þ h( x ) = ê ú = 0, " x Î R Þ x1 x 2 x 3 x 4 = 120 = 2 3 ´ 31 ´ 51.
ë 2 û
184 Textbook of Differential Calculus

Thus, to find number of positive integer solutions of 68. As, a15 and a is the root of x14 + x13 +...+ x + 1 = 0
x1 x 2 x 3 x 4 = 2 3 ´ 31 ´ 51. Now, f ( x ) = x13 + 2 x12 + 3 x11 + ...+13 x + 14
i.e. a + b + g + d = 3, a + b + g + d = 1, a + b + g + d = 1 f (x ) 14
\ = x12 + 2 x11 + 3 x10 + ...+
Þ 3+ 4 -1C 4 -1×1+ 4 -1C 4 -1×1+ 4 -1C 4 -1 = 6C 3×4 C 3×4 C 3 x x
= 20 ´ 4 ´ 4 = 320 \ d = 320 On subtracting, we get
æ 1ö 14
ç1 - ÷ f ( x ) = x + x + x + ...+ x + 1 -
13 12 11
65. –u , v è xø x

A
æ a -1ö 14 - a15 - 14 15
g(x)=Ax+u+v g(x)=–Ax–u+v Put x = a , ç ÷ f (a) = - a - =
14
=-
è a ø a a a
y=f(x)
15
P(1, 4) Q(3, 1) \ f (a) = .
1-a
y=g(x) 1514
f(x)=–Ax–c+d f(x)=Ax+c+d Hence, N =
(1 - a )(1 - a 2 )...(1 - a14 )
–c , d
— and we know that
A
From above figure, x15 - 1 = ( x - 1 )( x - a )( x - a 2 )( x - a 3 )...( x - a14 )
4 = A + u + v, 1 = -3 A - u + v. ...(i) x15 - 1
\ ( x - a )( x - a 2 )( x - a 3 )...( x - a14 ) =
and 1 = 3 A + c + d, 4 = - A - c + d ...(ii) x -1
From Eq. (i), 3 = 4 A + 2 u ...(iii) As, x ® 1.
From Eq. (ii), -3 = 4 A + 2c ...(iv) x15 - 1
On adding Eqs. (iii) and (iv), we get (1 - a )(1 - a 2 )(1 - a 3 )...(1 - a14 ) = lim = 15
x ®1 x - 1
8 A + 2u + 2c = 0
\ N = 15 3 = 313 × 513
u+c
\ = -4 Thus, number of divisors = (13 + 1 )(13 + 1 ) = 14 2 = 196.
A
ì 1 1ü
66. As, f ( x ) is one-one, if f ¢( x ) ³ 0, "x Î R. 69. Here, domain of f ( x ) Þ x Î í - , ý only
î 2 2þ
\ 3 x 2 + 2 x + 4 + a cos x - b sin x ³ 0, "x Î R.
1 æ1ö p
Þ 3 x 2 + 2 x + 4 ³ b sin x - a cos x , "x Î R \ f ( x ) is minimum when x = , i.e. f min ç ÷ =
2 è2ø 2
Þ 3 x 2 + 6 x + 4 ³ a 2 + b 2 , "x Î R 1 æ 1 ö 3p
and f ( x ) is maximum when x = - , i.e. f max ç - ÷ =
[as, b sin x - a cos x £ a + b ] 2 2 2 è 2ø 2
\Sum of maximum and minimum value of function is 2p.
Þ 3x 2 + 6x + 3 + 1 ³ a 2 + b 2 ," x Î R
70. Here, tan -1 ( x 2 - 18x + a ) > 0, " x Î R
Þ 3( x + 1 ) 2 + 1 ³ a 2 + b 2 , " x Î R
Þ x 2 - 18 x + a > 0, " x Î R
\ a 2 + b 2 £ 1 + 3( x + 1 ) 2, since 1 + 3( x + 1 ) 2 ³ 1
Þ (18 ) 2 - 4a < 0
Þ a +b £1
2 2
Þ a > 81
\ Greatest value of (a 2 + b 2 ) = 1 Þ a Î (81, ¥ )
p -1 x4 + x2 + 1 x 4 + 2x 2 + 1 - x 2 (x 2 + 1)2 - x 2 71. As, sin 2 x + sin x + 1 > 0, " x Î R.
67. Here, = 2 = = 2
p + 1 (x + x + 1)2 (x 2 + x + 1)2 (x + x + 1)2 1
\ is always exists.
( x 2 + x + 1 )( x 2 - x + 1 ) sin x + sin x + 1
2
=
(x 2 + x + 1)2
æ 1 ö
p -1 x2 - x + 1 For sin -1 ç 2 ÷ to exists,
\ = , using componendo and dividendo è | x - 1| ø
p + 1 x2 + x + 1
1
2 p 2( x 2 + 1 ) 0< £1
Þ = | x2 - 1 |
2 2x
1 Þ | x2 - 1 | ³ 1
Þ p=x+ …(i)
x Þ x 2 - 1 £ - 1 or x 2 - 1 ³ 1
1-x æ æ 1 öö 1 Þ x2 £ 0 or x 2 ³ 2
As, f ( x ) = Þ f ( f ( x )) + f ç f ç ÷ ÷ = x + ...(ii)
1+ x è è x øø x
Þ x = 0 or (x £ - 2 or x ³ 2 )
æ æ 1 öö
From Eqs. (i) and (ii), we get f ( f ( x )) + f ç f ç ÷ ÷ = p \ x Î ( - ¥, - 2 ] È [ 2, ¥ ) È { 0 }.
è è x øø
Chap 03 Functions 185

72. Here, log ( x 2 - x + 1) is defined, when 77. Functions which are not algebraic, are known as transcendal
functions.
x - x + 1 > 0 and x - x + 1 ¹ 1
2 2

Þ x Î R and x ¹ 0, 1 Þ x Î R - { 0, 1 } …(i) ( x - 1)2 + 1 - 2 x - 1


78. f ( x ) = ×x
Again, log (sin -1 x 2 + x + 1 ) exists, when x -1 -1
x -1 -1 ì x, when x Î [2, ¥ )
0 < x2 + x + 1 £ 1 = ×x =í
Þ x + x £ 0 Þ x Î [ -1, 0 ]
2
…(ii) x -1 -1 î - x, when x Î [1, 2 )
x Î [ -1, 0 ) ì 1, x Î [2, ¥ )
f ¢( x ) = í
73. Here, cos-1 x is defined for x Î [ -1, 1] for sin -1 (3x - 4x 3 ) î -1, x Î [1, 2 )
for x defined 0 £ 3 x - 4 x 3 £ 1 æ3ö
\ f ¢(10 ) = 1, f ¢ ç ÷ = -1
è2ø
\ 3 x - 4 x ³ 0 and 3 x - 4 x £ 1
3 3

æ 3ù é 3ù æ 2p ö
Þ x Î ç - ¥, - 1 + cos ç + 2x ÷
ú È ê 0, ú and x Î [ -1, ¥ ) 1 + cos2 x è 3 ø
è 2 û ë 2 û 79. f ( x ) = +
2 2
é 3ù é 3ù
Þ x Î ê -1, - ú È ê 0, ú é æ pö æpö
ë 2 û ë 2 û ê cos çè2 x + ÷ø + cos çè ÷ø

3 3
74. Here, f ( x ) exists only, if 2
2
( x - 2) 1é æ 2p ö æ p ö 1ù
4x + 8 3 - 52 - 2 2( x - 1) ³ 0. = ê2 + cos2 x + cos ç + 2 x ÷ - cos ç2 x + ÷ - ú
2ë è 3 ø è 3 ø 2û
Þ 2 2x + 2 2( x - 2) - 2 2( x - 1) ³ 52
1 é3 æ pö p æ p öù 3
Þ 2 2x ³ 64 Þ x ³ 3 = ê + 2 cos ç2 x + ÷ × cos - cos ç2 x + ÷ ú =
2 ë2 è 3ø 3 è 3 øû 4
75. y = | sin -1 (2x 2 - 1) | \ f ( x ) is even function, periodic function and
3
dy | sin -1 (2 x 2 - 1 ) | 4x f ( 0 ) = f (1 ) = .
\ = -1
× 4
dx sin (2 x - 1 ) | 2 x | 1 - x 2
2

80. (a) Let f ( x ) = sin(sin -1 x ) = x, " x Î [ - 1, 1]


which would exist, if
So, f ( x ) is one-one and onto.
| 2 x | ¹ 0 , sin -1 (2 x 2 - 1 ) ¹ 0 and 1 - x 2 > 0
2 2
Þ x ¹ 0, 2 x 2 - 1 ¹ 0 (b) Let f ( x ) = sin -1(sin x ) = x
p p
1 é 2 2ù
and | x | < 1 Þ x ¹ 0, ± and | x | < 1 The range is ê - , ú .
2 ë p pû
ì 1 ü
Þ x Î ( -1, 1 ) ~ í 0, ± ý So, f ( x ) is one-one and into.
î 2þ
ì x, x > 0
76. The given function is defined for 5x 2 - 8x + 4 > 0 which is ï
(c) Let f ( x ) = sgn ( x ) × log e x = í - x , x < 0
true, " x Î R. ï 0, x = 0
î
Since, coefficient of x 2 = 5 > 0 and D = 64 - 80 = - 16 < 0
\ Range is [0, 1] for x Î [- 1, 1]
Let g( x ) = 5 x 2 - 8 x + 4 So, f ( x ) is many-one and into.
Here, a =5 > 0 ì x3, x > 0
é D ö é -16 ö é4 ö ï
\Range of g( x ) = ê - , ¥ ÷ = ê - , ¥÷ = ê , ¥÷ (d) Let f ( x ) = x × sgn ( x ) = í - x 3 , x < 0
3

ë 4a ø ë 4 ´ 5 ø ë5 ø ï 0,
î x=0
4
As, £ 5 x 2 - 8 x + 4 < ¥ So, f ( x ) is many-one and into.
5
æ 4ö 81. (a) ( f + g ) (3.5) = f (3.5) + g(3.5) = ( - 0.5) + ( 0.5) = 0
\ log 5/4 ç ÷ £ log 5/4 (5 x 2 - 8 x + 4 ) < log 5/4 ¥
è5ø (b) f ( g(3 )) = f ( 0 ) = 3
Þ -1 £ log 5/4 (5 x - 8 x + 4 ) < ¥
2 (c) f ( g (2 )) = f ( -1 ) = 8
Þ tan -1 ( -1 ) £ tan -1 {log 5/4 (5 x 2 - 8 x + 4 )} < tan -1 ( ¥ ) (d) ( f - g )( 4 ) = f ( 4 ) - g( 4 ) = 0 - 26 = - 26
p p 82. Here, f ( x ) = x 2 - 2ax + a(a + 1)
Þ - £ f (x ) <
4 2 Þ f ( x ) = ( x - a ) 2 + a, x Î [a, ¥ )
é- p pö y = f (x ) = (x - a )2 + a
\ Rf = ê , ÷ Let
ë 4 2ø
186 Textbook of Differential Calculus

Clearly, y ³ a Clearly, the least value of f ( x ) is 4.


Þ ( x - a ) 2 = (y - a ) Þ x = a + y - a The number of integral solutions of f ( x ) = 4 are two, i.e. {2, 3 }.
12
\ f - 1( x ) = a + x -a Also, p - 1, e, Î {2, 3 }
5
-1
Now, f (x ) = f (x ) f ( p - 1 ) + f (e )
\ =1
Þ (x - a ) + a = a +
2
x -a æ 12 ö
2 fç ÷
è5ø
Þ (x - a ) = x - a
2

or ( x - a ) (( x - a ) 3 - 1 ) = 0
87. (a) Number of onto functions
= 4 5 - 4 × 3 5 + 6 × 2 5 - 4 = 240
Þ x = a or a + 1
(b) Number of onto functions, whose range is 3 elements
If a = 5049, then a + 1 = 5050
= 4C 3 (3 5 - 3 × 2 5 + 3 ) = 4 ´ 150 = 600
If a + 1 = 5049, then a = 5048
(c) Number of onto functions, whose range is 2 elements
83. Here, g( x ) = sin(sin -1 { x } ) + cos (sin -1 { x } ) - 1
= 4C 2(2 5 - 2 ) = 6 ´ 30 = 180
= { x } + cos (cos-1 1 - { x } ) - 1
(d) Number of onto functions, whose range is 1 element
= x + 1 - {x } - 1 = 4C1 = 4
If x Î I , then { x } = 0 Þ g( x ) = 0 88. Replacing x by 2,
Also, g( - x ) = g( x ) Þ g( x ) is even. æ1ö
Þ 2 f (2 ) + 2 f ç ÷ - 2 f (1 ) = 4
If x Ï I , then { - x } = 1 - { x } è2ø
Þ g( - x ) = 1 - { x } + { x 2 } - 1 = g( x ) æ1ö
Þ f (2 ) + f ç ÷ = 2 + f (1 ) …(i)
Þ g( x ) is even function. è2ø
\ g( x ) = 0 , x Î I and g( x ) = g( - x ), x Ï I Replacing x by 1,
Þ g( x ) is periodic function. f (1 ) = - 1 …(ii)
84. Given, f (a + x ) = f (a - x ) 1 æ1ö 1 5
Replacing x by , 2 f ç ÷ + f (2 ) + 2 =
2 è2ø 2 2
(a)f (2a - x ) = f (a + (a - x )) = f (a - (a - x )) = f ( x )
\ f (2a - x ) = f ( x ) …(i) 1 æ1ö 1
\ 2 f (2 ) + f ç ÷= …(iii)
(b) f (2a + x ) = f (a + (a + x )) = f (a - (a + x )) = f ( - x ) 2 è2ø 2
\ f (2a + x ) = f ( - x ) …(ii) From Eqs. (i) and (iii), we get
(c) f (2b + x ) = f ( - x ) [from Eq. (ii)] …(iii) æ1ö
f (2 ) = 1, f ç ÷ = 0
(d) From Eqs. (ii) and (iii), we get è2ø
f (2a + x ) = f (2b + x ) 89. Let g( x ) = f (100x ) , g(1) = 1
\Period is (2b - 2a ) . Þ g( x + 1 ) - g( x ) = x + 1
85. Here, f (1 + x ) = f (1 - x ) Putting x = 1, 2, 3,¼, 99 and adding, we get
Þ f ( x ) is symmetric about x = 1. 100
100
g(100 ) = 5050 = år = (100 + 1 ) = 5050
\ g( x ) = f (1 + x ) Þ g( - x ) = f (1 - x ) = f (1 + x ) = g( x ) 2
r =1
Þ g( x ) is an even function.
90. We have, f ( x ) = [1 + sin x ] + [1 + sin 2x ] + ¼ + [1 + sin nx ]
ì10 - 4 x, if -¥ < x < 1
ï 8 - 2 x, if 1 < x £ 2 = n + [sin x ] + [sin 2 x ] + ¼ + [sin nx ]
ïï Q x Î( 0, p )
86. f ( x ) = í 4, if 2 < x £ 3
ï 2 x - 2, if 3 < x £ 4 Þ 0 < sin x £ 1
ï ì n, if n is even
ïî 4 x - 10, if 4 < x < ¥ Þ f (x ) = í
în + 1 , if n is odd
Could be shown as
ì 1 , if x is rational
Y 91. (a) f ( x ) = í
î 0 , if x is irrational
ì 1 , if x is rational
y=4 Þ f (x + k ) = í
î 0 , if x is irrational
X′ X
0 1 2 3 4 Þ f ( x ) is periodic but period cannot be determined.
Y′
Chap 03 Functions 187

ì x - [ x ], 2n £ x < 2n + 1 Þ f (x + 4) = f (x )
ï
(b) f ( x ) = í 1 \ f ( x ) is periodic with period 4.
ïî 2 , 2n + 1 £ x < 2n + 2
94. f ( x ) = 1 - x - x 3
Y 1/2
Replacing x by f ( x ), f ( f ( x )) = 1 - f ( x ) - f 3( x )
1 Hence, the given equation is
X′
–2 –1 0 3 4
X f ( f ( x )) > f (1 - 5 x ), f ( x ) < 1 - 5 x
1 2
f (x ) = 1 - x - x 3
Y′ 1 - x - x 3 < 1 - 5x
\ f ( x ) is periodic with period 2. x 3 - 4x > 0
é 2x ù
ê ú
(c) f (x ) = (-1) ë pû x( x - 2 )( x + 2 ) > 0
é 2( x + p ) ù é 2x ù So, x Î ( -2, 0 ) È (2, ¥ )
ê ú ê ú
Þ f ( x + p ) = ( -1 ) ë p û
= (- 1)ë p û 95. ( x - y ) f ( x + y ) - ( x + y ) f ( x - y ) = 2y (( x - y )( x + y ))
\ f ( x ) is periodic with period p. Let x - y = u, x + y = v
æ px ö uf (v ) - vf (u ) = uv (v - u )
(d) f ( x ) = (ax + a ) - [ax + a ] + tan ç ÷
è 2 ø f (v ) f (u )
- =v -u
æ px ö v u
= {ax + a } + tan ç ÷
è 2 ø æ f (v ) ö æ f (u ) ö
Þ ç - v÷ = ç - u ÷ = constant
è v ø è u ø
ì 1 2ü
\ Period of f ( x ) is LCM of í , ý = 2 f (x )
î a 1þ Let - x = l Þ f ( x ) = ( lx + x 2 )
x
92. Here, ( x + 1) f ( x ) = x
f (1 ) = 2
Þ ( x + 1 ) f ( x ) - x = 0 is (n + 1 ) th degree
l + 1 =2 Þ l =1
Þ ( x + 1 ) f ( x ) - x = ( x - 0 ) ( x - 1 )( x - 2 ) K( x - n ) × k
f (x ) = x 2 + x
Put x = n + 1 Þ(n + 2 ) f (n + 1 ) - (n + 1 ) = (n + 1 )! (k ) …(i)
Put x = - 1 Þ1 = ( -1 )n + 1 × (n + 1 )!(k ) …(ii) 96. Period of sin x = 2p
From Eqs. (i) and (ii), we get 2p
and period of cos ( 4 - a 2 x ) = .
ì 1, when n is odd 4 - a2
(n + 1 ) + ( -1 )n + 1 ï
f (n + 1 ) = =í n æ ö
(n + 2 ) , when n is even 2p
ïî n + 2 Þ LCM ç2 p , ÷ = 4p [given]
ç 4 - a ÷ø
2
è
f (x ) - 5
93. We have, f ( x + 1) = …(i) p
f (x ) - 3 i.e. 4 - a 2 = , where p = 1, 3
2
f (x ) × f (x + 1) - 3 f (x + 1) = f (x ) - 5
3 f (x + 1) - 5 15 7 15 7
Þ f (x ) = Hence, a 2 = , ; a = ± ,±
f (x + 1) - 1 4 4 2 2
3 f (x ) - 5 97. f ( x + y ) = f ( x ) f (a - y ) + f (y ) f (a - x ) …(i)
On replacing x by ( x - 1 ), we have f ( x - 1 ) = …(ii) 1
f (x ) - 1 Put x = y = 0, we get f (a ) =
f (x + 1) - 5 2
Using Eq. (i), we get f ( x + 2 ) = Let y = 0
f (x + 1) - 3
Þ f ( x ) = f ( x ) f (a ) + f ( 0 ) × f (a - x )
f (x ) - 5
-5 1 1
f (x ) - 3 2 f (x ) - 5 Þ f ( x ) = f ( x ) + f (a - x )
= = …(iii) 2 2
f (x ) - 5 f (x ) - 2
-3 Þ f ( x ) = f (a - x )
f (x ) - 3
Put y = a - x in Eq. (i),
3 f (x - 1) - 5
Using Eq. (ii), we get f (x - 2) = f (a ) = ( f ( x )) 2 + ( f (a - x )) 2
f (x - 1) - 1
1
æ 3 f (x ) - 5 ö Þ ( f ( x )) 2 =
3ç ÷ -5 4
è f (x ) - 1 ø 2 f (x ) - 5 1 é 1ù
= = Þ f (x ) = ± êëQ f ( x ) ¹ - 2 úû
…(iv)
3 f (x ) - 5 f (x ) - 2 2
f (x ) - 1 1
Hence, f (x ) =
Using Eqs. (iii) and (iv), we have f ( x + 2 ) = f ( x - 2 ) 2
188 Textbook of Differential Calculus

98. Since, f ( g( x )) is one-one function. 101. We have, f ( x ) = cos-1( -{ -x })


Þ f ( g( x1 )) = f ( g( x 2 )) whenever x1 ¹ x 2 Domain of f ( x ) Î R.
Þ g( x1 ) = g( x 2 ) whenever x1 ¹ x 2 As, 0 £ { - x } < 1 for all x Î R
Þ g( x ) must be one-one. Þ -1 < - { - x } £ 0
Let f ( x ) = y is satisfied by x = x1 and x 2. ép ö
So, range of f ( x ) Î ê , p ÷ .
If g( x ) is such that its range has only one of x1 and x 2, then ë2 ø
f ( g( x )) can be one-one even, if f ( x ) is many-one. \ Range of f ( x ) contains two prime numbers 2 and 3. Graph
99. x sin x is a continuous function value of | x sin x | can be made of f ( x ) does not lie below X -axis. Clearly, f is neither even nor
as large as we like for sufficiently large values of x. odd but many-one.
[x ] \ f ( x + 1 ) = f ( x ) Þ f ( x ) is periodic.
Therefore, range of x sin x = R Þ =0
tan 2 x 102. If we take example of x + sin x, it is non-periodic whereas its
æ pö derivative 1 + cos x is periodic.
For x Î ç 0, ÷ , [ x ] = 0
è 4ø 103. The value of sin 2x + sin ax can be equal to 2, if sin 2x and
æ p ö [x ] sin ax both are equal to one but both are not equal one for any
For x Î ç - , 0 ÷ , > 0. common value of x.
è 4 ø tan 2 x
104. Clearly, f ( x ) is many-one and into function.
[x ]
Therefore, values of are never negative. p 2p
tan 2 x 105. f ( x ) = 2 cos sin x - 2 cos sin x
5 5
[x ]
Thus, range of ¹ R. é p 2p ù
tan 2 x = 2 sin x ê cos - cos
ë 5 5 úû
½ x ½ = 2 sin x [cos 36° - cos 72° ]
½ ½> 1, whenever defined.
½sin x½ = 2 sin x [cos 36° - sin 18° ]
Thus, range of
x
is not R. é 5 + 1 æ 5 - 1öù
sin x = 2 sin x ê -ç ÷ ú = sin x
ë 4 è 4 øû
|x|+ { x } is a continuous function and
\ Range is [ -1, 1 ] .
lim ([ x ] + { x } ) = ¥, 1 1 2p 2p
x®¥ 106. Period of 2 cos ( x - p ) and 4 sin ( x - p ) are ,
3 3 1 1
lim ([ x ] - { x } ) = - ¥
x ® -¥ 3 3
Thus, range of [ x ] + { x } = R. or 6 p , 6 p .
-1
\Period of their sums = 6p
100. (a) f ( x ) = e ln sec x
. g(1 ) = 0 but f (1 ) is not defined. 107. ( f (sin 2x )) 2 = sin 2 x + cos2 x + 2 sin x cos x
Thus, f and g are not identical. = 1 + sin 2 x
(b) f ( x ) = tan (tan -1 x ) = x, " x Î R Þ f ( x ) = 1 + x, " x Î [ -1, 1 ]
and g( x ) = cot (cot -1 x ) = x, " x Î R é p pù
Þ If x Î ê - , ú , then
Thus, f and g are identical. ë 4 4û
ì 1, x > 0 f (tan 2 x ) = 1 + tan 2 x = sec x
ï
(c) f ( x ) = í 0 , x = 0
ï- 1, x < 0 108. Fifth degree equation must have atleast one real root. If it had
î two real roots, f ¢( x ) = 0 must have one real root.
ì 1, x > 0 1
ï 109. Range of Î ( 0, 1 ]
g( x ) = sgn (sgn x) = sgn í 0 , x = 0 1 + x2
ï- 1, x < 0
î æ 1 ö
For domain R, log ç ÷ Î ( - ¥, 0 ]
ì 1, x > 0 è1 + x2 ø
ï
g( x ) = í 0 , x = 0
110. f ( x ) = 2 sin x + 2 cos x - 2 cos x + c = 2 sin x + c
ï- 1 , x < 0
î Þ f (0) = f (p ) = c
Thus, f and g are identical. Hence, many-one function.
cos2 x 111. If f ( x ) = a 0 + a1x + a 2x 2 + . . . + an xn , an ¹ 0.
(d) g( x ) = cot 2 x - cos2 x = - cos2 x
sin 2 x Then, f (n + 1) ( x ) = 0, for all x while any derivative of sin x is
= cot 2 x cos2 x = f ( x ), " x ¹ p never a zero function.
Thus, f and g are identical. Hence, sin x is not polynomial function.
Chap 03 Functions 189

112. f is injective, since x ¹ y ( x, y Î R ). \ [ x ] = 3 and [y ] = 2


Þ loga { x + x + 1 } ¹ loga {y + y + 1 }
2 2 Þ [ x ] = 2 and [y ] = 3
Þ [ x ] = - 1 and [y ] = 0
Þ f ( x ) ¹ f (y )
Þ [ x ] = 0 and [y ] = - 1
f is onto because loga ( x + x2 + 1) = y \ The x + y = [ x ][y ] becomes x + y = 6 Þ x + y = 0
a y - a -y \ Non-integer solution lies on x + y = 6
Þ x=
2 Þ x+y =0
æxö æxö 1é æ x öù
113. f ( x ) - 2 f ç ÷ + f ç ÷ = x 2 118. We have, f ( x ) = ê f ( xy ) + f ç ÷ ú …(i)
è2ø è 4ø 2ë èy øû
2
æxö æxö æxö æxö Interchanging x and y , we get
\ f ç ÷ -2f ç ÷ + f ç ÷ = ç ÷
è2ø è 4ø è8ø è2ø 1é æy öù
f (y ) = ê f ( xy ) + f ç ÷ ú …(ii)
2 2ë è x øû
æxö æxö æ x ö æxö
f ç ÷ -2f ç ÷ + f ç ÷ = ç ÷
è 4ø è8ø è 16 ø è 4 ø On subtracting, we get
1 ì æxö æ y öü
M M M M f ( x ) - f (y ) = í f ç ÷ - f ç ÷ý …(iii)
2 2 î èy ø è x øþ
æxö æ x ö æ x ö æxö
f ç n ÷ -2f ç n + 1 ÷ + f ç n + 2 ÷ = ç n ÷
è2 ø è2 ø è2 ø è2 ø From Eq. (i), put x = 1, we get
On adding, we get 1é æ 1 öù
f (1 ) = ê f (y ) + f ç ÷ ú …(iv)
æxö æ x ö æ x ö 2ë èy øû
f (x ) - f ç ÷ - f ç n + 1 ÷ + f ç n + 2 ÷
è2ø è2 ø è2 ø
æ1ö
æ 1 1 ö f (1 ) = 0 Þ f (y ) = - f ç ÷
= x 2 ç1 + 2 + . . . + 2n ÷ èy ø
è 2 2 ø
æxö æy ö
æ x ö 4x
2
Hence, f ç ÷=-f ç ÷
As n ® ¥, we get f ( x ) - f ç ÷ = èy ø èxø
è2ø 3
æxö
Repeating the same procedure again, we get \ From Eq. (iii) becomes f ( x ) - f (y ) = f ç ÷
èy ø
16 x 2
f (x ) - f (0) =
9 f (x + h ) - f (x )
119. f ¢( x ) = lim =
h®0
\ f (3 ) = 16 + f ( 0 ) h
Using the result in Q. No. 118.
114. f ( x ) - f ( 0) - x = 0
æ x + hö æ hö
16 x 2 9 f ç ÷ f ç1 + ÷
Þ - x = 0 Þ x = 0, è x ø è xø
9 16 lim = lim
h®0 h h®0 h
\Two solutions.
æ hö
32 x f ç1 + ÷
115. f ¢( x ) = \ f ¢( 0 ) = 0 è xø 1 1 2
9 = lim = f ¢(1 ) = × 2 =
h®0 h x x x
×x
116. For integral solution, x
x + y - [ x ][y ] = 0 Þ x + y - xy = 0 2
\ f ¢(3 ) =
Þ -1 + x + y - xy = - 1 Þ ( x - 1 )(y - 1 ) = 1 3
i.e. only possible, if 2
( x - 1 = 1, y - 1 = 1 ) or ( x - 1 = - 1, y - 1 = - 1 ) 120. From Q. No. 119, f ¢( x ) =
x
Þ x = 2, y = 2 or x = 0, y = 0 f ( x ) = 2 log x + C
\ Solutions are (0, 0) or (2, 2). Þ f (1 ) = 0 Þ C = 0
117. For non-integral solution, \ f ( x ) = 2 log x
Let x = [ x ] + f1 \ f (e ) = 2 log e = 2
and y = [y ] + f 2 121. f ( x ) = odd degree polynomial + bounded function
\ [ x ][y ] = [ x ] + f1 + [y ] + f 2
cot -1 x Î ( 0, p )
([ x ] - 1 )([y ] - 1 ) = f1 + f 2 + 1
Also, f ¢( x ) > 0
Now, 0 £ f1 + f 2 < 2
\ f ( x ) is one-one and range of f ( x ) Î R
\ f1 + f 2 = 1
\ f ( x ) is onto.
Þ ([ x ] - 1 )([y ] - 1 ) = 2
Þ f ( x ) is one-one onto.
Which is possible for
190 Textbook of Differential Calculus

1 2( x + 2 )
122. We have, f ( x ) = x 4 + 1 + Þ sin -1 ³1
x2 + x + 1 x -1
= even degree polynomial + bounded function 2( x + 2 ) p
i.e. 1 £ sin -1 £
1 æ 4ö x -1 2
Î ç 0, ÷
x + x + 1 è 3ø
2
2( x + 2 )
Þ sin 1 £ £1
4 x 3( x 2 + x + 1 ) 2 - 2 x - 1 x -1
f ¢( x ) =
(x 2 + x + 1)2 Solving this, we get
Þ f ¢( x ) = 0 has at least one root which is repeated odd ( 4 - sin 1 )
x£- or x > 1 …(ii)
number of times or it has one root which is not repeated, since 2 - sin 1
numerator of f ¢( x ) is a polynomial of degree 7. From Eqs. (i) and (ii), we get
Þ f ( x ) = 0 has a point of extreme. é ( 4 + sin 1 ) ù
\ f ( x ) is many-one into. x Î ê - 5, - ú
ë 2 - sin 1 û
123. f ( x ) = odd degree polynomial + bounded function sin 2x Þ
Sol. (Q. Nos. 130 to 132)
f ( x ) is onto.
P ( x ) + 1 = 0 has a thrice repeated root at x = 1.
f ( x ) is one-one, if f ¢( x ) ³ 0 or f ¢( x ) £ 0, " x
P ¢( x ) = 0 has a twice repeated root at x = 1.
Þ a ³ 1 È a £ - 1 Þa Î R - ( -1, 1 )
Similarly, P ¢( x ) = 0 has a twice repeated root at x = - 1.
124. Dh = { -1, 1}, as minimum occurs before maxima for f ( x ).
Þ P ¢( x ) is divisible by ( x - 1 ) 2( x + 1 ) 2
\ a3 = - 1
Q P ¢( x ) is degree at most 4.
Now, g( x ) = a 0 + a1x + a 2x 2 - x 3
Þ P ¢( x ) = a( x - 1 ) 2( x + 1 ) 2
g ¢( x ) = a1 + 2a 2x - 3 x 2
æ x5 2 3 ö
= - 3( x - 3 )( x + 3 ) = - 3 x 2 + 27 \ P (x ) = a ç - x + x÷ + c
è 5 3 ø
\ a1 = 27, a 2 = 0
\ a1 + a 2 = 27 15
Now, P(1 ) = - 1 and P( -1 ) = 1 Þ a = - and c = 0
8
125. Also, g( -3 ) > 0 and g(3 ) > 0
15 æ x 5 2 3 ö
Þ a 0 > 54 and a 0 < - 54 \ P (x ) = - ç - x + x÷
\ a 0 > 54 8 è 5 3 ø
126. Now, g( x ) = a 0 + 27 x - x 3 15 æ x 4 2 x 2 ö
=- xç - + 1÷
f ( x ) = a 0 + 27 x - x 3 8 è 5 3 ø
f (10 ) = a 0 + 270 - 1000 x 4 2x 2
has only one real root x = 0, as - + 1 has imaginary
Clearly, f (10 ) is defined for a 0 > 730. 5 3
roots.
127. Q ff -1( x ) = x 10
-1
( x )) 4 - 4 ( f -1 ( x )) 2 Also, sum of pairwise product of all roots = - .
2( f =x 3
Þ ( f -1( x )) 4 - 4( f -1( x )) 2 - log 2 x = 0 Also, P ¢¢( x ) = -
15 3
(x - x )
\ -1
( f ( x )) = 2 +2
4 + log 2 x 2
15
-1
\ Range of f ( x ) is [2, ¥ ). Let f (x ) = - (x 3 - x )
2
-1
\ f (x ) = 2 + 4 + log 2 x 15 15
f ¢( x ) = - (3 x 2 - 1 ) = - ( 3 x - 1 )( 3 x + 1 )
-1 2 2
\ f (x ) > 0
– + –
sin x + 4
128. g( x ) = 1
sin x - 2 Þ Maximum at x = .
-6 cos x é é p ùù 3
Þ g ¢( x ) = ³0 êQ x Î ê 2 , p ú ú
(sin x - 2 ) 2 ë ë ûû 130. (a) 131. (c) 132. (b)
Þ g( x ) is an increasing function, hence one-one function. Sol. (Q. Nos. 133 to 135)
1
é æpö ù Given, f - 1( x ) = …(i)
\ Range is ê g ç ÷ , g( p ) ú and lies in [ -5, - 2 ]. f (x )
ë è 2 ø û
Replace x by f ( x ).
129. x : x in domain of g -1( x ), 1 æ1ö
Þ f ( f ( x )) = Þ f -1 ç ÷ = f ( x )
g -1( x ) in domain of f -1( x ) x èxø
2( x + 2 ) æ1ö
g -1( x ) = sin -1 , " x Î [ -5, - 2 ] …(i) Þ f - 1( x ) = f ç ÷ ...(ii)
x -1 èxø
Chap 03 Functions 191

From Eqs. (i) and (ii), we get, p 3p


Þ - <x<
1 æ1ö æ1ö 10 10
= fç ÷ Þ f (x ) × f ç ÷ = 1 ...(iii)
f (x ) èxø èxø But x Î[ 0, p ] domain x Î[ 0, 3 p /10 ).
Putting x = 1, we get (B) 4 sin 2 x - 8 sin x + 3 £ 0, 0 £ x £ 2 p
( f (1 )) 2 = 1 Þ (2 sin x - 1 )(2 sin x - 3 ) £ 0 Þ 2 sin x - 1 ³ 0
1 é p 5p ù
133. f (1) = 1 [ as f (1 ) > 0 ] Þ sin x ³ Þ x Îê ,
2 ë 6 6 úû
134. If f ( x ) is continuous, then being bijective, it will be monotonic. (C) | tan x | £ 1; x Î [ - p , p ] , -1 £ tan x £ 1
1 p p
135. f ( f ( x )) would be increasing but f ( f ( x )) = , i.e. decreasing. np - £ x £ np +
x 4 4
Thus, contradicts the facts of f ( x ) being continuous. p p
Put n = 0, - £x£
Sol. (Q. Nos. 136 to 137) 4 4
f (m + n ) = f (m ) + f (n ) p p
Put n = 1, p- £x£p+
n = N -1 4 4
3p 5p
\ f ( N ) = f (1 ) + f ( N - 1 ) £x£
4 4
f ( N - 1 ) = f (1 ) + f ( N - 2 )
3p
f ( N - 2 ) = f (1 ) + f ( N - 3 ) But, from domain £x£p
4
M M M p p 5p 3p
Put x = - 1, - p - £ x £ - p + Þ - £x£-
f (2 ) = f (1 ) + f (1 ) 4 4 4 4
On adding all, we get 3p
But from domain - p £ x £ -
f ( N ) = Nf (1 ) 4
Now, if f (1 ) = 1 é 3p ù é p p ù é 3p ù
Finally, ê - p , - È - , È , pú
Þ f ( N ) = N , which contains even numbers ë 4 úû êë 4 4 úû êë 4 û
If f (1 ) = 2 Þ f ( N ) = 2 N , which contains even numbers é p ù é 3p ù
But, x Î[ 0, p ] \ x Î ê 0, ú È ê , p ú
\ f (1 ) is 1 or 2. ë 4û ë 4 û
136. (c) 137. (a) æ 1 sin x ö
(D) cos x - sin x ³ 1, 0 £ x £ 2 p Þ 2 ç cos x - ÷ ³1
138. (A) sin (cos x ), sin (cos x ) ³ 0 è 2 2 ø
2np £ cos x £ (2n + 1 ) p æ p pö 1
Þ ç cos x × cos - sin x × sin ÷ ³
0 £ cos x £ 1 è 4 4ø 2
é p pù æ pö 1
x Î ê2np - , 2np + ú Þ cos ç x + ÷ ³
ë 2 2û è 4ø 2
p p p
(B) ( cos (sin x ) ) -1 Þ 2np - £ x + £ 2np +
4 4 4
cos (sin x ) > 0
p
p p Þ 2np - £ x £ 2np
- < sin x < Þ x ÎR 2
2 2
On substituting suitable values of n, according to domain
(C) tan ( p sin x )
p é 3p ù
p sin x ¹ ± x Î ê , 2p ú È { 0}
2 ë 2 û
1 140. (A) When -1 £ x < 0,
sin x ¹ ±
2 f ( x ) Î [ 0, 1 ) and f ( x ) = x + 1
p ì pü
\ x ¹ np ± Þ x Î R - ín p ± ý \ f ( f ( x )) = ( x + 1 ) 2 - 1 = x 2 + 2 x
6 î 6þ
(cos 2 x + 1 )(sec 2 x + 2 tan x )
(D) ln (tan x ) (B)
p 2
tan x > 0 Þ 0 < x < æ 1 - tan 2 x ö æ 1 + tan 2 x + 2 tan x ö
2 =ç + 1 ÷ç ÷
p æ pö è 1 + tan 2 x øè 2 ø
np < x < np + Þ x Î çn p, n p + ÷
2 è 2ø
1 + tan 2 x + 2 tan x
=
139. (A) | 4 sin x - 1 | < 5 Þ - 5 < 4 sin x - 1 < 5 1 + tan 2 x
Þ 1 - 5 < 4 sin x < 1 + 5 æ 2 tan x ö 2 tan x
i.e. f ç ÷ =1 +
1- 5 1+ 5 è 1 + tan 2 x ø 1 + tan 2 x
Þ < sin x <
4 4
\ f (x ) = 1 + x
192 Textbook of Differential Calculus

(C) Put x = 0, y = 0, f (1 ) = (1 + 1 ) 2 = 2 2 Þ
1
= 2n
Put x = 0, y = 1, f (2 ) = (1 + 2 ) = 3 2 2 f (n )
1 1
By induction, f (x ) = (x + 1)2 Þ = 4020 Þ =2
f (2010 ) (2010 ) f (2010 )
éxù
(D) When 4 < x < 5, ê ú = 1 ax + b a
ë 4û 145. If f (x ) = ,x ¹
cx - a c
Þ f (x ) = 2x + 3
y -3 Then, f ( f ( x )) = x
\ y = 2x + 3 Þ x = æ æ 4 öö 4
2 \ f ( f ( x )) = x and f ç f ç ÷ ÷ =
x -3 è è x øø x
\ f - 1( x ) =
2 æ æ 4 öö 4
\ f ( f ( x )) + f ç f ç ÷ ÷ = x + ³ 4
141. (A) Simplifying the expression, we get f ( x ) = .
5 è è x øø x
4
(B) Period of tan (e { x } ) = 1, the period of
146. Here, g = 4 - ( a + b)
[ x + a ] - ( x + a ) + a - 5 is also 1, hence 1 is the period. Þ a 2 + b 2 + g 2 = a 2 + b 2 + ( 4 - ( a + b )) 2 = 6
(C) h( x ) - h( - x ) = 0, hence even function. Þ 2b 2 + 2b( a - 4 ) + (2 a 2 - 8 a + 10 ) = 0
(D) Simplifying the expression, k( x ) = 0. But, b ÎR Þ D ³ 0
142. Here, f ( x + y ) - kxy = f ( x ) + 2y 2 Þ 3a2 - 8a + 4 ³ 0
Put x = 0 Þ f (y ) = f ( 0 ) + 2y 2 Þ a Î[2 / 3, 2 ]
Þ f (x ) = 2x 2 + f (0) \Integer values of a are {1, 2}.
Now, f (1 ) = 2 + f ( 0 ) Þ f (0) = 0 147. Here, f ( x ) = ax + b
\ f (x ) = 2x 2 Þ a( x + f ( x )) + b = x + ax + b
æ 1 ö Þ a f (x ) = x
Hence, f ( x + y ) × f ç ÷=4 x
èx + yø Þ f (x ) =
a
143. Given, f ( x - f (y )) = f ( f (y )) + xf (y ) + f ( x ) - 1 …(i) x
Þ = ax + b
On putting x = 0 and f (y ) = 0, we get a
f (0) = f (0) + f (0) - 1 Þ b = 0 Þ a2 = 1
Þ f (0) = 1 …(ii) Þ a=±1
On putting x = a and f (y ) = a, we get Þ f ( x ) = ± x, i.e. 2 functions.
f ( 0 ) = f (a ) + a 2 + f (a ) - 1 148. Number of one-one functions = 5P3 = 60
a 2
x 2
Number of strictly monotonic functions = 10 + 10 = 20
Þ f (a ) = 1 - \ f (x ) = 1 -
2 2 Number of one - one functions 60
\ = =3
2 Number of strictly monotonic functions 20
(10 )
Þ - f (10 ) = - 1 + = 49
2 149. The number of different sets contains exactly 3 elements of
- f (10 ) B = 5C 3 = 10
Hence, =7
7 The number of onto functions from A to the set contains
144. Given, 3 elements = 10 [3 4 - 3(2 ) 4 + 3 ] = 360
f (1 ) + 2 f (2 ) + 3 f (3 ) + ¼ + nf (n ) = n (n + 1 ) f (n ) …(i) k
\ =6
On putting (n + 1 ) in place of n, we get 60
f (1 ) + 2 f (2 ) + ¼ + n f (n ) + (n + 1 ) f (n + 1 ) 150. Here, f (2) = a sin(2) + 2b cos(2) + 8
= (n + 1 ) (n + 2 ) f (n + 1 ) …(ii) and f ( -2 ) = - a sin(2 ) - 2b cos(2 ) + 8
On subtracting Eq. (i) from Eq. (ii), we get On adding, we get
(n + 1 ) f (n + 1 ) = (n + 1 ) (n + 2 ) f (n + 1 ) - n(n + 1 ) f (n ) f (2 ) + f ( -2 ) = 16
Þ (n + 1 ) f (n + 1 ) - nf (n ) = 0 \ f ( -2 ) = 1
Þ nf (n ) = (n + 1 ) f (n + 1 ) 151. Let y = f (x ) = x 5 + e x /3
Þ 2 f (2 ) = 3 f (3 ) = ¼ = nf (n ) Then, y( 0 ) = 1
1
From Eq. (i), Also, g ¢(y ) =
f (1 ) + [n f (n ) + nf (n ) + ¼ to (n - 1 ) terms] = n(n + 1 ) f (n ) f ¢( x )
Þ f (1 ) + (n - 1 ) n f (n ) = n(n + 1 ) f (n ) 1
\ g ¢(1 ) = =3
f ¢( 0 )
Þ f (1 ) = 2n f (n ) Þ 1 = 2n f (n )
Chap 03 Functions 193

152. f ( x ) = x 3 - 12 x + p 160. Clearly, x 2 + 4x ³ 0


Þ f ¢( x ) = 3 x 2 - 12 = 0 2x 2 + 3 ³ 0 Þ x 2 + 4x ³ 2x 2 + 3
Þ x=±2 and x is an integer.
\ f ( -2 ) = p + 16 and f (2 ) = p - 16 for all p Î {1, 2, …, 15} \ x Î {1, 2, 3 }
f ( - 2 ) > 0, f (2 ) < 0. It has 3 roots in each case. \ n =3
x2 + 4x
1 Now, maximum value C 2x 2 = 12
\ Sq = 3 ´ 15 = 45 Þ × Sq = 9 + 3
5
\ Y = | ln 12 |
153. Given, f (m ) - f (n ) = 3 \ [Y ] = 2 (ln e 2 < ln 12 < ln e 3 )
If m = 125, n = 124 Þ (m - n ) min = 1 \ [n + [Y ]] = [3 + 2 ] = 5
m = 24, n = 5
161. f ( x - 1) + f ( x + 1) = 3 f ( x )
Þ f (m ) - f (n ) = 3 Þ (m - n ) max = 19
Þ f (x ) + f (x + 2) = 3 f (x + 1)
(m - n ) max - (m - n ) min 19 - 1
\ = =9 Putting x = x + 2,
2 2
f (x + 1) + f (x + 3) = 3 f (x + 2)
154. As, x 2 + y 2 = 4
Þ x = 2 cos q, y = 2 sin q f ( x - 1 ) + 2 f ( x + 2 ) + f ( x + 3 ) = 3[ 3 f ( x + 1 )]
x + y3
3 f (x - 1) + f (x + 3) = f (x + 1)
\ = x 2 + y 2 - xy = 4 - 2 sin 2 q
x+y Again, putting x = x + 2,
f (x + 1) + f (x + 5) = f (x + 3)
æ x3 + y 3 ö
Þ ç ÷ = ( 4 - 2(sin 2 q)) max = 4 + 2 = 6 f (x - 1) + f (x + 5) = 0
è x + y ø max f (x + 5) = - f (x - 1)
155. f (2011 ) = (2 + 0 + 1 + 1 ) 2 = 16 f (x ) = - f (x + 6)
f ( x + 12 ) = f ( x )
f 2(2011 ) = (1 + 6 ) 2 = 49 19

f 3(2011 ) = ( 4 + 9 ) 2 = 169 Þ å f (5 + 12r ) = 20 f (5 ) = 20 ´ 10 = 200


r=0
f (2011 ) = (1 + 6 + 9 ) = 256
4 2
Hence, the sum of digits = 2 + 0 + 0 = 2
f 5(2011 ) = (2 + 5 + 6 ) = 169 162. Put x = 1,
æxö
2 f ( x ) = f ( xy ) + f ç ÷
M M
…(i)
f (2011 ) = 256
2n èy ø
\ f 2n + 1 (2011 ) = 169 æ1ö
2 f (1 ) = f (y ) + f ç ÷ …(ii)
f 2011(2011 ) - f 2010(2011 ) 169 - 256 èy ø
Þ = =1 æ1ö
f 2013(2011 ) - f 2012(2011 ) 169 - 256 Þ f (y ) = - f ç ÷
èy ø
x é x ù 2x é 2x ù
156. As, [sin x ] = - + -
2 p êë 2 p úû 5 p êë 5 p úû Replacing x by y and y by x in Eq. (i), we get
æy ö
ì x ü ì 2x ü 2 f (y ) = f (yx ) + f ç ÷ …(iii)
[sin x ] = í ý + í ý èxø
î 2p þ î 5p þ
From Eqs. (i) and (iii), we get
Thus, only one solution, i.e. x = 10p.
æxö ì æ x öü æxö
157. As, xy - 6( x + y ) = 0 Þ( x - 6 ) (y - 6 ) = 36 2{ f ( x ) - f (y )} = f ç ÷ - í - f ç ÷ý = 2 f ç ÷ …(iv)
èy ø î è y øþ èy ø
\( x - 6, y - 6 ) = (1, 36 ), (2, 18 ), (3, 12 ), ( 4, 9 ),(6, 6 ),
æxö
( - 36, - 1 ),( - 18, - 2 ), ( -12, - 3 ), ( - 4, - 9 ), ( -6, - 6 ), f ( x ) - f (y ) = f ç ÷ …(v)
Q x £ y Þ a = 10 èy ø
\ (a - 6) = 4 f (1 + h ) - f (1 )
lim = f ¢(1 ) = 1
h®0 h
158. Here, f ( x ) - 10x = ( x - 1) ( x - 2) ( x - 3) ( x - a )
f (1 + h )
\ f (12 ) + f ( - 8 ) = 19840 lim = 1, as f (1 ) = 0
h®0 h
f (12 ) + f ( - 8 )
Þ =1 æ hö
19840 f ç1 + ÷
f (x + h ) - f (x ) è xø 1
3 - cos 4 q + 4 sin 2 q f ¢( x ) = lim = lim =
159. On adding, a= = (1 + sin 2 q) 2 h®0 h h®0 h x
2
f ( x ) = log | x | + c
On subtracting, b = (1 - sin 2 q) 2
f (1 ) = 0 Þ c = 0
Þ ab = cos4 2 q £ 1 f (e ) = 1
194 Textbook of Differential Calculus

163. f (1) + 2 f (2) + 3 f (3) + . . . + n( f (n )) = n(n + 1) f (n ) Hence, given equation is equivalent to f ( x ) = f -1( x ).
Þ f (1 ) + 2 f (2 ) + 3 f (3 ) + . . . + (n + 1 ) f (n + 1 ) Û f (x ) = x [as f is an increasing function]
= (n + 1 )(n + 2 ) f (n + 1 ) 1+x 1+x
Þ ln =x Þ = e 2x
Þ n(n + 1 ) f (n ) = (n + 1 ) 2 f (n + 1 ) 1-x 1-x
Þ nf (n ) = (n + 1 ) f (n + 1 ) 1+x
Now, draw the graph of y = and y = e 2x . They intersect
i.e. 2 f (2 ) = 3 f (3 ) = . . . = n( f (n )) 1-x
1 each other at x = 0.
i.e. f (n ) =
n 3 x 2 + 9 x + 17 10
1 168. Let y = =1 + 2
\ 2126 f (1063 ) = 2126 ´ =2 3x + 9x + 7
2
3x + 9x - 7
1063 Now, 3 x 2 + 9 x + 7 = 3( x 2 + 3 x ) + 7
x4 + x2 + 1 2
164. Now, f (x ) = 2 Þ f (x ) = x 2 + x + 1 æ 3ö 1 1
x -x+1 = 3 ç x + ÷ + ³ for all x Î R
è 2ø 4 4
Now, f ( wn ) = w2n + wn + 1 = 3 10
Maximum value of 2 is 40.
Q wn = 1, when n is a multiple of 3. 3x + 9x + 7
æ1 - x ö Maximum value of y is 1 + 40 = 41
165. f 2( x ) × f ç ÷=x
3
…(i) \ 5k + 1 = 41 Þk = 8
è1 + x ø
1-x 169. The period of the function is found as follows
Replacing x by , we get Given, f ( x ) + f ( x + 4 ) = f ( x + 2 ) + f ( x + 6 ) …(i)
1+x
3 \ Replacing x by x + 2, we get
æ1 - x ö æ1 - x ö f (x + 2) + f (x + 6) = f (x + 4) + f (x + 8)
f2ç ÷ f (x ) = ç ÷ …(ii) …(ii)
è1 + x ø è1 + x ø From Eqs. (i) and (ii), we get
æ1 + x ö
3 f (x ) + f (x + 4) = f (x + 4) + f (x + 8)
By using Eqs. (i) and (ii), we get f 3( x ) = x 6 ç ÷ Þ f ( x ) = f ( x + 8 ) Þ Thus, 8 is the period.
è1 - x ø
170. Since, f ( x ) is symmetric about y = x.
æ1 + x ö 8
Þ f (x ) = x 3 ç ÷ Þ f ( -2 ) = Þ f - 1( x ) = f ( x )
è1 - x ø 3
\ f 2( x ) = ( f -1( x )) 2 - px × f ( x ) f -1( x ) + 2 x 2 f ( x )
Þ [ f ( -2 )] = 2 Þ |[ f ( -2 )]| = 2
Þ f 2( x ) = f 2( x ) - px × f ( x ) × f ( x ) + 2 x 2 f ( x )
166. f (2a - x ) = f ( x )
Þ f ( x ) × {2 x 2 - px × f ( x )} = 0
Þ f (2a - x ) = - f ( x )
Q f is odd Þ f ( x + 4a ) = f ( x ) 2x 2
Þ f (x ) = [as f ( x ) ¹ 0]
Þ f is periodic with period 4a. px
Þ f (1 + 4r ) = f (1 ) 2x
Þ f (x ) = Þ p =2
¥ p
1
Now, å[ f (1 )] r = 8 Þ =8
1 - f (1 ) 3 x 2 + mx + n mx + n - 3
r=0 171. Here, f ( x ) = =3+
7 x2 + 1 x2 + 1
Þ f (1 ) = Þ 8 f (1 ) = 7 mx + n - 3
8 \ y =3+
1 + x2
e x - e - x e 2x - 1 2
167. Consider, f ( x ) = -x
= 2x = 1 - 2x For y to lie in [ -4,3 ).
e +e
x
e +1 e +1 mx + n - 3 < 0,"x Î R.
4e 2x This is possible only if m = 0.
Þ f ¢( x ) = > 0, " x Î R
(e 2x + 1 ) 2 n -3
When m = 0, then y = 3 + .
Þ f ( x ) is an increasing function. 1 + x2
Þ Domain : R, Range : ( -1, 1 ) If x ® ¥ Þ y max ® 3 -
For f : R ® ( -1, 1 ), Now, y min occurs at x = 0. [ as 1 + x 2 is maximum ]
e x - e -x -x n -3
f (x ) = x , f : ( -1, 1 ) ® R y min = 3 +
=n ...(i)
e + e -x 1
Since, -4 £ y < 3
e y - e -y \ y min = -4
Þ x= ...(ii)
e y + e -y From Eqs. (i) and (ii), n = -4
1+x 1+x m 2 + n 2 16
Þ ey = Þ f -1( x ) = ln \ = =4
1-x 1-x 4 4
Chap 03 Functions 195

172. Since, f ( x ) is monotonic. Þ(1 ! + 2 ! + K + n !) + {(n + 1 )! + (n + 2 )!} = {Q(n ) + P (n )}


Þ f ¢( x ) < 0 or f ¢( x ) > 0, "x Î R. {1 ! + 2 ! + 3 ! + K + n !} + P (n ).(n + 1 )!
Þ f ¢( px ) < 0 or f ¢( px ) > 0, "x Î R. Equating coefficients of {1 ! + 2 ! + ... + n !} and (n + 1 )! on both
Þ f ( px ) is monotonic sides, we get
Þ f ( x ) + f (3 x ) + f (5 x ) + ...+ f ((2m - 1 ) x) Q (n ) + P (n ) = 1 and P (n ) = (n + 3 )
is a monotonic polynomial of odd degree (2m - 1), so it will So, P (n ) = n + 3
attains all real values only once. or P ( x ) = x + 3 and Q( x ) = 1 – P ( x )
\ f ( x ) + f (3 x ) + f (5 x ) + ...+ f ((2m - 1 ) x) = (2m - 1 ) has only Hence, P ( x ) = x + 3 and Q( x ) = – x – 2.
one root. ax
175. Given, f ( x ) = ...(i)
173. (i) We have, a + a x

( x ) 2 = [ x ]2 + 2 x …(i) a1 – x a
Now, f (1 – x ) = 1 – x = ...(ii)
Þ (i + f ) = [i + f ]2 = [i + f ]2 + 2i + 2 f
2
a + a a + ax
[Q x = i + f , where i is integer and f is fraction] From Eqs. (i) and (ii), we have f ( x ) + f (1 – x ) = 1 ...(iii)
Þ {i + f } 2 = (i ) 2 + 2i + 2 f ær ö æ 2n – r ö
Þ fç ÷+ fç ÷ =1
Þ i 2 + 2i + 1 = i 2 + 2i + 2 f è 2n ø è 2n ø
1
Þ f = Þ [ x ] = 0, ( x ) = 1
2n – 1
ær ö æ 2n – r ö
2n – 1
2 Þ å f ç ÷+ å f ç
è 2n ø r = 1 è 2 n ø
÷ = 2n – 1
r =1
Put the value of ( x ) and [ x ] in Eq. (i), we get
1
x = 0, n + , n Î Z
2n – 1
ær ö
2n – 1
æ t ö
2 Þ å f ç ÷+
è 2n ø å f ç ÷ = 2n – 1
è 2n ø
[putting 2n – r = t]
r =1 t =1
(ii) We have,
[2 x ] - 2 x = [ x + 1 ]
2n – 1
æ r ö
Þ - {2 x } = [ x ] + 1 Þ [ x ] + {2 x } = -1
Hence, å 2f ç ÷ = 2n – 1
è 2n ø
r =1
1
\ x = -1, - æ 3 ö
2 176. f ( x ) is defined if ç log| sin x| ( x 2 – 8x + 23) – ÷>0
è log 2|sin x | ø
(iii) We have, [ x ] + 2( x ) = 0, [ x ]2 = 3 x,0 £ x £ 2
2
æ x 2 – 8 x + 23 ö
For 0 £ x < 1, [ x ] = 0, [ x 2 ] = 0 Þ x = 0 Þ log| sin x| ç ÷>0
è 8 ø
1 £ x < 2, [ x ] = 1, [ x 2 ] = 1 Þ x = 1
é 3 log 28 ù
4 ê as = = log| sin x| 8 ú
2 £ x < 2, [ x ] = 1, [ x 2 ] = 2 Þ x = ë log 2 |sin x | log 2 |sin x | û
3
5 This is true, if
3 £ x < 2, [ x ] = 1, [ x 2 ] = 3 Þ x = x 2 – 8 x + 23
3 |sin x | ¹ 0, 1 and <1
8 8
x = 2, [ x ] = 2, [ x ] = 4 Þ x =
2
3 [as |sin x| < 1 Þ log| sin x | a > 0 Þa < 1]
ì 4 5ü é 8 ù x 2 – 8 x + 23
\ x Î í 0, 1, , ý <1
êëQ 3 Ï [ 0, 2 ]úû
Now,
î 3 3þ 8
Þ x 2 – 8 x + 15 < 0
(iv) We have, y = 4 - [ x ]2 and [y ] + y = 6
ì 3p ü
Now, [y ] + y = 6, Þ y = 3 Þ x Î (3, 5 ) – í p , ý
î 2þ
On substituting the value of y in y = 4 - [ x ]2
æ 3p ö æ 3p ö
Þ [ x ]2 = 1, x = ±1, ± 1 + k, where k is fraction. Hence, domain = (3, p ) È ç p , ÷ È ç , 5÷.
è 2 ø è 2 ø
(v) We have, 177. Given, nx + ny = xy
[ x ]+ | x - 2| £ 0, - 1 £ x £ 3
Þ xy – nx – ny + n 2 = n 2 Þ( x – n ) (y – n ) = n 2
For -1 £ x < 0, [ x ] = - Þ - 1 - x + 2 £ 0 x ³ 1, Not
possible. Þ( x – n ) and (y – n ) are two integral factors of n 2.
0 £ x £ 3, [ x ] and | x - 2| are positive. [as x, y , n Î N ]
Hence, no solution. Obviously, if d is one divisor of n 2, then for each such divisor,
174. Given, f (n ) = 1 ! + 2 ! + 3 ! +... + n ! there will be an ordered pair (x, y).
So, S (n ) = number of divisors of n 2.
\ f (n + 2 ) = Q (n ) f (n ) + P (n ) f (n + 1 )
Þ {1 ! + 2 ! + ... + (n + 2 )!} = Q(n ){1 ! + 2 ! + ... + n !} (i) For n = 6, we have d = 1, 2, 3, 6, 9, 12, 18, 36.
+ P (n ){1 ! + 2 ! + ...+ (n + 1 )!} Thus, S (6 ) = 9
196 Textbook of Differential Calculus

(ii) If n is prime, then d = 1, n and n 2. The above result holds if and only if,
Hence, S (n ) = 3, whenever n is prime. f (x ) = 1 + xn

178. We have,
1
+
1
= {x } +
1 If f ( x ) = an xn + an – 1 xn – 1 + ...+ a 0
[ x ] [2 x ] 3
Then, consider (1 + f ( x ))(1 – f (y )) = f ( xy )– 1
Case I x Î I , [ x ] = x, [2 x ] = 2 x { x } = 0 Compare constant term on either side, we have
1 1 1 3 1 9
\ + = Þ = Þ x= 1 – a0 = a0 – 1 Þ a0 = 1
x 2x 3 2x 3 2
Comparing coefficient of xny n , we get
9
Ï I , Not possible. an2 = an Þan = 1 or otherwise polynomial would not be of
2
Case II, x Î I + f , where I is an integer and f is fraction n degree.
1 Comparing coefficient of x, x1, ..., xn –1 on either sides, we have
[ x ] = I , [2 x ] = 2 I { x } = f , 0 £ f <
2 a1 = a 2 = ... = an –1 = 0
1 1 1 3 1 Þ an = 1 and f ( x ) = xn + 1
Þ + =f + Þ f = -
I 2I 3 2I 3
Given, f (2 ) = 5, i.e. 2n + 1 = 5
3 1 1 1 3 5
Þ 0£ =- < Þ £ < Þ n =2
2I 3 2 3 2I 6
9 9 Thus, f (x ) = x 2 + 1
Þ < I £ Þ I = 2, 3, 4
5 2 Þ f ( f (2 )) = f (5 ) = 5 2 + 1 = 26
5 1 1 a +b +c
When I = 2, f = ; I = 3, f = ; I = 4, f = 182. Using AM ³ GM, we have ³ (abc )1/3
12 6 24 3
ì 29 19 97ü a +b +c
\ x Î í , , ý. Þ ³ (a + b + c )1 / 3 [Qa + b + c = abc]
î 12 6 24þ 3
n n
Þ (a + b + c ) 2/3 ³ 3
179. The condition, å f –1 (xi ) = å xi can be written as; Þ (a + b + c ) ³ 3 3
i =1 i =1
n n
1 1 183. Clearly, g( x ) =
å f –1 (xi ) = n å xi
n i =1 i =1 ì x 2, – 2 £ x £ – 1
ï
i.e. AM of y-coordinates of f –1
= AM of x-coordinates of f ï – x, – 1 £ x £ –
1
The given two conditions hold if and only if ï 4
í x + 1 , – 1 £ x < 0 as graphically if,
x 2 + 3x – 3 = x ï 2 4
Þ x 2 + 2x – 3 = 0 ï x, 0 £ x £ 1
ï 2
So, x = –3, + 1 î x , 1 £ x £ 2 can be expressed as shown in the following figure
But x ³ 0 Þ x =1 [neglecting x = –3]
1 n Y
Hence, we can write å xi = 1, which is the required result. y = |x| y = x2 y = |x|
n i =1 (1,1) 1 (1,1)
y = 1/2
180. We have, f ( x ) = x 2 – 2x ...(i)
y = f(x)
and g ( x ) = f ( f ( x ) – 1 ) + f (5 – f ( x )) ...(ii) X′ X
–2 –3/2 –1 –1/2 0 1/2 1 3/2 2
\ g ( x ) = f ( x 2 – 2 x – 1 ) + f (5 – x 2 + 2 x )
= [{ x 2 – 2 x – 1 } 2 – 2 { x 2 – 2 x – 1 }] –1

+ [(5 – x + 2 x ) – 2 { 5 – x + 2 x }]
2 2 2
Y′

Þ g ( x ) = 2 x – 8 x – 4 x + 24 x + 18
4 3 2

To show g ( x ) ³ 0, we find its range. 184. Let g( x ) = f ( x ) – f ( x + p ) ...(i)


i.e. g ¢( x ) = 8 x 3 – 24 x 2 – 8 x + 24 let g ¢( x ) = 0 Y g(0)
Þ x = –1, 1 and 3
Þ g ( x ) ³ min { g (–1 ), g (1 ), g (3 )} = 0
Hence, g ( x ) ³ 0, "x Î R. (π, 0)
181. Given, 2 + f ( x ) f (y ) = f ( x ) + f (y ) + f ( xy ) X′
O X
or 1 – f ( x ) – f (y ) + f ( x ) f (y ) = f ( xy ) – 1
Y′ g(π)
or (1 – f ( x )) (1 – f (y )) = f ( xy ) – 1
Chap 03 Functions 197

At x = p , g( p ) = f ( p ) – f (2 p ) ...(ii) 187. PLAN


At x = 0, g( 0 ) = f ( 0 ) – f ( p ) ...(iii) (i) For such questions, we need to properly define the
Adding Eqs. (ii) and (iii), we have functions and then we draw their graphs.
g( 0 ) + g( p ) = f ( 0 ) – f (2 p ) = 0 (ii) From the graphs, we can examine the function for
Þ g( 0 ) = – g( p ) continuity, differentiability, one-one and onto.
Þ g ( 0 ) and g ( p ) are of opposite sign. ì - x, x < 0
f1( x ) = í x Þ f 2( x ) = x 2, x ³ 0
Þ There is a point c between 0 and p such that îe , x ³ 0
g (c ) = 0 ...(iv) ìsin x, x < 0 ì f 2( f1( x )), x<0
f 3( x ) = í Þ f 4 (x ) = í
From Eq. (i) putting x = c, we have î x , x ³ 0 î 2 1
f ( f ( x )) - 1, x³0
g (c ) = f (c ) – f ( p + c ) ...(v) ì x 2, x < 0 ì x 2, x<0
From Eqs. (iv) and (v), we have Now, f 2( f1( x )) = í 2x Þ f 4 = í 2x
î e , x ³ 0 î e - 1, x³0
f (c ) – f ( p + c ) = 0
Y
Hence, f (c ) = f ( p + c ) y=f1(x) y=f2(x)
Y

185. We have, g(t ) = | t - 1 | - | t | + | t + 1|


ì -t, t < -1 X X
ït + 2, -1 £ t < 0
ï
Þ g(t ) = í
ï2 - t , 0 £ t < 1
ïî t, t ³1 y=f3(x) Y
Graph of g(t ) is y=f4(x)
Y
Y
2 X
g(t)
X

1 y=f2 of1(x)
Y
1
t′ t
− 3/2 − 1/2 O 1
X

Y′
ì -3 ü æ -3 ö
f ( x ) = maxí g(t ) : £ t £ xý x Î ç , ¥ ÷ ì2 x , x<0
î 2 þ è 2 ø As, f 4 ( x ) is continuous, f ¢4 ( x ) = í 2x
î2e , x > 0
Clearly from the graph,
-3 -1 f 4 ¢( 0 ) is not defined. Its range is [ 0, ¥ ), thus f 4 is onto.
ì
ï 3 / 2, 2 < x £ 2 Also, horizontal line (drawn parallel to X-axis) meets the curve
ï -1 more than once, thus function is not one-one.
ï
f ( x ) = í x + 2, £x<0
188. Given, g { f ( x ) } = x Þ g ¢ { f ( x ) } f ¢ ( x ) = 1
2
ï
ï 2, 0 £ x £2 If f ( x ) = 1 Þ x = 0, f ( 0 ) = 1
ïî x, x ³2 Substitute x = 0 in Eq. (i), we get
1
186. n1n2 = 2n1 – n2 [given] g ¢(1 ) =
2n f ¢ (0)
Þ n2(n1 + 1 ) = 2n1 Þ n2 = 1 é 1 x /2 1ù
n1 + 1 Þ g ¢ (1 ) = 2 êëQ f ¢( x ) = 3 x + 2 e Þ f ¢( 0 ) = 2 úû
2

2
Þ n2 = 2 – Alternate Solution
n1 + 1
Given, f (x ) = x 2 + e x /2
Since, n1, n2 are integers.
1
2 Þ f ¢( x ) = 3 x 2 + e x / 2
\ ÎInteger 2
n1 + 1 1
For x = 0, f ( 0 ) = 1, f ¢( 0 ) = and g ( x ) = f -1 ( x )
Þ n1 + 1 = –2, – 1, 1, 2 2
Þ n1 = –3, – 2, 0, 1 Replacing x by f ( x ), we have
Þ n2 = 3, 4, 0, 1 g ( f ( x )) = x Þ g ¢( f ( x )) × f ¢( x ) = 1
Þ Integral solutions of the form (n1, n2 ) are 1
Put x = 0, we get g ¢(1 ) = =2
(–3, 3 ), (–2, 4 ), ( 0, 0 ), (1, 1 ). f ¢( 0 )
198 Textbook of Differential Calculus

1 - cos2 x 1
189. Given, F ( x ) = ò sin 2 x dx = ò dx 192. Given, fk ( x ) = (sin k x + cosk x ),
2 k
1
F ( x ) = (2 x - sin 2 x ) + C where x Î R and k > 1
4 1 1
f 4 ( x ) - f 6( x ) = (sin 4 x + cos 4 x ) - (sin 6 x + cos6 x )
Since, F ( x + p ) ¹ F ( x ) 4 6
Hence, Statement I is false. 1 1
= (1 - 2 sin x × cos x ) - (1 - 3 sin 2 x × cos2 x )
2 2

But Statement II is true, as sin 2 x is periodic with period p. 4 6


( x - 1 )( x - 5 ) 1 1 1
190. Given, f ( x ) = = - =
( x - 2 )( x - 3 ) 4 6 12
The graph of f ( x ) is shown as 193. Plan To check nature of function.
(i) One-one To check one-one, we must check whether
Y f ¢( x ) > 0 or f ¢( x ) < 0 in given domain.
(ii) Onto To check onto, we must check
y=1 Range = Codomain

X'
Description of Situation To find range in given domain
X
0 1 2 3 5 [a, b], put f ¢( x ) = 0 and find x = a1, a 2, …, an Î[a, b ]
Now, find { f (a ), f ( a1 ), f ( a 2 ), K, f ( an ), f (b )}
Y' its greatest and least values gives you range.
Now, f : [ 0, 3 ] ®[1,29 ]
A. If - 1 < x < 1 Þ 0 < f ( x ) < 1 f ( x ) = 2 x 3 - 15 x 2 + 36 x + 1
B. If 1 < x < 2 Þ f ( x ) < 0 \ f ¢( x ) = 6 x 2 - 30 x + 36 = 6 ( x 2 - 5 x + 6 )
C. If 3 < x < 5 Þ f ( x ) < 0 = 6 ( x - 2 )( x - 3 )
D. If x > 5 Þ 0 < f ( x ) < 1 + − +

1 - 2x + 5x 2
é p pù 2 3
191. Given, 2 sin t = , t Îê- , ú
3x 2 - 2x - 1 ë 2 2û For given domain [0, 3], f ( x ) is increasing as well as
decreasing Þ many-one
Put 2 sint = y Þ - 2 £ y £ 2 Now, put f ¢( x ) = 0
1 - 2x + 5x 2 Þ x = 2, 3
\ y =
3x 2 - 2x - 1 Thus, for range f ( 0 ) = 1, f (2 ) = 29, f (3 ) = 28
Þ (3y - 5 ) x 2 - 2 x(y - 1 ) - (y + 1 ) = 0 Þ Range Î[1, 29 ]
\ Hence, the function is into.
Since, x Î R - {1, - 1 / 3 }
194. f ( x ) = x 2, g( x ) = sin x
[ as, 3 x - 2 x - 1 ¹ 0 Þ( x - 1 )( x + 1 / 3 ) ¹ 0 ]
2
( gof )( x ) = sin x 2
\ D³0
go ( gof ) ( x ) = sin (sin x 2 )
Þ 4 (y - 1 ) 2 + 4 (3y - 5 ) (y + 1 ) ³ 0
( fogogof ) ( x ) = (sin (sin x 2 )) 2
Þ y -y -1 ³ 0
2
Given, ( fogogof ) ( x ) = ( gogof ) ( x )
æ 1ö 5
2
Þ (sin (sin x 2 )) 2 = sin (sin x 2 )
Þ çy - ÷ - ³ 0
è 2ø 4 Þ sin (sin x 2 ) {sin (sin x 2 ) - 1 } = 0
æ 1 5ö æ 1 5ö Þsin (sin x 2 ) = 0 or sin (sin x 2 ) = 1
Þ çy - - ÷ çy - + ÷³0
è 2 2 øè 2 2 ø p
Þ sin x 2 = 0 or sin x 2 =
1- 5 1+ 5 2
Þ y £ or y ³ \ x 2 = np [i.e. not possible as - 1 £ sin q £ 1 ]
2 2
1- 5 1+ 5 x = ± np
Þ 2 sin t £ or 2 sint ³
2 2 b-x
195. Here, f ( x ) = , where 0 < b < 1, 0 < x < 1
æ pö æ 3p ö 1 - bx
Þ sin t £ sin ç - ÷ or sin t ³ sin ç ÷
è 10 ø è 10 ø For function to be invertible, it should be one-one onto.
p 3p Check Range
Þ t£- or t ³ b-x
10 10 Let f (x ) = y Þ y =
é p p ù é 3p p ù 1 - bx
Hence, range of t is ê - , - ú È ê , ú .
ë 2 10 û ë 10 2 û Þ y - bxy = b - x Þ x (1 - by ) = b - y
Chap 03 Functions 199

b -y p p
Þ x= , where 0 < x < 1 Thus, g { f ( x ) } is bijective, if - £ 2x £
1 - by 2 2
b -y p p
\ 0< <1 Þ - £x£
1 - by 4 4
b -y b -y p
Þ > 0 and <1 200. Here, f ( x ) = sin -1 (2x ) + , to find domain we must have,
1 - by 1 - by 6
1 p é p pù
Þ y < b or y > sin -1 (2 x ) + ³ 0 -1
êë but - 2 £ sin q £ 2 úû
b 6
(b - 1 ) (y + 1 ) 1 …(i)
and < 0 Þ -1 <y < p p æ– p ö p
1 - by b - £ sin -1 (2 x ) £ Þ sin ç ÷ £ 2 x £ sin
…(ii) 6 2 è 6 ø 2
From Eqs. (i) and (ii), we get
y Î ( - 1, b ) Ì codomain –1
£ 2x £ 1
x 2
196. We have, e -x f ( x ) = 2 + ò t 4 + 1 dt, " x Î ( - 1, 1 ) –1 1
Þ £x£
0 4 2
On differentiating w.r.t. x, we get é– 1 1ù
Q x Îê , ú
e - x [ f ¢ ( x ) - f ( x )] = x 4 + 1 ë 4 2û
Þ f ¢( x ) = f ( x ) + x 4 + 1 ×ex x2 + x + 2
201. Let y = f ( x ) = , x ÎR
x2 + x + 1
Q f -1 is the inverse of f .
x2 + x + 2
\ f -1 { f ( x )} = x \ y =
x2 + x + 1
Þ [ f -1 { f ( x ) }]¢ f ¢ ( x ) = 1
1
1 y =1 + 2 [i.e. y > 1]…(i)
Þ [ f -1 { f ( x ) }]¢ = x +x+1
f ¢( x )
Þ yx 2 + yx + y = x 2 + x + 2
-1 1
Þ [f { f ( x ) }]¢ = Þ x 2 (y - 1 ) + x (y - 1 ) + (y - 2 ) = 0, " x Î R
f (x ) + x4 + 1 × ex
Since, x is real, D ³ 0
At x = 0, f ( x ) = 2
1 1 Þ (y - 1 ) 2 - 4 (y - 1 ) (y - 2 ) ³ 0
Þ { f -1 (2 ) }¢ = =
2+1 3 Þ (y - 1 ) {(y - 1 ) - 4 (y - 2 )} ³ 0
Þ (y - 1 ) ( - 3y + 7 ) ³ 0
197. Since, only (c) satisfy given definition,
7
i.e. f { f -1 ( B )} = B Þ 1 £y £ …(ii)
3
Only, if B Í f (x )
æ 7ù
ì x, x Î Q From Eqs. (i) and (ii), Range Î ç1 , ú
198. Let f( x ) = f ( x ) - g( x ) = í è 3û
î - x, x Ï Q
202. Given, f : [ 0, ¥ ) ® [ 0, ¥ )
Now, to check one-one.
Here, domain is [ 0, ¥ ) and codomain is [ 0, ¥ ). Thus, to check
Take any straight line parallel to X-axis which will intersect
one-one
f( x ) only at one point.
x
Þ f( x ) is one-one. Since, f (x ) =
1+x
To check onto
1
ì x, x Î Q Þ f ¢ (x ) = > 0, " x Î [ 0, ¥ )
As, f ( x ) = í , which shows (1 + x ) 2
î - x, x Ï Q
\ f ( x ) is increasing in its domain. Thus, f ( x ) is one-one in its
y = x and y = - x for rational and irrational values
domain. To check onto (we find range)
Þ y Î real numbers. x
\ Range = Codomain Þ onto Again, y = f (x ) =
1+x
Thus, f - g is one-one and onto.
Þ y + yx = x
199. By definition of composition of function, y y
g ( f ( x ) ) = (sin x + cos x ) 2 - 1, is invertible (i.e. bijective) Þ x= Þ ³0
1 -y 1 -y
Þ g { f ( x ) } = sin 2 x is bijective. Since, x ³ 0, therefore 0 £ y < 1
é p pù i.e. Range ¹ Codomain
We know, sin x is bijective, only when x Î ê - , ú
ë 2 2û \ f ( x ) is one-one but not onto.
200 Textbook of Differential Calculus

203. Given, f ( x ) = 2 x + sin x 207. Given, f ( x ) = (1 + b 2 ) x 2 + 2bx + 1


Þ f ' ( x ) = 2 + cos x æ b ö
2
b2
Þ f ' ( x ) > 0 , "x Î R = (1 + b 2 ) ç x + ÷ +1-
è 1+b ø
2
1 + b2
which shows f ( x ) is one-one, as f ( x ) is strictly increasing.
1
Since, f ( x ) is increasing for every x Î R. m (b ) = minimum value of f ( x ) = is positive and m (b )
1 + b2
\ f ( x ) takes all intermediate values between ( -¥, ¥ ).
varies from 1 to 0, so range = ( 0, 1 ]
Range of f ( x ) Î R.
log ( x + 3 ) log 2 ( x + 3 )
Hence, f ( x ) is one-to-one and onto. 208. Given, f ( x ) = 2 2 =
(x + 3x + 2) (x + 1) (x + 2)
204. The number of onto functions from
For numerator, x + 3 > 0
E = {1, 2, 3, 4 } to F = {1, 2 }
= Total number of functions which map E to F Þ x > -3 …(i)
- Number of functions for which map f ( x ) = 1 and and for denominator, ( x + 1 ) ( x + 2 ) ¹ 0
Þ x ¹ -1, -2 …(ii)
f ( x ) = 2 for all x Î E = 2 4 - 2 = 14
From Eqs. (i) and (ii), we get
205. It is only to find the inverse. Domain is ( - 3 , ¥ ) / { - 1, - 2 }
Let y = f ( x ) = ( x + 1 ) 2 for x ³ - 1 ax
209. Given, f ( x ) =
Þ ± y = x + 1, x ³ - 1 x+1
æ ax ö
Þ y = x + 1 Þ y ³ 0, x + 1 ³ 0 aç ÷
æ ax ö è x + 1ø
Þ x = y -1 f [ f (x ) ] = f ç ÷ =
è x + 1ø ax
+1
Þ f -1
(y ) = y - 1 x+1
ax 2
-1
Þ f (x ) = x - 1 x+1 a 2x
= = = x [given] …(i)
Þ x³ 0 a x + (x + 1) (a + 1) x + 1
1 x +1
2
x+1
206. Let y = x + Þy =
x x Þ a2 x = (a + 1) x 2 + x
Þ xy = x 2 + 1 Þ x [ a2 - (a + 1) x - 1] = 0
Þ x - xy + 1 = 0
2
Þ x( a + 1 )( a - 1 - x ) = 0
y ± y -4 2 Þ a - 1 = 0 and a + 1 = 0 Þ a = - 1
Þ x= But a = 1 does not satisfy the Eq. (i).
2
y ± y2 - 4
210. ( x ) = 1 + x - [ x ] ³ 1
g
Þ f -1 (y ) = since x - [ x ] ³ 0
2
f [ g ( x )] = 1, since f ( x ) = 1 for all x > 0

x2 - 4
Þ f -1 ( x ) = 211. Given, 2 x + 2 y = 2, " x , y Î R
2
Since, the range of the inverse function is [1, ¥), then But 2 x , 2 y > 0, " x , y Î R

x+ x2 - 4 Therefore, 2 x = 2 - 2 y < 2 Þ 0 < 2 x < 2


we take f -1 ( x ) =
2 Taking log on both sides with base 2, we get
x - x -4 2 log 2 0 < log 2 2 x < log 2 2 Þ - ¥ < x < 1
If we consider f -1 ( x ) = , then f -1 ( x ) > 1
2 212. It is given,
This is possible only if ( x - 2 ) 2 > x 2 - 4 f ( q) = sin q (sin q + sin 3 q)
Þ x + 4 - 4x > x - 4
2 2 = (sin q + 3 sin q - 4 sin3 q) sin q

Þ 8 > 4x = ( 4 sin q - 4 sin 3 q) sin q = sin 2 q( 4 - 4 sin 2 q )


Þ x < 2, where x > 2 = 4 sin 2 q cos 2 q = (2 sin q cos q) 2 = (sin 2 q) 2 ³ 0
Therefore, (a) is the correct answer. which is true for all q.
CHAPTER

04
Graphical
Transformations
Learning Part
Session 1
● When f (x) vertically transforms to f (x) ± a
Session 2
● When f (x) horizontally transforms to f ( x ± a ), where a is any positive constant
1
● When f ( x ) transforms to a f ( x ) or f ( x )
a
● When f ( x ) transforms to f ( ax ) or f ( x/a )
● When f ( x ) transforms to f ( − x )
● When f ( x ) transforms to – f ( x )
Session 3
● To draw y = | f ( x )| , when y = f ( x ) is given
● To draw y = f (| x|), when f ( x ) is given
● f ( x ) transforms to | f (| x|)|, i.e. f ( x ) → | f (| x|)|
● To draw the graph of| y | = f ( x ) when y = f ( x ) is given
Session 4
● To draw y = [ f ( x )] when the graph of y = f ( x ) is given and [⋅] denotes greatest integer or integral function
● To draw y = f ([ x]) when the graph of y = f ( x ) is given
● When f ( x ) transforms to y = f ({ x}), where {⋅} denotes fractional part of x, i.e. { x} = x − [ x] or f ( x ) → f ({ x})
● When f ( x ) and g ( x ) are two functions and are transformed to their sum
1
● When f ( x ) transforms to f ( x ) → = h ( x)
f ( x)
● To draw the graph for y = f ( x ) ⋅ sin x when graph of y = f ( x ) is given
● When f ( x ) and g( x ) are given, then find h ( x ) = max ( f ( x ), g ( x )) or h ( x ) = min ( f ( x ), g( x ))

Practice Part
● JEE Type Solved Examples
● Chapter Exercises

Arihant on Your Mobile !


Exercises with this #L
symbol can be practised on your mobile. See title inside to activate for free.
202 Textbook of Differential Calculus

Graph and diagram provide a simple and powerful approach to a variety of problems, from which we can solve them in
a very short time. In this chapter, we will discuss how the graph of a function may be transformed either by shifting,
stretching or compressing or reflection.

Graphical Transformations
A graphical transformation takes whatever its basic function f ( x ). The graph of new function is the graph of the
original function shifted vertically or horizontally.

Session 1
When f (x) Vertically Transforms to f (x) ± a
(Where a is Any Positive Constant)
For f ( x ) → f ( x ) + a, ∴ Locate the points (1, 1), (2, 2), (3, 3), (4, 4) and join
them.
First, draw the graph of f ( x ), then shift the graph of f ( x )
upwards through ‘a’ units such that the distance between For y = − x , x < 0
graphs of f ( x ) and f ( x ) + a at every point is same and is x −1 −2 −3
equal to ‘a’ units.
y 1 2 3
For f ( x ) → f ( x ) – a,
First, draw the graph of f ( x ), then shift the graph of f ( x ) ∴ Locate the points ( −1, 1), ( −2, 2), ( −3, 3) and join them.
downwards through ‘a’ units such that the distance So, we obtained graph of y = x .
between the graphs of f ( x ) and f ( x ) − a at every point is
Y
same and is equal to ‘a’ units.
Graphically, it could be stated as shown in figure. y =|x|= x,
x>0
Y y =|x|=–x,
3
x<0 2
a y = f(x)+a 1
a y = f(x) X
a –3 –2 –1 0 1 2 3
X , where a > 0
0 a y = f(x) – a

Now, to draw the graph of y = | x | + 2 and the graph of x is


Figure 4.1 shifted upwards by 2 units as shown below.

y Example 1 Plot y = | x | and y = | x | + 2. y = |x|+ 2 = – x + 2, y =|x|+ 2= x + 2,


x<0 Y x>0
Sol. We know, y = | x | (modulus function) is define as
 x, x ≥ 0 4
y=
− x , x < 0
y=|x|= – x, 3 y =|x|= x,
x<0 x>0
So, we have to plot two straight lines y = x and y = − x . 2
For y = x , x ≥ 0 1
X
x 1 2 3 4 –3 –2 –1 0 1 2 3
y 1 2 3 4
Chap 04 Graphical Transformations 203

y Example 2 Plot y = | x | and y = | x | − 2. y Example 3 Plot y = e x , y = e x + 1 and y = e x – 1 .


Sol. First, we plot the graph of y =| x | as explained in Example Sol. We know, y = e x (exponential function) could be plotted
1, then shift the graph of | x | through 2 units downwards as shown below which cuts the Y -axis at (0,1) .
as shown below.
Y
y = ex
Y

8
7 (0, 1)
y=–x, x<0 y=x, x>0 1
6 X
y=–x–2, x<0 5 y=x–2, x>0 0
4
⇒ y = e x + 1 is shifted upward by 1 unit such that it
3
intersects Y -axis at (0, 2) and y = e x –1 shifted downwards
2
1 by 1 unit. Such that it intersects Y -axis at (0,0) .
X¢ 0
X The graph of the given functions is shown below.
–8 –7 –6 –5 –4 –3 –2 –1 1 2 3 4 5 6 7 8
–1 Y y = ex+1
–2
–3 y = ex

)
–4

,2
(0
–5 2 ,1
) y = ex – 1
–6 (0
–7 1
–8 X
0 (0,0)
Y′

Exercise for Session 1


n
Directions (Q. Nos. 1 to 8) Plot the following functions.
1. y = x2 +1 2. y = x 2 −1

3. y = x3 +1 4. y = x 3 −1

5. y = sin x + 1 6. y = sin x − 1

7. y = (loge x ) + 1 8. y = (loge x ) − 1
Session 2
When f (x) Horizontally Transforms to f (x ± a), where a is any
Positive Constant, When f (x) Transforms to af (x) or 1 f(x),
a
When f (x) Transforms to f (ax) or f (x/a),
When f (x) Transforms to f (–x), When f (x) Transforms to –f (x)

For f ( x ) → f ( x ± a) y Example 5 Plot y = | x | and y = | x + 2| .


For f ( x ) → f ( x − a ), First, draw the graph of f ( x ), Sol. First we plot the graph of y = | x | as explained in Example 1, then
then shift the graph of f ( x ) through ‘a’ units shift the graph of | x | through ‘2’ units towards left.
towards right such that distance between the graphs y = |x + 2|
of f ( x ) and f ( x − a ) is same and is equal to ‘a’ units. Y
y = |x|
For f ( x ) → f ( x + a ), First, draw the graph of f ( x ), 5
then shift the graph of f ( x ) through ‘a’ units 4
towards left such that the distance between the
3
graphs of f ( x ) and f ( x + a ) is same and is equal to
‘a’ units. 2
Graphically, it could be stated as shown in figure. 1
X
–4 –3 –2 –1 0 1 2 3 4
y=f(x+a)
y=f(x) y=f(x – a)
π π
y Example 6 Plot y = sin  x +  and y = sin  x –  .
a a  4   4
Sol. Let us consider y = sin x , domain of sine functions is R and range
–a a a a is [− 1, 1]. sin x increases strictly from −1 to 1 as x increases from
π π π
a a
− to , decreases strictly from 1 to −1 as x increases from to
2 2 2

. Also, for y = sin x .
2
x −2 π 3π −π π 0 π π 3π 2π
Figure 4.2 − −
2 2 2 2
y Example 4 Plot y = | x | and y = | x – 2| . y 0 1 0 −1 0 1 0 −1 0

Sol. First we plot the graph of y = | x | as explained in  3π   π 


Locate the points ( −2 π,0),  − , 1 , ( − π,0),  − , − 1 , (0,0),
Example 1, then shift the graph of | x | through ‘2’  2   2 
units towards right as shown below. π   3π 
 , 1 , (π,0),  ,− 1 and (2 π,0) on the graph paper, then we get
2   2 
Y the graph of sin x as shown below.
y = |x|
2| 3π ,
Y
π,
–4 – – –1 –1
|x 2 2
1
–3 y= B D
y= sin x
–2
–1 X
–2π 3π – π π 0A π π 3π 2π C
– –
2 2 2 2
X
y= –1
– 4 – 3 – 2 –1 0 1 2 3 4 π, 3π ,
– –1 –1 –1
2 2
Chap 04 Graphical Transformations 205

 π
Now, for the graph of y = sin  x +  shift the graph of
 4 f ( x ) → a1 f ( x ) , a > 1
π
y = sin x through towards left as shown below.
4 First, draw the graph of f ( x ), then shrink the graph of
Y f ( x ), ‘a’ times along Y-axis. As shown in the figure.
Y
p 7p y = sin x

f(x)
4 p 4 2p

y=
X
–2p –p 0 3p
1
4 y= a f(x)
p
y = sin x+ X
4

 π
And for the graph of y = sin  x −  , shift the graph of
 4
π
y = sin x through towards right as shown below.
4
y Example 8 Plot y = sin x and y = 1 sin x .
p
Y y = sin x–
4
2
Sol. First we draw the graph of y = sin x as explained in
sin x Example 6, then shrink the graph of sin x by 2 times along
X 1
p p 5p Y -axis, we get the graph of sin x as shown below.
–2p –p 2
4 4 2p
Y

f ( x ) → {af ( x )}, a > 1 1 y=–1


First draw the graph of f ( x ), then 1/2
–π/2
stretch the graph of f ( x ), ‘a’ times along Y-axis. π X
–π 0 π/2 y = 1/2 sin x
Y y=–1 –1/2 y = sin x
f(x)

–1
y=

x= π
x =3π
x= –π

x =π

2
2

y=af(x)
x=
x=

f ( x )→ f (ax ) , a > 1
First draw the graph of f ( x ), then
y Example 7 Plot y = sin x and y = 2sin x . shrink the graph of f ( x ), ‘a’ times along X-axis as shown
in figure.
Sol. First we draw the graph of sin x as explained in Example
6, then stretch the graph of sin x by 2 times along the Y
Y -axis, we get the graph of 2 sin x as shown below. y = f(ax) y = f(x)

2
y = 2 sin x 1/a 1 X
1 –a –1 –1/a 0
y = sin x
–3p –p –p p X
– 2p 0 p 3p 2p
2 2 2 2
–1

–2
Figure 4.4
206 Textbook of Differential Calculus

y Example 9 Plot y = sin x and y = sin 2x . Y Y


Sol. First draw the graph of sin x as explained in Example 6, y =f(–x) y =f(x)
y =f(–x) y =f(x)
then shrink the graph of sin x, 2 times along X -axis, we
get the graph of sin2x as shown below.
X or X
Y 0 0
1 y=1 y =f(–x)
x
in2
y=s
X
–p – p 0p p y = sin x Figure 4.6
2 2
y = –1
y Example 11 Draw the graph of y = e –x , when the
graph of y = e x is known.
Sol. As y = e x is given, then y = e –x is the image in Y -axis as
 x
f (x ) → f   , a > 1 plane mirror. Thus, it can be drawn as shown in the
 a figure.

First draw the graph of f ( x ), then Y

stretch the graph of f ( x ), ‘a’ times along X-axis, as shown –x


y=e y = ex
in figure.
Y (0,1)
y = f(x) y =f x X
0
a

y Example 12 Draw graph of y = log (– x ), when the


1 a
X
–a –1 0 graph of y = log ( x ) is given.
Sol. As y = log ( x ) is given and y = log (–x ) is the image in
Y -axis as plane mirror. Thus, it can be drawn as shown in
figure.

y = log (–x) Y y = log (x)


Figure 4.5
x
y Example 10 Plot y = sin x and y = sin ⋅
2
0 X
(–1, 0) (1, 0)
Sol. First we draw the graph of sin x as explained in
Example 6, then stretch the graph of sin x, 2 times along
x
X -axis, we get the graph of sin as shown below.
2
Y f (x ) → − f (x )
To draw y = – f ( x ) take image of f ( x ) in the X-axis as
y=1
y =sin(x/2) plane mirror or turn the graph of f ( x ) by 180° about
y =sinx
X X-axis.
–2π 3π –π π 0 π π 3π 2π
– –
2 2 2 2 Y
y = –1
f(x)

X
f ( x ) → f (− x ) 0
– f(x)
To draw y = f (–x ), take the image of the curve y = f ( x ) in
the Y-axis as plane mirror or turn the graph of f ( x ) by
180° about Y-axis. Figure 4.7
Chap 04 Graphical Transformations 207

y Example 13 Draw the graph of y = – e x , when the y Example 14 Draw the graph of y = – log ( x ), when
graph of y = e x is known. the graph of y = log x is known.
Sol. As y = e x is given, then y = – e x is the image of e x in the Sol. As y = log x is given, then y = – log x is the image of
X -axis as plane mirror. log x in the X -axis as plane mirror. Thus, it can be drawn
as shown in figure.
Thus, it can be plotted as shown in figure
Y
Y y = ex

(0, 1) y = log x

0 X X
0
(0, –1) (1,0)
y = – log x
y = –ex

Exercise for Session 2


1. Consider the following function f, whose graph is given below.
Y

X
–2 –1 1 2
–1

Draw the graph of the following functions.


(i) f (x + 1) (ii) f (x − 1) (iii) −f (x ) (iv) f (− x )
(viii) f  
1 x
(v) 2f (x ) (vi) f (x ) (vii) f (2x )
2  2
Session 3
To Draw y = f (|x|),When f(x) is Given, f(x)
Transforms to Draw |f(|x|)|, i.e. f(x) → |f (|x|)| To Draw |y| = f (x)
f ( x ) → |f ( x )| Remark
When y = f ( x ) is given The above transformation of graph is very important in explaining
the differentiability of f ( x ).
 f ( x ), if f ( x ) ≥ 0
We know that | f ( x ) | =  From above example we could say, y = log x is differentiable for
– f ( x ), if f ( x ) ≤ 0 ( 0, ∞ ).
But y =|log x|is differentiable for ( 0, ∞ ) – {1}. As, we have a sharp
Hence, y = | f ( x )| is drawn in two steps
edge at x = 1(which indicates that it is not differentiable at x = 1).
(i) In the step I, leave the positive part of f ( x ), (i.e. the
part of f ( x ) above X-axis) as it is. f ( x ) → f (| x|)
(ii) In the step II, take the image of negative part of f ( x ), When f ( x ) is given
(i.e. the part of f ( x ) below X-axis) in the X-axis as
 f ( x ), if x ≥ 0
plane mirror. We know that, y = f | x | =  .
Or  f (–x ), if x ≤ 0
Take the mirror image (in X-axis) of the portion of the Hence, again f (| x | ) could be drawn in two steps
graph of f ( x ) which lies below X-axis.
(i) In the step I, leave the graph lying right side of the
Or Y-axis, as it is.
Turn the portion of the graph of f ( x ) lying below
(ii) In the step II, take the image of f ( x ) in the Y-axis as
X-axis by 180° about X-axis.
plane mirror.
Y Y
y = f(x) The part of f ( x ) lying left side of theY-axis (if it
exists) is omitted.
X X
–1 0 1 –1 0 1 e.g. Let us assume the graph of f ( x ) as shown
y = f(x)
f(x)
Figure 4.8

y Example 15 Draw the graph of y = |log x |, when the


graph of y = log ( x ) is known.
Sol. To draw the graph of y = | log x |, we have to proceed in
two steps.
Figure 4.9
(i) In the first step, leave the positive part of y = log x ,
i.e. the part of y = log x above X-axis as it is. Now, On the right of Y-axis Plot the graph of f ( x ) as such
Y y = log x
Y
X y = f(x)
0 1
Neglected

X
(ii) In the second step, take the image of negative part
of y = log x , i.e. the part below X-axis in the X-axis Image of
as plane mirror. Thus, the graph can be
f(x) about Y-axis
Y
y = log x when x>0
Figure 4.10
X
0 1
Chap 04 Graphical Transformations 209

On the left of Y-axis Take the mirror image of the graph Again, | f ( x )| → | f (| x | )| can be shown as,
of f ( x ) (of the portion lying on right of Y-axis). Y
Y
y = f( x ) y = f( x )
1

X
0
X –1 – 1 0 1 1
2 2
Figure 4.14

Figure 4.11 y Example 17 Draw the graph for y = | x| 2 − 2| x | − 3 ,


y Example 16 Draw the graph of y = log | x | , when if the graph for y = x 2 − 2x − 3 is given.
graph of y = log( x ) is given. Sol. First we draw the graph for y = x 2 − 2x − 3
Sol. We know, y = log | x | could be drawn in two steps ⇒ ( y + 4 ) = ( x − 1) 2
(i) In step I, leave the graph lying right side of Y-axis as it is. Y
y = x2 – 2x – 3
(ii) In step II, take the image of f ( x ) in the Y-axis as plane
mirror.
X
Thus, it can be plotted as shown in figure. 0 1 3
–1
Y Y –2
y = log x y = log x
–3
–4
(1, – 4)
X X
O 1 –1 O 1
Now, we form f ( x ) → f (| x | ),
i.e. y = x 2 − 2x − 3 ⇒ y = | x | 2 − 2 | x | − 3 as shown below

Y
Note f (|x|) is always an even function.

f ( x ) → |f (| x|)| –3 –1 0 1 3
X
–1
Here, we plot y = | f (| x | )| when graph of f (| x | ) is given, in
–2
two steps –3
(i) In step I, f ( x ) transforms to | f ( x )|.
(–1,– 4) (1,– 4)
(ii) In step II, | f ( x )| transforms to | f (| x | )|.
or In next step, we transform f ( x ) = | f (| x | )| ,
(i) f ( x ) → | f ( x )| (ii) f ( x ) → | f (| x | )| i.e. y = | x | 2 − 2 | x | − 3 → | | x | 2 − 2| x | − 3 | as shown below
e.g. Let graph for f ( x )
Y
Y (–1, 4) (1, 4)
y = f(x)
1
3 2
y = ||x| –2|x|–3|
1 1 X
0
2 X
–1 –3 –1 0 1 3

Figure 4.12 –3

Now, f ( x ) → | f ( x )| can be shown as,


Y
Remarks
1
y =|f(x)| Above mentioned transformations are very useful in explaining
continuity and differentiability of functions as, if y = log ( x ), then
X (i) y = log|x| can be plotted as shown in figure (4.15).
0 1
2 (ii) y =|log|x|| can be plotted as shown in figure (4.16).
Figure 4.13
210 Textbook of Differential Calculus

Y Y Finally, it is as shown
Y
y = |log|x|| (1, 1)
y = log |x| 1

X X
X 0 1 1
(–1, 0) 0 (1, 0) (–1, 0) 0 (1, 0)
2
–1
(1, –1)

Figure 4.15 Figure 4.16 Graph, | y | = f (x )


Figure 4.18
While discussing the case (i), we could say that it is continuous
for all x ∈ R – {0 } and differentiable for all x ∈ R – {0 }. Again,
y Example 18 Draw the graph for | y | = ( x – 1).
while discussing the case (ii), we could say that it is continuous Sol. We know, here | y | = ( x – 1) exists only when ( x –1) is
for all x ∈ R – {0 } and differentiable at all x ∈ R – {–10
, ,1}.
positive. Clearly y = x − 1 represent straight line the
(As now it has sharp edges at {–11 , } and is broken at {0 }). passing through (0, 1) and (0, −1).
∴ Neglecting negative values and will be reflected about
| y| = f ( x ) → y = f ( x ) X -axis. Thus, it can be drawn as shown.
When y = f ( x ) is given Y
|y| = x –1
Clearly, | y | > 0, if f ( x ) < 0, graph of | y | = f ( x ) would not
exist.
And if f ( x ) ≥ 0, | y | = f ( x ) would be given as y = ± f ( x ). O
X

Hence, the graph of | y | = f ( x ) exists only in the regions, (1, 0)


where f ( x ) is non-negative and will be reflected about neglecting (0, –1)
X-axis only when f ( x ) ≥ 0. Region where f ( x ) <0 will be
neglected. Finally, it is shown as
Or Y
To draw the graph of | y | = f ( x ), we use following steps
(i) Remove the portion of the graph, which lies below X-axis.
[Q the equation | y | = f ( x ) is not satisfied when 0 1
X
f ( x ) is negative.]
(ii) Plot the remaining portion of the graph and also take
its mirror image in the X-axis.
Graph for | y | = x − 1
[Q when f ( x ) > 0, then y = ± f ( x )]
y Example 19 Draw the graph for | y | = ( x – 1) ( x – 2) .
Y
(1, 1) y = f(x)
Sol. First we plot y = ( x − 1) ( x − 2)
1 2
 3  1
⇒ y = x 2 − 3x + 2 ⇒  x −  = y + 
 2  4
X
O ½ 1
3 1
It represent a parabola opening upwards having vertex  , −  .
2 4
–2 It intersect X -axis at (0, 1) and (2, 0) and Y -axis at (0, 2).
Thus, we can plot the figure as shown below.
Y
Y
(1, 1) y = f(x)
1
(0, 2)
1/2
X
O
(2, 0)
(1, 0)
–1 (1, –1) mirror image
neglecting O 3/2 3
–2 about X-axis X
–1 (3/2, –1/4)
Figure 4.17 4
Chap 04 Graphical Transformations 211

We know, | y | = ( x – 1) ( x – 2) exists only, when


( x – 1) ( x – 2) ≥ 0 and neglecting, if ( x – 1) ( x – 2) < 0. Summary
Now, f ( x ) will be reflected about X -axis, when • y = f ( x ) to y = − f ( x ) Flip about the X-axis.
( x – 1) ( x – 2) > 0 . • y = f ( x ) to y = f ( − x ) Flip about the Y-axis.
Thus, it can be plotted as shown. • y = f ( x ) to y = f ( x ) Reflect the parts of the graph that lie in
the lower half (negative parts) into the
Y
upper half of the axes.
• y = f ( x ) to y = f ( x ) Discard the left part of the graph (for
x < 0) and take a reflection of the right
part of the graph into the left half of the
axes.
O 1 2
• y = f ( x ) to y = f ( x ) Discard the lower part of the graph
X ( f ( x ) < 0 ) and take a reflection of the
3
upper part of the graph into the lower half
neglecting Image on X-axis, of the axes.
when (x – 1)(x – 2)
• y = f ( x ) to Shift the graph k units upwards or
y = f (x ) + k downwards depending on whether k is
positive or negative respectively.
• y = f ( x ) to Advance (shift left) or delay (shift right)
y = f (x + k ) the graph by k units depending on
Finally, | y | = ( x − 1)( x − 2) can be shown as whether k is positive or negative,
respectively.
• y = f ( x ) to y = kf ( x ) Stretch or compress the graph along the
Y-axis depending on whether k > 1 or
k < 1, respectively. Also, flip it about the
X-axis, if k is negative (this latter
0 1 X statement follows from part 1).
2 3
• y = f ( x ) to y = f (kx ) Stretch or compress the graph along the
X-axis depending on whether k < 1 or
k > 1, respectively. Also, flip it about the
Y-axis, if k is negative.

Exercise for Session 3


1. Consider the following function f, whose graph is given below.
Y
2

0
X
–1 –1 1 1 2
— —
2 2
–1
Draw the graph of the following functions.
(i) | f ( x )| (ii) f (| x | ) (iii) | f (| x | ) − 1|
2. Plot the following functions.
(i) y = |x 2 − 2x − 3 | (ii) y = x 2 − 2 |x |− 3 (iii) y = |log2 x | (iv) y = |log2 |x ||
(v) y = log2 |1− x | (vi) y = log2 (2 − x )2 (vii) y = |cos |x || (viii) y = |2 − 2x |
(ix) y = sin(|x |) (x) y = cos(|x |)
3. Plot the following functions.
(i) |f (x )| = log2 x (ii) |f (x ) | = log2 (− x )
4. Find the number of solutions of sin πx = |loge| x ||.
5. Find the number of solutions of
(i) 2 | x | = sin x 2 (ii) sin x = x 2 + x + 1
Session 4
To draw y = [f (x)], when the graph of y = f(x) is given and [.] denotes
greatest integer fuctions; To draw y = f([x]) when the graph of
y = f(x) is given; When f(x) transforms to y = f({x}); where {.} denotes
fractional part of x, i.e. {x} = x – [x]; When f(x) and g(x) are Two
Functions and are transformed to their Sum; To draw f(x) → 1/f(x)
= h(x), To draw y = f(x) . sin x, When f(x) & g(x) are given, then find
h(x) = max(f(x), g(x)) or h(x) = min(f(x),g(x))

To Draw y = [f ( x )] Y
Where, [⋅] denotes greatest integral function {i.e. now y
cannot be fraction}.
Here, in order to draw y =[ f ( x )] mark the integer on Y-axis. 3

Draw the horizontal lines through integers till they intersect 2


the graph. Draw vertical dotted lines from these intersection 1
X
points. 0

Finally, draw horizontal lines parallel to X-axis from any –1


intersection point to the nearest vertical dotted line with –2
x1 x2 x3 x4 x5 x6 x7 x8
blank dot at right end in case f ( x ) increases.
Or
Figure 4.19
To plot y =[ f ( x )] , we use the following steps
Step I Plot f (x ). y Example 20 Draw the graph of y = [ x 3 ] ,
Step II Mark the intervals of unit length with when − 21/ 3 ≤ x ≤ 21/ 3 .
integers as end points on Y-axis.
Sol. Here, in order to draw the graph of y = [ x 3 ] .
Step III Mark the corresponding intervals {with
the help of graph of f (x )} on X-axis. Step I First we draw the graph at y = x 3 .
Step IV Plot the value of [ f (x )] for each of the The function f ( x ) = x 3 is called the cube function.
marked intervals. The domain and range of cube function is R. Since,
y = x 3 is an odd function, so its graph is symmet-
Graphically, it could be shown as rical in opposite quardrant.
For y = x 3 , we have following table.
Y
3
x 0 1 21/3 −1 −21/3
2
y = f(x) y 0 1 2 −1 −2
1
Now, plot the points (0,0) (1,1), (21/ 3 ,2), ( −1, − 1) and
0 X
(−21/ 3 ,2) and join them by free hand, we get the
–1 graph of y = x 3 .
–2 Step II Draw horizontal lines through integers located on
Y -axis till they intersect the graph.
x1 x2 x3 x4 x5 x6 x7 x8
Chap 04 Graphical Transformations 213

Step III Draw vertical dotted lines from these intersection y = sin x y = [sin x]
Y
points to intersect X -axis.
Y y = x3 1
1
2 (2 3 , 2)

1 (1,1) y = [x3] X
0
(0, 0)
1 1 X
–23 –1 0 1 23 –1

y= in x
(–1,–1) –1 p p
5p 3p –p x = – p 3p 2p 5p 3p
–3p – –2p – x=
1
–2 2 2 2 2 2 2
(–2 3 , 2)

Y
Step IV Finally, draw horizontal lines parallel to X -axis from
any intersection point to the nearest vertical dotted 1
line with blank dot at right end.
X¢ X
Y –2p – 3p –p 0 p p 2p
2 2
–1
2

1 Y¢
Graph for y = [sin x ]
X¢ X
1/3 –1 0 1 1/3
–2 2
–1
To draw y = f [x ]
–2 when the graph of y = f ( x ) is given, following steps are
Y¢ used.
Graph for y = [x 3 ] , x ∈ [ −21/3, 21/3 ] Step I Plot the straight lines parallel toY-axis for integral
value of x, i.e. n = ...−3, − 2, − 1, 0, 1, 2, 3.
y Example 21 Draw the graph of y = [sin x ] . Step II Now, mark the points of which
x = − 3, x = − 2, x = − 1, x = 0, x = 1, K on the curve.
Sol.Step I First we draw the graph of y = sin x as (explained in
Example 6) shown below. Step III Take the lower marked point for x (say) if
n < x < n + 1, then take the point at x = n and draw
y = sin x y = [sin x]
Y
a horizontal line to the nearest vertical line formed
by x = n + 1, proceeding in this way, we get
required curve.
1 Mark the integers on the X-axis. Draw vertical
X
lines till they intersect the graph of f ( x ). From
–3π –2π –π 0 π 2π 3π
y = sin x these intersection points, draw horizontal lines
–1 (parallel to X-axis) to meet the nearest right
vertical line with a blank dot on each nearest right
vertical line which can be shown as in the figure.
Y
Step II Now, draw horizontal lines through integers located y = f(x)
as Y-axis till they intersect the graph.
Step III Draw vertical lines from the intersection points to y = f([x])
intersect X -axis. –4
–3 X
Step IV Finally, draw horizontal lines parallel to X -axis –2 –1 0 1 2 3 4
from any intersection point to the nearest vertical
dotted line with blank dot at right end as shown
below. Figure 4.20
214 Textbook of Differential Calculus

y Example 22. Draw the curve y = e [ x ] . Y


Sol. Step I. First we draw y = e x as shown below by figure.
y = (x–1)2
Y ex

X
–2 –1 0 1 2
(0, 1) (1, 0)
X Graph for y = (x − 1)2

Now, to plot y = ({ x } − 1)2 retain the graph for the


interval x ∈[0, 1) and repeat for length one as shown
Y′ below.
Step II. Draw straight lines parallel Y -axis for integer values Graph for y = ({ x } − 1)2
of x. Y
Step III. Now, mark the points as the curve for
n = − 4, − 3, − 2, − 1, 0, 1, 2, 3, K y = ({x} –1)
1
Step IV. Now, take the lower value of x ( = − 4 ) and draw a
horizontal line to the nearest vertical line with a blank
dot at left end, as shown in figure given below. X
–3 –2 –1 0 1 2 3 4
y = ex

Y
To Plot y = f ( x ) + g( x ), when a ≤ g( x ) ≤ b
There is no direct approach, but we can use the following
y= e[x] steps :
If maximum or minimum value of any one is known.
Step I Find the maximum and minimum value of g( x )
–4 –3 –2 –1 O 1 2 3
X say a ≤ g( x ) ≤ b.
Step II Plot the curve f ( x ) between f ( x ) + a to f ( x ) + b,
i.e. f ( x ) + a ≤ h( x ) ≤ f ( x ) + b.
Step III
To Plot y = f ({ x }) When g( x ) = 0
Graph of f ({ x }) can be obtained from the graph of f ( x ) by ⇒ h (x ) = f (x )
following rule. ‘‘Retain the graph of f ( x ) for values of x StepIV
lying between interval [0, 1). Now, it can be repeated for When g ( x ) > 0, then h ( x ) > f ( x ), i.e. the graph of
rest of the points. (taking periodicity 1). h ( x ) lies above the graph of f ( x ).
Now, obtained function is graph for y = f ({ x }). Step V When g ( x ) < 0, then h ( x ) < f ( x ), i.e. the graph of
Graphically, it could be stated as h( x ) lies below the graph of f ( x ).
(i) Graph for y = f ( x ) (ii) Graph for y = f ({ x }) The procedure is illustrated by following example.
Y y Example 24 Plot y = x + sin x .
Y
y1 Sol. Here, y = f ( x ) + g ( x ) = x + sin x
y1
X X where f ( x ) = x and g ( x ) = sin x .
0 1 –2 –1 0 1 2 3
As we know, g ( x ) = sin x ∈ [ −1, 1]
Figure 4.21 ∴ x − 1≤ y ≤ x + 1 …(i)
To sketch the curve between two parallel line y = x + 1 and
y Example 23. Draw the graph for y = ({ x } − 1) 2 . y = x − 1.
Sol. First we will draw the graph for y = ( x − 1)2 , clearly it Here, g ( x ) = 0 ⇒ y = x …(ii)
represent a parabola opening upward and having vertex Also, when g ( x ) > 0
(1, 0) which could be ploted as shown below. ⇒ x + sin x > x …(iii)
Chap 04 Graphical Transformations 215

and when g ( x ) < 0 1


(iv) As, f ( x ) → 0, i.e. → 0 as x → ∞
⇒ x + sin x < x …(iv) | x |–2
Using Eqs. (i), (ii), (iii) and (iv), we have the following graph. and when x → ∞ ⇒ y = | x |– 2 → ∞

x–
1 (v) y = ± 1 ⇒ f ( x ) = ± 1

1
y=

x+
=x ∴ f ( x ) could be plotted as,

y=
y
x (3π/2, 3π/2–1) Y
Y sin (π, π)
x+
y = (π/2,π/2+1)
y= 1
5 y= 1
|x| – 2 |x| – 2
1 4
3
y = |x| – 2
2
π X
–3π/2 –π –π/2 –1 (0,0) 1 π/2 3π/2 1
sin x 0
X
1 5 4 3 2 1 1 2 3 4 5
1
2
(–3π/2, –3π/2+1) (–π/2,–π/2–1)
(–π, –π)

y= 1
|x| – 2

1 To Plot the Graph of y = f ( x ) ⋅ sin x ,


When f ( x ) Transforms to = h( x )
f (x ) when Graph of y = f ( x ) is given
(i) when f ( x ) increases, then h ( x ) decreases. Clearly, – f ( x ) ≤ f ( x ) ⋅ sin x ≤ f ( x ) [as –1 ≤ sin x ≤ 1]
(ii) when f ( x ) decreases, then h ( x ) increases. Hence, graph for y = f ( x ) ⋅ sin x would be lying between the
graph of y = f ( x ) and y = – f ( x ). It amounts to just drawing
(iii) as f ( x ) → 0, h ( x ) → + ∞. graph of sin x in between the graphs of y = ± f ( x ).
(iv) as f ( x ) → ∞, h ( x ) → 0. The procedure is illustrated by following example.
(v) when f ( x ) = ± 1, then h ( x ) is also equal to ± 1.
 1 
1 y Example 26 Let f ( x ) =  x sin , x ≠ 0
y Example 25 Plot y = | x |– 2 and hence f ( x ) = ⋅ x 
| x |– 2  0, x = 0
Sol. First we plot y = | x |– 2 as explained in Example 2.  2 1 
x sin , x ≠ 0
The graph is shown below. and g ( x ) =  x .
Y  0, x = 0
y = – x – 2, x < 0
3 Discuss the graph for f ( x ) and g ( x ), and evaluate the
2 y = |x| – 2, x > 0 continuity and differentiability of f ( x ) and g ( x ) .
1 Sol. We will first try to graphically understand the behaviour
X′ 0 1 X of these two functions and then verify our results
4 3 2 1 2 3 4
1 analytically. Notice that no matter what the argument of
2 the sine function is, its magnitude will always remain
between − 1 and 1.
Y′ 1 1 2
Therefore, x sin ≤ x and x 2 sin ≤ x
x x
Here, (i) y = | x | – 2 increases when x > 0
1
1 This means that the graph of x sin will always lie
⇒ f (x ) = decreases when x > 0 x
| x |–2 between the lines y = ± x and the graph of x 2 sin 1 will
x
always lie between the two curves y = ± x 2 . Also, notice
and (ii) y = | x |– 2 decreases when x < 0
1 that as x increases, 1 decreases in a progressively slower
⇒ f (x ) = increases when x < 0 x 1
| x |–2 manner while when x is close to 0, the increase in is
x
very fast (as x decreases visualise the graph of y = 1 / x ).
(iii) As, y → 0 ⇒ x = ± 2 ⇒ f ( x ) → ∞ as x = ± 2
216 Textbook of Differential Calculus

This means that near the origin, the variation in the graph due to the shrinking envelope. The slope of the curve also keeps
on changing and does not approach a fixed value.
of sin 1 will be extremely rapid because the successive 1
x However, for g( x ) = x 2 sin , the slope of the envelope is itself
zeroes of the graph will become closer and closer. As we x
keep on increasing x, the variation will become slower and decreasing as we approach the origin, apart from shrinking in
width. This envelope will ‘compress’ or ‘harmmer out’ or ‘flatten’
slower and the graph will ‘spread out’.
the sin oscillations near the origin. What should therefore
1 happen to the derivative? It should become 0 at the origin!
For example, for x > there will not be finite root of the
π Let us ‘zoom in’ on the graphs of both the functions around the
1 origin, to see what is happening.
function. Only when x → ∞, sin will again approach 0.
x f(x) f(x)
f(x)=x sin 1
x
y=x

X X

X
1 1 1
3p 2p p
f(x) f(x)

y=–x

1 y=x2
f(x)=x2 sin x
X X

1 1 1 f(x) f(x)
1 1 1
p 2p 3p 3p 2p p

X X
y=–x2

Figure 4.22

Remarks Figure 4.23


Notice how the lines y = ± x ‘envelope’ the graph of the function These graphs are not very accurate and are only of an
in the first case and the curves y = ± x 2 ‘envelope’ the graph of approximate nature; but they do give us some feeling on the
the function in the second case. behaviour of these two functions near the origin.
1
The envelopes shrink to zero vertical width at the origin in both The x sin graph keeps ‘continuing’ in the same manner no
cases. Therefore, we must have x
1
1 1 matter how much we zoom in; however, in the x 2 sin graph, the
lim x sin = 0 and lim x 2 sin = 0. x
x→ 0 x x→ 0 x decreasing slope of the envelope itself tends to flatten out the
1 curve and make its slope tend to 0. Hence, the derivative of
(This is also analytically obvious; sin is a finite number between
x 1
x sin at the origin will not have any definite value, while the
− 1and 1; when it gets multiplied by x (where x → 0) the whole x
product gets infinitesimally small). 1
derivative of x 2 sin will be 0 at the origin.
Now lets try to get a ‘feel’ on what will happen to the derivatives x
of these two functions at the origin. Lets verify this analytically
1
For f ( x ) = x sin , the slope of the envelope is constant ( ± 1). 1
x (i) f ( x ) = x sin at x = 0.
x
Thus, the sinusoidal function inside the envelope will keep on 1
h sin
oscillating as we approach the origin, while shrinking in width f ( h) − f ( 0 ) h = lim  sin 1 
LHD = RHD = lim = lim
h→ 0 h h→ 0 h h→ 0  h
Chap 04 Graphical Transformations 217

This limit, as we know does not exist; hence, the derivative for Y y = max {2x, x2}
f ( x ) does not exist at x = 0
1
(ii) f ( x ) = x 2 sin
at x = 0 (2, 4)
x B
1 X′ X
h2 sin − 0 0 2
h 1
LHD = RHD = lim = lim h sin = 0.
h→ 0 h h→ 0 h
neglecting
To Find h( x ) max (f ( x ), g( x )) Y′

or h( x ) = min(f ( x ), g( x )) when f ( x ) and g( x ) Thus, from the given graph y = max {2x , x 2 }, we can say
are given y = max {2x , x 2 } is continuous for all x ∈ R.

(i) h ( x ) = max { f ( x ), g ( x )} But y = max {2x , x 2 } is differentiable for all x ∈ R – {0,2 }. As


 f ( x ), when f ( x ) > g ( x ) at x = 0, 2 curve has corner point.
Q h(x ) = 
 g ( x ), when f ( x ) < g ( x )
Remarks
To draw the graph of y = max{ f ( x ), g( x )}, first we draw One must remember the formula, we can write
the graphs of both the functions f ( x ) and g( x ) and find f (x) + g (x) f (x) – g (x)
max { f ( x ), g ( x )} = +
their point of intersection. 2 2
f (x) + g (x) f (x) – g (x)
Then, we find any two consecutive points of intersection. and min { f ( x ), g ( x )} = – .
2 2
In between these points f ( x ) > g( x ), then in order to draw
the graph of max { f ( x ), g( x )}, we take those segments of y Example 28 Draw the graph for y = | 2 – | x – 1|| .
f ( x ) for which f ( x ) > g( x ) between any two consecutive Sol. First we plot y = x − 1 , it is a straight line intersecting X
points of intersection of f ( x ) and g( x ). and Y -axes at (1, 0) and (0, − 1), respectively. The graph is
(ii) h ( x ) = min { f ( x ), g ( x )} shown below.
Y
 f ( x ), when f ( x ) < g ( x ) x–1=y
Q h(x ) = 
 g ( x ), when g ( x ) < f ( x )
B (1, 0)
Similarly, in order to draw the graph of min { f ( x ), g( x )} X′ X
0
we take those segments of f ( x ) for which f ( x ) < g( x ), (0, –1)

between any two consecutive points of intersection of


f ( x ) and g( x ). Y′
⇒ Now, we plot y = | x – 1| , For this, the portion of the
y Example 27 Draw graph for y = max{ 2x , x 2 } and graph remain same which lines above the X -axis and reflect
the negative portion of the graph about X -axis.
discuss the continuity and differentiability.
Y
Sol. Here, to draw, y = max {2x , x 2 }.
1
First, plot y = 2x and y = x 2 on graph and put 2x = x 2
y = |x – 1|
⇒ x = 0, 2 (i.e. their point of intersection).
Clearly, y = x 2 represent a parabola open upward with X′ X
0 1
vertex (0, 0) and y = 2x represent a straight line pointing
through (0, 0). Y′
Y
y=x2 ⇒ Now, to plot y = − | x − 1| take the mirror image of the
y = 2x graph of | x −1| in the X -axis as plane mirror.

A Y
y = – |x – 1|
B
X
(0, 0) 2 –1 1 2 3
X′ X
0

–1
Now, since y = max {2x , x 2 } we have to neglect the curve –2
below the point of intersections thus, the required graph is, Y′
as shown below.
218 Textbook of Differential Calculus

To find the graph of 2 − | x − 1| . Shift the graph of y = − | x − 1| Sol. Here, h ( x ) = min { x ; x 2 } can be drawn on graph in two
upward through 2 units. steps.
Y (a) Draw the graph of y = x and y = x 2 , also find their point
2 of intersection.
y = 2–|x–1|
i.e. x = x 2 ⇒ x = 0, 1
X′ X
(–1, 0) 0 1 (3, 0)
Y′
Thus, points of intersection are (0, 0) and (1,1).
Y
Y y = x2
To find the graph of | 2–| x – 1||, reflect the negative portion
of the graph of 2 − | x − 1| in X -axis. y=x
Thus, y = | 2–| x – 1 || can be plotted as shown below. 1

Y X
0 1
y = | 2 – |x – 1| |
(0, 2)

(b) To find h ( x ) = min { x ; x 2 } neglecting the graph above


X′ X the point of intersection, we get
(–1,0) 0 (1, 0) (2, 0)
Y
neglecting
Y′ g
ctin
egle
Remark 1 n
From above figure we could say y =|2 –|x – 1|| is not
neglecting
differentiable at x = {–1, 1, 3 } as sharp edges at x = – 1, 1, 3. X
(0, 0) (1, 0)
y Example 29 Let h ( x ) = min { x ; x 2 } for every real
number of x . Then, which one of the following is true?
[IIT JEE 1998]
Thus, from the given graph,
(a) h is not continuous for all x
 x , x ≤ 0 or x ≥ 1
(b) h is differentiable for all x h (x ) =  2
(c) h′ (x) = 1, for all x x , 0 ≤ x ≤ 1
(d) h is not differentiable at two values of x which shows h ( x ) is continuous for all x. But not
differentiable at x = { 0, 1}.
Thus, h ( x ) is not differentiable at two values of x.
Hence, (d) is the correct answer.

Exercise for Session 4


1. Plot the following, where [⋅] denotes greatest integer function.
(i) f (x ) = [x 2 ], when −2 ≤ x ≤ 2 (ii) f (x ) = [|x |] (iii) f (x ) = [|x − 2 |] (iv) f (x ) = [|x | − 2]
(v) f (x ) = sin−1 (sin |x |) (vi) f (x ) = [cos−1 x ] (vii) f (x ) = cos(x − [x ]) (viii) f (x ) = [sin−1 (sin x )]

2. Plot the graph for f ( x ) = min ( x − [ x ], − x − [ − x ]).

3. Find the area enclosed by the curves


(i) max (| x |, | y | ) = 1 (ii) max (2 | x |, 2 | y | ) = 1 (iii) max (| x + y |, | x − y | ) = 1
JEE Type Solved Examples :
Single Option Correct Type Questions

l Ex. 1 The number of real solutions of the equation y = | x | and y = cos x shown as;
e + x = 0 is
x
Y
y = –x y=x
(a) 0 (b) 1
(c) 2 (d) None of these
Sol. (b) It is evident from figure that the curves y = e x and X
O
y = − x intersect exactly at one point. So, the equation y = cos x
e x = − x or e x + x = 0 has one real solution.
Y y = ex It is evident from the graph that the two curves intersect at
y= –x
two points only.
 π π
∴ Two real solutions lying in  − ,  .
O
X  2 2

l Ex. 4 How many roots does the following equation


y = –x
possess 3 | x | {| 2 − | x | | } = 1?
(a) 1 (b) 2 (c) 3 (d) 4
l Ex. 2 The number of real solutions of the equation
Sol. (b) Here, 3 |x|
{| 2 − | x || } = 1
log a x = | x | , 0 < a < 1, is
(a) 0 (b) 1 ⇒ 2 − | x | = 3− | x |
(c) 2 (d) None of these In order to determine the number of roots, it is sufficient to
find the points of intersection of the curves y = 2 − | x | and
Sol. (b) The number of real solutions of the equation
loga x = | x | is equal to the number of points of intersec- y = 3− | x | , shown as;
tion of the curves. Y
y = loga x and y = | x | (0 < a < 1)
2
Y y=x
y = –x |–x|
y=3
X
–2 0 2
0 X
1

y=2–|x|

y = loga x (0 < a < 1)


We observe the two curves intersect at two points.
It is evident from the graph that the two curves intersect at ∴ Two real solutions ∈ ( −2, 2).
one point only. l Ex. 5 The number of real solutions of the equation
∴ One real solution lying in (0, 1). x 2 = 1 − | x − 5 | is
l Ex. 3 The number of solutions of the equation | x | = cos x is (a) 1 (b) 2
(c) 4 (d) None of these
(a) 0 (b) 1
Sol. (d) The number of real solutions of the equation
(c) 2 (d) 3
x2 = 1 − | x − 5|
Sol. (c) The number of real solutions of the equation
| x | = cos x is equal to the number of point of intersection is equal to the number of points of intersection of the
of the curves. curves.
220 Textbook of Differential Calculus

y = x 2 and y = 1 − | x − 5 | , shown as; Y

(5, 1)
1 X
– 2π –π 0 π 2π
X′ X
0

Clearly, the two curves intersect at three points.


y=1–|x–5|
∴ There are three solutions.
Y′

It is evident from the graph that the two curves do not l Ex. 8 The equation 3 x − 1 + 5 x − 1 = 34 has
intersect. (a) one solution (b) two solutions
∴ No solution. (c) three solutions (d) four solutions
Sol. (a) The total number of solutions is same as the number
l Ex. 6 Number of solutions for 2 sin | x | = 4 | cos x | in [ −π, π ] of points of intersection of the curves.
−1 −1
is equal to y = 3x + 5x and y = 34
(a) 2 (b) 4 Y
(c) 6 (d) 8
y = 34
Sol. (b) The total number of solutions for the given equation is
equal to the number of points of intersection of curves
y = 4| cos x | and y = 2sin | x | . 1/15
X
Clearly, the two curves intersect at four points. So, there 0
are four solutions of the given equation.
It is evident that these two curves intersect at exactly one
4
point.
∴ Exactly one solution.
2
l Ex. 9 The number of solutions of the equation
–π –π 0 π π cos [ x ] = e 2 x − 1 , x ∈[0, 2π ], where [.] denotes the greatest
2 2
integer function is
(a) 1 (b) 2
(c) 3 (d) 4
Sol. (a) It is evident from the graph the two curves intersect at
l Ex. 7 Number of roots of | sin | x | | = x + | x | in [ −2 π, 2 π ] only one point.
is Y
(a) 2 (b) 3 y = e 2x – 1
(c) 4 (d) 6
Sol. (b) The total number of solutions for the given equation is
equal to the number of points of intersection of the
curves. 1 y = cos [x]

y = | sin | x ||
2x , x ≥ 0 0 X
and y = x +|x |=  –2 –1 1 2 3
0, x <0
Chap 04 Graphical Transformations 221

JEE Type Solved Examples :


More than One Correct Option Type Questions
l Ex. 10 Let g ( x ) = x − 2k , ∀ 2k ≤ x < 2(k + 1 ) where, Then, (a) Graph of − f ( x ) + 2 is
k ∈I , then
5
(a) g ( x ) = x + 2, − 2 ≤ x < 0
4
(b) g ( x ) = x − 2, 2 ≤ x < 4 3
(c) g ( x ) = x , 0 ≤ x < 2 2
(d) period of g ( x ) is 2 1
Sol. (a,b,c,d) Given, g ( x ) = x − 2k , ∀ 2k ≤ x < 2 (k + 1), ∀ k ∈ I
–6 –5 –4 –3 –2 –1 1 2 3 4 5 6
 ..... –1
 ..... –2

 x + 2, − 2 ≤ x < 0 (b) Graph of − f ( x − 1) is
 x, 0≤ x < 2
= 3
 x − 2, 2 ≤ x < 4
2
 x − 4, 4 ≤ x < 6
 1
.....

 ..... –7 –6 –5 –4 –3 –2 –1 1 2 3 4 5 6
–1
The graph of g ( x ) is shown below
–2
Y –3

(c) Graph of f (| x| ) is
√x+2 √x √x–2 √x–4
X′ X 3
–2 0 2 4 6
2

Y′ 1

–6 –5 –4 –3 –2 –1 1 2 3 4 5 6
Clearly, g ( x ) is periodic with period 2. –1

l Ex. 11 The graph of f ( x ) is given below. (d) Graph of f ( x + 1) − 2 is

3 2
2 1

1
–7 –6 –5 –4 –3 –2 –1 1 2 3 4 5 6
–1
–6 –5 –4 –3 –2 –1 1 2 3 4 5 6
–1 –2

–2 –3

–3 –4
–5
222 Textbook of Differential Calculus

Sol. (a,c,d) (c) For y = f (| x | ), neglect the graph of y = f ( x )


(a) For the graph of − f ( x ) + 2, first flip the graph of y = f ( x ) about, for x < 0, and take the mirror image of
X -axis to get y = − f ( x ) y = f ( x ) for x > 0 about Y -axis,
keeping y = f ( x ) for x > 0
4
3
3
2
2
1
1
–6 –5 –4 –3 –2 –1 1 2 3 4 5 6
–1 –6 –5 –4 –3 –2 –1 1 2 3 4 5 6
–1
–2
–3

Now, shift the above graph 2 units upward to get y = 2 − f ( x ) (d) For y = f ( x + 1) − 2, shift the graph of
y = f ( x ), 1 unit left to get y = f ( x + 1)
5
4 3
3 2
2 1
1
–7 –6 –5 –4 –3 –2 –1 1 2 3 4 5
–1
–6 –5 –4 –3 –2 –1 1 2 3 4 5 6
–1 –2
–2 –3

(b) First shift the graph of f ( x ), through 1 unit right to get


y = f ( x − 1) Now, shift the above graph 2 units
downwards to get f ( x + 1) − 2
3
2 2
1 1

–6 –5 –4 –3 –2 –1 1 2 3 4 5 6 7 –7 –6 –5 –4 –3 –2 –1 1 2 3 4 5 6
–1 –1
–2 –2
–3 –3
–4
Now, flip the above graph about X -axis to get y = − f ( x − 1 )
–5
3
2
1

–5 –4 –3 –2 –1 1 2 3 4 5 6 7
–1
–2
–3
Chap 04 Graphical Transformations 223

JEE Type Solved Examples :


Single Integer Answer Type Questions
l Ex. 12 The number of solutions of the equation The graph of y = [sin x ] and y = cos x is shown below.
1 Y
[y + [y ]] = 2 cos x , where y = [sin x + [sin x + [sin x ]]],
3 2
where [⋅] denotes the greatest integer function, is 1
Sol. Here, [y + [y ]] = 2 cos x X¢ X
O π/2 π 3π/2 2π
or [y ] + [y ] = 2 cos x –1
[Q[ x + h ] = [ x ] + h , ∀ x ∈ I ]

or 2[y ] = 2 cos x ∴ The number of solution zero.
or [y ] = cos x …(i) l Ex. 13 The sum of the roots of the equation cos −1
Also, y=
1
[sin x + [sin x + [sin x ]]] (cos x ) = [ x ], where [x] denotes greatest integer function, is
3 Sol. The graph of cos −1 (cos x ) = [ x ] is shown below.
1 Y
y= (3 [sin x ])
3 3

y = [sin x ] …(ii) 2
1
From Eqs. (i) and (ii), we have
X
0 1 2 3 p 2p–3 2p
[[sin x ]] = cos x –1
–1
or [sin x ] = cos x
Solutions are clearly 0, 1, 2, 3 and 2π − 3.
Hence, there are 5 solutions.

Subjective Type Questions


l Ex. 14 Sketch the graph of y = log 0. 5 | x |. 1
l Ex. 15 Sketch the graph for y = –3 .
Sol. As we know, y = log 0. 5 x is a decreasing graph given as; x
Sol. Here, we follow certain steps to plot
Y
1 1
y = log0.5(x), x > 0 y= – 3 , as first we plot
x x
1
0
X 1 1 1
and then successively , – 3, –3 .
x x x
1
(i) The graph of is shown below
x
Since, f ( x ) → f (| x | ), then taking the images Y
aboutY -axis for x > 0
∴ y = log 0. 5 | x | could be plotted as; 1
y= x

Y X
y = log0.5 |x| 0

–1 1
0 X
224 Textbook of Differential Calculus

1 1 1 As we know, x 2 + y 2 = 1 is a circle and y =|sin x | is the


(ii) → To draw the graph of taking mirror
x x x image of negative values of y = sin x about X -axis. Thus,
1 we can plot them as; which shows the two curves intersects
images about X -axis for negative values of in the at two points.
x
1 ∴ Number of solutions is 2.
graph of .
x Y
Y
x2 + y 2 = 1 y = |sin x|
1 1
y=x y=x
–π π X
0
0 X

l Ex. 17 Find the number of solutions of 4 { x } = x + [ x ] .


Sol. Here, 4 { x } = x + [x ]
⇒ 4 ( x –[ x ]) = x + [ x ] [Q { x } = x − [ x ]]
1 1 ⇒ 3x = 5 [ x ]
(iii) → –3
x x 3
⇒ [x ] = x
1 5
Shift the graph of through 3 units downward. To find their solution we plot the graph of both y = [ x ]
x 3
Y
and y = x ,
5

Y 3
y= x
3 5

2
X 1
–1 0 1
1 3 3 1
y= –3 y= –3 X
x x –4 –3 –2 –1 0 1 2 3 4
–1
x = –3
–2

1 1
(iv) –3 → –3
x x
i.e. The two graphs intersects when [ x ] = 0 and 1
1
To draw the graph of − 3 , taking mirror images 5
x ⇒ x = 0 and x =
3
1
about X -axis for negative value of − 3 in the graph
x 1
l Ex. 18 Sketch the graph of sin x + .
1 2
of − 3.
x 1
Sol. We follow certain steps to plot sin x + ,
Y
2
1 1
1
y= x –3
i.e. sin x → sin x + → sin x +
2 2
(i) (ii) (iii)
X
– 13 0 1
3
Y
y = sin x
1
X
l Ex. 16 Find the number of solutions of the equations –2π –π 0 π 2π
y =| sin x | and x 2 + y 2 = 1 .
Sol. To find the number of solutions of two curves we should (i)
find the point of intersection of two curves.
Chap 04 Graphical Transformations 225

3
Y l Ex. 20 Sketch the region for y = sin ( x – [ x ]) .
2
Sol. We know, y = sin x is the periodic function. So, to plot
y = sin x + 1 the graph between x ∈[0, 1) and repeat for all values of x.
1 2
2 Y
X
0 1

2 1
(ii)

Y π X
y = sin x + 1 0 π

3/2 2 2
y = 3/2
y=1
1/2 i.e. To retain the graph between x ∈[0, 1).
y = 1/2
Y
X
0
(iii) sin (x–[x])

X
Remarks –2 –1 0 1 2 3 4
Plotting graph of f (x – [x ]):
Graph of f (x – [x ])can be obtained from the graph of f (x )by the
following rule. l Ex. 21 Sketch the region for | y | = sin x .
“Retain the graph of f (x )for values of x lying between interval
[0, 1). Now, it can be repeated for rest of points. Now, Sol. We know, y = sin x is a periodic function. So, to plot
obtained function is periodic with period 1.” | y | = sin x neglect all the points for which y is negative
and take the image for positive values of y about X-axis
2x shown as;
l Ex. 19 Sketch the graph of y = ⋅ Y
2 [x ]
1 y = sin x
Y x
y=2 X
–3 π –2π – π 0 π 2π 3π
–1
neglecting
2

1 Now, plotting, | y | = sin x .


X
0 1 Y

1
x
Sol. As we know, 2 is exponential function and we want to –4π –3π –2π –π 0 π 3π 4π 5π
X

transform it to 2x –[ x ], it retains, the graph for x ∈[0, 1) –1
and repeat for rest points.
Here, to retain graph between x ∈[0, 1), so we get
Y
l Ex. 22 Consider the following function f whose graph is
given below.
2 2x Y
y =—
2[x]
1 1

X X
–2 –1 0 1 2 3 –2 –1 0 1 2 3
–1
226 Textbook of Differential Calculus

Draw the graph of following functions. (e) f ( x ) → f (–x )


(a) f ( x ) + 1 (b) f ( x )–1 (c) – f ( x ) To draw the graph of f ( − x ) take the image of the
(d) | f ( x )| (e) f (–x ) (f) f (| x | ) curve f ( x ) in the Y -axis as plane mirror.
Y
(g) 2f ( x ) (h) f (2x ) (i) [ f ( x )]
(j) f ( x –[ x ]) 1
Sol. (a) f ( x ) → f ( x ) + 1 f (–x) f (x)
X
To draw the graph of f ( x ) + 1, shift the graph of f ( x ) –3 –2 –1 0 1 2 3
through 1 unit upward.
–1
Y
2 (f) f ( x ) → f (| x | )
f (x) + 1 To draw the graph of f | x | neglecting the graph for
1
negative values of f ( x ) and taking image for positive
f (x)
values of x about Y-axis.
X Y
–2 –1 0 1 2 3

–1 1
f(x)

0 X
–3 –2 –1 1 2 3
(b) f ( x ) → f ( x )– 1
To draw the graph of f ( x ) − 1, shift the graph of f ( x ) –1
through 1 unit downward.
(g) f ( x ) → 2 f ( x )
Y
To draw the graph of 2f ( x ) stretch the graph of f ( x ),
1 2 times along Y -axis.
f (x) Y
–2 –1
X 2
0 1 2 3 f (x) – 1
2 f (x)
–1 1
f (x)
–2 –2 –1 X
0 1 2 3
(c) f ( x ) → – f ( x )
–1
To draw the graph of − f ( x ). Take the image of f ( x )
in the X -axis as plane mirror. –2

Y
(h) f ( x ) → f (2x )
To draw the graph of f (2x ) , stretch the graph of f ( x )
1
f(x)
, 2 times along X-axis.
–2 –1 Y
0 X
1 2 3 – f(x)
–1 f (2x)
1
f (x)
X
(d) f ( x ) → | f ( x )|, To draw the graph of f ( x ) taking –2 –1 0 1 2
image for negative values of f ( x ) about X-axis. –1
Y
(i) f ( x ) → [ f ( x )]
1 To draw [ f ( x )] mark the integer on Y -axis. Draw the
|f (x)| horizontal lines till they intersect the graph, now
–2 –1
0 X draw vertical dotted lines from these intersection
1 2 3
f (x) point. Finally, draw horizontal lines parallel to X -axis
–1 from any intersection point to the nearest vertical
dotted line with blank dot at right end.
Chap 04 Graphical Transformations 227

Y As from the above graph,


 3π
sin x , –π < x ≤ –
 4
 3π π
 cos x , – ≤x≤
 4 4
–2 –1 max (sin x , cos x ) = 
X π 5π
0 1 2 3  sin x , ≤x≤
[f(x)]
 4 4
–1  5π 3π
 cos x , ≤x≤
 4 2
( j) f ( x ) → f ( x –[ x ])
Retain the graph of f ( x ) for values of x lying between
l Ex. 24 Sketch the graph for y = min {tan x , cot x }.
interval [0,1). Now, it can be repeated for rest of the Sol. First plot both f ( x ) = tan x and g ( x ) = cot x by a dotted
point taking periodicity ‘1’. curve as can be seen from the graph and then darken
Y those dotted lines for which f ( x ) < g ( x ) and g ( x ) < f ( x ).
Y
1
f (x – [x])
X
–2 –1 0 1 2 3
1
–1
–3π/2 –5π/4 –π/2 0 π/2 π 3π/2
–π
X
–3π/4 –π/4 π/4 3π/4 5π/4
l Ex. 23 Sketch the graph of y = max (sin x , cos x ), –1

 3π 
∀ x ∈  – π,  .
 2 
Sol. First plot both y = sin x and y = cos x by a dotted curve as
 3π 
can be seen from the graph in the interval –π,  and
 2 ∴ Graph of min {tan x , cot x }
then darken those dotted line for which f ( x ) > g ( x ) or Y
g ( x ) > f ( x ).
Y
1

1
π π π X
3π –π
– O 3π
cos x B – 2 2
sin x 2 2
–1
π π π X
–π
– O C 3π
2 2 2
A

–1 As from the graph, we have

∴ Graph of max {sin x , cos x }  ............................


 ............................
Y 
 tan x , – π < x ≤ – π
 2 4
y = max (sin x, cosx)  π
B  cot x , − ≤ x < 0
min {tan x , cot x } =  4
X π
–π
–π O π π 3π  tan x , 0 ≤ x ≤
2 2 2  4
A C  cot x , π ≤ x ≤ π
 4 2
 ............................

 ............................
228 Textbook of Differential Calculus

l Ex. 25 Sketch the graph of y = min {| x |, | x – 1 |, | x + 1 | }. Y Y


y=x
Sol. First plot the graph for, y = | x |, y = | x – 1| and y = | x + 1| by π π
cos– 1 x
a dotted curve as can be seen from the graph and then
darken those dotted lines for | x | < {| x – 1|, | x + 1| }, π/2 π/2
| x – 1| < {| x |, | x + 1| } and | x + 1| < {| x |, | x – 1| }. 1

X′ π X X′ X
1| | 1| –1 0 1 π/2 –1 0 1
Y
x + x | x–
| | –1 cos x
y= y
= y=
Y′ Y′ –1
y=cos x

1/2
1 01 1 X l Ex. 28 Sketch the graph for y = tan –1 x , ∀ x ∈ R.
–3 –2 –1 – — 2 3
2 —2 –π π –π π
Sol. As, x ∈ R ⇒ < tan –1 x < ⇒ <y<
2 2 2 2
As from the above graph, –π π
 – ( x + 1), x ≤ –1 ∴ tan –1 x is the reflection of the graph of tan x, <x<
2 2
 1
in y = x as a mirror.
 ( x + 1), – 1 ≤ x ≤ – 2
 1 y = tan x
 – ( x ), – ≤ x ≤0 Y
min {| x |, | x – 1|, | x + 1| } =  2 y=x
1
 x, 0≤ x ≤ π/2
 2 y = tan–1x
 – ( x – 1), 1 ≤ x ≤ 1
 2
 ( x – 1), 1≤ x –π/2 π/2
X

Remark
–π/2
To plot the graph of f –1( x), take reflection of f ( x) in y = x line as a
mirror.

l Ex. 26 Sketch the graph of y = sin –1 x , ∀ x ∈[–1, 1 ].


π π π π
Sol. As, – 1 ≤ x ≤ 1 ⇒ – ≤ sin –1 x ≤ ⇒ – ≤y≤ l Ex. 29 Sketch the graph for y = sin –1 (sin x ) .
2 2 2 2
sin –1 x is the reflection of sin x about y = x (as a mirror) Sol. As, y = sin –1(sin x ) is periodic with period 2π.
 π π ∴ To draw this graph we should draw the graph for one
when x ∈ – , .
 2 2 interval of length 2π and repeat it for entire values of x. As we
know,
x
sin –1

Y
 –π π
π y=x ( x ), ≤x≤

y=

2 2 2
 π 3π π 3π
1
f(x) = 
sin x sin (sin x ) = ( π − x ),
–1
≤ x ≤ or π – ≥ π – x ≥ π –
(–π) (–π/2,0) (–1,0) (1,0)(π/2,0) (π/0) 2 2 2 2
X′ X 
O  π π
or – ≤ π – x ≤
 2 2
–1
π –π 3π

2 Thus, it has been defined for ≤ x ≤ , that has length
2 2
Y′
2π. So, its graph could be plotted as;
l Ex. 27 Sketch the graph for y = cos –1 x , ∀ x ∈[–1, 1 ]. Y
π/2
Sol. As, –1 ≤ x ≤ 1 y = sin–1(sin x)
⇒ 0 ≤ cos x ≤ π
–1
⇒ 0≤ y ≤ π π
X
–2π –3π/2 –π –π/2 0 π/2 3π/2 2π 5π/2 3π
∴ cos –1 x is reflection of the graph of cos x , 0 ≤ x ≤ π in –π/2
y = x line as a mirror.
Chap 04 Graphical Transformations 229

l Ex. 30 Sketch the graph for y = cos –1 (cos x ) . Thus, to find the points for which f ( x ) = x 2 is less than or
equal to g ( x ) = | x – 1|.
Sol. As, we have that y = cos –1(cos x ) is periodic with
period 2π. Where the two functions f ( x ) and g ( x ) could be plotted as;
Y f(x) = x2
∴ To draw this graph we should draw the graph for one A g(x) = |x–1|
interval of length 2π and repeat it for entire values of x of
length 2π.
As we know, 1 B
x , 0≤ x ≤ π
cos –1(cos x ) =  X
(2π – x ), π < x ≤ 2π or 0 < 2π – x < π –1–√5 0 1
–1+√5
2
Thus, it has been defined for 0 < x < 2π, that has length 2π. 2
So, its graph could be plotted as;
Thus, from the graph f ( x ) ≤ g ( x ), when x ∈[ A , B ]. So that,
Y A and B are point of intersection of x 2 and 1– x.
π –1 – 5
∴ Solving x 2 = 1 – x , we get x = =A
2
y=cos–1(cos x) –1 + 5
and x= = B.
X 2
–2π –π 0 π 2π 3π 4π
x2 – 1 – 5 – 1 + 5 
∴ ≤ 1 is satisfied, ∀ x ∈  , .
x –1  2 2 

l Ex. 33 Find the values of x graphically satisfying


l Ex. 31 Sketch the graph for y = tan –1 (tan x ) .
[ x ] – 1 + x 2 ≥ 0 ; where [⋅] denotes the greatest integer
Sol. As, we have that y = tan –1(tan x ) is periodic with period π.
function.
∴ To draw this graph we should draw the graph for one
Sol. We have, [ x ]– 1 + x 2 ≥ 0 ⇒ x 2 – 1 ≥ – [ x ]
interval of length π and repeat for entire values of x.
Thus, to find the points for which f ( x ) = x 2 – 1 is greater
 –π π
As we know, tan –1(tan x ) =  x , <x< than or equal to g ( x ) = – [ x ].
 2 2
Where the two functions f ( x ) and g ( x ) could be plotted as;
–π π Y
Thus, it has been defined for < x < that has length π. f (x) = x2 – 1
2 2
So, its graph could be plotted as; 3
Y
2
–1 A
π/2 y = tan (tan x)
1

X
–2π –3π/2 –π –π/2 0 π/2 π 3π/2 2π 5π/2 X
–4 –3 –2 – √3 –1 1 2
–π/2
g (x) = –[x]
B
Finally.
Y
l Ex. 32 Find the values of x graphically which satisfy
x2
≤ 1.
x –1
2
x2
Sol. We have, ≤1 – Ö3 0 1
X
x –1
⇒ | x 2 | ≤ | x – 1|, ∀ x ∈ R – { 1 }
or x 2 ≤ | x – 1|, ∀ x ∈ R – { 1 }
230 Textbook of Differential Calculus

Thus, from the above graph f ( x ) ≥ g ( x ) when


x ∈ (–∞, A ] ∪ [ B, ∞ ), where A is point of intersection of
x 2 – 1 and – [ x ] when – [ x ] = + 2.
f (x) = 2 cos x
∴ x 2 – 1 = + 2, i.e. x = – 3 = A and B = 1 2

∴ [ x ]– 1 + x 2 ≥ 0 is satisfied, ∀ x ∈ (–∞, – 3 ] ∪ [1, ∞ ). 1 g (x) = |sin x|

l Ex. 34 Find the values of x graphically which satisfy;


0 π/2 π 3π/2 2π
–1 ≤ [ x ] – x 2 + 4 ≤ 2, where [.] denotes the greatest integer
–1
function.
Sol. We have, –1 ≤ [ x ]– x 2 + 4 ≤ 2 ⇒ x 2 – 5 ≤ [x ] ≤ x 2 – 2 –2

Thus, to find the points for which f ( x ) = x 2 – 5 is less than


or equal to g ( x ) = [ x ] and g ( x ) = [ x ] is less than or equal to Thus, from the above graph the two functions intersect at
two points between [0, 2π ] and we know that cos x is
h ( x ) = x 2 – 2.
periodic with period 2π, so it has same number of solutions
Where the three functions f ( x ), g ( x ) and h ( x ) could be for the interval [2π, 4 π ].
plotted as; ∴ Total number of solutions of 2cos x = |sin x | when
x2 – 5 x ∈[0, 4π ] is 4.
4
l Ex. 36 Sketch the curves
3
x2 – 2 (i) y = x – [ x ] (ii) y = [ x ] + x – [ x ]
(iii) y = |[ x ] + x – [ x ]|
2
(where [⋅] denotes the greatest integer function).
1 Sol. (i) Here, 0 ≤ x – [ x ] < 1,∀ x ∈ R and
–√2 √2 2 √7
x 2 ≤ x ≤ x , ∀x ∈ [0,1)
–3 –√5 –2 –1 1 √3 √5
3 ∴ x – [x ] ≤ x – [x ]
–1  x – 1, 1 ≤ x < 2

⇒ y = x – [x ] =  x, 0≤ x < 1
–2
 x + 1, – 1 ≤ x < 0

–3
and so on.
–4 In general,y = x – [ x ] = x , when 0 ≤ x < 1
y = x – [ x ] = x – 1, when 1 ≤ x < 2
–5
i.e. Shifting x by 1 unit on right side of X -axis
y = x – [ x ] = x – 2, when 2 ≤ x < 3,
Thus, from the above graph; x 2 – 5 ≤ [ x ] ≤ x 2 – 2, when
x ∈ [ A , B ] ∪ [C , D ]. then graph of y = x – [ x ] .
Where A and D is point of intersection of x 2 – 5 = ± 2 i.e. Shifting x – 1 by 1 unit on right side of X -axis
⇒ x = – 3, 7 B and C is point of intersection of x 2 –2 = ± 1 and so on. Thus, the graph for y = x – [ x ] is shown
⇒x = 3
as in figure, which is periodic with period 1.
∴ –1 ≤ [ x ]– x 2 + 4 ≤ 2 is satisfied, x ∈ [– 3, – 1] ∪ [ 3, 7 ].
Y

lEx. 35 Find the number of solutions of 2 cos x = | sin x |


when x ∈[0, 4π ] .
Sol. As we know, the graph of both f ( x ) = 2 cos x and
1
2

3
2
x+

x
x+

x–

x–
x–

g ( x ) = |sin x | X
–2 –1 0 1 2 3 4
∴ Their point of intersection are number of solutions.
Chap 04 Graphical Transformations 231

(ii) Again, y = [ x ] + x – [ x ] ⇒ y = k + x – k , (iii) Graph for y = |[ x ] + x – [ x ]| is obtained by


k ≤ x < k + 1 ; k ∈ integer −1 + x + 1,−1 ≤ x < 0 reflecting the portion lying below X -axis of the graph
 x, 0≤ x < 1 of y = [ x ] + x – [ x ]. About X -axis and keeping the

⇒ y =  1 + x – 1, 1 ≤ x < 2 portion lying above the X -axis (as it is).
 Thus, the graph for y = [ x ] + x − [ x ] is shown
2 + x – 2, 2 ≤ x < 3 and so on.
below.
Thus, the graph for y = [ x ] + x − [ x ] is obtained by
Y
the graph of y = x − [ x ] by translating it by [ x ] units
4
in upward or downward directions according as
[ x ] > 0 or [ x ] < 0. The graph is shown as 3
Y
2
4
x–3
3+ 1
3
x–2
2+ –3 –2 –1 0 1 2 3
X
2
x–1
1+
1
x
X
–1 10 1 2 3
x+
+
–1
232 Textbook of Differential Calculus

Graphical Transformations Exercise 1 :


Single Option Correct Type Questions
1. The number of real solutions of the equation 5. The equation e x = m (m + 1), m < − 1 has
e | x | − | x | = 0 is (a) no real root
(a) 0 (b) 1 (c) 2 (d) None of these (b) exactly one real root
(c) two real roots
2. The number of real solutions of the equation
(d) None of the above
3 − | x | − 2 | x | = 0 is
6. The number of real solutions of the equation
(a) 0 (b) 1 (c) 2 (d) 3 1 − x = [cos x ] is
3. The number of solutions of 3 | x | = | 2 − | x | | is (a) 1 (b) 2
(c) 3 (d) 4
(a) 0 (b) 2 (c) 4 (d) infinite
7. The number of roots of the equation 1 + 3 x / 2 = 2 x is
4. The total number of solutions of the equation
(a) 0
| x − x 2 − 1 | = | 2x − 3 − x 2 | is (b) 1
(a) 0 (b) 1 (c) 2
(c) 2 (d) infinitly many (d) None of the above

Graphical Transformations Exercise 2 :


More than One Option Correct Type Questions
8. The equation x 2 − 2 = [sin x ], where [⋅] denotes the (a) x 2, − 2 ≤ x ≤ − 1
greatest integer function, has 1
(b) 1 − x, − 1 < x ≤ −
(a) infinity many roots 4
1 1
(b) exactly one integer root (c) + x, − < x < 0
2 4
(c) exactly one irrational root
(d) 1 + x, 0 ≤ x < 1
(d) exactly two roots
 1 10. If f ( x ) is defined on [− 2, 2] and is given by
9. Consider the functionf ( x ) =  x − [x ] − 2 , if x ∉ I , f (x ) = 
 − 1, − 2 ≤ x < 0
and g ( x ) = f | x | + | f ( x )| , then
 0, if x ∈ I  x − 1, 0 < x ≤ 2
where [⋅] denotes greatest integer function and I is the g ( x ) is defined as
set of integers, then g ( x ) = max{x 2 , f ( x ), | x | }, − 2 ≤ x ≤ 2 (a) − x, − 2 ≤ x ≤ 0 (b) x, − 2 ≤ x ≤ 0
is defined as (c) 0, 0 < x ≤ 1 (d) 2( x − 1 ), 1 < x ≤ 2

Graphical Transformations Exercise 3 :


Statements I and II Type Questions
n
Directions (Q. Nos. 11 to 12) This section is based on 11. Statement I The graph of y = sec 2 x is symmetrical
Statement I and Statement II. Select the correct answer about the Y -axis.
from the code given below.
Statement II The graph of y = tan x is symmetrical
(a) Both Statement I and Statement II are correct and Statement
about the origin.
II is the correct explanation of Statement I
12. Statement I The equation |( x − 2) + a | = 4 can have four
(b) Both Statement I and Statement II are correct but Statement II
distinct real solutions for x if a belongs to the interval
is not the correct explanation of Statement I
( − ∞, − 4 ).
(c) Statement I is correct but Statement II is incorrect
Statement II The number of point of intersection of the
(d) Statement II is correct but Statement I is incorrect curve represent the solution of the equation.
Chap 04 Graphical Transformations 233

Graphical Transformations Exercise 4 :


Passage Based Questions
Passage I (Q. Nos. 13 to 14) Passage II (Q. Nos. 15 to 16)
Let f ( x ) = f 1 ( x ) − 2 f 2 ( x ), where  x − 1, − 1 ≤ x ≤ 0
Consider the functions f ( x ) =  2
 min {x 2 , | x | }, | x | ≤ 1  x , 0≤ x ≤ 1
f 1( x ) = 
 max {x , | x | }, | x | > 1
2
and g ( x ) = sin x .
 min {x 2 , | x | }, | x | > 1 If h 1 ( x ) = f (| g ( x )| )
f 2 (x ) =  ,
 max {x , | x | }, | x | ≤ 1
2
and h 2 ( x ) = | f ( g ( x ))|.
 min{ f (t ), − 3 ≤ t ≤ x , − 3 ≤ x < 0} 15. Which of the following is not true about h 1 ( x )?
and let g ( x ) = 
 max{ f (t ), 0 ≤ t < x , 0 ≤ x ≤ 3}. (a) It is a periodic function with period π
(b) The range is [0, 1]
13. For x ∈ ( − 1, 0), f ( x ) + g ( x ) is
(c) Domain R
(a) x 2 − 2 x + 1 (b) x 2 + 2 x − 1
(d) None of the above
(c) x + 2 x + 1
2
(d) x 2 − 2 x − 1
16. Which of the following is not true about h 2 ( x )?
14. The graph of y = g ( x ) in its domain is broken at (a) The domain is R (b) It is periodic with period 2π
(a) 1 point (b) 2 points (c) 3 points (d) None of these (c) The range is [0, 1] (d) None of these

Graphical Transformations Exercise 5 :


Matching Type Questions
n
Directions (Q.No. 17) Choices for the correct combination of elements from Column I and Column II are given as option
(a), (b), (c) and (d) out of which are correct.

17. Column I Column II


(Equation) (Number of Roots)
A. x 2 tan x = 1, x ∈[ 0, 2π ] p. 5
B. 2 cos x = |sin x|, x ∈[ 0, 2π ] q. 2
C. If f ( x ) is a polynomial of degree 5 with real coefficient such that r. 3
f ( x ) = 0 has 8 real roots, then the number of roots of f ( x ) = 0
D. 7| x| (5 − | x| ) = 1 s. 4

Codes
A B C D A B C D
(a) p q r s (b) q s p s
(c) q p s r (d) s p q r
234 Textbook of Differential Calculus

Graphical Transformations Exercise 6 :


Single Integer Answer Type Questions
18. Let f ( x ) = x + 2 | x + 1 | + 2 | x − 1|. If f ( x ) = k has exactly one real solution, then the value of k is
19. The number of roots of the equation x sin x = 1, x ∈ [− 2π, 0] ∪ (0, 2π ) is
π π
20. The number of solutions of tan x − mx = 0, m > 1 in  − ,  is
 2 2

Graphical Transformations Exercise 7 :


Subjective Type Questions
21. Find the number of solutions of the equation 27. Find a formula for the function f graphed as
x2
= 1, graphically.
1 − | x − 2| Y

22. Find the number of solutions for tan 4x = cos x , when (1, 1)
1
x ∈(0, π ).
23. Find number of solutions for equation [sin −1 x ] = x − [x ], 0 1 2 3 4
X
where [ ⋅ ] denotes the greatest integer function.
24. If x and y satisfy the equations 1
28. Find the domain for f ( x ) = ,
max (| x +y|,| x −y| ) = 1 and | y| = x −[x ] , the number of [ x −1 ]+[ 5− x ]− 4
ordered pairs ( x , y ). graphically.
25. Find the area enclosed by | x + y − 1 |+|2x + y + 1| = 1. 29. Draw the graph for y = {x } and y = {x }.
26. Find f ( x ) when it is given by
30. Draw the graph for y = − [x ] + {x }.
 1
f ( x ) = max  x 3 , x 2 ,  , ∀x ∈ [0, ∞ ).
 64 
Answers
Exercise for Session 1 5. Y
1.
2 y=sin x+1
Y
1
y=x2+1 X
5 –π 0 π 2π 3π
4 y=x2 –1 y=sin x
3
2
1
6. Y

X′ X 2
–7 –6 –5 –4 –3 –2 –1 0 1 2 3 4 5 6 7 1
–1 y=sin x
0 X
–2
–1 y=sin x–1
–3
–4 –2
–5
7. Y
Y′ y=loge x+1
y=log x
2. Y
5 y=x2 –1
X
0 1
4
3
2
1 8. Y
y=loge x
X′ X
–5 –4 –3 –2 –1 0 1 2 3 4 5
–1 X
0 1
–2
–3 y=loge x–1

–4
–5
Y′
Exercise for Session 2
3. Y y=x3+1 1. (i) f (x + 1) is shown as
y=x3 Y
1
1 1
X X
0 –3 –2 –1 0 1 2
–1

4. (ii) f (x − 1) is shown as
Y y=x3
Y
y=x3–1

X 1
1 0 X
–1 –2 –1 0 1 2 3
–1
236 Textbook of Differential Calculus

(iii) − f (x) is shown as Exercise for Session 3


Y
1. Graph for f (x)
1 Y
1 2 2
X
–2 –1 0
–1
1

0
(iv) f (− x) is shown as –1 –1 1 2 X
— 1
2 —
Y 2
–1
1
(i) | f (x)| is shown as
X Y
–2 –1 0 1 2
–1 2

(v) 2 f (x) is shown as 1


Y
X
2 –1 –1

0 1 1

2
2 2
1
X
–1 0 1 2 (ii) f (| x |) is shown as
–1 Y
–2 2

1
1
(vi) f (x) is shown as X
2 –2 –1 O 1/2 1 2
Y –1

–2
1
1/2 (iii) Y Y
X 1 1
–2 –1 0 1 2
–1/2
0 X X
–2 –1 1 2 –2 –1 0 1 2
–1
–1
Graph for f (| x |) − 1 Graph for | f (| x |) − 1 |
(vii) f (2x) is shown as
Y 2. (i) y = | x2 − 2x − 3| = |(x − 3)(x + 1)|
1 Y
X
–1 –1/2 0 1/2 1
–1
X
–1 0 3

(viii) f   is shown as
x
 2 (ii) y = (x + 1)(x − 3 ) = x2 − 2x − 3, y = x2 − 2| x|−3
Y
Y

1
X X
–4 –3 –2 –1 0 1 2 3 4 –3 0 3
–1

–3
Chap 04 Graphical Transformations 237

(iii) y =|log2 x| (x) y = cos | x |


Y Y

1
–π/2 π/2
X
–3π/2 0 3π/2
1 X
0
y = log 2 x

(iv) y =| log2 | x|| 3. (i) | f (x)| = log 2 x


Y Y

X
X 0 1
–1 0 1

(v) y = log2 (1 − x) ⇒ y = log2 |1 − x |


(ii) | f (x)| = log2 (− x)
Y
Y

0 X
1
X
–1 0

(vi) y = log2 (2 − x)2= 2 log2 | 2 − x |


4. sin πx = |log e | x||
Y
Y

y= loge x

X
0 1 2 3 X
–3 –2 –1 0 1 2 3 4 5
y=sin px

∴ Number of solutions is 6.
5. (i) 2 | x | = sin x2
(vii) y =|cos| x ||
Y
1 Y
|x|
2 =y

π π 3π X
–3π –— 0 — —
— 2 2 2 1
2
X
–√π 0 √π

(viii) y = 2 − 2x ⇒ y = |2 − 2x |
Y
2 ∴No solution
1 (ii) sin x = x2 + x + 1
0 X
1
Y
(ix) y = sin| x |
y=x2+x+1
Y

1 1
y=sin x
X X
–2π –π 0 π 2π 0
–1 –1

∴No solution
238 Textbook of Differential Calculus

Exercise for Session 4 (v) y = sin −1 (sin | x|)


1. (i) Here, f (x) = [ x2 ]

Y
π
y=
2
3

2 y=0
3π –π – π

π
π 3π
2 2 2 2
1 π
y =–
X′ X 2

√3 – √2 –1 O 1 √2 √3

(vi) y = [cos−1 x]
Y
Y′
π
(ii) Here, f (x) = [| x|] –2
Y
1

X′ X
–1 O 1

X′ X
O

Y′
Y′ (vii) f (x) = cos (x − [ x])

(iii) f (x) = [(x − 2)] Y


y = cos (x –[x])
Y

X′ X
2 –π O 1 π2
2 2
1
y = cos x
X′ X Y′
O 1 2

(viii) f (x) = [sin −1 (sin x)]

Y
Y′
π
(iv) f (x) = [| x|−2] y=2
Y 1
X′ X
–π –π
– 3π –π π 3π
0
2 2 2 2
π
X′ X y =–2
O

Y′

Y′
Chap 04 Graphical Transformations 239

2. Here, f (x)= min(x – [x],–x–[–x]) 6. (b) 7. (b) 8. (b, c, d) 9. (a, b, c, d)


Y 10. (a, c, d) 11. (a) 12. (a) 13. (b)
–2 14. (a) 15. (d) 16. (c) 17. (b) 18. (3)
19. (4) 20. (3) 21. 0 22. 5 23. 1
1  1 , 0 < x≤ 1
 64 8
X′ X 
24. 0 25. 4 sq units 26. f (x) =  2 1
–5 –4 –3 –2 –1 1 2 3 4 x , < x≤ 1
–1  8

–2  x3 , x > 1
Y′ x, 0≤ x≤ 1

3. (i) 4 sq units (ii) 1 sq unit (iii) 2 sq units 27. f (x) = 2 − x, 1 < x ≤ 2
 0, x> 2

Chapter Exercises
28. D = R / {(0,1], 2, 3, 4, [ 5, 6}.
1. (a) 2. (b) 3. (b) 4. (b) 5. (b)

= 2− x
x
3. Given, 3

Solutions The graph of y = 3 x


and y = 2 − x
Y 3
x
is shown below

2– x

− x =0 ⇒e = x
x x
1. Given, e X′ X
–2 O 2
Now, the number of solution will be the point of intersection
of y = e and y = x .
x
Y′
Both of the curves shown below Clearly, both the curve intersect of two points.
e–x ex ∴ Number of solutions is 2.
Y
4. Given, x − x 2 − 1 = 2x − 3 − x 2
y=x
Let f ( x ) = x − x 2 − 1 and g( x ) = 2 x − 3 − x 2
)
(0 ,1
First we draw the graph of f ( x ) and g ( x )
X′ X
(0, 0) then, transforms it into | f ( x )| and | g( x )|.
2
 1  3
Here, f ( x ) = x − x 2 − 1 = y ⇒  x −  = − y + 
 2  4
Y′  1 3
Clearly, it represents a parabola having vertex  − , −  .
Clearly, from the graph we see that the both curves do not  2 4
intersect. Also, g( x ) = 2 x − 3 − x 2 = y ⇒ ( x − 1 ) 2 = − (y + 2 )
∴ No solution.
It also represent a parabola having vertex (1, − 2 ).
2. Given, 3 − x
−2
x
=0 ⇒3
− x
=2
x

− x
The graph of both curve is given below
Now, we plot the graph y = 3 Y
and y = 2
x
as shown below 3

Y x 2
2
1
X′ X
–3 –2 –1 1 2 3
–1
X′ X 1 , –3
O –x 2 4
3 x – x2–1 –2
(1, –2)

–3 2x– 3– x2

Y′
Y′
Clearly, the two curves intersect at one points. Clearly, both the curves intersect at only one point.
Hence, the number of solutions is 1. ∴ Number of solutions is 1.
240 Textbook of Differential Calculus

5. Given, e x = m (m + 1) Clearly, the curve y = x 2 − 2 intersect the curves y = [sin x ] at


two points ( 2, 0 ) and ( − 1, − 1 ).
ex
Y ∴ Therefore, the solution has exactly one integer root and one
irrational root.
 1
X′ X 9. Given, f ( x ) = x − [ x ] − 2 , x ∉ I
 0, x ∈I
Y′ and g( x ) = max{ x , f ( x ), | x| }
2

Clearly, for m < − 1, it will represent a straight line parallel to First we draw the graph of x 2, f ( x ) and | x| as shown below
X -axis which intersect the e x at only one point. Y
x2 x
Hence, exactly one real root.
6. Given, 1 − x = [cos x ]
The graph of both the curves is given below
Y
y =1–x f(x)
X′ 1 2 X
–3 –2 –1 O 3
1
y =–
2

X′ X
3π π π 3π 2π
–2π – 2 – 2 2 2
Y′

Clearly, from the graph


Y′
 x 2, −2 ≤ x ≤ −1
y = 1 − x intersect the graph of y = [cos x ] at exactly two  1
points.  1 − x, − 1 < x ≤ −
 4
∴ Number of solutions is 2. g( x ) = 1 + x, − 1 < x < 0
7. Given, 1 + 3 x /2 = 2 x 2 4
The graph is shown below 1 + x, 0 ≤ x <1
 2
x/2
3 +1  x , 1 ≤ x ≤2
2x
Y  −1 −2 ≤ x < 0
10. f ( x ) = 
) x − 1, 0 < x ≤ 2
(0, 2
g( x ) = f (| x| ) + | f ( x )|
(0, 1) The graph of f ( x ) is shown below
X′ X Y
O
1
Y′ X′ X
–2 –1 0 1 2
Clearly, both the curves intersect at one point. y = –1
–1
8. Given, x 2 − 2 = [sin x ]
The graph of two curves is given below Y′
Y ∴ g( x ) = | f ( x )| + f (| x| ) is shown below
Y
1

X′ X
–2π
O π π 3π 2π X′
0 1
X
–2 –1 2
2 2
–1

(0, –2)

Y′ Y′
Chap 04 Graphical Transformations 241

Clearly, from the graph  f ( x ), − 3 ≤ x < − 1


 − x, −2 ≤ x ≤ 0  − 1, −1 ≤ x < 0

 Now, g( x ) = 
g( x ) =  0, 0 < x <1 0 ≤ x ≤2
 0,
2( x − 1 ) 1 < x ≤ 2  f ( x ), 2 < x ≤3

11. We have the graph of y = tan x as shown below The graph of g( x ) is shown below
Y
y = tan x

X′ –2 –1 X

–π O π
2 2 –1
Y′
x 2 + 2 x , − 3 ≤ x < − 1

 − 1, −1 ≤ x < 0
Clearly, from the graph it is symmetrical about the origin. g( x ) = 
 0, 0 ≤ x ≤2
Also, derivative of an odd function is an even function and
 x 2 − 2 x, 2 < x <3
sec 2 x is derivative of tan x.
Hence, both the statements are true and Statement II is a 13. For x ∈ ( − 1, 0), f ( x ) + g( x ) = x 2 + 2x − 1
correct explanation of Statement I. 14. Obviously, the graph is broken at x = 0, i.e. in one point.
12. We know that, the number of point of intersection of two 15. Clearly, h1( x ) = f (| g( x )| ) = sin 2 x has period π, range [ 0, 1] and
curve is the real solutions of the equation involving these
functions. domain R.
Also, || x − 2| + a| = 4 16. h2( x ) = | f ( g( x ))|
⇒ | x − 2| = ± 4 − a The graph for h2( x ) is
Y x–2 Y

y = f(x)

X
X′ O 2

X′ X
O π π 2π 3π
Y′ 2
For four real roots 4 − a > 0 and − 4 − a > 0
Y′
⇒ a ∈ ( − ∞, − 4 )
It is clear from the graph that the function is periodic with
Both Statement I and Statement II are correct and Statement II
period 2π and has range [0, 2].
is the correct explanation of Statement I
1
Sol. (Q.Nos. 13 to 14) 17. Let y = tan x = 2
x
Here, f1( x ) = x 2 1
We plot the graph for tan x and 2 as shown below
f 2( x ) = | x | x
or f ( x ) = f1( x ) − 2 f 2( x )
Y
= x 2 − 2| x|
2
Graph for f ( x ) 0.5
Y 1
f(x) = 1/x2
0.5
X′ π 3π/2 X
π/2 2π
–0.5
–1
O –1.5
X′ X
–2 –1 1 2 –2
Y′

Y′ From the graph, it is clear that, it will have two real roots.
(b) 2 cos x = |sin x|, x ∈[ 0, 2π ]
242 Textbook of Differential Calculus

Plotting the graph ofy = 2 cos x andy = |sin x |, as shown below 1


19. Here, x sin x = 1 or y = sin x =
x
Y
The roots of Eq. (i) will be given by the points of intersection
y = 2cos x 1
2 of the graphs y = sin x and y = , which is shown below.
x
Y
y = 1/x
1
y = sin x 3
2
2
X
1
O π/2 π 3π/2 2π
X′ π/2 π/2 π X
–2π –3π–2–π 3π/2 2π
–1
y = sin x
–2
From the graph we see that the two curve meet at four
points for x ∈[ 0, 2π ]. –3
So, the equation 2 cos x = |sin x| has four solutions. Y′

(c) Given, f (| x| ) = 0 has a real root or f ( x ) = 0 has four Graphically, it is clear that the given equation has four roots.
positive roots. Since, f ( x ) is a polynomial of degree 5,
 π 
f ( x ) cannot have even number of real roots. Hence, f ( x ) 20. In  − , 0 , the graph of y = tan x lies below the line y = x,
 2 
has all the five roots real and one root is negative.
 π
(d) Here, 7| x| (|5 − | x|| ) = 1 or |5 − | x|| = 7 −| x| which is the tangent at x = 0 and in  0,  , it lies above the
 2
Draw the graph of y = 7 −| x| and y = |5 − | x|| line y = x.
Y y = tan x
Y y = mx
y=x
5
y=5–x (–π/2, 0)
X′ X
O (π/2, 0)

1
Y′
X
–5 5  π 
For m > 1, the line y = mx lie, below y = x in  − , 0 and
y =7– x  2 
From the graph, the number of real roots is 4.  π
above y = x in  0,  .
Hence, a → q; b → s; c → p; d → s.  2
18. Let f ( x ) = x + 2| x + 1| + 2| x − 1| Thus, graph of y = tan x and y = mx, m > 1, meet at three
 x − 2( x + 1 ) − 2( x − 1 ), x < −1  − 3 x, x < −1  π π
  points including x = 0 in  − ,  independent of m.
=  x + 2( x + 1 ) − 2( x − 1 ), − 1 ≤ x ≤ 1 = x + 4, − 1 ≤ x ≤ 1  2 2
x + 2( x + 1 ) + 2( x − 1 ), x >1  5 x, x >1 x2
  21. We have, =1
The graph of f ( x ) is shown below 1 −| x − 2|
Y x2 x2
(i) When x ≥ 2, =1⇒ =1
1 − (x − 2) 3−x
Y
7
6 4
5
3
4
3 2
2 1
1 X
X′ X 3 2 1 1 2 3 4
–3 –2 –1 0 1 2 3

Y′
Clearly, from the graph, y = k can intersect y = f ( x ) at exactly ⇒ x 2 = 3 − x, x ≠ 3
one point only if k = 3.
No point of intersection.
Chap 04 Graphical Transformations 243

x2 x2 24. Now, | x + y | = 1
(ii) When x < 2, =1⇒ =1
1 − (2 − x ) −1 + x ⇒ x + y = ±1 …(i)
⇒ x 2 = −1 + x , x ≠ 1 and | x − y| = 1
Y ⇒ x − y = ±1 …(ii)
Also, |y | = x − [x ]
4 ⇒ ± y = x − [x ]
Here, we see that there is no point of intersection between the
3 two curves.
2 Here, no ordered pair of ( x, y ) satisfy the given equations.

1
Y max (|x+y|,|x–y|=1
X 3
–4 –3 –2 –1 1 2 3 4
–1 2
y=x–[x]

X′ X
–3 –2 –1 1 2 3
No point of intersection. –y=x–[x]
–1
Here, the number of solution is zero. –2
22. Given, tan 4x = cos x –3
The number of solutions will be the point of intersection of Y′
curves y = tan 4 x and cos x
The graph of y = tan 4 x and y = cos x is given below. 25. We have, | x + y − 1| +|2x + y + 1| = 1
Y ⇒ ± ( x + y − 1 ) ± (2 x + y + 1 ) = 1
(i) ( x + y − 1 ) + (2 x + y + 1 ) = 1
tan 4x 1
⇒ 3 x + 2y = 1 …(i)
(ii) + ( x + y − 1 ) − (2 x + y + 1 ) = 1
X′ X
⇒ −x = 3 …(ii)
0 π π 3π π 5π 3π 7π π
8 4 8 2 8 4 8 cos x
(iii) −( x + y − 1 ) + (2 x + y + 1 ) = 1
–1 ⇒ x = −1 …(iii)
(iv) −( x + y − 1 ) − ( +2 x + y + 1 ) = 1
Y′ ⇒ −3 x − 2y = 1 …(iv)

3x+2y=1
From the graph, it is clear that both curve intersect at five points.
Y
∴Number of solutions is 5.
3x+2y=–1
23. Given, [sin − 1 x ] = x − [ x ] 4

The number of solution will be the point of intersection of 3


y = [sin − 1 x ] and y = x − [ x ] = { x }
2
The graph is shown below
1
π X′ X
2 sin–1x –5 –4 –3 –2 –1 0 1 2 3 4 5
1 –1

–2

–3 –2 –1 1 2 3 x=–3 x=–1 –3
[sin–1x] –4
–1
–π –π
2 Y′
−3
∴ Area of shaded region = ∫ [(3 x + 2y − 1 ) − (3 x + 2y + 1 )]dx
−1
Clearly, the two graphs intersect at only one point. −3
= ∫ − 2 dx = −2 [ x ]−−31 = −2 [ −3 + 1 ] = 4 sq units
∴ Number of solutions is 1. −1
244 Textbook of Differential Calculus

 1 Y
26. Given, f ( x ) = max x 3, x 2, , ∀ x ∈ ( 0, ∞ ) 6
 64 
5
1
Now, we draw the graph for the curves y = x 3, y = x 2 and y = 4
64
3 2 3
Y y =x y =x
2
1
X
0 1 2 3 4 5 6
1
y=
64
X′ X We see that the value of (graph A + graph B) is 4 for the
O 1,1,1
84 following values of x : ( 0, 1 ], {2, 3, 4 }, [5, 6 ).
Hence, domain D = R / {( 0, 1 ], 2, 3, 4, [5, 6 )}.
29. Let us first draw the graph for y = { x }. Now, { x } is the same
Y′ as x for 0 ≤ x < 1. In this interval, { x } will be the same as x .
Y
1 1
,0<x≤ 1
64 8
 1 X′ X
Clearly, from the graph f ( x ) =  x 2, < x < 1 0 1
 8
 x 3, x > 1 Y′
 We can easily see that this same curve will be repeated in every
27. The line through ( 0, 0) and (1, 1) has slope m = 1 and y-intercept previous and subsequent unit interval, since { x } is the same in all
such intervals. Hence, we obtain the graph of y = { x }.
b = 0, so its equation is y = x. Thus, for the part of the graph f
that joins ( 0, 0 ) to (1, 1 ), we have f ( x ) = x, if 0 ≤ x ≤ 1 Y
The line through (1, 1 ) and (2, 0 ) has slope m = − 1, so its point 1
slope form is y − 0 = ( − 1 ) ( x − 2 ) or y = 2 − x.
So, we have f ( x ) = 2 − x, if 1 < x ≤ 2
X
We also see that the graph of f coincides with the X -axis for –3 –2 –1 1 2 3 4
x >2. Putting this information together, we have the following
 x, if 0 ≤ x ≤ 1
 Now, we can easily draw y = { x } by taking a reflection in
three-piece formula for f ( x ) = 2 − x, if 1 < x ≤ 2
 0, if x > 2 the X -axis.

Y
28. The denominator is a bit complicated and we need to analyse it
in detail to determine, where it can become zero. 1
The fastest and easiest way would be to visualise the graph.
Draw the graphs for x − 1 and 5 − x , apply the greatest X′ X
integer function of these graphs separately, then add them and –2 –1 0 1 2 3
find the values of x for which this sum becomes 4. For these
–1
values of x, the denominator of f ( x ) becomes 0. Y′
Y
Y 30. In any interval n ≤ x < n + 1 , (where n is an integer) [x] has
x–1 the value n. In any interval, therefore the graph of
1 (Graph A)
X Þ X y = − [ x ] + { x } will be the graph of { x } − integer n.
1 –2 –1 0 1 2 3 4 5
e.g. 0 ≤ x < 1 ⇒ y = { x } ; 1 ≤ x < 2 ⇒ y = −1 + { x } ;

Y
−1 ≤ x < 0 ⇒ y = 1 + { x }. The graph is drawn below
4
(Graph B) Y
Y 3
2
5 2
5–x 1
XÞ X
5 1 2 3 4 5 6 7 8 X
–1 O 1 2

Now, add the graphs of A and B point by point –1


CHAPTER

05
Limits
Learning Part
Session 1
● Definition of Limits

● Indeterminate Forms

● L’Hospital’s Rule

● Evaluation of Limits

Session 2
● Trigonometric Limits

Session 3
● Logarithmic Limits

● Exponential Limits

Session 4
● Miscellaneous Forms

Session 5
● Left Hand and Right Hand Limits

Session 6
● Use of Standard Theorems/Results

● Use of Newton-Leibnitz’s Formula in Evaluating the Limits

● Summation of Series Using Definite Integral as the Limit

Practice Part
● JEE Type Examples
● Chapter Exercises

Arihant on Your Mobile !


Exercises with this #L
symbol can be practised on your mobile. See title inside to activate for free.
Session 1
Definition of Limits, Indeterminate Forms,
L’Hospital’s Rule, Evaluation of Limits

Definition of Limits (b) When the left tendency is not the same as its
right tendency Here, left tendency is l1 and right
Let lim f ( x ) = l. It would mean that when we approach
x ®a tendency is l 2 , clearly left tendency (l 1 ) is not same as
the point x = a from the values which are just greater than right tendency (l 2 ). In this case, we say that the limit of
or smaller than x = a, f ( x ) would have a tendency to move f ( x ) at x = a will not exist.
closer to the value ‘l ’.
i.e. lim f ( x ) = Doesn’t exist.
This is same as saying ‘difference between f ( x ) and l can x ®a

be made as small as we feel like by suitably choosing x in


Y
the neighbourhood of x = a’.
Mathematically, we write this, as lim f ( x ) = l, which is y = l2
x ®a
equivalent of saying that, | f ( x ) – l | < e , " x whenever y = l1
0 < | x - a | < d and e and d sufficiently small +ve
numbers.
X
O x=a
It is clear from the above discussion that, if we are x=a+h
interested in finding the limit of f ( x ) at x = a, the first x=a–h
thing we have to make sure that f ( x ) is well defined in
the neighbourhood of x = a and not necessarily at x = a Figure 5.2
(that means x = a may or may not be in the domain of
f ( x )), because we have to examine its behaviour or (c) When the left tendency and/or right tendency is
tendency in the neighbourhood of x = a. not fixed As shown in the figure 5.3, it is clear that
in this case, the function has erratic behaviour in the
Following possibilities may arise : neighbourhood of x = a and it will not be possible to
(a) Left tendency is same as its right tendency As talk about the left and right tendencies of the function
shown in figure 5.1, when we approach x = a from in the neighbourhood of x = a.
the values which are just less than a, f ( x ) has a In this case, we conclude that the limit of f ( x ) at
tendency to move towards the value l (left tendency). x = a will not exist.
Similarly, when we approach x = a from the values
i.e. lim f ( x ) = Doesn’t exist.
which are just greater than a, f ( x ) has a tendency to x ®a
move towards the value l (right tendency). Y
In this case, we say f ( x ) has limit l at x = a,
i.e. lim f ( x ) = l .
x ®a
Y y = f(x)
y = f(x)

y=l

X x a
O
x=a
Figure 5.3
Figure 5.1
Chap 05 Limits 247

Remarks Infinity ( ¥) is a symbol and not a number. It is a symbol


1. Normally, students have the Y for the behaviour of a variable which continuously
perception that limit should be a increases and passes through all limits. Thus, the
finite number. But, it is not always
so. It is quite possible that f ( x ) has
statement x = ¥ is meaningless, we should write x ® ¥.
infinite limit at x = a. Similarly, – ¥ is a symbol for the behaviour of a variable
If xlim
® a
f ( x ) = ¥, it would simply O X which continuously decreases and passes through all
mean that function has tendency to limits. Thus, the statement x = – ¥ is meaningless, we
assume very large positive values in Figure 5.4
neighbourhood of x = a (as shown in
should write x ® – ¥.
Y
figure 5.4). 1 1
1 Also, ® 0, if x ® + ¥ and ® 0, if x ® – ¥.
For example lim =¥ O
X x x
x ® 0 | x|

which indicates the left tendency as We cannot plot ¥ on paper. Infinity does not obey laws of
well as right tendency are the same. elementary algebra.
Again, if lim f ( x ) = - ¥, it would (i) ¥ + ¥ = ¥ is indeterminate
x ® a
simply mean that the function has Figure 5.5
tendency to assume very large
(ii) ¥ – ¥ is indeterminate.
negative values in the neighbourhood of x = a as shown in
figure 5.5.

For example (as shown in figure 5.5) lim


–1
=– ¥
L’Hospital’s Rule
x ® 0 |x|
f (x ) 0 ¥
At the end we discuss the case when left tendency is (– ¥) and This rule states that, if lim , reduces to or ×
right tendency is ( + ¥) (i.e. f ( x ) does not have unique
x ®a g( x ) 0 ¥
tendency). Then, differentiate numerator and denominator till this
Thus, in this case limit does not exist. form is removed.
1 Y
For example lim does not exist, f (x ) f ¢ (x )
x ® 0 x
i.e. lim = lim , provided the later limit exists.
since left tendency is (– ¥) and right x ® a g( x ) x ® a g ¢ (x )
tendency is ( + ¥) as shown in X
figure 5.6. O æ 0 ¥ö
But, if it again take form ç or ÷ ,
2. If f ( x ) is well defined at x = a, it è 0 ¥ø
doesn’t imply that Figure 5.6 f (x ) f ¢ (x ) f ¢¢( x )
lim f ( x ) = f ( a). Because, it is quite then lim = lim = lim
x ® a
possible that f ( x ) is well
x ®a g ( x ) x ® a g ¢ ( x ) x ® a g ¢¢( x )
defined at x = a but not in the neighbourhood of x = a or f ( x ) is æ0 ¥ö
well defined in the neighbourhood of x = a, but doesn’t have a and this process is continued till ç or ÷ form is removed.
è0 ¥ø
unique tendency.

Remark
Indeterminate Forms L’Hospital’s rule is applicable to only two indeterminate
¥
forms æç or ö÷ .
0
If direct substitution of x = a while evaluating lim ( x ) è0 ¥ø
x ®a
leads to one of the following forms
0 ¥ x 6 – 24 x – 16
, , ¥ – ¥, 1 ¥ , 0 0 , ¥ 0 , ¥ ´ 0 then it is called e.g. Evaluate lim ×
0 ¥ x ®2 x 3 + 2x – 12
indeterminate form. x 6 – 24 x – 16 é0 ù
Sol. We have, lim êë 0 formúû
x2 –1 0 x ®2 x + 2x – 12
3
e.g. lim = indeterminate form.
x ®1 x – 1 0 6x 5 – 24
= lim [by L’Hospital’s rule]
x n – an 0 x ®2 3x 2 + 2
lim = indeterminate form.
x ®a x – a 0 6 (2)5 – 24 168
= = = 12
sin x 0 3 ( 2) + 2
2
14
lim = indeterminate form.
x ®0 x 0
248 Textbook of Differential Calculus

é 23 x 3 25 x 5 ù é x3 x5 ù
Frequently Used Series Expansions x ê2x +
3
+2
15
+...ú –2x ê x +
3
+2
15
+...ú

1. ex = 1 +
x
+
x2 x3
+ + ... = lim ë û ë û
1! 2 ! 3!
x ®0 (2 sin 2 x )2

x × log a (log a) 2 x 2 æ8 2ö æ 64 4ö
2. ax = 1 + + + ... x 4 ç - ÷ + x 6 ç - ÷ + ...
1! 2! è3 3ø è 15 15 ø
= lim
[where, a Î R+ ] x ®0
æ x3 x5 ö
4

nx n ( n – 1) x 2 4 çx - + – ...÷
3. ( 1 + x ) n = 1 + + è 3! 5! ø
1! 2!
2 + 4 x 2 + ... 2 1
+
n ( n – 1) ( n – 2) x 3
+ ... n Î R and|x| < 1
= lim 4
= =
x ®0 4 2
3! æ x 2 ö
4 ç1 - + ...÷
4. log ( 1 + x ) = x –
x 2
+
x

x 3
+ ...
4
è 3 ! ø
2 3 4
[where –1 £ x £ 1]

5.
x n – an
x–a
= x n – 1 + x n – 2a + x n – 3a2 + ... + an – 1 Evaluation of Limits
Now, according to our plan, first of all we shall learn the
æ x 11x 2 ö
6. ( 1 + x )1 /x = e çç1 – + + ...÷÷ evaluation of limits of different forms and then learn the
è 2 24 ø existence of limits.
x 3 x5 x 7 There are eight indeterminate or meaningless forms,
7. sin x = x – + – + ...
3! 5 ! 7! which are
x 2 x4 x6 0 ¥
8. cos x = 1 – + – + ... (i) (ii) (iii) ¥ – ¥ (iv) ¥ ´ ¥
2! 4 ! 6 ! 0 ¥
x3 2 5 17 7 (v) ¥ × 0 (vi) 0 0 (vii) ¥ 0 (viii) 1 ¥
9. tan x = x + + x + x + ...
3 15 315 We will divide the problems of evaluation of limits in five
12 3 12 × 32 5 12 × 32 × 52 7 categories, which are
10. sin–1 x = x + x + x + x + ...
3! 5! 7! 1. Limit of algebraic functions
x 3 x5 2. Trigonometric limits
11. tan–1 x = x – + + ...
3 5
3. Logarithmic limits
x2 5x 4 61x 6
12. sec –1 x = 1 + + + + ... 4. Exponential limits
2! 4! 6!
5. Miscellaneous forms
2 2 2 × 22 4 2 × 22 × 4 2 6
13. (sin–1 x ) 2 = x + x + x + ... Now, we discuss one by one in details.
2! 4! 6!
x 3 x 4 2x 6
14. x cot x = 1 –
3
+ –
45 945
+ ...
Limit of Algebric Functions
2 4 6
15. sec x = 1 +
x
+
5x
+
61x
+ ...
In this section, we evaluate limit of algebraic functions
2 24 720 when variable tends to a finite or infinite value. While
x2 7x4 31x 6 0 ¥
16. x cosec x = 1 + + + + ... evaluating algebraic limits the form , and ¥ - ¥ arise,
6 360 15120 0 ¥
which we will discussed here.

x tan 2x – 2x tan x 0
y Example 1 Evaluate lim 2
× (i) Form
x ®0 (1 – cos 2x ) 0
[IIT JEE 1999] This form can be resolved by factorisation method,
x tan 2x – 2x tan x rationalisation method or by using the formula
Sol. We have, lim
x ®0 (1 – cos 2x )2 x n - an
lim = na n - 1 , which are discussed below.
x ®a x - a
Chap 05 Limits 249

(a) Factorisation Method x+h– x


y Example 4 Evaluate lim .
In this method, numerators and denominators are h ®0 h
factorised. The common factors are cancelled and the rest x +h – x
output is the result. Sol. Method I We have, lim
h ®0 h
x 2 - 3x + 2 x +h – x x +h + x é0 ù
y Example 2 Evaluate lim × = lim ´ êë 0 form úû
x ®1 x –1
h ®0 h x +h + x

x 2 – 3x + 2 ( x + h )–( x ) h
é0 ù = lim = lim
Sol. Method I We have, lim êë 0 form úû h ®0 h( x + h + x ) h ® 0 h( x + h + x )
x ®1 x –1
( x – 1)( x – 2) 1 1
= lim = lim =
x ®1 ( x – 1) h ®0 x +h + x 2 x
[as x 2 - 3x + 2 = ( x - 1)( x - 2)] Method II (L’Hospital’s rule) We have,
= lim ( x – 2) [as x –1 ¹ 0] x +h – x é0 ù
x ®1
L = lim êë 0 form úû
h ®0 h
= 1–2 = - 1 \ Applying L’Hospital’s rule,
x 2 – 3x + 2 é0 ù 1
Method II We have, L = lim êë 0 form úû –0
x ®1 x –1 2 x +h
L = lim
So, applying L’Hospital’s rule, h ®0 1
2x – 3 2 – 3 [differentiating numerator and denominator w.r.t. h]
L = lim = =–1
x ®1 1 1 1
=
[i.e. differentiating numerator and 2 x
denominator separately]
x - 2a + x - 2a
y Example 5 Evaluate lim .
x 3 – x 2 log x + log x – 1 x ® 2a
y Example 3 Evaluate lim . x 2 - 4a 2
x ®1 x 2 –1 x - 2a + x - 2a é0 ù
Sol. We have, lim êë 0 form úû
x 3 – x 2 log x + log x – 1 x ® 2a x - 4a
2 2
Sol. We have, lim
x ®1 x2 –1 x - 2a
x - 2a +
( x 3 – 1)–( x 2 – 1) log x é0 ù x + 2a
= lim = lim
x ®1 ( x 2 – 1) êë 0 form úû x ® 2a x - 2a x + 2a
1 x - 2a
( x –1){ x 2 + x + 1 – ( x + 1) log x } = lim +
= lim x ® 2a x + 2a x + 2a ( x + 2a )
x ®1 ( x –1)( x + 1)
éQ x 3 - 1 = ( x - 1) ( x 2 + x + 1),ù 1 1
= =
ê ú 4a 2 a
êë x - 1 = ( x - 1) ( x + 1)
2
úû
x 2 + x + 1 – ( x + 1) log x (c) Based on Standard Formula
= lim
x ®1 ( x + 1) x n – an
lim = na n – 1 , where n is a rational number.
12 + 1 + 1 – (1 + 1) log 1 3 x ®a x –a
= = [as log 1 = 0]
1+1 2 x n – an
Proof Let f ( x ) =
x –a
(b) Rationalisation Method
= x n – 1 + ax n – 2 + a 2 x n – 3 + ...+ a n – 1
Rationalisation is followed when we have fractional
1 1 \ lim f ( x ) = lim ( x n – 1 + ax n – 2 + a 2 x n – 3 + ...+ a n – 1 )
x ®a x ®a
powers (like , etc.) on expressions in numerator or
2 3 =a + a × a n – 2 + a 2 × a n – 3 + ...+ a n – 1
n–1
denominator or in both. After rationalisation the terms are 1444444 424444444 3
factorised which on cancellation gives the result. n terms
= a n – 1 + a n – 1 + a n – 1 + K upto n terms = n × a n – 1
250 Textbook of Differential Calculus

f (x) = ax, when 0<a<1 f (x) = ax, when a = 1


x 3 – 23 Y Y
y Example 6 Evaluate lim .
x ®2 x – 2
1
x 3 – 23 é0 ù X
Sol. We have, lim êë 0 formúû O
X
O
x ®2 x –2

é x n – an ù
= 3 ( 2) 3 – 1 ê x ®a
Q lim = n × an – 1 ú
ë x –a û Figure 5.7 Figure 5.8
= 3 (2) = 12
2
f(x) = ax,when a>1
Y
x + x 2 + ... + x n - n
y Example 7 Evaluate lim .
x ®1 x -1
X
x + x 2 + x 3 + ... + x n - n O
Sol. We have, lim
x ®1 x -1
Figure 5.9
( x - 1) + ( x 2 - 12 ) + ( x 3 - 13 )+...+( x n - 1n )
= lim
x ®1 ( x - 1) Now, see the graph for a x , when a > 1 . This graph appears
ì x - 1 x 2 - 12 x 3 - 13 x n - 1n ü to touch X-axis in the negative side of X-axis and
= lim í + + +...+ ý
x ®1 x - 1 x -1 x -1 x -1 þ thereafter it increases rapidly. This is why because
î
lim a x ® 0, again you will also find the result,
= 1 + 2 (1)2 -1 + 3 (1)3 -1 + ... + n (1)n -1 x ®-¥
lim a x ® ¥
= 1 + 2 + 3 + ... + n x ®¥

=
n ( n + 1) ì¥, if a > 1
ï
2 Thus, we have lim a = í1, if a = 1
x
x ®¥
3 ï0, if 0 £ a < 1
y î
y Example 8 The value of lim as
x ®1 x3 - y 2 -1 This type of problems are solved by taking the highest
y ®0
power of the terms tending to infinity as common from
( x , y ) ® (1, 0) along the line y = x - 1 is numerator and denominator. That is after they are
(a) 1 cancelled and the rest output is the result (or apply
(b) -1 L’Hospital’s rule).
(c) 0 x2 + 5
(d) Doesn’t exist y Example 9 Evaluate lim .
x ®¥ x 2 + 4x + 3
Sol. As, y ® x - 1 or x ® y + 1 x2 + 5
Sol. Let L = lim
(y )3 x ®¥ x + 4x + 3
2
\ lim
y ®0 ( y + 1) - y - 1
3 2
Dividing numerator and denominator by x 2 , we get
5
Using L, Hospital’s rule, 1+ 2
x 1+0
3y 2 6y L = lim = =1
lim = lim x ®¥ 4
1+ + 2
3 1 + 0 + 0
y ®0 3 (y + 1) - 2y
2 y ®0 6 ( y + 1) - 2 x x
0 K
=0 = [because ® 0, when x ® ¥, where K is any constant]
6 x
Hence, (c) is the correct answer. x2 + 5 é¥ ù
Aliter We have, L = lim êë ¥ form úû
x ®¥ x + 4x + 3
2
(ii) Algebraic Function of ¥ Type
2x é¥ ù
¥ Applying L’Hospital’s rule, L = lim
® ¥ 2x + 4 êë ¥ form úû
(a) Form x

¥ 2
Again, applying L’Hospital’s rule, L = lim =1
First we should know the limiting values of a x (a > 0 ) as x ® ¥2

x ® ¥. See the graphs of this function.


Chap 05 Limits 251

(n + 2)! + (n + 1)! An Important Result


y Example 10 Evaluate lim .
n ®¥ (n + 2)! – (n + 1)! If m, n are positive integers and a0, b0 ¹ 0 and non-zero real
(n + 2)(n + 1) ! + (n + 1) ! numbers, then
Sol. We have, lim a x m + a1x m – 1 + K + am – 1x + am
n ®¥ (n + 2)(n + 1) ! – (n + 1) ! lim 0 n
x ® ¥ b x + b xn – 1 + K + b
n – 1x + bn
(n + 1) ![ n + 2 + 1] ( n + 3) é¥ ù 0 1
= lim = lim êë ¥ form úû ì 0, m<n
n ®¥ (n + 1) ![ n + 2 – 1] n ® ¥ (n + 1) ï a
1 + 3/n 1 + 0 é 1 ù ïï 0 , m=n
= lim = =1 êëQ n ® 0, as n ® ¥ úû
= í b0
n ® ¥ 1 + 1/n 1+0 ï ¥, m > n when a0b0 > 0
ï
ïî– ¥, m > n when a0b0 < 0
(b) ¥ - ¥ Form
Such problems are simplified (generally rationalised) first, ax 2 + b
y Example 13 Evaluate lim , when a ³ 0.
æ ¥ö x ®¥ x + 1
thereafter they generally acquire ç ÷ form.
è ¥ø
ax 2 + b
Sol. Here, if a ¹ 0 ; lim
y Example 11 Evaluate lim ( x – x 2 + x ) . x ®¥ x + 1
x ®¥ [as degree of numerator > degree of denominator]
Sol. We have, lim ( x – x 2 + x ) ax 2 + b
x ®¥ = lim =¥ [as a > 0]
x ®¥ x + 1

x – x2 + x x + x2 + x 0× x 2 + b
= lim ´ [ ¥ – ¥ form] Again, if a = 0 ; lim = lim
b
=0
x ®¥ 1 x + x2 + x x ®¥ x +1 x ®¥ x + 1

x 2 – (x 2 + x ) –x [as degree of numerator < degree of denominator]


= lim = lim ax 2 + b ì ¥, a > 0
x ®¥
x+ x +x 2 x ®¥
x + x2 + x \ lim =í
x ®¥ x + 1 î0, a = 0
–x
= lim æx2 +1 ö
x ®¥ ì 1ü y Example 14 If lim ç – ax – b ÷ = 0, find the
x í1 + 1 + ý x ®¥ è x + 1
î xþ ø
values of a and b.
–1 1 é 1 ù æx2 + 1 ö
= lim =– êëQ x ® 0, as x ® ¥ úû
x ®¥ 1 2 Sol. Given, lim ç – ax – b ÷ = 0
1+ 1+ x ®¥ è x + 1 ø
x
x 2 + 1 – ax 2 – ax – bx – b
Þ lim =0
y Example 12 Evaluate lim ( x + x + 1 – x + 1 ). 2 2
x ®¥ x +1
x ®¥
x 2 (1 – a ) – x (a + b ) + (1 – b )
Sol. We have, lim ( x 2 + x + 1 – x 2 + 1 ) [ ¥ – ¥ form ] Þ lim =0
x ®¥
x ®¥ x +1
Since, the limit of above expression is zero.
( x 2 + x + 1 – x 2 + 1) ( x 2 + x + 1 + x 2 + 1)
= lim ´ \ Degree of numerator < Degree of denominator.
x ®¥ 1 ( x 2 + x + 1 + x 2 + 1) So, numerator must be a constant, i.e. a zero degree
polynomial.
( x 2 + x + 1) – ( x 2 + 1)
= lim \ 1 – a = 0 and a + b = 0
x ®¥ x2 + x + 1 + x2 + 1 Hence, a = 1 and b = –1
x æx2 +1 ö
= lim y Example 15 If lim ç – ax – b ÷ = 2, find the
x ®¥ x2 + x + 1 + x2 + 1 x ®¥ è x + 1 ø
1 1 1 values of a and b.
= lim = =
x ®¥ 1 1 1 1+1 2 æx2 + 1 ö
1+ + 2 + 1+ 2 Sol. We have, lim ç – ax – b ÷
x x x x ®¥ è x + 1 ø
é 1 ù x 2 (1 – a ) – x (a + b ) + (1 - b )
= lim =2
êëQ x ® 0, as x ® ¥ úû x ®¥ ( x + 1)
252 Textbook of Differential Calculus

Since, limit of above expression is a finite non-zero number. n ( n + 1)


Sol. As, Sn =
\ Degree of numerator = Degree of denominator 2
Þ 1–a = 0 Þ a = 1 n ( n + 1) n 2 + n - 2 ( n + 2) ( n - 1)
Þ Sn - 1 = -1= =
Putting a = 1 in above limit, we get 2 2 2
- x (1 + b) + (1 + b ) Sn æ n ö æn + 1ö
lim =2 \ =ç ÷ç ÷
x ®¥ x +1 Sn - 1 è n - 1 ø è n + 2 ø
Þ – (1 + b ) = 2 Þ b = –3
æ2 3 4 n öæ3 4 5 n + 1ö
Hence, a = 1 and b = - 3. Þ Pn = ç × × K ÷ç × × K ÷
è1 2 3 n - 1ø è 4 5 6 n + 2ø
æx2 +1 ö
y Example 16 If lim ç – ax – b ÷ = ¥, find a and æn ö æ 3 ö
x ®¥ è x + 1
=ç ÷ç ÷
ø è 1 ø èn + 2ø
b. 3n
\ lim Pn = lim =3
n ®¥ n ® ¥n + 2
æx2 + 1 ö
Sol. Given, lim ç – ax – b ÷ = ¥
x ®¥ è x + 1 ø 1
y Example 18 If lim -1
= 1, x lies in the
Þ lim
x (1 – a ) – x (a + b ) + (1 – b )
2

n ®¥ (sin x )n + 1
x ®¥ x +1 interval
The limit of above expression is infinity. (a) ( - sin 1, sin 1) (b) ( - 1, 1)
\ Degree of numerator > Degree of denominator (c) (0, 1) (d) ( - 1, 0)
Þ 1–a > 0 Þa ¹ 1 1
Sol. Here, lim -1
= 1 is possible only, if
Hence, a < 1 and b can assume any real value. n ®¥ (sin x )n + 1

y Example 17 Let S n = 1 + 2 + 3 + L + n -1 < sin -1 x < 1

S2 S S S Þ x Î ( - sin 1, sin 1)
and Pn = × 3 × 4 K n , Hence, (a) is the correct answer.
S 2 - 1 S 3 - 1 S 4 - 1 Sn - 1
where n Î N (n ³ 2). Find lim Pn .
n ®¥

Exercise for Session 1


g( x ) f (a ) - g(a ) f ( x )
1. If f (a ) = 2, f ¢ (a ) = 1, g(a ) = - 1, g ¢ (a ) = - 2, then lim is
x ®a x -a
(a) - 5 (b) 3 (c) - 3 (d) 5
x cos x - log (1 + x )
2. The value of lim is
x ®0 x2
1 1
(a) 1 (b) (c) (d) None of these
4 2
2
e x - cos x
3. The value of lim is
x ®0 x2
3 3 1 1
(a) (b) - (c) (d) -
2 2 2 2
cos x - cos a
4. The value of lim is
x ®a cot x - cot a
(a) - sin3 a (b) cos3 a (c) sin3 a (d) cot a
Chap 05 Limits 253

æ 1 ö
5. The value of lim ç 2 - cot x ÷ is
x ®0 èx ø
(a) 0 (b) 1
1
(c) (d) None of these
4

6. The value of lim ( a 2x 2 + ax + 1 - a 2x 2 + 1), (a > 0) is


x ®¥
1 1
(a) (b) -
2 2
(c) Doesn’t exist (d) None of these
13 + 23 + 33 + . . . + n 3
7. The value of lim is
n®¥ (n 2 + 1)2
1 1
(a) (b)
4 2
1
(c) (d) None of these
2 2
1× n + 2 × (n - 1) + 3 × (n - 2) + . . . + n × 1
8. The value of lim is
n®¥ 12 + 22 + . . . + n 2
(a) 1 (b) -1
1 1
(c) (d)
2 2
an + b n
9. The value of lim , (where a > b > 1) is
n ® ¥ an - b n

(a) 1 (b) -1
1 1
(c) (d)
2 2
Session 2
Trigonometric Limits
Trigonometric Limits y Example 21 Evaluate lim 2 –x sin (2 x ).
x ®¥
To evaluate trigonometric limits the following results are
1
given below Sol. Since, 2–x = . We know that, as x ® ¥, 2x ® ¥
2x
sin x tan x
(i) lim =1 (ii) lim =1 \ The given limit = 0 ´
x ®0 x x ®0 x
[A finite number between –1 and +1] =0
sin–1 x tan–1 x sin (2x )
(iii) lim =1 (iv) lim =1 Hence, lim =0
x ®0 x x ®0 x x ®¥ ( 2x )
sin x 0 p
(v) lim = (vi) lim cos x = 1 y Example 22 Evaluate lim e x sin (d /e x ).
x ®0 x 180° x ®0
x ®¥
sin ( x – a ) tan ( x – a )
(vii) lim = 1 (viii) lim =1 Sol. When x ® ¥, e ® ¥ x
x ®a x –a x ®a x –a d finite
But, angle of sine = x = =0
1 – cos x e ¥
y Example 19 Evaluate lim . sin (d/e x ) sin d/e x
x ®0 x2 \ The given limit = lim = lim ´d
1–cos x
x ®¥ 1/e x d
® 0 d/e
x

Sol. We have, lim ex


x ®0 x2 =1 ´ d = d
2 sin 2 x/ 2 2 sin 2 x/ 2 é0 ù
= lim = lim êë 0 formúû
x – sin x
x ®0 x 2 x ®0 4 x 2/ 4 y Example 23 Evaluate lim .
2
x ®¥ x + cos 2 x
1 æ sin x / 2 ö 1 2 1
= lim ç ÷ = ( 1) = æ sin x ö
x ® 0 2 è x /2 ø 2 2 x ç1 – ÷
x – sin x è x ø
Sol. We have, lim = lim
1 - cos (1 - cos x ) x ®¥ x + cos 2 x x ®¥ æ cos 2 x ö
y Example 20 Solve lim x ç1 + ÷
x ®0 sin 4 x è x ø
æ xö sin x
1 - cos ç2sin 2 ÷ 1–
1 - cos (1 - cos x ) è 2ø sin 4 x = lim x =
1–0
=1
Sol. lim = lim
x ®0 sin 4 x x ®0 x4 x4 x ®¥ 2
cos x 1+0
1+
x
æ xö
1 - cos ç2sin 2 ÷ 4
è 2ø æ sin x ö
= lim lim ç ÷ sin 2 x – sin 2 y
x ®0 x4 x ®0è x ø y Example 24 Evaluate lim .
x ®y x2 – y 2
æ xö
2sin 2 çsin 2 ÷ sin 2 x – sin 2 y sin ( x + y ) sin ( x –y )
è 2ø Sol. lim = lim
= lim 1
x ®0 x4 x ®y 2
x –y 2 x ®y ( x + y ) ( x –y )
2 sin( x + y ) sin( x –y ) sin(2y )
æ æ 2 xö xö = lim ´ lim = ´1
ç sin çsin ÷ sin 2 ÷ x ®y (x +y ) x ® y ( x –y ) 2y
è 2 ø
= lim 2 × ç × 2÷
x ®0 ç 2 x x 2
÷ [as x ® y Þ ( x – y ) ® 0, but x + y ® 2y ]
ç sin 4× ÷
è 2 4 ø =
sin 2y
1 1 2y
=2´ =
42 8
Chap 05 Limits 255

y Example 25 \ a is minimum, when x = –1, i.e. a = 2


lim [( x + 5) tan -1 ( x + 5) - ( x + 1) tan -1 ( x + 1)] is equal to Again, b = lim
2 sin 2 q/ 2
= lim
2 sin 2 q/ 2
× 2
x ®¥ q®0 q2 q®04 q /4
(a) p (b) 2p
2 sin 2 q/ 2 1
p = lim × =
(c) (d) None of these q ® 0 4 (q/ 2)2 2
2
n n n –r
lim (( x + 5) tan -1( x + 5) - ( x + 1) tan -1( x + 1)) æ1ö
Sol. Here,
x ®¥
Hence, åar bn – r = å 2r × çè 2 ÷ø
r =0 r =0
ìp ü
= lim ( x + 5) í - cot -1( x + 5)ý - ( x + 1) 1
x ®¥ î2 þ Þ n
{20 + 22 + 24 + ...+ 22n }
2
ìp -1 ü
í - cot ( x + 1)ý 1 ì 1 ( 4n + 1 – 1) ü 4n + 1 – 1
î 2 þ Þ í ý =
2n î 4 –1 þ 2n ´ 3
æ 1 ö -1 æ 1 ö
tan -1 ç ÷ tan ç ÷ [i.e. sum of (n + 1) terms of GP]
è x + 5ø è x + 1ø
= lim 2p - + = 2p
x ®¥ æ 1 ö æ 1 ö
n
4n + 1 – 1
ç
è x + 5ø
÷ ç
è x + 1ø
÷ \ åar bn – r =
2n ´ 3
r =0

Hence, (b) is the correct answer.


y Example 28 Evaluate
sin ( p cos 2 x ) æxö æxö æxö æxö
y Example 26 Evaluate lim . lim cos ç ÷ cos ç ÷ cos ç ÷ ... cos ç n ÷ .
x ®0 x2 n ®¥ è2ø è4ø è8ø è2 ø
[IIT JEE 2001]
æxö æ x ö æx ö æx ö æx ö
sin ( p cos 2 x ) sin { p (1–sin 2 x )} Sol. Here, lim cos ç n ÷ cos ç n -1 ÷ ...cos ç ÷ cos ç ÷ cos ç ÷
Sol. lim = lim n ®¥ è2 ø è2 ø è8ø è4ø è2ø
x ®0 x2 x ®0 x2
sin ( p – p sin 2 x ) sin 2n A
= lim We know, cos A cos 2A cos 22 A ...cos 2n -1 A =
x ®0 x2 2n sin A
ì sin ( p sin 2 x ) p sin 2 x ü æxö æ x ö æx ö æx ö æx ö
sin ( p sin 2 x ) Thus, lim cos ç n ÷ cos ç n -1 ÷ ...cos ç ÷ cos ç ÷ cos ç ÷
= lim = lim í ´ ´ ý n ®¥ è2 ø è2 ø è8ø è4ø è2ø
x2 î p sin x
2
x ®0 x ®0 1 x2 þ
æxö
sin ( p sin 2 x ) sin 2 x sin 2n ç n ÷
= lim ´ p ´ lim è2 ø sin x
= lim = lim
x ®0 p sin 2 x x ®0 x2 n ®¥ n æxö n ®¥ æxö
2 sin ç n ÷ 2n sin ç n ÷
=1 ´ p ´ 1 = p è2 ø è2 ø
1 x
y Example 27 Let a = min { x 2 + 2x + 3, x ÎR} and = sin x × lim ×
n ®¥ n æxö x
1 – cos q n 2 sin ç n ÷
b = lim . The value of S a r ×b n – r is è2 ø
q®0 q 2 r = 0
sin x 1
= × lim ×x
Sol. Here, a = min { x 2 + 2x + 3, x Î R } x n ®¥ æxö
2n sin ç n ÷
i.e. x 2 + 2x + 3 = x 2 + 2x + 1 + 2 è2 ø

= ( x + 1) 2 + 2 sin x sin x é x q ù
= ×1 = n ® ¥, n ® 0 and lim = 1ú
x x êë 2 q ® 0 sin q û
Exercise for Session 2
1. If lim ( x -3 sin 3x + ax -2 + b ) exists and is equal to zero, then
x ®0
9 9
(a) a = - 3, b = (b) a = 3, b =
2 2
-9
(c) a = - 3, b = (d) None of these
2
x sin a - a sin x
2. The value of lim is
x ®a x -a
(a) a sin a - cos a (b) sin a - a cos a
(c) cos a + a sin a (d) sin a + a cos a
2 + cos x - 1
3. The value of lim is
x ®p ( p - x )2
1 1
(a) (b)
4 2
(c) 2 (d) Doesn’t exist
( 2 - cos q - sin q)
4. The value of lim is
q ® p/ 4 (4q - p )2
1 1
(a) (b)
16 2 16
1 1
(c) (d)
8 2 2 2
(cos x + sin x )3 - 2 2
5. The value of lim is
x ® p/ 4 1 - sin 2x
3 3
(a) (b) -
2 2
1 1
(c) (d) -
2 2
Session 3
Logarithmic Limits, Exponential Limits

Logarithmic Limits y Example 32 Evaluate lim


log e (1 + 2h ) – 2 log e (1 + h )
.
In this section, we will deal with the problems based on h ®0 h2
expansion of logarithmic series, which is given below [IIT JEE 1999]
x2 x 3 loge (1 + 2h ) – 2 loge (1 + h )
log (1 + x ) = x – + + ... ¥ Sol. We have, lim
h ®0 h2
2 3
é (2h )2
(2h )3 ù æ h2 h3 ö
where, –1 £ x £ 1 and it should be noted that the expansion ê(2h ) – + – ... ¥ ú –2 çh – + – ...÷
is true only if the base is e. To evaluate the logarithmic = lim ë 2 3 û è 2 3 ø
log (1 + x ) h ®0 h2
limit, we use lim = 1.
x ®0 x –h + 2h – ...
2 3
= lim
h ®0 h2
log { 1 + ( x – a )}
y Example 29 Evaluate lim × h 2 {–1 + 2h – ...}
x ®a (x – a) = lim
h ®0 h2
log {1 + ( x - a )} = lim {–1 + 2h – ...} = - 1
Sol. We have, lim h ®0
x ®a ( x - a)
ì 1 öü
Let x – a = y , when x ® a; y ® 0 y Example 33 Solve lim í x - x 2 × log æç 1 + ÷ ý.
x ®¥ î è x øþ
log {1 + y }
\ The given limit = lim =1 1 ì æ 1 öü
y ®0 y Sol. Here, put x = in lim í x - x 2 × log ç1 + ÷ý
y x ®¥ î è x øþ
log 10 (1 + h )
y Example 30 Evaluate lim × ì 1 log (1 + y )ü
= lim í -
h ®0 h ý
y ®0 y
î y2 þ
log10 (1 + h ) loge (1 + h ) ´ log10 e
Sol. We have, lim = lim y - log(1 + y )
h ®0 h h ®0 h = lim
loge (1 + h )
y ®0 y2
= lim ´ log10 e
h ®0 h ì y2 y3 y4 ü
y - íy - + - + Ký
é log(1 + x ) ù î 2 3 4 þ
= log10 e ´ 1 = log10 e êëQ xlim = 1ú = lim
®0 x û y ®0 y 2

log ( 5 + x ) – log ( 5 – x ) ì1 y ü
y Example 31 Evaluate lim . y 2 í - + Ký
x ®0 x = lim î 2 3 þ
ì æ x öü ì æ x öü y ®0 y2
log í5 ç1 + ÷ý – log í5 ç1 – ÷ý
î è ø
5 þ î è 5 øþ ì1 y ü 1
Sol. We have, lim = lim í - + Ký =
x ®0 x y ® 0î2 3 þ 2
æ xö æ xö
log 5 + log ç1 + ÷ – log 5– log ç1 – ÷
è 5ø è 5ø Remark
= lim
x ®0 x If we solve above question as
æ xö æ xö ì 1 log ( 1 + y ) ü ì 1 log ( 1 + y ) 1 ü
log ç1 + ÷ log ç1 – ÷ lim í - ý = ylim í - × ý
è 5ø è 5ø 1 1 2 y®0
îy y2 þ ® 0î y y yþ
= lim – = + =
x ®0 æx ö æ xö 5 5 5 é log ( 1 + y ) ù
5ç ÷ –5 ç – ÷ êas ylim = 1ú
è5ø è 5ø ë ® 0 y û
æ1 1ö
= lim ç - ÷ = 0, then it is not correct.
é log (1 + x ) ù y®0 èy yø
êëQ xlim = 1ú
®0 x û
258 Textbook of Differential Calculus

Since, limit is finite, so (a – b ) = 0 Þ b = a


Exponential Limits ax 2
There are two types of exponential limits discussed below xa + + ... ¥
2!
\ lim =2
x ®0 x
(i) Based on Series Expansion ax
Þ lim a + + ... ¥ = 2 Þ a = 2
x2 x 3 x ®0 2!
ex =1+ x + + + ... ¥
2! 3! \ b=2
To evaluate the exponential limit, we use the following a sin x - bx + cx 2 + x 3
results. y Example 38 Solve lim , if it
log (1 + x ) - 2x 3 + x 4
x ®0 2x 2
ex –1 ax –1
(a) lim =1 (b) lim = log e a exists and is finite, also find a, b and c.
x ®0 x x ®0 x a sin x - bx + cx 2 + x 3
Sol. Let L = lim
ax – b x x ® 0 2x 2 log (1 + x ) - 2x 3 + x 4
y Example 34 Evaluate lim ×
x ®0 x æ x3 x5 ö
aç x - + - ...÷ - bx + cx 2 + x 3
ax – bx ( a x –1 ) – ( b x –1 ) è 3 ! 5! ø
Sol. We have, lim = lim = lim
x ®0 x x ®0 x x ®0 æ x2 x3 x4 ö
x x 2x 2 ç x - + - + ...÷ - 2x 3 + x 4
a –1 b –1 è 2 3 4 ø
= lim – lim = log a – log b = log (a / b )
x ®0 x x ®0 x
æ aö a
(a - b )x + cx 2 + ç1 - ÷ x 3 + x 5 + ...
(ab ) x – a x – b x + 1 è 3! ø 5!
y Example 35 Evaluate lim × = lim
x ®0 2 5 1 6
x ®0 x2 x - x +...
3 2
(ab )x – a x –b x + 1 a x b x – a x –b x + 1
Sol. We have, lim = lim a
x ®0 x 2 x ® 0 x2 For finite limit, a - b = 0, c = 0, 1 - = 0, i.e. a = b = 6, c = 0]
x x
a (b – 1)–(b –1) x 3!
= lim a
x ®0 x2 + higher powers of x
x
( a – 1) ( b x –1 ) \ L = lim ! 5
= lim ´ lim = log a ´ log b x ®0 2 1
- x +L
x ®0 x x ®0 x 3 2
é ax - 1 ù a 3 a 6 3
ê x ®0
Q lim = loge a ú = × = = = [Qa = 6]
ë x û 5! 2 80 80 40
a sin x - bx + cx 2 + x 3 3
e tan x – e x So, lim =
y Example 36 Evaluate lim × x ®0 2x log (1 + x ) - 2x + x
2 3 4
40
x ® 0 tan x – x
Where, a = 6 = b, c = 0
e tan x – e x e x ´ e (tan x – x ) – e x
Sol. We have, lim = lim y Example 39 Find the values of a, b and c such that
x ® 0 tan x – x x ®0 ( tan x – x )
e x {e tan x – x – 1} axe x – b log (1 + x ) + cxe – x
= lim
x ®0 (tan x – x ) lim = 2.
x ® 0 x 2 sin x
= e 0 ´ 1 [as x ® 0, tan x – x ® 0]
axe x - b log (1 + x ) + c xe - x
=1 ´ 1 =1 Sol. We have, lim =2
x ®0 x 2 sin x
ae x – b Using the expansion, we have
y Example 37 Evaluate lim = 2 . Find a and b.
x ®0 x é æ x2 ö æ x2 x3 ö ù
êax ç1 + x + 2 ! + ...÷ –b ç x – 2 + 3 –...÷ ú
ê è
æ x x2 ö ø è ø
ú
a ç1 + + + ... ¥ ÷ –b
è ø ê æ x x2 x3 öú
ae x - b 1! 2!
ê + cx ç1 – + - + ...÷ ú
Sol. Given, lim = lim =2
x ®0 x x ®0 x êë è 1! 2! 3! øú
lim û =2
ax 2 x ®0 æ 3 5 ö
(a –b ) + xa + + ... ¥ x 2 çx –
x
+
x
–...÷
2! è ø
Þ lim =2 3! 5!
x ®0 x
Chap 05 Limits 259

æ ö px
b æa b c ö tan
y Example 42 Evaluate lim æç 2 – ö÷
x (a – b + c )– x 2 çça + – c ÷÷ + x 3 ç – + ÷ + ... a 2a
è 2 ø è2 3 2ø .
x®a è xø
lim =2
x ®0 æ x x 3 ö 5
px
x 2 çx – + –...÷ tan
è 3! 5! ø æ aö 2a
Sol. We have, lim ç2 – ÷
x ®a è xø
Now, above limit would exist, if least power in numerator is
px æ aö px
greater than or equal to least power in denominator, i.e. tan lim ç 1 – ÷ × tan
ì æ a öü 2a è xø
Coefficient of x and x 2 must be zero and coefficient of x 3 = lim í1 + ç1 – ÷ý =e x®a 2a
[1¥ form]
x ®a î è x øþ
should be 2.
æ x –a ö px
b a b c lim ç ÷ × tan lim f ( x )
i.e. a – b + c = 0, a + – c = 0, – + =2 =e x ® a
è x ø 2a
=e x ® a …(i)
2 2 3 2
On solving, we get a = 3, b = 12, c = 9. Let x –a = h , we get
æ h ö p h æ p ph ö
lim ç ÷ × tan (a + h ) lim × tan ç + ÷
èa +h ø 2a a +h è 2 2a ø
(ii) Evaluation of Exponential Limits eh ® 0 = eh ® 0
of the Form 1¥ lim
h®0
h é
a + h êë
æ ph ö ù
- cot ç ÷
è 2a ø úû
lim
h®0
–h p
× ×
1
(a + h ) tan ( ph/ 2a ) 2a p/ 2a
=e =e
To evaluate the exponential limits of the form 1 ¥ , we use –2a
×
ph/ 2a –2a
lim
h ® 0 p (a + h ) tan ( ph/ 2a ) é x ù
the following results. =e = e p (a ) = e – 2 /p ê as lim = 1ú
If lim f ( x ) = lim g( x ) = 0, ë x ® 0 tan x û
x ®a x ®a
x +4
lim
f (x)
æ x + 6ö
then lim {1 + f ( x )} 1 /g ( x )
=e x ®a g(x) y Example 43 Evaluate lim ç ÷ .
x ®a x ®¥ è x + 1ø
x +4
or when lim f ( x ) = 1 and lim g ( x ) = ¥ æ x + 6ö
x ®a x ®a Sol. We have, lim ç ÷
x ®¥ è x + 1 ø
Then, lim { f ( x )} g ( x ) = lim [1 + f ( x ) – 1]g ( x ) x +6
x ®a x ®a As x ® ¥, lim = 1 and ( x + 4 ) ® ¥ [1¥ form]
x ®¥ x + 1
lim (f ( x ) – 1 ) g ( x )
= e x ®a x +4 x +4
æ x + 6ö é æ x + 6 öù
Particular Cases \ lim ç ÷ = lim ê1 + ç –1 ÷ ú
x ® ¥ è x + 1ø x ®¥
ë è x + 1 øû
x
æ 1ö x +4 æ 5 ö
(i) lim (1 + x ) 1 /x = e (ii) lim ç 1 + ÷ = e é 5 ù lim ç
x®¥
÷ × (x + 4 )
è x + 1ø
x ®0 x ®¥ è xø = lim ê1 + =e
x ®¥ ë x + 1 úû
x x +4
æ lö lim 5 ×
é x+4 ù
(iii) lim (1 + lx ) 1 /x = e l (iv) lim ç 1 + ÷ = e l =e
x®¥ x +1
=e 5 × (1)
=e 5 ê as x ® ¥; lim = 1ú
x ®0 x ®¥ è xø x ®¥ x + 1
ë û
x
y Example 40 Evaluate lim æç 1 + ö÷ .
1
2
x®¥ è
y Example 44 Evaluate lim (1 + tan 2 2
x) x .
xø x®0
1 1
x 2 lim tan 2 x ×
lim × x
æ 2ö Sol. We have, lim (1 + tan 2
x 2
) x =e x®0 2x
Sol. We have, lim ç1 + ÷ = e x ® ¥ x = e 2 [1¥ form] x ®0
x ®¥ è xø 2
1 æ tan x ö
lim ç ÷
2 x®0 è x ø é tan x ù
y Example 41 Evaluate lim (log 3 3x )log x 3 . =e = e 1/ 2 êë as xlim = 1ú
x ®1
®0 x û
1
log x 3
= lim (log 3 3 + log 3 x )log x 3 ì p üx
y Example 45 Evaluate lim í tan æç + x ö÷ ý .
Sol. We have, lim (log 3 3x )
x ®1 x ®1

é x®0î è4 øþ
1/ log 3 x 1 ù
= lim (1 + log 3 x ) ê as logb a = log b ú [IIT JEE 1993]
x ®1 ë a û 1
1 ì p üx 1
ï tan 4 + tan x ï
lim log 3 x ´
=e
x ®1 log 3 x ì æp öü x
Sol. We have, lim í tan ç + x ÷ý = lim í ý
x ® 0î è4 øþ x ®0 p
= e1 ï 1 – tan tan x ï
î 4 þ
260 Textbook of Differential Calculus

1 1
ì 1 + tan x ü x ì 2 tan x ü x y Example 49 The value of
= lim í ý = lim í1 + ý
x ® 0 î 1 – tan x þ x ®0î 1 - tan x þ lim (2x 2 - 9 x + 8 ) cot ( 2x - 7 ) is equal to
x ® 7/2
æ 2 tan x ö 1 tan x 1
÷× lim 2× ×
(b) e -5 / 2
lim ç
x ® 0 è 1 – tan x ø x x®0 (a) e 5 / 2
=e =e x 1 – tan x
= e2
(c) e 7 / 2 (d) e 3 / 2
1 /x 2 - 7)
æ 1 + 5x 2 ö Sol. Here, lim (2x 2 - 9 x + 8)cot (2 x [1¥ form]
y Example 46 Evaluate lim ç ÷ . x ® 7/2
x ® 0 è 1 + 3x 2 ø - 7)
[IIT JEE 1996] = lim {1 + (2x 2 - 9 x + 7 )} cot (2 x
1/x 2 1/x 2 x ® 7/2
æ 1 + 5x 2 ö æ 2x 2 ö
Sol. We have, lim ç ÷ = lim ç1 + ÷ lim ( 2 x 2 - 9 x + 7 ) × cot ( 2 x - 7 )
x ® 0 è 1 + 3x 2 ø x ®0 è 1 + 3x 2 ø = e x ® 7/2
[1¥ form] 4x - 9
æ 2x 2 ö 1 lim
2 x ® 7/2
lim ç ÷. sec 2 ( 2 x - 7 ) . 2
x ® 0 è 1 + 3x 2 ø x 2 1+ 0 =e = e 5/2
=e =e = e2
x Hence, (a) is the correct answer.
æ x – 3ö
y Example 47 Evaluate lim ç ÷ . [IIT JEE 1
x ® ¥ è x + 2ø p
æ ö log x
y Example 50 The value of lim ç tan æç + log x ö÷ ÷
2000]
x x
x ®1 è è4 øø
æ x –3ö æ (– 5) ö
Sol. We have, lim ç ÷ = lim ç1 + ÷ is equal to
x ® ¥ è x + 2ø x ®¥ è x + 2ø
–5
(a) e (b) e -1
(d) e -2
æ –5 ö lim
lim ç ÷× x
x®¥
1+
2 (c) e 2
x®¥ è x + 2ø
=e = e x = e –5 1
æ æp ö ö log x
Sol. Here, lim ç tan ç + log x ÷ ÷ [1¥ form]
y Example 48 The value of x ®1 è è4 øø
p
sec 2 1
ì æ p öü 2 - bx
æ 2 tan (log x ) ö log x lim ×
2 tan (log x ) 1
lim ísin 2 ç ÷ý is equal to = lim ç1 + ÷ =e
x ® 1 {1 - tan (log x )} log x

x ®0
î è 2 - ax ø þ x ®1 è 1 - tan (log x ) ø

(a) e - a /b (b) e - a
2
/b 2 tan (log x ) 1
(c) a 2 a /b (d) e 4 a /b 2 lim ×
1 - tan (log x )
= e 2 × (1) = e 2
x ®1
æ p ö =e log x
sec 2 ç ÷
æ æ p öö è 2 - bx ø [as x ® 1, log x ® 0]
Sol. Here, lim çsin 2 ç ÷÷
x ®0 è è 2 - ax ø ø Hence, (c) is the correct answer.
æ p ö 2m
sec 2 ç ÷
y Example 51 The value of lim æç sin + cos ö÷
ì æ p öü è 2 - bx ø x 3x x
= lim í1 - cos 2 ç ÷ý [1¥ form ]
x ®0 è 2 - ax øþ x ®0 è m mø
î
ì ü is equal to
ï ï
ï 2 æ
lim - í cos ç
p ö
÷×
1 ï
ý (a) e (b) 1 (c) e -1 (d) e 2
x®0
ï è 2 - ax ø æ p öï 2m
cos 2 ç ÷
ïî è 2 - bx ø ïþ
=e ï æ x 3x ö x
Sol. Here, lim çsin + cos ÷
ì æ p ö
ç
æ p ö
÷ cos ç ÷
pa
´
ü x ®0 è m mø
ï 2 sin ï
ï è 2 - ax ø è 2 - ax ø ( 2 - ax ) 2 ï 2m
lim - í × ý
pb
x®0
ï 2 sin æ p ö æ p ö
÷ ´ ï ì æ x 3x öü x
ïî
ç ÷ cos ç
è 2 - bx ø è 2 - bx ø ( 2 - bx ) 2 ïþ = lim í1 + çsin + cos - 1÷ý [1¥ form ]
=e ï x ® 0î è m m øþ
[using L’Hospital’s rule] 2m æ x
+ cos
3x ö
- 1÷
lim ç sin
x è ø
æ 2p ö = ex® m m
0
sin ç ÷
è 2 - ax ø a ( 2 - bx ) 2
- lim × × æ1 æxö 3 3x ö
x®0 æ 2 p ö b ( 2 - ax ) 2 a ( 2 - bx ) 3 2m ç cos ç ÷ - sin ÷
sin ç ÷ - lim × a èm èm ø m m ø æ1ö
è 2 - bx ø x®0 b ( 2 - ax ) 3 - lim 2m ç ÷
=e =e =e b
= ex®0 1 = e
èm ø
= e2
Hence, (a) is the correct answer. Hence, (d) is the correct answer.
Chap 05 Limits 261

n æ n b - 1ö
æa - 1+ n b ö lim ç ÷ ×n
[1¥ form ]
y Example 52 The value of lim ç ÷ =e
n ® ¥è a ø
æ - 1ö
n ®¥ è a ø 1/n
b log b ç 2 ÷
1 b1/n - 1 1 èn ø
lim × lim ×
(a > 0, b > 0) is equal to =e n®¥a 1/n
=e
n®¥ a -1/n 2
a b
(a) b (b) a (c) b (d) a 1
log e b 1/ a

æa - 1 + b ö n æ
n
b - 1ö n
n = ea = e log e b = b1/a
Sol. Here, lim ç ÷ = lim ç1 + ÷ Hence, (a) is the correct answer.
n ®¥ è a ø n ® ¥ è a ø

Exercise for Session 3


e x - e x cos x
1. The value of lim is
x ® 0 x + sin x

(a) 0 (b) 1 (c) -1 (d) None of these


x -y
y x
2. The value of lim is
x ®y xx - yy
1 - log x 1 - log y log x - log y
(a) (b) (c) (d) None of these
1 + log x 1 + log y log x + log y

px - q x
3. The value of lim is
x ®0 r x - sx
1 - log p log p - log q log p × log q
(a) (b) (c) (d) None of these
1 + log p log r - log s log r × log s

4. The value of lim ( x + 2) tan-1 ( x + 2) - ( x tan-1 x ) is


x ®¥
p p
(a) (b) Doesn’t exist (c) (d) None of these
2 4
(cos a )x - (sin a )x - cos 2a æ pö
5. The value of lim , a Î ç0, ÷ is
x ®4 x -4 è 2ø
(a) log (cos a) + (sin a)4 log (sin a) (b) (cos4 a) log (cos a) - (sin a)4 log (sin a)
(c) (cos4 a) log (cos a) (d) None of these
a/ x
æ 1x + 2x + 3x + . . . + n x ö
6. The value of lim ç ÷ is
x ®0è n ø
(a) (n !)a / n (b) n ! (c) a n! (d) Doesn’t exist
7. If lim (1 + ax + bx ) 2 2/ x
= e , the values of a and b are
3
x ®0
3 1
(a) a = , b ÎR (b) a = , b ÎR (c) a ÎR, b ÎR (d) None of these
2 2
-a
8. If a and b are roots of ax 2 + bx + c = 0, the value of lim (1 + ax 2 + bx + c )2/ x is
x ®a
2a
( a -b )
(a) e 2 a( a - b ) (b) e a( a - b ) (c) e 3 (d) None of these
9. . ) + [(1 + 0.0001)
The value of lim ((15 n 10000 n 1/ n
] ) , where [×] denotes the greatest integer function, is
n®¥
1
(a) 1 (b) (c) Doesn’t exist (d) 2
2
10. The value of lim | x |[cos x ] , where [×] denotes the greatest integer function, is
x ®0

(a) 0 (b) Doesn’t exist (c) 1 (d) None of these


Session 4
Miscellaneous Forms
Miscellaneous Forms = lim
n ®¥
log e – 0
[by L’Hospital’s rule]
1
(i) 00 Form log A = 1
1/n
When lim f ( x ) ¹ 1 but f ( x ) is positive in the æen ö
x ®a Þ A = e or lim ç ÷
1
=e
n ® ¥è p ø
neighbourhood of x = a.
In this case, we write { f ( x )} g ( x ) = e loge {f ( x )}
g (x )
y Example 56 Evaluate lim (cosec x ) x .
x ®0
lim g ( x ) loge f ( x ) Sol. Let A = lim (cosec x )x [¥ 0 form]
Þ lim [ f ( x )]g ( x ) = e x ® a x ®0
x ®a
Þ log A = lim x log ( cosec x )
x ®0
sin x
y Example 53 Evaluate lim | x | . log ( cosec x ) é¥ ù
x ®0 = lim êë ¥ form úû
log e | x |
x ®0 1
lim
Sol. lim | x | sin x = lim e sin x log e | x | = e x ® 0 cosec x [0° form] x
x ®0 x ®0 1
× (– cosec x cot x )
1/ x cosec x
lim
x®0 - cos ec x cot x = lim
=e [by L’ Hospital’s rule] x ®0 1
2
– 2
sin x 2 æ sin x ö æ x ö x
lim - lim - ç ÷ ×ç ÷
x®0 è x ø è cos x ø [by L’Hospital’s rule]
=e x®0 x cos x =e x2
- (1) 2× ( 0 )
= lim =0
=e =e =10 x ® 0 tan x

\ log A = 0 or A = 1 Þ lim ( cosec x )x = 1


y Example 54 Evaluate lim ( pn ) 2/n . x ®0
n ®¥

Sol. Let A = lim ( pn )2 /n


n ®¥
[¥ 0 form] (ii) 10 Form
\ log A = lim
2 log ( pn ) é¥ ù (1 + x )1/x - e
n ®¥ n êë ¥ form úû y Example 57 Solve lim .
x ®0 x
1
2× ×p (1 + x )1/x - e
pn Sol. Let L = lim
= lim [by L’Hospital’s rule] x ®0 x
n ®¥ 1 ìï log (1 + x ) -1üï
2 e íe x ý
= lim =0
e log (1 + x )
1/x
n ®¥ n -e ïî ïþ
= lim = lim
\ loge A = 0 Þ A = 1 x ®0 x x ®0 x
1 /n e {e M - 1} log(1 + x ) - x
æen ö = lim × lim ,
y Example 55 Evaluate lim ç ÷ . x ®0 M x ®0 x2
n ®¥ è p ø
log(1 + x )
where M = -1
1/n x
æen ö
Sol. Let A = lim ç ÷ [¥ 0 form] 1
n ® ¥è p ø -1
e × {e M - 1} 1+ x
\ L = lim × lim
1 æen ö M ®0 M x ®0 2x
\ log A = lim log ç ÷
n ® ¥n èpø -x e
= e ´ 1 ´ lim = - [as x ® 0 Þ M ® 0]
x ® 0 2x ( 1 + x ) 2
n log e – log p é¥ ù
= lim êë ¥ form úû
n ®¥ n
Chap 05 Limits 263

Exercise for Session 4


1. Evaluate lim + (sin x )x . 2. Evaluate lim + (sin x )tanx .
x ®0 x ®0

æe n ö 1 /n
3. Evaluate lim ç ÷ .
n® ¥è p ø

e - (1 + x )1/ x
4. The value of lim is
x ®0 tan x
11e e
(a) e (b) (c) (d) None of these
24 2

Session 5
Left Hand and Right Hand Limits
Left Hand and Right Hand Limits Now, RHL of f ( x ) at x = 4
Let y = f ( x ) be a given function and x = a is the point = lim + f ( x ) = lim f ( 4 + h )
x ®4 h ®0
under consideration. Left tendency of f ( x ) at x = a is
| 4 + h – 4| |h |
called its left hand limit and right tendency is called its = lim = lim =1
h ®0 4 + h – 4 h ®0 h
right hand limit.
LHL of f ( x ) at x = 4
Left tendency (left hand limit) is denoted by f (a – 0 ) or
= lim - f ( x ) = lim f ( 4 – h )
f (a–) and right tendency (right hand limit) is denoted by x ®4 h ®0
f (a + 0 ) or f (a +) and are written as |4 – h – 4| |h | h
= lim = lim = lim = -1
h ® 0 – h h ® 0 -h
f (a – 0 ) = lim f (a – h ) ü h ®0 4 –h – 4
h®0 ï
ý Thus, RHL ¹ LHL. So, lim f ( x ) does not exist.
f (a + 0 ) = lim f (a + h )ï x ®4
h®0 þ
where, h is a small positive number. ì 5x – 4, 0 < x £ 1
y Example 59 If f ( x ) = í 3 , show that
Thus, for the existence of the limit of f ( x ) at x = a, it is î4 x – 3x , 1 < x < 2
necessary and sufficient that lim f ( x ) exists.
f (a – 0 ) = f (a + 0 ) , if these are finite or f (a – 0 ) and x ®1
ì5x – 4, 0 < x £ 1
f (a + 0 ) both should be either + ¥ or –¥. Sol. We have, f ( x ) = í 3
î 4 x – 3x , 1 < x < 2
y Example 58 Evaluate the right hand limit and left LHL of f ( x ) at x = 1 = lim - f ( x ) = lim f (1 – h )
x ®1 h ®0
hand limit of the function
= lim 5(1 – h )– 4 = lim 1 – 5h = 1
ì| x – 4 | h ®0 h ®0
ï ,x ¹4
f (x ) = í x – 4 . RHL of f ( x ) at x = 1 = lim + f ( x ) = lim f (1 + h )
ï0, x ®1 h ®0
î x = 4
= lim 4 (1 + h ) –3(1 + h ) = 4 (1)3 – 3 (1) = 1
3
ì| x – 4 | h ®0
ï ,x ¹4
Sol. Given, f ( x ) = í x – 4 Thus, RHL = LHL = 1. So, lim f ( x ) exists and is equal to 1.
ï0, x =4 x ®1
î
264 Textbook of Differential Calculus

e 1/x – 1
y Example 60 Show that lim does not exist. é sin x ù
x ®0 e 1/x + 1 y Example 62 Solve (i) lim [sin –1 x ] (ii) lim + ê
e 1/x
–1 x ®1 x ®0 ë x ú û
Sol. Let f (x ) = .
e 1/x
+1 é sin x ù
(iii) lim - ê
Then, LHL = lim - f ( x ) = lim f (0 – h ) x ®0 ë x ú û
x ®0 h ®0
e –1/h – 1 (where [ × ] denotes greatest integer function.)
= lim
h ®0 e –1/h
+1 Sol. (i) Here, L = lim [sin –1 x ]
x ®1
(1/e 1/h – 1) 0–1
= lim = = –1 Put sin x = t –1
h ®0 (1/e 1/h + 1) 0 + 1
p
1 \ x = sin t and t ® as x ® 1
[as h ® 0 Þ ® ¥ Þ e 1/h ® ¥ Þ 1/e 1/h ® 0 ] ...(i) 2
h
épù
RHL = lim + f ( x ) = lim f (0 + h ) Þ L = lim [t ] = ê ú = 1
x ®0 h ®0 t ® p/ 2 ë2û

e 1/h – 1 (1–1/e 1/h ) \ lim [sin –1 x ] = 1


= lim = lim x ®1
h ®0 e 1/h + 1 h ®0 (1 + 1/e 1/h )
é sin x ù
[dividing numerator and denominator both by e 1/h ] (ii) Here, L = lim + ê
x ®0 ë x ú û
1–0
= =1 [using Eq. (i)] sin x
1+0 Put = t Þ t ® 1– as x ® 0+ and [ t ] = 0
x
Clearly, lim - f ( x ) ¹ lim + f ( x )
x ®0 x ®0
Þ L = lim- [t ] = [1 – h ] = 0
t ®1
Hence, lim f ( x ) doesn’t exist. [as x ® 0 + h Þ t ® 1 – h ]
x ®0

1 – cos 2 ( x – 1) é sin x ù
(iii) Here, L = lim - ê
y Example 61 Evaluate lim . x ®0 ë x ú û
x ®1 ( x – 1)
sin x
[IIT JEE 1998] Put =t
x
1 – cos 2 ( x – 1)
Sol. We have, lim Þ t ® 1– as x ® 0–
x ®1 ( x – 1)
\ L = lim- [t ] = [1 – h ] = 0
2sin 2 ( x – 1) 2 |sin ( x – 1)| t ®1
= lim = lim [as x ® 0 – h Þ t ® 1 – h \ [t ] = 0]
x ®1 ( x – 1) x ®1 ( x – 1)
2 |sin ( x – 1)| y Example 63 Solve (i) lim [tan -1 x ]
\ LHL = lim - x ®¥
(ii) lim [tan -1 x ]
x ®1 ( x – 1)
x ®-¥
2 |sin (– h )| 2 sin h
= lim = lim =– 2 (where [ × ] denotes greatest integer function.)
h ®0 (– h ) h ®0 –h
Sol. (i) Here, lim [tan –1 x ]
2 |sin ( x – 1)| x ®¥
Again, RHL = lim +
x ®1 ( x – 1) p
Put tan x = t Þ t ®
–1
as x ® ¥
2
2 |sin (h )|
= lim épù
h ®0 h \ lim [ t ] = ê ú = 1
t ® p/ 2 ë2û
2 sin h (ii) Here, lim [tan –1 x ]
= lim = 2 x ®- ¥
h ®0 h
p
Clearly, lim f ( x ) ¹ lim - f ( x ) Put tan –1 x = t Þ t ® – as x ® – ¥
x ® 1+ x ®1 2
é pù
Hence, lim f ( x ) doesn’t exist. \ lim [t ] = ê– ú = [–1.57] = – 2
x ®1
t ® - p/ 2 ë 2û
Chap 05 Limits 265

y Example 64 Solve (i) lim + é


tan x ù Sol. (i) Here, lim [cot x ]
x ®0
x ®0 ë x ú
ê û Put cot x = t , now as x ® 0 ; cot x exhibits two values
é tan x ù for x ® 0+ and x ® 0–. i.e. cot x ® + ¥ and
(ii) lim - ê
x ®0 ë x ú û cot x ® – ¥, respectively.
(where [ × ] denotes greatest integer function.) \ We should apply right hand and left hand limit;
é tan x ù i.e. lim + [cot x ] = lim [t ] = ¥
Sol. (i) Here, lim + ê úû x ®0 t ®+ ¥
x ®0 ë x
[Q cot x = t Þ t ® + ¥ as x ® 0+ ]
tan x
Put = t Þ t ® 1+ as x ® 0+ and lim - [cot x ] = lim [t ] = – ¥
x x ®0 t ®- ¥

\ lim+ [ t ] = [1 + h ] = 1 [as x ® 0 + h Þ t ® 1 + h ] [Q cot x = t Þ t ® – ¥ as x ® 0–]


t ®1
é tan x ù \ Limit doesn’t exist.
(ii) Here, lim - ê úû
x ®0 ë x (ii) Here, lim [cot –1 x ] = lim+ [t ]
x ®+ ¥ t ®0
tan x
Put = t Þ t ® 1+ as x ® 0–
x [Q cot –1
x = t Þ t ® 0+ as x ® + ¥ ]
\ lim+ [ t ] = [1 + h ] = 1 [as x ® 0 – h Þ t ® 1 + h ] = lim [0 + h ] = lim 0 = 0
t ®1 h ®0 h ®0

y Example 65 Solve é | x |ù
-1 -1 y Example 67 Solve lim êsin ú , where [ × ] denotes
(i) lim- [sin (sin x )] (ii) lim [sin (sin x )] x ®0ë x û
x ®1 x ® p/ 2

(where [× ] denotes greatest integer function.) greatest integer function.


é | x |ù
Sol. (i) Here, lim - [sin (sin –1 x )] Sol. Here, lim êsin , since we have greatest integer
x ®1 x ®0ë x úû
= lim - [ x ] [as sin (sin –1 x ) = x , if –1 £ x £ 1] function, we must define function.
x ®1 é sin | 0 + h | ù
Now, RHL (put x = 0 + h ) = lim ê
= lim [1 – h ] [as x ® 1– Þ x = 1 – h ] h ®0 ë 0 + h úû
h ®0
sin h
We know that, ® 1 as h ® 0 but less than 1.
=0 h
\ lim [sin (sin –1 x )] = lim - [sin (sin –1 x )] é æ sin h ö ù
x ®1 x ®1 \ RHL = lim 0 = 0 êQ çè h ÷ø = 0 as h ® 0ú
h ®0 ë û
and no need to check for lim + [sin (sin –1 x )]
x ®1 é | 0 –h | ù
Again, LHL (put x = 0 – h ) = lim êsin ,
(ii) lim [sin (sin x )] = lim - [sin (sin x )]
–1 –1 h ®0 ë 0–h úû
p sin h
x ® x ®
p we know ® – 1 as h ® 0 but greater than –1.
2 2 –h
ép ù é æ sin h ö ù
= lim - [ x ] = lim ê – h ú = 1 \ LHL = lim – 1 = –1 êQ çè h ÷ø = - 1 as h ® 0 ú
p h ®0 ë 2 û h ®0
x ® ë û
2
Thus, Limit doesn’t exist, as RHL = 0 and LHL = –1.
Remark
é sin | x | ù
If lim [sin (sin–1 x )], it means you have to calculate only left
x ®1
y Example 68 Solve lim ê
x ®0 ë |x | û
ú , where [ × ] denotes
hand limit and not right hand limit as for x > 1, sin (sin–1 x ) is
not defined.
greatest integer function.
é sin | x | ù
Sol. Here, lim ...(i)
y Example 66 Solve (i) lim [cot x ] x ®0ê
ë | x | úû
x ®0
sin x sin | x |
(ii) lim [cot–1 x ] We know that, ® 1 as x ® 0 or ® 1 as x ® 0
x ®+¥ x |x |
from right or left,
(where [ × ] denotes greatest integer function.) sin| x |
i.e. at x = 0 + h or x = 0 – h , ® 1–
|x |
266 Textbook of Differential Calculus

é sin | x | ù sin | x | (a) -1 (b) 4


\ ê | x | ú =0 as | x | < 1
ë û (c) 5 (d) None of these
From Eq. (i) whether we find RHL or LHL Sol. We know, when x ® 0
x -x
é sin | x | ù é sin | x | ù Þ <1 Þ > -1
lim ê ú = lim 0 = 0 Þ lim ê ú, tan x tan x
x ® 0 ë |x | û x ®0 x ® 0 ë |x | û
- 2x
exists and is 0. Þ > -2
tan x
é - 2x ù é - 2x ù
= -2
y Example 69 lim ê ú , where [ × ] denotes
So, lim ê
x ® 0 ë tan x û x ® 0 ë tan x ú
û
greatest integer function is Hence, (d) is the correct answer.

Exercise for Session 5


1. The value of lim {1 - x + [ x - 1] + [1 - x ]} (where [ × ] denotes the greatest integral function) is
x ®1

(a) -1 (b) Doesn’t exist


(c) 1 (d) None of these
sin [ x ]
2. The value of lim (where [ × ] denotes the greatest integer function) is
x ®0 [x ]
(a) 1 (b) sin 1
(c) Doesn’t exist (d) None of these

3. The value of lim sin-1 {x } (where { × } denotes fractional part of x ) is


x ®0
p
(a) 0 (b)
2
(c) Doesn’t exist (d) None of these
é x2 ù
4. The value of lim ê
x ® 0 sin x tan x
ú (where [ × ] denotes the greatest integer function) is
ë û
(a) 0 (b) 1
(c) Doesn’t exist (d) None of these
Session 6
Use of Standard Theorems/Results,
Use of Newton-Leibnitz’s Formula in Evaluating
the Limits, Summation of Series Using Definite
Integral as the Limit

Use of Standard Theorem 2 :


Theorems/Results Limits of Trigonometric Functions
If x is small and is measured in radians, then
Theorem 1 : lim
sin x
= 1 = lim
x
Sandwich/Squeeze Play Theorem x ®0 x x ® 0 sin x

General The Squeeze principle is used on limit problems sin -1 x x


= lim x cosec x = lim = lim
where the usual algebraic methods, factorisation or x ®0 x ®0 x x ® 0 sin -1 x
algebraic manipulation etc., are not effective. However, it Proof Consider a circle with unit radius.
requires to “squeeze” our problem in between two other
simpler function, whose limits can be easily computed and Area of DOAP < Area of sector OAP < Area of DOAT
equal. Use of Squeeze principle requires accurate analysis,
T
indepth algebra skills and careful use of inequalities.
Statement If f , g and h are three functions such that Y
P

tan x
f ( x ) £ g ( x ) £ h ( x ) for all x in some interval containing
the point x = c and if 1
sin x
x
lim f ( x ) = lim h ( x ) = L X′
O cos x A
X
x ®c x ®c
1
Y
x2
h (x) =1+ –
2 Y′
Figure 5.11
y = g(x) (0, 1)
sin x x tan x x 1
x2 < < Þ 1< < [Q0 < x < p/ 2 ]
f(x) =1 – –
4
2 2 2 sin x cos x
sin x
XN O X Þ cos x < <1
x
YN Now, using Sandwich theorem,
Figure 5.10 sin x
lim cos x < lim+ <1
x ®0 x ®0 x
Then, lim g ( x ) = L
x ®c sin x
Obviously, we have lim+ =1
From the figure, note that lim g ( x ) = 1. x ®0 x
x ®0
sin y
Remark Put x = - y , lim =1
y ® 0- y
The quantity c may be a finite number, + ¥ or - ¥. Similarly, L may
also be finite number, + ¥ or - ¥. sin x
Hence, lim =1
x ®0 x
268 Textbook of Differential Calculus

sin x x y Example 72 Evaluate


Similarly, lim = 1 = lim = lim x cosec x
x ®0 x x ® 0 sin x x ® 0 æ n n n n ö
lim ç 2 + 2 + 2 + ... + 2 ÷.
sin -1 x x n ®¥ è n + 1 n + 2 n + 3 n + nø
= lim = lim …(i)
x ®0 x x ® 0 sin -1 x
n n n n
Sol. Let f (n ) = + + + ... +
Using Eq. (i), we can deduce n +1
2
n +2
2
n +3
2
n +n
2

tan x x
lim = 1 = lim = lim x cot x Note that f (n ) has n terms which are decreasing.
x ®0 x x ® 0 tan x x ®0
Suppose
tan -1 x x æ n n n n ö
= lim = lim h (n ) = ç 2 + 2 + 2 + ... + 2 ÷ , n terms
x ®0 x x ® 0 tan -1 x èn + 1 n + 1 n + 1 n + 1ø
n2
Important Results = [obviously f (n ) < h (n )]
n +1
2
sin x
The lim always approaches 1 from its left hand,
x®0 x and
i.e. 0.9999.... æ n n n n ö
g (n ) = ç 2 + 2 + 2 + ... + 2 ÷, n terms
Þ lim éê
sin x ù
= 0, where [ × ] denotes step up function. èn + n n + n n + n n +nø
x®0 ë x ú û
é æ sin x ö ù n2
êNote that çè xlim ÷ = 1ú = [obviously g (n ) < f (n )]
ë ® 0 x ø û n2 + n
tan x
Note that the lim
x®0 x
approaches 1 from RHS. Hence, lim g (n ) < lim f (n ) < lim h (n )
n®¥ n®¥ n®¥
Þ lim éê ù = 1, where [ × ] denotes step up function.
tan x
x®0 ë x ú û Since, lim g (n ) = 1 = lim h (n )
n ®¥ n ®¥

2 Hence, using Sandwich theorem, lim f (n ) = 1.


y Example 70 Evaluate lim x 3 cos . n ®¥
x ®0 x x éb ù
y Example 73 The value of the lim (a ¹ 0)
2 x ®0 a êë x úû
Sol. Here, lim x 3 cos
x ®0 x (where [ × ] denotes the greatest integer function) is
2 2
As, - 1 £ cos £ 1 Þ - x 3 £ x 3 cos £ x 3 for x > 0 (a) a (b) b
x x b b
2
(c) (d) 1 -
and x £ x cos £ - x for x < 0,
3 3 3 a a
x b éb ù b
2 Sol. Since, -1< ê ú £
Thus, lim x 3 cos = 0, as in both the cases limit is zero. x ëx û x
x ®0 x x
Now, we have two cases depending upon the value of .
a
x 2 (2 + sin 2 x ) x
Case I For > 0
y Example 71 Evaluate lim . a
x ®¥ x + 100
æb ö x éb ù x b x
x 2 (2 + sin 2 x ) Þ lim ç - 1÷ < lim ê ú £ lim ×
Sol. We have, lim x ®0 è x ø a x ®0 ë x û a x ® 0 x a
x ®¥ x + 100
x éb ù b
2x 2 x 2 (2 + sin 2 x ) 3x 2 Using Squeeze play theorem, we have = lim =
Now, £ £ , as 0 £ sin 2 x £ 1 x ®0 a êë x úû a
x + 100 x + 100 x + 100
x
2x 2 x 2 (2 + sin 2 x ) 3x 2 Case II For <0
\ lim £ lim £ lim a
x ® ¥ x + 100 x ®0 x + 100 x ® ¥ x + 100
æb öx éb ù x b x
Þ lim ç - 1÷ > lim ê ú ³ lim ×
x 2 (2 + sin 2 x ) x ®0 è x ø a x ®0 ë x û a x ® 0 x a
Þ lim =¥
x ®¥ x + 100 Using Squeeze play theorem, we have
é 2x 2 3x 2 ù x éb ù b
êQxlim = lim = ¥ú lim =
®¥ x + 100 x ®¥ x + 100 ë x úû a
x ®0 a ê
ë û
Hence, (c) is the correct answer.
Chap 05 Limits 269

y Example 74 Evaluate Use of Newton-Leibnitz’s Formula


[ x ] + [2x ] + [ 3x ] + ...+ [nx ]
lim
n ®¥ n2
, in Evaluating the Limits
Let us consider the definite integral,
where [ × ] denotes the greatest integer function. y (x)
I (x ) = ò f (t ) dt
f (x)
Sol. We know that, x – 1 < [x ] £ x
Newton-Leibnitz’s formula states that,
Þ 2x – 1 < [2x ] £ 2x d ìd ü ìd ü
Þ 3x – 1 < [3x ] £ 3x { I ( x )} = f { y ( x )} × í y ( x )ý – f { f ( x )} í f ( x )ý
.............................
dx îdx þ îdx þ
.............................
Þ nx – 1 < [nx ] £ nx y Example 75 Evaluate
\ ( x + 2x + 3x + ... + nx ) – n < [ x ] + [2x ] + ... + [nx ] æ 1 x 2 1 1 ö
£ ( x + 2x + ... + nx ) lim ç 5 ò e –t dt – 4 + 2 ÷.
x®0 è x 0 x 3x ø
xn (n + 1) n
x × n ( n + 1)
Þ – n < å [r x ] £ æ 1 x -t 2 1 1 ö
2 r =1 2 Sol We have, lim ç 5
x ®0 è x ò0 e dt -
x4
+ ÷
3x 2 ø
[ x ] + [2x ] + ...+ [nx ] x
Thus, lim 3 ò e –t dt – 3x + x 3
2
n ®¥ n2 0 é0 ù
= lim êë 0 form úû
xæ 1ö 1 [ x ] + [2x ] + ... + [nx ] x ®0 3x 5
Þ lim ç1 + ÷ – < lim
n ®¥ 2 è ø
n n n ® ¥ n2 d x
ò0
2

xæ 1ö 3 e –t dt – 3 + 3x 2
£ lim ç1 + ÷ = lim dx [by L’Hospital’s rule]
n ®¥ 2 è nø x ®0 15x 4
x [ x ] + [2x ] + ...+ [nx ] x Applying Newton-Leibnitz’s formula,
Þ < lim £
2 n ®¥ n2 2 d x –t 2 d d
dx ò 0
2 2
e dt = e – x × (x ) – e –0 ( 0) = e – x
[ x ] + [2x ] + ...+ [nx ] x dx dx
\ lim =
n ®¥ n2 2 d x 2
3 ò e –t dt – 3 + 3x 2 2

Aliter We know that, [ x ] = x – { x } dx 0 3e – x – 3 + 3x 2


\ lim = lim
n x ®0 15x 4 x ®0 15x 4
år x = [x ] + [2x ] + ...+ [nx ] é0 ù
r =1 êë 0 form úû
= x – { x } + 2x – { 2x } + ... + nx –{ nx }
2
= ( x + 2x + 3x + ...+ nx ) – ({ x } + { 2x } + ... + { nx }) – 3(2x )e – x + 6x
xn (n + 1) = lim [again, apply L’Hospital’s rule]
= – ({ x } + { 2x } + ... + { nx }) x ®0 60 x 3
2 2
–6 x ( e – x – 1 )
2
–(e – x – 1) æe –x 2 – 1ö
1 n x æ 1 ö { x } + { 2x } + ...+ { nx } = lim = lim =
1
ç ÷
\ 2 å [rn ] = ç1 + ÷ – lim ç –x 2 ÷
n r =1 2 è nø n2 x ®0 60x 3 x ®0 10x 2 10 x ® 0 è ø
Since, 0 £ { rx } < 1 1 1 é æe x - 1ö ù
n = ´1 = êQ lim ç ÷ = 1ú
\ 0£ å {rx } < n 10 10 êë x ®0 è x ø úû
r =1
x
x – ò cos t 2 dt
n
å {rx } y Example 76 Evaluate lim
0
.
r =1
Þ lim 2
=0 x®0 x 3 – 6x
n ®¥ n
x
x – ò cos t 2 dt
n n
å[rx ] å { rx } Sol. Let L = lim
0 é0 ù
r =1 x æ 1ö r =1 x ®0 3 êë 0 form úû
\ lim = lim ç1 + ÷ – lim x – 6x
n ®¥ n2 n ®¥ 2 è n ø n ® ¥ n2
Applying L’Hospital’s rule, we get
n
å[rx ] 1–
d x
ò 0 cos t dt
2
r =1 x L = lim dx
Þ lim =
n ®¥ n2 2 x ®0 3x 2 – 6
270 Textbook of Differential Calculus

Applying Newton-Leibnitz’s rule, 1 2n


where, a = lim = 0 and b = lim =2
d x n ®¥ n n ®¥ n

dx ò 0
(cos t 2 ) dt = cos ( x 2 ) × 1 – 0 = cos( x 2 ) 2n
1 r/n x
å
2
d x \ L = lim = ò0 dx
ò 0 cos t dt
n ®¥
1– 2 n r =1 1 + (r/n ) 2
1 + x2
dx 1 – cos ( x 2 )
\ L = lim = lim 2 2
x ®0 3x 2 – 6 x ® 0 3( x 2 – 2) = ( 1 + x )0 = 5 – 1
1 – cos 0 1 – 1 0
= = = =0 y Example 79 Evaluate
3 (0 – 2) 3 (–2) –6
æ n n n ö
x2 lim ç 2 2 + 2 + + ÷.
ò0 cos t 2 dt K
n ®¥ èn + 1 n + 22 n2 + n2 ø
y Example 77 Evaluate lim .
x®0 x sin x æ n n n ö
Sol. Let S = lim ç 2 + 2 + ... + 2 ÷
[IIT JEE 1997] n ® ¥ èn + 1 2
n +2 2
n + n2 ø
Sol. Applying Newton-Leibnitz’s rule, followed by L' Hospital’s n
n 1 n n2
rule, we get = lim
n ® ¥ r = 1n2
å +r2
= lim å
n ® ¥ n r = 1n2 + r 2
cos ( x 2 )2 × { 2x } - 0 2 cos x 4
lim = lim [dividing numerator and denominator both by n]
x ®0 x cos x + sin x x ®0 æ sin x ö
cos x + ç ÷
è x ø 1 n 1
2 cos 0 2 = lim
n ®¥ n
å 1 + (r/n )2
= = =1 r =1
cos 0 + 1 1 + 1
n
r 1
= x ; = dx ; lim å = ò
b
Replace
n n n ®¥r =1 a

Summation of Series Using where, a = lim


1
= 0 and b = lim
n
=1
Definite Integral as the Limit 1
n ®¥ n
n
1
x ®¥ n

The expression of the form, we get, S = lim


n ® ¥n
å 1 + (r/n )2
y (x) r =1
1 ær ö b
lim
n®¥ n
å f ç ÷ = ò f ( x ) dx
ènø a =ò
1 1
dx =[tan –1 x ] 10 = tan –1(1) – tan –1(0) =
p
r = f (x) 01 +x 2
4
where, (i) S is replaced by ò,
1/n
y Example 80 The value of lim æç n ö÷
n!
r is equal to
(ii) is replaced by x, n ®¥ èn ø
n
1 1 1
(iii) is replaced by dx, (a) (b) e (c) e 2 (d)
n e e2
f (x ) y (x ) 1/n
(iv) To obtain, a = lim and b = lim æ n! ö
n®¥ n n®¥ n Sol. Let A = lim ç n ÷
n ® ¥ èn ø

The value so obtained is the required sum of the given 1 æ n ( n - 1) ( n - 2) 3 2 1ö


series. \ log A = lim log ç × × ... × × ÷
n ®¥ n èn n n n n nø
1 2n r
y Example 78 Evaluate lim å . 1 n -1
æn - r ö
n ®¥ n
r =1 n + r
2 2 = lim
n ® ¥n
å log çè n ø
÷
r =0
[IIT JEE 1997] 1
1 2n
r = ò0 1 × log (1 - x ) dx = - 1
Sol. Let L = lim
n ®¥ n
å dividing numerator and
n -1
n2 + r 2
r =1 r 1 0
by n and by dx lim å =
b

denominator both by n, we get


[replace
n n n ®¥
r =0
òan
= 0,
, where

1 2n
r/n n -1
L = lim
n ®¥
å a = lim , b = lim
n ®¥ n ®¥ n
= 1]
n r =1 1 + (r/n )2
2n Þ loge A = - 1 Þ A = e -1
r 1
Put = x ; = dx ; lim å = ò
b

n n n ®¥r =1 a Hence, (a) is the correct answer.


Chap 05 Limits 271

Exercise for Session 6


1 2
1. The value of lim ([1 x + 12 ] + [22 x + 22 ] + . . . + [n 2x + n 2 ]) (where [ × ] denotes the greatest integer
n®¥ n3
function) is
x 1
(a) (b) x +
3 3
x 1
(c) + (d) None of these
3 3
ì [12 (sin x )x ] + [22 (sin x )x ] + . . . + [n 2 (sin x )x ]ü
2. The value of lim í lim ý (where [ × ] denotes the greatest integer
x ®¥ n®¥
î n3 þ
function) is
x sin x x
(a) + (b) + (sin x )x
3 3 3
1
(c) (d) 0
3
x
ò1 | t - 1| dt
3. The value of lim + is
x ®1 sin ( x - 1)
(a) 0 (b) 1
(c) Doesn’t exist (d) None of these
n 1/ n
æ Kö
4. The value of lim
n®¥
å log çè1 + n ÷ø is
K =1

(a) loge æç ö÷ (b) loge æç ö÷


e 4
è 4ø èe ø
(c) loge 4 (d) None of these
æ 1 1 1 1ö
5. The value of lim ç + + + ... + ÷ is
n ® ¥ è na na + 1 na + 2 nb ø

(a) log æç ö÷ (b) log æç ö÷


a b
èb ø èa ø
(c) log (ab ) (d) None of these
JEE Type Solved Examples :
Single Option Correct Type Questions

x sin (sin x ) - sin 2 x 2 tan K 1x


× K 1x
l Ex. 1 The value of lim equals 2 tan K 1x K 1x
x ®0 x 6
Þ lim = 1 Þ lim =1
x ® 0 tan K 2 x x ® 0 tan K 2 x
1 1 × K 2x
(a) (b) K 2x
6 12
2K 1
(c)
1
(d)
1 Þ = 1 Þ 2K 1 = K 2
18 24 K2
x sin (sin x ) - sin 2 x 1
Sol. (c) Here, lim l Ex. 3 Let f (q ) = {(1 + tan q ) 3 + ( 2 + tan q ) 3 + ...
x ®0 x6 tan q 2

sin -1(t ) × sin t - t 2 + (10 + tan q ) 3 } - 10 tan q.


= lim -1 6
[put sin x = t ]
t ®0 (sin t ) Then, lim f (q ) is equal to
sin -1
(t ) × sin t - t 2 t6 p-

= lim ×
(sin -1 t )6
2
t ®0 t6
(a) 170 (b) 166 (c) 165 (d) None of these
ì t 3 9 t 5 5t 7 ü ì t3 t5 ü Sol. (c) Here,
ít + + + + Ký × ít - + - Ký - t 2 (1 + tan q )3 1 + 3 tan q + 3 tan 2 q + tan 3 q
6 120 112 þ î 3! 5! = lim
= lim î þ lim
´1 p- tan q2
p- tan 2 q
t ®0 t6 q®
2

2
æ1 1 9 ö
t6ç - + ÷ + higher powers to t [(a + b )3 = a 3 + b 3 + 3a 2b + 3ab 2 ]
è 5! 6 × (3!) 120 ø
= lim æ 3 tan q + 1 ö
t ®0 t6 = lim 3 + tan q + ç ÷ = lim - (3 + tan q )
p - è tan 2 q ø p
1 9 1 1 q® q®
= + - = 2 2
120 120 36 18 é ù
êas lim æ 3 tan q + 1 ö = 0ú
ç ÷
æp ö ê p- è tan q ø
2 ú
log e cot ç - K 1 x ÷ êë 0 ® 2 úû
è4 ø
l Ex. 2 If lim = 1 , then
x ®0 tan K 2 x Þ lim f (q ) = lim [(3 + tan q ) + (3(2) + tan q )+K
p- p-
q® q®
(a) K 1 = K 2 (b) 2K 1 = K 2 2 2
+ (3(10) + tan q )] -10 lim tan q
(c) K 1 = 2K 2 (d) K 1 = 4K 2 p-

2
é æp öù
loge ê cot ç - K 1x ÷ ú = lim 3(1 +2 +3 +... +10) +10 × lim (tan q ) - 10 × lim (tan q )
ë è 4 øû p- p- p-
Sol. (b) Here, lim =1 q®
2

2

2
x ® 0 tan K 2 x
3 ´ 10 ´ 11
= = 165
é æp ö ù 2
log ê cot ç - K 1x ÷ - 1 + 1ú
ë è 4 ø û =1
Þ lim
x ®0 tan K 2 x l Ex. 4 For positive integers K =1, 2 , 3 , ..., n. Let Sk denotes
æ 2 tan K 1x ö the area of DOABK (where ‘O’ is the origin) such that
log ç1 + ÷ Kp
è 1 - tan K 1x ø ÐAOBK = , OA = 1 and OBK = K . The value of the
Þ lim =1 2n
x ®0 tan K 2 x n
× å SK is
1
æ 2 tan K 1x ö 2 tan K 1x lim
n ®¥ n2
log ç1 + ÷ K =1
è 1 - tan K 1x ø 1 - tan K 1x
Þ lim × =1 2 4 8 1
x ®0 2 tan K 1x tan K 2 x (a) (b) (c) (d)
1 - tan K 1x p 2
p 2
p 2
2p 2
Chap 05 Limits 273

Kp p p p
Sol. (a) Here, OB K = K and ÐAOB K = Let M = cos × cos 4 K cos n - 1
2n 23 2 2
1 æ Kp ö é 1 ù æ p ö æpö
\ SK = × (1) ( K ) sin ç ÷ using, D = ab sinq ú sin ç2n - 3 × n - 1` ÷ sin ç 2 ÷
2 è 2n ø êë 2 û =
è 2 ø
=
è2 ø
æ p ö æ p ö
Y 2n - 3 × sin ç n - 1 ÷ 2n - 3 × sin ç n - 1 ÷
BK è2 ø è2 ø
Kp 1
2n 3
K
\ Sn = lim 2 ´ ì2n - 3 × sin p ü ´ 1
í n - 1ý
A n ®¥
æ 1 ö
3
î 2 þ æpö
ç ÷ cosn ç n ÷
1 è 2ø è2 ø
X
O 3
æ p ö
1 K æ Kp ö ç sin n - 1 ÷ æ p ö 3 1
Then, L = lim × × sin ç ÷ = lim 2 × ç 2 ÷ ×ç ÷ ´
n ® ¥ n2 2 è 2n ø n ®¥
ç p ÷ è 4 ø n æpö
cos ç n ÷
1 K æp K ö 1 1 æp ö è 2n - 1 ø è2 ø
= × lim × sin ç × ÷ = × ò x × sin ç x ÷ dx
2n n ® ¥ n è2 n ø 2 0 è2 ø æpö 1 p
33
= 2ç ÷ × =
éæ 1 ù è 4 ø 1 32
ö
1 ê ç -2 px ÷ 2 1 px ú
= ê ç × x × cos
2 êç1 p4 2 ÷ +p ò0 cos 2 × dx ú x + 7 - 3 2x - 3
4244 3÷ ú l Ex. 6 The value of lim is
êë è 0 ø0 úû x ®2 3 x + 6 - 23 3 x - 5
1 é 2 2 æ px ö ù
1 33 34 54
2
= ê × × çsin ÷ ú= 2 (a) (b) (c) (d) None of these
2 êp p è ø
2 0ú p 23 23 25
ë û
x + 7 - 3 2x - 3 é0 ù
Sol. (b) We have, L = lim
l Ex. 5 If x ®2 3 x + 6 - 2 3 3x - 5 êë 0 form úû
æ pöæ pö æ pö x - 2 = t , such that x ® 2 Þ t ® 0
Sn = ç1 - tan 4 ÷ ç1 - tan 4 K ç1 - tan 4
4÷ ÷. Let
è 3ø è ø è 2n ø
2 2 (t + 9 )1/ 2 - 3(2t + 1)1/ 2
The value of lim Sn , is \ L = lim
n ®¥ (t + 8)1/ 3 - 2(3t + 1)1/ 3
t ®0
p 3
p 3
(1 + t /9 )1/ 2 - (2t + 1)1/ 2
3 é0 ù
(a) (b) = × lim êë 0 form úû
4 16 2 t ®0 æ t ö1/ 3
ç1 + ÷ - (1 + 3t )
1/ 3
p 3
p3
(c) (d) è 8ø
32 256
æ 1 tö æ 1 ö
Sol. (c) Here, ç1 + × ÷ - ç1 + × (2t )÷
3 è 2 9ø è 2 ø
æ pöæ pö æ pö = × lim ;
Sn = ç1 - tan 4 3 ÷ ç1 - tan 4 4 ÷ K ç1 - tan 4 n ÷ 2 t ®0 æ 1 t ö æ 1 ö
è 2 ø è 2 ø è 2 ø ç 1 + × -
÷ ç 1 + × ( 3t ) ÷
è 3 8ø è 3 ø
æ 2 p 2 p ö æ 2 p 2 p ö
ç cos 3 - sin 3 ÷ ç cos 4 - sin 4 ÷ [using (1 + x )n = 1 + nx , as x ® 0]
è 2 2 øè 2 2 ø
p pö t æ1 ö
æ -t ç - 1÷
K ç cos 2 n - sin 2 n ÷ 3 3 è 18 ø 34
è 2 2 ø = × lim 18 = =
= 2 t ®0 t - t 2 æ 1 ö 23
æ 4 p 4 p 4 p ö ç - 1÷
ç cos 3 × cos 4 K cos n ÷ 24 è 24 ø
è 2 2 2 ø
æ pö æ pö æ p ö f (x + a ) f ( x + 2a ) f ( x + 3a )
ç cos 2 ÷ × ç cos 3 ÷K ç cos n - 1 ÷
è 2 ø è 2 ø è 2 ø
= l Ex. 7 Let D ( x ) = f (a ) f ( 2a ) f (3a ) for
æ p p p ö
ç cos 3 × cos 4 K cos n ÷
4 4 4
è 2 2 2 ø f ¢ (a ) f ¢ ( 2a ) f ¢ (3a )
1 some real values differential function f and constant a,
2 1 D (x )
= × …(i) lim is equal to
4 p
3 x ®0 x
æ p p p ö
ç cos 3 × cos 3 K cos n - 1 ÷ cos 2n
è 2 2 2 ø (a) 0 (b) 1 (c) 2 (d) 3
274 Textbook of Differential Calculus

f (a ) f (2a ) f (3a ) (1 + h )a - a(1 + h ) + a - 1


lim
Sol. (a) Here, D (0) = f (a ) f (2a ) f (3a ) h ®0 h2
f ¢ (a ) f ¢ (2a ) f ¢ (3a ) æ a(a - 1) 2 ö
ç1 + ah + h + ...÷ - a - ah + a - 1
è 2! ø
D(x ) é0 ù = lim
Thus, lim êë 0 form úû h ®0 h2
x ®0 x
a(a - 1)
D ¢( x ) \ f (a ) = ; f (101) = 5050
= lim = D ¢ ( 0) [applying L’Hospital’s rule] 2
x ®0 1
D(x ) nx n +1 - (n + 1) x n + 1
\ lim = D ¢ ( 0) ...(i) l Ex. 9 lim , where n =100 is equal to
x ®0 x x ®1 (e x - e ) sin px
f (x + a ) f ( x + 2a ) f ( x + 3a )
5050 100 5050 4950
Given, D( x ) = f (a ) f ( 2a ) f ( 3a ) (a) (b) (c) - (d) -
pe pe pe pe
f ¢ (a ) f ¢ ( 2a ) f ¢(3a )
nx n ( x - 1) - ( x n - 1)
Using definition of differentiation of determinant, Sol. (c) Let l = lim
x ®1 (e x - e ) sin px
f ¢ ( x + a ) f ¢ ( x + 2a ) f ¢ ( x + 3a )
Put x = 1 + h , so that as x ® 1, h ® 0
D¢ ( x ) = f (a ) f ( 2a ) f ( 3a )
h × n (1 + h )n - {(1 + h )n - 1}
f ¢ (a ) f ¢ ( 2a ) f ¢ ( 3a ) \ l = - lim
h ®0 e (e h - 1) sin ph
f (x + a ) f ( x + 2a ) f ( x + 3a )
Þ l = - lim
+ 0 0 0 x ®1

f ¢ (a ) f ¢ ( 2a ) f ¢ ( 3a ) n × h (1 + n C 1h + n C 2h 2 + n C 3h 3 + ...) - (1 + n C 1h
f (x + a ) f ( x + 2a ) f ( x + 3a ) + n C 2h 2 + n C 3h 3 + K-1)
+ f (a ) f ( 2a ) f ( 3a ) æ e h - 1 ö æ sin ph ö
pe ( h 2 ) ç ÷ç ÷
0 0 0 è h ø è ph ø

[as a is constant Þ
d
(a ) = 0] n 2 - nC 2 é 2n 2 - n (n - 1) ù n2 + n n ( n + 1)
dx =- = -ê ú=- =-
pe ë 2p e û 2( pe ) 2( pe )
f ¢( x + a ) f ¢ ( x + 2a ) f ¢ ( x + 3a )
æ 5050 ö
\ D¢ ( x ) = f (a ) f ( 2a ) f ( 3a ) If n = 100 Þ l = - ç ÷
è pe ø
f ¢ (a ) f ¢ ( 2a ) f ¢ ( 3a )
f ¢ (a ) f ¢ ( 2a ) f ¢ ( 3a ) 1 2 n + 2 2 (n - 1) + 3 2 (n - 2 ) + ... + n 2 × 1
l Ex. 10 lim is
or D¢ (0) = f (a ) f ( 2a ) f ( 3a ) = 0 n ®¥ 13 + 2 3 + 3 3 + ... + n 3
f ¢ (a ) f ¢ ( 2a ) f ¢ ( 3a ) equal to
[Q R1 and R 3 are identical] 1 2 1 1
D(x ) (a) (b) (c) (d)
Hence, lim = D ¢ ( 0) = 0 3 3 2 6
x ®0 x Sol. (a) We have,
x a - ax + a - 1 12 n + 22 (n - 1) + 32 (n - 2) + ... + n 2 {n - (n - 1)}
l Ex. 8 Let lim = f (a ). The value of f (101) lim
n ®¥ Sn 3
x ®1 ( x - 1) 2
Numerator = n (12 + 22 + ... + n 2 )
equals
- {1 × 22 + 2 × 32 + 3 × 4 2 + ... + (n - 1)n 2 }
(a) 5050
n
(b) 5151 = n Sn 2 - S ( r - 1 ) × r 2
r =2
(c) 4950
n
= n Sn 2 - S ( r 3 - r 2 )
(d) 101
r =1
x a - ax + a - 1
Sol. (a) We have, lim
( x - 1)
x ®1 2 = n S n - [ S n 3 - S n 2 ] = ( n + 1 ) Sn 2 - S n 3
2

Put x = 1 + h , we have
Chap 05 Limits 275

æ b (bt - 1) - a (at - 1) ö
( n + 1 ) Sn 2 - Sn 3 lim ç ÷ b ln b - a ln a
\ l = lim t ® 0 çè t (b - a ) ÷
ø b-a
n ®¥ Sn 3
=e =e
4 (n + 1) n (n + 1) (2n + 1) 4 1
= lim - 1= - 1 = b
b 1
n ®¥ 6n (n + 1) n (n + 1) 3 3 ln bb - ln aa
ln
a a æ bb ö b - a
ln çç ÷÷
1

b-a b-a è aa ø æ bb ö b - a
12 n + 22 (n - 1) + ...+ n 2 × 1 =e =e =e = ç a÷
Aliter l = +1-1 èa ø
13 + 23 + 33 +...+ n 3
14442444 3
cot -1 ( x + 1 - x )
12 (n + 1) + 22 (n + 1) +...+ n 2 (n + 1) l Ex. 13 lim is equal to
l= -1 x ®¥ ìïæ 2 x + 1 ö x üï
Sn 3
sec -1 íç ÷ ý
( n + 1 ) Sn 2
l= -1 ïîè x - 1 ø ïþ
Sn 3
(n + 1) × n (n + 1) (2n + 1) 4 1 p
Þ lim -1= -1= (a) 1 (b) 0 (c) (d) None of these
n ®¥ n ( n + 1)
2 2
3 3 2
p
6× Sol. (a) As, lim x + 1 - x = 0 Þ cot -1 (0) =
4 x ®¥ 2
-1 -a
cot (x log a x ) æ 2x + 1 ö
x
p
l Ex. 11 The value of lim (a > 1) is Þ sec -1 ( ¥ ) =
lim ç ÷ ®¥
x ®¥ sec - 1 (a x log x a ) x ®¥ è x - 1 ø 2
equal to \ l =1
p
(a) 1 (b) 0 (c) (d) Doesn’t exist
2 l Ex. 14 lim cos ( p n 2 + n ) (when n is an integer) is
n ®¥
æ loga x ö
cot -1 ç a ÷ equal to
è x ø
Sol. (a) We have, lim ;
æ ax ö
x ®¥ (a) 1 (b) - 1 (c) 0 (d) Doesn’t exist
-1
sec ç ÷
è loga x ø Sol. (c) Let l = lim ± cos (np - p n + n ) 2
n ®¥
æ log x ö æ ax ö = lim ± cos ( p (n - n 2 + n ))
as lim ç aa ÷ ® 0 and ç ÷®¥ n ®¥
x ®¥ è x ø è loga x ø
æ ( p ( + n )) ö
[using L’ Hospital’s rule] = lim ± cos ç ÷
n ®¥ çn + n2 + n ÷
p è ø
\ l = 2 =1 æ ö
p ç ÷
np
2 = lim ± cos ç ÷
n ®¥ ç 1÷
çn + n 1 + ÷
l Ex. 12 Suppose that a and b (b ¹ a ) are real positive è nø
1/t æ ö
æ bt + 1 - at + 1 ö ç ÷
numbers, thene the value of lim ç ÷ is equal p p
t ®0 è b -a = lim ± cos ç ÷ = cos ® 0
ø n ®¥ ç 1 ÷ 2
ç1 + 1 + ÷
to è n ø
a ln b - b ln a b ln b - a ln a Aliter
(a) (b)
b -a b -a
We have, p n 2 + n
1
1/ 2
æ bb öb - a æ 1ö é 1 1 æ1 ö1 1 ù
(c) b ln b - a lna (d) ç a ÷ = pn ç1 + ÷ = np ê1 + + ç - 1÷ + ...ú
èa ø è nø ë 2n 2 è 2 ø 2 ! n 2
û
1 é 1 1 æ1 ö1 1 ù
æ bt +1 - at +1 ö t = p ên + + ç - 1÷ + ...ú
Sol. (d) Here, lim ç ÷ . ë 2 2 è 2 ø 2 ! n û
t ®0 è b -a ø
p é æ1 ö 1 1 ù p
As n ® ¥; × ê2n + 1 + ç - 1÷ + ...ú = (2n + 1)
Obviously, limit is of the form 1¥ . 2 ë è2 ø 2! n û 2
1 é bt + 1 - at + 1 ù æ bt + 1 - at + 1 - b + a ö
lim ê - 1ú lim ç
t ® 0 çè
÷
÷ æ pö
Hence, l = e
t ® 0 t êë b-a úû
=e
t (b - a ) ø \ lim cos ç(2n + 1) ÷ = 0
n ®¥ è 2ø
276 Textbook of Differential Calculus

(tan({ x } - 1)) sin { x } 1 - cos ax sin 2 ax 1


l Ex. 15 The value of lim , where { x } Now, - lim = - lim ×
x ®0 { x } ({ x } - 1) x ®0 sin bx2 x ®0 sin bx 1 + cos ax
2

denotes the fractional part function is a2


-
a2 2b 2
(a) 1 (b) tan 1 (c) sin 1 (d) Doesn’t exist =- , l =e
2b 2
(tan({ x } - 1)) sin { x }
Sol. (d) Let f ( x ) = lim n
r
x ®0 { x } ({ x } - 1) l Ex. 19 lim
n ®¥
å n 2 + n + r equals
tan (h - 1) × sin h tan ( - 1) r =1
lim f ( x ) = lim = = tan1
x ®0 +
h ®0 + h ( h - 1) -1 (a) 0 (b) 1/3 (c) 1/2 (d) 1
tan ((1 - h ) - 1) sin (1 - h ) sin 1 1 2 n
LHL = lim = = sin 1 Sol. (c) Let f (n ) = + + ... +
h ®0 ( 1 - h ) ( 1 - h - 1) 1 n2 + n + 1 n2 + n + 2 n2 + n + n
Hence, lim f ( x ) doesn’t exist. 1 2 n
x ®0 Consider g (n ) = + + ... +
n +n +n
2
n +n +n
2
n +n +n
2

Ex. 16 lim ( - ln({ x } + |[ x ]| )) {x} 1 + 2 + 3 + ...+ n n ( n + 1)


l is equal to = =
x ® 0- n 2 + 2n 2 (n 2 + 2n )

(a) 0 (b) 1 (c) ln2 (d) ln


1 g (n ) < f (n ) ...(i)
2 1 2 n
Similarly, h (n ) = 2 + + ...+ 2
Sol. (d) We have, lim = ( - ln ({ x } + |[ x ]| )){ x } n + n + 1 n2 + n + 1 n +n +1
x ®0 -
n ( n + 1)
= lim ( - ln ({0 - h } + |[ - h ]| )){ - h } =
x ® 0- 2(n 2 + n + 1)
æ1ö \ f (n ) < h (n ) …(ii)
= lim ( - ln (1 - h + 1))1 - h = - ln 2 = ln ç ÷
x ®0 - è2ø From Eqs. (i) and (ii), g (n ) < f (n ) < h (n )
1
2 + 2 x + sin 2 x But lim g (n ) = lim h (n ) = ×
l Ex. 17 lim is equal to n ®¥ n ®¥ 2
x ®¥ ( 2 x + sin 2 x ) e sin x 1
Hence, using Sandwich theorem, lim f (n ) = .
(a) 0 (b) 1 (c) - 1 (d) Non-existent n ®¥ 2
2 + 2x + sin 2x
Sol. (d) We have, lim
+ sin 2x )e sin x
x ®¥ (2x l Ex. 20 The value of lim ( n 2 + n + 1 - [ n 2 + n + 1 ]),
n ®¥
2 sin 2x
+2+ where [×] denotes the greatest integer function is
= lim x x , as x ® ¥
x ®¥ æ sin 2x ö sin x (a) 0 (b) 1/2 (c) 2/3 (d) 1/4
ç2 + ÷e
è x ø Sol. (b) We know that, n < n + n + 1 < n + 1. 2

2 1 [ n2 + n + 1] = n
Þ l = lim = oscillatory between and Hence,
x ®¥ 2 × e sin x e
n +1 1
1 \ l = lim ( n 2 + n + 1 - n ) = lim =
Þ Non-existent n ®¥ n ®¥ n +n +1 +n
2 2
e -1
2
l Ex. 18 The value of lim (cos ax ) cosec bx
is sin 2 ( x 3 + x 2 + x - 3 )
x ®0 l Ex. 21 lim has the value equal to
æ 8b 2 ö æ 8a 2 ö
x ®1 1 - cos ( x 2 - 4 x + 3 )
ç- ÷ ç- ÷
ç ÷ ç ÷
è a2 ø è b2 ø (a) 18 (b) 9/2 (c) 9 (d) None of these
(a) e (b) e
æ a2 ö æ b2 ö sin ( x + x + x - 3)
2 3 2
ç-
ç
÷
÷
ç-
ç
÷
÷
Sol. (a) We have, lim
(c) e
è 2b 2 ø
(d) e
è 2a 2 ø x ®1 1 - cos( x 2 - 4 x + 3)
sin 2 ( x 3 + x 2 + x - 3) ( x 3 + x 2 + x - 3) 2
= lim ×
Sol. (c) Let l = lim(cos ax )cosec
2
bx x ®1 ( x + x + x - 3)
3 2 2
1 - cos ( x 2 - 4 x + 3)
x ®0
( x 2 - 4 x + 3) 2 ( x 3 + x 2 + x - 3) 2
lim
x®¥
= (1) lim ×
Þ l =e cosec bx (cos ax - 1)
2 x ®1 1 - cos ( x - 4 x + 3)
2
( x 2 - 4 x + 3) 2
Chap 05 Limits 277

2
æ x 3 + x 2 + x - 3ö (a) 1 (b) 2
= (1) (2) lim ç ÷ = 2l
2
(c) 7 (d) None of these
x ®1 è x 2 - 4x + 3 ø

3x 2 + 2x + 1 log (1 + 7 f ( x )) - sin ( f ( x )) é0 ù
where, l = lim [using L’ Hospital’s rule] Sol. (b) Here, lim êë 0 formúû
x ®1 2x - 4 x ® ea 3f (x )
6
= = - 3 Þ l = 2 ( - 3)2 = 18 7 f ¢ ( x ) - {cos ( f ( x )) × f ¢ ( x )} {1 + 7 f ( x )}
-2 = lim
x ®e a 3 f ¢ ( x ) × {1 + 7 f ( x )}
l Ex. 22 The graph of function y = f ( x ) has a unique
[using L’Hospital’s rule]
tangent at (e a , 0 ) through which the graph passes, then
7 - cos ( f ( x )) {1 + 7 f ( x )} 7 - 1
log (1 + 7 f ( x )) - sin ( f ( x )) = lim = =2
lim equals x ® ea 3{1 + 7 f ( x )} 3
a
x ®e 3 f (x )

JEE Type Solved Examples :


More than One Correct Option Type Questions
3 x 2 + ax + a + 1 (a) lim f ( x ) = 1 (b) lim f ( x ) = 1
l Ex. 23 If f ( x ) = , which of the following x ® 0+ x ® 0-
x2 +x -2
æ ö
2
can be correct?
(c) cot -1 ç lim f ( x )÷ = 1 (d) None of these
(a) lim f ( x ) exists Þ a = - 2 èx ® 0 - ø
x ®1
ì tan 2 { x }
(b) lim f ( x ) exists Þ a = 13 ï 2 , x >0
ï x - [x ]
x ®- 2 2

(c) lim f ( x ) = 4 / 3 Sol. (a, c) We have, f ( x ) = í 1 , x =0


x ®1
ï { x } cot { x } , x < 0
(d) lim f ( x ) = - 1 /3 ï
x ®- 2
3x 2 + ax + a + 1 î
Sol. (a, b, c, d) Given, f ( x ) =
( x + 2) ( x - 1) tan 2 {h } tan 2 h
RHL = lim f ( x ) = lim = lim =1
As x ® 1, Dr. ® 0, hence as x ® 1, Nr. ® 0 x ® 0+ h ®0 h 2 - [h ]2 h ®0 h2
\ 3 + 2a + 1 = 0 Þ a = - 2 LHL = lim f ( x ) = lim { - h } cot { - h }
x ® 0- h ®0
As x ® - 2, Dr. ® 0, hence as x ® - 2, Nr. ® 0
\ 12 - 2a + a + 1 = 0 Þ a = 13 = lim (1 - h ) cot (1 - h ) = cot1
h ®0
3x - 2x - 1
2
æ ö
2
Now, lim f ( x ) = lim \ cot -1 ç lim f ( x )÷ = cot -1 ( cot 1 )2 = 1
x ®1 x ® 1 ( x + 2) ( x - 1)
èx ® 0- ø
( 3x + 1) ( x - 1) 4
= lim =
x ® 1 ( x + 2) ( x - 1) 3 lEx. 25 Given that the derivative f ¢ (a ) exists. Indicate
3x + 13x + 14
2
( 3x + 7 ) ( x + 2) 1 which of the following statement(s) is/are always true?
and lim = lim =-
x ®- 2 ( x + 2) ( x - 1) x ® - 2 ( x + 2) ( x - 1) 3 f (h ) - f (a )
(a) f ¢ (a ) = lim
h ®a h -a
ì tan 2 { x }
ï 2 , for x > 0 (b) f ¢ (a ) = lim
f (a ) - f (a - h )
ïï x - [ x ] 2
h ®0 h
l Ex. 24 Let f ( x ) = 1 , for x = 0, where [ x ] is
í f (a + 2 t ) - f (a )
ï { x } cot { x } , for x < 0 (c) f ¢ (a ) = lim
t ®0 t
ï
ïî f (a + 2 t ) - f (a + t )
(d) f ¢ (a ) = lim
the step up function and { x } is the fractional part function t ®0 2t
of x, then
278 Textbook of Differential Calculus

Sol. (a, b) Here, options (a) and (b) are true by definition. n
[(r + 1)x + 1)] - (rx + 1)
= lim å
= 0 (rx + 1) [(r + 1) x + 1]
Option (c) is false, as n ®¥
r
f ( a + 2t ) - f ( a )
lim = 2 f ¢ (a ) n
æ 1 1 ö
t ®0 t = lim
n ®¥
å çè rx + 1 - (r + 1) x + 1 ÷ø
f ( a + 2t ) - f ( a + t ) 1 1 r =0
and lim = [2 f ¢ (a ) - f ¢ (a )] = f ¢ (a )
t ®0 2t 2 2 éæ1 1 ö æ 1 1 ö ù
Hence, option (d) is false. êç - ÷+ç - ÷ ú
è 1 x + 1 ø è x + 1 2x + 1 ø
= lim ê ú
n n ®¥ ê æ 1 1 ö ú
x ê + ...+ ç -
l Ex. 26 Let f ( x ) = lim
n ®¥
å (rx + 1) {(r + 1) x + 1} . Then, ë è nx + 1 ( n + 1 ) x + 1
÷
ø
ú
û
r =0
é 1 ù ( n + 1) x
(a) f (0) = 0 (b) f (0) = x = lim ê1 - = lim
n ®¥ ë (n + 1)x + 1 úû n ® ¥ (n + 1)x + 1
(c) f (0+ ) = 1 (d) f (0- ) = 1
ì0, x = 0
Þ f (0) = 0, f (0+ ) = f (0- ) = 1.
n
x \ f (x ) = í
Sol. (a, c, d) Given, f ( x ) = lim
n ®¥
å (rx + 1) {(r + 1) x + 1} î1, x ¹ 0
r =0

JEE Type Solved Examples :


Passage Based Questions
Passage I n
Sol. (Q. Nos. 27 to 29)
(Q. Nos. 27 to 29) - sin h + tan h + cos h - 1
-0
2h 2 + ln (2 - h ) - tan h
ì 2
e { x } - 1, x >0 LHD = lim
ï h ®0 -h
ï sin x - tan x + cos x - 1 sin h tan h 1 - cos h
Let f ( x ) = í , x < 0, - + ´h
ï 2 x + ln( 2 + x ) + tan x h2
2
= lim h 2 h =0
ïî 0, x =0 h ®0 2h + ln (2 - h ) - tan h
2 2
where { } represents fractional part function. Suppose lines L1 eh - 1 - 0 eh - 1
and L 2 represent tangent and normal to curve y = f ( x ) at f ¢ (0+ ) = RHD = lim =h ´ =0
h ®0 h h2
x = 0 . Consider the family of circles touching both the lines
L1 º y = 0 and L 2 º x = 0
L1 and L 2 .
27. (a) ( x - r )2 + (y - r )2 = r 2 [family of circle]
l Ex.27 Ratio of radii of two circles belonging to this x 2 + y 2 - 2rx - 2ry + r 2 = 0
family cutting each other orthogonally is
Y
(a) 2 + 3 (b) 3
(c) 2 + 2 (d) 2 - 2

l Ex.28 A circle having radius unity is inscribed in the


triangle formed by L1 and L 2 and a tangent to it. Then, the
minimum area of the triangle possible is O X

(a) 3 + 2 (b) 2 + 3
Q 2(r1r 2 + r1r 2 ) = r12 + r 22
(c) 3 + 2 2 (d) 3 - 2 2
or 4r1r 2 = r12 + r 22
l Ex.29 If centres of circles belonging to family having 2
æ r2 ö æ r2 ö
equal radii r are joined, the area of figure formed is ç ÷ - 4 ç ÷ +1=0
è r1 ø è r1 ø
(a) 2r 2 (b) 4r 2
r 2 4 ± 12
(c) 8r 2 (d) r 2 Þ = =2± 3
r1 2
Chap 05 Limits 279

28. (c) 2 [ D 1 + D 2 + D 3 ] Such that each horizontal row is arithmetic progression and
each vertical column is a geometrical progression. It is
1 æ æp qö q ö é 1 ù
D =2´ ç cot ç - ÷ + cot + 1÷ êë using, 2 ab úû known that each column in geometric progression have the
2 è è4 2ø 2 ø
1 3
same common ratio. Given that a 24 = 1, a 42 = and a 43 = .
æp qö
cos ç - ÷ cos q 8 16
è4 2ø 2 +1
D= +
æ p q ö sin q n
S
sin ç - ÷
è4 2ø 2
l Ex.30 Let Sn = å a 4 j , nlim n
®¥ n2
is equal to
j =1
Y 1 1 1 1
p/2 – q (a) (b) (c) (d)
p/4 – q/2 4 8 16 32
Sol. (d) Here, a 43 = a 42 + d
è
r cot è4 – 2 è
p q

D1 3 1 1
Þ = +d Þ d =
è

16 8 16
[common difference of 4th row]
1 1 1
r \ a 41 = a 42 - d = - =
D3
8 16 16
r (r, r) 1 2 3 n
q/2
q \ a 41 = , a 42 = , a 43 = ,...,a 4n =
D2 q/2 16 16 16 16
O X Now, all elements of 4th row are known
r cot (q/2)
n
n ( n + 1) S 1
p
2 sin Sn = å a 4 j = ; lim n2 =
2 ( 16 ) n ®¥ n 32
4 j =1
Þ D =1+
q æp qö
2 sin × sin ç - ÷
2 è4 2ø l Ex.31 Let d i be the common difference of the elements in
n
2
D =1+ ith row, then å d i is
æ pö æpö
cos çq - ÷ - cos ç ÷ i =1
è 4ø è4ø
1 1 1 n +1
p (a) n (b) - (c) 1 - (d)
D is minimum, if denominators is maximum when q = , 2 2n + 1 2n
2n
4
4
2 2 Sol. (c) Also, a 24r 2 = a 44 =
D min = 1 + =1+ = 1 + 2 ( 2 + 1) = 3 + 2 2 16
1-
1 2 -1
4 1 é 1ù
2 Þ r2 = Þ r = êë common ratio of all GP is 2 úû
16 2
29. (b) Area = (2r )2 = 4r 2
Y é1 2 3 nù
ê2 ...
2 2 2ú
ê1 2 3 nú
ê 2 ... 2 ú
2r ê2 22 22 2 ú
\ A=ê1 2 3 n
X' X
... 3 ú
ê 23 23 23 2 ú
ê M M M M Mú
ê1 2 3 nú
Y' êë 2n ... n ú
2n 2n 2 ûn ´ n
Passage II 1
Now, di = common difference in i th row =
(Q. Nos. 30 to 32) 2i
Let A be n ´ n matrix given by 1æ 1ö
ç1 - n ÷
n
2 èn
2 ø =1- 1
1
éa 11 a 12 a 13 K a 1n ù \ ådi = å 2i =
êa a 22 a 23 K a 2n ú i =1 i =1 1-
1 2n
A = ê 21 ú 2
ê M M M ú
ëa n1 a n 2 a n3 ... a nn û
280 Textbook of Differential Calculus

n Y
l Ex.32 The value of lim
n ®¥
å a ii is equal to
i =1
1 1 Case IV
(a) (b) (c) 1 (d) 2 f (a )
4 2
n
1 2 3 n
åaii = S = 2 + 22
X
Sol. (d) Given, + + K+ …(i) O a
i =1 23 2n
1 1 2 3 n -1 n If f (a ) is an integer, the limit will exist in Case III and
Þ Sn = 2 + 3 + 4 + ...+ n + n + 1 …(ii)
2 2 2 2 2 2 Case IV but not in Case I and Case II. If f (a ) is not an
On subtracting Eq. (ii) from Eq. (i), we get integer, the limit exists in all the cases.
1 æ1 1 1 1ö n
Sn = ç + 2 + 3 + K+ n ÷ - n + 1 é 1ù
2 è 2 2 2 2 ø 2 l Ex.33 If f ¢ (1) = - 3 and lim ê f ( x ) - ú does not exist,
x ®1 ë 2û
1æ 1ö
ç1 - n ÷ (where [× ] denotes the greatest integer function), then
1 2è 2 ø - n Þ S = 2æ1 - 1 ö - n
Þ Sn = ç ÷
è 2n ø 2n
n
2 1-
1 2n + 1 (a) f (1) may be integer
2 1
n
(b) { f ( x )} = , " x Î R ({×} fractional part of x )
2 2n
\ lim å aii = lim 2 - n - n = 2 - 0 - 0 = 2
2
n ®¥ n ®¥ 2 2 (c) f (1) is not an integer
i =1
n (d) None of the above
Þ lim
n ®¥
åaii = 2 é
Sol. (c) lim f ( x ) -

does not exist, when f (1) is not an
x ®1 ê 2 úû
i =1
ë
Passage III integer, as f ¢ (1) = - 3, i.e. decreasing in the neighbour-
hood at x = 1.
(Q. Nos. 33 to 35)
To evaluate lim [ f (x )], we must analyse the f ( x ) in right é sin x ù
x®a l Ex.34 lim ê(1 - e x ) × (where [ × ] denotes the great-
hand neighbourhood as well as in left hand neighbourhood x ®0 ë | x | úû
of x = a. e.g. In case of continuous function, we may come est integer function), equals
across following cases.
(a) 0 (b) 1 (c) -1 (d) Doesn’t exist
Y
ì sin x
ï (1 - e ) × x , x > 0
x

Sol. (c) Let f ( x ) = í .


sin x
Case I f(a) ï ( e x - 1) × , x <0
î x
ì < 0, for x > 0
\ f (x ) = í Þ lim [ f ( x )] = - 1
X
î < 0, for x < 0 x ®0
O a
-1/(1- x )
Y é px ù
l Ex.35 lim êcosec ú is equal to (where [× ]
x ®1 ë 2û
denotes the greatest integer function).
Case II
(a) 0 (b) 1 (c) ¥ (d) Doesn’t exist
f(a)
æ px ö
Sol. (b) Let f ( x ) = cosec ç ÷
X è 2 ø
O a
æp ö
Y Þ f (1+ ) = lim cosec ç (1 + h )÷ > 1
h ®0 è2 ø
æp ö
and f (1- ) = lim cosec ç (1 - h )÷ > 1
Case III h ®0 è2 ø
f(a)
-1
é px ù é px ù 1 - x
a X \ lim ê cosec ú = 1 Þ lim ê cosec ú =1
O x ®1 ë 2 û x ®1 ë 2 û
JEE Type Solved Examples :
Statements I and II Type Questions
n Directions (Ex. Nos. 36 to 38) This section is based on l Ex. 37 Statement I lim lim {sin 2m n ! pn } = 0 m, n Î N ,
Statements I and II. Select the correct answer from the m ®¥n ®¥
codes given below. when x is rational.
(a) Statement I is true, Statement II is true; Statement II Statement II When n ® ¥ and x is rational n ! x , is integer.
is correct explanation for Statement I
b
(b) Statement I is true, Statement II is true; Statement II Sol. (a) When n ® ¥ and x is rational x = .
is not the correct explanation for Statement I q
(c) Statement I is true, Statement II is false p
Where p and q are integers and q ¹ 0 n ! x = n !´ is integer
(d) Statement I is false, Statement II is true q
as n ! has factor q, when n ® ¥
l Ex. 36 Also, when n ! x is integer, sin(n ! p x ) = 0.
sin(cot 2 x ) 1 Therefore, the given limit is zero.
Statement I lim = .
(p - 2x )
x ® p /2 2 2
ì sin x ü
sin q tan q l Ex. 38 Statement I If lim í f ( x ) + ý does not exist,
Statement II lim = 1 and lim = 1 , where q x ®0î x þ
q®0 q q®0 q
is measured in radians. then lim f ( x ) does not exist.
x ®0
æ e 1/ x - 1 ö
Statement II lim ç ÷ does not exist.
sin (cot 2 x ) cot 2 x p x ® 0 è e 1/ x + 1 ø
Sol. (d) lim × ; put x = -h
x ® p/ 2 cot x 2
( p - 2x ) 2
2 æ sin x ö
Sol. (b) If lim f ( x ) exist, then lim ç f ( x ) + ÷ always exist as
tan 2 h 1 x ®0 x ®0 è x ø
Þ lim =
h ® 0 4h 2 4 sin x
lim exists finitely.
\ Statement I is false and Statement II is true. x ®0 x
Hence, lim f ( x ) must not exist.
x ®0

JEE Type Solved Examples :


Matching Type Questions
1- y -1
l Ex. 39 Match the following. 1 æ 1 ö
1/ y lim
y ® 0 y (1 + y )
(A) Put x = , lim ç ÷ =e = e -1
Column I Column II y y ® 0 è1 + y ø
æ x ö
x
(p) e2 lim
sin y + cos y - 1
(A) lim ç ÷ equals y®0
x ® ¥ è1 + xø (B) lim (sin y + cos y ) 1/ y
=e y
=e
y ®0
x -1/ 2
æ 1 1ö (q) e
(B) lim ç sin + cos ÷ equals cos x - 1
x®¥ è x xø
lim
x ® 0 tan 2 x 2
×x 1
2 2 -
(C) lim (cos x )cot x
equals (r) e (C) e x =e 2
x®0

é æp öù
1/ x
(s) e-1 lim
tan (( p/ 4 ) + x ) - tan( p/ 4 )
lim tan ç + x÷ ú equals
x®0ê
(D)
ë è4 øû (D) e x ® 0 x

tan x [1 + tan (( p/ 4 ) + x )×1]


Sol. (A) ® (s); (B) ® (r); (C) ® (q); (D) ® (p) lim
x®0
=e x
= e2
282 Textbook of Differential Calculus

l Ex. 40 Match the following. æ sin 3 x ö


(C) lim [ln sin 3 x - ln ( x 4 + ex 3 )] = lim ln ç 3 ÷
x ® 0+ x ®0 è x (x + e )ø
Column I Column II
æ1ö
(A) lim ( x + x - x - x ) equals
(p) -2 = ln ç ÷ = - 1
x®¥
èe ø
sin 2x - 2 tan x (q) -1 æp ö
(B) The value of the lim is (D) tan(2p | sin q | ) = tan ç - | cos q | 2p ÷
x®0 ln (1 + x 3 ) è2 ø
(C) lim (ln sin 3 x - ln(x 4 + ex 3 )) equals (r) 0 p
x ® 0+ Þ 2p |sin q | = n p + - | cos q | 2p
2
(D) Let tan (2p | sin q |) = cot (2p | cos q |), (s) 1
p
where q Î R and f (x ) = (| sin q | + |cos q |) . x Þ 2p (|sin q | + | cos q | ) = np +
2
é 2 ù n 1
The value of lim ê ú equals (here, [×] Þ |sin q | + | cos q | = + …(i)
x ® ¥ ë f (x ) û 2 4
n 1
represents greatest integer function) Since, 1 £ |sin q | + | cos q | £ 2; 1 £ + £ 2
2 4
Sol. (A) ® (s); (B) ® (p); (C) ® (q); (D) ® (r) 4 £ 2n + 1 £ 4 2
(x + x ) - (x - x ) 3 4 2 -1
(A) lim Þ £n £
x ®¥ x+ x + x- x 2 2
2 x Thus, n = 2 is only possible value. Putting in Eq. (i),
= lim =1
x ®¥ x+ x + x- x 5
| sin q | + | cos q | = ,
sin 2x - 2 tan x x3 4
(B) lim lim
x ®0 x3 x ®0 log (1 + x 3 )
é æ 4 öx ù
2 sin x (cos x - 1)
2
2 tan x 3 g ( x ) = lim ê2 ç ÷ ú = 0
= lim × 1 = lim - = -2 x ®¥
êë è 5 ø úû
x ®0 cos x × x 3 x ®0 x3

JEE Type Solved Examples :


Single Integer Answer Type Questions
l Ex. 41 If lim ( x -3 sin 3 x + ax -2 + b ) exists and is equal l Ex. 42 For a certain value of c,
x ®0
lim [( x 5 + 7 x 4 + 2 ) c - x ] = l, is finite and non-zero.
to zero, the value of a + 2b is ……… . x ®-¥

sin 3 x a sin 3x + ax + bx 3 The value of 3c + l is ……… .


Sol. (6) We have, lim + + b = lim
x ®0 x 3
x 2 x ®0 x 3 Sol. (2) We have, lim [( x 5 + 7 x 4 + 2)c - x ]
x ® –¥
sin 3x æ c ö
3 + a + bx 2 æ 7 2 ö
3 x = lim çç x 5c ç1 + + 5 ÷ - x ÷÷
= lim for x ®- ¥
è è x x ø ø
x ®0 x2
existence of limit 3 + a = 0 Þ a = - 3 æ æ 7 2 ö
c ö
= lim x çç x 5c - 1 ç1 + + 5 ÷ - 1÷÷
sin 3x - 3x + bx 3 x ®- ¥
è è x x ø ø
\ l = lim
x ®0 x3 This will be of the form ¥ ´ 0 only, if
sint - t 1 1
= 27 × +b=0 [Q3x = t ] 5c - 1 = 0 Þ c = substituting c = , l becomes
t3 5 5
7 2
27 9 l = lim x [(1 + x )1/ 5 - 1], where x = + 5
=- +b=0 Þ b= x ®- ¥ x x
6 2 é x ù æ7 2 ö 1 7
= lim x ê1 + + ....- 1ú = lim x ç + 5 ÷ × =
Hence,
9
a + 2b = - 3 + 2 ´ = 6 x ®- ¥ ë 5 û x ® - ¥ è x x ø 5 5
2 1 7
[using L’ Hospital’s rule] Hence, c = and l = Þ 3c + l = 2
5 5
Chap 05 Limits 283

Ex. 43 Consider the curve y = tan -1 x and a point x 2 t dt


ò
l 1
l Ex. 44 If lim = ,
x ®0 0
æ pö (e x - 1 - x )
2a t
- + 104
19
A ç1 , ÷ on it. If the variable point Pi ( x i , y i ) moves on the
è 4ø 3 2
a
curve for i = 1 , 2 , 3 , K, n (n Î N ) such that then equals ........... .
r
æ 1 ö
2010
y r = å tan -1 ç ÷ and B ( x , y ) be the limiting position x 2t
m =1
è 2m 2 ø ò0 2a t
dt
- + 104
of variable point Pn as n ® ¥, the value of Sol. (1) Here, lim 3 2 =
1
reciprocal of the slope of AB will be …......... . x ®0 e -1- x
x
19
[using L’Hospital’s rule]
n
æ 1 ö
Sol. (2) Here, y = lim yn = lim
n ®¥ n ®¥
å tan -1 çè 2m 2 ÷ø Þ lim
2x
=
1
m =1
x ®0 2a x 19
- + 104 (e x - 1)
n
æ 2 ö
= lim
n ®¥
å tan -1 çè 1 + (2m + 1) (2m - 1) ÷ø 3 2
m =1 1 x 1
Þ 2 lim × lim x =
n
é (2m + 1) - (2m - 1) ù x ®0 2a x x ® 0 e - 1 19
= lim
n ®¥
å tan -1 ê 1 + (2m + 1)(2m - 1) ú 3
- + 104
2
m =1 ë û
n 1 1
Þ 2× ×1 =
= lim
n ®¥
å (tan -1(2m + 1) - tan -1(2m - 1)) 2a
+ 104
19
m =1
3
= lim (tan -1 3 - tan -1 1) + (tan -1 5 - tan -1 3) 2a a
n ®¥
Þ + 104 = 1444 Þ 2a = 4020 Þ =1
3 2010
+ ...+ (tan -1(2n + 1) - tan -1 (2n - 1))
= lim (tan -1 (2n + 1) - tan -1(1)) l Ex. 45 Evaluate lim log sin x sin 2 x .
n ®¥
x ® 0+
p p p
= - = log sin 2x
2 4 4 Sol. (1) Here, lim log sin x sin 2x = lim
x ® 0+ x ® 0+ logsin x
é pö é–¥ ù
Q B ® ê1, ÷ , i.e. coordinates of B approaches towards
ë 4ø ê – ¥ form ú
ë û
those of A. 1
× 2 cos 2x
Chord AB approaches to the tangent to y = f ( x ) at A. = lim
sin 2x
[using L’Hospital’s rule]
x ® 0+ 1
æd ö × cos x
\ Slope of AB = ç tan -1 x ÷ sin x
è dx ø at x =1
é ( 2x ) ù
æ 1 ö 1 ê sin(2x ) ú cos 2x
=ç ÷ = = lim ë û = lim
cos 2x
=1
è 1 + x 2 ø at x =1
2
x ® 0+ æ x ö x ® 0 + cos x
ç ÷ cos x
Þ (Slope of AB)-1 = 2 è sin x ø
é sin x ù
êëQ xlim = 1ú
®0 x û
Subjective Type Questions
l Ex. 46 Evaluate a , b, c and d, if which shows (n ! px ) is not an integral multiple of p and so,
cos (n ! px ) will lie between –1 and +1.
lim ( x 4 + ax 3 + 3 x 2 + bx + 2
x ® ¥ Thus, lim lim [1 + cos 2m (n ! px )]
m ®¥ n ®¥
– x 4 + 2 x 3 – cx 2 + 3 x – d ) = 4.
= lim lim [1 + (a value between - 1 and + 1)2m ]
m ®¥n ®¥
Sol. Here, lim ( x + ax + 3x + bx + 2
4 3 2
x ®¥
= lim [1 + 0] = 1 [as 0 < x < 1 Þ x ¥ = 0]
n ®¥
– x 4 + 2x 3 – cx 2 + 3x – d ) = 4 [ ¥ – ¥ form]
ì2, x ÎQ
lim [1 + cos (n ! px )] = í
2m
Rationalising Thus, lim
m ®¥ n ®¥ î1, x ÏQ
( a – 2) x 3 + ( 3 + c ) x 2 + ( b – 3) x + ( 2 + d )
lim =4
x ®¥ é x 4 + ax 3 + 3x 2 + bx + 2 ù 4 + 3a n
ê ú
l Ex. 48 If a 1 = 1 and a n + 1 = , n ³1 , show that
ê + x 4 + 2x 3 – cx 2 + 3x – d ú 3 + 2a n
ë û
a n + 2 > a n + 1 and if a n has a limit l as n ® ¥, then evaluate
Since, limit is finite, the degree of the numerator must be 2.
So, a – 2 = 0, i.e. a = 2. lim a n .
n ®¥
( 3 + c ) x 2 + ( b – 3) x + ( 2 + d ) a1 = 1
Þ lim =4 Sol Here,
x ®¥ é
x 4 + ax 3 + 3x 2 + bx + 2 ù 4 +3 7
ê ú \ a2 = = >1
êë + x 4 + 2x 3 – cx 2 + 3x – d úû 3 +2 5
\ a 2 > a1
On dividing numerator and denominator by x 2 , we get
Assuming an + 1 > an
(3 + c ) + (b – 3)/x + (2 + d )/x 2
lim 4 + 3an + 1 4 + 3an
x ®¥ a 3 b 2 2 c 3 d \ an + 2 – an + 1 = –
1 + + 2 + 3 + 4 + 1 + – 2 + 3– 4 3 + 2an + 1 3 + 2an
x x x x x x x x
3+c ( 4 + 3an + 1 ) (3 + 2an ) – ( 4 + 3an ) (3 + 2an + 1 )
=4 Þ =4 =
2 (3 + 2an + 1 ) (3 + 2an )
Þ c =5 an + 1 – an
\ c = 5, a = 2 = >0 [Qan + 1 > an ]
(3 + 2an + 1 ) (3 + 2an )
Hence, a = 2, c = 5 and b, d are any real numbers.
\ an + 2 – an + 1 > 0
l Ex. 47 If x is a real number in [0, 1]. Find the value of Þ an + 2 > an + 1
lim lim [1 + cos (n ! px )].
2m whenever an + 1 > an .
m® ¥ n® ¥
\ The sequence of values an is increasing and since a1 = 1,
Sol. If x is a real number in [0, 1], then we have two cases an > 0, for all n.
either.
Case I x Î Q [rational number] Now, let l = lim an = lim an + 1
n ®¥ n ®¥
1 1 1
As x Î Q , we have x = 0, , , , ..., 1 4 + 3an
2 3 4 \ l = lim
n ®¥ 3 + 2an
which shows (n ! px ) is integral multiple of p for large
values of n. 4 + 3l éQ lim a = l ù
Þ l=
ëê n ® ¥ úû
n
\ cos (n ! px ) = ± 1 3 + 2l
Thus, lim lim [1 + cos 2m (n ! px )] = lim lim (1 + 1) = 2 Þ 3l + 2l 2 = 4 + 3l
m ®¥ n ®¥ m ®¥ n ®¥
Þ l2 = 2
Case II x Ï Q [irrational number]
1 1 1 \ l= 2 [ neglecting l = – 2, as l > 0]
As x Ï Q , we have x = , , , ...
2 3 5
Chap 05 Limits 285

l Ex. 49 Let a 1 , a 2 , K, a n be sequence of real numbers l Ex. 50 A square is inscribed in a circle of radius R, a
with a n + 1 = a n + 1 + a n2 and a 0 = 0. Prove that circle is inscribed in this square then a square in this circle
and so on, n times. Find the limit of the sum of areas of all
æ a ö 4 the squares as n ® ¥.
lim ç n ÷ = .
n ® ¥ è 2n – 1 ø p Sol. Let the side of a square, S1 be ‘a’ units.
Sol. Here, an + 1 = an + 1 + an2 , let an = cot (a n )
R
Þ an + 1 = cot (a n ) + cosec (a n )
cos (a n ) + 1
Þ an + 1 =
sin (a n )
O
2 cos 2 (a n / 2) æa ö
= = cot ç n ÷
2 sin (a n / 2) cos (a n / 2) è 2 ø
S1
Putting n = 1, a1 = cot (a 1 ) C1
and a1 = a 0 + 1 + a 02 = 1
a
p Then, a 2 = 2R Þ R = is radius of circle C 1.
Þ cot (a 1 ) = 1 or a 1 = 2
4
If a1 be the side of another square, then
a
æ ö æpö
Again, a 2 = cot ç 1 ÷ = cot ç ÷ a1 2 = a Þ a1 =
a
è 2 ø è8ø
2
æa ö æ p ö a1 a
a 3 = cot ç 2 ÷ = cot ç ÷ a 2 2 = a1 Þ a 2 = = ×
è 2 ø è 4 × 22 ø 2 2
æa ö æ p ö ................................................
a 4 = cot ç 3 ÷ = cot ç ÷ ................................................
è 2 ø è 4 × 23 ø
................................................ So, sum of areas of all the squares,
................................................ Sn = a 2 + a12 + a 22 + . . . upto n terms
æ p ö a2 a2
an = cot ç ÷; = a2 + + + . . . upto n terms
è 4 × 2n – 1 ø 2 4
ì 1ü
Hence, 1– n ï
æ 1 1 1 ö 2 ï
æ p ö = a ç1 + + + + K upto n terms ÷ = a í 2 ý
2
cot ç ÷ è 2 4 8 ø 1
æ a ö è 4 × 2n – 1 ø 1 ï 1– ï
lim ç n ÷ = lim = lim î 2þ
n ® ¥ è 2n – 1 ø n ®¥ 2n – 1 x ®0 æp ö æ 1ö
tan ç x ÷ = 2a 2 ç1 – n ÷
è4 ø è 2 ø
x æ 1ö
\ lim Sn = lim 2a 2 ç1 – n ÷ = 2 a 2 = 4 R 2
é 1 ù n ®¥ n ®¥ è 2 ø
êë put 2n – 1 = xú
û é 1 ù
\
æ a ö 4
lim ç n n– 1 ÷ = êë as n ® ¥ Þ 2n ® 0úû
n ® ¥ è2 ø p
#L Limits Exercise 1 : Single Option Correct Type Questions
sin ( p cos 2 (tan (sin x ))) 8. Let f ( x ) be a real valued function defined for all x ³ 1,
1. lim is equal to
x ®0 x2 1
satisfying f (1) = 1 and f ¢ ( x ) = ; then
p x + ( f ( x ))
2
(a) p (b)
4
lim f ( x )
p x ®¥
(c) (d) None of these
2 (a) doesn’t exist
1 - ( 1 + t )t p
2. lim is equal to (b) exists and less than
t ® 0 ln (1 + t ) - t 4
p
1 1 (c) exists and less than 1 +
(a) (b) - 4
2 2 (d) exists and equal to 0
(c) 2 (d) - 2
9. The quadratic equation whose roots are the minimum
3. If I 1 = lim (tan -1 px - tan -1 x ) cos x and 1
x ®¥ value of sin 2 q - sin q + and lim ( x + 1) ( x + 2) - x is
2 x ®¥
I 2 = lim (tan -1 px - tan -1 x ) cos x , then ( I 1 , I 2 ) is
x ®0 (a) 3 x 2 - 7 x + 3 = 0 (b) 8 x 2 - 14 x + 3 = 0
(a) ( 0, 0 ) (c) x 2 - 7 x + 3 = 0 (d) 2 x 2 - 7 x + 3 = 0
(b) ( 0, 1 )
xn
(c) (1, 0 ) 10. If x 1 = 3 and x n + 1 = , " n Î N , then
(d) None of the above 1 + 1 + x n2
4. If f ( x ) = 0 is a quadratic equation such that lim 2n x n equal to
x ®¥
æpö 3p 2
3 2 2p 3p
f ( - p ) = f ( p ) = 0 and f ç ÷ = - , (a) (b) (c) (d)
è2ø 4 2p 3p 3 2
f (x ) x -b - a -b
then lim
x ® - p sin (sin x )
is equal to 11. lim- , (a > b ) is
x ®a (x 2 - a 2 )
(a) 0 (b) p 1 1
(c) 2p (d) None of these (a) (b)
4a a a -b
3 1 + sin 2 x - 4 1 - 2 tan x (c)
1
(d)
1
5. lim is equal to 2a a - b 4a a - b
x ®0 sin x + tan 2 x
(a) - 1 (b) 1 12. lim (sin n 1 + cos n 1)n is equal to
1 1 n ®¥
(c) (d) -
2 2 (a) cot 1 (b) tan 1 (c) cos 1 (d) sin 1
1 + xn 2/ x 2
6. If x n + 1 = and | x 0 | < 1, n ³ 0, then æ 2 ö
13. The value of lim ç 3 (sin -1 x - tan -1 x )÷ equals
2 x ®0 èx ø
æ 1 - x 02 ö 1 1
lim ç ÷ is equal to (a) e (b) e (c) (d)
n ® ¥ ç x 1 x 2 x 3 ... x n + 1 ÷ e e
è ø
n
n-K æ 4K ö
(a) - 1
(b) 1
14. The value of lim
n ®¥
å n 2
cos ç
è n ø
÷
k =1
(c) cos-1 ( x 0 + 1 ) equals
(d) cos-1( x 0 ) 1 1 1
(a) sin 4 + cos 4 -
x 4 16 16
7. For n Î N , let f n ( x ) = tan (1 + sec x ) (1 + sec 2x ) 1 1 1
2 (b) sin 4 - cos 4 +
f (x ) 4 16 16
(1 + sec 4 x )... (1 + sec 2n x ). Then, lim n is 1
x ®0 2x (c) (1 - sin 4 )
16
(a) 0 (b) 2n 1
(c) 2n - 1 (d) 2n + 1 (d) (1 - cos 4 )
16
Chap 05 Limits 287

1 - cos 2x × 3 cos 3x × 4 cos 4 x ... n cos nx 23. If f ( x ) = lim


1
, then the set of values
15. If lim has the n ®¥ 2n
x ®0 x 2
æ3 -1 ö
ç tan 2x ÷ +5
value equal to 10, the value of n is èp ø
(a) 6 (b) 7 (c) 8 (d) 9 of x for which f ( x ) = 0, is
z -1 (a) | 2 x | > 3 (b) | 2 x | < 3
16. lim
ò1/ 2 [ cot x ]dx
, where [×] denotes the greatest (c) | 2 x | ³ 3 (d) | 2 x | £ 3
z ®¥ é
z 1ù
ò1/ 2 êë1 + x úû dx 24. The integer ‘n’ for which the
x3
integer function, equals cos 2 x - cos x - e x cos x + e x -
(a) 0 (b) 1 lim 2 is a finite
(c) cot 1 (d) not defined x ®0 xn
17. If a and b are roots of x 2 - ( 1 - cos 2 q ) x + q = 0, where non-zero number, is
(a) 2 (b) 3
p æ 1 1ö
0<q < . Then, lim ç + ÷ is (c) 4 (d) None of these
2 q®0 è a
+ bø
tan -1 x sin -1 x
(a)
1
(b) - 2 25. If I 1 = lim -
x ®0 x x
2
(c) 2 (d) None of these sin -1 x tan -1 x
and I 2 = lim - ,
x ®0
1æ 5 ö x x
18. If f ( x ) = ç f ( x + 1) + ÷ and f ( x ) > 0, " x Î R,
3è f ( x + 2) ø where | x | < 1, which of the following statement is true?
then lim f ( x ) is (a) Neither I 1 nor I 2 exist
x ®¥ (b) I 1 exists and I 2 doesn’t exist
2 5 (c) I 1 doesn’t exist and I 2 exists
(a) 0 (b) (c) (d) ¥
5 2 (d) None of the above

19. Let f :(1, 2) ® R satisfies the inequality éx ù


êë 2 úû
cos (2x + 4 ) - 33 x 2 4x - 8 26. The value of lim is equal to
< f (x ) < , " x Î (1, 2). Then, x ® p/ 2 log (sin x )
2 x -2
(a) 0 (b) 1
lim f ( x ) is (c) -1 (d) doesn’t exist
-
x ®2
(a) 16 (b) -16 1öææ 1ö æ 1ö
27. The value of lim ç1 + ÷ ç1 + ÷ ... ç1 + ÷ , where
(c) 8 (d) doesn’t exist n ®¥ a1 ø è a 2 ø è an ø
è
20. Let f ( x ) be polynomial of degree 4 with roots 1, 2, 3, 4 a1 = 1 and an = n (1 + an – 1 ), " n ³ 2 , is
and leading coefficient 1 and g ( x ) be the polynomial of e
(a) e (b)
1 1 1 2
degree 4 with roots 1, , and with leading coefficient
2 3 4 (c) 2e (d) 3e
f (x ) 28. If f ( x + y ) = f ( x ) + f (y ) for all x , y Î R and f (1) = 1,
1. Then, lim equals
x ®1 g ( x )
2 f (tan x ) – 2 f (sin x )
1 1 1 then lim equals
(a) (b) -24 (c) (d) - x ®0 x 2 f (sin x )
24 12 12
(a) log 2 (b) log 4 (c) log 2 (d) log 8
4 2 - (cos x + sin x ) 5
21. The value of lim é 1 æ 1ö
29. The value of lim n –n ê(n + 1) æçn + ö÷ çn +
is 2

p 1 - sin 2x ÷
4 n ®¥ è 2ø è
ë 22 ø
(a) 2 (b) 3 5 n
(c) 5 2 (d) -5 2 æ 1 öù
... çn + n – 1 ÷ ú is
è 2 øû
n × 3n
1
22. If lim = , where n Î N , the
n +1 (a) e
- 2) + n × 3 - 3
n ®¥ n ( x n 3 n
(b) e 2
number of integer(s) in the range of ‘x’ is (c) e 3
(a) 3 (b) 4 (d) e 4
(c) 5 (d) infinite
288 Textbook of Differential Calculus

ì sin [x ] (a) x 2 (b) x 3 + 2 x 2


, for [x ] ¹ 0
30. If f ( x ) = ïí [x ] , where [x ] denotes the (c) - x + 2 x
2 3
(d) None of these
ï 0, for [x ] = 0
î 36. Let [ ] represents the greatest integer function less than
greatest integer less than or equal to x, then æ é n sin x ù é n tan x ù ö
or equal to x. The value of lim ç ê + ÷
lim f ( x ) equals
x ® 0
x ® 0 èë x úû êë x úû ø
(a) 1 (b) 0 is
(c) -1 (d) doesn’t exist (a) n + 1 (b) 2n
(c) n - 1 (d) 2n - 1
| x 3 – 6x 2 + 11x – 6 |
31. Let f ( x ) = , then the number of
37. The value of lim [ 2 – x + 1 + x ] , where a Î éê0, ùú and
1
x 3 – 6x 2 + 11x – 6
x ®a ë 2û
solutions of ‘a’, where lim f ( x ) doesn’t exist is
(a) 3 (b) 2 x®a
(c) 1 (d) 4 [ × ] denotes the greatest integer function is
(a) 1 (b) 2
32. Let the rth term, t r of a series is given by (c) 3 (d) 4
n
r
tr =
1+r2 +r 4
. The value of lim
n ®¥
åt r is 38. The value of
r =1 (tan x – sin x )
1 1
(a) 2 (b) (c) 1 (d) –1 + (tan x – sin x ) + (tan x - sin x )
2 4 +
+ . .. ¥
æ 3 1ö
n çr – r + ÷ lim
x®0 +
is
33. The value of lim
n ®¥
å cot –1
ç
ç 2
r ÷ is
÷
–1 + x 3 + x 3 + x 3 + ... ¥
r =1 1 1
è ø (a) (b)
2 4
p 1
(a) p (b) (c) (d) 1
4 8
p
(c) (d) p 39. The value of
2
cos 2 (1 – cos 2 (1 – cos 2 (1 ... cos 2 q ))... )
34. Let (tan a ) x + (sin a ) y = a and lim is
q ®0 æ p ( q + 4 – 2) ö
(a cosec a ) x + (cos a ) y = 1 be two variable straight sin çç ÷÷
lines, a being the parameter. Let P be the point of è q ø
intersection of the lines. In the limiting position, when (a) 2 (b) 2
a ® 0, the point of intersection of straight lines is (c)
1
(d)
1
(a) (2, - 1 ) (b) (2, 1 ) 2 2
(c) ( -2, 1 ) (d) ( -2, - 1 )
40. The value of lim an when an + 1 = 2 + an ,
35. The polynomial of least degree, such that n ®¥
1/x
æ x + f (x )ö 2 n = 1 , 2 , 3 , K is
lim ç1 + ÷ = e 2 is (a) 1 (b) 2
x ®0 è x2 ø (c) 3 (d) 4

Limits Exercise 2 : More than One Option Correct Type Questions


æp x + 1ö 1 - cos ( x 2 )
41. If lim 4 x ç - tan -1 ÷ = y + 4y + 5, then y can
2
42. lim is equal to
x ®¥ è4 x + 2ø x ®0 x 3 ( 4 x - 1)
be equal to 1
(a) ln 2 (b) ln2
(a) 1 (b) - 1 2
(c) - 4 (d) - 3 1 æe2 ö
(c) ln 4 (d) 1 - ln ç ÷
2 è4ø
Chap 05 Limits 289

43. If f ( x ) = e [cot x ] , where [y ] represents the greatest where [×] denotes the greatest integer less than or equal
integer less than or equal to y, then to x, then
1 (a) lim f ( x ) = sin 1
(a) lim f ( x ) = 1 (b) lim f ( x ) =
p +
p + e x ® 0-

2

2
(b) lim f ( x ) = 0
x ® 0+
1
(c) lim f ( x ) = (d) lim f ( x ) = 1 (c) limit does not exist at x = 0
p- e p-
x® x® (d) limit exists at x = 0
2 2
50. lim f ( x ) does not exist when
44. lim éêm
sin x ù x ®c
is equal to (where m Î I and [ × ] denotes
x ®0 ë x úû (a) f ( x ) = [[ x ]] - [2 x - 1 ] , c =3 (b) f ( x ) = [ x ] - x, c =1
greatest integer function) tan(sgn x )
(c) f ( x ) = { x } 2 - { - x } 2, c = 0 (d) f ( x ) = , c =0
(a) m, if m < 0 (sgn x )
(b) m - 1, if m > 0 (where [×] and {×} denotes greatest integer and fractional
part of x)
(c) m - 1, if m < 0
(d) m, if m > 0 51. Identify the correct statement.
é n 1ù
45. If lim (1 + ax + bx 2 ) 2 / x = e 3 , then (a) lim ê S r ú = 1
x ®0 n ®¥ ë r = 1 2 û

(a) a = 3, b = 0
3
(b) a = , b = 1 (b) If f ( x ) = ( x -1 ) { x }, where [×] and {×} denotes greatest integer
2 function and fractional part of x respectively, the limit of
3 f ( x ) does not exist at x =1
(c) a = , b = 4 (d) a = 2, b = 3
2 é tan x ù
(c) lim ê
x ®0 ë x û
+ ú =1
46. The graph of the function y = f ( x ) is shown in the
é tan x ù
adjacent figure, then correct statement is (d) ê lim =1
ë x ® 0 + x úû
Y
a cot x - a cos x
2 52. For a > 0, let l = lim and
x ® cot x - cos x
p
(–1, 1) 1 2

4 m = lim ( x 2 + ax - x 2 - ax ), then
X x ®-¥
–1 0 1 2
(a) l > m, for all a > 0 (b) l > m, only when a ³1
(c) l > m, for all a > e -a (d) el + m = 0
x=3
x
æ ax + 1 ö
53. Consider the function f ( x ) = ç ÷ , where a, b > 0 ,
(a) lim f ( x ) = 1 (b) lim f ( x ) = 2 è bx + 2 ø
x ® 0+ x ®1
(c) lim f ( x ) = does not exist (d) lim f ( x ) = 4 the lim f ( x ) is
x ®3 x ®4 x ®¥
(a) exists for all values of a and b
æ1ö (b) zero for a < b
cot -1 ç ÷
èx ø (c) non-existent for a > b
47. For lim
x ®0 x (d) e -(1/a ) or e -(1/b ) , if a = b
(a) RHL exists (b) LHL does not exist x ×2x - x æ log 2 ö
(c) limit does not exist as RHL is 1 and LHL is -1 54. If f ( x ) = and g ( x ) = 2 x × sin ç x ÷, then
1 - cos x è 2 ø
(d) limit does not exist as RHL and LHL both are non-existent
x (a) lim f ( x ) = log 2 (b) lim g( x ) = log 4
æ x + 1ö x ®0 x ®¥
48. If l = lim ç ÷ , the value of {l } and [l ] are
x ®¥ è x - 1 ø
(c) lim f ( x ) = log 4 (d) lim g( x ) = log 2
x ®0 x ®¥
(a) 7 (b) 7 -e 2 (c) -7 (d) e 2 - 7
x 3 + cx 2 + 5x + 12
ì sin [x ] 55. If lim = l (finite real number), then
,[x ] ¹ 0 x 2 - 7 x + 12
49. If f ( x ) = ïí [x ]
x ®3
,
ï0, (a) l = 4 (b) c = - 6
î [x ] = 0 (c) c = 4 (d) x Î R
Limits Exercise 3 : Passage Based Questions
1/n
Passage I (Q. Nos. 56 to 58) æ K1 ö
If lim f ( x ) = 1 and lim g ( x ) = ¥,then
62. The value of lim ç ÷ is
n ®¥ èK2 ø
x ®a x ®a
lim ( f ( x ) - 1) ´ g ( x ) 4 1
(a) e 2 (b) log 4 - 1 (c) (d)
lim { f ( x )} g (x ) = e x ® a . e e
x ®a
sin x
æ K1 ö
56. lim æç
sin x ö x - sin x 63. The value of lim ç ÷ is
÷ is equal to n ®¥ èK2 + K3 ø
x ®0 è x ø

(a) 1/e (b) - 1/e (a) e (b) 1


(c) e (d) - e (c) 0 (d) does not exist
1
æ x - 1 + cos x ö x Passage IV (Q. Nos. 64 to 65)
57. lim ç ÷ is equal to
x ®0 è x ø Let f : N ® R and g : N ® R be two functions and
1 f (1) = 0.8, g (1) = 06
. ; f (n + 1) = f (n ) cos( g (n )) - g (n ) sin( g (n ))
(a) e1/2 (b) e -1/2 (c) e1 (d) and g (n + 1) = f (n ) sin( g (n )) + g (n )cos( g (n )) for n ³ 1.
e
2
64. lim f (n ) is equal to
æa x + b x + c x ö x n ®¥
58. lim ç ÷ is equal to
x ®0 è 3 ø (a) -1 (b) 0
(c) 1 (d) does not exist
(a) a 2/3 + b 2/3 + c 2/3 (b) abc
(c) (abc ) 2/3 (d) 1 65. lim g (n ) is equal to
n ®¥
(a) -1 (b) 0
Passage II (Q. Nos. 59 to 61) (c) 1 (d) does not exist
n
æ xö
Let f ( x ) = lim ç cos ÷ , g ( x ) = lim (1 – x + x e ) .
n n
Passage V (Q. Nos. 66 to 68)
n ®¥ è nø n ®¥
A circular arc of radius 1 subtends an angle of x radian as shown
Now, consider the function y = h ( x ), where in figure. The centre of the circle is O and the point C is the
h ( x ) = tan - 1 ( g - 1 f - 1( x )). intersection of two tangent lines at A and B. Let T ( x ) be the area
ln ( f ( x )) of DABC and S( x ) be the area of shaded region.
59. lim is equal to C
x ®0 ln ( g ( x ))
1 1
(a) (b) - B
2 2
(c) 0 (d) 1
60. Domain of the function y = h( x ) is A
(a) ( 0, ¥ ) (b) R x
(c) ( 0, 1 ) (d) [ 0, 1 ] O
61. Range of the function y = h( x ) is T (x )
66. lim is
æ pö æ p ö x ®0 x3
(a) ç 0, ÷ (b) ç - , 0 ÷
è 2ø è 2 ø 1 1 1 1
(a) (b) (c) (d)
æ p pö 2 3 4 8
(c) R (d) ç - , ÷
è 2 2ø S( x )
67. lim is
x ®0 x
Passage III (Q. Nos. 62 to 63) 1
(a) 0 (b) (c) 1 (d) None of these
Let K 1 = Total number of ways of selecting of ball from a bag 2
which contains n balls of first colour, (n + 1) balls of second colour, 68. lim
T (x )
is
(n + 2) balls of third colour, …, (2n - 1) balls of n colour. x ® 0 S( x )
K 2 = number of n-digit numbers using the digits 1, 2, 3, …, n and 1 3 3
(a) (b) (c) (d) 0
K 3 = number of ways of arranging (n + 1) objects on a circle. 4 4 2
Chap 05 Limits 291

Limits Exercise 4 : Matching Type Questions


69. Match the statements of Column I with values of Column II. 71. Match the column.
Column I Column II Column I Column II
(A) lim tan -1 (tan x ) (p) 0 (A) 1 - cos 2 x (p) 1
p+
lim equals
x®0 x2

2 e -e +xx

é n 1ù æ (3 / x ) + 1ö
1/ x
(q) 2
lim ê å r ú ([ × ] denotes the
(B) (q) Doesn’t exist (B) If the value of lim ç ÷ can be
n® ¥ ê + è (3 / x ) - 1 ø
ë r = 1 2 úû x®0

greatest integer function) expressed in the form of e p / q , where p and


æ x ö (r) p q are relative prime, then ( p + q) is equal to
(C) lim sec- 1 ç ÷ -
x®¥ è x + 1ø 2 tan 3 x - tan x 3 (r) 4
(C) lim equals
cos x (s) p x®0 x5
(D) lim

p (1 - sin x )2/ 3 2 x + 2 sin x (s) 5
2 (D) lim
x®0 x + 2 sin x + 1 - sin 2 x - x + 1
2

70. A right angled triangle has legs 1 and x. The hypotenuse equals
is y and the angle opposite to the side x is q. Shown as
72. Match the column.
θ
Column I Column II
y
1
(A) lim cos (p (3 n + n + 2 n - n)),
2 3 2
(p) 1
n® ¥
2
where n is an integer, equals
x
Match the column. (B) lim n sin (2 p 1 + n2 ) (n Î N ) (q) 1
n® ¥
4
Column I Column II equals
(A) lim ( y - x ) (p) 0 (C) lim (- 1)n sin (p n2 + 0. 5n + 1) (r) p
p
q® n® ¥
2

(B) lim ( y - x ) (q) 1 æ (n + 1) p ö



p ç sin ÷ is (where n Î N )
2 2 è 4n ø
(C) lim ( y2 - x 2 ) (r) 1 x
p æ x + aö
q® (D) If lim ç ÷ = e, where a is some (s) non
x ® ¥ è x - aø
2
-existent
(D) lim ( y3 - x 3 ) (s) ¥ real constant, the value of a is equal to
p

2

Limits Exercise 5 : Single Integer Answer Type Questions


æ æ 1 öö 75. Let f (n ) = éê n + ùú, where [×] denotes greatest integer
1
73. Let L = lim ç x log x + 2x × log sin ç ÷ ÷, then the
è
x ®¥ è x øø ë 2û
æ 2ö function, " n Î N.
value of ç - ÷ is ………… .
è Lø ¥ - f (n )
(n + xn ) 2 f (n ) + 2
74. For n Î N , let x n be defined as æç1 + ö÷
1
= e , then
Then, å 2n
is equal to ……… .
è nø n =1
2 lim ( x n ) equals ………… .
n ®¥
292 Textbook of Differential Calculus

76. If the arithmetic mean of the product of all distinct pairs æ1ö 3
(3x 4 + 2x 2 ) × sin ç ÷ + x + 5
n 2 èx ø k
of positive integers whose sum is n, is S n , then lim 85. If lim 3 2
= k , then is
x ® -¥
n ® ¥ Sn x + x + x +1 2
must equal to ………… .
……........ .
¥
6n
77. If k = å , the value of k is ………… . é 2x æ x öù
(3n - 2n )(3n +1 - 2n +1 ) 86. If f ( x ) = lim ê × tan -1 ç 2 ÷ ú, then f (1) is ……...... .
n =1 t ®0 ë p èt øû
tan x – sin {tan –1 (tan x )} æ 1 ö
78. The value of lim is ……… . 1 1
p tan x + cos 2 (tan x ) 87. The value of lim ç + +¼+ ÷ is
n ®¥ ç
n 2 + 2n ÷ø

2 è n
2
n2 +1
79. The figure shows two regions in the first quadrant. A(t) 1

is the area under the curve y = sin x 2 from 0 to t and 1- x x sin ( x - [x ])


88. If l = lim+ 2 -2 and m = lim (where
B (t ) is the area of the triangle with vertices 0, P and x ®1 x -1x ®1+

A (t ) m [×] denotes greatest integer function). Then, (l +m ) is …… .


M (t , 0). If lim = , then (m + n ) is ………… .
t ® 0 B (t ) n é sin x × tan x ù
89. The value of lim ê ú is …….......
Y Y x ®0 ë x2 û
P(t, sin t2) P(t, sin t2)
(where [×] denotes greatest integer function).
x2

n kr
90. If lim S = 1, then k 2 is
sin

n ®¥ r =1 1 ´ 3 ´ 5 ´ ¼ ´ (2r - 1) ´ (2r + 1)
y=

A(t) B(t)
……...... .
X X
O t O M (t, 0) 1 1
91. If f ( x ) = lim sin 4 x + sin 4 2x + ¼ + × sin 4 (2n x ) and
æ 2t sin x ö ö
n ®¥ 4 4n
80. If the two lines AB : ç ò æç + 1÷ dx ÷ x + y = 3t and
è0 è x ø ø g ( x ) is a differentiable function satisfying g ( x ) + f ( x ) = 1,
AC : 2tx + y = 0 intersect at a point A, the x-coordinate of then the maximum value of ( f ( x ) + g ( x )) 4 is ……...... .
p
a point A as t ® 0, is equal to (p and q are in their 92. If f ( x + y + z ) = f ( x ) + f (y ) + f (z ) with f (1) = 1 and
q n
lowest form), the ( p + q ) is ………… . ( 4r ) f (3r )
f (2) = 2 and x, y, z ÎR, the value of lim S is ….
n ®¥ r =1 n3
x2
81. Consider a parabola y = and the point F(0, 1). Let n × 3n 1
4 93. If lim = , the number of the
n +1
A 1 ( x 1 , y 1 ), A 2 ( x 2 , y 2 ), A 3 ( x 3 , y 3 ), K, A n ( x n , y n ) are n ®¥ n × ( x
- 1) + n × 3 - 3
n 3 n

‘n’ points on the parabola such that x k > 0 and integral values of x is ............ .
kp
ÐOFA k = (k = 1, 2, K , n ) . If the value of 94. The value of
2n
lim ((( x - 1)( x - 2)( x + 3)( x + 10)( x + 15))1/ 5 - x ) is ...... .
1 n m x ®¥
lim × å FA k = , then m is ………… .
n ®¥ n p tan x
k =1 95. If lim ([ f ( x )] + x 2 ){ f ( x )} = k , where f ( x ) = and [×]
x ®¥ x
cos (tan -1 (tan x ))
82. If L = lim+ , then cos (2 p L ) is ……… . , {×} denotes greatest integer and fractional part of x

p x - p /2 respectively, the value of [k / e ] is ............. .
2
96. The value of lim {( 3 + 1) 2n } is ......... (where {×} denotes
83. Number of solutions of the equation cosecq = k n ®¥

n ær 3 - 1ö fractional part of x).


in [0, p], where k = lim P ç 3 ÷, is ………… .
97. If f ( x ) is a polynomial of degree 4, having extremum at
n ®¥ r = 2 è r + 1 ø
æ f (x )ö
84. If C satisfies the equation x = 1, 2 and lim ç1 + 2 ÷ = 2 .
x ® 0è x ø
x
æx +c ö ec Then, f (2) is ............ .
lim ç ÷ = 4, then is ……... .
x®¥ èx -c ø 2
Chap 05 Limits 293

98. If a is the number of solutions of | x | = log ( x - [x ]), 99. Suppose x 1 = tan -1 2 > x 2 > x 3 > ¼ are the real
(where [×] denotes greatest integer function) and numbers satisfying sin ( x n + 1 - x n ) + 2 - (n + 1) × sin x n
x e ax - b sin x × sin x n + 1 = 0 for all n > 1 and l = lim x n , the value of
lim is finite, the value of (a + b ) is n ®¥
x ®a x3 [4l ] is ........ (where [×] denotes greatest integer function).

Limits Exercise 6 : Questions Asked in Previous 10 Years’ Exams


(i) JEE Advanced & IIT-JEE
x 2 sin (bx )
100. Let a , b Î R be such that lim = 1. Then, F(x ) 1 æ1ö
x ® 0 ax - sin x If lim = , then the value of f ç ÷ is
x ® 1 G( x ) 14 è2ø
6 (a + b ) equals [Integer Answer Type, 2016 Adv]
[Integer Answer Type, 2015 Adv]
101. Let m and n be two positive integers greater than 1. If 103. The largest value of the non-negative integer a for
æ e cos (an ) - e ö 1- x
lim ç ÷ = - æç e ö÷ , then the value of m is ì - ax + sin( x - 1) + a ü 1- x 1
a®0 ç a m ÷ è2ø n which lim í ý = , is
è ø x ®1 î x + sin( x - 1) - 1 þ 4
[Integer Answer Type, 2015 Adv]
[Integer Answer Type , 2014 Adv]
102. Let f : R ® R be a continuous odd function, which
x2
1
vanishes exactly at one point and f (1) = . Suppose that a - a2 - x 2 -
2 104. Let L = lim 4 , a > 0 . If L is finite, then
x x ®0 x4
F ( x ) = ò f (t )dt for all x Î [-1 , 2] [More than One Correct Option, 2009]
–1
x (a) a = 2 (b) a = 1
and G( x ) = ò t | f { f (t )}| dt for all x Î [-1 , 2]. 1 1
–1
(c) L = (d) L =
64 32

(ii) JEE Main & AIEEE


cot x - cos x æx 2 + x + 1 ö
105. The lim equals 110. If lim ç - ax - b ÷ = 4, then
p ( p - 2x ) 3
x® [2017 JEE Main] x ®¥ è x +1 ø [2012 AIEEE]
2

(a)
1
(b)
1
(c)
1
(d)
1 (a) a = 1, b = 4 (b) a = 1, b = - 4
4 24 16 8 (c) a = 2, b = - 3 (d) a = 2, b = 3
106. Let p = lim+ (1 + tan 2 x )1/ 2 x , then log p is equal to 111. Let a (a ) and b (a ) be the roots of the equation
x ®0
[2016 JEE Main]
1 1 ( 3 1 + a - 1) x 2 - ( 1 + a - 1) x + ( 6 1 + a - 1) = 0, where
(a) 2 (b) 1 (c) (d)
2 4
a > - 1. Then, lim a (a ) and lim b (a ) are
1/n a ® 0+ a ®0 +
é (n + 1)(n + 2) K 3n ù [2012 AIEEE]
107. lim ê ú is equal to 5 1
n ®¥ ë n 2n û [2016 JEE Main] (a) - and 1 (b) - and -1
2 2
18 27 9
(a) 4 (b) 2 (c) 2 (d) 3 log 3 - 2 7 9
e e e (c) - and 2 (d) - and 3
2 2
sin( p cos 2 x ) 1
108. lim 2
is equal to 112. If lim [1 + x log (1 + b 2 )] x = 2b sin 2 q, b > 0
x®0 x [2014 JEE Main] x ®0
p
(a) (b) 1 (c) -p (d) p and q Î ( - p, p ], then the value of q is [2011 AIEEE]
2 p p
(a) ± (b) ±
(1 – cos 2x )(3 + cos x ) 4 3
109. lim is equal to
[2013 JEE Main] p p
x ®0 x tan 4 x (c) ± (d) ±
1 6 2
(a) 4 (b) 3 (c) 2 (d)
2
294 Textbook of Differential Calculus

é sin x ù (cos x - 1) (cos x - e x )


113. For x > 0, lim ê(sin x )1/ x + æç ö÷
1 116. The integer n for which lim
ú is is a
x ®0
ê èx ø úû
x ®0 xn
ë [2006 AIEEE]
finite non-zero number, is [2002 AIEEE]
(a) 0 (b) – 1 (c) 1 (d) 2
(a) 1 (b) 2 (c) 3 (d) 4
f (2h + 2 + h 2 ) - f (2) 117. Let f : R ® R be such that f (1) = 3 and f ¢ (1) = 6. Then,
114. lim , given that f ¢ (2) = 6 and
h ®0 f (h - h 2 + 1) - f (1) 1/ x
é f (1 + x ) ù
f ¢ (1) = 4, [2003 AIEEE] lim ê
x ® 0 ë f (1) û
ú equals [2002 AIEEE]
(a) does not exist (b) is equal to -3/2 1
(c) is equal to 3/2 (d) is equal to 3
(a) 1 (b) e 2 (c) e 2 (d) e 3
{(a - n ) nx - tan x } sin nx x
115. If lim = 0, where n is æx - 3 ö
x ®0 x2 118. For x Î R , lim ç ÷ is equal to
x ®¥ èx + 2 ø
non-zero real number, then a is equal to [2003 AIEEE] [2000 AIEEE]
-1
(a) e (b) e
n+1 1 (c) e -5 (d) e 5
(a) 0 (b) (c) n (d) n +
n n

Answers
Exercise for Session 1 Exercise for Session 4
1. (c) 2. (c) 3. (a) 4. (c) 5. (d) 6. (a) 1. (1) 2. (1) 3. (e) 4. (c)
7. (a) 8. (d) 9. (a)
Exercise for Session 5
Exercise for Session 2 1. (a) 2. (c) 3. (c) 4. (a)
1. (a) 2. (b) 3. (a) 4. (a) 5. (b)
Exercise for Session 6
Exercise for Session 3 1. (c) 2. (c) 3. (a) 4. (b) 5. (b)
1. (a) 2. (b) 3. (b) 4. (d) 5. (b) 6. (a)
7. (a) 8. (a) 9. (d) 10. (c)

Chapter Exercises
1. (a) 2. (c) 3. (a) 4. (c) 5. (c) 6. (d) 7. (c) 8. (c) 9. (b) 10. (c)
11. (d) 12. (d) 13. (d) 14. (d) 15. (a) 16. (a) 17. (c) 18. (c) 19. (b) 20. (b)
21. (c) 22. (c) 23. (a) 24. (c) 25. (c) 26. (a) 27. (a) 28. (c) 29. (b) 30. (d)
31. (a) 32. (b) 33. (c) 34. (a) 35. (c) 36. (d) 37. (b) 38. (a) 39. (b) 40. (b)
41. (b, d) 42. (b, d) 43. (b, d) 44. (a, b) 45. (b, c) 46. (a, b, c) 47. (a, b) 48. (a, d)
49. (a,b) 50. (b, c) 51. (c, d) 52. (b, c, d) 53. (b, c,d) 54. (c,d) 55. (a, b) 56. (a)
57. (b) 58. (c) 59. (b) 60. (c) 61. (d) 62. (c) 63. (b) 64. (a) 65. (b) 66. (d)
67. (a) 68. (c)
69. (A) ® (r); (B) ® (p); (C) ® (q); (D) ® (q)
70. (A) ® (p); (B) ® (p); (C) ® (r); (D) ® (s)
71. (A) ® (r); (B) ® (s); (C) ® (p); (D) ® (q)
72. (A) ® (q); (B) ® (r); (C) ® (p); (D) ® (p)
73. (6) 74. (1) 75. (3) 76. (6) 77. (2) 78. (1) 79. (5) 80. (5) 81. (4) 82. (1)
83. (0) 84. (1) 85. (1) 86. (1) 87. (2) 88. (2) 89. (1) 90. (4) 91. (4) 92. (4)
93. (3) 94. (5) 95. (7) 96. (1) 97. (0) 98. (1) 99. (3) 100. (7) 101. (2) 102. (7)
103. (2) 104. (c) 105. (c) 106. (b) 107. (d) 108. (c) 109. (b) 110. (d) 111. (b) 112. (d)
113. (c) 114. (d) 115. (d) 116. (c) 117. (c) 118. (c)
æ 1 + cos 2 x ö
= tan x ç ÷ (1 + sec 4 x ) ... (1 + sec 2 x )
n

Solutions
è cos 2 x ø
= tan 2 x (1 + sec 2 2 x ) K (1 + sec 2n x )
= tan 2n - 1 x (1 + sec 2n x ) = tan 2n x
fn ( x ) tan 2n x n - 1
Now, lim = lim ×2 = 2n - 1
x®0 2x x ® 0 2n x
sin( p - p sin 2 tan (sin x ))
1. lim
x®0 x2 8. As, f ¢( x ) > 0 Þ f ( x ) is increasing.
sin( p sin 2 tan(sin x )) æ p sin 2 tan (sin x ) ö So, for t > 1; f (t ) > 1
= lim ç ÷ 1 1
x®0 p sin 2 tan (sin x ) è tan 2 (sin x ) ø Now, f ¢(t ) = <
t 2 + f (t ) 1 + t 2
æ tan 2 (sin x ) ö æ sin 2 x ö
ç ÷ç ÷=p x x dt
è sin 2 x ø è x 2 ø \ f (x ) = 1 + ò1 f ¢(t ) dt < 1 + ò1 1 + t2
1 - (1 + t )t ¥ dt p
2. lim
1 - (1 + t )t
= lim t2
Þ lim f ( x ) < 1 +
x®¥ ò1 1+t 2
Þ lim f ( x ) < 1 +
x ® ¥ 4
t ® 0 ln(1 + t ) - t t ® 0 ln (1 + t ) - t
2
t2 1 æ 1ö 1
= çsin q - ÷ +
9. E = sin 2 q - sin q +
1 -e t (ln (1 + t )) 2 è 2ø 4
= lim =2 1
t ®0 t2 Þ Minimum value is .
-1/2 4
K 2 - x 2 3x + 2
3. I 1 = lim (tan -1 px - tan -1 x ) Let K = ( x + 1 ) ( x + 2 ), then K - x = =
x®¥ K +x K +x
2
æ (p - 1) x ö 3+
= lim tan -1 ç ÷ = 0 Þ I 1 = 0 and I 2 = 0 lim ( K - x ) = lim Þ lim xK
=1
x®¥ è 1 + px 2 ø x®¥ K x®¥ x®¥ x
+1
4. From given f ( x ) = x 2 - p 2, x
x2 - p2 (- p + h )2 - p 2 3+0 3 é K ù
lim = lim \ lim ( K - x ) = = êë xlim = 1ú
x ® - p sin (sin x ) x ® - p sin sin ( - p + h ) x®¥ 1+1 2 ®¥ x û
7 3
- 2hp + h 2 h - 2p The required equation is x 2 - x + = 0,
= lim = lim = 2p 4 8
h®0 - sin (sin h ) h ® 0 - sin (sin h ) sin h
´ i.e. 8 x - 14 x + 3 = 0.
2
sin h h
5. Using approximations 10. Let xn = tan qn
sin 2 x 2 tan x x2 2x xn tan q
+ + Now, tan qn + 1 = xn + 1 = =
L = lim 3 4 = lim 3 4 1+ 1+ xn2 1 + 1 + tan 2 qn
x ® 0 sin x + tan 2 x x ® 0 x + x2
tan qn sin qn q
æ x 1ö Þ tan qn + 1 = = = tan n
xç + ÷ 1 + sec qn 1 + cos qn 2
è 3 2ø 1
= lim = q qn - 1
x ® 0 x (1 + x ) 2 Þ qn + 1 = n Þ qn =
q q 2 2
6. Let x 0 = cos q, x1 = cos , x 2 = cos 2 p p æ 2p ö
2 2 Now, q1 = Þ qn = Þ xn = tan ç n ÷
3 3 ×2n -1 è 3 ×2 ø
æ ö
æ 1 - x 02 ö÷ ç sin q ÷ æ 2p ö
= lim ç = lim ç ÷ tan ç n ÷
n ® ¥ ç x1x 2x 3¼xn + 1 ÷ è 3 × 2 ø 2p
è ø n ® ¥ ç cos q cos q ... cos q ÷ Þ lim 2 xn = lim
n
=
è 2 2 2
2n + 1 ø n®¥ n®¥ æ1ö 3
ç n÷
æ n qö æ qö æ ö è2 ø
ç 2 × sin n ÷ ç sin n ÷ ç q ÷ ( x - b ) - (a - b ) 1
= lim ç 2 ÷ = lim ç 2 ÷ ç ÷ 11. L = lim = ´ 2
n®¥ç q ÷ n®¥ç q ÷ ç q ÷ x ®a - x - b + a - b (x - a 2 )
cos n + 1 cos n + 1
è 2 ø è 2 ø è
n
2 ø 1 1
= lim =
= q = cos-1 ( x 0 ) x ® a - (x + a ) { x - b + a - b } 4a a - b

x æ 1 + cos x ö 12. lim (sinn 1 + cosn 1)n = sin 1 lim (1 + cotn 1)n
7. fn ( x ) = tan ç ÷ (1 + sec 2 x ) (1 + sec 4 x ) n®¥ n®¥
2 è cos x ø n 1
...(1 + sec 2n x ) lim
n ® ¥ tann 1
lim
n ® ¥ tann 1× ln (tan 1)
= sin 1 × e = sin 1 × e = sin 1
296 Textbook of Differential Calculus

x 3 3x 5 5 7 As, Y
13. As, sin -1 x = x + + + x +L
6 40 112 π
3 5 7
x x x
and tan -1 x = x - + - +L
3 5 7
2
\ lim 3 (sin -1 x - tan -1 x ) 1
x ® 0x
X
2 éæ x 3 3x 5 ö æ x3 x5 öù 0 1/2 cot 1 z
= lim 3 ê ç x + + + L÷ - ç x - + + L÷ ú
x ® 0 x êè
ë 6 40 ø è 3 5 ø úû Q [cot -1 x ] = 1, when 1 / 2 < x < cot 1
2 é æ1ö æ 3 1ö ù and [cos-1 x ] = 0, when cot 1 < x < z
= lim 3 ê x 3 ç ÷ + x 5 ç - ÷ + Lú = 1
x ® 0x ë è2ø è 40 5 ø û
cot 1

2/ x 2 Þ lim
ò1/2 1 × dx
= lim
(cot 1 - 1 / 2 )
=0
é 2 ù z ®¥ 1 z z ®¥ 2 (1 - 1 / 2 ) + (z - 1 )
\ lim ê 3 (sin -1 x - tan -1 x ) ú ò1/2 2 × dx + ò 1 × dx
¥
[1 form]
x ®0 ëx û 1

é 2 ù 2
lim ê (sin -1 x - tan -1 x ) -1ú× æ 1 1ö a+b 2 sin q
x ® 0 ë x3 û x2
17. lim ç + ÷ = lim = lim = 2
=e + èa ø
b q ® 0 ab
+ q
q®0 q ® 0+
é æ x 3 3x 5 ö æ x3 x5 öù
2 êçx + + + L÷ - ç x - + - L÷ ú -x 3 1é 5 ù
2 êë çè 6 5 ÷ ç
ø è 3 5 ÷
ø úû 18. Here, lim f ( x ) = lim ê f ( x + 1) + ú
lim
2
x ®¥ x ®¥ 3 ë f (x + 2) û
= ex ® 0x x3
1 æ 5ö 1
2 é 1 5 ù
ê - x + higher powers of x ú
Þ l = × çl + ÷ Þ l 2 = (l 2 + 5 )
3 è lø
lim
x ® 0 x5 ë 4 û -1/ 2 1 3
=e =e =
e 5
1 n
æ kö æk ö Þ 2l 2 = 5 Þ l =
14. We have, S = × å ç1 - ÷ × cos 4ç ÷ 2
n k =1
è nø ènø
cos (2 x + 4 ) - 33 x 2 4x -8
1 æ sin 4 x ö 1 1
1 19. lim = -16 and lim = - 4( 4 ) = - 16
S = ò (1 - x ) cos 4 x × dx = ç(1 - x ) ×
4 ø 0 4 ò0
÷ + sin 4 x dx x ® 2- 2 x ® 2- 2
0 è
By Sandwich theorem, lim f ( x ) = - 16
1 1 1 1 x ® 2-
=0+ × (cos 4 x )10 = - (cos 4 - cos 0 ) = (1 - cos 4 )
4 ( - 4) 16 16 20. As, f ( x ) being polynomial having roots 1, 2, 3, 4 and leading
coefficient 1.
15. Let y = cos 2x × 3 cos 3x × 4 cos 4x K n cos nx \ f (x ) = (x - 1) (x - 2) (x - 3) (x - 4)
1 1 1
loge y = log cos 2 x + log cos 3 x + log cos 4 x æ 1ö æ 1ö æ 1ö
2 3 4 Similarly, g( x ) = ( x - 1 ) ç x - ÷ ç x - ÷ ç x - ÷
1 è 2ø è 3ø è 4ø
+ ...+ log cos nx
n f (x ) (x - 1) (x - 2) (x - 3) (x - 4)
On differentiating both sides, we get \ lim = lim
1 ( -2 sin 2 x ) 1 æ -3 sin 3 x ö 1 é ( -n sin nx ù
x ®1 g( x ) x ® 1 æ 1ö æ 1ö æ 1ö
1 (x - 1) ç x - ÷ ç x - ÷ ç x - ÷
× dy / dx = × + ×ç ÷ + ... + ê ú è 2ø è 3ø è 4ø
y 2 cos2 x 3 è cos 3 x ø n ë cos nx û
( -1 ) ( -2 ) ( -3 )
dy n n
= = - 24
\ = Õ (cos rx )1/r × å[ - tan (rx )] ...(i) æ1ö æ2ö æ 3 ö
dx r = 2 ç ÷ç ÷ç ÷
r =2 è2ø è3ø è 4ø
n
1 - Õ [cos(rx )]1/r 4 2 - (cos x + sin x ) 5
r=2
21. We have, lim
\ L = lim x®
p 1 - sin 2 x
x ®0 x2 4
n n
[(cos x + sin x ) 2 ]5/2 - (2 ) 5/2 (1 + sin 2 x ) 5/2 - (2 ) 5/2
- Õ (cos rx )1/2 å[ - tan (rx )] = lim = lim
r =2 r =2 x®
p (1 + sin 2 x ) - 2 x®
p (1 + sin 2 x ) - 2
= lim [using L’ Hospital’s rule] 4 4
x® 0 2x
Put y = 1 + sin 2 x
1 é tan 2 x + tan 3 x + L+ tan nx ù
= lim × ê úû y 5/2 - 2 5/2 5 2 -1 é
5
p ù
x ®0 2 ë x = lim = (2 ) = 5 2 ê as x ® Þ y ® 2 ú

p y -2 2 ë 4 û
1 n2 + n - 2 4
= [2 + 3 + L + n ] = = 10 [given]
2 4 n × 3n 1
Þ (n + 7 )(n - 6 ) = 0 Þ n = 6 22. We have, lim =
n ®¥ n ( x - 2 ) + n × 3n + 1 - 3n
n
3
z -1 cot 1 z

16. Here, lim


ò1/2 [cot ò 1 × dx + òcot1 0 × dx
= 1/21
x ] dx
Þ lim
1
=
1
n ®¥ æ x - 2 ö n 1 3
z ®¥ z é 1ù z
ò1/2 êë1 + x úû dx ò1/2 2 × dx + ò1 1 × dx ç
è 3 ø
÷ +3-
n
Chap 05 Limits 297

1 éxù
For, limit to exists and is equal to . êë 2 úû
3
x -2 Þ lim = lim 0 = 0
We must have, -1 < < 1. x ® p /2 log sin x x ® p /2
3
æ a1 + 1 ö æ a 2 + 1 ö æ an + 1 ö
Þ -3 < x - 2 < 3 Þ -1 < x < 5, as x Îinteger 27. We have, lim ç ÷ç ÷ ... ç ÷
n®¥ è a1 ø è a 2 ø è an ø
\ x Î { 0, 1, 2, 3, 4 }
a
Þ Number of integer solutions = 5. and an – 1 + 1 = n …(i)
1 n
23. Given, f ( x ) = lim æa ö æa ö æa ö æ an + 1 ö 1
n ® ¥æ 3
-1 ö
2n
\ lim ç 2 ÷ ç 3 ÷ ç 4 ÷ K ç ÷×
ç tan 2 x ÷ + 5 n®¥è 2 ø è 3 ø è 4 ø è n + 1 ø a1 ×a 2 ... an
èp ø
an + 1 1 + an
æ3 ö
2 = lim = lim [using Eq. (i)]
Now, f ( x ) = 0, if and only if ç tan -1 2 x ÷ > 1 n ® ¥ (n + 1 ) ! n®¥ n!
èp ø
æ1 a ö æ1 1 an – 1 ö
p p = lim ç + n ÷ = lim ç + + ÷ [using Eq. (i)]
Þ tan -1 2 x > or tan -1 2 x < - n ® ¥ èn ! n ! ø n ® ¥ è n ! (n – 1 ) ! (n – 1 ) ! ø
3 3
Þ 2 x > 3 or 2 x < - 3 Þ | 2 x | > 3 æ1 1 1 1 1 a ö
= lim ç + + + ... + + + 1÷
3
x n ® ¥ èn ! (n – 1 ) ! (n – 2 ) ! (2 ) ! 1 ! 1 ! ø
cos2 x - cos x - e x cos x + e x - [a1 =1, given]
24. We have, lim 2
x®0 xn æ1 1 1 1 1 1ö
= lim ç + + + ... + + + ÷
x3 n ® ¥ èn ! (n – 1 ) ! (n – 2 ) ! (2 ) ! 1 ! 1 ø
(cos x - 1 )(cos x - e x ) -
= lim 2 é 1 1 1 ù
=e ê as e = 1 + + + + ... ¥ ú
x®0 xn ë 1 ! 2 ! 3 ! û
éæ x2 x4 ö ù 28. Here, f ( x + y ) = f ( x ) + f (y )
ê ç1 - + -. . .÷ ú
æ x2 x4 x6 ö êè 2! 4! ø ú x3 f (2 ) = f (1 ) + f (1 ) = 2
ç1 - + - + . . .- 1÷ ê -
è 2! 4! 6! ø æ 2 öú 2 f (3 ) = f (1 + 2 ) = f (1 ) + f (2 ) = 3
ê -ç1 + x + x + . . .÷ ú
êë è 1! 2! ø úû f ( 4 ) = f (1 + 3 ) = f (1 ) + f (3 ) = 4
= lim n MMMMMMMMMMMMMMMMMMMMMMMMMMMMMM
x®0 x
æ x2 x4 x6 ö f ( x ) = x, for all x Î R
ç- + - + . . .÷
è 2! 4! 6! ø Þ f (tan x ) = tan x, f (sin x ) = sin x

æ ö x3 2 f (tan x ) – 2 f (sin x ) 2 tan x – 2 sin x


x 3 2x 5 \ lim = lim
ç-x - x2 - - - . . .÷ - x®0 2
x f (sin x ) x®0 x 2 sin x
è 3! 5! ø 2
= lim 2 sin x × {2 tan x – sin x – 1 } tan x – sin x
x®0 xn = lim ´
æ x3 x4 x5 ö x3 x®0 sin x × x 2 tan x – sin x
ç + + - . . .÷ - x4 x5 x5
è 2 2 12 ø 2 + - {2 tan x – sin x – 1 } ì tan x – sin x ü
= lim = lim 2 12 24 = lim ´í 2 ý ´2
sin x
x®0 tan x – sin x
x®0 x n x®0 x n
î x sin x þ
= a non-zero finit number, if n = 4. 2 tan x – sin x – 1 1 – cos x
= lim ´ 2 ´ 2 sin x
25. We know that, x ® 0 tan x – sin x x cos x
tan -1 x sin -1 x
< 1 and > 1, " x Î R 2 tan x – sin x – 1 2 sin 2 x/ 2 1
x x = lim ´ ´ 2 sin x ´
x ® 0 tan x – sin x 4 ( x/ 2 ) 2 cos x
tan -1 x sin -1 x sin -1 x tan -1 x
\ - < 0 and - >0 1 1
x x x x = log 2 ´ ´ 1 = log 2
2 2
Þ I 1 doesn’t exist and I 2 exists. n
–n 2 é æ 1ö æ 1 öù
[ x / 2] 29. We have, lim n ê (n + 1 ) ç n + ÷ ... ç n + ÷
26. Here, lim
p log(sin x )
n®¥ ë è 2 ø è 2n – 1 ø úû
x® n
2 é æ 1ö æ 1 öù
p x p x p ê (n + 1 ) çèn + 2 ÷ø ... çèn + 2n – 1 ÷øú
As, x ® Þ ® , where ® < 1 = lim ê ú
2 2 4 2 4 n®¥ ê nn ú
p+ p- êë úû
for x® or x ® n
2 2 n
æ 1ö æ 1 ö
éxù æx pö n çn + ÷ ç n + n –1 ÷
\ æn + 1ö
êë 2 úû = 0, since 0 < çè 2 ® 4 ÷ø < 1 = lim ç ÷ ×ç 2 ÷ ... ç 2 ÷
n®¥è n ø
ç n ÷ ç n ÷
è ø è ø
298 Textbook of Differential Calculus

æ 1ö æ
n
1ö æ 1 ö
n n
Shows limit exists at all points except at x = 1, 2, 3.
= lim ç1 + ÷ × ç1 + ÷ ... ç1 + n – 1 ÷ [1 ¥ form]
n®¥è n ø è 2n ø è 2 nø Graphically, this is shown in given figure.
2n 2n –1 ×n
n Y
æ 1ö æ 1ö2 æ 1 ö 2n –1
= lim ç1 + ÷ × ç1 + ÷ ... ç1 + n – 1 ÷
n®¥è nø è 2n ø è 2 nø
Y = f(x)
n –1 é æ 1ö
an ù 1
= e1 × e1/ 2 × e1/ 4 ... e1/ 2 K¥ ê using lim ç1 + ÷ = e ú
a

êë n ® ¥ è n ø úû XN X
0
1 1 2 3
1
1– –1
= e (1 + 1/ 2 + 1/ 4 + K) =e 2 = e2
YN
30. Here, limit can be calculated only after removing greatest
integral function (i.e. [ x ]). which shows limit doesn’t exist at x = 1, 2, 3.
\ LHL at x = 0 r
sin [ x ] sin [ 0 – h ] 32. Here, tr =
= lim – f ( x ) = lim – = lim 1 + r2 + r4
x®0 x®0 [x ] h ® 0 [0 – h]
1æ 2r ö 1é 2r ù
sin (–1 ) = ç ÷= ê ú
= lim = sin 1 2 è 1 + 2r 2 + r 4 – r 2 ø 2 ë (1 + r 2 ) 2 –(r 2 ) û
h ® 0 (–1 )
1é 2r ù
[as –1 £ [ 0 – h ] < 0 \ [ 0 – h ] = – 1, i.e. [ x ] ¹ 0] = ê ú
Again, RHL at x = 0 2 ë (1 + r + r )(1 + r – r ) û
2 2

é ì sin [ x ] ù 1 é (r 2 + r + 1 )–(r 2 – r + 1 ) ù
ï , [x ] ¹ 0 ú = ê ú [Q 2r = (r 2 + r + 1 )–(r 2 – r + 1 )]
= lim f ( x ) = lim 0 = 0 ê as f ( x ) = í [ x ] 2 ë (1 + r 2 + r )(1 + r 2 – r ) û
x ® 0+ x ® 0+ ê ú
êë ï 0, [ x ] = 0 úû
î 1é 1 1 ù 1é 1 1 ù
= ê – ú= ê – ...(i)
\ lim f ( x ) = 0 2 ë 1 + r – r 1 + r + r û 2 ë 1 + r (r – 1 ) 1 + r (r + 1 ) úû
2 2
x ® 0+
1æ 1 1 ö
Here, 0 £ [ 0 + h ] < 1 Thus, tr = ç – ÷
2 è 1 + r (r – 1 ) 1 + r (r + 1 ) ø
\ [ x ] = 0 Þ f (x ) = 0
So, lim f ( x ) ¹ lim f ( x ). Thus, limit doesn’t exist. 1æ 1 1 ö 1æ 1ö
x ® 0- x ® 0+
t1 = ç – ÷ = ç1 – ÷
2 è1 + 0 1 + 2ø 2 è 3ø
|( x – 1 ) ( x – 2 ) ( x – 3 )|
31. We write, f ( x ) = 1æ 1 1 ö 1 æ1 1ö
(x – 1) (x – 2) (x – 3) t2 = ç – ÷= ç – ÷
2 è 1 + 2 1 + 2 ×3 ø 2 è 3 7 ø
ì–1, x <1
ï 1, 1 < x < 2 1 æ1 1 ö 1 æ1 1 ö
ï t3 = ç – ÷ = ç – ÷
f (x ) = í 2 è 7 13 ø 2 è 7 13 ø
ï–1, 2 < x < 3 ....................................................
ïî 1, x >3
....................................................
[leaving x =1, 2, 3 as denominator ¹ 0] 1æ 1 1 ö
Using Wavy-curve method, as shown in figure tn = ç – ÷
2 è 1 + n (n – 1 ) 1 + n (n + 1 ) ø
–∞ ∞ n
1æ 1 ö
1 2 3 \ lim
n®¥
åtr = 2 çè1 – 1 + n (n + 1) ÷ø …(ii)
r =1
When x > 3, then n
1æ 1 ö 1æ 1ö
|( x – 1 ) ( x – 2 ) ( x – 3 )| = + ( x – 1 ) ( x – 2 ) ( x – 3 ) Hence, lim
n®¥
åtr = nlim ç1 –
®¥2 è
÷
1 + n (n + 1 ) ø
= ç1 – ÷
2è ¥ø
When 2 < x < 3, then r =1
|( x – 1 ) ( x – 2 ) ( x – 3 )| = – ( x – 1 ) ( x – 2 ) ( x – 3 ) n
1
When 1 < x < 2, then \ lim
n®¥
åtr = 2
r =1
|( x – 1 ) ( x – 2 ) ( x – 3 )| = + ( x – 1 ) ( x – 2 ) ( x – 3 )
When x < 1, then æ 3 1ö
n çr – r + ÷ n æ 2r ö
|( x – 1 ) ( x – 2 ) ( x – 3 )| = – ( x – 1 ) ( x – 2 ) ( x – 3 )
x <1
33. Here, lim
n®¥
å cot –1
ç r ÷ = lim
® ¥
å tan –1 ç
è1 – r + r ø
2 4
÷
ì–1, r =1 ç 2 ÷ n
r =1
ï 1, 1 < x < 2 è ø
ï
Thus, f (x ) = í n æ 2r ö
ï–1, 2 < x < 3 = lim
n®¥
å tan–1 çè 1 + (r 2 –r ) (r 2 + r ) ÷ø
ïî 1, x >3 r =1
Chap 05 Limits 299

n æ (r 2 + r )–(r 2 – r ) ö æ x 2 + f (x ) ö 1
lim ç ÷× x 2 + f (x )
= lim
n®¥
å tan–1 çè 1 + (r 2 + r ) (r 2 –r ) ÷ø Þ e
x ® 0 çè x2 ÷ x
ø
=e Þ e
2
lim
x ®0 x3 =e 2
r =1
n
x 2 + f (x ) x 2 + a 2x 2 + a 3x 3 + ...
= lim
n®¥
å {tan–1 (r 2 + r ) – tan–1(r 2 – r )} \
x®0
lim
x 3
= 2 Þ lim
x®0 x3
=2
r =1

= lim [( tan –1 2 – tan –1 0 ) + ( tan –1 6 – tan –1 2 ) + (tan –1 12 – tan –1 6 ) Þa 2 = – 1, a 3 = 2 and a 4 , a 5 are any arbitrary constants. Since,
n®¥ we want polynomial of least degree.
+ ... + {( tan –1 (n 2 + n )– tan –1 (n 2 – n )}] Hence, f (x ) = – x 2 + 2x 3
= lim {tan –1 (n 2 + n )– tan –1( 0 )}
n®¥ 36. We know, n £ [ x ] < n + 1 Þ [ x ] = n
p n sin x
® n, as x ® 0 but less than n.
= tan –1 ( ¥ )– tan –1( 0 ) = Here,
2 x
æ 3 1ö n tan x
n çr – r + ÷ p Also, ® n, as x ® 0 but more than n.
x
\ lim
n®¥
å cot -1
ç
2
r÷=
é n sin x ù
r =1 ç ÷ 2 Thus, n – 1 < ê < n, as x ® 0
è ø ë x úû
34. Here, two straight lines (tan a ) x + (sin a ) y = a and é n sin x ù
Þ êë x úû = n – 1
( a cos ec a ) x + (cos a ) y = 1
have their point of intersection, as é n tan x ù
a cos a – sin a a – x tan a Again, n £ ê < n + 1, as x ® 0
x= and y = ë x úû
sin a – a sin a
é n tan x ù
\ When a ® 0, we obtain the point P. Þ êë x úû = n
a cos a – sin a é0 ù
i.e. lim x = lim êë 0 form úû æ é n sin x ù é n tan x ù ö
a®0 a®0 sin a – a Thus, lim ç ê + ÷ = (n – 1 ) + (n ) =(2n – 1 )
x ® 0 èë x úû êë x úû ø
– a sin a + cos a - cos a
= lim [applying L’Hospital’s rule] æ é n sin x ù é n tan x ù ö
a®0 cos a – 1 \ lim ç ê + ÷ = (2n - 1 )
x®0 è ë x úû êë x úû ø
æ a aö 2a
a ç2 sin cos ÷ 37. Here, lim [ 2 – x + 1 + x ] and we know, we could only
è 2 2ø a
= lim = lim = lim 2 = 2 x ®a
a®0 2a a®0 a a®0 a apply limit after defining greatest integral function.
2 sin tan tan
2 2 2 é 1ù
Thus, finding range of [ 2 – x + 1 + x ], when x Î ê 0, ú .
a – x tan a ë 2û
Again, as lim y = lim
a®0 a®0 sin a i.e. Let f (x ) = 2 – x + 1 + x
æ a x ö a x
= lim ç – ÷ = lim – lim 1æ 1 1 ö
a ® 0 è sin a cos a ø a ® 0 sin a a ® 0 cos a For range f ¢( x ) = ç – + ÷
2è 2– x 1+ xø
é lim ù 1æ 2– x– 1 + x ö
=1–2 êë as a ® 0 x = 2 úû = ç ÷
2è 2– x × 1 + x ø
\ lim y = – 1
a®0 Þ f ¢( x ) will be the +ve for 2 – x > 1 + x .
Hence, in limiting position P (2, – 1 ). Þ f ¢( x ) will be the +ve for 2 – x > 1 + x.
æ x + f (x ) ö
2 1/x
Þ f ¢( x ) will be the +ve for 2 x < 1.
35. Here, lim ç1 + ÷ 1
x®0 è x2 ø Þ x <
1/x
2
æ x 2 + f (x ) ö 1
Þ lim ç1 + ÷ = L (say) \ f ( x ) will be increasing for x < ×
x®0è x2 ø 2
exists only when \ f (0) = 2 + 1
x 2 + f (x ) Þ f (1 / 2 ) = 6
lim =0 [it converts to 1 ¥ form]
x®0 x2 é 1ù
So, the least degree in f ( x ) is of degree 2. which shows range of f ( x ) is [1 + 2, 6 ], when x Î ê 0, ú .
ë 2û
i.e. f ( x ) = a 2x 2 + a 3x 3 + ... \ [ 2– x + 1 + x]=2
1/x
æ x 2 + f (x ) ö Þ lim [ 2 – x + 1 + x ] = lim 2 = 2
Now, lim ç1 + ÷ = e2 x ®a x ®a
x ® 0è x2 ø
300 Textbook of Differential Calculus

(tan x – sin x ) cos2 (sin 2 (sin 2 ... (sin 2 q)) ... )


38. Let y = (tan x – sin x ) + = lim
+ (tan x – sin x ) + ... ¥ q®0 æ p ( q + 4 – 2) ö
sin ç ÷
è q ø
Þ y = (tan x – sin x ) + y
cos2 (sin 2 (sin 2 ... (sin 2 q)) ... ) cos2 ( 0 )
Þ y – y –(tan x – sin x ) = 0
2 = lim = = 2
q®0 æ q ö æpö
sin ç p lim ÷ sin çè ÷ø
1 + 1 + (tan x – sin x ) 4 è q ® 0 q( q + 4 + 2) ø 4
Þ y = [as y > 0] ...(i)
2
40. Here, lim an = lim an + 1.
n®¥ n®¥
Again, let Z = x + x + x + ... ¥
3 3 3
So, let l = lim an = lim an + 1
n®¥ n®¥
Þ Z = x3 + Z
\ l = lim an + 1 = lim 2 + an
Þ Z 2 – Z – x3 = 0 n®¥ n®¥

1 + (1 + 4 x 3 ) é ù
Þ Z = [as Z > 0] ...(ii) or l = 2+l êëQnlim an = l ú
2
®¥ û
Þ l2 = 2 + l Þ l2 – l – 2 = 0
–1 + (tan x – sin x ) + (tan x – sin x ) + ... ¥
\ lim Þ (l - 2 ) (l + 1 ) = 0 Þ l = 2, – 1
x ®0+
–1 + x 3 + x 3 + x 3 + ... ¥ \ lim an = 2 or –1
n®¥
æ 1 + 1 + 4 (tan x –sin x ) ö é ù
Þ lim an = 2 êë neglecting nlim an = – 1, as an > 0 ú
–1 + ç ÷ n®¥ ®¥ û
è 2 ø
= lim [from Eqs. (i) and (ii)] æ 1 ö
x ®0+ æ 1 + 1 + 4x 3 ö tan -1 ç ÷
–1 + ç ÷ è2x + 3ø 1
ç 2 ÷ 41. lim 4x ´ =2
è ø x®¥ æ 1 ö 2x + 3
ç ÷
–1 + 1 + 4 (tan x –sin x ) è2x + 3ø
= lim
x®0 +
–1 + 1 + 4 x 3 Þ y 2 + 4y + 5 = 2
Rationalising numerator and denominator, we get Þ y = - 1, - 3.
æ 2ö æ 2 ö2
x x
4 (tan x – sin x )(1 + 1 + 4 x 3 ) 2 sin 2 ç ÷ ç ÷
lim è 2 ø è 2 ø x 1 1 æe2 ö
x®0 +
4 x 3 (1 + 1 + 4 (tan x –sin x ) ) 42. lim × x × 4 = ln 4 = ln 2 = 1 - ln ç ÷
x®0 2 (4 - 1) x 2 2 è4ø
æ sin x sin x ö
ç – ÷ (1 + 1 + 4 x )
3 43. f ( x ) = e [cot x ], as cot x is negative in the II quadrant and
è cos x 1 ø p 1
= lim cot = 0 [cot x ] = - 1 Þ lim f ( x ) = e - 1 =
x ®0+ x 3 (1 + 1 + 4 (tan x –sin x ) ) 2 p+ e

2
sin x (1 – cos x ) (1 + 1 + 4 x 3 ) p-
= lim × As x ® , cot x is positive (being in I quadrant) and hence
x ®0+
3
x cos x (1 + 1 + 4 (tan x –sin x ) ) 2
x [cot x ] = 0 Þ lim f ( x ) = e 0 = 1
2 sin 2 p-
sin x 2× 1 × (1 + 1 + 4 x 3 ) x®
= lim × 2
x ®0+ x 4x 2 cos x (1 + 1 + 4 (tan x –sin x ) ) 44. If m < 0, then for values of x sufficiently close to 0.
4 1 sin x sin x
1 (1 + 1 ) 1 1+ < <1 Þ m + 1 >m >m
=1 × × 1 × = m x x
2 (1 + 1 ) 2 é sin x ù é sin x ù
\ êëm x úû = m Þ lim êm =m
(tan x – sin x ) x®0ë x úû
–1 + (tan x – sin x ) + (tan x – sin x ) If m > 0, then for values of x sufficiently close to 0, we can
+ have
+ ... ¥ 1
\ lim = 1 sin x
x®0 + 2 1- < <1
–1 + x 3 + x 3 + x 3 + ... ¥ m x
sin x
cos2 (1 – cos2(1 – cos2 (1 ... cos2 q))... ) \ m -1 <m <m
39. Here, lim x
q®0 æ p ( q + 4 – 2) ö
sin ç ÷ é sin x ù
è q ø \ lim m =m -1
x®0ê ë x úû
Chap 05 Limits 301

2 2
(ax + bx 2 ) é1 1 1ù
lim
51. (a) lim ê + 2 + ¼+ n ú
45. lim (1 + ax + bx 2 ) x = e 3 Þ e x ® ¥ x = e3 n ®¥ ë 2 2 2 û
x®0
Þ e 2a = e 3 é1 æ 1 öù
ê 2 çè1 - 2n ÷ø ú é 1ù
Since, limit value does not involve b. = lim ê ú = lim 1 - =0
n ®¥ ê 1 ú n ®¥ ëê 2n úû
3 1-
\ b can have any value. Thus, a = , b Î R êë 2 úû
2
46. (a) lim f ( x ) = 1 (b) lim ( x -1 ) { x }
n ®¥
x ® 0+
RHL = lim(1 + h - 1 ) {1 + h } = lim h (h ) = 0
(b) lim f ( x ) = 2, as RHL = LHL = 2 (at x =1) h®0 h®0
x ®1
LHL = lim(1 - h - 1 ) {1 - h } = lim - h (1 - h ) = 0
(c) lim f ( x ) ®¥ and lim f ( x ) ®- ¥ h®0 h®0
x ® 3- x ® 3+ \ lim( x - 1 ) { x } = 0
x ®1
\ lim f ( x ) does not exists. tan x
x ®3 (c) As, >1
(d) lim f ( x ) = 0 x
x ®4
cot -1(1 / x ) tan -1 é tan x ù é tan x ù
\ ê ®1, as x ® 0 + Þ lim ê =1
47. RHL = lim = lim =1 ë x úû ë x úû
x ® 0+
x ® 0+ x x ® 0+ x
cot -1(1 / x ) p + tan -1 x é tan x ù
LHL = lim = lim =¥
(d) ê lim ú = [1 + h ] = 1
x ®0 x x ®0 x ë x ®0 x û
+

\ RHL exists and LHL does not exists. p


52. Put x = - h
lim 2
x x x ® ¥ 2x
æ x +1ö æ 2 ö x -1 a tan h - a sin h a sin h (a tan h - sin h - 1 )
48. As, l = lim ç ÷ = lim ç1 + ÷ =e = e2 l = lim = lim = loge a
x ®¥ è x - 1 ø x ®¥ è x -1 ø h ® 0 tan h - sin h h ®¥ tan h - sin h
\ [l ] =7 and {l } = e 2 - 7. and m = lim ( x 2 + ax - x 2 - ax )
x ®-¥
sin[ x ] sin ( -1 )
49. (a) lim f ( x ) = lim = lim = sin 1 ( x 2 + ax ) - ( x 2 - ax )
x ® 0- x ® 0- [ x ] h ® 0 ( -1 ) = lim
x ®-¥ x 2 + ax + x 2 - ax
(b) lim f ( x ) = lim[ 0 + h ] = 0
x ® 0+ h®0 2ax
= lim = -a
\ Limit does not exist at x = 0. x ®-¥ æ a aö
x ç 1+ + 1- ÷
50. (a) lim [[ x ]] - [2x - 1] è x xø
x ®3
x
RHL = lim[[3 + h ]] - [6 + 2h - 1 ] = lim 3 - 5 = - 2 é ax + 1 ù
53. f ( x ) = lim ê1 + -1
bx + 2 úû
h®0 h®0
x ®¥ ë
LHL = lim[[3 - h ]] - [6 - 2h - 1 ] = lim 2 - 4 = - 2 x
h®0 h® 0 x é (a - b ) x - 1 ù
\ Limit exists. é (a - b ) x - 1 ù lim ê
bx + 2 û
ú
= lim ê1 + ú = e x ®¥ë
x ®¥ ë bx + 2 û
(b) lim[ x ] - x
x ®1 lim -
x 1 1
- -
bx + 2
RHL = lim[1 + h ] - (1 + h ) = lim(1 - 1 - h ) = 0 When a = b, f ( x ) = e x ® ¥ =e b or e a .
h®0 h®0
LHL = lim[1 - h ] - (1 + h ) = lim 0 - 1 + h = - 1 When a > b, f ( x ) = e ¥ = ¥, does not exist.
h®0 h®0
\Limit does not exist. When a < b, f ( x ) = e -¥ = 0.
1
(c) lim { x } 2 - { - x } 2 2× x2
x × (2 x - 1 ) 4 × 2 - 1 = 2 × log 2 = log 4
x
x ®0 54. lim f ( x ) = lim = lim e e
x ®0 x ® 0 2 sin 2 x / 2 x ®0 x x
RHL = lim{h } 2 - { - h } 2 = lim h 2 - (1 - h ) 2 = - 1 sin 2
h®0 h®0 2
sin (log 2 / 2 x )
LHL = lim { - h } 2 - {h } 2 = lim(1 - h ) 2 - h 2 = 1 and lim 2 x ×sin (log 2 / 2 x ) = lim × loge 2 = loge 2
h®0 h®0 x ®¥ x ®¥ (log 2 / 2 x )

\ Limit does not exist. x 3 + cx 2 + 5 x + 12


55. As, lim =l . [finite]
tan(sgn x ) tan(1 ) x ®3 x 2 - 7 x + 12
(d) lim ; RHL = lim = tan 1
x ®0 sgn x h®0 1 \ 27 + 9c + 15 + 12 = 0
tan( - 1 ) Þ 9c + 54 = 0 Þ c = -6
LHL = lim = tan 1
h®0 -1 x 3 + cx 2 + 5 x + 12 3 x 2 + 2cx + 5
\ lim = lim
\ Limit exists. x ®3 x - 7 x + 12
2 x ®3 2x -7
302 Textbook of Differential Calculus

27 + 6c + 5 27 - 36 + 5 60. Domain of h ( x ) is (0, 1).


= =
9 -7 -1
61. h ( x ) = tan -1 (ln (ln 1 / x 2 )), for 0 < x < 1
\ c = -6 and l = 4 1 1
sin x 1<
2
< ¥ Þ 0 < ln 2 < ¥
x x
sin x x
56. lim = lim =¥ Þ - ¥ < ln (ln (1 / x 2 )) < ¥
x ® 0 x - sin x x®0 sin x
1- \ Range of h( x ) is ( - p / 2, p / 2 ).
x
æ sin x ö sin x - sin x
lim ç - 1÷ lim
1 Sol. (Q. Nos. 62 to 63)
x ®0 è x ø x - sin x
Hence, L = e =e x ®0 x
= e -1 = Here, K1 = (n + 1 )(n + 2 )(n + 3 ) ... (2n )
e
x K 2 = n ´ n ´ n ´ L ´ n times = nn and K 3 = (n )!
2 sin 2
x - 1 + cos x 2 =1 æ K1 ö
1/n
æ (n + 1 )(n + 2 ) K (2n ) ö
1/n
57. lim = lim 1 - 62. L = lim ç = lim ç
x®0 x x®0 x ÷ ÷
n ®¥ è K 2 ø n ®¥ è n ´ n ´ K ´ n times ø
æ x - 1 + cos x ö 1
lim ç - 1÷ ´
Hence, L =e x ®0 è x ø x Taking log on both sides, we get
1 n æ rö 1
æ xö
ç - 2 sin 2 ÷ log L = lim × å log ç1 + ÷ = ò log (1 + x ) dx = log( 4 / e )
lim ç 2÷ n ®¥ n
r =1
è nø 0
x ®0 ç x2 ÷ 2
ç ÷ -
=e è ø
=e 4 = e -1/ 2 \ L = 4/ e
K1 (n + 1 )(n + 2 ) K (n + n )
æa x + b x + c x
ç
ö 2
- 1÷
æa x - 1
ç +
b - 1 c - 1 ö÷ 2
x
+
x
63. Also, M = lim = lim
lim
x ® 0 çè 3 ÷ x
ø
lim
x ® 0 çè x x x ÷ø 3 n ®¥ K 2 + K 3 n ®¥ nn + n !
58. L = e =e
nn ((1 + 1 / n )(1 + 2 / n ) K (1 + n / n ))
2
(log a + log b + log c )
2 = lim =1
= e3 = e3
log abc
= (abc ) 2/3 n ®¥ é 1 ×2 ×3 K n ù
nn ê1 +
ë n × n K n úû
Sol. (Q. Nos. 59 to 61)
n n
æ xö æ æ x öö Sol. (Q. Nos. 64 to 65)
f ( x ) = lim ç cos ÷ = lim ç1 + ç cos - 1÷ ÷
n®¥è nø n®¥è è n øø Here, f (1 ) = 0 × 8 and g(1 ) = 0 × 6 Þ( f (1 )) 2 + ( g(1 )) 2 = 1
æ1
sin 2 çç

÷×n
\ Let f (1 ) = cos a and g(1 ) = sin a, then
n ÷ø
2
è2 æ1 xö
- lim 2 ´ çç ÷ f (2 ) = f (1 ) cos( g(1 )) - g(1 )sin( g(1 ))
æ ö n®¥ æ1 x ö
2 è2 n ÷ø
lim çç cos
x
- 1÷÷n çç ÷÷ = cos a × cos( g(1 )) - sin a × sin( g(1 )) = cos ( a + g(1 ))
n®¥ è ø è2 n ø
=e n
=e Similarly, f (n + 1 ) = cos ( a + g(1 ) + g(2 ) + K + g(n )) ...(i)
2
æ1
çç

÷ Here, for all x Î( 0, p )
è2 n ÷ø
-2 lim 1 x /n
sin x < p - x Þ x + sin x < p
n®¥ 1 - 2 lim x
-
n ® ¥ 4 1/n
=e n =e =e 2 Y
- x /2 π
y = f (x ) = e , x ³ 0, range = ( 0, 1 ]
Þ g( x ) = lim (1 - x + x n e )n
n®¥

(e1 /n - 1) π X
lim x 0
n®¥
=e 1/n
= e , " x ÎR
x sin x
y=π–x
Þ h( x ) = tan -1 ( g -1 ( f -1( x ))
Þ a + g(1 ) + g(2 ) + K + g(n ) < p
Let y 2 = f ( x ) Þ y = e - x /2
\ As n ® ¥, a + g(1 ) + g(2 ) + K + g(n ) ® p ...(ii)
x 1
- = ln y Þ x = 2 ln 64. lim f (n ) = lim cos( a + g(1 ) + g(2 ) + K + g(n ))
2 y n ®¥ n ®¥

1 = cos p = -1
Þ f -1 ( x ) = 2 ln , for 0 < x £ 1
x 65. Similarly, g(n ) = sin( a + g(1 ) + g(2 ) + K + g(n - 1 ))
y = g( x ) = e x Þ x = ln y , g -1 ( x ) = ln x Again, g(2 ) = sin( a + g(1 ))
æ 1ö æ æ 1 öö As, lim g(n ) = lim sin( a + g(1 ) + g(2 ) + K + g(n - 1 ))
\ g -1 ç2 ln ÷ = ln ç2 ln ç ÷ ÷ , for 0 < x < 1 n ®¥ n ®¥
è x ø è è x øø
= sin( p ) = 0 [using Eq. (ii)]
æ æ 1 öö
\ h( x ) = tan -1 ç ln ç ln 2 ÷ ÷ , for 0 < x < 1 Sol. (Q. Nos. 66 to 68)
è è x øø
1æ æxö æxö æ p x öö
- x /2 Here, T ( x ) = çsin ç ÷ × tan ç ÷ × cosç - ÷ ÷ ´ 2
59. lim
ln ( f ( x ))
= lim =-
1 2è è2ø è2ø è 2 2 øø
+ ln ( g ( x )) ® x 2
x®0 x 0
Chap 05 Limits 303

æxö æxö (D) lim (y 3 - x 3 ) = lim (sec 3 q - tan 3 q)


= sin 2 ç ÷ × tan ç ÷ p p
è2ø è2ø q® q®
2 2
1 1 - sin 3 q
\ S ( x ) = (1 ) 2 ×( x - sin x ) = lim
2 p cos3 q

x x 2
sin 2 × tan
T (x )
= lim 2 2 =
1 -3 sin 2 q cos q sin q
66. lim = lim = [using L’ Hospital’s rule]
x ®0 x 3 x ®0 x 3
8 p -3 cos2 q sin q cos q

S (x ) 1 ( x - sin x ) 2
67. lim = lim =0 = lim tan q = ¥
x ®0 x x ®0 2 x p

T (x ) sin 2( x / 2 ) × tan( x / 2 ) 1 x3 3 2 2
ex - 1 + x - ex + 1
68. lim = lim = lim × = 71. (A) l = lim
x ® 0 S (x ) x ® 0 1 x ® 0 4 ( x - sin x ) 2
( x - sin x ) x®0 sin 2 x 2
2 2 ×x
x2
p
(A) lim tan -1 (tan x ) = - , as lim (tan x ) = - ¥
1é x - ex + 1ù
69. 2
ex - 1
p+ 2 p+ = ê lim + lim ú
x® x®
2 2 2 êx ® 0 x2 x®0 x2 úû
ë
n
æ
1 1ö é n 1ù
(B) å 2r= ç1 - n ÷ < 1, for all n Î N .Thus, lim ê å r ú = 0
è 2 ø n®0 ê

= ê1 - lim
e - x - 1ù
x
ú
r =1 ër = 1 2 úû 2ë x ® 0 x2 û
x
(C) As x ® ¥, ® 1- 1é 1ù 1 1
x+1 = ê1 - ú = Þ = 4
2ë 2û 4 l
æ x ö 1 æ3 + x ö
and hence lim sec -1 ç ÷ doesn’t exist. æ3 + x ö
1/ x lim ç - 1÷÷
è x + 1ø x çè 3 - x ø
x®¥
(B) l = lim ç ÷ = ex ® 0
x ® 0 è3 - x ø
p
(D) Put - x = q, then the given limit is lim
2x
2
sin q sin q = e x ® 0 x ( 3 - x ) = e 2/3 Þ2 + 3 = 5
lim = lim
q ® 0 (1 - cos q) 2/ 3 q ® 0 2/ 3 q (tan 3 x - x 3 ) - (tan x 3 - x 3 )
2 × sin 4 /3 (C) lim
2 x®0 x5
q q q tan x - x 3
3
tan x 3 - x 3
2 sin cos cos = lim - lim
= lim 2 2 = 2 lim
1/ 2 2 , x®0 x 5 x®0 x5 3
q ® 0 2/ 3 q q®0 q 144244
2 × sin 4 /3 sin1/3 zero (by expansion)
2 2
(tan x - x ) (tan 2 x + x tan x + x 2 ) 1
which doesn’t exist as for lim limit is ¥ and for = lim × = ´3 =1
q ® 0+ x®0 x3 x2 3
lim limit is - ¥.
q ® 0+ (D) Rationalising gives
70. Here, y = sec q and x = tan q ( x + 2 sin x ) [ ( x 2 + 2 sin x + 1 ) + sin 2 x - x + 1 ]
lim
(A) lim ( y - x ) = lim ( sec q - tan q )
x®0 ( x 2 + 2 sin x + 1) - (sin 2 x - x + 1 )
p p
q® q® x + sin 2 x
2 2 = 2 × lim
x®0 x 2 - sin 2 x + 2 sin x + x
1 - sin q
= lim sin 2 x

p cos q 1+
x æ1 + 2ö
2
= 2 × lim =2 ç ÷ =2
x®0 sin 2 x è 3 ø
æ cos q ö
3/ 2 x- +2+1
= lim ç ÷ =0 [using L’Hospital’s rule] x
p è sin q ø

2 72. (A) l = lim cos2 ( p ( 3 n 3 + n 2 + 2n - n ))
n®¥
1 - sin q
(B) lim (y - x ) = lim (sec q - tan q) = lim Consider lim [(n 3 + n 2 + 2n )1/3 - n ]

p

p

p cos q n®¥
2 2 2
cos q é æ æ1 2 öö
1/ 3 ù
= lim =0 [using L’Hospital’s rule] = lim ên ç1 + ç + 2 ÷ ÷ - n ú
p sin q n®¥ê è èn n øø úû

2
ë
(C) lim (y 2 - x 2 ) = lim (sec 2 q - tan 2 q) = 1 éì æ1 2 öü
1 / 3 ù
p p = lim n êí1 + ç + 2 ÷ý - 1 ú
q® q® n ® ¥ êî è n n øþ úû
2 2 ë
304 Textbook of Differential Calculus

é 1 æ1 2ö ù 1 æ1ö 2 1
= lim n ê1 + ç + 2 ÷ + ... - 1 ú = lim × log ç 2 ÷ + 2 × log(sin q) [put x = 2 ]
n®¥ ë 3 èn n ø û q2
q®0 èq ø q q
1 1 1 1 1 2 log(sin q) - 2 log q
= + terms containing , 2 , 3 , … = = lim
3 n n n 3 q®0 q2
2 æpö 1 é 1ù
\ l = cos ç ÷ = 2 ê cot q - ú
è3ø 4 ë qû
= lim [using L’ Hospital’s rule]
(B) l = lim n sin (2 p 1 + n 2 - 2n p )
q®0 2q
n®¥ 2 [ q cot q - 1 ] q - tan q
= lim = 2
é 2p ( 1 + n 2 - n ) ù q®0 2 q2 q tan q
= lim n sin ê ( 1 + n 2 + n )ú q - tan q q q - tan q
n®¥ êë ( 1 + n 2 + n ) úû = lim × = lim
q®0 q3 tan q q ® 0 q3
é æ 2p ö ù
ê n sin ç ÷ ú q - (q + q /3 + L )
3

ç 1 + n2 + n ÷ æ L = lim = -1 / 3
ê è ø 2 p öú q®0 q3
= lim ê ç ÷
ç 1 + n2 + n ÷ ú -2
n®¥
ê æç 2p ö
÷
è øú \ =6
ê ç ÷ ú L
êë è 1 + n + n ø
2
úû n+ x
æ 1ö n
2np 2p 74. We have, ç1 + ÷ =e ...(i)
= lim = =p è nø
n®¥ æ 1 ö 2 On taking log both sides of Eq. (i), we get
n ç 1 + 2 + 1÷
è n ø æ 1ö
(n + xn ) loge ç1 + ÷ = 1
æ n ö è nø
(C) l = lim ( - 1 )n ( - 1 )n - 1 sin çnp - p n 2 + + 1 ÷ 1 1
n®¥ è 2 ø Þ n + xn = Þ xn = -n
æ 1 ö æ 1ö
= lim ( - 1 ) 2n - 1 sin p log ç1 + ÷ log ç1 + ÷
n®¥
è nø è nø
éæ öæ öù n+1
n n Let = m Þ nm = n + 1
ê çn - n + + 1 ÷ çn + n + + 1 ÷ ú
2 2
n
êè 2 øè 2 øú
1
ê n ú Þ n=
ê n+ n + +1
2
ú m -1
ë 2 û æ 1 1 ö
æ ö \ lim xn = lim ç - ÷
n n ®¥ m ®1 è log m m -1 ø
ç n2 - n2 - - 1 ÷
= lim ( - 1 ) 2n - 1
sin p ç 2 ÷ (m - 1 ) - log m é0 ù
n®¥ ç æ 1 1 ö÷ = lim êë 0 form úû
ç n ç1 + 1 + + ÷÷ m ®1 (m - 1 ) log m
è è 2n n 2 ø ø 1
1-
m
æ 1 1 ö = lim [using L’ Hospital’s rule]
ç + ÷ m ®1 1
(m - 1 ) × + log m
= lim ( - 1 ) sin p ç
2n 2 n ÷ m
n®¥ ç 1 1 ÷
ç1 + 1 + + ÷ 1 /m 2 1
è 2n n 2 ø = lim =
m ®1 1 / m 2 + 1 / m 2
p 1
= (1 ) sin =
4 2 \2 lim xn = 1
n ®¥
(n + 1 ) p 1
Also, as n ® ¥, sin ® é 1ù 1
4n 2 75. Let ê n + ú = K Þ K £ n + < ( K + 1)
1 ë 2û 2
\ Final answer is . Þ (K - 1 /2)2 £ n < (K + 1 /2)2
2
æx + a ö æ 2a ö Þ K 2 - K + 1 / 4 £ n < K 2 + K + 1 / 4, K Î N , n Î N
lim x çç - 1÷÷ lim x çç ÷÷
x ® ¥ è x -a ø x ® ¥ èx -aø
(D) l = e =e Þ K2 -K +1 £n £ K2 + K
¥ ¥
æ 2a ö 2 f (n ) + 2 - f (n ) 2K + 2-K
=e
lim x çç
x ®¥
÷÷
è 1 - (a / x ) ø
=e 2a So, S = å 2n
= å
2n
n =1 n =1
\ e 2a
= e Þa = 1 / 2 æ 2 + 2 -1 21 + 2 -1 ö
=ç + ÷
æ 1 ö è 2 22 ø
73. Here, L = lim x log x + 2x × logsin ç ÷,
x ®¥ è xø æ 2 2 + 2 -2 2 2 + 2 -2 2 2 + 2 -2 2 2 + 2 -2 ö
+ç + + + ÷
è 23 24 25 26 ø
Chap 05 Limits 305

ìK = 1 Þ 1 £ n £ 2 n
1 3
ïK = 2 Þ 3 £ n £ 6
ï
= lim
n ®¥
å n
- n
n=1 æ2ö æ2ö
As, í 1-ç ÷ 3 -2× ç ÷
ï K = 3 Þ 7 £ n £ 12 è3ø è3ø
ïî M M é 1 3 ù é 1 3 ù
¥ K2+K ¥ K +K 2 = lim ê - ú+ê - 2ú
+K
2K + 2-K 1 n ®¥ ë 1 - (2 / 3 ) 3 - 2 × (2 / 3 ) û
ë 1 - (2 / 3 ) 2
3 - 2 × (2 / 3 ) û
\S=å å n
= å(2 K + 2 - K ) × å n
K =1 n =K 2 -K +1
2 K =1 n =K 2 -K +1
2 é 1 3 ù
¥
+ê - nú
é 1 1 1 ù ë 1 - (2 / 3 ) 3 - 2 × (2 / 3 ) û
n
= å(2K + 2-K ) × ê 2
-K +1
+ 2
+ L+ 2 ú
K =1 2K -K + 2
ë 2K 2K +K
û é 1 1 ù
= lim ê - ú = 3 -1 = 2
ì 1 ü 1 n ®¥ 1 - 2 / 3 1 - (2 / 3 )n + 1
ë û
¥ í1 - 2 K ý
K 2 -K +1 î 2 þ
= å(2 K + 2 -K ) 2 tan x – sin {tan –1(tan x )}
K =1 1 - 1 / 2 78. We have, lim
¥ x®
p tan x + cos2(tan x )
å(2K + 2-K ) × (2-K
2 2
+K
= - 2-K -K
) 2
p
K =1 Now, RHL at x = ,
¥ 2
å(2-K
2 2 2 2
+ 2K
= - 2-K + 2-K - 2-K - 2K
)
tan x – sin {tan –1(tan x )}
K =1 = lim
¥ ¥ p+ tan x + cos2(tan x )
å(2-K å(2-K (K -2) - 2-K (K + 2) )
2
+ 2K 2 x®
= - 2-K - 2K
)= 2
K =1 K =1 tan x – sin ( x – p )
= lim
= (21 -2 -3 ) + (2 0 -2 -8 ) + (2 -3 -2 -15 ) + (2 -8 - 2 -24 ) + ... ¥ x®
p+ tan x + cos2(tan x )
=2+1=3 2
é pù
êëQ tan (tan x ) = x – p , when x > 2 úû
–1
1 × (n - 1 ) + 2 × (n - 2 ) + K+ (n - 1 ) × 1
76. Arithmetic mean =
(n - 1 )
sin x
n -1 n -1 n -1 1+
år (n - r ) n × år - år 2
= lim
tan x
=
1+0
=1
Sn = r =1
= r =1 r =1 p + 2
cos (tan x ) 1+0
(n - 1 ) (n - 1 ) x®
2
1 +
tan x
n (n - 1 ) n(n - 1 )(2n - 1 ) p
n× - Again, LHL at x = ,
2! 6 n 2 n(2n - 1 ) 2
= Þ Sn = -
(n - 1 ) 2 6 tan x – sin {tan –1(tan x )}
2 2 = lim
n n 1 p– tan x + cos2(tan x )
\ lim = = =6 x®
n ®¥ Sn n n(2n - 1 ) 1 / 2 - 2 / 6
2 2
-
2 6 tan x – sin ( x ) é pù
¥ = lim êëQtan (tan x ) = x, when x < 2 úû
–1
6n tan x + cos2(tan x )
77. Here, k = å n n n + 1 n + 1

p

n =1 (3 - 2 )(3 -2 ) 2
sin x
¥ 1–
1 1+0

tan x
= lim = =1
n =1 é æ2ö
nù é
æ3ö
n ù p– cos2(tan x ) 1+0
ê1 - ç ÷ ú ê3 × ç ÷ - 2ú x® 1+
êë è ø
3 úû êë è 2 ø úû
2
tan x
¥

1 tan x – sin {tan –1(tan x )}
\ lim =1
n =1 é æ2ö ù æ3ö é æ2ö ù
n n n

p tan x + cos2(tan x )
ê1 - ç ÷ ú × ç ÷ × ê3 - 2 × ç ÷ ú
êë è 3 ø úû è 2 ø êë è3ø ú 2
û t t sin t 2
¥
(2 / 3) × (3 - 2 )
n
79. We have, A(t ) = ò sin x 2 × dx ; B (t ) = ×
=å 0 2
n =1 é æ2ö ù é æ2ö ù
n n t t
ê1 - ç ÷ ú ê3 - 2 × ç ÷ ú A (t ) 2 ò sin x 2 × dx 2 ò sin x 2dx
êë è 3 ø úû êë è3ø ú \ = lim = lim
0 0
û lim
t ® 0 B (t ) t ®0 t sin t 2 t ®0 sin 2 t
é ù é t3 × 2
æ2ö ù
n n
æ2ö
ê 3 - 2 ç ÷ ú - 3 ê1 - ç ÷ ú t
2 ò sin x 2 × dx
t
¥
ê è3ø ú êë è 3 ø úû 2 × sin (t 2 )
=åë û = lim 0 = lim
t ®0 t3 t ®0 3t 2
n=1 é æ2ö ù é æ2ö ù
n n
ê1 - ç ÷ ú ê3 - 2 × ç ÷ ú
êë è 3 ø úû êë è3ø ú
û
[applying Newton-Leibnitz’s rule followed
by L’ Hospital’s rule]
306 Textbook of Differential Calculus

\
A(t ) 2
= cos(tan -1(tan( p / 2 + h )))
lim
t ®0
82. Here, L = lim
B(t ) 3 h®0 h
Then, m+n =2+3 =5 æ p ö
cos ç - + h ÷
é æ sin x ö ù cos(tan -1( - cot h )) è 2 ø sin h
80. Here, AB : ê ò ç
2t
+ 1 ÷ dx ú x + y = 3t = lim = lim = =1
ë 0 è x ø û h®0 h h®0 h h
\ cos(2 pL ) = cos(2 p ) = 1
and AC : 2tx + y = 0
On solving, we get
n (r - 1 )(r 2 + r + 1 )
83. Here, k = lim P
é 2t æ sin x ù n ®¥ r = 2 (r + 1 )(r 2 - r + 1 )
ö
AB : ê ò ç + 1 ÷ dx - 2t ú x = 3t [Q y = -2tx ]
ë 0 è x ø û = lim
2
´
n2 + n + 1 2
= .
3t n ®¥ n (n + 1 ) 3 3
\ xA =
æ sin x 2t ö 2
ò0 çè x + 1÷ø dx - 2t So, cosec q=
3
Þ Number of solution is zero.

3 3 lim æ ö
x ® ¥ ç 2c ÷ = 4
= = æ x +c ö
x
ç ÷
æ sin 2t ö sin 2t è x -c ø
ç + 1÷ 2 - 2 2 × 84. Here, lim ç ÷ =4 Þ e
è 2t ø x ®¥ è x - c ø
2t
3 3 Þ e 2c = 4 Þ ec =2 [only positive value]
Now, lim = [using L’Hospital’s rule]
t ®0 æ sin 2t ö 2 e c
2× ç ÷ \ =1
è 2t ø 2
p 3 æ1ö
\ = Þ p+q =5 (3 x 4 + 2 x 2 ) ×sin ç ÷ - x 3 + 5
q 2 èxø
85. lim =k
81. Let Ak = (2t, t 2 ) x ®-¥ -x3 + x2 -x +1
t2 - 1 æ1ö
\ Slope of FAk = sin ç ÷
2t - 0 æèxø 2 ö 5
ç3 + 2 ÷ ×
-1 + 3
è x ø
1 x
æp ö
= tan ç + qk ÷ Þ x
è2 ø lim
1 1 1
x ®-¥
-1 + - 2 + 3
Yy x x x
(3 - 1 ) k
Þ =k Þ =1
-1 2
F(0,1)
2x æxö
86. f ( x ) = lim tan -1 ç 2 ÷
Ak(2t, t2) t ®0 p èt ø
Case I When x > 0,
X′ Xx
O (0,0) 2x æ x ö 2x p
f ( x ) = lim × tan -1 ç 2 ÷ = ´ =x
Y′ t ®0 p èt ø p 2
2t Case II When x < 0,
tan( qk ) = = tan(2 f ), where t = tan f
1 - t2 2x æ x ö 2x æ - p ö
f ( x ) = lim × tan -1 ç 2 ÷ = × ç ÷ = -x
q kp t ®0 p èt ø p è 2 ø
\ f = k = , where tan f = t.
2 4n Case III When x = 0 Þ f ( x ) = 0
Also, FAK = (t - 1 ) + (2t ) 2 2 2 \ f ( x ) = ( x ) Þ f (1 ) = 1
æ 1 ö
= t 2 + 1 = 1 + tan 2 f 87. Let f ( x ) = çç +
1
+
1
+¼+
1 ÷
æ kp ö è n
2
n +12
n +2
2
n + 2n ÷ø
2
= sec 2 ç ÷
è 4n ø 1 1 1
As, < =
1 n
1 æ kp ö
n n + 2n
2
n 2
n2
\ lim × å FAk = lim × å sec 2 ç ÷
n®¥n
k =1
n®¥n
k =1
è 4n ø 1 1 1
< <
1 æ px ö n + 2n
2
n +12
n2
= ò sec 2 ç ÷dx
0 è 4 ø M
1 On adding, we get
4 é æ px ö ù 4
= × tan ç ÷ ú = 2n + 1 2n + 1
p êë è 4 øû0 p <
1
+
1
+ ¼+
1
<
n 2 + 2n n2 n2 + 1 n 2 + 2n n2
Hence, m = 4.
Chap 05 Limits 307

On applying, limit n ®¥ ( x - 1 )( x - 2 )( x + 3 )( x + 10 )( x + 15 ) - x 5
æ 1 1 1 1 ö = -1 - 2 + 3 + 10 + 15 = 25
2 < lim ç + + +¼+ ÷ <2
n ®¥ ç n 2 n2 + 1 n2 + 2 n 2 + 2n ÷ø \ lim (( x - 1 )( x - 2 )( x + 3 )( x + 10 )( x + 15 )1/3 - x )
è x®¥
æ 1 1 1 ö 25
\ lim ç + +¼+ ÷ =2 = =5
n ®¥ ç 2
n +1
2
n + 2n ÷ø
2 5
è n
1 1
95. Here, k = lim (1 + [ f ( x )] + x 2 - 1) { f ( x )}
x®¥
-21 - x - 2 -h
88. l = lim 2 = lim 2 =20 =1 é tan x ù 2
x ®1+ h®0 ê ú + x -1 1 + x2 - 1
ë x û
lim lim
(1 + h ) ×sin h x ®¥ ì tan x ü x ® ¥ tan x

é tan x ù
and m = lim =1 í
î x þ
ý
ë x û
ú
h®0 h =e =e x

\ l + m =2 lim
2
x
x ®¥æ 2 4 ö
x ç1 + x + x -1÷
4 × tan 4
ç
è 3 15 ÷ø
sin x tan x 2 =e = e3
89. As, = 2
x2 x ×(1 - tan 4 x / 2 )
\ [k / e ] = [e 2 ] = 7
sin x × tan x sin x × tan x
Hence, when x ®0, ®1. But >1 96. Here, ( 3 + 1) 2n = I + f …(i)
x2 x2
é sin x tan x ù Let f = ( 3 - 1) 2n
…(ii)
\ lim úû = 1
x ®0 ê
ë x2 \ I + f +F = even integer, 0 < f , f < 1
k n 2r Þ f +F =1 Þ 0 < f + F <2
90. Let Sn = S
2 r =1 1 ´ 3 ´ 5 ´¼ ´ (2r - 1 ) ´ (2r + 1 ) Þ f +F =1
k n é 1 1 ù Þ {( 3 + 1 )} 2n = f =1 - f
= Sê - ú
2 r =1 ë 1 ´ 3 ´ 5 ´¼´ (2r - 1 ) 1 ´ 3 ´ 5 ´¼´ (2r + 1 ) û \ lim f = lim 1 - f = 1 - lim ( 3 - 1 ) 2n = 1 - 0 = 1
n®¥ n®¥ n®¥
ké 1 ù
= ê1 - ú , as n ®¥. 97. Here, f ( x ) = a 0x 4 + a1x 3 + a 2x 2 + a 3x + a 4 .
2 ë 1 ´ 3 ´ 5 ´¼´ (2n + 1 ) û f (x ) f (x )
k As, lim 1 + = 2 Þ lim 2 = 1
lim Sn = = 1 Þ k 2 = 4 x®0 x2 x®0 x
n ®¥ 2 So, a 3 = a 4 = 0 and a 2 = 1
1 f ( x ) = a 0x 4 + a1x 3 + x 2
91. Here, sin 4 x = sin 2 x - ×sin 2 2x
4
æ 1 ö æ1 1 ö Þ f ¢( x ) = 4a 0x 3 + 3a1x 2 + 2 x
\ Sn = çsin 2 x - ×sin 2 2 x ÷ + ç ×sin 2 2 x - 2 ×sin 2 2 2 x ÷
è 4 ø è4 4 ø As, f ( x ) has extremum at x = 1 and x = 2.
æ1 1 ö f ¢(1 ) = 4a 0 + 3a1 + 2
+ ¼+ ç n ×sin 2 2n x - n + 1 ×sin 2 2n + 1 x ÷
è4 4 ø and f ¢(2 ) = 32a 0 + 12a1 + 4
1 1
= sin 2 x - ×sin 2 2n + 1 x Þ f ( x ) = sin 2 x, g( x ) = cos2 x
n +1
So, a 0 = and a1 = - 1
4 4
\ [ f ( x ) + g( x ) ]4 = 4 Þ f (2 ) = 0
92. Here, f (3) = 3 f (1) = 1 98. Here, | x | = log({ x }) has no solution.
f ( 4 ) = f (2 + 1 + 1 ) = f (2 ) + f (1 ) + f (1 ) = 2 + 1 + 1 = 4 and so on. Hence, a = 0
æa2 b ö
In general, f ( x ) = x, for x ÎN (1 - b ) x + ax 2 + x 3 ç - ÷
xe ax
- b sin x è 2 6ø
n ( 4r ) f (3r ) 12n (n + 1 )(2n + 1 ) \ lim = lim
\ lim S = =4 x®0 x3 x®0 x3
n ®¥ r =1 n3 6n 3
Þ a = 0, b = 1
x -1 Þ a + b =1
93. As, 0 £ < 1 Þ1 £ x £ 4
3
n 1 x -1 99. Here, sin( xn + 1 - xn ) + 2 -(n + 1) sin xn × sin xn + 1 = 0
Then, lim = , when 0 £ <1
n ®¥ æ x -1 ö
n
3 3 Þ cot xn + 1 - cot xn = 2 - (n + 1)
n×ç ÷ + n × 3 - 1
è 3 ø Þ cot xn - cot xn - 1 = 2 -n
Þ x =1, 2, 3 1 1 1
\ Number of integral values = 3. Þ cot xn + 1 - cot x1 = 2
+ 3 + ¼+ n +1
2 2 2
94. As, x 5 - y 5 = ( x - y )( x 4 + x 3y + x 2y 2 + xy 3 + y 4 ) 1 1 1
Þ cot xn + 1 = + 2 + ¼ n+1
and the coefficient of xn in 2 2 2
308 Textbook of Differential Calculus

Þ lim cot xn + 1 = 1 x
n®¥
and G (x ) = ò–1t f { f (t )} dt
p Þ G ¢( x ) = x f { f ( x )} …(iii)
\ lim xn + 1 =
n®¥ 4 F (x ) F ¢( x ) f (x )
p \ lim = lim = lim
x ®1 G ( x ) x ®1 G ¢( x ) x ®1 x f { f ( x )}
l = Þ 4l = p
4 1
\ [ 4l ] = 3 f (1 )
= = 2 ...(iv)
x 2 sin (bx ) 1 f { f (1 )} æ1ö
100. Here, lim =1 fç ÷
x ® 0 ax - sin x è2ø
æ (bx ) 3 (bx ) 5 ö F (x ) 1
x 2 ç bx - + - K÷ Given, lim =
è 3! 5! ø x ®1 G( x ) 14
Þ lim =1
x®0 æ x3 x5 ö 1
ax - ç x - + - K÷ 2 1 æ1ö
è 3! 5! ø \ = Þ f ç ÷ =7
æ1ö 14 è2ø
fç ÷
æ b 3x 2 b 5x 4 ö è2ø
x 3 çb - + - K÷
è 3 ! 5 ! ø (1 + x ) (1 - x )
Þ lim 3 5
=1 ì sin ( x - 1 ) + a (1 - x )ü 1- x 1
x®0 x x 103. Given, lim í ý =
(a - 1)x + + -K x ® 1 î ( x - 1 ) + sin ( x - 1 ) þ 4
3! 5!
1+
Limit exists only, when a - 1 = 0 ì sin ( x - 1 ) ü
x

Þ a =1 …(i) ïï ( x - 1 ) - a ïï 1
lim í =
3æ b 3x 2 b 5x 4 ö x ®1 sin( x - 1 ) ý 4
x çb - + - K÷ ï1 + ï
è 3! 5! ø ïî ( x - 1 ) ïþ
\ lim =1
3æ 1 ö
x®0 2 2
x ç -
x
- K÷ æ1 - a ö 1
Þ ç ÷ =
è 3! 5! ø è 2 ø 4
Þ 6b = 1 …(ii) Þ (a - 1 ) 2 = 1
From Eqs. (i) and (ii), we get Þ a = 2 or 0
6( a + b ) = 6 a + 6 b = 6 + 1 = 7 Hence, the maximum value of a is 2.
é e cos ( an ) - e ù e
101. Given, lim ê ú=- x2
a®0 ê am a - a2 - x2 -
ë ú
û
2 4 ,a > 0
104. L = lim
n x®0 x4
e {e cos( a ) -1 - 1 } cos( an ) - 1 - e
Þ lim × =
a ® 0 cos( an ) - 1 am 2 é 1 æ1 ö ù
ê 1 x 2 2 çè 2 - 1 ÷ø x 4 ú x2
an
a - a × ê1 - × 2 + × 4 - ...ú -
ìï e cos( an ) -1 - 1üï -2 sin 2 ê 2 a 2 a ú 4
Þ lim e í 2 =-e
ý × lim êë úû
a®0
îï
cos( a n
) - 1
þï
a®0 am
2 = lim
x®0 x4
æ an ö
sin 2 ç ÷ x2 1 x4 x2
è 2 ø a 2n -e + × 3 + ... -
Þ e ´ 1 ´ ( -2 ) lim × m = = lim 2a 8 a 4 4
a®0 a 2n 4a 2 x®0 x
4
Since, L is finite ⇒ 2a = 4 ⇒ a = 2
a 2n - m - e
Þ e ´ 1 ´ - 2 ´ 1 ´ lim = 1 1
a®0 4 2 ∴ L = lim =
x ® 0 8× a 3 64
m
For this to be exists, 2n - m = 0 Þ =2 cot x (1 - sin x )
n 105. lim
p 3
F (x ) 1 x® æ pö
102. Here, lim = 2 - 8 çx - ÷
x ®1 G ( x ) 14 è 2ø
F ¢( x ) 1 æp ö æ æp öö
Þ lim = [using L’Hospital’s rule] …(i) tan ç - x ÷ ç1 - cos ç - x ÷ ÷
x ®1 G ¢( x ) 14 è2 ø è è2 øø 1 1 1
= lim = ×1 × =
x p æ p ö æp ö
2
8 2 16
As, F ( x ) = ò f (t ) dt x®
2 8 ç - x÷ ç - x÷
–1 è2 ø è2 ø
Þ F ¢( x ) = f ( x ) …(ii)
Chap 05 Limits 309

1
(1 – cos2 x )(3 + cos x ) 2 sin 2 x (3 + cos x )
106. Given, p = lim (1 + tan 2 x ) 2x [1 ¥ form] 109. We have, lim = lim
x ® 0+
x ®0 x tan 4 x x ®0 tan 4 x
x´ ´ 4x
2 4x
tan 2 x 1 æ tan x ö
lim
x ® 0+
lim ç
2 x ® 0 + çè
÷
x ÷ø
1
2 sin 2 x (3 + cos x ) 1
=e
2x
=e = e2 = lim ´ lim ´
x ®0 x 2 x ®0 4 tan 4x
1 lim
1 x ® 0 4x
\ log p = log e 2 = 4 é sin q tan q ù
2 = 2 ´ ´1 = 2 êëQ qlim = 1 and lim = 1ú
1 4 ®0 q q®0 q û
é (n + 1 ) × (n + 2 ) K (3n ) ù n
107. Let l = lim ê úû æ ¥ö
n®¥ë n 2n 110. Plan ç ÷ form
è ¥ø
1
é (n + 1 ) ×(n + 2 ) ... (n + 2n ) ù n ì 0, if n < m
= lim ê úû ï a0
n®¥ë n 2n n -1
a 0x + a1x + K + an ï ,
n if n = m
1 lim = í b0
x ® ¥ b 0x m + b1x m - 1 + K + bm
éæn + 1ö æn + 2ö æn + 2n ö ù n ï + ¥, if n > m and a 0b0 > 0
= lim ê ç ÷ç ÷Kç ÷
n ® ¥ ëè n ø è n ø è n ø úû ï - ¥,
î if n<m and a 0b0 < 0
Taking log on both sides, we get Description of Situation As to make degree of numerator
1 é ìæ 1ö æ 2ö æ 2n öü ù equal to degree of denominator.
log l = lim ê log í ç1 + ÷ ç1 + ÷ ... ç1 + ÷ý ú
n ® ¥n ë î è n ø è n ø è n øþ û æ x2 + x +1 ö
\ lim ç - ax - b ÷ = 4
Þ log l = lim
1 x ® ¥è x +1 ø
n ® ¥n

é 2n ö ù x 2 + x + 1 - ax 2 - ax - bx - b
æ 1ö æ 2ö æ Þ lim =4
ê log çè1 + n ÷ø + log çè1 + n ÷ø + ... + log çè1 + n ÷ø ú x®¥ x +1
ë û
2n x 2(1 - a ) + x(1 - a - b ) + (1 - b )
1 æ rö Þ =4
å log çè1 + n ÷ø
lim
Þ log l = lim x®¥ x +1
n ® ¥n
r =1
2 Here, we make degree of numerator = degree of denominator
Þ log l = ò log (1 + x ) dx \ 1 -a = 0
0
2 Þ a =1
é 1 ù
Þ log l = ê log (1 + x ) × x - ò × x dx ú x (1 - a - b ) + (1 - b )
ë 1+x û0 and lim =4
x®¥ x +1
2 x + 1 -1
Þ log l = [log (1 + x ) × x ]20 - ò dx Þ 1 -a -b = 4
0 1+x
Þ b = -4 [Q (1 - a ) = 0 ]
2æ 1 ö
Þ log l = 2 × log 3 - ò ç1 - ÷ dx 111. Plan To make the quadratic into simple form, we should
0 è 1 + xø eliminate radical sign.
Þ log l = 2 × log 3 - [ x - log 1 + x ]20 Description of Situation As for given equation, when a ® 0
the equation reduces to identity in x.
Þ log l = 2 × log 3 - [2 - log 3 ]
i.e. ax 2 + bx + c = 0, " x Î R or a = b = c ® 0
Þ log l = 3 × log 3 - 2
Þ log l = log 27 - 2 Thus, first we should make above equation independent from
coefficients as 0.
\ l = e log 27 - 2
Let a + 1 = t 6. Thus, when a ® 0, t ® 1.
27
= 27 × e - 2 = 2 \ (t 2 - 1 ) x 2 + (t 3 - 1 ) x + (t - 1 ) = 0
e
Þ (t - 1 ) {(t + 1 ) x 2 + (t 2 + t + 1 ) x + 1 } = 0, as t ® 1
sin( p cos2 x ) sin p (1 - sin 2 x )
108. lim 2
= lim 2x 2 + 3x + 1 = 0
x ®0 x x ®0 x2
sin( p - p sin 2 x ) Þ 2x 2 + 2x + x + 1 = 0
= lim
x ®0 x2 Þ (2 x + 1 ) ( x + 1 ) = 0
sin( p sin x ) 2 Thus, x = - 1, - 1 / 2
= lim [Q sin ( p - q) = sin q]
x ®0 x2 or lim a (a ) = - 1 / 2
a ® 0+
sin p sin x 2 æ sin x ö 2
é sin q ù
= lim ´ (p ) ç 2 ÷ = p = 1ú and lim b (a ) = - 1
x ®0 p sin 2 x è x ø êëQ qlim
®0 q û a ® 0+
310 Textbook of Differential Calculus

112. Here, lim {1 + x log (1 + b 2 )}1/x [1 ¥ form] {(a - n ) nx - tan x } sin nx


x®0 115. Given, lim =0
1
x®0 x2
lim { x log (1 + b 2 )} ×
= ex ® 0 x ì tan x ü sin n x
Þ lim í(a - n ) n - ý ´n = 0
x ® 0î x þ nx
2
= e log (1 + b )
= (1 + b 2 ) …(i) Þ {(a - n ) n - 1 } n = 0
Given, lim {1 + x log (1 + b )} 2 1/ x
= 2b sin q 2
Þ (a - n ) n = 1
x®0
1
Þ a =n +
Þ (1 + b 2 ) = 2b sin 2 q n
1 + b2 (cos x - 1 ) (cos x - e x )
\ sin 2 q = …(ii) 116. lim
2b x®0 xn
b+
1
1/ 2 ìæ x2 x4 ö ü
b ³ æçb × 1 ö÷ ï ç1 - + - ...÷ ï
By AM ³ GM, è 2 ! 4 ! ø
è bø æ 2 xöï ï
2 ç - 2 sin ÷í ý
è 2øï æ x 2
x 3 öï
b2 + 1 - ç 1 + x + + + ... ÷
Þ ³1 …(iii) ï è 2! 3 ! øïþ
2b = lim î
From Eqs. (ii) and (iii), we get x®0 xn
sin 2 q = 1 æ 2 xöæ 2x 2 x 3 ö
ç - 2 sin ÷ ç - x - - - ...÷
è ø
2 è 2! 3! ø
p = lim
Þ q=± , as q Î ( - p , p ] x®0 2
2 æ ö
x
4 ç ÷ xn - 2
sin x è2ø
æ1ö
113. Here, lim (sin x )1/x + lim ç ÷
x®0 x®0 èxø x æ x2 ö
sin x sin 2 ç1 + x + + K÷
æ1ö
log ç ÷ lim
log (1/ x ) é lim (sin x )1/x ® 0 ù 2 è 3! ø
= 0 + lim e èxø
= ex ® 0 cosec x êx ® 0 ú = lim 2
x ®0 æxö
x®0 ê as, (decimal) ¥ ® 0 ú 2 ç ÷ xn - 3
ë û è2ø
Applying L’ Hospital’s rule, we get Above limit is finite, if n - 3 = 0, i.e. n = 3.
æ 1 ö
x ç- ÷ 1/ x
è x2 ø sin x é f (1 + x ) ù
lim
x ®0 -
lim
x ®0 x
tan x 117. Let y = ê ú
e cosec x cot x
=e =e = 1
0
ë f (1 ) û
f (2h + 2 + h 2 ) - f (2 ) Þ log y =
1
[log f (1 + x ) - log f (1 )]
114. Here, lim
h®0 f (h - h 2 + 1 ) - f (1 ) x
[Q f ¢ (2 ) = 6 and f ¢ (1 ) = 4, given] é 1 ù
Þ lim log y = lim ê × f ¢ (1 + x ) ú
x®0 x ® 0 ë f (1 + x ) û
Applying L’ Hospital’s rule,
{ f ¢ (2h + 2 + h 2 )} × (2 + 2h ) - 0 [using L’Hospital’s rule]
= lim f (1 ) 6
h ® 0 { f ¢ (h - h 2 + 1 )} × (1 - 2h ) - 0 = =
f (1 ) 3
f ¢ (2 ) × 2
=
f ¢ (1 ) × 1 æ ö
Þ log ç lim y ÷ = 2 Þ lim y = e 2
6 .2 è x ®0 ø x ®0
= =3 [using f ¢(2 ) = 6 and f ¢ (1 ) = 4]
4 .1 æ x - 3ö
x
(1 - 3 / x ) x e -3
118. For x Î R , lim ç ÷ = lim = 2 = e -5
x ® ¥ è x + 2ø x ®¥ (1 + 2 / x ) x e
CHAPTER

06
Continuity and
Differentiability
Learning Part
Session 1
● Continuous Function

● Continuity of a Function at a Point

Session 2
● Continuity in an Interval or Continuity at End Points

Session 3
● Discontinuity of a Function

Session 4
● Theorems Based on Continuity

● Continuity of Composite Function

Session 5
● Intermediate Value Theorem

Session 6
● Differentiability : Meaning of Derivative

Session 7
● Differentiability in an Interval

Practice Part
● JEE Type Examples
● Chapter Exercises

Arihant on Your Mobile !


Exercises with this #L
symbol can be practised on your mobile. See title inside to activate for free.
Session 1
Continuous Function,
Continuity of a Function at a Point
Introduction to Continuity (i) lim – f ( x ) and lim+ f ( x ) exist but are not equal.
x ®a x ®a
A real function is continuous at a fixed point if we can (ii) lim – f ( x ) and lim+ f ( x ) exist and are equal but not
draw the graph of the function around that point without x ®a x ®a
lifting the pen from the plane of the paper. In case one has equal to f (a ) .
to lift the pen at a point, the graph of the function is said (iii) f (a ) is not defined.
to have a break or discontinuous at that point, say x = a.
(iv) Atleast one of the limit doesn’t exist.
Different types of situations, which may come up at x = a
along the graph, can be shown as below Now, we will discuss different conditions for a function to be
Y Y
discontinuous in detail
(i) lim- f ( x ) and lim+ f ( x ) exists but are not equal.
x ®a x ®a

Here, lim f ( x ) = l 1
x ®a–

x=a X x=a X and lim f ( x ) = l 2


O O x ®a+
Continuous at x = a Discontinuous at x = a \ lim – f ( x ) and lim+ f ( x ) exist but are not equal.
Y x ®a x ®a
Y
Thus, f ( x ) is discontinuous at x = a. It does not
matter whether f (a ) exist or not.
Graphically This conditions can be illustrated as
Y
x=a X
X O l1
O x=a f (x)
Discontinuous at x = a Discontinuous at x = a
Y Y l2

x=a X
O

Figure 6.2
x=a X X
O O |x|
x=a y Example 1 If f ( x ) = . Discuss the continuity at x = 0.
Discontinuous at x = a Discontinuous at x = a x
Figure 6.1
|x |
l There should be heading ‘‘continuity of a function at Sol. Here, f (x ) =
x
point’’
\ RHL at x = 0
l A function f ( x ) is said to be continuous at x = a; where
a Î domain of f ( x ), if lim – f ( x ) = lim+ f ( x ) = f (a ) Let x =0+h
x ®a x ®a | 0 + h| h
i.e. LHL = RHL = Value of a function at x = a i.e. lim f ( x ) = lim f (0 + h ) = lim = lim = 1
x ® 0+ h ®0 h ®0 0 + h h ®0 h
or lim f ( x ) = f (a )
x ®a Þ lim f ( x ) = 1
x ® 0+
l If f ( x ) is not continuous at x = a, we say that f ( x ) is
discontinuous at x = a, f ( x ) will be discontinuous at x = a Again, LHL at x = 0
in any of the following cases Let x = 0–h
Chap 06 Continuity and Differentiability 313

i.e. lim f ( x ) = lim f (0 – h ) ì2x + 3, when x < 0


x ® 0– h ®0
| 0 – h| h ï
= lim = lim
®
= –1 y Example 2 If f ( x ) = í0, when x = 0.
h ®0 0–h h 0 –h
ï x 2 + 3, when x > 0
Þ lim f ( x ) = – 1 Þ lim + f ( x ) ¹ lim – f ( x ). î
x ® 0– x ®0 x ®0
Discuss the continuity at x = 0.
Thus, f ( x ) is discontinuous at x = 0.
Graphically Here, ì2x + 3, when x < 0
ï
Y
Sol. Here, f ( x ) = í0, when x = 0
ï x 2 + 3, when x > 0
î
1
\ RHL at x = 0, let x = 0 + h
X
O i.e. lim f ( x ) = lim f (0 + h )
–1 x ® 0+ h ®0
= lim {(0 + h )2 + 3 } = 3
h ®0

ì x Þ lim + f ( x ) = 3
, x >0
|x | ï x ì 1, x > 0 x ®0
f (x ) = =í =í Again, LHL at x = 0, let x = 0 – h
–1, x < 0
– , x <0 î
x ï x
î x i.e. lim – f ( x ) = lim f (0 – h )
x ®0 h ®0
0
and f (0) = [indeterminate form] = lim {2(0 – h ) + 3 } = 3
0 h ®0

Þ It is not defined. Þ lim f ( x ) = 3


x ® 0–
Which shows, the graph is broken at x = 0. But f ( 0) = 0
Where, lim – f ( x ) = – 1 and lim + f ( x ) = – 1
x ®0 x ®0 Therefore, lim f ( x ) = lim – f ( x ) = 3 ¹ f (0)
x ® 0+ x ®0
Thus, lim f ( x ) doesn’t exist and hence function is
x ®0 Thus, f ( x ) is discontinuous at x = 0.
discontinuous. Graphically
Remark Y
0
Here, f ( x ) is not defined at x = 0, as f( 0 ) = . [indeterminate form]
0
So, we could say directly that the function is discontinuous at x = 0 3 f (x) = x2 + 3

(ii) lim- f ( x ) and lim+ f ( x ) exist and are equal but


x ®a x ®a
not equal to f (a ) . X′ X
O
Here, lim f ( x ) = lim+ f ( x ) = L f (x) = 2x+3 f (0) = 0
x ®a– x ®a

f (a ) is also defined but f (a ) ¹ L.


Y′
So, limit of f ( x ) exists at x = a. But, it is not
continuous at x = a. Here, lim – f ( x ) = 3
x ®0
Graphically This conditions can be illustrated as lim f ( x ) = 3
x ® 0+
Y f ( 0) = 0
f (x) Thus, lim f ( x ) = lim + f ( x ) = 3 ¹ f (0)
x ® 0– x ®0
f (a) Hence, f ( x ) is discontinuous at x = 0.
L (iii) f (a ) is not defined.
Here, lim+ f ( x ) = L
x ®a
X′ a X
O
and lim f ( x ) = L
x ®a–

Y′ But, f (a ) is not defined. So, f ( x ) is discontinuous


Figure 6.3 at x = a.
314 Textbook of Differential Calculus

Graphically This conditions can be illustrated as ì2x + 3, – 3 £ x < – 2


ï
Y Sol. Graphically f ( x ) = í x + 1, – 2 £ x < 0 is plotted as shown
ï x + 2, 0£ x £1
î
L
Y
x+2
3

X′ x=a X 2
O
1
f(x) = x+1
X′ X
–2 –1 0 1
Y′ –1
Figure 6.4
–2
x2 –1 f(x)=2x+3
y Example 3 If f ( x ) = . Y′
–3
x–1
Here, if we observe the graph we could conclude that at
Discuss the continuity at x = 1.
x = 0, lim – f ( x ) = 1 and lim + f ( x ) = 2, which shows that
x2 –1 x ®0 x ®0
Sol. Here, f (x ) =
x –1 the function is discontinuous at x = 0 and continuous at
every other point in [– 3,1].
x2 –1
lim f ( x ) = lim
x ®1 x ®1 x – 1 y Example 5 Examine the function,
( x – 1)( x + 1) ì cos x
= lim ï , x ¹ p/ 2
x ®1 ( x – 1) f ( x ) = í p/ 2 – x for continuity at x = p/ 2.
ï1, x = p/ 2
= lim ( x + 1) = 2 î
x ®1
cos x
0 Sol. We have, lim f ( x ) = lim
But f ( 1) = [indeterminate form] x ® p/ 2 x ® p/ 2 p/ 2 – x
0
[as x ¹ p/ 2 but x ® p/ 2]
\ f (1) is not defined at x = 1.
cos x é0 ù
Hence, f ( x ) is discontinuous at x = 1. = lim êë 0 form úû
p/ 2 – x
x ® p/ 2
Graphically
–sin x
= lim [applying L’Hospital’s rule]
Y x ® p/ 2 0 – 1

p
Þ lim f ( x ) = sin =1
2 x2 – 1 (x + 1) x ® p/ 2 2
f (x) = =
x–1
Also, f ( p / 2) = 1
1
\ lim f ( x ) = f ( p/ 2)
X′ X x ® p/ 2
0 1
Thus, f ( x ) is continuous at x = p/ 2.

Y′ y Example 6 Discuss the continuity of f ( x ) = [tan –1 x ].


Here, lim f ( x ) = 2 but f (1) is not defined. Sol. We know, y = tan –1 x could be plotted as
x ®1
Y
So, f ( x ) is discontinuous at x = 1. π

2 tan–1 x
y Example 4 Show that the function
ì2x + 3, – 3 £ x < – 2 1
ï X′ X
f ( x ) = í x + 1, – 2 £ x < 0 –tan 1 0 tan 1
ï x + 2, 0£ x £1 –1
î
–π
is discontinuous at x = 0 and continuous at every other –
2
point in interval [– 3, 1]. Y′
Chap 06 Continuity and Differentiability 315

Thus, f ( x ) = [tan –1 x ] could be plotted as y Example 8 Let


Y e tan x - e x + log (sec x + tan x ) - x
π
– f (x ) = be a
2
tan x - x
1
continuous function at x = 0. The value of f (0) equals
1 2
X′
0 tan 1
X (a) (b)
–tan 1 2 3
–1
3
(c) (d) 2
2 2
Y′ Sol. For continuity at x = 0, we have
Which clearly shows the graph is broken at f (0) = lim f ( x )
{–tan 1, 0, tan 1}. x ®0
\ f ( x ) is not continuous when x Î {–tan 1, 0, tan 1}. e tan x - e x log(sec x + tan x ) - x
= lim + lim
x ® 0 tan x - x x ®0 tan x - x
y Example 7 Let y = f ( x ) be defined parametrically as
y = t 2 + t |t|, x = 2t – |t |, t ÎR . Then, at x = 0, find f ( x ) (e tan x - x - 1) sec x - 1
= lim e x + lim
x ®0 (tan x - x ) x ®0 sec 2 x - 1
and discuss continuity.
Sol. As, y = t 2 + t | t | and x = 2t – | t | [using L’Hospital’s rule]
1 1 3
Thus, when t ³ 0 = 1 + lim =1+ =
x ® 0 sec x + 1 2 2
Þ x = 2t – t = t , y = t 2 + t 2 = 2t 2
Hence, (c) is the correct answer.
\ x =t and y = 2t 2 Þ y = 2x 2 , " x ³ 0
1
Again, when t < 0 y Example 9 If f ( x ) = -1
, then f ( x ) is
tan ( x - 4 x + 3) 2
Þ x = 2t + t = 3t and y = t 2 – t 2 = 0 Þ y = 0, " x < 0
continuous for
ì2x 2 , x ³ 0
Hence, f (x ) = í (a) (1, 3) (b) (- ¥, 0)
î 0, x < 0 (c) (- ¥, 1) È (3, ¥) (d) None of these
which is clearly continuous for all x as shown graphically. Sol. Here,
Y
1
28 f (x ) = -1
26 tan ( x - 4 x + 3)
2
24
-1
22 For domain, tan ( x - 4 x + 3) > 0
2

20
18 Þ x 2 - 4x + 3 > 0
16 Þ ( x - 1)( x - 3) > 0
14
12 + +
10
1 – 3
8
6 \ x Î ( - ¥, 1) È (3, ¥ )
4 Since, every general function is continuous in its domain.
2
X′ X \ f ( x ) is continuous for x Î ( - ¥, 1) È (3, ¥ ).
O 1 2 3 4 5 6 7 8 9 10 11 12 13 14 15
Hence, (c) is correct answer.
Y′
316 Textbook of Differential Calculus

Exercise for Session 1


1+ x - 3 1+ x
1. If function f ( x ) = is continuous function at x = 0, then f (0) is equal to
x
1 1 1
(a) 2 (b) (c) (d)
4 6 3
ì 1
ï , x ¹0
2. If f ( x ) = í e1/ x + 1 , then
ïî 0, x =0
(a) lim- f (x ) = 0 (b) lim+ f (x ) = 1
x ®0 x ®0
(c) f (x ) is discontinuous at x = 0 (d) f (x ) is continuous at x = 0
ì x 2 - (a + 2)x + 2a
ï , x ¹2
3. If f ( x ) = í x -2 is continuous at x = 2 , then a is equal to
ïî 2, x =2
(a) 0 (b) 1 (c) -1 (d) 2
ì log (1+ 2ax ) - log(1- bx )
ï , x ¹0
4. If f ( x ) = í x is continuous at x = 0, then k is equal to
îï k, x =0
(a) 2a + b (b) 2a - b (c) b - 2a (d) a + b
ì[ x ] + [ - x ], x ¹ 2
5. If f ( x ) = í and f is continuous at x = 2, where [ × ] denotes greatest integer function, then l is
î l, x =2
(a) - 1 (b) 0 (c) 1 (d) 2
Session 2
Continuity in an Interval or Continuity at End Points
Let a function y = f ( x ) be defined on [a, b ]. Then, Here, in the graph f ( x ) is continuous at all points, where
the function f ( x ) is said to be continuous at the left end 1 < x < 2.
x =a To check continuity at x = 1 and x = 2
If f (a ) = lim f ( x ) [need not check LHL] (i) To check continuity at x = 1
x ®a+ Here, f (1) = 1 and RHL at x = 1
The function f ( x ) is said to be continuous at the Þ lim f ( x ) = 1, i.e. f (1) = lim f ( x )
x ® 1+ x ® 1+
right end x =b
Thus, f ( x ) is continuous at x = 1.
If f (b ) = lim f ( x ) [need not check RHL]
x ®b –
(ii) To check continuity at x = 2
y Example 10 If f ( x ) = [ x ] , where [ × ] denotes greatest Here, f ( 2) = 2
integral function. Then, check the continuity on [1, 2]. But LHL at x = 2 Þ lim f ( x ) = 1
x ® 2–

Sol. Graphically f ( x ) = [ x ] could be plotted as which shows f (2) ¹ lim – f ( x )


x ®2
Y
Thus, f ( x ) is discontinuous at x = 2.
2
From the above information, it becomes clear that f ( x ) is
1 continuous at all points on [1, 2] except at x = 2,
i.e. f ( x ) is continuous for x Î[1, 2).
X
0 1 2

Exercise for Session 2


ì p
ï -2 sin x for -p £ x £ -
2
ïï p p
1. Let f ( x ) = ía sin x + b for - <x < . If f is continuous on [ -p , p ), then find the values of a and b .
ï 2 2
ïcos x p
for £x £p
ïî 2
2. Draw the graph of the function f ( x ) = x - | x - x 2|, - 1 £ x £ 1 and discuss the continuity or discontinuity of f in
the interval - 1 £ x £ 1.
ì| 4x - 5| [ x ] for x > 1
3. Discuss the continuity of ‘f ’ in [0, 2] , where f ( x ) = í ; where [ x ] is greatest integer not
î [cos px ] for x £ 1
greater than x.
ì Ax - B, x £ –1
ï
4. Let f ( x ) = í2x 2 + 3Ax + B , x Î ( - 1, 1]
ï x >1
î 4,
3 1
Statement I f ( x ) is continuous at all x, if A = and B = - .
4 4
Statement II Polynomial function is always continuous.
A. Both Statement I and Statement II are correct and Statement II is the correct explanation of Statement I
B. Both Statement I and Statement II are correct but Statement II is not the correct explanation of Statement I
C. Statement I is correct but Statement II is incorrect
D. Statement II is correct but Statement I is incorrect
(a) A (b) B (c) C (d) D
Session 3
Discontinuity of a Function
A function f which is not continuous, said to be Shown as,
discontinuous functions, i.e. if there is a break in the Y

function of any kind, then it is discontinuous functions.


There are two types of discontinuity
1. Removable discontinuity
2. Non-removable discontinuity X′ X
–2 0 2
Y′

Removable Discontinuity sin x


Figure 6.6

In this type of discontinuity, lim f ( x ) necessarily (iii) f ( x ) = has missing point discontinuity at x = 0.
x ®a x
exists, but is either not equal to f (a ) or f (a ) is not defined. Shown as,
Such function is said to have a removable discontinuity of Y
the first kind.
In this case, it is possible to redefine the function in such a 1
manner that lim f ( x ) = f (a ) and thus making function
2
continuous. x ® a π

These discontinuities can be further classified as X′ π π 0 π π X


2 2
(i) Missing point discontinuity
Y′
(ii) Isolated point discontinuity Figure 6.7

Examples of Missing Point Discontinuity Examples of Isolated Point Discontinuity


Here, lim f ( x ) exists finitely but f (a ) is not defined. e.g. Here, lim f ( x ) exists and f (a ) also exists but
x ®a x ®a
( x - 1) (9 - x )
2
lim f ( x ) ¹ f (a ). e.g.
(i) Let f ( x ) = , x ®a
( x - 1)
ì 0, if x Î I
0 (i) Let f ( x ) = [ x ] + [ - x ] Þ f ( x ) = í
clearly f (1) ® form î- 1, if x Ï I
0
where x = Integer, has isolated point discontinuity,
\ f ( x ) has missing point discontinuity. Shown as, can be shown as
Y Y

X′ X
–4 –3 –2 –1 0 1 2 3 4 5

X′ X –1
–3 0 1 3

Y′ Y′
Figure 6.5 Figure 6.8
(ii) Let f ( x ) = sgn (cos 2 x - 2 sin x + 3 )
x2 - 4
(ii) f ( x ) = has missing point discontinuity at Þ f ( x ) = sgn (1 - 2 sin2 x - 2 sin x + 3 )
x -2
x = 2. = sgn (2 (2 + sin x ) (1 - sin x ))
Chap 06 Continuity and Differentiability 319

ì p |sin x |
(ii) f ( x ) = at x = 0
ï0, if x = 2np + 2 x

p
ï1, if x ¹ 2np + RHL i.e. f (0 + ) = 1 ü
î 2 ý jump = 2
LHL i.e. f (0 - ) = - 1þ
p
\ f ( x ) has an isolated point discontinuity at x = 2np + .
2
Shown as,
Y
(ii) Infinite Discontinuity
If for a function f ( x ) : lim - f ( x ) = L1 and lim+ f ( x ) = L2
x ®a x ®a
and either L1 or L2 is infinity, then such function is said
to have infinite discontinuity.
X′ X
7π 3π π 5π
2 2
O
2 2 In other words, If x = a is a vertical asymptote for the
graph of y = f ( x ), then f is said to have infinite
Y′ discontinuity at a.
Figure 6.9
Examples of Infinite Discontinuity
x
(i) f ( x ) = , at x = 1
Non-removable Discontinuity 1- x
In this type of discontinuity lim f ( x ) doesn’t exist and
x ®a RHL i.e. f (1 + ) = - ¥
therefore, it is not possible to redefine the function in any
LHL i.e. f (1 - ) = ¥
manner and make it continuous. Such function is said to
1
have non-removable discontinuity or discontinuity of (ii) f ( x ) = , at x = 0
x2
second kind.
RHL i.e. f (0 + ) = ¥
Such discontinuities can further be classified into three
types. LHL i.e. f (0 - ) = ¥

(i) Finite Discontinuity (iii) Oscillatory Discontinuity


If for a function f ( x ) : lim - f ( x ) = L1 and lim+ f ( x ) = L2 If for a function f ( x ) : lim f ( x ) doesn’t exist but oscillate
x ®a
x ®a x ®a between two finite quantities, then such function is said to
and L1 ¹ L2 , then such function is said to have a finite
have oscillatory discontinuity.
discontinuity or a jump discontinuity.
Examples of Oscillatory Discontinuity
In this case, the non-negative difference between the two
æ 1ö
limits is called the Jump of Discontinuity. A function (i) f ( x ) = sin ç ÷
èxø
having a finite number of jumps in a given interval is called
a Piecewise Continuous or Sectionally Continuous Þ lim f ( x ) = a value between - 1 to 1.
x ®0
Function.
\ Limit doesn’t exist, as it oscillates between - 1 and
Examples of Finite Discontinuity 1 as x ® 0.
æ 1ö
(i) f ( x ) tan -1 ç ÷ at x = 0 æ 1ö
(ii) f ( x ) = cos ç ÷
èxø èxø
p ü
RHL i.e. f (0 + ) = æ 1ö
2 ï jump = p Þ lim cos ç ÷ = a value between - 1 to 1.
èxø

x ®0
LHL i.e. f (0 - ) = - ï
2þ \ Limit doesn’t exist, as it oscillates between - 1 to 1
at x ® 0.
320 Textbook of Differential Calculus

ì e 1/x – 1
List of Continuous Functions ï
Sol. We have, f ( x ) = í e 1/x + 1
, when x ¹ 0
Function f ( x ) Interval in which f (x ) ï 0, when x = 0
is continuous î
1. constant c (– ¥, ¥ ) \ RHL at x = 0, let x = 0 + h
2. xn , n is an integer ³ 0 (– ¥, ¥ ) 1 1
–n
(– ¥, ¥ )– { 0 } e0 +h –1 eh –1
3. x , n is a positive integer Þ lim f ( x ) = lim f (0 + h ) = lim 1
= lim 1
x ® 0+ h ®0 h ®0 h ®0
4. | x – a | (– ¥, ¥ )
e0 +h +1 eh + 1
5. P ( x ) = a 0xn + a1xn – 1 + ...+ an (– ¥ , ¥ ) 1
1–
6. p ( x ) , where p ( x ) and q ( x ) are e 1/h 1 –0
(– ¥, ¥ )– { x :q ( x ) = 0 } Þ lim f ( x ) = lim Þ lim f ( x ) = =1
q (x ) x ®0 + h ®0
1+
1 x ®0 + 1+0
polynomial in x e 1/h
7. sin x (– ¥, ¥ ) 1
8. cos x (– ¥, ¥ ) [as h ® 0 ; ® ¥ Þ e 1/h ® ¥; 1/e 1/h ® 0]
h
9. tan x ì p ü
(– ¥, ¥ )– í(2n + 1 ) : n Î I ý \ lim f ( x ) = 1
î 2 þ x ® 0+
10. cot x (– ¥, ¥ )– { np : n Î I }
11. sec x (– ¥, ¥ )– {(2n + 1 ) p/ 2 : n ÎI } Again, LHL at x = 0, let x = 0 – h
12. cosec x (– ¥, ¥ )– { np : n Î I } Þ lim f ( x ) = lim f (0 – h )
13. ex (– ¥, ¥ ) x ® 0– h ®0

14. loge x (0, ¥) e –1/h – 1 0–1


= lim = = –1
ì x – 1, x < 0 h ® 0 e –1/h +1 0+1
ï
y Example 11 Examine the function, f (x) = í 1/ 4 , x = 0 [as h ® 0; e –1/h ® 0]
ï 2
î x – 1, x > 0 lim f ( x ) = – 1
x ® 0–
Discuss the continuity and if discontinuous remove
the discontinuity. Þ lim f ( x ) ¹ lim f ( x ) .
x ® 0+ x ® 0–
Sol. lim f ( x ) = lim + f ( x ) = – 1 Thus, f ( x ) has non-removable discontinuity.
x ® 0– x ®0

But f (0) = 1/ 4. Thus, f ( x ) has removable discontinuity 1


and f ( x ) could be made continuous by taking y Example 13 Show f ( x ) = has discontinuity of
|x|
ì x – 1, x < 0
ï
f (0) = – 1 Þ f ( x ) = í–1, x =0
second kind at x = 0.
ï x 2 – 1, x > 0 Sol. Here, f (0) =
1
î , which shows function has discontinuity
|0|
Graphically f ( x ) could be plotted as showing of second kind.
Y Graphically Here, the graph is broken at x = 0 as
x ®0
f (0) = 1/4 Þ f (x ) ® ¥
f (x) = x2 – 1,
1/4 when x > 0 Y
X′ X
0 1
1 1
1, –1 f (x) = f (x) =
x– |x| |x|
) = <0
f (x n x
e
wh Y′ X′
O X
ì e1 /x – 1
y Example 12 Show the function, f (x) = ïí e1 /x + 1 , when x ¹ 0
ï0 , when x = 0 Y′
î
has non-removable discontinuity at x = 0. Therefore, f ( x ) has discontinuity of second kind.
Chap 06 Continuity and Differentiability 321

Exercise for Session 3


1. Which of the following function(s) has/have removable discontinuity at the origin?
(b) f (x ) = cos æç
1 |sin x |ö
(a) f (x ) = ÷
1+ 2 cot x è x ø
p 1
(c) f (x ) = x sin (d) f (x ) =
x ln |x |

2. Function whose jump (non-negative difference of LHL and RHL) of discontinuity is greater than or equal to one.
is/are
ì (e1/ x + 1) ì (x1/ 3 - 1)
ïï 1/ x ; x<0 ï 1/ 2 ; x>0
- 1) ï - 1)
(a) f (x ) = í (e (b) g (x ) = í (x
ï (1 - cos x ) ; x > 0 ï ln x ; 1
<x<1
ïî ïî (x - 1) 2
x
ì sin-1 2x
; x Î æç 0, ùú
1
ïï ì log3 (x + 2); x>2
-1
(c) u (x ) = í tan 3x è 2û (d) v (x ) = í
îlog1/ 2 (x + 5); x < 2
2
ï |sin x | x<0
ïî ;
x
ì - x , if x < 0
ï
3. Consider the piecewise defined function f ( x ) = í 0, if 0 £ x £ 4, choose the answer which best describes
ï x - 4, if x >4
î
the continuity of this function.
(a) the function is unbounded and therefore cannot be continuous.
(b) the function is right continuous at x = 0.
(c) the function has a removable discontinuity at 0 and 4, but is continuous on the rest of the real line.
(d) the function is continuous on the entire real line.

4. If f ( x ) = sgn (cos 2x - 2 sin x + 3), where sgn () is the signum function, then f ( x )
(a) is continuous over its domain.
(b) has a missing point discontinuity.
(c) has isolated point discontinuity.
(d) has irremovable discontinuity
2 cos x - sin 2x e - cos x - 1
5. f (x ) = ; g( x ) =
( p - 2x ) 2
8x - 4p
h( x ) = f ( x ) for x < p /2 = g( x ) for x > p /2, then which of the followings does not hold?
(a) h is continuous at x = p/ 2
(b) h has an irremovable discontinuity at x = p/ 2
(c) h has a removable discontinuity at x = p/ 2
æ p+ ö æ p- ö
(d) f ç ÷ = g ç ÷
è 2ø è 2ø
Session 4
Theorems Based on Continuity;
Continuity of Composite Function
Theorem 1 Sum, difference, product and quotient of two 1/x
ì æp öü
continuous functions is always a continuous function. Sol. Here, lim f ( x ) = lim í tan ç + x ÷ý
x ®0 x ®0î è4 øþ
f (x )
However, h ( x ) = is continuous at x = a only if 1/x
g( x ) é 1 + tan x ù
Þ lim f ( x ) = lim ê ú [1¥ form]
®
g(a ) ¹ 0. x ®0 x 0 ë 1 – tan x û
1/x
Theorem 2 If f ( x ) is continuous and g( x ) is é æ 1 + tan x ö ù
discontinuous at x = a, then the product function Þ lim f ( x ) = lim ê1 + ç –1 ÷ ú
x ®0 x ®0 ë è 1 – tan x ø û
f ( x ) = f ( x )× g( x ) is not necessarily be discontinuous at
æ 2 tan x ö 1
x = a. lim ç
x®0
÷×
è 1 – tan x ø x
Þ lim f ( x ) = e
ì 1 x ®0
ïsin , x ¹ 0
e.g. (i) f ( x ) = x and g( x ) = í x tan x
2 lim
îï 0, x =0 x®0
Þ lim f ( x ) = e x (1 – tan x )
=e 2
x ®0
where f ( x ) is continuous and g( x ) is discontinuous at
Here, f ( x ) is continuous at x = 0, when
x = 0.
lim f ( x ) = f (0)
ì 1 x ®0
ïx × sin , x ¹ 0 Þ k = e2
But f ( x ) = f ( x ) × g( x ) = í x
ïî 0, x =0
y Example 15 A function f ( x ) is defined by,
is continuous at x = 0.
ì[x 2 ] – 1
p ï , for x ¹ 1
2
(ii) f ( x ) = cos (2 x - 1) is continuous at x = 1 and f (x ) = í x 2 – 1
2
ï 0, for x = 1
2
g( x ) = [ x ] is discontinuous at x = 1. î
æ 2x - 1ö Discuss the continuity of f ( x ) at x = 1.
But f ( x ) = f ( x ) × g( x ) = [ x ]cos ç ÷ p is
è 2 ø ì [x 2 ] – 1
continuous at x = 1. ï , for x 2 ¹ 1
Sol. We have, f (x ) = í x 2 – 1
Theorem 3 If f ( x ) and g( x ) both are discontinuous at ï 0, for x = 1
2
î
x = a, then the product function f ( x ) = f ( x ) × g( x ) is not
ì –1
ï x 2 – 1 , for 0 < x < 1
2
necessarily be discontinuous at x = a.
ì 1, x ³ 0 ì- 1, x ³ 0 ï
ï
e.g. f ( x ) = í and g( x ) = í Þ f (x ) = í 0, for x 2 = 1
î- 1, x < 0 î 1, x < 0 ï 1–1
ï 2 , for 1 < x 2 < 2
\ f ( x ) = f ( x ) × g( x ) = - 1, " x Î R ïî x – 1
f ( x ) is continuous, where f ( x ) and g( x ) are discontinuous
at x = 0. ì –1
ï x 2 – 1 , for 0 < x < 1
2

ìæ 1 /x ï
p ö ï
ï ç tan æç + x ö÷ ÷ , x ¹ 0 Þ f (x ) = í 0, for x 2 = 1
y Example 14 f ( x ) = í è è4 øø . ï 0, for 1 < x 2 < 2
ïk , ï
î x =0 ïî
For what value of k, f ( x ) is continuous at x = 0? \ RHL at x = 1
Chap 06 Continuity and Differentiability 323

Þ lim f ( x ) = 0 Check continuity at x = (2n + 1 )


x ® 1+
LHL = lim – f ( x ) = 0 and f (2n + 1) = 1
Also, LHL at x 2 = 1 x ® ( 2n + 1)
Thus, LHL ¹ f (2n + 1)
Þ lim f ( x ) = lim f (1 – h ) Þ f ( x ) is discontinuous at x = (2n + 1)
x ® 1– h ®0
–1 [i.e. at odd integers]
= lim = –¥
h ® 0 (1 – h )2 –1 Hence, f ( x ) is discontinuous at x = (2n + 1) ; n Î integer.
\ lim f ( x ) doesn’t exist. [as RHL ¹ LHL ]
x ®1 y Example 18 Let
Hence, f ( x ) is not continuous at x = 1. ì{ 1 + |sin x |}a /|sin x | , – p/ 6 < x < 0
ï
y Example 16 Discuss the continuity of the function f ( x ) = íb , x =0
log (2 + x ) – x 2n sin x ï tan 2x /tan 3x
f ( x ) = lim at x = 1. îe , 0 < x < p/ 6
n ®¥ 1 + x 2n Determine a and b such that f ( x ) is continuous at
log 3 – sin 1 1 x = 0.
Sol. We have, f (1) = lim = (log 3 – sin 1) …(i) [IIT JEE 1994]
n ®¥ 2 2
Sol. Since, f is continuous at x = 0.
ïì0, if x < 1
2
We know that, lim x 2n = í Therefore, RHL = LHL = f (0)
n ®¥ ïî ¥, if x 2 > 1 RHL at x = 0
\ For x 2 < 1, we have lim f ( x ) = lim f (0 + h ) = lim e tan 2h/ tan 3h
x ® 0+ h ®0 h ®0
log (2 + x ) – x 2n sin x tan 2h 3h 2
f ( x ) = lim = log (2 + x ) × ×
n ®¥ 1 + x 2n = lim e 2h tan 3h 3
= e 2/3 …(i)
h ®0
Again, for x > 1, we have
2
Again, LHL at x = 0
1
2n
log (2 + x ) – sin x lim – f ( x ) = lim f (0 – h )
x ®0 h ®0
f ( x ) = lim x = –sin( x )
n ®¥ 1 = lim {1 + |sin (0 – h )| }a /| sin (0 – h )|
1+ h ®0
x 2n a

Here, as x ® 1 = lim {1 + |sin h | }| sin h |


h ®0
lim – f ( x ) = log (3) lim | sin h |×
a
x ®1 h®0
=e | sin h |
=ea …(ii)
and lim + f ( x ) = –sin (1)
x ®1 and f ( 0) = b …(iii)
So, lim f ( x ) ¹ lim f ( x ) Thus, e 2/3
= e = b Þ a = 2/ 3 and b = e
a 2/3
x ® 1– x ® 1+

Therefore, f ( x ) is not continuous at x = 1. y Example 19 Fill in the blanks so that the resulting
statement is correct.
y Example 17 Discuss the continuity of f ( x ) , where
æ p ö
æ px ö
2n Let f ( x ) = [ x ] sin ç ÷, where [ × ] denotes greatest
f ( x ) = lim ç sin ÷ . è [ x + 1] ø
n ®¥ è 2 ø
integral function. The domain of f is ............. and the
ì0, | x | < 1 points of discontinuity of f in the domain are
Sol. Since, lim x 2n = í
n ®¥ î1, | x | = 1 ................ . [IIT JEE 1996]
ì px æ p ö
æ æ px ö ö ï0, sin 2 < 1
2n Sol. Let f ( x ) = [ x ] sin ç ÷
è [ x + 1] ø
ï
\ f ( x ) = lim çsin ç ÷ ÷ =í
n ®¥ è è 2 øø
ï1, sin px = 1 Domain of f ( x ) is x Î R excluding the points where
ïî 2 [ x + 1] = 0 [Q denominator can’t be zero]
Thus, f ( x ) is continuous for all x, except for those values Þ 0 £ x + 1 < 1 Þ –1 £ x < 0
px i.e. for all x Î[–1, 0) , denominator is zero.
of x for which sin = 1, i.e. x is an odd integer. So, domain is x Î R – [–1, 0).
2
Þ Domain is x Î (–¥, – 1) È [0, ¥ )
Þ x = (2n + 1), where n Î I and the internal point discontinuity Î[–1, 0).
324 Textbook of Differential Calculus

y Example 20 Let f ( x + y ) = f ( x ) + f ( y ) for all x and y. y Example 22 Discuss the continuity for
If the function f ( x ) is continuous at x = 0, show that 1– u2
f (x ) = , where u = tan x .
f ( x ) is continuous for all x. 2 + u2
Sol. As, the function is continuous at x = 0, we have p
Sol. Here, u = tan x is discontinuous at np ± ,n ÎI
lim – f ( x ) = lim + f ( x ) = f (0) 2
x ®0 x ®0
1 – u2
and f (x ) = is continuous at every u Î R.
Þ lim f (0–h ) = lim f (0 + h ) = f (0) 2 + u2
h ®0 h ®0
Þ lim { f (0) + f (–h )} = lim { f (0) + f (h )} = f (0) ì p ü
h ®0 h ®0 Hence, f ( x ) is continuous on; x Î R – ínp ± , n Î I ý.
î 2 þ
[using f ( x + y ) = f ( x ) + f (y )] 2
1–u
Also, lim f ( x ) = 2lim
Þ lim f (–h ) = lim f (h ) = 0 …(i) x ® np ±
p u ® ¥ 2 + u2
h ®0 h ®0 2
Now, consider some arbitrary point x = a 1
2
–1
LHL = lim f (a – h ) = lim f (a ) + f (–h ) = lim u = –1
h ®0 h ®0 u ®¥ 2
+1
[using f ( x + y ) = f ( x ) + f (y ) ] u2
p
= f (a ) + lim f (–h ) Hence, the points np ± , n Î I have removable
h ®0 2
LHL = f (a ) + 0 = f (a ) discontinuity.
[as lim f (–h ) = 0, using Eq. (i)] i.e. If f ( x ) is defined as
h ®0
ì1 – u2 p
RHL = lim f (a + h ) = lim f (a ) + f (h ) ïï , x ¹ np ± and u = tan x
h ®0 h ®0
f (x ) = í 2 + u 2
2
, then
= f (a ) + lim f (h ) ï – 1, x = np ± p
h ®0
ïî 2
RHL = f (a ) + 0 = f (a )
f ( x ) is continuous for all x Î R.
[as lim f (h ) = 0, using Eq. (i)]
h ®0
At any arbitrary point ( x = a ), y Example 23 Find the points of discontinuity of
LHL = RHL = f (a ) 1 1
y= 2 , where u = .
Therefore, function is continuous for all values of x, if it is u +u–2 x –1
continuous at 0. 1
Sol. The function u = f ( x ) = is discontinuous at the point
y Example 21 Let f ( x ) be a continuous function x –1
x = 1. …(i)
defined for 1 £ x £ 3. If f ( x ) takes rational values for 1 1
The function y = g ( x ) = 2 = is
all x and f (2) = 10, then find the value of f (1 . 5). u + u – 2 ( u + 2) ( u – 1)
[IIT JEE 1997] discontinuous at u = –2 and u = 1.
Sol. As, f ( x ) is continuous in [1, 3], f ( x ) will attain all values 1
when u = – 2, = u = –2
between f (1) and f (3). As, f ( x ) takes rational values for x –1
all x and there are innumerable irrational values between 1
Þ x –1 = –
f (1) and f (3) which implies that f ( x ) can take rational 2
values for all x, if f ( x ) has a constant rational value at all Þ x = 1/ 2 …(ii)
points between x = 1 and x = 3. 1
when u = 1, =u =1
So, f (2) = f (1 . 5) = 10 x –1
Continuity of composite function Þ x –1 = 1
If the function u = f ( x ) is continuous at the point x = a Þ x =2 …(iii)
and the function y = g (u ) is continuous at the point Hence, the composite function y = g ( f ( x )) is
u = f (a ), then the composite function 1
discontinuous at three points x = , 1 and 2.
y = ( gof ) ( x ) = g ( f ( x )) is continuous at the point x = a. 2
Chap 06 Continuity and Differentiability 325

Exercise for Session 4


1 æ 2 ö
1. If f ( x ) = , then f ç ÷ is discontinuous at x =
x 2 - 17x + 66 è x - 2ø
7 24
(a) 2 (b) (c) (d) 6, 11
3 11

2. Let f be a continuous function on R.


n2
If f (1/4n ) = (sin e n ) e - n + 2
2
, then f (0) is
n +1
(a) not unique (b) 1
(c) data sufficient to find f (0) (d) data insufficient to find f (0)

3. f ( x ) is continuous at x = 0, then which of the following are always true?


(a) lim f (x ) = 0
x ® 0
(b) f (x ) is non continuous at x = 1
(c) g (x ) = x 2f (x ) is continuous at x = 0
(d) lim + (f (x ) - f (0)) = 0
x ® 0
épù æp ö
4. If f ( x ) = cos ê ú cos ç ( x - 1)÷ ; where [x] is the greatest integer function of x, then f ( x ) is continuous at
ëxû è2 ø
(a) x = 0 (b) x = 1
(c) x = 2 (d) None of these
ì 0 , x ÎZ
5. Let f ( x ) = [ x ] and g( x ) = í 2 , then (where [.] denotes greatest integer function)
î x , x ÎR - Z
(a) lim g (x ) exists, but g (x ) is not continuous at x = 1
x ®1
(b) lim f (x ) does not exist and f (x ) is not continuous at x = 1
x ®1
(c) gof is continuous for all x
(d) fog is continuous for all x
ì a sin2n x , for x ³ 0 and n ® ¥
6. Let f ( x ) = í , then
î b cos 2m
x - 1, for x < 0 and m ® ¥
(a) f (0- ) ¹ f (0+ ) (b) f (0+ ) ¹ f (0)
(c) f (0- ) = f (0) (d) f is continuous at x = 0

x `- sin x
n n
7. Consider f ( x ) = lim for x > 0, x ¹ 1, f (1) = 0, then
n®¥ x n + sin x n
(a) f is continuous at x = 1
(b) f has a finite discontinuity at x = 1
(c) f has an infinite or oscillatory discontinuity at x = 1
(d) f has a removal type of discontinuity at x = 1
Session 5
Intermediate Value Theorem
If f ( x ) is continuous in [a, b ] and f (a ) ¹ f (b ), then for any Therefore, using intermediate value theorem, there exists
value c Î( f (a ), f (b )), there is atleast one number x 0 in some c Î[a, b ], such that
(a, b ) for which f ( x 0 ) = c . a+b
f (c ) =
Y Y 2

f (b ) f (b) y Example 25 Suppose that f ( x ) is continuous in [0,


c = f (x0) c
1] and f (0) = 0, f (1) = 0 . Prove that f (c ) = 1 - 2c 2 for
f (a) f (a)
some c Î(0, 1).
X X Sol. Let g ( x ) = f ( x ) + 2x 2 - 1 in [0, 1], g (0) = – 1, g (1) = 1
O x=a x0 x = b O x=a x0 x=b
\ By intermediate value theorem, there exists some
Y c Î(0, 1); g (c ) = 0 Þ f (c ) = 1 - 2c 2
f (b)
Continuity for Rational and Irrational Function
Functions should be continuous only at one point and to
C
be defined everywhere. [Single Point Continuity]
e.g.
x=a
X ìx , if x Î Q
x0 O x0 x0 x = b
(i) f ( x ) = í , is continuous only at x = 0.
î 0, if x Ï Q
f (a )
ì x , if x Î Q
Figure 6.10 (ii) f ( x ) = í , is continuous only at x = 0
î- x , if x Ï Q
y Example 24 Show that the function and defined everywhere.
a+b ì x , if x Î Q 1
f ( x ) = ( x - a ) 2 ( x - b ) 2 + x takes the value for (iii) f ( x ) = í , is continuous only at x =
2 î1 - x , if x Ï Q 2
some value of x Î[a, b ]. and defined everywhere.
Sol. Here, f (a ) = a and f (b ) = b
ìx 2 , if x Î Q
a+b (iv) f ( x ) = í , is continuous only at x = 1 or
Also, f ( x ) is continuous in [a, b ] and Î [a, b ]. î 1, if x Ï Q
2
x = - 1 and defined everywhere.

Exercise for Session 5


x x x
1. Examine the continuity at x = 0 of the sum function of the infinite series + + + ¼¥
x + 1 ( x + 1) (2x + 1) (2x + 1) (3x + 1)

2. If g : [a, b ] onto [a, b ] is continuous, then show that there is some c Î[a, b ] such that g(c ) = c.

3. Show that (a) a polynomial of an odd degree has atleast one real root.
(b) a polynomial of an even degree has atleast two real roots, if it attains atleast one value opposite in sign
to the coefficient of its highest-degree term.
x2 x2 x2
4. Let y n ( x ) = x 2 + + + ¼+ and y ( x ) = lim y n ( x ) . Discuss the continuity of
1+ x 2
(1 + x )
2 2
(1 + x 2 )n - 1 n®¥

y n( x ) (n = 1, 2, 3, ¼ n ) and y ( x ) at x = 0.
Session 6
Differentiability : Meaning of Derivative
The instantaneous rate of change of function with respect (a) Right hand derivative The right hand derivative of
to the independent variable is called derivative. Let f ( x ) f ( x ) at x = a denoted by f (a + ) is defined as
be a given function of one variable and Dx denotes a f (a + h ) - f (a )
number (positive or negative) to be added to the f ¢ (a + ) = lim , provided the limit exists
h ®0 h
number x. and is finite (h >0 ).
Let Df denotes the corresponding change of f , then (b) Left hand derivative The left hand derivative of
Df = f ( x + Dx ) - f ( x ).
f ( x ) at x = a denoted byf (a - ) is defined as
Df f ( x + Dx ) - f ( x )
= f (a - h ) - f (a )
Dx Dx f ¢ (a - ) = lim , provided the limit exists
h ®0 -h
Df
If approaches a limit as Dx approaches to zero, this and is finite (h >0 ).
Dx
limit is the derivative of f at the point x. The derivative of Hence, f ( x ) is said to be derivable or differentiable at
df x = a.
a function f is denoted by symbols such as f ¢ ( x ), ,
dx If f ¢ (a + ) = f ¢ (a - ) = finite quantity and it is denoted by
d d f (x ) f ¢ (a ) ; where f ¢ (a ) = f ¢ (a - ) = f ¢ (a + ) and it is called
f ( x ) or .
dx dx derivative or differential coefficient of f ( x ) at x = a.
df Df f ( x + Dx ) - f ( x )
Þ = lim = lim
dx Dx ®0 Dx Dx ®0 Dx
The derivative evaluated at a point a is written, Relation between Continuity
æ d f (x ) ö
f ¢ (a ), ç ÷ , ( f ¢ ( x )) x =a etc.
and Differentiability
è dx ø x =a If a function is differentiable at a point, then it should be
continuous at that point as well and a discontinuous
function cannot be differentiable. This fact is proved in the
Existence of Derivative at x = a following theorem.
The derivative of a function f ( x ) exists at x = a, if Theorem If a function is differentiable at a point, it is
f ( x ) - f (a ) necessarily continuous at that point. But the converse is
lim exists finitely.
x ®a (x - a ) not necessarily true.
Or
f ( x ) - f (a ) f ( x ) - f (a )
or lim– = lim+ f ( x ) is differentiable at x = c Þ f ( x ) is continuous at
x ®a (x - a ) x ®a (x - a ) x = c.
f (a - h ) - f (a ) f (a + h ) - f (a ) Proof Let a function f ( x ) be differentiable at x = c .
or lim = lim
h ®0 -h h ®0 h f ( x ) – f (c )
Then, lim exists finitely.
Y y = f(x) x ®c x –c
f (a+h)
f ( x ) – f (c )
f (a–h)
Let lim = f ¢ (c ) …(i)
x ®c x –c
In order to prove that f ( x ) is continuous at x = c , it is
f (a)
sufficient to show that lim f ( x ) = f (c ).
x ®c
é æ f ( x ) – f (c ) ö ù
O a–h a a +h
X Now, lim f ( x ) = lim ê ç ÷ ( x – c ) + f (c )ú
x ®c x ®c è x –c ø
(h→0)
ë û
(h→0)
Figure 6.11
328 Textbook of Differential Calculus

éì f ( x ) – f (c ) ü ù = lim
f ( 0 – h ) – f ( 0)
= lim êí ý × ( x – c )ú + f (c ) h ®0 ( 0 – h ) – ( 0)
x ®c î x –c þ
ë û
–h sin(–1/h )
f ( x ) – f (c ) = lim
= lim × lim ( x – c ) + f (c ) h ®0 (– h )
x ®c x –c x ®c
æ1ö
= – lim sin ç ÷
= f ¢ (c ) ´ 0 + f (c ) h ®0 èh ø
= f (c ) [a number which oscillates between –1 and +1]
\ (LHD at x = 0) doesn’t exist.
Hence, f ( x ) is continuous at x = c . Similarly, it could be shown that RHD at x = 0 doesn’t exist.
Converse The converse of above theorem is not Hence, f ( x ) is continuous but not differentiable.
necessarily true, i.e. a function may be continuous at a
point but may not be differentiable at that point. ì é æ 1 1 öù
e.g. The function f ( x ) = | x | is continuous at x = 0 but it is ï x exp ê– ç + ÷ ú , x ¹ 0
y Example 27 Let f ( x ) = í ë è | x | x øû
not differentiable at x = 0, as shown by figure. ï0,
î x =0
Y f (x) =|x|
Test whether (a) f ( x ) is continuous at x = 0
(b) f ( x ) is differentiable at x = 0.
[IIT JEE 1997]
ì é æ 1 1 öù
X′ X ï x exp ê– ç + ÷ú,x ¹ 0
O Sol. Here, f ( x ) = í ë è|x | x øû
ï0, x =0
î
ì – ìí 1 + 1 üý
Y′ ï xe îï x x þ , x > 0
Figure 6.12 ï ì 1 1ü
ïï – í + ý é ì x , x ³ 0ù
Which shows we have sharp edge at x = 0, hence not Þ f ( x ) = í xe îï – x x þ
,x <0 êQ| x | = í–x , x < 0ú
differentiable but continuous at x = 0. ï ë î û
0, x =0
ï
ì 1 ï
ï x sin , when x ¹ 0
y Example 26 Show that f ( x ) = í x ïî
ïî0, when x = 0 ì xe – 2 / x , x > 0
ï
is continuous but not differentiable at x = 0. Þ f (x ) = í x , x <0 …(i)
ï 0, x =0
Sol. (a) To check continuity at x = 0 î
Here, f ( 0) = 0 (a) To check continuity of f ( x ) at x = 0
Also, lim f ( x ) = lim x sin
1 LHL = lim f ( x ) = lim – x
x ® 0– x ®0
x ®0 x ®0 x
= 0 ´ (A finite quantity that lies between –1 to +1) = lim (0 – h ) = 0
h ®0
1
=0 [as n ® 0, sin ® sin ¥, which is a finite RHL = lim f ( x ) = lim + xe –2 / x
x x ® 0+ x ®0

quantity between –1 to +1] h


= lim = 0, f (0) = 0
\ lim f ( x ) = f (0) and hence, f ( x ) is continuous. h ® 0 e 2 /h
x ®0
Now, \ f ( x ) is continuous at x = 0.
(b) To check differentiability at x = 0 (b) To check differentiability at x = 0
(LHD at x = 0) f ( 0 – h ) – f ( 0)
LHD = Lf ¢ (0) = lim ,h > 0
f ( x ) – f ( 0) h ®0 –h
= lim –
x ®0 x –0 (– h ) – 0
= lim =1
h ®0 –h
Chap 06 Continuity and Differentiability 329

f ( 0 + h ) – f ( 0) Similarly, at x = –1 it is clear that RHD = 0, while LHD = –2


RHD = Rf ¢ (0) = lim
h ®0 h Aliter In this method, first of all we differentiate the
–2 function and from the derivative equality sign should be
–2
he h –0 removed from doubtful points.
= lim = lim e h
h ®0 h ®0 x < –1
h ì – 2,
–¥ ï
=e =0 Here, f ¢ ( x ) = í 0, – 1 < x < 1 [no equality on –1 and +1]
ï 2, x >1
\ Lf ¢ (0) ¹ Rf ¢ (0) î
Therefore, f ( x ) is not differentiable at x = 0. Now, at x = 1, f ¢ (1+ ) = 2, while f ¢ (1– ) = 0 and
at x = –1, f ¢ (–1+ ) = 0, while f ¢ (–1– ) = – 2
Some Standard Results Thus, f ( x ) is not differentiable at x = ± 1.
on Differentiability Remark
This method is not applicable when function is discontinuous.
Functions f ( x ) Intervals in which
f ( x ) is differentiable
y Example 29 Discuss the continuity and
1. Polynomial (– ¥ to ¥ ) differentiability for f ( x ) = [sin x ] when x Î[0, 2p ] ;
2. Exponential (a x , a > 0) (– ¥ to ¥ ) where [ × ] denotes the greatest integer function x.
3. Constant (– ¥ to ¥ ) Sol. In last chapter, we have discussed the plotting of curves, so
4. Logarithmic Each point in its domain y = [sin x ] could be plotted as
5. Trigonometric Each point in its domain
6. Inverse trigonometric Each point in its domain Y

y Example 28 Let f ( x ) = | x – 1| + | x + 1|. Discuss the


continuity and differentiability of the function. 1
y = sin x
Sol. Here, f ( x ) = | x – 1| + | x + 1| X
0 π 2π
ì ( x – 1) + ( x + 1), when x >1
ï –1
Þ f ( x ) = í–( x – 1) + ( x + 1), when – 1 £ x £ 1
ï–( x – 1) – ( x + 1), when x < –1
î
ì 2x , when x >1 Or
ï
Þ f ( x ) = í 2, when – 1 £ x £ 1 Y
ï–2x , when x < –1
î
Graphically The graph of the function is shown below. 1
Y
–2x 2x π X
0 π


– 2π
2 2
–1
2

X which shows f ( x ) = [sin x ] is discontinuous


–1 0 1
p
for x = , p, 2p, when x Î[0,2p ] and f ( x ) is not
From the graph, it is clear that the function is continuous at 2
p
all real x, also differentiable at all real x except at x = ± 1. differentiable at x = , p, 2p.
Since, it has sharp edges at x = –1 and x = 1. 2
At x = 1 we see that the slope from the right, i.e. RHD = 2, As we know, function is neither differentiable nor
while slope from the left, i.e. LHD = 0. continuous at those points for which graph is broken.
330 Textbook of Differential Calculus

y Example 30 If f ( x ) = {| x| – | x – 1| } 2 , draw the graph of ì (3 – x ), x <0


ï 2
f ( x ) and discuss its continuity and differentiability of f ( x ). ï ( x – 3x + 2), 0 £ x < 1
| f ( x )| = í …(ii)
ï–( x – 3x + 2), 1 £ x < 2
2
ì–x + x – 1, x < 0 ï ( x 2 – 3x + 2), 2 £ x
ï î
Sol. We know, | x | – | x – 1| = í x + x – 1, 0 £ x < 1
ï x – ( x – 1), 1 £ x Now, from Eqs. (i) and (ii), g ( x ) = f (| x | ) + | f ( x )|
î
ì x 2 + 2x + 5, x <0
ì – 1, x < 0 ï 2
ï ï2x – 6x + 4, 0 £ x < 1
Þ | x | – | x – 1| = í2x – 1, 0 £ x < 1 g(x ) = í
ï 1, 1 £ x ï0, 1£ x < 2
î ï2x 2 – 6x + 4, x ³ 2
î
\ f ( x ) = {| x | – | x – 1| } 2
ì2x + 2, x < 0
ì 1, when x < 0 or x ³ 1 ï 4 x –6, 0 < x < 1
=í ï
and g ¢( x ) =í
î(2x – 1) , when 0 £ x < 1
2
ï 0, 1 < x <2
Graphically f ( x ) could be shown as ïî 4 x –6, x > 2
Therefore, g ( x ) is continuous in R – { 0 } and g ( x ) is
Y
differentiable in R – {0, 1, 2 }.

y=1 y=1 y Example 32 Let f ( x ) = [n + p sin x ], x Î (0, p ), n Î Z


1
y = (2x–1)2 and p is a prime number, where [ × ] denotes the
X
greatest integer function. Then, find the number of
0 1 1 points, where f ( x ) is not differentiable.
2
Sol. Here, f ( x ) = [n + p sin x ] is not differentiable at those
points where n + p sin x is an integer.
From above figure, it is clear that f ( x ) is continuous for As, p is a prime number.
x Î R; but f ( x ) is not differentiable at x = 0, 1.
Þ n + p sin x is an integer if sin x = 1, – 1, r/p
Þ f ( x ) is continuous for all x Î R.
p p r r
Þ f ( x ) is differentiable for all x Î R – {0, 1}. i.e. x = , – , sin –1 , p – sin –1
2 2 p p
ì x – 3, x <0
y Example 31 If f ( x ) = í 2 where 0 £ r £ p –1
î x – 3x + 2 , x ³ 0 But
p
x ¹ – ,0
and let g ( x ) = f (| x | ) + | f ( x )| . Discuss the 2
differentiability of g ( x ) . \ Function is not differentiable at
p r r
x = , sin –1 , p – sin –1
ì| x | – 3, |x | < 0 2 p p
Sol. f (| x | ) = í 2
î| x | – 3| x | + 2, | x | ³ 0 where 0<r £ p -1
Where, | x | < 0 is not possible, thus neglecting, we get So, the required number of points are
f (| x | ) = {| x | 2 – 3| x | + 2, | x | ³ 0 = 1 + 2 ( p – 1) = 2p – 1.
ìï x 2 + 3x + 2, x <0 y Example 33 If f ( x ) = || x | – 1| , then draw the graph of
f (| x | ) = í …(i)
ïî x 2 – 3x + 2, x ³0 f ( x ) and fof ( x ) and also discuss their continuity and
ì| x – 3|, x <0 3
Again, | f ( x )| = í 2 differentiability. Also, find derivative of ( fof ) 2 at x = ×
î| x – 3x + 2|, x ³0 2
Chap 06 Continuity and Differentiability 331

Sol. The graph of f ( x ) is shown as y Example 34 Draw the graph of the function
Y
g ( x ) = f ( x + l ) + f ( x – l ) , where
ì ì | x| ü
1 ïk í1 – ý, for | x | £ l
f (x ) = í î l þ
ï 0, for | x | > l
Y′ X î
–1 0 1 Also, discuss the continuity and differentiability of the
function g ( x ).
Y′
ì æ |x |ö
It is clear from the graph that, f ( x ) is continuous for all x, ïk ç1 – ÷, | x | £ l
but f ( x ) is not differentiable at x = {– 1, 0,1 }. Sol. We have, f ( x ) = í è l ø
ï 0, |x| > l
Now, fof ( x ) = || f ( x )| – 1| = | f ( x ) – 1| [as f ( x ) ³ 0 for all x] î
Now, if f ( x ) ® f ( x ) – 1, shift the graph one unit below the The graph of f ( x ) is shown as
X-axis, i.e., as shown;
Y
Y

X′ X
–2 2 X′ X
0 –l 0 l
–1 Graph for f(x)
Graph for f(x) – 1 Y′
Y′
Thus, for graph of fof ( x ) = | f ( x ) – 1| is taking image of the Now, to plot g ( x ) = f ( x + l ) + f ( x – l ), we shall first plot
graph of f ( x ) – 1 below X-axis and leaving the portion the graphs of f ( x + l ) and f ( x – l ) which are given as
above Y-axis as it is. Y Y
\ Graph for fof ( x ) is shown as
Y k
k

X′ X′
y= X X
2 x 2– x –2 0 l 2l –2l –l 0
x+
y=

y=

y= x y= Graph for f (x – l) Graph for f (x + l)


y=
–x

–x

Y′ Y′
–2

X′ X
–2 –1 0 1 2
Thus, the graph of g ( x ) = f ( x + l ) + f ( x – l ) is obtained by
adding graph of f ( x + l ) and f ( x – l ) as
Y′ Y

which is clearly continuous for all x Î R, but not


differentiable at x = { - 2, – 1, 0, 1, 2 } . k
Also, fof ( x ) = 2 – x , 1 £ x £ 2
\ ( fof )2 = (2 – x )2 , when 1 £ x £ 2 X′ X
–2l –l 0 l 2l
d
Þ ( fof )2 = 2 (2 – x )(–1), when 1 £ x £ 2 Graph for g(x) = f (x + l) + f (x – l)
dx Y′
d
\ ( fof )2 (when x = 3/ 2) = – 2 (2 – 3/ 2) = – 1 From the above figure, g ( x ) is continuous for all x Î R
dx
d and g ( x ) is differentiable for all x Î R – { ± 2l , ± l , 0 } .
Þ {( fof )2 } at x = 3 / 2 = – 1
dx
332 Textbook of Differential Calculus

ìï x{ 5 + | 1 – t |} dt, if x > 2
y Example 35 Let f ( x ) = í ò 0
y Example 36 Draw the graph of the function and
. discuss the continuity and differentiability at x = 1 for,
ïî 5x + 1, if x £ 2 ì 3 x , when – 1 £ x £ 1
Test f ( x ) for continuity and differentiability for all real x. f (x ) = í
î4 – x , when 1 < x < 4
Sol. Here, f ( x ) for x > 2.
ì 3x , when – 1 £ x £ 1
x
f (x ) = í
f (x ) = ò 0 {5 + | 1 – t | }dt Sol. Here,
î 4 – x , when 1 < x < 4
1 x
= ò0( 5 + 1 – t )dt + ò ( 5 + t – 1 )dt [since x > 2]
1
is shown below graphically.
1 x Y
æ t2 ö æ t2 ö
= ç6t – ÷ + ç 4t + ÷ .
è 2 ø0 è 2 ø1 3

1 x2 1
= 6– + 4x + –4–
2 2 2 1
2
x
= 1 + 4x + X
2 –1 0 1 4
ì1 + 4 x + x 2 / 2 , if x > 2
\ f (x ) = í
î 5x + 1, if x £ 2 From the graph, it is clear that it is continuous for all x in
\ RHL (at x = 2) [–1, 4 ) and not differentiable at x = 1.
ì (2 + h )2 ü Because at x = 1, LHD > 0, while RHD < 0
= lim í1 + 4(2 + h ) + ý
h ®0 2 þ Mathematically,
î
f ( 1 + h ) – f ( 1)
= 1 + 8 + 2 = 11 RHD = lim
h ®0 h
and LHL (at x = 2) = lim { 5(2 – h ) + 1}
h ®0 4 –(1 + h ) – 3
= 10 + 1 = 11 = lim = –1
h ®0 h
\ f ( x ) is continuous at x = 2. [as f (2) = 11 ] f ( 1 – h ) – f ( 1)
LHD = lim
Also, RHD (at x =2) h ®0 –h
f ( 2 + h ) – f ( 2)
Rf ¢ (2) = lim 3(1 – h ) – 3 æ 3–h – 1 ö
h ®0 h = lim = lim 3 ç ÷
h ®0 –h h ® 0 è –h ø
(2 + h )2
1 + 4( 2 + h ) + – 11 =3 loge 3
= lim 2
h ®0 h Since, LHD ¹ RHD, f ( x ) is not differentiable at x = 1.
h2 But f ( x ) is continuous at x = 1, because the derivatives
11 + 6h + – 11
= lim 2 =6 from both the sides are finite and definite.
h ®0 h Aliter
LHD (at x = 2) The given function is continuous.
f ( 2 – h ) – f ( 2)
Lf ¢ (2) = lim ì3x log 3, – 1 £ x < 1
h ®0 –h Hence, f ¢( x ) = í
5( 2 – h ) + 1 – 11 î–1, 1< x < 4
= lim
h ®0 -h Here, f ¢ ( 1+ ) = – 1
11 – 5h – 11
= lim =5 and f ¢ (1– ) = lim 3x log 3 = 3 log 3
h ®0 -h x ® 1–

\ f ( x ) is not differentiable at x = 2. which shows, f ¢ ( 1+ ) ¹ f ¢ ( 1– )


Thus, f ( x ) is continuous for all x Î R and differentiable for Therefore, f ( x ) is not differentiable at x = 1.
all x Î R – { 2 }.
Chap 06 Continuity and Differentiability 333

y Example 37 Match the column I with column II. Sol. The function f ( x ) = ( x 2 – 1) | x 2 – 3x + 2| + cos (| x | ) …(i)
Column I Column II Here, | x | is not differentiable at x = 0, but
(i) sin (p [ x ]) (A) differentiable everywhere ì cos (–x ), x < 0
cos (| x | ) = í
(ii) sin {(x –[ x ]) p} (B) nowhere differentiable î cos ( x ), x ³ 0
ì cos ( x ), x ³ 0
(C) not differentiable at –1 and +1 Þ cos (| x | ) = í
î cos ( x ), x < 0
where [ ] denotes greatest integral function. [IIT JEE 1992]
\ cos (| x | ) is differentiable at x = 0 …(ii)
Sol. (i) We know, [ x ] Î I , " x Î R
Again, | x – 3x + 2| = |( x – 1) ( x – 2)|
2
\ sin ( p [ x ]) = sin ( Ip ) = 0, " x Î R
ì ( x – 1) ( x – 2), if x < 1
By theory, we know that every constant function is ï
differentiable in its domain. = í–( x – 1) ( x – 2), if 1 £ x < 2 …(iii)
ï ( x – 1) ( x – 2), if x ³ 2
Thus, sin ( p [ x ]) is differentiable everywhere. î
Hence, (i) « (A) ì ( x 2 – 1) ( x – 1)( x –2) + cos x , if – ¥ < x < 1
(ii) Again, f ( x ) = sin {( x –[ x ]) p } ïï
So, f ( x ) = í–( x 2 – 1) ( x – 1)( x –2) + cos x , if 1 £ x < 2
Here, we know x – [ x ] = {x }, ï 2
( x – 1) ( x – 1)( x –2) + cos x , if 2 £ x < ¥
then p ( x – [ x ]) = p { x } îï
which is not differentiable at integral points. Now, to check differentiability at x = 1, 2
\ f ( x ) = sin { p ( x – [ x ])} is not differentiable at x Î I . [using shortcut method]
[integers] ì( x – 1)(2x – 3) + (2x ) ( x – 3x + 2) – sin x , – ¥ < x < 1
2 2

Hence, (ii) « (C) ïï


f ¢ ( x ) = í–( x 2 – 1)(2x – 3) – (2x ) ( x 2 – 3x + 2) – sin x , 1 £ x < 2
ï 2
ïî( x – 1)(2x – 3) + (2x ) ( x – 3x + 2) – sin x , 2 £ x < ¥
2
y Example 38 Fill in the blank, in the statement given
below.
Thus, for f ¢(1), we have
Let f ( x ) = x | x | . The set of points, where f ( x ) is twice
ì–sin 1, x < 1
differentiable is ............. . [IIT JEE 1992] f ¢ ( 1) = í
Sol. The function f ( x ) = x | x | can be written as, î–sin 1, x > 1
ì x ( x ), if x ³ 0 Thus, f ( x ) is differentiable at x = 1
f (x ) = í ì– 3 – sin 2, x < 2
î x (–x ), if x < 0 Also, f ¢ ( 2) = í
ìï x 2 , î 3 – sin 2, x > 2
if x ³ 0
Þ f (x ) = í Thus, f ( x ) is not differentiable at x = 2.
ïî– ( x ) , if x < 0
2
Hence, (d) is the correct answer.
f ( x ) is differentiable, " x Î R. n
ì 2x , if x > 0 y Example 40 If f ( x ) = S ar | x | r , where ai ’ s are
Again, f ¢( x ) = í r =1
î– 2x , if x < 0
real constants, then f ( x ) is
ì 2, if x > 0
f ¢ ¢( x ) = í (a) continuous at x = 0, for all ai
î– 2, if x < 0 (b) differentiable at x = 0, for all ai Î R
Here, f ² ( x ) is discontinuous at x = 0 (c) differentiable at x = 0, for all a 2k + 1 = 0
\ f ² ( x ) is not differentiable at x = 0. (d) None of the above
Hence, f ( x ) is twice differentiable, " x Î R – {0}. Sol. We know that, | x |r , r = 0, 1, 2, K are all continuous every-
where.
y Example 39 The function n
f ( x ) = ( x 2 – 1)| x 2 – 3x + 2| + cos (| x | ) is not \ f ( x ) = S ar | x |r is everywhere continuous.
r =1
differentiable at Since, | x |, | x | , | x | 5 , ... are not differentiable at x = 0,
3

(a) –1 (b) 0 (c) 1 (d) 2


whereas | x | 2 , | x | 4 , ... are everywhere differentiable.
[IIT JEE 1992]
334 Textbook of Differential Calculus

n
ì x, 1 £ x < e
\ f ( x ) = S ar | x |r is not differentiable at x = 0, if anyone ï
r =1 \ f ( x ) = í x /2, x = e
of a1, a 3 , a 5 , K is non-zero. ï 0, e < x £ 3
î
Thus, for f ( x ) to be differentiable at x = 0, we must have
a1 = a 3 = a 5 K = 0 Þf ( x ) is neither continuous nor differentiable at x = e = k .
i.e. a 2k + 1 = 0 \ [k 2 ] = 7
Hence, (a) and (c) are the correct answers. Hence, (c) is the correct answer.

y Example 43 If f ( x ) = | 1 - x |, then the points where


y Example 41 Let f and g be differentiable functions
satisfying g ¢ (a ) = 2, g (a ) = b and fog = I (Identity sin -1 ( f (| x | )) is non-differentiable, are
function ). Then, f ¢(b ) is equal to (a) {0, 1} (b) {0, - 1}
2 1 (c) {0, 1, - 1} (d) None of these
(a) 2 (b) (c) (d) None of these
3 2 Sol. Here, f ( x ) = |1 - x |
Sol. We have, fog = I ì x - 1, x >1
Þ f { g ( x )} = x , for all x Î R ï 1 - x, 0< x <1
ï
f (| x | ) = |1-| x || = í
\ f ¢ { g ( x )} × g ¢ ( x ) = 1, for all x Î R ï 1 + x, -1 £ x £ 0
1 1 ïî - x - 1, x < -1
Þ f ¢ ( g (a )) = Þ f ¢ (b ) =
g ¢ (a ) 2
Here, for domain of sin -1( f (| x | ))
Hence, (c) is the correct answer.
-1 £ f (| x | ) £ 1
x Þ | 1-| x || £ 1
y Example 42 If f ( x ) = , " x Î [1, 3]
1 + (log x )(log x ) ... ¥ Þ - 1 £ 1 -| x | £ 1
is non-differentiable at x = k. Then, the value of [k 2 ], is Þ - 2 £ -| x | £ 0
(where [ × ] denotes greatest integer function). Þ 2 ³ |x | ³ 0
(a) 5 (b) 6 (c) 7 (d) 8 \ x Î [ -2, 2]
Then, sin -1( f (| x | )), for x Î [ -2, 2] is not differentiable at
Sol. Let g ( x ) = (log x ) × (log x ) ... ¥
x = {0, 1, - 1}.
ì 1, x = e
ï Hence, (c) is the correct answer.
\ g ( x ) = í 0, 1 £ x < e
ï ¥, x > e
î
Chap 06 Continuity and Differentiability 335

Exercise for Session 6


ì -x , x <0
ï 2
1. If a function f ( x ) is defined as f ( x ) = í x , 0 £ x £ 1, then
ïx 2 - x + , x >
î 1 1
(a) f (x ) is differentiable at x = 0 and x = 1 (b) f (x ) is differentiable at x = 0 but not at x = 1
(c) f (x ) is not differentiable at x = 1but not at x = 0 (d) f (x ) is not differentiable at x = 0 and x = 1

2. If f ( x ) = x 3sgn (x ), then
(a) f is differentiable at x = 0 (b) f is continuous but not differentiable at x = 0
(c) f ¢ (0- ) = 1 (d) None of these

3. Which of the following is continuous everywhere in its domain but has atleast one point where it is not
differentiable?
x
(a) f (x ) = x1/ 3 (b) f (x ) =
x
(c) f (x ) = e - x (d) f (x ) = tan x

ì x + {x } + x sin {x } , for x ¹ 0
4. If f ( x ) = í , where {x } denotes the fractional part function, then
î 0, for x = 0
(a) f is continuous and differentiable at x = 0 (b) f is continuous but not differentiable at x = 0
(c) f is continuous and differentiable at x = 2 (d) None of these
ì æ e1/ x - e -1/ x ö
ïx ç ÷ , x ¹0
5. If f ( x ) = í è e1/ x + e1/ x ø , then at x = 0, f ( x ) is
ï x =0
î 0,
(a) differentiable (b) not differentiable
(c) f ¢ (0+ ) = -1 (d) f ¢ (0- ) = 1
Session 7
Differentiability in an Interval
(i) A function f ( x ) defined in an open interval (a, b ) is Remark
said to be differentiable or derivable in open interval The above example, can also be solved as follows
(a, b ), if it is differentiable at each point of (a, b ) . æ 2x ö
y = f ( x ) = sin–1 ç ÷, let x = tan q
(ii) A function f ( x ) defined in a close interval [a, b ] is è1 + x 2 ø
said to be differentiable or derivable at the end points æ 2 tan q ö
\ y = sin–1 ç ÷ Þ y = sin–1(sin 2q)
a and b, if it is differentiable from the right at a and è 1 + tan2 q ø
from the left at b. In other words, lim f ( x ) – f (a ) \ y = 2q or y = 2 tan–1 x
x ®a+ x –a dy
=
2
, which states f ¢( x ) exists for all x Î R. “Which is
dx 1 + x 2
and lim f ( x ) – f (b ) both exist. wrong as we have not checked the domain of f ( x ) .” So, students
x ®b– x –b are advised to solve these problems carefully, while applying this
method.
y Example 44 Discuss the differentiability of y Example 45 Let [ ] denotes the greatest integer
æ 2x ö function and f ( x ) = [tan 2 x ], then
f ( x ) = sin –1 ç ÷. [IIT JEE 1993]
è1+ x ø
2
(a) lim f (x) doesn’t exist (b) f (x) is continuous at x = 0
x®0
æ 2x ö (c) f (x) is not differentiable at x = 0
Sol. We have, f ( x ) = sin –1 ç ÷ (d) f ¢ (0) = 1
è1+ x 2 ø
Sol. Here, [ ] denotes the greatest integral function.
1 d æ 2x ö –45° < x < 45°
Þ f ¢( x ) = ´ ç ÷ Thus,
æ 2x ö
2 dx è 1 + x 2 ø Þ tan(– 45° ) < tan x < tan( 45° )
1– ç ÷
è1 + x 2 ø Þ –1 < tan x < 1 Þ 0 < tan 2 x < 1
(1 + x 2 ) é ( 1 + x 2 ) ( 2) – 2x ( 2x ) ù Since, f ( x ) = [tan 2 x ] = 0
= ´ê ú
(1 + x 2 )2 – 4 x 2 êë (1 + x 2 )2 ûú Therefore, f ( x ) is zero for all values of x from (– 45° ) to
( 45° ). Thus, f ( x ) exists when x ® 0 and also it is
(1 + x )
2
( 2 + 2x – 4 x )
2 2
= ´ continuous at x = 0, f ( x ) is differentiable at x = 0 and has a
1 + 2x 2 + x 4 – 4 x 2 (1 + x 2 )2 value 0. (i.e. f (0) = 0).
(1 + x 2 ) ( 2 – 2x 2 ) Hence, (b) is the correct answer.
= ´
1 – 2x 2 + x 4 (1 + x 2 )2

=
(1 + x 2 )
´
( 2 – 2x 2 )
=
(1 + x 2 )
´
2( 1 – x 2 ) Theorems of Differentiability
(1 – x ) 2 2 (1 + x )2 2 2
|1 – x | (1 + x 2 )2 Theorem 1 If f ( x ) and g( x ) are both derivable at
f (x )
[since 1 + x 2 ¹ 0] x = a, f ( x ) ± g( x ), f ( x ) × g( x ) and will also be
2 g( x )
1 2 (1 – x )
Þ f ¢( x ) = ´ …(i) ì f (x )ü
2
|(1 – x )| (1 + x 2 ) derivable at x = a íonly if g(a ) ¹ 0 for ý.
î g (x ) þ
Here, in Eq. (i), f ¢( x ) exists only if, |1– x 2 | ¹ 0
Theorem 2 If f ( x ) is derivable at x = a and g( x ) is not
Þ 1 – x 2 ¹0
differentiable at x = a, then f ( x ) ± g( x ) will not be
Þ x 2 ¹1 Þ x ¹ ± 1 derivable at x = a.
Thus, f ¢( x ) exists only, if x Î R – {–1,1}. e.g. f ( x ) = cos | x | is derivable at x = 0 and g( x ) = | x | is not
\ f ( x ) is differentiable for all x Î R – { 1, – 1}. derivable at x = 0.
Chap 06 Continuity and Differentiability 337

Then, cos | x | + | x | is not derivable at x = 0. y Example 46 Let h( x ) = min { x , x 2 } for every real
However, nothing can be said about the product function, number of x. Then, [IIT JEE 1998]
as in this case (a) h is not continuous for all x
f ( x ) = x is derivable at x = 0 (b) h is differentiable for all x
g( x ) = | x | is not derivable at x = 0
(c) h¢ (x) = 1for all x
ì x 2 , if x ³ 0
But, f ( x ) × g( x ) = í 2 (d) h is not differentiable at two values of x
î- x , if x < 0 Sol. Here, h ( x ) = min { x , x 2 } can be drawn on graph in two
which is derivable at x = 0. steps.
Theorem 3 If both f ( x ) and g( x ) are non-derivable, then (a) Draw the graph of y = x and y = x 2 also find their
nothing can be said about the sum/difference/product point of intersection.
function. Clearly, x = x 2 Þ x = 0, 1
e.g. f ( x ) = sin | x |, not derivable at x = 0 Y
g( x ) = | x |, not derivable at x = 0 y = x2
Then, the function y=x
F ( x ) = sin | x | + | x |, not derivable at x = 0 1
G ( x ) = sin | x | - | x |, derivable at x = 0
Theorem 4 If f ( x ) is derivable at x = a and f (a ) = 0 and X′
O 1
X
g( x ) is continuous at x = a.
Then, the product function F ( x ) = f ( x ) × g( x ) will be
derivable at x = a. Y′
f (a + h ) × g(a + h ) - 0 (b) To find h ( x ) = min { x , x 2 } neglecting the graph above
Proof F ¢ (a + ) = lim = f ¢ (a ) × g(a )
h®0 h the point of intersection, we get
f (a - h ) × g(a - h ) - 0
F ¢ (a - ) = lim = f ¢ (a ) × g(a ) Y
h®0 –h
\ Derivable at x = a. x
1
Theorem 5 Derivative of a continuous function need not
be a continuous function. X′
x2
X
(1, 0)
ì 2 (0, 0)
ïx × sin æç ö÷ , if x ¹ 0
1
e.g. f (x ) = í èxø
ïî 0, if x = 0 x
Y′
Here, f (0 + ) = 0 and f (0 - ) = 0
Thus, from the above graph,
\ Continuous at x = 0.
ì x , x £ 0 or x ³ 1
ì
ï2 x × sin - x 2 × cos æç ö÷ ×
1 1 1 , x ¹0 h (x ) = í 2
and f ¢ (x ) = í x èx ø x2 î x , 0£ x £1
ïî 0, x =0 which shows h ( x ) is continuous for all x. But not
differentiable at x = {0, 1}.
Þ f ¢ ( x ) is not continuous at x = 0.
Thus, h ( x ) is not differentiable at two values of x.
é as lim f ¢ ( x ) doesn’ t existù
ëê x ® 0 ûú Hence, (d) is the correct answer.

Remark y Example 47 Let f :R ® R be a function defined by


One must remember the formula which we can write as f ( x ) = max { x , x 3 }. The set of all points where f ( x ) is
f ( x ) + g( x ) f ( x ) – g( x )
max {f ( x ), g( x )} = + not differentiable, is
2 2 [IIT JEE 2001]
f ( x ) + g( x ) f ( x ) – g( x ) (a) {–1, 1} (b) {–1, 0 }
min {f ( x ), g( x )} = –
2 2 (c) {0, 1} (d) {–1, 0, 1}
338 Textbook of Differential Calculus

Sol. f ( x ) = max { x , x 3 }. Consider the graph separately of y Example 49 The total number of points of
y = x 3 and y = x and find their point of intersection; non-differentiability of
x3 = x ì 3ü
Clearly, f ( x ) = max ísin 2 x , cos 2 x , ý in [0, 10p], is
Þ x = 0, 1, – 1 î 4þ
Y (a) 40
y = x3
y=x (b) 30
A
(1, 1) (c) 20
O
(d) 10
X′ X
ì 3ü
Sol. Here, f ( x ) = max ísin 2 x , cos 2 x , ý
(–1, –1) î 4þ
B Y
Y′ max {sin2x, cos2x, 3 }
4
1
Now, to find f ( x ) = max { x , x 3 } neglecting the graph below 3/4
the point of intersection, we get the required graph of
f ( x ) = max { x , x 3 }. X′ X
O π π
2
Y Y′
y = x3
y=x Since, sin 2 x and cos 2 x are periodic with period p
1
A and in [0, p ] , there are four points of non-differentiability of
(1, 1) f ( x ).
–1
X′ X \ In [0, 10p ], there are 40 points of non-differentiability.
O 1
B
(–1, –1) Hence, (a) is the correct answer.
–1
y Example 50 If f ( x ) = | x + 1|{| x| + | x – 1|}, then draw
Y′
the graph of f ( x ) in the interval [–2 , 2] and discuss the
ì x , if x Î (–¥, – 1] È [0,1] continuity and differentiability in [–2 , 2].
Thus, from above graph, f ( x ) = í 3
î x , if x Î [–1, 0] È [1, ¥ ) Sol. Here, f ( x ) = | x + 1| {| x | + | x – 1| }
which shows f ( x ) is not differentiable at 3 points, i.e. ì( x + 1) (2x – 1), – 2 £ x < – 1
x = {–1, 0, 1}. (Due to sharp edges) ï–( x + 1) (2x – 1), – 1 £ x < 0
ï
f (x ) = í
ï( x + 1), 0£ x <1
Hence, (d) is the correct answer.
ïî( x + 1) (2x – 1), 1£ x £2
y Example 48 Let f ( x ) be a continuous function,
Thus, the graph of f ( x ) is
" x ÎR, f (0) = 1 and f ( x ) ¹ x for any x ÎR, then
show f ( f ( x )) > x , " x ÎR + . Y
(2, 9)

Sol. Let g ( x ) = f ( x ) – x
2)
So, g ( x ) is continuous and g (0) = f (0)–0. (1,
Þ g(0) = 1 1
x+
Now, it is given that g ( x ) ¹ 0 for any x Î R 1
[as f ( x ) ¹ x for any x Î R]
So, g ( x ) > 0, " x Î R + X′
–2 –1 0 1 2
X
+
i.e. f (x ) > x, " x Î R
Y′
Þ f ( f ( x )) > f ( x ) > x , " x Î R +
Clearly, continuous for x Î R and has differentiability for
or f ( f ( x )) > x , " x Î R + x Î R – {– 1, 0, 1}
Chap 06 Continuity and Differentiability 339

y Example 51 If the function From graph of f ( x ),


ì f ( x ), for –2 £ x < – 1
é ( x - 2) 3 ù ï –1,
f (x ) = ê ú sin ( x - 2) + a cos( x - 2), ï for –1 £ x < 0
g(x ) = í
ë a û 0, for 0£ x <1
ï
(where [ ] denotes the greatest integer function) is ïî f ( x ), for x ³1
continuous and differentiable in (4, 6 ), then ì x 2 + 2x , for –2 £ x < – 1
(a) a Î [8, 64] (b) a Î (0, 8] ï
ï –1, for –1 £ x < 0
(c) a Î [64, ¥) (d) None of these Þ g(x ) = í
ï 0, for 0£ x <1
Sol. We have, x Î( 4, 6) Þ 2 < x - 2 < 4 ï x 2 - 2x , for x ³1
î
8 ( x - 2)3 64
Þ < < [Qa > 0] Thus, graph of g ( x ) is
a a a
Y
For f ( x ) to be continuous and differentiable in ( 4, 6),
é ( x - 2) 3 ù
ê ú must attain a constant value for x Î( 4, 6).
êë a úû
Clearly, this is possible only when a ³ 64. X′ 0
X
–2 –1 1 2 3
In that case, we have
–1
f ( x ) = a cos( x - 2), which is continuous and differentiable
\ a Î [64, ¥ )
Hence, (c) is the correct answer. Y′
From above figure, it is clear that g ( x ) is not
y Example 52 If f ( x ) = x – 2| x | and
2
continuous at x = 0, 1.
ìmin { f (t ) :– 2 £ t £ x , – 2 £ x £ 0}
g (x ) = í y Example 53 Let f ( x ) = f ( x ) + y ( x ) and f ¢ (a ), y ¢ (a )
îmax{ f (t ) : 0 £ t £ x , 0 £ x £ 3}
are finite and definite. Then,
(i) Draw the graph of f (x ) and discuss its continuity and
(a) f (x) is continuous at x = a
differentiability.
(b) f (x) is differentiable at x = a
(ii) Find and draw the graph of g (x ). Also, discuss the
(c) f ¢ (x) is continuous at x = a
continuity.
(d) f ¢ (x) is differentiable at x = a
ìï x 2 – 2x , x ³ 0
Sol. (i) Graph of f ( x ) = í is shown as Sol. We know that the sum of two continuous (differentiable)
îï x + 2x , x < 0
2 functions is continuous (differentiable).
Y \ f ( x ) is continuous and differentiable at x = a.
Hence, (a) and (b) are the correct answers.

y Example 54 If f ( x ) = x + tan x and g ( x ) is the inverse


X′ 0
X of f ( x ), then g ¢( x ) is equal to
–2 –1 1 2 1 1
(a) (b)
–1
y = x2 – 2|x| 1 + (g (x) - x) 2
2 + (g (x) + x) 2
1
(c) (d) None of these
Y′ 2 + (g (x) - x) 2
which shows f ( x ) is continuous for all x Î R and Sol. We have, f ( x ) = x + tan x
differentiable for all x Î R – {0}. -1 -1 -1
Þ f (f ( x )) = f ( x ) + tan( f ( x ))
(ii) We know that,
If f ( x ) is an increasing function on [a, b ] , then Þ x = g ( x ) + tan( g ( x )) …(i)
-1
max { f (t ); a £ t £ x , a £ x £ b } = f ( x ) [Q g ( x ) = f ( x )]
min { f (t ); a £ t £ x , a £ x £ b } = f (a ) 1 = g ¢ ( x ) + sec 2 ( g ( x )) × g ¢ ( x )
If f ( x ) is decreasing function on [a, b ], then
1
max { f (t ); a £ t £ x , a £ x £ b } = f (a ) Þ g ¢( x ) =
1 + sec 2 ( g ( x ))
min { f (t ); a £ t £ x , a £ x £ b } = f ( x )
340 Textbook of Differential Calculus

1 Also, f (1) = 1
Þ g ¢( x ) =
2 + tan 2 ( g ( x )) \ f ( x ) is Continuous at x = 1
Þ g ¢( x ) =
1 ì x, 0£ x <1
[from Eq. (i)] f (x ) = í 2 , non-differentiable at x = 1
2 + ( x - g ( x ))2 î x + x - 1 , 1£ x <2
Hence, (c) is the correct answer. lim f ( x ) = lim- x 2 sgn[ x ] + { x }
x ®2 - x ®2
y Example 55 If f ( x ) is differentiable function and
= 4 ´1+1 = 5
( f ( x ) × g ( x )) is differentiable at x = a, then
lim f ( x ) = lim+ sin x + | x - 3| = 1 + sin 2
(a) g (x) must be differentiable at x = a x ®2 + x ®2

(b) g (x) is discontinuous, then f (a) = 0 Thus, f ( x ) is neither continuous nor differentiable at x = 2.
(c) f (a) ¹ 0, then g (x) must be differentiable Hence, (c) and (d) are the correct answers.
(d) None of the above
y Example 58 A real valued function f ( x ) is given as
éd ù
Sol. ê ( f ( x ) × g ( x ))ú = f ¢ (a ) g (a ) ì x
ë dx ûx =a ï ò0 2{ x } dx , x + { x } Î I
g (a + h ) - g (a )
+ lim × f (a ) ï 1 1 3
h® 0 h f ( x ) = í x 2 - x + , < x < and x ¹ 1,
ï 2 2 2
If f (a ) ¹ 0 Þ g ¢(a ) must exist. 1
Also, if g ( x ) is discontinuous, f (a ) must be 0 for f ( x ) × g ( x ) ï x - x + , otherwise
2
î 6
to be differentiable.
where [ ] denotes greatest integer less than or equals
Hence, (b) and (c) are the correct answers.
to x and { } denotes fractional part function of x. Then,
y Example 56 If f ( x ) = [ x -2 [ x 2 ]], (where [ × ] denotes
æ 1 1ö
(a) f (x) is continuous and differentiable in x Î ç - , ÷
the greatest integer function) x ¹ 0, then incorrect è 2 2ø
statement
é 1 1ù
(a) f (x) is continuous everywhere (b) f (x) is continuous and differentiable in x Î ê - , ú
ë 2 2û
(b) f (x) is discontinuous at x = 2
(c) f (x) is non-differentiable at x = 1 é 1 3ù
(c) f (x) is continuous and differentiable in x Î ê , ú
(d) f (x) is discontinuous at infinitely many points ë2 2 û
(d) f (x) is continuous but not differentiable in x Î (0, 1)
Sol. Here, 0 £ [x 2 ] £ x 2
Sol. Here, x + {x } Î I Þ x + x - [x ] Î I
Þ 0 £ x -2 [ x 2 ] £ 1 Þ [ x -2 [ x 2 ]] = 0 or 1
n
f ( x ) is discontinuous at x 2 = n , n Î N Þ x = n Þ 2x - [ x ] Î I , possible for x = , n Î I
2
\ f ( x ) is neither continuous nor differentiable at æ1ö 1/ 2 1 æ3ö 3/2 5
\ f ç ÷ = ò 2 { x }dx = , f ç ÷ = ò 2 { x } dx =
x = n, n Î N. è2ø 0 4 è2ø 0 4
Hence, (b), (c) and (d) are the correct answers. æ -1 ö - 1/ 2 -3
and fç ÷=ò 2 { x } dx = , f ( 1) = 1
è 2 ø 0 4
ì x 2 (sgn [ x ]) + { x }, 0 £ x £ 2
y Example 57 If f ( x ) = í , ì 1 1
x=
î sin x + | x - 3|, 2 £ x £ 4 ï 4
,
2
ï
where [ ] and { } represents greatest integer and ï
5
, x=
3
fractional part function respectively, then ï 4 2
(a) f (x) is differentiable at x = 1 ï -3 -1
ï , x =
(b) f (x) is continuous but non-differentiable at x = 2 Then, f ( x ) = í 4 2
ï 1, x =1
(c) f (x) is non-differentiable at x = 2 ï 1 1 3
(d) f (x) is discontinuous at x = 2 ï x 2 - x + , < x < and x ¹ 1
ï 2 2 2
Sol. For continuity at x = 1 ï 1
ïî x 2
- x + , otherwise
lim f ( x ) = lim+ x 2 sgn[ x ] + { x } = 1 + 0 = 1 6
x ®1+ x ®1
Clearly, continuous for x Î(0, 1) but not differentiable.
lim- f ( x ) = lim- x 2 sgn[ x ] + { x } = 0 + 1 = 1 Hence, (d) is the correct answer.
x ®1 x ®1
Chap 06 Continuity and Differentiability 341

Exercise for Session 7


æ p pö
1. If f ( x ) = sin( p ( x - [ x ])), " x Î ç - , ÷ , where [ × ] denotes the greatest integer function, then
è 2 2ø
p p
(a) f (x ) is discontinuous at x = {-1, 0, 1} (b) f (x ) is differentiable for x Î æç - , ö÷ - {0}
è 2 2ø
p p
(c) f (x ) is differentiable for x Î æç - , ö÷ - {-1, 0, 1} (d) None of these
è 2 2ø

ì x - 1, -1 £ x < 0
2. Let f ( x ) = í 2 , g( x ) = sin x and h( x ) = f (| g( x ) | ) + | f ( g( x )) | . Then,
î x , 0 £ x £1
(a) h (x ) is continuous for x Î [-1, 1] (b) h (x ) is differentiable for x Î [-1, 1]
(c) h (x ) is differentiable for x Î [-1, 1] - {0} (d) h (x ) is differentiable for x Î (-1, 1) - { 0}
ì |1 - 4x 2|, 0 £ x < 1
3. If f ( x ) = í 2 , where [ ] denotes the greatest integer function, then
î[ x - 2x ], 1 £ x < 2
(a) f (x ) is continuous for all x Î[0, 2) (b) f (x ) is differentiable for all x Î [ 0, 2 ) - { 1}
(c) f (x ) is differentiable for all x Î [0, 2) - ìí , 1üý
1
(d) None of these
î2 þ
1
4. Let f ( x ) = ò | x - t | t dt , then
0
(a) f (x ) is continuous but not differentiable for all x ÎR (b) f (x ) is continuous and differentiable for all x ÎR
(c) f (x ) is continuous for x ÎR - ìí üý and f (x ) is differentiable for x ÎR - ìí , üý
1 1 1
î 2þ î 4 2þ
(d) None of these
5. Let f ( x ) be a function such that f ( x + y ) = f ( x ) + f ( y ) for all x and y and f ( x ) = (2x 2 + 3x ) × g( x ) for all x, where
g( x ) is continuous and g(0) = 3. Then, f ¢( x ) is equal to
(a) 6 (b) 9 (c) 8 (d) None of these
6. If a function g( x ) which has derivatives g ¢( x ) for every real x and which satisfies the following equation
g( x + y ) = e y g( x ) + e x g( y ) for all x and y and g ¢ (0) = 2, then the value of {g ¢ ( x ) - g( x )} is equal to
2 1
(a) e x (b) e x (c) e x (d) 2e x
3 2
æ xy ö f ( x ) × f ( y )
7. Let f : R ® R be a function satisfying f ç ÷ = , " x , y ÎR
è 2 ø 2
and f (1) = f ¢ (1) ¹ 0. Then, f ( x ) + f (1 - x ) is (for all non-zero real values of x)
1
(a) constant (b) can’t be discussed (c) x (d)
x
æ x + y ö f (x ) + f (y )
8. Let f ( x ) be a derivable function at x = 0 and f ç ÷= (K Î R, K ¹ 0, 2). Then, f ( x ) is
è K ø K
(a) even function (b) neither even nor odd function
(c) either zero or odd function (d) either zero or even function
æx + yö
9. Let f : R - ( - p , p ) be a differentiable function such that f ( x ) + f ( y ) = f ç ÷.
è 1 - xy ø
p f (x )
If f (1) = and lim = 2 . Then, f ( x ) is equal to
2 x ®0 x
1 -1 p
(a) 2 tan-1 x (b) tan x (c) tan-1 x (d) 2p tan-1 x
2 2
ìïmax {f (t ), 0 £ t £ x }, for 0 £ x £ p
10. Let f ( x ) = sin x and g( x ) = í 1 - cos x , . Then, g( x ) is
ïî for x > p
2
(a) differentiable for all x ÎR (b) differentiable for all x ÎR - {p}
(c) differentiable for all x Î (0, ¥) (d) differentiable for all x Î (0, ¥) - {p}
JEE Type Solved Examples :
Single Option Correct Type Questions
l Ex. 1 The values of a and b so that the function = lim (a cos 2x – b sin x )
p+
ìx + a 2 sin x , 0 £ x < p/ 4 x ®
2
ï
f ( x ) = í2 x cot x + b, p/ 4 £ x £ p/ 2 is continuous for é æp ö æp öù
= lim êa cos 2 ç + h ÷ – b sin ç +h ÷ú
ïa cos 2 x – b sin x , p/ 2 < x £ p h ®0
ë è2 ø è2 øû
î
x Î[0, p ], are RHL = – a – b
p p p p p p
(a) a = ,b = - (b) a = - , b = and f ( p/ 2) = 2 cot + b = b
6 6 6 12 2 2
p p For continuity, LHL (at x = p / 2 ) = RHL (at x = p /2) = f ( p / 2 )
(c) a = , b = - (d) None of these Þ b = – a – b Þ a + 2b = 0 ...(ii)
6 12
On solving Eqs. (i) and (ii), we get
Sol. (c) Since, f ( x ) is continuous for x Î[0, p ].
a = p/ 6, b = – p/ 12
\ f ( x ) is continuous at x = p/ 4 and x = p/ 2 and hence to
discuss continuity at x = p/ 4 and x = p/ 2.
l Ex. 2 Let f be an even function and f ¢ (0 ) exists, then
Now, at x = p/ 4 ¢
f (0 ) is
Left hand limit (at x = p/ 4) (a) 1 (b) 0
(c) -1 (d) -2
= lim f ( x ) = lim ( x + a 2 sin x ) Sol. (b) Since, f is an even function.
p– p–
x ® x ®
4 4 So, f (–x ) = f ( x ), for all x ...(i)
= lim {( p/ 4 – h ) + a 2 sin ( p/ 4 – h )} Also, f ¢(0) exists.
h ®0
p So, Rf ¢(0) = Lf ¢(0)
\ LHL = lim f ( x ) = +a
p- 4 f ( 0 + h ) – f ( 0) f ( 0 – h ) – f ( 0)
x ® Þ lim = lim
4 h ®0 h h ®0 –h
Right hand limit at x = p/ 4
f ( h ) – f ( 0) f (– h ) – f (0)
= lim f ( x ) = lim (2x cot x + b ) Þ lim = lim
p +
p + h ®0 h h ®0 –h
x® x®
4 4
f ( h ) – f ( 0) f ( h ) – f ( 0)
æp ö æp ö Þ lim = lim
= lim 2 ç + h ÷ cot ç + h ÷ + b h ®0 h h ®0 –h
h ®0 è4 ø è4 ø
p [using Eq. (i), f (–h ) = f (h )]
Þ RHL = lim f ( x ) = + b
p+ 2 f ( h ) – f ( 0) f ( h ) – f ( 0)

4
Þ lim = – lim
h ®0 h h ®0 h
æpö p
Also, f ç ÷ = + b f ( h ) – f ( 0)
è4ø 2 Þ 2 lim =0
h ®0 h
æpö
For continuity, LHL (at x = p / 4 ) = RHL (at x = p /4) = f ç ÷ Þ 2 f ¢(0) = 0 Þ f ¢(0) = 0
è4ø
p p Ex. 3 The set of points where x 2 | x | is thrice
Þ +a= +b l
4 2
p differentiable, is
Þ a –b = …(i) (a) R (b) R - { 0, 1}
4
At x = p/ 2 (c) [0, ¥ ) (d) R - { 0 }
Left hand limit (at x = p/ 2) Sol. (d ) Let f ( x ) = x 2 | x | which could be expressed as
æp ö æp ö
= lim (2x cot x + b ) = lim 2 ç –h ÷ cot ç –h ÷ + b ì– x 3 , x < 0
p – h ® 0 è 2 ø è2 ø ï

2 f ( x ) = í 0, x = 0
LHL = b ï x 3, x > 0
Right hand limit (at x = p/ 2) î
Chap 06 Continuity and Differentiability 343

ì–3x 2 , x < 0 As, f ( x ) is differentiable in [0, 2].


ï Þ f ( x ) is continuous and differentiable at x = 1
This gives, f ¢( x ) = í 0, x = 0
ï 3x 2 , x > 0 Þ lim f ( x ) = lim f ( x ) = f (1)
î x ® 1– x ® 1+

So, f ¢( x ) exists for all real x. p


a + b = –2 + =a+b
ì–6x , x < 0 4
¢¢ ï p
f ( x ) = í 0, x = 0 Þ a + b = –2 + ...(i)
ï 6x , x > 0 4
î
Again, since f ( x ) is differentiable at x = 1
So, f ¢¢ ( x ) exists for all real x.
(LHD at x = 1 ) = (RHD at x = 1)
ì–6, x < 0 f ( x ) – f ( 1) f ( x ) – f ( 1)
ï Þ lim = lim
f ¢¢¢ ( x ) = í 0, x = 0 x ®1 – x –1 x ®1+ x –1
ï 6, x > 0
î (ax 3 + b ) – (a + b )
Þ lim
However, f ¢¢¢(0) doesn’t exist, since f ¢¢¢(0- ) = - 6 and x –1

x ®1

f ¢¢¢(0+ ) = 6 which are not equal. (2 cos px + tan –1 x ) – (a + b )


= lim
Thus, the set of points where f ( x ) is thrice differentiable is x ® 1+ x –1
R – { 0 }. 1
–2p sin px +
|x + 2| 1 + x2
l Ex. 4 The function f ( x ) = , is continuous for Þ 3a = lim
1 +
x ®1
tan –1 ( x + 2 )
1 1
(a) x Î R (b) x Î R - { 0 } Þ 3a = or a = ...(ii)
2 6
(c) x Î R - { - 2 } (d) None of these
From Eqs. (i) and (ii), we get
Sol. (c) Clearly, f is continuous except possibility at x = –2
1 p 13
RHL (at x = - 2) = lim f ( x ) a= and b = –
x ® –2 + 6 4 6
( x + 2)
= lim =1 x m f (1 ) + h( x ) + 1
x ® –2 + tan –1( x + 2) l Ex. 6 If g ( x ) = lim is continuous at
m ®¥ 2 x m + 3x + 3
LHL (at x = -2) = lim f (x )
x ® –2 – x =1 and g (1 ) = lim {log e (ex )} 2 / loge x , then the value of
–( x + 2) x ®1
= lim = –1
x ® –2 –
tan ( x + 2)
–1 2 g (1 ) + 2 f (1 ) – h(1 ) when f ( x ) and h( x ) are continuous at
x =1, is
So, f is not continuous at x = –2.
(a) 0 (b) 1 (c) 2 (d) 3
ì sin [ x 2 ]p Sol. (b) Here, g (1) = lim {log e + loge x } 2 / loge x
ï 2 + ax 3 + b, for 0 £ x £ 1 x ®1
l Ex. 5 If f ( x ) =
í x – 3 x – 18
ï = lim {1 + loge x } 2 / loge x
î2 cos px + tan x , for 1 < x £ 2
–1 x ®1
2
lim loge x
differentiable function in [0, 2], where [ × ] denotes the great- = ex ®1 loge x
est integer function, then
g ( 1) = e 2 ...(i)
1 p 13 1 p
(a) a = , b = - (b) a = - , b = ì x f ( 1) + h ( x ) + 1ü
m
6 4 6 6 4 Also, lim g ( x ) = lim lim í ý
1 p 13 x ®1–
x ®1–m ®¥
î 2x + 3x + 3 þ
m
(c) a = - , b = - (d) None of these
6 4 6
ì x m f ( 1) + h ( x ) + 1ü
Sol. (a) Here, [ x 2 ] = 0, for all 0 £ x < 1 = lim í lim ý
m ® ¥ x ® 1–
î 2x m + 3x + 3 þ
and [ x 2 ] = 1, for x = 1
h ( 1) + 1
\ sin [ x 2 ]p = 0, for 0 £ x £ 1 = [as x < 1 Þ lim x m = 0]
3+3 m ®¥
ìï ax 3 + b, 0£ x £1
Hence, f ( x ) = í h ( 1) + 1
ïî2 cos px + tan –1 x , 1< x £2 lim g ( x ) = ...(ii)
x ® 1– 6
344 Textbook of Differential Calculus

ì x m f ( 1) + h ( x ) + 1ü On substituting x = 1, 2, 3, K, N in Eq. (iii) and adding


Again, lim g ( x ) = lim lim í ý
x ® 1+ x ® 1+ m ® ¥ î 2x m + 3x + 3 þ æ 1ö æ1ö ì 1 1 1 ü
g ¢ ¢ ç N + ÷ - g ¢ ¢ ç ÷ = - 4 í1 + + +K+ ý
è 2ø è2ø î 9 25 ( 2 N - 1) þ
2
ì x m f ( 1) + h ( x ) + 1ü
= lim í lim ý
m ® ¥ x ® 1+
î 2x m + 3x + 3 þ l Ex. 8 Let y = f ( x ) be a differentiable function, " x Î R
f (1) + h ( x )/x m
+ 1/x m
f ( 1) and satisfies;
= lim lim =
m ® ¥ x ® 1+ 2 + 3/x m –1 + 3/x m 2
f ( x ) = x + ò x 2 z f ( z ) dz + ò x z 2 f ( z ) dz , then
1 1
0 0
f ( 1)
\ lim g ( x ) = ...(iii) 20x 20x
+ 2
x ®1 (a) f ( x ) = ( 2 + 9x ) (b) f ( x ) = ( 4 + 9x )
119 119
From Eqs. (i), (ii) and (iii), we get 10x 5x
h ( 1) + 1 f ( 1) (c) f ( x ) = ( 4 + 9x ) (d) f ( x ) = ( 4 + 9x )
e2 = = [as g ( x ) is continuous at x = 1] 119 119
6 2 1 1
Þ h (1) = 6e 2 – 1 and f (1) = 2e 2
Sol. (b) We have, f ( x ) = x + ò0 x 2 z f (z ) dz + ò0 x z 2 f (z )dz
1 1
\ 2g (1) + 2 f (1) – h (1) = 2e 2 + 4e 2 – 6e 2 + 1 = 1 f (x ) = x + x 2 ò 0 z f (z )dz + x ò 0 z
2
or f (z )dz
Þ 2g ( 1) + 2 f ( 1) – h ( 1) = 1 1 1 2
Let l1 = ò 0 z f (z )dz and l 2 = ò 0 z f (z )dz
l Ex. 7 Let g ( x ) = log f ( x ), where f ( x ) is a twice
\ f ( x ) = x + x 2 l1 + xl 2 ...(i)
differentiable positive function on (0, ¥) such that 1
f ( x + 1 ) = xf ( x ). Then, for Now, l1 = ò 0 z f (z )dz
æ 1ö æ1ö
N = 1 , 2 , 3 , K, g ¢ ¢ ç N + ÷ - g ¢ ¢ ç ÷ is equal to 1
l1 = ò 0 z (z + z l1 + zl 2 )dz
2
è 2ø è 2ø
[IIT JEE 2008] [using Eq. (i), as f (z ) = z + z 2 l1 + zl 2 ]
ì ü 1 1 3
Þ l1 = (1 + l 2 ) ò z 2 dz + l1 ò 0 z dz
1 1 1
(a) - 4 í1 + + +K+ 2ý
î 9 25 ( 2 N - 1) þ
0
1 1
ì ü æz3 ö æz4 ö
1 1
(b) 4 í1 + + +K+
1 Þ l1 = ( 1 + l 2 ) ç ÷ + l1 ç ÷
2ý è 3 ø0 è 4 ø0
î 9 25 ( 2 N - 1) þ
ì ü 1 + l 2 l1
1 1
(c) - 4 í1 + + +K+
1 Þ l1 = +

î 9 25 ( 2 N + 1) þ 3 4
Þ 9 l1 – 4 l 2 = 4 ...(ii)
ì 1 1 1 ü
(d) 4 í1 + + +K+ 2ý
1 2 1 2
î 9 25 ( 2 N + 1) þ Also, l 2 = ò 0
z f (z ) dz or l 2 = ò 0
z {z + z 2 l1 + zl 2 } dz

Sol. (a) We have, g ( x ) = log f ( x ) …(i) [using Eq. (i), as


Þ f (x ) = e g (x ) f ( z ) = z + z 2 l 1 + zl 2 ]
+ 1) 1 3 1 4
Þ f ( x + 1) = e g ( x
+ 1)
Þ l2 = ò0z (1 + l 2 )dz + l1 ò0z dz
Þ x f ( x ) = e g (x [given f ( x + 1) = x f ( x )]
( 1 + l 2 ) l1
+ 1) Þ l2 = +
Þ log x + log f ( x ) = log e g (x [taking log both sides] 4 5
Þ log x + g ( x ) = g ( x + 1) [from Eq. (i)] or 15l 2 – 4 l1 = 5 ...(iii)
i.e. g ( x + 1) - g ( x ) = log x …(ii) From Eqs. (ii) and (iii), we get
1 9 l1 – 4 l 2 = 4 and 15l 2 – 4 l1 = 5
On replacing x by x - in Eq. (ii), we get
2 80 61
l1 = , l2 = ...(iv)
æ 1ö æ 1ö æ 1ö 119 119
g ç x + ÷ - g ç x - ÷ = log ç x - ÷
è 2ø è 2ø è 2ø Thus, Eq. (i) becomes
= log (2x - 1) - log 2 f (x ) = x +
80 2
x +
61
x [from Eq. (iv)]
æ 1ö æ 1ö -4 119 119
\ g ¢ ¢ çx + ÷ - g ¢ ¢ çx - ÷ = …(iii)
è 2ø è 2 ø ( 2x - 1) 2 20x
Hence, f (x ) = (4 + 9x )
119
Chap 06 Continuity and Differentiability 345

l Ex. 9 A function f : R ® R satisfies the equation Sol. (c) As we know, [ x ] is neither continuous nor differentia-
ble at integer values.
f ( x + y ) = f ( x ) × f (y ) for all , f ( x ) ¹ 0. Suppose that the
So, f ( x ) is continuous and differentiable in (7, 9).
function is differentiable at x = 0 and f ¢ (0 ) = 2 . Then,
é ( x - 5) 2 ù
(a) f ¢ ( x ) = 2 f ( x ) (b) f ¢ ( x ) = f ( x ) If ê ú =0
(c) f ¢ ( x ) = f ( x ) + 2 (d) f ¢ ( x ) = 2 f ( x ) + x ë A û
Sol. (a) We are given that Þ A > ( x - 5) 2 …(i)
f ( x + y ) = f ( x ) × f (y ) ...(i)
As, x Î(7, 9 ) Þ x - 5 Î(2, 4 )
f ( h ) – f ( 0 )
and f ¢(0) = lim =2 ...(ii) Þ ( x - 5)2 Î ( 4, 16) …(ii)
h ®0 h
f (x + h) – f (x ) From Eqs. (i) and (ii), we get
Now, f ¢( x ) = lim
h ®0 h A ³ 16
f ( x + h ) – f ( x + 0)
Þ f ¢( x ) = lim l Ex. 12 If f ( x ) = [ 2 + 5 | n | sin x ], where n Î I has exactly
h ®0 h
f ( x ) × f ( h ) – f ( x ) × f ( 0) 9 points of non-derivability in (0 , p ), then possible values of
Þ f ¢( x ) = lim
h ®0 h n are (where [ x ] denotes greatest integer function)
[using Eq. (i)] (a) ±3 (b) ± 2
¢ f ( h ) – f ( 0) (c) ± 1
Þ f ( x ) = f ( x ) lim (d) None of these
h ®0 h Sol. (c) We have, [2 + 5| n |sin x ] = 2 + [5| n |sin x ] , let
Þ ¢
f (x ) = 2 f (x ) [using Eq. (ii)] y = 5| n |sin x
\ Number of points of non-derivability
l Ex. 10 Let f be a function such that = 2(5| n | - 1) + 1 = 10| n | - 1
f ( x + f (y )) = f ( x ) + y , " x , y Î R, then find f (0 ). According to the question, 10 | n | - 1 = 9
If it is given that there exists a positive real d, such that Þ 10 | n | = 10
f (h ) = h for 0 < h < d, then find f ¢( x ) , Þ |n | = 1
(a) 0, 1 (b) -1, 0 (c) 2, 1 (d) -2, 0 Here, n = ± 1
Sol. (a) Let x = 0, y = 0 in f ( x + f (y )) = f ( x ) + y
f (0 + f (0)) = f (0) + 0 l Ex. 13 The number of points of discontinuity of
Þ f ( f (0)) = f (0) f ( x ) = [ 2 x ] 2 - { 2 x } 2 (where [ × ] denotes the greatest integer
Þ f (a ) = a orf (0) = 0 function and { × } denotes the fractional part of x) in the
Given, f (h ) = h interval ( -2, 2 ) are
f (x + h) – f (x )
f ¢( x ) = lim
(a) 6 (b) 8
Then, for 0 < h < d
h ®0 h (c) 4 (d) 3
f ( x + f (h )) – f ( x ) Sol. (a) Here, f ( x ) = ([2x ] + {2x })([2x ] - {2x }) = 2x (2x - 2 {2x })
Þ f ¢( x ) = lim
h ®0 h = 4 x 2 - 4 x {2x } …(i)
[given f (h ) = h ]
We know, {2x } is discontinuous at integers.
f (x ) + h – f (x )
Þ f ¢( x ) = lim Here, -2 < x < 2
h ®0 h
Þ - 4 < 2x < 4
h
Þ f ¢( x ) = lim \ Integer values of 2x = { -3, - 2, - 1, 0, 1, 2, 3}, but
h ®0 h
f ( x ) = 4 x ( x - {2x }) is continuous at x = 0
Þ ¢
f (x ) = 1 \ Total number of points of discontinuity are 6.

Ex. 11 If the function of Ex. 14 If x Î R + and n Î N , we can uniquely write


l
l

é ( x - 5) 2 ù x = mn + r , where m ÎW and 0 £ r < n. We define x mod


f (x ) = ê ú sin( x - 5 ) + a cos ( x - 2 ), where [ × ]
ë A û n = r . e.g. 10.3 mod 3 = 1 .3 . The number of points of
denotes the greatest integer function, is continuous and discontinuity of the function, f ( x ) = ( x mod 2 ) 2 + ( x mod 4 )
differentiable in (7, 9 ), then in the interval 0 < x < 9 is
(a) A Î[8, 64 ] (b) A Î[0, 8 ) (a) 0 (b) 2
(c) A Î [16, ¥ ) (d) A Î[8, 16] (c) 4 (d) None of these
346 Textbook of Differential Calculus

Sol. (c) Here, f ( x ) = ( x mod 2)2 + ( x mod 4 ) æ 12 -9 ö æ 12 9 ö


(a) ç , ÷ (b) ç , ÷
è 25 25 ø è 25 25 ø
ì x 2 + x, 0 < x < 2
ï æ 16 -8 ö æ 24 -7 ö
ïï (x - 2) 2 + x , 2 £ x < 4 (c) ç , ÷ (d) ç , ÷
è 25 25 ø è 25 25 ø
= í( x - 4 )2 + ( x - 4 ), 4 £ x < 6
ï ( x - 6) 2 + ( x - 4 ), 6 £ x < 8 Sol. (d) We have,
ï
ïî ( x - 8)2 + ( x - 8), 8 £ x < 9 y × lim f (cos 3 x - cos 2 x ) = x × lim f (sin 2 x - sin 3 x )
x ®0 x ®0
- +
Clearly, f (2 ) = 6, f (2 ) = 2 Þ discontinuous at x = 2.
Y
f ( 4 - ) = 8, f ( 4 + ) = 0 Þ discontinuous at x = 4.
(0, 5)
f (6- ) = 6, f (6+ ) = 2 Þ discontinuous at x = 6.
(0, 4)
f (8- ) = 8, f (8+ ) = 0 Þ discontinuous x = 8.
(0, 3)
\ Number of points of discontinuity = 4.
X′ X
0
l Ex. 15 Let f : R ® R be a differentiable function at x = 0 Y′
satisfying f (0 ) = 0 and f ¢ (0 ) = 1 , then the value of
1 ¥ Here, cos 3 x - cos x ® 0 from LHS
æxö
lim × å ( -1) n × f ç ÷ , is
x ®0 x
n =1
ènø \ lim f (cos 3 x - cos x ) = 4
x ®0
(b) -log 2
sin 2 x - sin 3 x ® 0 from RHS
(a) 0
(c) 1 (d) e
f ( x /n ) f ( x / n ) - f ( 0) \ lim f (sin 2 x - sin 3 x ) ® 3
Sol. (b) Let L = lim = lim , x ®0
x ®0 x x ® 0 n × x /n
\ Equation of straight line is 4y = 3x …(i)
[using f (0) = 0]
Y y = 3x
1 f ( x / n ) - f ( 0) 
\ L= lim 4
n x ®0 x /n

Þ L=
1
× f ¢ ( 0) =
1
[as f ¢ (0) = 1]
(0, 1)
( 1225 , 259
n n
X′ X
1 ¥ æx ö ¥ 1 0
\ lim × å ( -1)n f ç ÷ = å ( -1)n ×
x ®0 x
n =1
è n ø n =1 n ( 24 , –7
25 25

1 1 1 1 1 Y′
= -1 + - + - + - ...
2 3 4 5 6 3
Equation of line perpendicular to y = x and passing
æ 1 1 1 1 1 ö 4
= - ç1 - + - + - +¼÷ = - log2 -4
è 2 3 4 5 6 ø through (0, 1), is (y - 1) = ( x - 0)
3
l Ex. 16 Let f ( x ) is a function continuous for all x Î R -4
Þ y -1 = x …(ii)
3
except at x = 0 such that f ¢ ( x ) < 0, "x Î ( - ¥, 0 ) and
On solving Eqs. (i) and (ii), we get
f ¢ ( x ) > 0, " x Î (0, ¥). If lim f ( x ) = 3, lim f ( x ) = 4 and 12 9
x ®0 + x ®0 - x = ,y =
f (0 ) = 5, then the image of the point (0, 1) about the line, 25 25
æ 24 -7 ö
y × lim f (cos 3 x - cos 2 x ) = x × lim f (sin 2 x - sin 3 x ), is Hence, image point is ç , ÷.
x ®0 x ®0 è 25 25 ø
Chap 06 Continuity and Differentiability 347

JEE Type Solved Examples :


More than One Correct Option Type Questions
l Ex. 17 If f ( x ) be such that f ( x ) = max (| 3 - x |, 3 - x 3 ), (c) f ( x ) is differentiable for all x Î R
then (d) f ( x ) is differentiable for all x Î R - { 2 }
(a) f ( x ) is continuous " x Î R
Sol. (a,d) Here, x = 2t – | t – 1| and y = 2t 2 + t | t |
(b) f ( x ) is differentiable " x Î R
Now, when t < 0,
(c) f ( x ) is non-differentiable at three points only
x = 2t – {– (t – 1)} = 3t – 1
(d) f ( x ) is non-differentiable at four points only
1
Sol. (a,d) From the graph of f ( x ), f ( x ) is continuous, " x Î R and y = 2t 2 – t 2 = t 2 Þ y = ( x + 1) 2
9
and f ( x ) is not differentiable at x = - 1, 0, 1, 3.
When 0 £ t < 1,
y = 3 – x3
x = 2t –(– (t – 1)) = 3t – 1
Y 1
and y = 2t 2 + t 2 = 3t 2 Þ y = ( x + 1) 2
3
When t ³ 1,
x = 2t – ( t – 1) = t + 1
y = |3 – x| and y = 2t 2 + t 2 = 3t 2 Þ y = 3 ( x – 1) 2
ì1
ï 9 ( x + 1) , x < –1
2
X′ X
–1 0 1 31/3 3 ï
ï1
l Ex. 18 Let f ( x ) = | x - 1 |([ x ] - [ -x ]), then which of the Thus, y = f ( x ) = í ( x + 1) 2 , – 1 £ x < 2
ï3
following statement(s) is/are correct. (where [ × ] denotes ï3 ( x – 1)2 , x ³2
greatest integer function.) ïî
(a) f ( x ) is continuous at x = 1 Now, to check continuity at x = –1 and 2.
(b) f ( x ) is derivable at x = 1
Continuity at x = –1 ,
(c) f ( x ) is non-derivable at x = 1
1
(d) f ( x ) is discontinuous at x = 1 LHL = lim f (–1 – h ) = lim (–1 – h + 1)2 = 0
h ®0 h ®0 9
Sol. (a, c) We have, f ( x ) = | x - 1| ([ x ] - [ - x ])
1
Here, [ x ] = 0 , when 0 < x < 1 RHL = lim f (–1 + h ) = lim (–1 + h + 1)2 = 0
h ®0 h ®0 3
[ - x ] = -1 , when 0 < x < 1
f (–1) = 0
[ x ] = 1 , when 1 < x < 2
\ f ( x ) is continuous at x = –1.
[ - x ] = -2 , when 1 < x < 2
Now, to check continuity at x =2 ,
[ x ] = 1 , [ - x ] = -1 , when x = 1
1
ì ( x - 1)(1 + 2), when 1 < x < 2 LHL = lim f (2 – h ) = lim (2 – h + 1)2 = 3
h ®0 h ®0 3
ï
\ f ( x ) = í 0 , when x = 1
RHL = lim f (2 + h ) = lim 3 (2 + h – 1)2 = 3
ï(1 - x ) , when 0 < x < 1 h ®0 h ®0
î
f ( 2) = 3
3(h ) - 0
Þ f ¢ (1+ ) = lim =3 Thus, f ( x ) is continuous at x = 2.
h ®0 h
(1 - (1 - h )) - 0 Now, to check differentiability at x = –1 and 2.
and f ¢ (1- ) = lim = -1 Differentiability at x = –1 ,
h ®0 -h
f (–1 – h ) – f (–1)
Þ f ( x ) is continuous at x = 1 and non-derivable at x = 1. LHD = Lf ¢ (–1) = lim
h ®0 –h
1
l Ex. 19 If y = f ( x ) defined parametrically by (–1 – h + 1) – 0
2

= lim 9 =0
x = 2 t – | t – 1 | and y = 2 t 2 + t | t | , then h ®0 –h
(a) f ( x ) is continuous for all x Î R f (–1 + h ) – f (–1)
RHD = Rf ¢ (–1) = lim
(b) f ( x ) is continuous for all x Î R - { 2 } h ®0 h
348 Textbook of Differential Calculus

1 4 2018

3
(–1 + h + 1)2 – 0 and ò-2010 f ( x )dx = - ò4 f ( x ) dx
= lim =0
h ®0 h Þ Option (b) is true.
Hence, f ( x ) is differentiable at x = –1. Also, D = ( f ¢ (10))2 + 4 f ¢ (10) > 0
Differentiability at x = 2 , As, f ¢ ( - 100) > 0 Þ f ¢ (10) ³ 0
f ( 2 – h ) – f ( 2)
LHD = Lf ¢ (2) = lim Þ x 2 - f ¢ (10)x - f ¢ (10) = 0 has real roots.
h ®0 –h
1 Þ Option (c) is false.
( 2 – h + 1) – 3
2
As, f ¢( 4 + x ) = f ¢( 4 - x )
= lim 3 =2 f ¢( x ) is symmetric about x = 4
h ®0 –h
f ( 2 + h ) – f ( 2) Þ f ¢ (10) = f ¢ ( - 2) = 20
RHD = Rf ¢ (2) = lim Þ Option (d) is true.
h ®0 h
3( 2 + h – 1) 2 – 3 Ex. 22 Let f be a real valued function defined on the
= lim =6 l
h ®0 h x
interval (0, ¥), by f ( x ) = ln x + ò 1 + sin t dt . Then, which
Hence, f ( x ) is not differentiable at x = 2. 0
\ f ( x ) is continuous for all x and differentiable for all x, of the following statement(s) is/are true? [IIT JEE 2010]
except x = 2. (a) f ¢ ¢ ( x) exists for all x Î (0, ¥)
(b) f ¢ ( x) exists for all x Î (0, ¥) and f ¢ is continuous on
l Ex. 20 f ( x ) = sin -1 [e x ] + sin -1 [e - x ], where [ × ] (0, ¥) but not differentiable on (0, ¥)
greatest integer function, then (c) There exists a > 1such that| f ¢ ( x)| < | f ( x)| for all
(a) domain of f ( x ) = ( - ln 2, ln 2) x Î ( µ, ¥)
(b) range of f ( x ) = { p } (d) There exists b > 0 such that| f ( x)| + | f ¢ ( x)| £ b for all
x Î (0, ¥)
(c) f ( x ) has removable discontinuity at x = 0
1
(d) f ( x ) = cos -1 x has only solution Sol. (b, c) Here, f ¢ ( x ) = + 1 + sin x , x > 0 but f ( x ) is not
x
Sol. (a, c) 0 < e x < 2 and 0 < e - x < 2 differentiable in (0, ¥) as sin x may be - 1 and then
1 1 cos x
Þ - ¥ < x < loge 2 and - ¥ < - x < loge 2 f ¢ ¢( x ) = - 2 + will not exist.
x 2 1 + sin x
Þ ( - ¥ < x < loge 2)
Þ f ¢( x ) is continuous for all x Î (0, ¥ ) but f ¢( x ) is not
and ( - loge 2 < x < ¥ ) differentiable on (0, ¥ ).
Þ - loge 2 < x < loge 2 \ Option (b) is true.
ìï p, x =0 Also, f ¢ ( x ) £ 3, if x > 1
\ f (x ) = í p
, x Î ( - loge 2, 0) È (0, loge 2) and f ( x ) > 3, if x > e 3
ïî 2
\ Let a = e3
l Ex. 21 f : R ® R is one-one, onto and differentiable Þ Option (c) is true.
function and graph of y = f ( x ) is symmetrical about the (d) is not possible, as f ( x ) ® ¥ when x ® ¥.
point (4, 0), then æ x +y ö
-1 -1
l Ex. 23 If f ( x ) + f (y ) = f ç ÷ for all x , y Î R
(a) f ( 2010) + f ( - 2010) = 8 è 1 – xy ø
(b) ò
2018
f ( x ) dx = 0 ( xy ¹1) and lim
f (x )
= 2, then
-2010
x ®0 x
(c) if f ¢ ( - 100) > 0, then roots of
x 2 - f ¢ (10) x - f ¢ (10) = 0 may be non-real æ 1 ö p æ 1 ö p
(a) f ç ÷ = (b) f ç ÷ = -
è 3ø 3 è 3ø
(d) if f ¢ (10) = 20, then f ¢ ( - 2) = 20
3
(c) f ¢ (1) = 1 (d) f ¢ (1) = -1
Sol.(a, b, d) Graph is symmetrical about (4, 0).
Þ f (4 + x ) = - f (4 - x ) æx +yö
Sol. (a, c) f ( x ) + f (y ) = f ç ÷ ...(i)
Þ f ( x ) = - f (8 - x ) …(i) è 1 – xy ø
Now, let f ( x ) = 2010, then f (8 - x ) = - 2010 On putting x = y = 0, we get f (0) = 0
-1 -1
Þ f (2010) + f ( -2010) = 8 On putting y = – x , we get f ( x ) + f (–x ) = f (0)
Þ Option (a) is true. Þ f (–x ) = – f ( x ) ...(ii)
Chap 06 Continuity and Differentiability 349

f (x ) f ( 0 + h ) – f ( 0) f (h )
Also, lim =2 Also, f ¢(0) = lim = lim ...(iv)
x ®0 x h ®0 h h ®0 h

f (x + h) – f (x ) f (0 + (h 1/ 3 )3 ) – f (0)
Now, f ¢( x ) = lim …(iii) Let I = f ¢(0) = lim
h ®0 h h ®0 (h 1/ 3 )3
f ( x + h ) + f (–x )
= lim ( f (h 1/ 3 ))3
3
æ f (h 1/ 3 ) ö
h ®0 h = lim = lim ç 1/ 3 ÷ = I 3
[using Eq. (ii), – f ( x ) = f (–x )] h ®0 (h 1/ 3 )3 h ® 0 è (h ) ø
æ x +h–x ö Þ I = I 3 or I = 0, 1, – 1 as f ¢(0) ³ 0
f ç ÷
è 1 – ( x + h ) (–x ) ø
f ¢( x ) = lim [using Eq. (i)] \ f ¢(0) = 0, 1 ...(v)
h ®0 h
f (x + h) – f (x )
é æ h öù Thus, f ¢( x ) = lim
h ®0 h
êf ç ÷ú
è1 + x (x +h )ø
Þ f ¢( x ) = lim ê ú f ( x + (h 1/ 3 )3 ) – f ( x )
h ®0 ê h ú = lim
ê ú
h ®0 (h 1/ 3 )3
ë û
f ( x ) + ( f (h 1/ 3 ))3 – f ( x )
æ h ö f ¢( x ) = lim [using Eq. (i)]
f ç ÷ h ®0 (h 1/ 3 )3
è 1 + xh + x 2 ø æ 1 ö
Þ ¢
f ( x ) = lim ´ç 2÷
3
h ®0 æ ö è 1 + xh + x ø æ f (h 1/ 3 ) ö
ç
h
÷ Þ f ¢( x ) = lim ç 1/ 3 ÷ = ( f ¢ (0))3
h ® 0 è (h ) ø
è 1 + xh + x 2 ø
æ h ö Þ f ¢( x ) = 0 or 1 [as f ¢ (0) = 0 or 1, using Eq. (v)]
f ç ÷
è 1 + xh + x 2 ø 1 On integrating both sides, we get
Þ f ¢( x ) = lim ´ lim f ( x ) = C or x + C as f (0) = 0
h ®0 æ h ö h ®0 1 + xh + x 2
ç 2÷ Þ f ( x ) = 0 or x
è 1 + xh + x ø
Thus, f (10) = 0 or 10
é f (x ) ù
êë using xlim = 2ú
®0 x û l Ex. 25 Let f ( x ) = x 3 – x 2 + x + 1
Þ f ¢( x ) = 2 ´
1
Þ f ¢( x ) =
2 ìmax f (t ), 0 £ t £ x for 0 £ x £ 1
1 + x2 1 + x2 and g ( x ) = í
î3 – x , 1< x £ 2
On integrating both sides, we get Then, g ( x ) in [0, 2] is
f ( x ) = 2 tan –1( x ) + C (a) continuous for x Î [0, 2] - { 1}
Where f ( 0) = 0 Þ C = 0 (b) continuous for x Î[0, 2]
Thus, f ( x ) = 2 tan –1 x (c) differentiable for all x Î[0, 2]
æ 1 ö æ 1 ö p p (d) differentiable for all x Î [0, 2] - { 1 }
Hence, f ç ÷ = 2 tan –1 ç ÷ = 2 × =
è 3ø è 3ø 6 3 Sol. (b, d) Here, f ( x ) = x 3 – x 2 + x + 1

and f ¢(1) =
2
=
2
=1 Þ f ¢( x ) = 3x 2 – 2x + 1, which is strictly increasing in (0, 2).
1+1 2
2
ì f ( x ), 0 £ x £ 1
\ g (x ) = í
l Ex. 24 Let f : R ® R is a function which satisfies condi- î3 – x , 1 < x £ 2

tion f ( x + y 3 ) = f ( x ) + [ f (y )]3 for all x , y Î R. If f ¢(0 ) ³ 0, [as f ( x ) is increasing, so f ( x ) is


maximum when 0 £ t £ x]
then ì x 3 – x 2 + x + 1, 0 £ x £ 1
(a) f ( x ) = 0 only (b) f ( x ) = x only So, g (x ) = í
î3 – x , 1< x £2
(c) f ( x ) = 0 or x only (d) f (10) = 10
ì3x 2 – 2x + 1, 0 £ x £ 1
Sol. (c, d) Given, f ( x + y 3 ) = f ( x ) + [ f (y )]3 ...(i) Also, g ¢( x ) = í
î– 1, 1< x £2
and f ¢(0) ³ 0 ...(ii)
On replacing x , y by 0, Which clearly shows g ( x ) is continuous for all x Î[0, 2] but
g ( x ) is not differentiable at x = 1.
f ( 0) = f ( 0) + f ( 0) 3 Þ f ( 0) = 0 ...(iii)
350 Textbook of Differential Calculus

JEE Type Solved Examples :


Passage Based Questions
Passage I 1
Sol. (d) Here, f (x ) =
(Ex. Nos. 26 to 27) 1- x
In the given figure graph of Þ x ¹1
1
y = P ( x ) = ax 5 + bx 4 + cx 3 + dx 2 + ex + f , is given. f ( f ( x )) =
1 - f (x )
Y
(–2, 2) 1 x -1
= =
(0, 1) 1 x
1-
1- x
Þ x ¹ 0, 1
X′ X 1
0 1
–3 –2 –1 1 2 -1
2 f (x ) - 1 1 - x
and f ( f ( f ( x ))) = = = x , where x ¹ 0, 1.
f (x ) 1
x -1
Y′ Here, point of discontinuity are x = 0, 1.
26. If P ¢¢ ( x ) has real roots a, b, g, then [a ] + [b] + [ g ], is 29. The domain of f ( g ( x )), is
(a) -2 (b) -3 (c) -1 (d) 0 (a) x Î R
27. The minimum number of real roots of equation (b) x Î R - {1}
(c) x Î R - {0, 1}
(P ¢¢ ( x )) 2 + P ¢ ( x ) × P ¢¢¢( x ) = 0, is (d) x Î R - {0, 1, -1}
(a) 5 (b) 7 (c) 6 (d) 4 Sol. (c) Here, g ( x ) = f ( f ( f ( x ))) = x , x Î R - {0, 1}
ì 1ü 1
Sol. (Ex. Nos. 26 to 27) Here, P ¢ ( x ) = 0 at x = í -2, - 1, 0, ý f ( g ( x )) = f ( x ) =
î 2þ 1- x
æ 1ö Þ x ¹ 0, 1
Þ P ¢¢( x ) will have real roots Î ( -2, - 1 ), ( -1, 0), ç0, ÷ .
è 2ø \ Domain of f ( g ( x )) Î R - {0, 1}
Þ [a ] + [b ] + [ g ] = -2 - 1 + 0 = -3 30. If point P (a , a 2 ) lies on the same side as that of (a, b)
Also, let f ( x ) = ( P ¢¢ ( x ))2 + P ¢( x ) × P ¢¢ ¢ ( x ) with respect to line x + 2y - 3 = 0, then
d æ 3 ö
\ f (x ) = ( P ¢( x ) × P ¢¢( x )) (a) a Î ç - , 1÷ (b) a Î R
dx è 2 ø
Since, P ¢( x ) has 4 real roots. æ 3 ö
(c) a Î ç - , 0÷ (d) a Î(0, 1)
Þ P ¢ ¢( x ) has 3 real roots. è 2 ø
Þ f ( x ) has 6 real roots. æ 3 ö
Sol. (a) From graph, a Î ç - , 1÷ , as (a , b ) = (0, 1).
26. (b) 27. (c) è 2 ø
y = x2
Passage II Y

(Ex. Nos. 28 to 30)


x+2
y – 3=
If a, b (where a < b) are the points of discontinuity of the (0, 3/2)
0
1
function g ( x ) = f ( f ( f ( x ))), where f ( x ) = and P (a , a 2 ) (0, 1)
1- x (a , a 2 )
X′ X
is any point on XY -plane. Then, –3 –1 0 1
2
28. The points of discontinuity of g ( x ) is
Y′
(a) x = 0, - 1 (b) x = 1only (c) x = 0 only (d) x = 0, 1
Chap 06 Continuity and Differentiability 351

JEE Type Solved Examples :


Matching Type Questions
ì[ x ], - 2 £ x < 0 ì ì pü
l Ex. 31 Let f ( x ) = í ïsec ( -2), x Î [ - 2, - 1) - í - 2 ý
î| x |, 0 £ x £ 2 ï î þ
gof = í sec ( -1), x Î [ -1, 0)
(where [ × ] denotes the greatest integer function)
ï ì pü
g ( x ) = sec x , x Î R - ( 2n + 1) p / 2. ï sec x , x Î [0, 2] - í 2 ý
Match the following statements in Column I with their î î þ
Limit of gof doesn’t exist at x = - 1.
æ 3p 3p ö
values in Column II in the interval ç - , ÷ .
è 2 2 ø l Ex. 32 Match the functions in Column I with the
Column I Column II properties Column II.
(A) Limit of fog exists at (p) -1 Column I Column II
(B) Limit of gof doesn’t exist at (q) p
(A) g : R ® Q (Rational number), f : R ® Q (p) 1
(C) Points of discontinuity of fog is/are (r) 5p
(Rational number); f and g are
6
continuous functions such that
(D) Points of differentiability of fog is/are (s) -p 3 f (x ) + g (x ) = 3, then
Sol. (A) ® (p, q, s), (B) ® (p), (C) ® (q, s), (D) ® (p, r) (1 - f (x ))3 + (g (x ) - 3)3 is
ì -2, - 2 £ x < - 1
ì[ x ], - 2 £ x < 0 ï (B) If f (x ), g (x ) and h(x ) are continuous and (q) 0
f (x ) = í Þ f ( x ) = í -1, - 1 £ x < 0 positive functions such that
î | x |, 0 £ x £ 2 ï x, 0 £ x £ 2
î f (x ) + g (x ) + h (x ) = f (x )g (x )
p + g (x )h(x ) + h(x ) f (x ),
g ( x ) = sec x , x Î R - (2n + 1)
2 then f (x ) + g (x ) - 2h(x ) is
ì -2, - 2 £ sec x < - 1 (C) y = f (x ) satisfies the equation (r) 2
ï
Þ fog ( x ) = í -1, - 1 £ sec x < 0 y3 - 2 y2 (x + 1) + 4 xy + (x 2 - 1)
ïsec x , 0 £ sec x £ 2 ( y - 2) = 0, then y¢ (1) + y (1) would be
î
equal to
Y
(D) If y = f (x ) satisfies (xf (x ))99 + (xf (x ))98 (s) 3
+ ... + (xf (x )) + 1 = 0, then (1 + f (1)) is
–3π/2 –π –π/2 1 π/2 π 3π/2 (t) -1
X
–1 O –1 Sol. (A) ® (p), (B) ® (q), (C) ® (p, r,s), (D) ® (q)
–2 –2
(A) On comparing, f ( x ) = 0, g ( x ) = 3
\ (1 - f ( x ))3 + ( g ( x ) - 3)3 = 1

ì (B) Here, f ( x ) + g ( x ) + h ( x )
é 4 p 2p ù é 2p 4 p ù
ï -2, x Î ê - 3 , - 3 ú È ê 3 , 3 ú - { - p, p } = f ( x )× g ( x ) + g ( x ) × h ( x ) + h ( x ) × f ( x )
ï ë û ë û
\ fog = í -1, x = - p, p 1
ïsec x , Þ {( f ( x ) - g ( x ))2 + ( g ( x ) - h ( x ))2
é p pù
ï x Î ê- , ú 2
î ë 3 3û + ( h( x ) - f ( x ))2 } = 0
Limit of fog exist at x = - p, p, - 1 points of discontinuity of
5p Þ f ( x ) = g ( x ) = h( x )
fog are - p, p and points of differentiability of fog are - 1, .
6 \ f ( x ) + g ( x ) - 2h ( x ) = 0
352 Textbook of Differential Calculus

(C) y 3 - 2y 2 ( x + 1) + 4 xy + ( x 2 - 1) (y - 2) = 0 Sol. (A) ® (q), (B) ® (q), (C) ® (s), D ® (s)


Put x = y = 0 Þ f ( 0) = 0
Put y = 2 Þ 8 - 8 ( x + 1) + 8x = 0
f (x + h) - f (x )
\ y = 2 is solution. Now, f ¢ ( x ) = lim
h ®0 h
Put y = ( x + 1) f ( x ) + f (h )
- f (x )
Þ ( x + 1) 3 - 2 ( x + 1) 3 + 4 x ( x + 1) + ( x 2 - 1) ( x - 1) 1 + f ( x ) f (h )
= lim
= - ( x + 1) 3 + 4 x ( x + 1) + ( x + 1) ( x - 1) 2 h ®0 h
= - ( x + 1) 3 + ( x + 1) { 4 x + ( x - 1) 2 } f (h ) [1 - { f ( x )} 2 ]
= lim
h ®0 h [1 + f ( x ) f (h )]
= - ( x + 1) 3 + ( x + 1) 3 = 0
ì f (h ) - f (0)ü é 1 - { f ( x )} 2 ù
\ y = ( x + 1) is the solution. = lim í ýê ú
h ® 0î h - 0 þ ë 1 + f ( x ) f (h ) û
Similarly, y = ( x - 1) is the solution
Þ y = 2, x + 1, x - 1 = f ¢ (0) [1 - { f ( x )} 2 ] = 1 - { f 2 ( x )}
dy \ f ¢ ( x ) = 1 - { f 2 ( x )}
\ = 0, 1 ...(i)
dx
On integrating both sides, we get
ì y ¢ (1) + y (1) = 0 + 2 = 2, when y = 2
ï 1 é1 + f (x )ù
Þ íy ¢ (1) + y (1) = 1 + 2 = 3, when y = x + 1 ln = x +c
2 êë 1 - f ( x ) úû
ïy ¢ (1) + y (1) = 1 + 0 = 1, when y = x - 1
î
1 + f (x )
( xf ( x ))100 - 1 or = ke 2 x
(D) = 0 Þ xf ( x ) = - 1 1 - f (x )
xf ( x ) - 1
Now, f ( 0) = 0 Þ k = 1
As, xf ( x ) ¹ 1 e 2x - 1 e x - e -x
\ xf ( x ) = - 1 \ f (x ) = =
e 2x
+1 e x + e -x
Þ f (1) = - 1 or 1 + f (1) = 0
Clearly, f ( x ) is differentiable for all x Î R and from Eq. (i ),
l Ex. 33 Suppose a function f ( x ) satisfies the following f ¢ ( x ) > 0 for all x Î R. Again, f ( x ) is an odd function,
f ( x ) = 0 Þ x = 0.
conditions x
f ( x ) + f (y ) æe x - e -x ö
f (x + y ) = , " x , y and f ¢ (0 ) = 1 . Also, Now, lim [ f ( x )] = lim ç x
x
÷
1 + f ( x ) f (y ) x ®¥ x ® ¥ èe + e -x ø

- 1 < f ( x ) < 1 , " x Î R. æ e x -e - x ö


lim ç -1÷÷ x
æ xe - x
-2 lim çç
ö
÷
Match the entries of the following two columns. x ® ¥ çè e x +e - x ø x ® ¥ è e x +e - x ÷
ø
=e =e
Column I Column II æ x ö
- 2 lim ç ÷
x ® ¥ è e 2 x +1 ø
(A) f (x ) is differentiable over the set (p) R - (-1, 0, 1) =e
(B) f (x ) increases in the interval (q) R æ 1 ö
- 2 lim ç ÷
x ® ¥ è 2e 2 x ø
(C) Number of the solutions of f (x ) = 0 is (r) 0 =e =e0 =1
(D) The value of the limit lim [ f (x )]x is (s) 1
x®¥
Chap 06 Continuity and Differentiability 353

JEE Type Solved Examples :


Single Integer Answer Type Questions
ì1 – cos 4 x
1 + x2 1 + x2
ï , x <0
ï x
2 Y
l Ex. 34 Let f ( x ) =
ï 4
í a, x =0
ï x
ï , x >0 3
ïî 16 + x – 4
2
Then, the value of a if possible, so that the function is
continuous at x = 0 , is ..... . 1
Sol. (8) As, f ( x ) is continuous at x = 0.
\ We must have, X′
0 √3
X
–√3 –1 1
RHL (at x = 0) = LHL (at x = 0) = f (0)
–1
x
RHL (at x = 0) = lim f ( x ) = lim Y′
x ® 0+ x ® 0+ 16 + x – 4
Or
0+h 16 + h + 4
= lim ´ 2
x +1
Y x2 + 1
h ®0 16 + 0 + h – 4 16 + h + 4
4
[putting x = 0 + h ]
h { 16 + h + 4 } 3
= lim
h ®0 16 + h – 16
2
= lim { 16 + h + 4 } = 8
h ®0
1
1 – cos 4 x
Also, LHL (at x = 0) = lim f ( x ) = lim
x ® 0– x ® 0– x2 X′
0
X
1 – cos 4 (0–h ) –√3 √3
= lim [putting x = 0 - h ] Y′
h ®0 ( 0 –h ) 2
Thus, from above graph, we can simply say
1 – cos 4 h
= lim f ( x ) is not differentiable at x = ± 3.
h ®0 h2
\ Not differentiable at 2 points.
2 sin 2 2h
= lim
h ®0 h2 l Ex. 36 Let f ( x ) = x n , n being non-negative integer. Then,
2
æ sin 2h ö
= lim 8 ç ÷ =8 the number of value of n for which the equality
h ® 0 è 2h ø
f ¢(a + b ) = f ¢(a ) + f ¢(b ) is valid for all a , b > 0, is
And f ( 0) = a
Sol. (2) Since, f ( x ) = x n , n being non-negative integer.
Since, f ( x ) is continuous at x =0
Then, f ¢( x ) = nx n – 1
Þ f (0) = RHL = LHL
or f (0) = 8 or a = 8 f ¢(a ) = nan – 1, f ¢(b ) = nbn – 1,
f ¢(a + b ) = n (a + b )n – 1
l Ex. 35 Let f ( x ) = maximum { 4, 1 + x , x – 1 } , " x ÎR.
2 2
Now, the equality f ¢(a + b ) = f ¢(a ) + f ¢(b ) holds, if
Then, the total number of points, where f ( x ) is not n (a + b )n – 1 = nan – 1 + nbn – 1
differentiable, …… . or (a + b )n – 1 = an – 1 + bn – 1 ...(i)
Sol. (2) We have, discussed in the last chapter for sketching Above statement is true, only if (n – 1) = 1 Þ n =2
maximum { 4, 1 + x 2 , x 2 – 1} as
i.e. (a + b )n – 1 = an – 1 + bn – 1 [if n = 2]
354 Textbook of Differential Calculus

or (a + b )1 = a1 + b1 | x ||sin x |
Thus, | x f ( x )| +|| x - 2| - 1| = + || x - 2 | -1| is not
Also, when n = 1, then LHS < RHS |x |
differentiable at { p, 2p, 0, 1, 3}.
and when n = 3, 4, 5, ¼, then LHS > RHS
\ Number of points are 5.
Again, when n = 0, f ¢( x ) = 0 for all x.
So, the equality is true for n = 0 and 2. æ xy ö f ( x ) × f (y )
\ Number of values is 2.
l Ex. 39 If f ç ÷ = ; x , y Î R, f (1) = f ¢ (1).
è 2 ø 2
l Ex. 37 Find the number of points where f (3 )
Then, is …… .
f ( x ) = [sin x + cos x ] (where [ × ] denotes greatest integral f ¢ (3 )
function), x Î[0, 2p ] is not continuous, is/are…… . f (x + h) - f (x )
Sol. (3) Here, f ¢ ( x ) = lim
h ®0 h
Sol. (5) We know [ × ] is not continuous at integral points.
Thus, f ( x ) = [sin x + cos x ] will be discontinuous at those æ 2x + 2h ö
fç ÷ - f (x )
points, where sin x + cos x is an integer, i.e. è 2 ø
= lim
h ®0
ì p 3p 3p 7 p ü h
x Î í , , p, , ý
î 2 4 2 4þ æ 2x (1 + h / x ) ö æ 2x × 1 ö
fç ÷-f ç ÷
Thus, the number of points at which f ( x ) is discontinuous è 2 ø è 2 ø
= lim
is 5. h ®0 h
f ( 2x ) × f ( 1 + h / x ) f ( 2x ) × f ( 1)
l Ex. 38 The number of points where | xf ( x )| + | |x - 2 |-1 || -
= lim 2 2
is non-differentiable in x Î(0, 3 p ), where h ®0 h
æ æ 2x ö ö f ( 2x ) f (1 + h / x ) - f (1)
¥ ç
1 + 2 cos ç ÷ ÷ = × lim
è 3k ø ÷ 2x ® 0 h /x
f (x ) = Õ ç
h
, is …… .
ç ÷ f ( 2x )
k =1
ç
3
÷ = × f ¢ ( 1)
è ø 2x
f ( 2x )
Sol. (5) We know, = × f (1), given f (1) = f ¢ (1)
2x
sin 3q = 3sin q - 4 sin 3 q = sin q (1 + 2 cos 2q )
æ 2x × 1 ö
fç ÷
\ sin 3q = sin q (1 + 2 cos 2q ) …(i) è 2 ø æ xy ö f ( x ) × f (y )
= , using f ç ÷ =
x x x è 2 ø 2
Now, on putting 3q = x , , 2 , … one by one in Eq. (i), we
3 3 f (x )
get \ f ¢( x ) =
x
æx ö æ 2x ö
Þ sin x = sin ç ÷ × ç1 + 2 cos ÷ Þ
f (x )
=x Þ
f ( 3)
=3
è3ø è 3 ø f ¢( x ) f ¢ ( 3)
x æxö æ 2x ö
and sin = sin ç 2 ÷ × ç1 + 2 cos 2 ÷
3 è 3 ø è 3 ø l Ex. 40 Let f : R ® R be a differentiable function satisfy-
x æxö æ 2x ö ing f ( x ) = f (y ) f ( x - y ), "x , y Î R and f ¢ (0 ) = ò { 2 x } dx ,
4
\ sin 2 = sin ç 3 ÷ × ç1 + 2 cos 3 ÷ … and so on
3 è3 ø è 3 ø 0
where { × } denotes the fractional part function and
æxö n æ 2x ö
Þ sin x = sin ç n ÷ × Õ ç1 + 2 cos k ÷ f ¢ ( -3 ) = ae b . Then, | a + b | is equal to ……… .
è 3 ø k =1 è 3 ø
Sol. (4) Given, f ( x ) = f (y ) f ( x - y )
æ æ 2x ö ö
ç1 + 2 cos ç k ÷ ÷ On replacing x by ( x + y ),
sin x sin x /3n n è è 3 øø
Þ
x
= lim
n ®¥ n Õ
x / 3 k =1 3n
f ( x + y ) = f (y ) × f ( x )
Þ f ( x ) = e kx Þ f ¢ ( x ) = ke kx
æ æ 2x ö ö 4
sin x ¥ ç 1 + 2 cos ç k ÷÷
è3 ø
But, f ¢ ( 0) = ò0 {2x }dx = 2
Þ
x
= Õ çç 3
÷ = f (x )
÷
…(ii)
Þ f ¢ (0) = k = 2 Þ f ¢ ( x ) = 2e 2 x
k =1
ç ÷
è ø Þ f ¢ ( - 3) = 2e -6 Þ | a + b | = 4
Chap 06 Continuity and Differentiability 355

l Ex. 41 Let f ( x ) is a polynomial function and So, for x Î[0, 25]


f ( x ) = 101 at x = 0, 6, 12, 18, 24
f (x ) é f ¢ (x )ù
( f (a )) 2 + ( f ¢ (a )) 2 = 0, then find lim ú , where Total numbers = 5
x ® a f ¢ (x ) ê
ë f (x ) û Since, f (2 - x ) = f (2 + x )
[ × ] denotes greatest integer function, is ……. Þ f ( x ) is symmetric about x = 2 line.
Sol. (1) Here, ( f (a ))2 + ( f ¢ (a ))2 = 0 Þ f (a ) = f ¢ (a ) = 0 Due to symmetry in one period length.
\ a is repeated root of f ( x ). f ( x ) = 101 has one solution at x = 4 other than 0 and 6.
f ( x ) é f ¢( x ) ù f ( x ) æ f ¢ ( x ) ì f ¢ ( x )ü ö Now, f ( x ) = 101 at x = 4, 10, 16, 22
Now, lim = lim ç -í ý÷
x ®a f ¢ ( x ) êë f ( x ) úû x ® a f ¢ ( x ) è f ( x ) î f ( x ) þ ø Total numbers = 4
Hence, atleast minimum possible number of values of x = 9.
æ f ( x ) ì f ¢ ( x )ü ö
= lim ç1 - ×í ý÷
x ® aè f ¢( x ) î f ( x ) þø l Ex. 43 If f ( x ) is a differentiable function for all x Î R
such that f ( x ) has fundamental period 2. f ( x ) = 0 has
é as , x = a is the repeated root and ì f ( x ) ü is bounded;
ëê
í ý exactly two solutions in [0, 2], also f (0 ) ¹ 0. If minimum
î f ¢ ( x )þ
number of zeros of h( x ) = f ¢ ( x ) cos x - f ( x ) sin x in (0, 99 )
f (x ) ù is 120 + k, then k is …… .
\ lim = 0ú
x ®a f ¢( x ) û d
Sol. (7) h ( x ) = ( f ( x ) × cos x )
= 1-0 = 1 dx
First find the minimum number of zeros of
l Ex. 42 Let f : R ® R is a function satisfying ( f ( x ) × cos x ) = 0.
f (10 - x ) = f ( x ) and f ( 2 - x ) = f ( 2 + x ), " x Î R. If f ( x ) = 0 has minimum 98 roots in [0, 99) and cos x = 0 has
f (0 ) = 101. Then, the minimum possible number of values of 31 roots is [0, 99).
x satisfying f ( x ) = 101 , x Î[0, 25 ] is ……… . Maximum common possible root is only 1.
Hence, minimum number of roots of f ( x ) cos x = 0 is 128.
Sol. (9) Since, f (10 - x ) = f ( x ) = f ( 4 - x )
d
Þ f (10 - x ) = f ( 4 - x ) Thus, ( f ( x ) cos x ) = 0 has minimum 127 roots.
dx
Say, 4 - x = t Þ f (6 + t ) = f (t ) 127 = 120 + 7
Þ f ( x ) is periodic function with period 6. Hence, value of k is 7.

Subjective Type Questions


l Ex. 44 Let f ( x ) = max { 2 sin x , 1 – cos x }, " x Î(0, p), or 4 (1 + cos x ) = (1 – cos x )
then discuss differentiability of f ( x ) in (0, p ). or 4 + 4 cos x = 1 – cos x
Sol. We know that, f ( x ) = maximum {2 sin x , 1 – cos x } can be or cos x = – 3/ 5
plotted as Þ x = cos –1(– 3/ 5)
Y
\ f ( x ) is not differentiable at
1– cos x
2
2 sin x x = p – cos –1(3/ 5).

1 l Ex. 45 Discuss the continuity of the function


g ( x ) = [ x ] + [– x ] at integral values of x.
π π X Sol. Here, x can assume two values
0 –
2 –3 (a) integers (b) non-integers
π –cos–1 5
(a) If x is an integer
Thus, f ( x ) = maximum {2 sin x , 1 – cos x } is not [ x ] = x and [– x ] = – x
differentiable. Þ g (x ) = x – x = 0
When, 2 sin x = 1 – cos x (b) If x is not an integer
or 4 sin 2 x = (1 – cos x )2 Let x = n + f , where n is an integer and f Î(0, 1)
356 Textbook of Differential Calculus

Þ [ x ] = [n + f ] = n l Ex. 47 Let f : R ® R satisfying | f ( x )| £ x 2 , " x Î R, then


Þ [–x ] = [–n – f ] = [(–n – 1) + (1 – f )] = (–n – 1) show that f ( x ) is differentiable at x = 0.
[Q0 < f < 1 Þ (1 – f ) < 1] Sol. Since, | f ( x )| £ x 2 , " x Î R
Hence, g ( x ) = [ x ] + [–x ] = n + (–n – 1) = – 1
\ At x = 0, | f (0)| £ 0
ì 0, if x is an integer
\ g(x ) = í Þ f ( 0) = 0 ...(i)
î - 1, if x is not an integer f ( h ) – f ( 0) f (h )
\ f ¢(0) = lim = lim …(ii)
Let us discuss the continuity of g ( x ) at a point x = a h ®0 h h ®0 h
[where a Î integer] [f (0) = 0, from Eq. ( i) ]
LHL = lim g ( x ) = – 1 ½ f ( h )
½ £ |h |
x ® a– Now,
[Q as x ® a –, x is not an integer] ½ h ½
RHL = lim g ( x ) = – 1 f (h )
Þ – |h | £ £ |h |
x ® a+ h
[Q as x ® a + , x is not an integer] f (h )
Þ lim ®0
But g (a ) = 0 because a is an integer. h ®0 h

Hence, g ( x ) has a removable discontinuity at [using Cauchy-Squeeze theorem] …(iii)


integral values of x. \ From Eqs. (ii) and (iii), we get f ¢ (0) = 0
sin 2 x + A sin x + B cos x i.e. f ( x ) is differentiable at x = 0
l Ex. 46 If f ( x ) = is continuous
x3 l Ex. 48 Show that the function defined by
at x = 0. Find the values of A and B. Also, find f (0 ). ìx 2 sin 1/x , x ¹ 0
Sol. As, f ( x ) is continuous at x = 0. f (x ) = í
î0 , x =0
f (0) = lim f ( x ) and both f (0) and lim f ( x ) are finite.
x ®0 x ®0 is differentiable for every value of x, but the derivative is not
sin 2x + A sin x + B cos x continuous for x = 0.
Þ f (0) = lim
x ®0 x3 Sol. For x ¹ 0,
As, denominator ® 0 as x ® 0, æ 1 ö æ1ö
f ¢( x ) = 2x sin (1/x ) + x 2 ç– 2 ÷ cos ç ÷
Numerator should also ® 0 as x ® 0 è x ø èx ø
1 1
Which is possible only if ( for f (0) to be finite). f ¢( x ) = 2x sin – cos
Þ sin 2(0) + A sin (0) + B cos (0) = 0 x x
Þ B=0 For x = 0,
f ( 0 + h ) – f ( 0)
sin 2x + A sin x f ¢( x ) = lim
\ f (0) = lim h ®0 h
x ®0 x3
1
æ sin x ö æ 2 cos x + A ö h 2 sin – 0
Þ f (0) = lim ç ÷ç ÷ = lim h
x ®0 è x øè x2 ø h ®0 h
æ 2 cos x + A ö 1
= lim ç ÷ Þ f ¢( x ) = lim h sin = 0
x ®0 è x2 ø h ®0 h
Again, we can see that denominator ® 0 as x ® 0. ì 1 1
ï2x sin –cos , x ¹ 0
\ Numerator should also approach to 0 as x ® 0 Thus, f ¢ ( x ) = í x x
[Q for f (0) to be finite] ïî 0, x =0
Þ 2+ A =0 Now, f ¢( x ) is continuous at x = 0, if
Þ A = –2
æ 2 cos x –2 ö æ – 4 sin 2 x/ 2 ö (i) lim f ¢( x ) exists (ii) lim f ¢( x ) = f ¢(0)
Þ f (0) = lim ç ÷ = lim ç ÷ x ®0 x ®0
x ®0 è x 2 ø x ®0 è x2 ø æ 1 1ö
Again, lim f ¢( x ) = lim ç2x sin – cos ÷, doesn’t exist
æ –sin 2 x/ 2 ö x ®0 x ®0è x xø
= lim – ç ÷ =–1
x ® 0 è x 2/ 4 ø 1
Since, lim cos doesn’t exist.
So, we get A = – 2, B = 0 x ®0 x
and f ( 0) = – 1 Hence, f ¢( x ) is not continuous at x = 0.
Chap 06 Continuity and Differentiability 357

ìx – [ x ], x Ï I and thus g ( x ) Ï I , should be neglected.


l Ex. 49 If f ( x ) = í
î 1, x ÎI Þ g { g ( x )} = 0, x Î R and could be plotted as,
where I is an integer and [ × ] represents the greatest integer Y
function and
{ f ( x )} 2n – 1 g{g(x)} = 0, x ∈ R
g ( x ) = lim , then
n ® ¥ { f ( x )} 2n + 1 X
O

(a) Draw graphs of f ( 2x ), g ( x ) and g { g ( x )}.


(b) From the above three graphs, we have
(b) Find the domain and range of these functions.
Domain of f (2x ) Î R
(c) Are these functions periodic? If yes, find their
periods. and Range of f (2x ) Î (0,1]
ì x – [ x ], x Ï I Domain of g ( x )Î R
Sol. Here, f (x ) = í and Range of g ( x ) Î { 0, – 1 }
î 1, x ÎI
Domain of g ( g ( x )) Î R
(a) Graph of f (2 x )
and Range of g ( g ( x )) Î {0}
ì2x – [2x ], 2x Ï I
As, f ( 2x ) = í (c) Here, f (2x ) and g ( x ) are periodic with period 1/2 and
î 1, 2x ÎI 1. Also, g { g ( x )} is constant function. Therefore,
Y periodic and its period is undetermined.

1 l Ex. 50 Prove that f ( x ) = [tan x ] + tan x – [tan x ].


(where [ × ] denotes greatest integer function) is
é pö
continuous in ê0, ÷ .
ë 2ø
X′ X
–1 0 1 1

2 2
f(2x) Sol. Here, f ( x ) = [tan x ] + tan x – [tan x ]
Y′
or g ( x ) = [ x ] + x – [ x ], where x = tan x ³ 0
Graph of g ( x )
Then, for a Î N . We discuss continuity of f ( x ) as
{ f ( x )} 2n –1 LHL (at x = a)
Here, g ( x ) = lim
n ®¥ { f ( x )} 2n + 1 = lim g ( x ) = lim ([a – h ] + a – h – [a – h ])
x ® a– h ®0
ìï 0, x Î I
g(x ) = í = lim ((a – 1) + a – h – a + 1 ) = a – 1 + 1 = a
–1, x Ï I , as lim { f ( x )} 2n = 0
îï
h ®0
n ®¥

Y ì where , a – 1 < a – h < a


í
2 î\[a – h ] = a – 1
Now, RHL (at x = a)
1
∈I = lim g ( x ) = lim ([a + h ] + a + h – [a + h ])
0, x x ® a+ h ®0
x) =
X′ –2 –1 0 1 2 g( = lim (a + a + h – a ) = a
X h ®0
I
, x∉
) = –1 [Qa < a + h < a + 1 Þ [a + h ] = a ]
–1 g (x
and g (a ) = [a ] + a – [a ] =a
Y′ as a ÎN
Graph of g { g ( x ) }
So, g ( x ) is continuous, " a Î N , now g ( x ) is clearly
ì 0, x Î I continuous in (a – 1, a), " a Î N .
We have, g(x ) = í
î–1, x ÏI Hence, g ( x ) is continuous in [0, ¥).
ì 0, g ( x ) Î I Now, let f( x ) = tan x which is continuous in [0, p/ 2].
Þ g { g ( x )} = í
î–1, g ( x )ÏI So, g { f ( x )} is continuous in [0, p/ 2).
where g ( x ) Î {0, – 1} Hence, f ( x ) = [tan x ] + tan x – [tan x ] is continuous in
[0, p/ 2).
358 Textbook of Differential Calculus

x x
l Ex. 51 Determine the values of x for which the following or x ò g 2 (t ) dt + x 2 g 2 ( x ) = 4 xg ( x ) ò g (t ) dt ...(ii)
0 0
functions fails to be continuous or differentiable
ì(1 – x ), x <1 Using Eq. (i),
ï 2
f ( x ) = í(1 – x ) ( 2 – x ), 1 £ x £ 2 2 æç ò g (t ) dt ö÷ + x 2 g 2 ( x ) = 4 xg ( x ) ò g (t ) dt
x x

ï(3 – x ), è 0 ø 0
î x >2
2
Þ 2 æç ò g (t ) dt ö÷ – 4 xg ( x ) ò g (t ) dt + x 2 g 2 ( x ) = 0
x x
Justify your answer. [IIT JEE 1995]
è 0 ø 0
Sol. By the given definition, it is clear that the function f is x
continuous and differentiable at all points except which is quadratic in ò g (t ) dt ,
0
possibility at x = 1 and x = 2.
Continuity at x = 1 x 2± 2
\ ò0 g (t ) dt =
2
xg ( x )
LHL = lim f ( x ) = lim (1– x ) = lim (1–(1–h )) = 0
x ®1– x ®1– h ®0 On differentiating both sides w.r.t. x, we get
RHL = lim f ( x ) = lim (1– x )(2– x ) g ¢( x ) 1 ± 2
x ®1
+
x ®1
+ =
g(x ) x
= lim {1–(1 + h )} {2–(1 + h )} = 0
h ®0 Þ g ( x ) = kx 1 ± 2
, since g(1) = 1
Also, f (1) = 0. Þ k =1
Hence, LHL = RHL = f (1) \ g(x ) = x1 ± 2
Therefore, f ( x ) is continuous at x = 1.
which is continuous in [0, ¥ ).
Differentiability at x = 1
f (1–h )– f (1) 1–(1–h )–0
L f ¢(1) = lim = lim = –1 ìx + a , if x < 0
h ®0 -h h ® 0 –h l Ex. 53 Let f ( x ) = í and
f (1 + h )– f (1) î| x – 1 |, if x ³ 0
R f ¢(1) = lim x + 1 , if x < 0
h ®0 h ì
g( x ) = í
{1–(1 + h )} {2–(1 + h )}–0 î( x – 1 ) + b, if x ³ 0
2
= lim = –1
h ®0 h where a and b are non-negative real numbers. Determine the
Since, L { f ¢ (1)} = R { f ¢ (1)} so, we get f ( x ) is differentiable composite function gof . If ( gof ) ( x ) is continuous for all
at x = 1. real x. Determine the values of a and b. Further for these
Continuity at x = 2 values of a and b, is gof differentiable at x = 0. Justify your
LHL = lim f ( x ) = lim (1– x )(2– x ) answer. [IIT JEE 2002]
x ®2 – x ®2 –
= lim (1–(2–h )) (2–(2–h )) = 0 ì x + a, if x < 0
Sol. Here, f (x ) = í
h ®0
î| x – 1|, if x ³ 0
RHL = lim f ( x ) = lim (3– x ) = lim 3–(2 + h ) = 1
x ®2
+
x ®2
+ h ®0 ì x + 1, if x < 0
and g(x ) = í
Since, RHL ¹ LHL
î( x – 1) + b, if x ³ 0
2
Therefore, f ( x ) is not continuous at x = 2.
As such f ( x ) cannot be differentiable at x = 2. ì g ( x + a ), x < 0
\ gof ( x ) = g { f ( x )} = í
Hence, f ( x ) is continuous and differentiable at all points î g (| x – 1| ), x ³ 0
except at x = 2. x + a + 1, x + a < 0
ì
ï
l Ex. 52 If g ( x ) is continuous function in [0, ¥) satisfying = í ( x + a – 1)2 + b, x + a ³ 0
ï {| x – 1| – 1} 2 + b, x ³0
î
g (1 ) = 1 . If ò 2 x × g 2 (t ) dt = æç ò 2 g ( x – t ) dt ö÷ , find g ( x ).
2
x x
0 è 0 ø ì x + a + 1, x < –a
ï
ï( x + a – 1) + b, 0 > x ³ –a
x x 2
Sol. Here, ò0 g (t ) dt = ò0 g ( x – t ) dt , the given equation could =í
ï x 2 + b, 0£ x <1
be written as
2 ï ( x – 2)2 + b, x ³1
x ò g 2 (t ) dt = 2 æç ò g (t ) dt ö÷
x x
...(i) î
0 è 0 ø
Þ gof ( x ) is continuous for all real x.
On differentiating both sides w.r.t. ‘ x ’, we get
\ gof ( x ) is continuous at x = – a, 0, 1.
g 2 (t ) dt + xg 2 ( x ) = 4 g ( x ) ìí ò g (t ) dt üý
x x
ò0 î 0 þ
Since, gof ( x ) is continuous at x = – a.
Chap 06 Continuity and Differentiability 359

Þ lim gof ( x ) = lim gof ( x ) = gof (–a ) 1


x ® -a – x ® -a + Þ x = 0, x = and x = 1 may be the points at which f ( x )
2
Þ lim ( x + a + 1) = lim ( x + a – 1)2 + b may be discontinuous ...(i)
x ® –a –
x ® –a +
Now, consider x Î(1, 2]
= (–a + a – 1)2 + b f ( x ) = [ x – 2] | 2x – 3|
Þ – a + a + 1 = (–a + a – 1)2 + b = (–a + a – 1)2 + b If x Î(1, 2), then [ x - 2] = - 1 and if x = 2, then [ x - 2] = 0
Þ b=0 Also, | 2x – 3| = 0 Þ x =3/ 2
and gof ( x ) is continuous at x = 0. Þ x = 3/ 2 and 2 may be the points at which f ( x ) may be
Þ lim gof ( x ) = lim gof ( x ) = gof (0) discontinuous ...(ii)
x ® 0– x ® 0+
On combining Eqs. (i) and (ii), we have
Þ ( a – 1) 2 + b = b Þ a = 1 x = 0, 1/ 2, 1, 3/ 2, 2
d On dividing f ( x ) by about the 5 critical points, we get
Now, (LHD at x = 0) = {( x + a – 1)2 + b } at x =0
dx ì 1, x = 0 Q cos ( p 0 ) = 1
ï
= 2 ( a – 1) = 0 [as a = 1] ï 0, 0 < x £
1
Q 0 £ cos px < 1 Þ [cos px ] = 0
d ï 2
Again, (RHD at x = 0) = {( x 2 + b )} at x =0 =0 ï 1
ï–1, 2 < x £ 1 Q – 1 £ cos px < 0 Þ [cos px ] = – 1
dx
\ gof ( x ) is differentiable at x = 0. ï
f ( x ) = í–(3 – 2x ), 1 < x £ 3/ 2 Q | 2x – 3| = 3 – 2x
ï
l Ex. 54 If a function f :[– 2a , 2a ] ® R is an odd function ï and [ x – 2] = – 1
such that f ( x ) = f ( 2a – x ) for x Î[a , 2a ] and the left hand ï–(2x – 3), 3/ 2 < x < 2 Q | 2x – 3| = 2x – 3
ï
derivative at x = a is 0, then find the left hand derivative at ï and [ x – 2] = – 1
x =–a [IIT JEE 2003] ïî0, x = 2 Q [ x – 2] = 0
Sol. It is given that, LHD (at x = a) = 0 Checking continuity at x = 0
f (a – h ) – f (a ) RHL = lim f ( x ) = lim 0 = 0
\ lim =0
h ®0 –h x ® 0+ x ® 0+
And f ( 0) = 1
f (a – h ) – f (a )
Þ lim =0 ...(i) As RHL ¹ f (0)
h ®0 h
Thus, f ( x ) is discontinuous at x = 0
Now, LHD (at x = – a)
f (–a – h ) – f (–a ) Checking continuity at x = 1 / 2
= lim LHL = lim f ( x ) = lim 0 = 0
h ®0 –h 1– 1–
x® x®
– f (a + h ) + f (a ) 2 2
= lim [as f (–x ) = – f ( x ), given]
h ®0 –h RHL = lim f ( x ) = lim (–1) = – 1
1+ 1+
f (a + h ) – f (a ) f [2a – (a - h )] – f (a ) x® x®
= lim = lim 2 2
h ®0 h h ®0 h As RHL ¹ LHL
Therefore, f ( x ) is discontinuous at x = 1/ 2
f (a – h ) – f (a ) Checking continuity at x = 1
= lim =0 [as f ( x ) = f (2a – x )]
h ®0 h LHL = lim f ( x ) = lim (–1) = – 1
[using Eq. (i)] x ® 1– x ® 1–

\ LHD (at x = – a) = 0 RHL = lim f ( x ) = lim –(3 – 2x ) = – 1


x ® 1+ x ® 1+
And f (1) = – 1
lEx. 55 Discuss the continuity of f ( x ) in [0, 2], where
As RHL = LHL = f (1)
ì[cos px ], x £1
f (x ) = í Therefore, f ( x ) is continuous at x = 1
î| 2 x – 3 |[ x – 2 ], x > 1 Checking continuity at x = 3 / 2
where [ × ] denotes the greatest integral function. LHL = lim (2x –3) = 0
x ® 3/2–
Sol. First we shall find the points where f ( x ) may be
discontinuous. RHL = lim ( 3 –2 x ) = 0
x ® 3/2+
Consider x Î[0, 1] And f ( 3/ 2) = 0
f ( x ) = [cos px ] is discontinuous where cos px Î I . As RHL = LHL = f (3/ 2)
In [0, 1], cos px is an integer at x = 0, 1/ 2, 1. Hence, f ( x ) is continuous at x = 3/ 2
360 Textbook of Differential Calculus

Checking continuity at x = 2 h –1 –
h
+x ì æ hö ü
f ( x )(e h – 1) + e x
íf ç1 + ÷ – f (1)ý
LHL = lim – (3 – 2x ) = – 1 è xø
x ®2 = lim î þ
h ®0 h
And f ( 2) = 0
[Q f (1) = 0 ]
As LHL ¹ f (2) h
Hence, f ( x ) is discontinuous at x = 2
h –1 – + x ì æ hö ü
e x
í f ç1 + ÷ – f (1)ý
î è xø
h
Thus, f ( x ) is continuous when x Î[0, 2] – { 0, 1/ 2, 2 }. = lim
f ( x )(e – 1)
+ lim þ
h ®0 h h ®0 h
×x
l Ex. 56 Let f is a differentiable function such that x
x
f ( x ) = x 2 + ò e –t f ( x – t ) dt . Find f ( x ). ex –1
× f ¢(1)
= f (x ) +
0 x
x –t é æ hö ù
Sol. We have, f ( x ) = x 2 + ò 0 e f ( x – t ) dt ê f ç1 +
è xø
÷ – f ( 1)
eh – 1 ú
x êQ lim = f ¢(1) and lim = 1ú
Þ f ( x ) = x 2 + ò e – (x –t ) f ( x –( x – t )) dt ê h ®0 h h ®0 h ú
0
é using a f ( x )dx = a f (a – x )dx ù êë x úû
ëê ò0 ò0 ûú ex
x t = f (x ) + × f ¢(1)
Þ f (x ) = x + e ò
2 –x ex
e f (t ) dt ...(i)
0
ex ex
On differentiating both the sides, we get \ f ¢( x ) = f ( x ) + Þ = f ¢( x ) – f ( x ) [Qf ¢(1) = e ]
x t
x x
f ¢( x ) = 2x + e –x [e x f ( x )] – e –x ò0 e f (t ) dt ...(ii) 1 e x f ¢( x ) – f ( x ) × e x
Þ =
[using Leibnitz rule] x e 2x
Þ f ( x ) + f ¢( x ) = x 2 + 2x + f ( x ) é e x f ¢( x ) – f ( x ) × e x ù
Þ f ¢( x ) = x 2 + 2x ...(iii) ê as by quotient rule, we can write ú
ê (e x )2 ú
On integrating both the sides of Eq. (iii), we get ê d ì f ( x )ü ú
x3 ê = í x ý ú
f (x ) = + x2 + C ë dx î e þ û
3
But f ( 0) = 0 Þ C = 0 1 d ì f ( x )ü
\ = í ý
x3 x dx î e x þ
Hence, f (x ) = + x2
3 On integrating both the sides w.r.t. ‘x’, we get
+ f (x )
l Ex. 57 Let f : R ® R satisfies the functional equation log| x | + C = x
e
f ( xy ) = e xy – x – y {e y f ( x ) + e x f (y )}, " x , y Î R + . If
f ( x ) = e x {log | x | + C }
f ¢ (1 ) = e, determine f ( x ).
Sol. Given that, Since, f ( 1) = 0 Þ C = 0
+
f ( xy ) = e xy – x – y
{e f ( x ) + e f (y )}," x , y Î R
y x
...(i) Thus, f ( x ) = e x log | x |
On putting x = y =1, we get l Ex. 58 Let f is a differentiable function such that
f (1) = e –1 {e 1 f (1) + e 1 f (1)} Þ f (1) = 0 ...(ii)
4–e2
f ¢( x ) = f ( x ) + ò f ( x ) dx , f (0 ) =
2
f (x + h) – f (x ) , find f ( x ).
Now, f ¢( x ) = lim 0 3
h ®0 h
Sol. It is given that
ì æ h öü
f í x ç1 + ÷ý – f ( x ) f ¢( x ) = f ( x ) +
2
è x øþ ò 0 f ( x ) dx
= lim î
h ®0 h
f ¢( x ) = f ( x ) + c é\ c = 2 ù
æ hö æ hö
ìï 1 + h
Þ
êë ò 0 f ( x ) dx úû
x ç1 + ÷ – x – ç1 + ÷
è xø è xø æ h öüï
e í e x f ( x ) + e x f ç1 + ÷ý – f ( x ) f ¢( x )
ïî è x øï Þ =1
= lim þ f (x ) + c
h ®0 h
h
On integrating both the sides of above expression,
h –1 – +x æ hö
e f (x ) + e
h x f ç1 + ÷ – f ( x ) loge { f ( x ) + c } = x + c Þ f ( x ) + c = ke x ...(i)
è xø
= lim [k being constant of integration]
h ®0 h
Chap 06 Continuity and Differentiability 361

4 –e2
Since, f (0) = and f (0) + c = k Hence, at any arbitrary point ( x = a )
3
æ4 –e2 ö LHL = RHL = f (a )
4 –e2
\ k= +c Þ c = k –ç ÷ ...(ii) Therefore, function is continuous for all values of x, if it is
3 è 3 ø
continuous at 1.
From Eqs. (i) and (ii), we get
æ x + y ö f ( x ) + f (y )
æ4 –e2 ö l Ex. 60 Let f ç ÷= for all real x and y.
f (x ) + k –ç ÷ = k (e )
x
è 2 ø 2
è 3 ø If f ¢ (0 ) exists and equals to –1 and f (0 ) = 1 , then find f ¢( x ).
æ4 –e2 ö [IIT JEE 1995]
Þ f ( x ) = k ( e x – 1) + ç ÷ ...(iii)
è 3 ø
æ x + y ö f ( x ) + f (y )
Sol. Given equation is f ç ÷= ...(i)
Now, to find constant of integration k. è 2 ø 2
On integrating both the sides from 0 to 2, we get On putting y = 0 and f (0) = 1 in Eq. (i), we have
2 2 x 4 –e2 2
ò0 f ( x ) dx = k ò0 ( e – 1 ) dx +
3 ò0
dx æx ö 1
f ç ÷ = [ f ( x ) + 1] [Q f (0) = 1]
è2ø 2
4 –e2
Þ c = k ( e 2 – 2 – 1) + ( 2)
3 æx ö
Þ f ( x ) = 2 f ç ÷ –1 ...(ii)
2 è2ø
Þ c = ( 4 – e 2 ) + k ( e 2 – 3) ...(iv)
3 f (x + h) – f (x )
From Eqs. (ii) and (iv), we get Now, f ¢( x ) = lim
h ®0 h
æ4 –e2 ö 2
k–ç ÷ = ( 4 – e ) + k ( e – 3) Þ k = 1
2 2 æ 2x + 2h ö
f ç ÷ – f (x )
è 3 ø 3 è 2 ø
= lim
On putting k = 1 in Eq. (iii), we get h ®0 h
4 –e2 f (2x ) + f (2h )
f ( x ) = ( e x – 1) + – f (x )
3 = lim 2 [using Eq. (i)]
h ®0 h
( e 2 – 1)
Hence, f (x ) = e –
x
3 f (2x ) + f (2h )–2 f ( x )
= lim
h ®0 2h
l Ex. 59 A function f ( x ) satisfies the following property 2 f ( x ) – 1 + f (2h )–2 f ( x )
= lim [using Eq. (ii)]
f ( x + y ) = f ( x ) × f (y ). h ®0 2h
Show that the function is continuous for all values of x, if it f (2h ) – 1
is continuous at x =1 . lim = f ¢(0)
h ®0 2h
Sol. As the function is continuous at x = 1, we have
Therefore, f ¢( x ) = –1
lim f ( x ) = lim f ( x ) = f (1)
x ® 1– x ® 1+
l Ex. 61 Let f ( x ) = 1 + 4 x – x 2 , " x Î R
Þ lim f (1 - h ) = lim f (1 + h ) = f (1)
h ®0 h ®0
g ( x ) = max { f (t ); x £ t £ ( x + 1); 0 £ x < 3 }
[using f ( x + y ) = f ( x ) × f (y )]
= min {( x + 3 ); 3 £ x £ 5 }
Þ lim f (1) f ( -h ) = lim f (1) × f (h ) = f (1) Verify continuity of g ( x ), for all x Î[0, 5 ] .
h ®0 h ®0

Þ lim f (–h ) = lim f (h ) = 1 ...(i) Sol. Here, f ( t ) = 1 + 4t – t 2


h ®0 h ®0
f ¢(t ) = 4 – 2t
Now, consider some arbitrary points x = a.
When f ¢ (t ) = 0, then t = 2
Left hand limit
At t = 2, f (t ) has a maxima.
Þ lim f (a –h ) = lim f (a ) × f (–h ) = f (a ) lim f (–h )
h ®0 h ®0 h ®0 Since, g ( x ) = max { f (t ) for t Î [ x , x + 1], 0 £ x < 3 }
LHL = f (a ) [as lim f (–h ) = 1, using Eq. (i)] ì f ( x + 1), if [ x , x + 1] < 2
h ®0 ï
Right hand limit \ g ( x ) = í f (2), if x £ 2 £ x + 1
Þ lim f (a + h ) = lim f (a ) × f (h ) = f (a ) lim f (h ) ï f ( x ), if 2 < [ x , x + 1]
h ®0 h ®0 h ®0
î
RHL = f (a ) [as lim f ( h ) = 1, using Eq. (i)]
h ®0
362 Textbook of Differential Calculus

ì 4 + 2x – x 2 , if 0 £ x < 1 11 1 1
ï
We get, ò0 2 dt £ ò 0 f (t )dt £ ò 0 1 dt
ï 5, if 1 £ x £ 2
g (x ) = í 1 1
ï1 + 4 x – x , if 2 < x < 3
2
Þ
2
£ ò0 f (t )dt £ 1 ...(ii)
ïî 6, if 3 £ x £ 5
1
Again, 0 £ f (t ) £ , for t Î [1, 2]
Which is clearly continuous for all x Î[0, 5] except x = 3. 2
\ g ( x ) is continuous for x = [0, 3) È (3, 5] 2 2 2 1
Þ ò1 0dt £ ò1 f (t )dt £ ò1 2 dt
l Ex. 62 Let f ( x ) = x 4 – 8 x 3 + 22 x 2 – 24 x and 2 1
Þ 0 £ ò f (t )dt £ ...(iii)
ìmin f ( x ); x £ t £ x + 1 , – 1 £ x £ 1 1 2
g( x ) = í
îx – 10 ; x >1 On adding Eq. (ii) and (iii), we get
Discuss the continuity and differentiability of g ( x ) in [–1 , ¥). 1 1 2
0 + £ ò f (t )dt + ò f (t )dt £ 1 +
1
2 0 1 2
Sol. Here, f ( x ) = x 4 – 8x 3 + 22x 2 – 24 x
1 3 é 1 3 ù
Þ f ¢( x ) = 4 x 3 – 24 x 2 + 44 x – 24 or £ g ( 2) £ Þ g ( 2) Î ê , ú
2 2 ë2 2û
or f ¢( x ) = 4 ( x – 1) ( x – 2) ( x – 3)
Which shows f ( x ) is increasing in [1, 2] È [3, ¥ ) and l Ex. 64 Let f be a one-one function such that
decreasing in (–¥, 1] È [2, 3].
f ( x ) × f (y ) + 2 = f ( x ) + f (y ) + f ( xy ), " x , y Î R – { 0 }
Thus, minimum f ( x ); x £ t £ x + 1, –1 £ x £ 1
and f (0 ) = 1 , f ¢ (1 ) = 2. Prove that
ì f ( x + 1), – 1 £ x £ 0 3 ( ò f ( x ) dx ) – x ( f ( x ) + 2 ) is constant.
Þ Minimum f ( x ) = í
î f (1), 0< x £1
Sol. We have, f ( x ) × f (y ) + 2 = f ( x ) + f (y ) + f ( xy ) ...(i)
ì f ( x + 1), – 1 £ x £ 0
ï On replacing x , y ® 1, we get
Thus, g ( x ) = í f (1), 0< x £1
ï x –10, ( f (1))2 + 2 = 3 f (1)
î x >1
Þ f 2 (1) – 3 f (1)+ 2 = 0
ì( x + 1)4 –8( x + 1)3 + 22( x + 1)2 –24 ( x + 1), – 1 £ x £ 0
ï Þ f (1) = 2 or 1
=í 1–8 + 22–24 , 0< x £1
ï But f (1) cannot be equal to one as f (0) = 1
x –10, x >1
î Þ f ( 1) = 2 ...(ii)
ì x 4 – 4 x 3 + 4 x 2 – 9, – 1 £ x £ 0 On replacing y by 1/x in Eq. (i), we get
ï
g (x )= í – 9, 0< x £1 f ( x ) × f (1/x ) + 2 = f ( x ) + f (1/x ) + f (1)
ï x –10, x >1 Þ f ( x ) × f (1/x ) + 2 = f ( x ) + f (1/x ) + 2 [using Eq. (ii)]
î
Þ f ( x ) × f (1/x ) = f ( x ) + f (1/x )
ì 4 x 3 – 12x 2 + 8x , – 1 £ x £ 0
ï f (1/x )
Also, g ¢( x ) = í 0, 0< x £1 Þ f (x ) =
f (1/x ) – 1
ï 1, x >1
î f (x )
and f (1/x ) = ...(iii)
Which clearly shows g ( x ) is continuous in [–1, ¥ ) but not f (x ) – 1
differentiable at x = 1 . f (x + h) – f (x )
Now, f ¢( x ) = lim
h ®0 h
x
l Ex. 63 Let g ( x ) = ò f (t ) dt , where f is such that f (1/x )
0
f (x + h) +
1 / 2 £ f (t ) £ 1 for t Î [0, 1 ] and 0 £ f (t ) £ 1 / 2 for t Î[1, 2 ]. = lim
1– f (1/x )
h ®0 h
Then, find the interval in which g( 2 ) lies. [IIT JEE 2000]
x 2 f ( x + h ) – f ( x + h ) × f (1/x ) + f (1/x )
Sol. Here, g (x ) = ò0 f (t )dt Þ g (2) = ò 0 f (t )dt = lim
h ®0 h {1 – f (1/x ) }
1 2
Þ g ( 2) = ò 0 f (t )dt + ò1 f (t )dt ...(i) æx + hö
f ( x + h ) – f ( x + h )– f (1/x )– f ç ÷ + 2 + f (1/x )
è x ø
Now, 1 / 2 £ f (t ) £ 1 for t Î[0, 1] = lim
h ®0 h {1 – f (1/x ) }
[using Eq. (i)]
Chap 06 Continuity and Differentiability 363

æ hö
f ç1 + ÷ – 2 \ f ¢( x ) = 1 + e x – 1 + xe x + 2x
è xø f (1 + h/x ) – f (1)
= lim = lim or f ¢( x ) = e x ( x + 1) + 2x ...(ii)
h ® 0 h { f (1/x ) – 1} h ®0 h
× x { f (1/x ) – 1} On integrating Eq. (ii) both sides, we get
x
[using Eq. (ii)] f ( x ) = ò e x ( x + 1) dx + 2 ò x dx
f ¢(1) x2
= = ( x + 1)e x – ò 1 × e x dx + 2 +C
x { f (1/x ) – 1} 2
é ù = ( x + 1) e x – e x + x 2 + C
æ1ö
ê f (x )f ç ÷ ú Þ f ( x ) = x 2 + xe x + C
èx ø
ê Q from Eq. (iii), f ( x ) × f æç ö÷ =
1
ú
ê èx ø ì æ1ö ü ú But f ( 0) = 0 Þ C = 0
ê í f ç ÷ – 1ý { f ( x ) – 1} ú f ( x ) = x 2 + x ×e x
ë î è x ø þ û So,
2 { f ( x ) – 1}
f ¢( x ) = Þ xf ¢( x ) = 2 { f ( x ) – 1} l Ex. 66 Let f be a function such that
x
On integrating both the sides of the above expression, we f ( xy ) = f ( x ) × f (y ), " y Î R and f (1 + x ) = 1 + x (1 + g ( x )),
2 f (x )
where lim g ( x ) = 0 . Find the value of ò
1
get × dx .
1 f ¢ (x )
x f ( x ) – ò f ( x ) dx = 2 ò f ( x ) dx – 2x + l, x®0 1+ x2
where l is the constant of integral f ( x + h )– f ( x )
Sol. We know, f ¢( x ) = lim
Þ 3ò f ( x ) dx = x {2 + f ( x )} – l h ®0 h
Hence, 3ò f ( x ) dx – x {2 + f ( x )} = l (constant) æ æ h öö
f ç x ç1 + ÷ ÷ – f ( x )
è è x øø
= lim
h ®0 h
l Ex. 65 Let f : R ® R, such that f ¢ (0 ) =1 and
æ hö
f ( x + 2y ) = f ( x ) + f ( 2y ) + e x + 2 y ( x + 2y ) – x × e x f ( x ) × f ç1 + ÷ – f ( x )
è xø
– 2y × e 2 y + 4 xy , " x , y Î R. Find f ( x ). Þ f ¢( x ) = lim
h ®0 h
Sol. We have, [given f ( xy ) = f ( x ) × f (y )]
+ 2y
f ( x + 2y ) = f ( x ) + f (2y ) + e x ( x + 2y ) – x × e x ì h ü
f ( x ) × í1 + (1 + g (h / x ))ý – f ( x )
– 2y × e 2 y + 4 xy î x þ
Þ f ¢( x ) = lim
h ®0 h
On replacing x , y ® 0, we get
f ( 0) = f ( 0) + f ( 0) + 0 – 0 – 0 + 0 Þ f ( 0) = 0 [given f (1 + x ) = 1 + x (1 + g ( x ))]
On replacing 2y ® – x , we get ì h ü
f ( x ) × í1 + (1 + g (h / x ))–1ý
f (0) = f ( x ) + f (–x ) – x × e x + xe –x – 2x 2 Þ f ¢( x ) = lim î x þ
h ®0 h
Þ – f ( x ) = f (–x ) – x × e x + xe –x – 2x 2 ...(i) f (x )
f (x + h) – f (x ) \ f ¢( x ) = [as lim g ( x ) = 0] ...(i)
Now, f ¢( x ) = lim x x ®0
h ®0 h 2 f (x ) 1 2 x
To find, ò × dx = ò dx [using Eq. (i)]
f ( x + h ) + f (–x ) – xe x + x × e –x – 2x 2 1 f ¢( x ) 1 + x 2 1 1 + x2
= lim
h ®0 h 1 1
= [log| 1 + x 2 | ]1 = [log (5 / 2)]
2
é f (h ) – e × h + ( x + h ) e (x + h ) – xe –x + ù
h
2 2
ê ú
êë 2 ( x + h ) x – xe x + xe –x – 2x 2 úû
Þ f ¢( x ) = lim l Ex. 67 If f ( x ) = ax 2 + bx + c is such that | f (0 )| £ 1 ,
h ®0 h
f (h ) – e h × h + x × e x × e h + h × e x × e h + 2hx – xe x | f (1 )| £ 1 and | f (–1 )| £ 1 , prove that
= lim | f ( x )| £ 5 / 4, " x Î[–1 , 1 ] .
h ®0 h
f (h ) – f (0) h (e x – 1) h xe x (e h – 1) 2hx Sol. We have, f ( x ) = ax 2 + bx + c …(i)
= lim + e + +
h ®0 h h h h Þ f (–1) = a – b + c ...(ii)
¢
= f ( 0) + ( e – 1) + x × e + 2x
x x Þ f ( 0) = c ...(iii)
Þ f ( 1) = a + b + c ...(iv)
364 Textbook of Differential Calculus

2 2
From Eqs. (ii), (iii) and (iv), we get
1
= ò0 f ( x ) dx + ò0 {2– f ( x )} dx – 4
a = [ f (–1) + f (1) – 2 f (0)] , [given, f (2 + x ) + f ( x ) = 2]
2
2 2 2
1
b = [ f (1) – f (–1)] and c = f (0)
Þ p= ò0 f ( x ) dx + 2ò 0 dx – ò 0 f ( x ) dx – 4 = 0 …(v)
2
Then, from Eqs. (v) and (iii), p = 0, q = 1
On substituting the values of a, b and c in Eq. (i), we get
Let the roots of equation ax 2 – bx + c = 0 be a and b.
1
Þ f ( x ) = [ f (–1) + f (1) – 2 f (0)] x 2
2 \ f ( x ) = ax 2 – bx + c = a ( x – a )( x – b ) ...(vi)
1 Since, equation f ( x ) = 0 has both roots lying between 0 and 1.
+ [ f (1) – f (– 1)] x + f (0)
2 \ f ( 0) × f ( 1) > 0 ...(vii)
x ( x - 1) But f (0) × f (1) = c (a – b + c ) = an integer ...(viii)
f (x ) = f (–1) – ( x + 1) ( x – 1) f (0)
2 \ Least value of f (0) × f (1) = 1 ...(ix)
( x + 1) x Now, from Eq. (vi),
+ f ( 1) ...(v)
2 f (0) × f (1) = a ab a(1 – a ) (1 – b )
Since, | f (– 1)| , | f (0)| and | f (1)| are £ 1, we have = a 2 ab (1 – a ) (1 – b ) ...(x)

2 | f ( x )| £ | x ( x - 1)| + 2 | x 2 – 1| + | x ( x + 1)| ...(vi) As we know,


1 1
In the interval, x Î[– 1, 1] a (1 – a ) has greatest value at a = and b (1 – b ) has
4 2
0 £ 1 + x £ 2 , 0 £ 1 – x £ 2 and 0 £ 2(1 – x 2 ) £ 2 greatest value
1 1
at b = . But a ¹ b
4 2
Þ 2 | f ( x )| £ | x | (1 – x + 1 + x ) + 2 (1 – x 2 )
a2
2 | f ( x )| £ 2 (| x | + 1 – x ) 2 Thus, from Eq. (x) greatest value of f (0) × f (1) < ...(xi)
16
æ 1ö
2
a2
Therefore, | f ( x )| £ ç| x | – ÷ + 5/ 4 £ 5/ 4 \ From Eqs. (ix) and (xi), we get 1 <
è 2ø 16
5 Þ a 2 – 16 > 0
Þ | f ( x )| £ , " x Î [–1, 1]
4 Þ a < – 4 or a > 4 [Qa Î N ]
Þ Least value of a = 5 [as a Î natural number]
l Ex. 68 Let a + b =1, 2 a 2 + 2 b 2 = 1 and f ( x ) be a
continuous function such that f ( 2 + x ) + f ( x ) = 2 for all l Ex. 69 Prove that the function
a f ( x ) = a x - 1 + b 2 x - 1 - 2 x 2 - 3 x + 1, where
x Î[0 , 2 ] and p = ò f ( x ) dx – 4, q = . Then, find the least
4
0 b a + 2b = 2 and a , b Î R always has a root in (1 , 5 ), " b Î R.
positive integral value of ‘a’ for which the equation Sol. Let b > 0, then f (1) = b > 0
ax 2 – bx + c = 0 has both roots lying between p and q, where and f (5) = 2a + 3b - 6 = 2(a + 2b ) - b - 6
a , b, c Î N . = 4 - b - 6 = - (2 + b ) < 0
Sol. Given, a +b =1 ...(i) Hence, by IVT, $ some c Î(1, 5) such that
2 a 2 + 2 b2 = 1 ...(ii) Þ f (c ) = 0
On solving Eqs. (i) and (ii), we get If b = 0, then a = 2
1 a f ( x ) = 2 x - 1 - 2x 2 - 3x + 1 = 0
a =b = Þq = =1 ...(iii)
2 b
Þ 4( x - 1) = 2x 2 - 3x + 1 = ( 2x - 1) ( x - 1)
and given f (2 + x ) + f ( x ) = 2," x Î [0, 2] ...(iv)
4 2 4
Þ ( x - 1) ( 2x - 5) = 0
Now, p= ò0 f ( x ) dx – 4 = ò0 f ( x ) dx + ò f ( x ) dx – 4
2 Þ x=
5
2 2 2
= ò0 f ( x ) dx + ò f (t + 2) dt – 4
0 5
Hence, f ( x ) = 0, if x = , which lies in (1, 5).
[ let x = t + 2 for second integration] 2
Chap 06 Continuity and Differentiability 365

If b < 0, f (1) = b < 0 and f (2) = a + b 3 - 3 f ( x )– f (a )


Þ = g (x )
= (a + 2b ) + ( 3 - 2)b - 3 (x –a )
f ( x )– f (a )
= (2 - 3 ) - (2 - 3 )b Þ lim = lim g ( x ) Þ f ¢(a ) = g (a )
x ®a ( x –a ) x ®a
= (2 - 3 )(1 - b ) > 0 [as b < 0]
Therefore, f ( x ) is differentiable at x = a ...(ii)
Hence, f (1) and f (2) have opposite signs Conversely, f is differentiable at x = a, then
$ some c Î (1, 2) Ì (1, 5) for which f (c ) = 0. f ( x )– f (a )
lim exists finitely.
x ®a ( x –a )
l Ex. 70 Let a Î R.. Prove that a function f : R ® R is
ì f ( x )– f (a )
differentiable at a, if and only if there is a function g : R ® R ï ,x ¹a
Let g ( x ) = í x –a
which is continuous at x = a and satisfies ï f ¢ (a ),
î x =a
f ( x ) – f (a ) = g ( x ) ( x – a ) for all x Î R. [IIT JEE 2001]
Sol. If g ( x ) is continuous at x = a. Clearly, lim g ( x ) = f ¢ (a )
x ®a
Þ lim g ( x ) = g (a ) ...(i) Thus, g ( x ) is continuous at x = a.
x ®a
Hence, f ( x ) is differentiable at x = a , iff g ( x ) is
and f ( x ) – f (a ) = g ( x ) ( x – a ), " x Î R continuous at x = a.
366 Textbook of Differential Calculus

Continuity and Differentiability Exercise 1 :


Single Option Correct Type Questions
ì px ì 1 + a x × x × log a - a x
,x <1
1. If f ( x ) = ïí for x £ 0
sin ï ,
2 , where [x] denotes the greatest ax ×x 2
ï
ïî[x ], x ³1 8. If g ( x ) = í
ï 2 x × a x - x log 2 - x log a - 1
integer function, then ï , for x > 0
(a) f ( x ) is continuous at x =1 î x2
(b) f ( x ) is discontinuous at x =1 where a > 0, then a for which g ( x ) is continuous, is
(c) f (1 + ) = 0 (a) -
1
(b)
1
(c) 2 (d) -2
- 2 2
(d) f (1 ) = - 1
ìæ p -1 2 ö -1
ì8 x - 4 x - 2 x + 1
x >0 ï çè 2 - sin (1 - {x } )÷ø × sin (1 - {x })
ï
9. Let f ( x ) = ïí
,
2. Consider f ( x ) = í x2 , x ¹0,
ï x ï 2 ({x } - {x } 3 )
îe sin x + px + k log 4, x < 0 ïî
Then, f (0) so that f ( x ) is continuous at x = 0, is p
, x = 0, where {×} is fractional part of x, then
(a) log 4 (b) log2 2
(c) (log 4 )(log 2 ) (d) None of these p
(a) f ( 0 + ) = -
ì 2
ï a(1 - x sin x ) + b cos x + 5 , x < 0 - p
(b) f ( 0 ) =
ï x2 4 2
ï (c) f ( x ) is continuous at x = 0
3. Let f ( x ) = í 3, x =0
ïé æ 3 öù
1/ x (d) None of the above
ï ê1 + ç cx + dx ÷ ú , x >0 ìsgn ( x ) + x , - ¥ < x < 0
ïê è x 2 ø ú
îë û 10. Let f ( x ) = ïí -1 + sin x , 0 £ x £ p / 2 , then the number of
If f is continuous at x = 0, then (a + b + c + d ) is ï cos x , p /2£ x < ¥
(a) 5 (b) -5 (c) loge 3 - 5 (d) 5 - loge 3
î
points, where f ( x ) is not differentiable, is/are
ì cos {cot x }, x < p / 2
-1
4. If f ( x ) = í , then the jump of (a) 0 (b) 1
î p [x ] - 1 , x ³ p /2 (c) 2 (d) 3
discontinuity, (where [×] denotes greatest integer and {×} ì1
ï , x ³1
denotes fractional part function) is 11. Let f ( x ) = í x be continuous and
p ïax 2 + b, x < 1
(a) 1 (b) p /2 (c) -1 (d) 2
2 î
5. Let f :[0, 1] ¾onto
¾¾® [0, 1] be a continuous function, then differentiable for all x. Then, a and b are
1 3 1 3
f ( x ) = x holds for (a) - , (b) , -
(a) at least one x Î[ 0, 1 ] (b) at least one x Î[1, 2 ] 2 2 2 2
1 3
(c) at least one x Î[ -1 , 0 ] (d) can’t be discussed (c) , (d) None of these
2 2
x +1 1
6. If f ( x ) = and g ( x ) = , then ( fog )( x ) is ì A + Bx 2 , x <1
x -1 x -2 12. If f ( x ) = í , then A and B,
discontinuous at î 3Ax - B + 2, x ³1
(a) x =3 only (b) x =2 only so that f ( x ) is differentiable at x = 1, are
(c) x =2 and 3 (d) x =1 only (a) -2, 3 (b) 2, -3
x 2
x 2
x 2 (c) 2, 3 (d) -2, -3
7. Let y n ( x ) = x 2 + + + ¼+ and ì x - 1 ([x ] - x ), x ¹ 1
(1 + x ) (1 + x )
2
(1 + x 2 )n -1
2 2
13. If f ( x ) = í , then
y( x ) = lim y n ( x ) , then y n ( x ), n = 1, 2, 3, …, n and y( x ) is î 0, x =1
n ®¥
(a) continuous for x ÎR (b) continuous for x ÎR - { 0 } (a) f ¢(1 + ) = 0 (b) f ¢(1 - ) = 0
(c) continuous for x ÎR - {1 } (d) data unsufficient (c) f ¢(1 - ) = -1 (d) f ( x ) is differentiable at x =1
Chap 06 Continuity and Differentiability 367

ì[cos px ], x £ 1 12
14. If f ( x ) = í , where [ × ] and { × } denote If f ( x 2 ) = x 4 + x 5 , then å f (r 2 ) is equal to
î 2{x } - 1, x > 1 r =1
greatest integer and fractional part of x, then (a) 216 (b) 219
(a) f ¢(1 - ) = 2 (b) f ¢(1 + ) = 2 (c) 222 (d) 225
(c) f ¢(1 - ) = - 2 (d) f ¢(1 + ) = 0 ì1 p
, if x =
x - 3, x <0 22. For x > 0, let h( x ) = ïí q q , where
ì
15. If f ( x ) = í , then g ( x ) = f (| x | ) is ïî 0, if x is irrational
î x 2
- 3x + 2, x ³0
p and q > 0 are relatively prime integers, then which one
(a) g ¢( 0 + ) = - 3
of the following does not hold good?
(b) g ¢( 0 - ) = - 3 (a) h( x ) is discontinuous for all x in ( 0, ¥ )
(c) g ¢( 0 + ) = g ¢( 0 - ) (b) h( x ) is continuous for each irrational in ( 0, ¥ )
(d) g( x ) is not continuous at x = 0 (c) h( x ) is discontinuous for each rational in ( 0, ¥ )
(d) h( x ) is not derivable for all x in ( 0, ¥ )
ìì 1ü
ïïí x + 3ý[sin px ], 0 £ x < 1 g( x )
23. Let f ( x ) =
16. If f ( x ) = í î þ , where [ × ] and { × }
, where g and h are continuous functions
h( x )
ï[2x ]sgn æç x - 4 ö÷, 1 £ x £ 2
ïî è 3ø on the open interval (a, b ). Which of the following
statements is true for a < x < b?
denote greatest integer and fractional part of
(a) f is continuous at all x for which x ¹ 0
x respectively, then the number of points of (b) f is continuous at all x for which g ( x ) = 0
non-differentiability, is (c) f is continuous at all x for which g( x ) ¹ 0
(a) 3 (b) 4 (c) 5 (d) 6 (d) f is continuous at all x for which h( x ) ¹ 0
17. Let f be differentiable function satisfying 2 cos x - sin 2x e - cos x - 1
æx ö 24. If f ( x ) = , g( x ) = and
f ç ÷ = f ( x ) - f (y ) for all x , y > 0. If f ¢ (1) = 1, ( p - 2x ) 2 8x - 4p
èy ø ì f ( x ), for x < p / 2
then f ( x ) is h( x ) = í ,
î g ( x ), for x > p / 2
1
(a) 2 loge x (b) 3 loge x (c) loge x (d) loge x
2 then which of the following holds?
(a) h is continuous at x = p/2
18. Let f ( x + y ) = f ( x ) + f (y ) - 2xy - 1 for all x and y. If
(b) h has an irremovable discontinuity at x = p/2
f ¢ (0) exists and f ¢ (0) = - sin a, then f { f ¢ (0)} is (c) h has a removable discontinuity at x = p/2
(a) -1 (b) 0 æ p+ ö æ p- ö
(c) 1 (d) 2 (d) f ç ÷ = gç ÷
è 2 ø è 2 ø
19. A derivable function f : R + ® R satisfies the condition
æx ö x - e x + cos 2x
f ( x ) - f (y ) ³ logç ÷ + x - y, " x , y Î R + . 25. If f ( x ) = , x ¹ 0 is continuous at x = 0,
èy ø x2
then
If g denotes the derivative of f , then the value of the 5
100
(a) f ( 0 ) = (b) [ f ( 0 )] = - 2
sum å g æç 1 ö÷ is
2
è ø (c) { f ( 0 )} = - 0.5 (d) [ f ( 0 )] × { f ( 0 )} = - 1.5
n =1 n
(a) 5050 (b) 5510 (c) 5150 (d) 1550 where, [x ] and {x } denote the greatest integer and
d ( f ( x )) fractional part function.
20. If = e - x f ( x ) + e x f ( - x ), then f ( x ) is, (given é x {x } + 1, if 0 £ x < 1
dx 26. Consider the function f ( x ) = ê ,
f (0) = 0) ë 2 - {x }, if 1 £ x £ 2
(a) an even function where {x } denotes the fractional part function. Which
(b) an odd function one of the following statements is not correct?
(c) neither even nor odd function (a) lim f ( x ) exists
x ®1
(d) can’t say
(b) f ( 0 ) ¹ f (2 )
21. Let f :(0, ¥ ) ® R be a continuous function such that (c) f ( x ) is continuous in [0, 2]
x (d) Rolle’s theorem is not applicable to f ( x ) in [ 0, 2 ]
f ( x ) = ò t f (t ) dt .
0
368 Textbook of Differential Calculus

é 2x + 23- x - 6 33. The total number of points of non-differentiability of


ê , if x > 2
ê
-x
- 1- x é 1ù
27. Let f ( x ) = 2 2 , then f ( x ) = min ê|sin x |, | cos x |, ú in (0, 2p ) is
ê x2 -4 ë 4û
ê , if x < 2 (a) 8 (b) 9 (c) 10 (d) 11
êë x - 3x - 2
(a) f (2 ) = 8 Þ f is continuous at x = 2 34. The function f ( x ) = [x ]2 - [x 2 ] , where [y ] is the
(b) f (2 ) = 16 Þ f is continuous at x = 2 greatest integer less than or equal to y, is discontinuous
(c) f (2 - ) ¹ f (2 + ) Þ f is discontinuous at
(d) f has a removable discontinuity at x = 2 (a) all integers
(b) all integers except 0 and 1
28. Let [x ] denotes the integral part of g ( x ) = x - [x ], x Î R. (c) all integers except 0
Let f ( x ) be any continuous function with f (0) = f (1), (d) all integers except 1
then the function h( x ) = f ( g ( x ))
(a) has finitely many discontinuities 35. The function f ( x ) = ( x 2 - 1)| x 2 - 6x + 5 | + cos | x | is
(b) is discontinuous at some x = c not differentiable at
(c) is continuous on R (a) -1 (b) 0 (c) 1 (d) 5
(d) is a constant function ì -1
36. Let f ( x ) = ïíe x , if x ¹ 0
2
29. Let f be a differentiable function on the open interval
(a, b ). ïî 0, if x = 0
I. f is continuous on the closed interval [a, b ]. Then, f ¢ (0) is equal to
II. f is bounded on the open interval (a, b ).
(a) 0 (b) 1
III. If a < a1 < b1 < b and f (a1 ) < 0 < f (b1 ), then there (c) - 1 (d) doesn’t exist
exists a number c such that a1 < c < b1 and f (c ) = 0.
e x - cos 2x - x
Which of the above statements must be true? 37. Given f ( x ) = , for x Î R - {0}
(a) I and II x2
(b) I and III 1
(c) II and III f ({x }), for n < x < n +
2
(d) Only III 1
f (1 - {x }), for n + £ x < n + 1, n Î I
2
30. Number of points, where the function g( x ) =
f ( x ) = ( x 2 - 1) | x 2 - x - 2 | + sin (| x | ) is not 5
, otherwise
differentiable, is 2
(a) 0 (b) 1
(c) 2 (d) 3 ì where {x } denotes
í then g ( x ) is
x î fractional part function,
31. Consider function f : R - {- 1, 1} ® R. f ( x ) = .
1 - | x| (a) discontinuous at all integral values of x only
(b) continuous everywhere except for x = 0
Then, which of the statements is incorrect? 1
(a) It is continuous at the origin (c) discontinuous at x = n + , n Î I and at some x Î I
2
(b) It is not derivable at the origin
(d) continuous everywhere
(c) The range of the function is R
(d) f is continuous and derivable in its domain ì x + b, x < 0
38. The function g ( x ) = í cannot be made
32. If the functions f : R ® R and g : R ® R are such that î cos x , x ³ 0
f ( x ) is continuous at x = a and f (a ) = a and g ( x ) is differentiable at x = 0,
discontinuous at x = a but g ( f ( x )) is continuous at x = a, (a) if b is equal to zero (b) if b is not equal to zero
where f ( x ) and g ( x ) are non-constant functions. (c) if b takes any real value (d) for no value of b
(a) x = a is a extremum of f ( x ) and x = a is an extremum of 39. The graph of function f contains the point P (1, 2) and
g( x ) Q (s, r ). The equation of the secant line through P and Q is
(b) x = a may not be an extremum of f ( x ) and x = a is an æ s 2 + 2s - 3 ö
extremum of g( x ) y=ç ÷ x - 1 - s. The value of f ¢ (1) is
(c) x = a is an extremum of f ( x ) and x = a may not be an è s -1 ø
extremum of g( x ) (a) 2 (b) 3
(d) None of the above (c) 4 (d) non-existent
Chap 06 Continuity and Differentiability 369

40. Consider (a) Not continuous at any point


é 2(sin x - sin x ) + |sin x - sin x | ù
3 3
p (b) Continuous at 3/2
f (x ) = ê ú, x ¹ for (c) Discontinuous at 2
êë 2(sin x - sin x ) - |sin x - sin x | úû
3 3 2 (d) Differentiable at 4/3
x Î(0, p ), f ( p / 2) = 3, where [ ] denotes the greatest ìb ([x ]2 + [x ]) + 1, for x ³ - 1
integer function, then 46. If f ( x ) = í , where [x ]
(a) f is continuous and differentiable at x = p /2 îsin ( p( x + a )), for x < - 1
(b) f is continuous but not differentiable at x = p /2 denotes the integral part of x, then for what values of a, b
(c) f is neither continuous nor differentiable at x = p /2 the function is continuous at x = - 1?
(d) None of the above (a) a = 2n + (3 / 2 ); b Î R; n Î I
(b) a = 4n + 2; b Î R; n Î I
41. If f ( x + y ) = f ( x ) + f (y ) + | x | y + xy 2 , " x , y Î R and (c) a = 4n + (3 / 2 ); b Î R + ; n Î I
f ¢ (0) = 0, then (d) a = 4n + 1; b Î R + ; n Î I
(a) f need not be differentiable at every non-zero x
(b) f is differentiable for all x Î R 47. If both f ( x ) and g ( x ) are differentiable functions at
(c) f is twice differentiable at x = 0 x = x 0 , then the function defined as,
(d) None of the above h( x ) = max{ f ( x ), g ( x )}
(a) is always differentiable at x = x 0
42. Let f ( x ) = max{|x 2 - 2|x||, | x | } and (b) is never differentiable at x = x 0
g ( x ) = min {| x 2 - 2 | x ||, | x | }, then (c) is differentiable at x = x 0 when f ( x 0 ) ¹ g( x 0 )
(a) both f ( x ) and g( x ) are non-differentiable at 5 points (d) cannot be differentiable at x = x 0, if f ( x 0 ) = g( x 0 )
(b) f ( x ) is not differentiable at 5 points whether g( x ) is 48. Number of points of non-differentiability of the function
non-differentiable at 7 points g ( x ) = [x 2 ]{cos 2 4 x } + {x 2 }[cos 2 4 x ] + x 2 sin 2 4 x
(c) number of points of non-differentiability for f ( x ) and g( x )
are 7 and 5 points, respectively +[x 2 ][cos 2 4 x ] + {x 2 } {cos 2 4 x } in ( -50, 50),
(d) both f ( x ) and g( x ) are non-differentiable at 3 and 5 points, where [ × ] and { × } are greatest integer function and
respectively fractional part of x, is equal to
(a) 98
ì3x 2 - 4 x + 1, for x < 1
43. Let g ( x ) = í . If g ( x ) is the (b) 99
î ax + b, for x ³ 1 (c) 100
(d) 0
continuous and differentiable for all numbers in its
{ x } g( x ) 1
domain, then 49. If f ( x ) = is a periodic function with period ,
(a) a = b = 4 (b) a = b = - 4 { x } g( x ) 4
(c) a = 4 and b = - 4 (d) a = - 4 and b = 4 where g ( x ) is a differentiable function, then (where { × }
denotes fractional part of x).
44. Let f ( x ) be continuous and differentiable function for all
æ1 5ö
reals and f ( x + y ) = f ( x ) - 3xy + f (y ). (a) g ¢( x ) has exactly three roots in ç , ÷
è 4 4ø
f (h )
If lim = 7, then the value of f ¢ ( x ) is k
(b) g ( x ) = 0 at x = , where k Î I
h ®0 h
4
(a) - 3x (b) 7 (c) g( x ) must be non-zero function
(c) - 3 x + 7 (d) 2 f ( x ) + 7 (d) g( x ) must be periodic function
45. Let [x ] be the greatest integer function and æx ö f (x )
1 50. If f ç ÷ = for all x , y Î R, y ¹ 0 and f ¢ ( x ) exists for
sin p[x ] èy ø f (y )
f (x ) = 4 . Then, which one of the following does all x , f (2) = 4. Then, f (5) is
[x ] (a) 3 (b) 5
not hold good? (c) 25 (d) None of these
370 Textbook of Differential Calculus

Continuity and Differentiability Exercise 2 :


More than One Option Correct Type Questions
x 55. Let [x ] be the greatest integer function, then
51. Indicate the correct alternative, if f ( x ) = - 1 , then on
2 1
sin p [x ]
the interval [0, p ] , 4
1 f (x ) = is
(a) tan( f ( x )) and are both continuous [x ]
f (x ) 3
1 (a) not continuous at any point (b) continuous at x =
(b) tan( f ( x )) and are both discontinuous 2
f (x ) 4
(c) discontinuous at x = 2 (d) differentiable at x =
(c) tan( f ( x )) and f -1( x ) are both continuous 3
1
(d) tan( f ( x )) is continuous but is not continuous ì æ1ö
(sin -1 x ) 2 cos ç ÷, x ¹ 0
f (x ) 56. If f ( x ) = ïí èx ø , then f ( x ) is
52. On the interval I = [-2, 2], if the function ïî 0, x =0
ì -ç
æ 1 1ö
+ ÷ (a) continuous nowhere in - 1 £ x £ 1
ï è|x | x ø
f ( x ) = í( x + 1) e , x ¹ 0 then which of the (b) continuous everywhere in - 1 £ x £ 1
ï x =0 (c) differentiable nowhere in - 1 £ x £ 1
î 0 ,
following hold good? (d) differentiable everywhere in - 1 £ x £ 1
(a) f ( x ) is continuous for all values of x Î I 57. Let f ( x ) = cos x and
(b) f ( x ) is continuous for x Î I - { 0 }
ì min ( f (t ) : 0 £ t < x ) , for 0 £ x £ p / 2
(c) f ( x ) assumes all intermediate values from f ( -2 ) to f (2 ) ï
H(x ) = í p p , then
(d) f ( x ) has a maximum value equal to 3/e - x, for < x £ 3
ïî 2 2
ì é -1 æ 2x 3 - 3 ö ù
ï3 - ê cot ç ÷ ú , for x > 0 (a) H ( x ) is continuous and derivable in [ 0, 3 ]
53. If f ( x ) = í ê è x 2 ø úû , where {x } and p
ë (b) H ( x ) is continuous but not derivable at x =
ï 2
2 1/ x
î { x } cos (e ), for x < 0 p
(c) H ( x ) is neither continuous nor derivable at x =
[x ] denote fractional part and the greatest integer 2
function respectively, then which of the following (d) maximum value of H ( x ) in [ 0, 3 ] is 1
statements does not hold good?
58. If f ( x ) = 3 (2x + 3) 2 / 3 + 2x + 3, then
(a) f (0 - ) = 0
3
(b) f ( 0 + ) = 0 (a) f ( x ) is continuous but not differentiable at x = -
2
(c) f ( 0 ) = 0 Þ Continuous at x = 0
(b) f ( x ) is differentiable at x = 0
(d) Irremovable discontinuity at x = 0
(c) f ( x ) is continuous at x = 0
ìb ([x ]2 + [x ]) + 1 , for x > - 1 3
54. If f ( x ) = í , where [x ] (d) f ( x ) is differentiable but not continuous at x = -
î sin( p ( x + a )), for x < - 1 2
denotes the integral part of x, then for what values of a ì x × ln (cos x )
, x ¹0
and b, the function is continuous at x = - 1? 59. If f ( x ) = ïí ln (1 + x 2 ) , then
3 ïî 0, x =0
(a) a = 2n + ; b Î R, n Î I
2
(a) f is continuous at x = 0
(b) a = 4n + 2 ; b Î R, n Î I
(b) f is continuous at x = 0 but not differentiable at x = 0
3
(c) a = 4n + ; b Î R + , n Î I (c) f is differentiable at x = 0
2
(d) f is not continuous at x = 0
(d) a = 4n + 1; b Î R + , n Î I
Chap 06 Continuity and Differentiability 371

60. Let [x ] denotes the greatest integer less than or 66. If f ( x ) = sec 2x + cosec 2x , then f ( x ) is discontinuous at
equal to x. If f ( x ) = [x sin px ] , then f ( x ) is all points in
(a) continuous at x = 0 ì p ü
(a) {np , n Î N } (b) í(2n ± 1 ) , n Î I ý
(b) continuous in ( - 1, 0 ) î 4 þ
(c) differentiable at x = 1 ì np ü ì p ü
(d) differentiable in ( - 1, 1 ) (c) í , n Î I ý (d) í(2n ± 1 ) , n Î I ý
î 4 þ î 8 þ
61. The function f ( x ) = [| x | ] - |[x ]|, where [x ] denotes ì æ 1 ö
x n sin ç 2 ÷ , x ¹ 0
greatest integer function 67. Let f ( x ) = ïí èx ø , (n Î I ), then
(a) is continuous for all positive integers ïî 0, x =0
(b) is discontinuous for all non-positive integers
(a) lim f ( x ) exists for every n > 1
(c) has finite number of elements in its range x®0

(d) is such that its graph does not lie above the X-axis (b) f is continuous at x = 0 for n > 1
62. The function f ( x ) = 1 - 1 - x 2 (c) f is differentiable at x = 0 for every n > 1
(d) None of the above
(a) has its domain - 1 £ x £ 1
ì ex, x£0
(b) has finite one sided derivates at the point x = 0 68. A function is defined as f (x ) = í , then f ( x ) is
(c) is continuous and differentiable at x = 0 î| x - 1 | x > 0
,
(d) is continuous but not differentiable at x = 0 (a) continuous at x = 0 (b) continuous at x = 1
63. Consider the function f ( x ) = | x + 1 |. Then, 3 (c) differentiable at x = 0 (d) differentiable at x = 1
x
(a) domain of f ( x ) Î R 69. Let f ( x ) = ò | t + 1| dt , then
-2
(b) range of f is R +
(a) f ( x ) is continuous in [ - 1, 1 ]
(c) f has no inverse (b) f ( x ) is differentiable in [ - 1, 1 ]
(d) f is continuous and differentiable for every x Î R (c) f ¢( x ) is continuous in [ - 1, 1 ]
64. f is a continuous function in [a, b ] , g is a continuous (d) f ¢( x ) is differentiable in [ - 1, 1 ]
function in [b, c ]. A function h ( x ) is defined as 70. A function f ( x ) satisfies the relation
ì f ( x ) for x Î [a, b ) f ( x + y ) = f ( x ) + f (y ) + xy ( x + y ), " x , y Î R . If
h( x ) = í . If f (b ) = g (b ), then
î g ( x ) for x Î (b, c ] f ¢ (0) = - 1, then
(a) f ( x ) is a polynomial function
(a) h( x ) has a removable discontinuity at x = b
(b) f ( x ) is an exponential function
(b) h( x ) may or may not be continuous in [a, c ]
(c) f ( x ) is twice differentiable for all x Î R
(c) h (b - ) = g(b + ) and h (b + ) = f (b - )
(d) f ¢(3 ) = 8
(d) h (b + ) = g(b - ) and h (b - ) = f (b + )
ì3x 2 + 12x - 1 , -1 £ x £ 2
65. Which of the following function(s) has/have the same 71. If f ( x ) = í , then
î 37 - x , 2< x £3
range?
1 (a) f ( x ) is increasing on [ - 1, 2 ]
(a) f ( x ) =
1+x (b) f ( x ) is continuous on [ - 1, 3 ]
1 (c) f ¢(2 ) doesn’t exist
(b) f ( x ) =
1 + x2 (d) f ( x ) has the maximum value at x = 2
1
(c) f ( x ) = 72. If f ( x ) = 0 for x < 0 and f ( x ) is differentiable at x = 0,
1+ x
then for x > 0, f ( x ) may be
1
(d) f ( x ) = (a) x 2 (b) x (c) - x (d) - x 3/2
3-x
372 Textbook of Differential Calculus

Continuity and Differentiability Exercise 3 :


Statements I and II Type Questions
æ e 2 x - e -2 x ö
n Directions (Q. Nos. 73 to 82) For the following 77. Statement I Range of f ( x ) = x ç -2 x
÷ + x2 + x4
questions, choose the correct answer from the codes (a), is not R. èe + e
2x
ø
(b), (c) and (d) defined as follows : Statement II Range of a continuous even function
(a) Statement I is correct, Statement II is also correct;
cannot be R.
Statement II is the correct explanation of Statement I
(b) Statement I is correct, Statement II is also correct; 78. Let h( x ) = f 1 ( x ) + f 2 ( x ) + f 3 ( x ) + K + f n ( x ), where
Statement II is not the correct explanation of Statement I f 1 ( x ), f 2 ( x ), f 3 ( x ), K , f n ( x ) are real valued functions
(c) Statement I is correct, Statement II is incorrect of x.
(d) Statement I is incorrect, Statement II is correct
Statement I f ( x ) = | cos | x || + cos -1 (sgn x ) + | ln x | is
73. Statement I f ( x ) = sin x + [x ] is discontinuous at x = 0. not differentiable at 3 points in (0, 2p ).
Statement II If g ( x ) is continuous and f ( x ) is Statement II Exactly one function , isf i ( x ), i = 1, 2, K , n
discontinuous, then g ( x ) + f ( x ) will necessarily be is not differentiable and the rest of the function is
discontinuous at x = a . differentiable at x = a makes h( x ) not differentiable at
ì2 sin(a cos -1 x ), if x Î (0, 1) x = a.
ï 79. Statement I f ( x ) = | x | sin x is differentiable at x = 0.
74. Consider f ( x ) = í 3, if x = 0
ï ax + b, if x < 0 Statement II If g ( x ) is not differentiable at x = a and
î
2 h ( x ) is differentiable at x = a, then g ( x ) × h( x ) cannot be
Statement I If b = 3 and a = , then f ( x ) is continuous differentiable at x = a.
in ( -¥, 1). 3
p
Statement II If a function is defined on an interval I and 80. Statement I f ( x ) = | cos x | is not derivable at x = .
2
limit exists at every point of interval I, then function is
continuous in I. Statement II If g ( x ) is differentiable at x = a
and g (a ) = 0, then | g ( x )| is non-derivable at x = a .
ì cos x - e x 2
/2

75. Let f ( x ) = ïí , x ¹0 81. Let f ( x ) = x - x 2 and g ( x ) = {x }, " x Î R,


x3
ï x = 0, then where { } denotes fractional part function.
î 0,
Statement I f ( g ( x )) will be continuous, " x Î R.
Statement I f ( x ) is continuous at x = 0.
2 Statement II f (0) = f (1) and g ( x ) is periodic with
cos x - e - x /2
1 period 1.
Statement II lim =-
x ®0 x3 12 ì - ax 2 - b | x | - c , - a £ x < 0
3 82. Let f ( x ) = í , where a, b, c
î ax + b | x | + c , 0 £ x £ a
x 2 2
76. Statement I The equation - sin px + = 0 has
4 3 are positive and a > 0, then
atleast one solution in [- 2, 2].
Statement I The equation f ( x ) = 0 has at least one real
Statement II Let f : [a, b ] ® R be a function and c be a root for x Î [- a , a ].
number such that f (a ) < c < f (b ), then there is at least
one number n Î(a, b ) such that f (n ) = c . Statement II Values of f ( - a ) and f (a ) are opposite
in sign.
Chap 06 Continuity and Differentiability 373

Continuity and Differentiability Exercise 4 :


Passage Based Questions
Passage I (Q. Nos. 83 to 85) 87. The range of the function D = f (| x | ) is
A man leaves his home early in the morning to have a walk. He (a) [ 0, 1 ] (b) [ 0, 1 )
arrives at a junction of roads A and B as shown in figure. He takes (c) ( 0, 1 ] (d) None of these
the following steps in later journies : 88. The function y = f ( x ) is
N
Road B (a) odd (b) even
W E (c) increasing (d) decreasing
S 89. If h( x ) = f ¢ ( x ), then h( x ) is given by
1
(a) - f ( x ) (b) (c) f ( x ) (d) e f ( x )
Road A f (x )

Home
Passage III (Q. Nos. 90 to 92)
(i) 1 km in North direction. Let y = f ( x ) be defined in [a, b ] , then
(ii) Changes direction and moves in North-East direction for (i) Test of continuity at x = c , a < c < b
2 2 km. (ii) Test of continuity at x = a
(iii) Changes direction and moves Southwards for distance of (iii) Test of continuity at x = b
2 km. Case I Test of continuity at x = c, a < c < b
(iv) Finally, he changes the direction and moves in If y = f ( x ) be defined at x = c and its value f (c ) be equal to
South-East direction to reach road A again.
limit of f ( x ) as x ® c , i.e. f (c ) = lim f ( x )
Visible/invisible path The path traced by the man in the x ®c
or lim f ( x ) = f (c ) = lim f ( x )
direction parallel to road A and road B is called invisible path, the x ® c- x ® c+
remaining path is called visible.
or LHL = f (c ) = RHL
Visible points The point about which the man changes direction Then, y = f ( x ) is continuous at x = c.
are called visible points, except the point from where he changes
Case II Test of continuity at x = a
direction last time.
If RHL = f (a )
Now, if roads A and B are taken as X -axis and Y-axis, then visible Then, f ( x ) is said to be continuous at the end point x = a.
point representing the graph of y = f ( x ).
Case III Test of continuity at x = b, if LHL = f (b )
83 The value of x at which the function is discontinuous, is Then, f ( x ) is continuous at right end x = b.
(a) 2 (b) 0 (c) 1 (d) 3
ì sin x , x £0
84. The value of x for which fof is discontinuous, is ï tan x , 0 < x < 2p
(a) 0 (b) 1 (c) 2 (d) 3 90. If f ( x ) = ïí , then f ( x ) is discontinuous
ï cos x , 2p £ x < 3p
85. If f ( x ) is periodic with period 3, then f (19 ) is ïî 3 p , x = 3p
(a) 2 (b) 3 at
(c) 19 (d) None of these p 3p p 3p
(a) , , 2p (b) 0, , p, , 3p
Passage II (Q. Nos. 86 to 89) 2 2 2 2
p
Let f be a function that is differentiable everywhere and that has (c) , 2 p (d) None of these
2
the following properties :
(i) f ( x ) > 0 (ii) f ¢ (0) = - 1 91. Number of points of discontinuity of [2x 3 - 5] in [1, 2) is
1 (where [ × ] denotes the greatest integral function)
(iii) f ( - x ) = and f ( x + h ) = f ( x ) × f (h )
f (x ) (a) 14 (b) 13
f ¢( x ) (c) 10 (d) None of these
A standard result is dx = log| f ( x )| + C .
f (x ) 92. Max ([x ], | x | ) is discontinuous at
(a) x = 0
86. Range of f ( x ) is (b) f
(a) R (b) R - { 0 } (c) x = n, n Î I
(c) R + (d) ( 0, e ) (d) None of the above
374 Textbook of Differential Calculus

Passage IV (Q. Nos. 93 to 95) Passage V (Q. Nos. 96 to 99)


f ( x ) = cos x and H 1 ( x ) = min { f (t ), 0 £ t < x }; Let f ( x ) be a real valued function not identically zero, which
p p p satisfied the following conditions
0 £ x £ = - x; < x £ p
2 2 2 I. f ( x + y 2n +1 ) = f ( x ) + ( f (y )) 2n +1 , n Î N , x , y are any real
f ( x ) = cos x and H 2 ( x ) = max { f (t ), 0 £ t £ x }; numbers.
p p p II. f ¢ (0) ³ 0
0£ x £ = - x; < x £ p
2 2 2
96. The value of f (1) is
g ( x ) = sin x and H 3 ( x ) = min { g (t ), 0 £ t £ x };
(a) 0 (b) 1
p p p (c) 2 (d) Not defined
0 £ x £ = - x; < x £ p
2 2 2 97. The value of f ( x ) is
g ( x ) = sin x and H 4 ( x ) = max { g (t ), 0 £ t £ x }; (a) 2x (b) x 2 + x + 1
p p p (c) x (d) None of these
0 £ x £ = - x; < x £ p
2 2 2 98. The value of f ¢ (10) is
93. Which of the following is true for H 2 ( x ) ? (a) 10 (b) 0 (c) 2n + 1 (d) 1
(a) Continuous and derivable in [ 0, p ] 99. The function f ( x ) is
p
(b) Continuous but not derivable at x = (a) odd
2
(b) even
p
(c) Neither continuous nor derivable at x = (c) neither even nor odd
2
(d) both even as well as odd
(d) None of the above
94. Which of the following is true for H 3 ( x )? Passage VI (Q. Nos. 100 to 101)
(a) Continuous and derivable in [ 0, p ] If f : R ® (0, ¥ ) is a differentiable function f ( x ) satisfying
p f ( x + y ) - f ( x - y ) = f ( x ) × { f (y ) - f ( -y )}, " x , y Î R,
(b) Continuous but not derivable at x =
2 (f (y ) ¹ f ( -y ) for all y Î R) and f ¢(0) = 2010. Now, answer the
p following questions
(c) Neither continuous nor derivable at x =
2
(d) None of the above 100. Which of the following is true for f ( x ) ?
(a) f ( x ) is one-one and into
95. Which of the following is true for H 4 ( x )?
(b) { f ( x )} is non-periodic, where { × } denotes fractional part of x
(a) Continuous and derivable in [ 0, p ] (c) f ( x ) = 4 has only two solutions
p
(b) Continuous but not derivable at x = (d) f ( x ) = f ¢( x ) has only one solution
2
p f ¢( x )
(c) Neither continuous nor derivable at x = 101. The value of is
2 f (x )
(d) None of the above (a) 2016 (b) 2014 (c) 2012 (d) 2010
Chap 06 Continuity and Differentiability 375

Continuity and Differentiability Exercise 5 :


Matching Type Questions
102. Match the column.
(B) The number of points where the (q) 0
Column I Column II
function
ì sin{x}; x<1 f (x) = min {1, 1 + x3 , x2 - 3x + 3} is
(A) If f (x) = í , where {x} (p) 1
îcos x + a; x ³ 1 non-derivable, is
denotes the fractional part function, such (C) The number of points where (r) 4
that f (x) is continuous at x = 1. If f (x) = (x + 4)1/ 3 is non-differentiable,
a
|K | = , then K is is
(4 - p )
2 sin
4 (D) Consider (s) 1
ì -p æ 2x ö + p , 0 < x £ p
1 - cos(sin x) (q) 0 ï log ç ÷
(B) If the function f (x) = is f (x) = í 2 èpø 2 2
x2 ï sin -1 (sin x), p 3p
<x£
continuous at x = 0, then f (0) is î 2 2
-1 æ 3p ö
ì x, x Îq (r) Number of points in ç 0, ÷, where
(C) If f (x) = í , then the values of x è 2 ø
î1 - x, x Ï q f (x) is non-differentiable, is
at which f (x) is continuous, is

(D) If f (x) = x + {- x} + [ x] , where [ x] and {x} (s) 1 105. Match the entries of the following two columns.
represent greatest integer and fractional part 2
Column I Column II
of x, then the values of x at which f (x) is
é x + 1, if x < 0
discontinuous, is (A) f (x) = ê at (p) continuous
ë cos x, if x ³ 0
103. Match the column. x = 0 is
Column I Column II (B) For every x Î R, the function (q) differentiability
sin(p[ x - p ])
(A) Number of points where the function (p) 0 g (x) = , where [ x]
ì é px ù 1 + [ x]2
ï1 + êë cos 2 úû , 1 < x £ 2
ï denotes the greatest integer
f (x) = í 1 - {x}, 0 £ x < 1 and f (1) = 0 function, is
ï |sin px |, -1 £ x < 0 (r) discontinuous
ï (C) h(x) = {x}2 , where {x}
î
denotes fractional part
is continuous but non-differentiable, where [ × ]
function for all x Î I , is
denotes greatest integer function and {×}
denotes fractional part of x, is ì 1
ï ln x
(D) k (x) = íx , if x ¹ 1 at (s) non-derivable
ìx2e1/ x , x ¹ 0
(B) If f (x) = í , then f (0- ) is ï e, if x = 1
(q) 1 î
î 0, x=0
x = 1 is
(C) The number of points at which (r) 2
1 106. Match the entries of the following two columns.
f (x) = is not differentiable,
1 + 2 / f (x)
Column I Column II
1
where f (x) = , is æ
1 + 1/ x x4 + 1 2
+ 1) ö (p) e
(A) lim ç e - e(x ÷ is
x® ¥ è ø
(D) Number of points where tangent does not (s) 3
(B) For a > 0, let
exist for the curve y = sgn(x2 - 1), is (q) e2
é ax + a- x - 2
f (x) = ê , if x > 0.
ê x2
104. Match the column. ë 3 ln(a - x) - 2, if x £ 0
Column I Column II If f is continuous at x = 0, then a equals
(A) The number of values of x in (0, 2p ), (p) 2 xx - aa
(C) Let L = lim and (r) non-
where the function x® a x- a existent
tan x + cot x tan x - cot x xx - ax
f (x) = - is M = lim (a > 0).
2 2 x® a x- a
continuous but not differentiable, is If L = 2 M , then the value of a is equal to
376 Textbook of Differential Calculus

Continuity and Differentiability Exercise 6 :


Single Integer Answer Type Questions
107. Number of points of discontinuity of ì æ 3 ö 1/ x æ 3 ö -1/ x
f ( x ) = tan x - sec x in (0, 2p ) is ……… .
2 2 ï ç ÷ -ç ÷
ïx è 4 ø è4ø
,x ¹0
108. Number of point(s) of discontinuity of the function 114. Let f ( x ) = ïí æ 3 ö 1/ x æ 3 ö -1/ x
f ( x ) = [x 1/ x ], x > 0, (where [ ] denotes the greatest ï çè ÷ø + çè ÷ø
ï 4 4
integral function) is ……… . ïî 0, x =0
- +
109. Let f ( x ) = x + cos x + 2 and g ( x ) be the inverse If P = f ¢ (0 ) - f ¢ (0 ), then
function of f ( x ) , then g ¢ (3) equals to ……… . æ [ x +1] ö
ç (exp(( x + 2) log 4 )) 4 - 16 ÷
110. Let f ( x ) = x tan -1 ( x 2 ) + x 4 . Let f k ( x ) denotes kth 4 ç lim ÷ is …… .
ç x ®P - 4 x - 16 ÷
derivative of f ( x ) w. r. t. x, k Î N . è ø
If f 2m
(0) ¹ 0, m Î N , then m equals to ……… . (where [x ] denotes greatest integer function.)
111. Let f 1 ( x ) and f 2 ( x ) be twice differentiable functions, 115. Let f ( x ) = - x 3 + x 2 - x + 1 and
where F ( x ) = f 1 ( x ) + f 2 ( x ) and ì min ( f (t )), 0 £ t £ x and 0 £ x £ 1
g (x ) = í
G( x ) = f 1 ( x ) - f 2 ( x ), " x Î R, f 1 (0) = 2 î x - 1, 1< x £2
and f 2 (0) = 1. If f 1¢ ( x ) = f 2 ( x ) and Then, the value of lim g ( g ( x )) is…… .
x ®1
f 2¢ ( x ) = f 1 ( x ) , " x Î R, then the number of
116. The number of points at which the function
9x 4 f ( x ) = ( x -| x | ) 2 (1 - x + | x | ) 2 is not differentiable in the
solutions of the equation ( F ( x )) 2 = is ……… .
G( x ) interval ( -3, 4 ) is …… .
112. Suppose the function f ( x ) - f (2x ) has the derivative ìp -1 -1
ï 2 - sin (1 - {x } )sin (1 - {x })
2

5 at x = 1 and derivative 7 at x = 2. The derivative of ï ,x >0


the function f ( x ) - f ( 4 x ) - 10x at x = 1 is equal to ï 2({x } - {x } 3 )
……… . 117. If f ( x ) = ïí k, x =0
ï -1 -1
113. In a DABC, angles A, B, C are in AP. ï A sin (1 - {x }) cos (1 - {x }) , x < 0
ï 2 {x }(1 - {x })
3 - 4 sin A sin C ï
If f ( x ) = lim , then f ¢ ( x ) is equal î
A ®C | A - C| æ Ap ö
is continuous at x = 0, then the value of sin 2 k + cos 2 ç ÷,
to ……… . è 2ø
is…… (where { × } denotes fractional part of x).

Continuity and Differentiability Exercise 7 :


Subjective Type Questions
118. Discuss the continuity of the function 120. Examine the continuity and differentiability at points x = 1
f ( x ) = [[x ]]– [x – 1], where [ ] denotes the greatest and x = 2.
integral function.
The function f defined by
119. Examine the continuity or discontinuity of the ì x [x ], 0 £ x < 2
following : f (x ) = í (where [ × ] denotes the greatest
x 2n – 1 î( x – 1) [x ], 2 £ x £ 3
(i) f ( x ) = [x ] + [– x ] (ii) g ( x ) = lim
n ®¥ x 2n + 1 integral function less than or equal to x).
Chap 06 Continuity and Differentiability 377

121. Let f be twice differentiable function, such that 123. A function f : R ® R, where R is a set of real numbers,
æ x + y ö f ( x ) + f (y ) + f (0)
f ¢¢ ( x ) = - f ( x ) and f ¢ ( x ) = g ( x ), satisfying the equation f ç ÷=
è 3 ø 3
h( x ) = [ f ( x )]2 + [g ( x )]2 . Find h(10), if h(5) = 11.
for all x , y in R. If the function is differentiable at x = 0,
122. A function f : R ® R satisfies the equation then show that it is differentiable for all x in R.
f ( x + y ) = f ( x ) × f (y ) for all x , y in R and f ( x ) ¹ 0 for any 124. Let f ( x + y ) = f ( x ) + f (y ) + 2xy – 1 for all real x, y and
x in R. Let the function be differentiable at x = 0 and f ( x ) be differentiable functions. If f ¢ (0) = cos a, then
f ¢ (0) = 2. Show that f ¢( x ) = 2 f ( x ) for all x in R. Hence, prove that f ( x ) > 0, " x Î R.
determine f ( x ).

Continuity and Differentiability Exercise 8 :


Questions Asked in Previous 10 Years’ Exams
(i) JEE Advanced & IIT-JEE
125. For every pair of continuous function f , g : [0, 1] ® R (c) an - bn + 1 = 1 (d) an - 1 - bn = -1
such that max { f ( x ): x Î [0, 1]} = max {g ( x ): x Î [0, 1]}. The 129. Let f : R ® R be a function such that
correct statement(s) is/are f ( x + y ) = f ( x ) + f (y ), "x , y Î R. If f ( x ) is differentiable
[More than One Correct Option, 2014] at x = 0, then [More than One Correct Option, 2011]
(a) [ f (c )]2 + 3 f (c ) = [ g(c )]2 + 3 g(c ) for some c Î[ 0, 1 ] (a) f ( x ) is differentiable only in a finite interval containing zero
(b) [ f (c )]2 + f (c ) = [ g(c )]2 + 3 g(c ) for some c Î[ 0, 1 ] (b) f ( x ) is continuous for all x Î R
(c) [ f (c )]2 + 3 f (c ) = [ g(c )]2 + g(c ) for some c Î[ 0, 1 ] (c) f ¢( x ) is constant for all x Î R
(d) f ( x ) is differentiable except at finitely many points
(d) [ f (c )]2 = [ g(c )]2 for some c Î[ 0, 1 ]
ì p p
126. Let f : R ® R and g : R ® R be respectively given by ï -x - 2, x £- 2
ï p
f ( x ) =| x | + 1 and g ( x ) = x 2 + 1. Define h : R ® R by 130. If f ( x ) = ïí - cos x , - < x £ 0, then
ï 2
ì max{ f ( x ), g ( x )}, if x £ 0
h( x ) = í . ï x - 1, 0< x £1
î min{ f ( x ), g ( x )}, if x > 0 ïî ln x , x >1
The number of points at which h( x ) is not differentiable [More than One Correct Option, 2011]
is p
[Integer Answer Type, 2014] (a) f ( x ) is continuous at x = -
2
ì 2 p
, x ¹ 0 x Î R, (b) f ( x ) is not differentiable at x = 0
127. Let f ( x ) = ïí
x cos
x (c) f ( x ) is differentiable at x = 1
ï 0, 3
î x = 0, then f is (d) f ( x ) is differentiable at x = -
[One Correct Option, 2012] 2
1
(a) differentiable both at x = 0 and at x = 2 131. For the function f ( x ) = x cos , x ³ 1,
(b) differentiable at x = 0 but not differentiable at x = 2 x
[More than One Correct Option, 2009]
(c) not differentiable at x = 0 but differentiable at x = 2
(a) for at least one x in the interval [1, ¥ ), f ( x + 2 ) - f ( x ) < 2
(d) differentiable neither at x = 0 nor at x = 2
(b) lim f ¢ ( x ) = 1
x®¥
128. For every integer n, let an and bn be real numbers. Let
(c) for all x in the interval [1, ¥ ), f ( x + 2 ) - f ( x ) > 2
function f : R ® R be given by (d) f ¢( x ) is strictly decreasing in the interval [1, ¥ )
ì an + sin px , for x Î [2n, 2n + 1] ( x - 1)n
f (x ) = í , 132. Let g ( x ) = ; 0 < x < 2, m and n are
îbn + cos px , for x Î (2n - 1, 2n ) log cos m ( x - 1)
for all integers n. integers, m ¹ 0, n > 0 and let p be the left hand
If f is continuous, then which of the following hold(s) derivative of | x - 1| at x = 1 . If lim g ( x ) = p , then
for all n ? [More than One Correct Option, 2012] x ®1+
[One Correct Option, 2008]
(a) an -1 - bn -1 = 0 (b) an - bn = 1
(a) n = 1 , m = 1 (b) n = 1 , m = -1
378 Textbook of Differential Calculus

(c) n = 2 , m = 2 (d) n > 2, m = n (d) f ( x ) is not differentiable at one point


133. Let f and g be real valued functions defined on interval 136. Let f ( x ) = || x | - 1 |, then points where, f ( x ) is not
( -1, 1) such that g ¢¢ ( x ) is continuous, g (0) ¹ 0, g ¢ (0) = 0, differentiable is/are [One Correct Option, 2005]
g ¢¢(0) ¹ 0 and f ( x ) = g ( x )sin x . (a) 0, ± 1 (b) ± 1
Statement I lim [g ( x ) cos x - g (0)] cosec x = f ¢¢(0). (c) 0 (d) 1
x ®0
137. If f is a differentiable function satisfying f æç ö÷ = 0, "
1
Statement II f ¢ (0) = g (0).
è ø n
For the above question, choose the correct answer from
the codes (a), (b), (c) and (d) defined as follows. n ³ 1, n Î I , then [One Correct Option, 2005]
[Assertion and Reason, 2008] (a) f ( x ) = 0, x Î ( 0, 1 ]
(a) Statement I is true, Statement II is also true; Statement II (b) f ¢( 0 ) = 0 = f ( 0 )
is the correct explanation of Statement I (c) f ( 0 ) = 0 but f ¢( 0 ) not necessarily zero
(b) Statement I is true, Statement II is also true; Statement II (d) | f ( x ) | £ 1, x Î ( 0, 1 ]
is not the correct explanation of Statement I 138. The domain of the derivative of the functions
(c) Statement I is true; Statement II is false ì tan -1 x , if | x | £ 1
ï
(d) Statement I is false; Statement II is true f ( x ) = í1 is
134. In the following, [x] denotes the greatest integer less ïî 2 (| x | - 1), if | x | > 1
[One Correct Option, 2002]
than or equal to x. [Matching type Question, 2007]
(a) R - { 0 } (b) R - {1 }
Column I Column II (c) R - { -1 } (d) R - { -1, 1 }
A. x| x| p. continuous in (– 1, 1) 139. The left hand derivative of f ( x ) = [x ]sin ( p x ) at
B. | x| q. differentiable in (– 1, 1) x = k , k is an integer, is [One Correct Option, 2001]
C. x + [ x] r. strictly increasing in (– 1, 1) (a) ( -1 )k (k - 1 ) p
D. | x - 1| +| x + 1| s. not differentiable atleast at (b) ( -1 )k - 1 (k - 1 ) p
one point in (– 1, 1)
(c) ( -1 )k kp
135. If f ( x ) = min { 1, x 2 , x 3 }, then (d) ( -1 )k - 1 kp
[More than One Correct, 2006 ] 140. Which of the following functions is differentiable
(a) f ( x ) is continuous everywhere at x = 0 ? [One Correct Option, 2001]
(b) f ( x ) is continuous and differentiable everywhere (a) cos (| x | ) + | x | (b) cos (| x | ) - | x |
(c) f ( x ) is not differentiable at two points (c) sin (| x | ) + | x | (d) sin (| x | ) - | x |

(ii) JEE Main & AIEEE


141. For x Î R, f ( x ) = | log 2 - sin x | and g ( x ) = f ( f ( x )), then 143. If f and g are differentiable functions in (0, 1) satisfying
(a) g is not differentiable at x = 0 [2016, JEE Main] f (0) = 2 = g (1), g(0) = 0 and f (1) = 6, then for some
(b) g ¢( 0 ) = cos (log 2 ) c Î]0, 1 [ [2014, JEE Main]

(c) g ¢( 0 ) = - cos (log 2 ) (a) 2 f ¢(c ) = g ¢(c ) (b) 2 f ¢(c ) = 3 g ¢(c )


(c) f ¢(c ) = g ¢(c ) (d) f ¢(c ) = 2 g ¢(c )
(d) g is differentiable at x = 0 and g ¢( 0 ) = - sin (log 2 )
144. If f : R ® R is a function defined by
ìk x + 1, 0 £ x £ 3 æ 2x - 1 ö
142. If the function g ( x ) = í is f ( x ) = [x ] cos ç ÷ p, where [x ] denotes the greatest
î mx + 2, 3 < x £ 5 è 2 ø
differentiable, then the value of k + m is [2015, JEE Main] integer function, then f is [2012 AIEEE]
16 (a) continuous for every real x
(a) 2 (b)
5 (b) discontinuous only at x = 0
10 (c) discontinuous only at non-zero integral values of x
(c) (d) 4
3 (d) continuous only at x = 0
Answers
Exercise for Session 1 13. (a) 14. (b) 15. (a) 16. (c) 17. (c) 18. (c)
1. (c) 2. (c) 3. (a) 4. (a) 5. (a) 19. (c) 20. (b) 21. (b) 22. (b) 23. (d) 24. (b)
25. (d) 26. (c) 27. (c) 28. (c) 29. (d) 30. (c)
Exercise for Session 2 31. (b) 32. (c) 33. (d) 34. (d) 35. (d) 36. (a)
1. a = - 1, b = 1 2. f is continuous in -1 £ x £ 1. 37. (d) 38. (d) 39. (c) 40. (a) 41. (b) 42. (b)
1
3. f (x) is continuous everywhere in [ 0, 2] except for x = , 1 and 2. 43. (c) 44. (c) 45. (c) 46. (a) 47. (c) 48. (d)
2 49. (b) 50. (c) 51. (c,d) 52. (b,c,d) 53. (a,b,c) 54. (a,c)
4. (d) 55. (b,c,d) 56. (b,d) 57. (a,d) 58. (a,b,c) 59. (a,c) 60. (a, b, c, d)
61. (a, b, c, d) 62. (a, b, d) 63. (a, c) 64. (a,c)
Exercise for Session 3 65. (b, c) 66. (a, b, c) 67. (a, b, c)
1. (b,c,d) 2. (a,c,d) 3. (d) 4. (c) 5. (a,c,d) 68. (a, b) 69. (a, b, c, d) 70. (a, c, d)
71. (a, b, c, d) 72. (a,d) 73. (a) 74. (c)
Exercise for Session 4 75. (a) 76. (c) 77. (a) 78. (a) 79. (c) 80. (c)
1. (a,b,c) 2. (b,c) 3. (c,d) 4. (b,c) 5. (a,b,c) 6. (a) 7. (b) 81. (a) 82. (d) 83. (a) 84. (b,c) 85. (a) 86. (c)
87. (a) 88. (d) 89. (a) 90. (a) 91. (b) 92. (b)
Exercise for Session 5 93. (c) 94. (b) 95. (c) 96. (b) 97. (c) 98. (d)
99. (a) 100. (b) 101. (d)
1. discontinuity at x = 0 102. (A) ® (p, r ); (B) ® (s); (C) ® (s); (D) ® (p, q, r )
4. yn (x) is continuous at x = 0 for all n and y(x) is discontinuous
103. (A) ® (q); (B) ® (p); (C) ® (s); (D) ® (p)
at x = 0.
104. (A) ® (r); (B) ® (p); (C) ® (s); (D) ® (q)
Exercise for Session 6 105. (A) ® (p, s); (B) ® (p, q); (C) ® (r, s); (D) ® (p, q)
1. (d) 2. (a) 3. (a) 4. (d) 5. (b) 106. (A) ® (r); (B) ® (p, q); (C) ® (p)
107. (2) 108. (1) 109. (1) 110. (2) 111. (2)
Exercise for Session 7 112. (9) 113. (0) 114. (2) 115. (1) 116. (0) 117. (2)
1. (c) 2. (c) 3. (c) 4. (b) 5. (b) 6. (d) 125. (a,d) 126. (3) 127. (b) 128. (b, d) 129. (b,c) 130. (a, b, c, d)
7. (a) 8. (c) 9. (a) 10. (c) 131. (b, c, d) 132. (c) 133. (b)
134. (A) ® (p, q, r); (B) ® (p, s); (C) ® (r, s); (D) ® (p, q)
Chapter Exercises
135. (a,d) 136. (a) 137. (b) 138. (d) 139. (a) 140. (d)
1. (a) 2. (c) 3. (c) 4. (c) 5. (a) 6. (c)
141. (b) 142. (a) 143. (d) 144. (a)
7. (b) 8. (b) 9. (b) 10. (b) 11. (a) 12. (c)
x +1

Solutions
6. Here, f ( x ) = , discontinuous at x =1
x -1
1
g( x ) = , discontinuous at x =2
x -2
g( x ) + 1
f ( g( x )) = , discontinuous at g( x ) =1
ì px g (x ) -1
ïsin , x < 1
1. f ( x ) = í 2 1
ïî[ x ] , x ³ 1 Þ =1
x -2
f (1 ) = 1, f (1 + ) = lim[1 + h ] = 1 Þ x =3
h®0
\ ( fog )( x ) is discontinuous at x = 2 and 3.
p æ p phö ph
f (1 - ) = lim sin (1 - h ) = lim sin ç - ÷ = lim cos = 1 ì 2é ù
h®0 2 h®0 è 2 2 ø h®0 2 1
-1ú
ïx ê
+ û = (1 + x 2 ) × é1 - ù
2 n
\ f ( x ) is continuous at x =1. ï ë (1 x )
ê
1
2 nú
,
ï 1
- ë (1 + x ) û
8h - 4h - 2h + 1 ( 4h - 1 ) (2h - 1 ) ï 1
2. f ( 0 + ) = lim 2
= lim 7. Here, yn ( x ) = í 1+ x2
h®0 h h®0 h h ï when x ¹ 0, n Î N
= (log 4 )(log 2 ) ï
f ( 0 - ) = lim(e -h sin( -h ) + p ( -h ) + k log 4 ) = k log 4 ï
h®0 ï 0, when x = 0,n ÎN
î
Since, f ( x ) is continuous at x = 0 ì(1 + x 2 ), x ¹ 0
Þ k log 4 = (log 4 )(log 2 ) = f ( 0 ). \ y ( x ) = lim yn ( x ) = í
n ®¥
î0 , x =0
\ f ( 0 ) = (log 4 )(log 2 ), when k = log2
a (1 - x sin x ) + b cos x + 5 \ y ( x ) is discontinuous at x = 0 .
-
3. f ( 0 ) = lim = f (0)
x ® 0- x2 a h - 1 - h log a
8. g( 0 - ) = lim
æ bö h®0 h2
(a + b + 5 ) + ç - a - ÷ x 2 + ¼
è 2ø é h2 ù
= lim 2
=3 ê1 + h log a + (log a ) +¼ú - 1 - h log a
2
-
x ®0 x 2 !
b = lim ë û
Þ a + b + 5 = 0 and - a - = 3 h®0 h2
2
Þ a = -1, b = - 4 é (log a ) 2 (log a ) 3 ù (log a ) 2
= lim ê ×1 + ×h ¼ú =
h®0
é æ cx + dx 3 ö ù
1/ x
ë 2! 3! û 2
Þ f ( 0 + ) = lim ê1 + ç ÷ú exists. 2h ah - h log 2 - h log a - 1
ë è x ø úû
2
x ® 0+ ê g( 0 + ) = lim
h®0 h2
cx + dx 3
Þ lim =0 Þc=0 é h2 2 ù
x ® 0+ x2 ê1 + h log(2a ) + (log(2a )) ¼ú - h log 2 - h log a -1
2!
Now, lim (1 + dx )1/x = lim ((1 + dx )1/dx )d = ed = lim ë û
x ® 0+ x ® 0+ h®0 h2
So, e = 3 Þ d = loge 3
d (log(2a )) 2
=
\ a + b + c + d = loge 3 - 5 2
Since, g( x ) is continuous.
æ p+ ö ép ù
4. f ç ÷ = lim p ê + h ú - 1 = ( p - 1) (log a ) 2 (log(2a )) 2
è 2 ø h®0 ë 2 û Þ = Þ (log a ) 2 = (log 2 + log a ) 2
2 2
æ p- ö ì æ p öü p 1
f ç ÷ = lim cos-1í cot ç - h ÷ý = lim cos-1 {tan h } = Þ log a = - log 2 = log 2 -1/2
è 2 ø h®0 î è 2 ø þ h ® 0 2 2
p p 1
\ Jump of discontinuity = ( p - 1 ) - = -1 Þ a=
2 2 2
g( x ) = f ( x ) - x æp -1 2 ö -1
5. Consider ç - sin (1 - h ) ÷ ×sin (1 - h )
+ è2 ø
g( 0 ) = f ( 0 ) - 0 = f ( 0 ) ³ 0 [Q 0 £ f ( x ) £ 1] 9. f ( 0 ) = lim
h®0 2 (h - h 3 )
g(1 ) = f (1 ) - 1 £ 0
Þ g( 0 ) × g(1 ) £ 0 (cos-1(1 - h 2 )) ×sin -1(1 - h ) p
= lim =
Þ g( x ) = 0 has at least one root in [0, 1]. h®0 2 (1 - h 2 ) ×h 2
Þ f ( x ) = x for at least one root in [0, 1].
Chap 06 Continuity and Differentiability 381

æp 2 ö f (1 + h ) - f (1 )
-1 -1
ç - sin (1 - (1 - h )) ÷ × sin (1 - (1 - h )) 13. f ¢(1 + ) = lim
- è2 ø h®0 h
f ( 0 ) = lim
h®0 2 ((1 - h ) - (1 - h ) 3 ) 1 + h - 1 ([1 + h ] - (1 + h )) h (1 - 1 - h )
= lim = 0 = lim =0
h®0 h®0
p -1 h h
sin h f (1 - h ) - f (1 ) - h ([1 - h ] - (1 - h ))
p
= lim 2 = f ¢(1 - ) = lim = lim
h®0 2 (1 - h )(2 - h ) h 4 2 h®0 -h h®0 -h
ì - 1 + x, -¥<x <0 h (0 -1 + h )
= lim =1
ï h®0 -h
10. f ( x ) = í - 1 + sin x, 0 £ x < p / 2
ï cos x, p /2£x <¥ Þ f ¢(1 + ) = 0, f ¢(1 - ) = 1
î
ì1, -¥<x <0 \ f ( x ) is not differentiable at x =1.
ï f (1 - h ) - f (1 ) [cos p (1 - h )] + 1
f ¢( x ) = í cos x , 0 < x < p / 2
14. f ¢(1 - ) = lim = lim
ï - sin x , p / 2 < x < ¥ h®0 -h h® 0 -h
î
-1 + 1
f ¢( 0 - ) = 1, f ¢( 0 + ) = 1 = lim =0
h ® 0 -h
Þ f ( x ) is differentiable at x = 0 f (1 + h ) - f (1 )
æ p– ö f ¢(1 + ) = lim
h®0
f ¢ ç ÷ = 0, f ¢( p / 2 + ) = -1 h
è 2 ø 2 {1 + h } - 1 + 1
= lim
p h® 0 h
Þ f ( x ) is not differentiable at x = .
2 2h
= lim =2
\Number of points of non-differentiability is 1. h®0 h

ì 1 \ f ¢(1 - ) = 0, f ¢(1 + ) = 2
ï- x , x £ -1
ï ì x - 3, x<0
11. f ( x ) = íax 2 + b , - 1 < x < 1 15. f ( x ) = í
î x 2
- 3 x + 2, x³0
ï1
ï , x ³1 ì | x | - 3, | x | < 0 not possible
f (| x | ) = í 2
îx î | x | - 3 | x | + 2, | x | ³ 0
Since, function is continuous everywhere.
ì x 2 + 3 x + 2, x £ 0
\ LHL = RHL at x = -1 Þ g( x ) = í 2
Þ f ( -1 - ) = 1, f ( - 1 + ) = a + b î x - 3 x + 2, x ³ 0
Þ g( x ) is continuous at x = 0.
Þ a + b =1 …(i)
ì2 x + 3, x £ 0
f ( x ) is differentiable at x = -1 g ¢( x ) = í
ì1 î 2 x - 3, x ³ 0
ï 2 , x < -1 g ¢( 0 + ) = - 3, g ¢( 0 - ) = 3
ïï x
f ¢( x ) = í2ax , - 1 < x < 1 ì 0, 0 £ x < 1 / 2
ï 1 ï5 / 6, x =1/2
ï - 2 , x >1 ï
ïî x ï 0, 1 / 2 < x < 1
ïï -2, 1 £ x < 4 / 3
f ¢( -1 - ) = 1, f ¢( -1 + ) = -2a Þ - 2a = 1, a = -1 / 2 …(ii) 16. f ( x ) = í
0, x = 4/3
From Eqs. (i) and (ii), we get ï
ï 2, 4 / 3 < x <3/2
1 3
a = - ,b = ï 3, 3/2 £ x <2
2 2 ï
ïî 4, x =2
12. f (1 - ) = A + B, f (1 + ) = 3A - B + 2
Since, continuous A + B = 3 A - B + 2 Hence, f ( x ) is neither continuous nor differentiable at
1 4 3
Þ 2 A - 2 B = -2 Þ A - B = -1 …(i) x = , 1, , , 2 .
2 3 2
ì2 Bx , x < 1
f ¢( x ) = í \ Number of points is 5.
î3 A, x > 1
17. Put x = y = 1 in given rule
f ¢(1 + ) = 3 A, f ¢(1 - ) = 2 B
Þ f (1 ) = f (1 ) - f (1 ) = 0
Þ 3A =2B …(ii) æ x + hö
fç ÷
On solving Eqs. (i) and (ii), we get f (x + h ) - f (x ) è x ø
A = 2, B = 3 \ f ¢( x ) = lim = lim
h ®0 h h®0 h
382 Textbook of Differential Calculus

æ æ hö ö Þ
5
f (x 2 ) = 2 + x
çf ç1 + ÷ - f (1 ) ÷ …(ii)
è xø ÷ 1 = f ¢(1 ) = 1
2
= lim ç 12 12
h ®0 ç h ÷x x x æ 5 ö 5 (12 )(13 )
ç ÷ \ å f (r 2 ) = å ç2 + r ÷ = 24 + × = 24 + 195 = 219
è x ø r =1 r =1
è 2 ø 2 2
[given, f ¢(1 ) = 1] 12

On integrating both sides, f ( x ) = log x + C Hence, å f (r 2 ) = 219


r =1
On putting x = 1, we get C = 0 Þ f ( x ) = loge x
f (x + h ) - f (x ) 2
18. f ¢( x ) = lim 22. Let x = , which is rational.
h ®0 h 3
f ( x ) + f (h ) - 2 xh - 1 - f ( x ) æ2ö 1
= lim Þ hç ÷=
è3ø 3
h®0 h
æ f (h ) - 1 ö æ2 ö
f ¢( x ) = lim ç -2 x + ÷ = - 2 x + f ¢( 0 ) lim h ç + t ÷ = 0 Þ Discontinuous at x Î Q
h ® 0è ø t ®0 è3 ø
h
On integrating both sides, we get Let x = 2 ÏQ
Þ f ( x ) = - x 2 + f ¢( 0 ) x + c h( 2 ) = 0, consider 2 = 1.4142135624
Þ f ( x ) = - x 2 - (sin a ) x + c [given f ¢( 0 ) = - sin a ] æ 14142135624 ö 1
h( 2 ) = h ç ÷ = 10 ® 0
è 1010 ø 10
Þ f (0) = c Þ c = 1
So, f { f ¢( 0 )} = f { - sin a } = - sin 2 a + sin 2 a + 1 Hence, h is continuous for all irrational.

=1 23. By theorem, if g and h are continuous functions on the open


interval (a, b ), then g/h is also continuous at all x in the open
æ x + hö interval (a, b ), where h( x ) ¹ 0.
log ç ÷+x+h-x
f (x + h ) - f (x ) è x ø
19. f ¢( x ) = lim ³ lim ì 2 cos x - sin 2 x p
h®0 h h®0 h ïï ( p - 2 x ) 2 , x < 2
æ hö 24. h( x ) = í - cos x
log ç1 + ÷ ïe -1 p
è xø 1 1 , x>
= lim + 1 ³ + 1 …(i) ïî 8 x - 4 p 2
h®0 h x x
LHL at x = p/2
x
2 sin h - sin 2h 2 sin h (1 - cosh )
æ x - hö LHL = lim = lim =0
log ç ÷ + x -h - x h®0 4h 2 h®0 4h 2
f (x - h ) - f (x ) è x ø
f ¢( x ) = lim £ lim e sin h - 1
h®0 -h h®0 -h RHL = lim
h ® 0 8(( p/2 ) + h ) - 4 p
æ hö
log ç1 – ÷
è xø 1 e sin h - 1 sin h 1
= lim + 1 £ + 1 …(ii) = lim × =
h®0 -h x h®0 8h sin h 8
1
\ From Eqs. (i) and (ii), we get f ¢( x ) = + 1 = g( x ) Þh ( x ) is discontinuous at x = p /2.
x Irremovable discontinuity at x = p / 2 .
100
æ1ö æ1ö æ1ö æ 1 ö æ p+ ö æ p- ö 1
Þ ågç ÷ = gç ÷ + gç ÷ + ¼ + gç ÷ fç ÷ = 0 and g ç ÷ =
n =1 n
è ø è1ø è2ø è 100 ø è 2 ø è 2 ø 8
= (1 + 1 ) + (2 + 1 ) + (3 + 1 ) + ¼ + (100 + 1 ) æ p+ ö æ p- ö
= (1 + 2 + ¼ + 100 ) + 100 Þ fç ÷ ¹ gç ÷
è 2 ø è 2 ø
= 5050 + 100 = 5150
x - e x + 1 - (1 - cos 2 x ) 1 5
20. Here,
d ( f ( x ))
= e -x f (x ) + e x f (-x ) 25. lim =- - 2 = -
dx
x®0 x2 2 2
5
d ( f ( x )) Hence, for continuity, f ( 0 ) = -
Let = g( x ), where g( x ) = e - x f ( x ) + e x f ( - x ) 2
dx
ì 5ü 1
\ g( - x ) = e x f ( - x ) + e - x f ( x ) = g( x ) \ [ f ( 0 )] = - 3; { f ( 0 )} = í - ý =
î 2þ 2
Þ g( x ) is even function. 3
Hence, f ( x ) should be an odd function as f ( 0 ) = 0. Hence, [ f ( 0 )] × { f ( 0 )} = - = - 1.5
2
x2
21. We have, f ( x 2 ) = ò t f (t ) dt = x 4 + x 5 …(i) 26. f (1 + ) = f (1 - ) = f (1) = 2
0
On differentiating both the sides w.r.t. x, we get f ( 0 ) = 1, f (2 ) = 2
x 2 f (x 2 ) × 2x = 4x 3 + 5x 4 f (2 - ) = 1,
Þ f is not continuous at x = 2.
Chap 06 Continuity and Differentiability 383

27. f (2 + ) = 8, f (2 - ) = 16 Þ g[ f ( a - )] = g [ f ( a + )] = g (a )
28. g( x ) = x - [ x ] = { x } Þ g( x ) takes same limiting values at
f is continuous with f ( 0 ) = f (1 ) Y f ( a - ), f ( a + ) and f ( a ).
h( x ) = f ( g( x )) = f ({ x }) f (x) Þ f ( a - ) = f ( a + ) Þ x = a is an extremum of f ( x )
Let the graph of f is as shown in and x = a may not be an extremum of g( x ).
f (0)
the figure satisfying
33. From the graph, number of points of non-differentiability = 11
f ( 0 ) = f (1 )
Now, X Y
O 1
h( 0 ) = f ({ 0 }) = f ( 0 ) = f (1 )
h( 0.2 ) = f ({ 0.2 }) = f ( 0.2 ) y = |cos x| y = |sin x|
y = 1/4
h(1.5 ) = f ({1.5 }) = f ( 0.5 ) etc.
Hence, the graph of h( x ) will be a periodic graph as shown π X
O 2π

Y 34. Let n be any integer other than 1.


h (x)
lim f ( x ) = lim ([n - h ] 2 - [(n - h ) 2 ])
x ®n - h ®0

= (n - 1 ) 2 - (n 2 - 1 ) = 2 – 2n

X
lim f ( x ) = lim [n + h ]2 - [(n + h ) 2 ] = n 2 - n 2 = 0
O x ®n + h ®0
1 2 3
\ LHL ¹ RHL unless n = 1.
Þh is continuous in R. Hence, f ( x ) is discontinuous at all integral values except 1.
29. Statements I and II are false. The function f ( x ) = 1 /x, 0 < x < 1 {( x - 1 ) 2 ( x + 1 ) | x - 5| + cos| x| } - cos5
35. At x = 5, f ¢ ( x ) = lim
is a counter example. x®5 x -5
Statement III is true. Apply the intermediate value theorem to 96 | x - 5|
= lim = + 96, if x > 5 and - 96, if x < 5
f on the closed interval [a1, b1 ]. x®5 x -5

Hence, f ¢(5 ) doesn’t exist.


a1 c This ambiguity doesn’t occur at other points.
a b1 b
\ f ( x ) is not differentiable at x = 5.
-1
2
30. Since, the given function is not differentiable, at x = 0 and 2. f (x ) - f (0) ex - 0 1/x
36. f ¢ ( 0 + ) = lim = lim = lim 2
+ x + x +
Hence, the number of points is 2. x®0 x®0 x ® 0 e 1/ x

ì x -1 / x 2
x
ïï 1 - x , if x ³ 0, x ¹ 1 = lim
x®0 + 2 æ 2
= lim
ö x ® 0 2e /x 2
+ 1
=0
31. f ( x ) = í e 1/ x . ç - 3 ÷
ï
x è x ø
, if x < 0, x ¹ - 1
ïî 1 + x
As, f is even, so f ¢( 0 - ) = f ¢( 0 + ) = 0. Thus, f ¢( 0 ) = 0
Y eh - cos2h - h
37. lim g(n + h ) = lim
h®0 h®0 h2
1 e -h - 1
h
(1 - cos 2h ) 1 5
= lim + lim ×4= +2=
h®0 h2 h®0 4h 2 2 2
X′ X 1- {n -h }
–1 O 1 e - cos2 (1 - {n - h }) - (1 - {n -h })
lim g(n - h ) =
–1
h®0 (1 - {n - h }) 2
e - cos2h - h
h
5
= lim [{n - h } = { -h } = 1 - h ] =
Y′ h®0 h 2
2
5
ì 1 g(n ) = . Hence, g( x ) is continuous, " x Î I .
ïï (1 - x ) 2 , if x > 0, x ¹ 1
2
and f ¢( x ) = í Hence, g( x ) is continuous, " x Î R.
1 cosh - 1
ï , if x < 0, x ¹ - 1 38. g ¢ ( 0 + ) = lim =0
ïî (1 + x ) 2 h®0 h
-h + b -1
32. g[ f ( x )] is continuous at x = a g ¢ ( 0 - ) = lim for existence of limit b = 1
h®0 -h
Þ g[ f ( a )] = g (a )
Thus, g ¢( 0 - ) = 1
Also, lim g( f ( x )) = g(a )
x®a Hence, g cannot be made differentiable for no value of b.
384 Textbook of Differential Calculus

39. By definition f ¢ (1) is the limit of the slope of the secant line p[ x ]
sin
when s ® 1. 45. f ( x ) = 4
s 2 + 2s - 3 Y [x ]
Thus, f ¢ (1 ) = lim
s ®1 s -1 Obviously, continuity at x = 3/2
Q (s, r)
p 1
(s - 1 ) (s + 3 ) f (2 - ) = sin =
= lim 4 2
s ®1 s -1 At x = 2, p
= lim (s + 3 ) = 4 sin
s ®1 P (1, 2)
X f (2 ) = 2 =1
Aliter O 2 2
By substituting x = s into the Hence, f ( x ) is discontinuous at x = 2.
equation of the secant line and cancelling by s - 1. Again, 46. f ( -1) = b (1 - 1) + 1 = 1
we get y = s 2 + 2s - 1.
and lim f ( - 1 + h ) = 1
This is f (s ) and its derivative is f ¢(s ) = 2s + 2, so f ¢(1 ) = 4. h®0

40. In the immediate neighbourhood of lim f ( - 1 - h ) = lim sin (( - 1 – h + a ) p ) = - sin pa


x = p/2, sin x > sin 3 x Þ |sin x - sin 3 x| = sin x - sin 3 x h®0 h®0
æ 3p ö
For continuity, sin pa = - 1 = sin ç2np + ÷
Hence, for x ¹ p / 2, è 2 ø
é 2(sin x - sin 3 x ) + sin x - sin 3 x ù 3p
f (x ) = ê 3 ú Þ pa = 2np +
ë 2(sin x - sin x ) - sin x + sin x û
3
2
3 sin x - 3 sin 3 x 3
= =3 Þ a = 2n +
2
sin x - sin 3 x
3
Hence, f is continuous and differentiable at x = p/2. Hence, a = 2n + , n Î I and b Î R
2
f (x + h ) - f (x ) f (h ) + | x | h + xh 2
41. f ¢( x ) = lim = lim 47. Consider the graph of h( x ) = max ( x, x 2 ) at x = 0 and x = 1
h®0 h h®0 h
For (d ) : h ( x ) = max ( x 2, - x 2 )
Q f (0) = 0
é f (h ) - f ( 0 ) ù 48. Here, g( x ) = [ x 2 ]{cos2 4x } + { x 2 }[cos2 4x ] + x 2 sin 2 4x
Hence, f ¢( x ) = lim ê + | x| + xhú
h ® 0ë û
h + [ x 2 ] [cos2 4 x ] + { x 2 } {cos2 4 x }
Þ f ¢ ( x ) = f ¢( 0 ) + | x | = | x |
= [ x 2 ]({cos2 4 x } + [cos2 4 x ]) + { x 2 }([cos2 4x ]
42. f ( x ) is non-differentiable at x = a, b, 0, g, d
+ {cos2 4x }) + x 2 sin 2 4x
and g( x ) is non-differentiable at x = a, - 2, b, 0, g, 2, d.
Y = ([ x 2 ] + { x 2 })({cos2 4 x } + [cos2 4 x ]) + x 2 sin 2 4 x
f (x) = x 2 cos2 4 x + x 2 sin 2 4 x
= x2
Clearly, g( x ) is always differentiable.
α –2 β γ2 δ X
0 \ Number of points of non-differentiability is 0.
{ x } g( x )
ì3 x 2 - 4 x + 1, for x < 1 49. Here, f ( x ) = = 1 , when { x } g( x ) ¹ 0
43. We have, g( x ) = í { x } g( x )
î ax + b, for x ³ 1 1
If f ( x ) is periodic with period , then { x } g( x ) ¹ 0 with
For differentiability at x = 1, g ¢(1 + ) = g ¢(1 - ) 4
1
4 period .
a =6 - 4
2 1
k
Þ a =6 -2 = 4 Þ g( x ) = 0 at x = , where k Î I .
4
For continuity at x = 1, g(1 ) = g(1 - )
+
f (x + h ) - f (x )
50. Here, f ¢( x ) = lim
a + b = 3 - 4 + 1 Þa + b = 0 h®0 h
Þ b = -4 æ æ h öö æ 1 ö
f ç x ç1 + ÷ ÷ - f ç ÷
Hence, a = 4 and b = - 4 è è x øø è 1/ x ø
f (x + h ) - f (x ) - 3 xh + f (h ) = lim
44. f ¢( x ) = lim = lim h®0 h
h®0 h h®0 h æ1 + h/ x ö æ 1 ö
ì f (h )ü f (h ) f ç ÷-f ç ÷
= lim í - 3 x + ý = - 3 x + lim è 1/ x ø è 1/ x ø
h®0î h þ h®0 h = lim
h®0 h
= - 3x + 7
Chap 06 Continuity and Differentiability 385

f (1 + h / x ) f (1 ) æ 1ö
-
= lim h 2 cos çe h ÷ = 0
= lim
f (1 / x ) f (1 / x ) h® 0 ç ÷
è ø
h®0 h
f (1 + h / x ) - f (1 ) 1 1 f (x ) and f ( 0 ) = 0
= lim × = f ¢(1 ) × ×
h®0 h/ x æ1ö x f (1 ) Hence, f ( x ) is continuous at x = 0.
xf ç ÷
èxø 54. lim f ( x ) = lim (b([ x ] 2 + [ x ]) + 1 )
x ® -1+ x ® -1+
dy y f ¢(1 )
\ = k × , where = k, f ( x ) = y . = lim (b ([ - 1 + h ]2 + [ - 1 + h ]) + 1 )
dx x f (1 ) h® 0
dy dx = lim (b(( -1 ) 2 - 1 ) + 1 ) = 1
Þ ò y
= òk
x h® 0

Þ log y = k log x + log C Þ b ÎR


Þ log y = log( xk × C ) lim f ( x ) = lim sin( p ( x + a ))
x ® -1- x ® -1-
Þ y =C ×x k
= lim sin( p (( -1 - h ) + a ))
h® 0
\ f (x ) = C × xk
\ sin pa = - 1
æ x ö f (x ) 3p 3
Put x = 2, y = 1 in f ç ÷ = Þ pa = 2np + Þ a = 2n +
è y ø f (y ) 2 2
f (2 ) ì p
Þ f (2 ) = Þ f (1 ) = 1 ï sin 4 1
f (1 ) = , 1 £ x <2
ïï
55. f ( x ) = í 1 2
Also, f ( x ) = C × xk , put x = 1 p
ï sin
Þ f (1 ) = C Þ C = 1 ï 2 1
\ f ( x ) = xk , now f (2 ) = 4 ïî 1 = 2 , 2 £ x < 3
3 4
Þ f (2 ) = 2k Hence, f ( x ) is continuous at , differentiable at and
2 3
4 = 2k Þ k =2 discontinuous at 2.
\ f (x ) = x Þ f (5 ) = 25 56. Since, sin -1 x and cos(1 / x ) are continuous and differentiable in
2

æx ö x Î [ - 1, 1 ] - { 0 }.
51. tan( f ( x )) = tan ç - 1 ÷, x Î[ 0, p ] Now, at x = 0
è2 ø
f (- h ) - f (0)
x p f ¢( 0 - ) = lim =0
0 £ x £ p Þ-1 £ -1 £ -1 h®0 -h
2 2
\tan( f ( x )) is continuous in [ 0, p ]. æ 1ö
(sin -1( -h )) 2 cos ç - ÷ - 0
1 2 è hø
= is not defined at x = 2 Î [ 0, p ]. = lim =0
f (x ) x - 2 h®0 -h
x -2 æ1ö
y = Þ f -1( x ) = 2 x + 2 is continuous in R. (sin -1 h ) 2 cos ç ÷ - 0
èhø
2 f ¢( 0 + ) = lim =0
2 h®0 h
- (x + 1)
52. lim ( x + 1) e x = lim =0 Hence, LHD = RHD. So, f ( x ) is continuous and differentiable
x ® 0+ x ® 0+ e 2/ x everywhere in - 1 £ x £ 1.
lim ( x + 1 ) e 0 = 1 ì p
x ® 0-
ï cos x , 0 £ x < 2
Hence, continuous for x Î I - { 0 }, assumes all intermediate 57. H ( x ) = í
p p
3 ï - x, < x £3
values from f ( -2 ) to f (2 ) and maximum value at x = 2. î2 2
e
æ p -ö
æ é æ 2x 3 - 3 ö ù ö H ¢ ç ÷ = - sin x = - 1
53. RHL = lim çç3 - ê cot -1 ç ÷ú÷ è2 ø
x ® 0+ è è êë x 2 ø úû ÷ø
-1
æp+ö
= 3 - [cot ( - ¥ )] = 3 - 3 = 0 H ¢ç ÷ = -1
è2 ø
æ -1ö
LHL = lim { x 2 } cos çe x ÷ Hence, H ( x ) is continuous and derivable in [0, 3] and has
x ® 0- ç ÷
è ø maximum value 1 in [0, 3].
386 Textbook of Differential Calculus

58. Here, f ( x ) = 3(2x + 3) 2/3 + 2x + 3 Also, f (b - ) = f (b ), g(b + ) = g(b )


4 [using Eqs. (i) and (ii)] …(iv)
Þ f ¢( x ) = +2
(2 x + 3 )1/3 \ h(b ) = f (b ) = f (b ) = g(b ) = g(b + ) = h(b + )
- -

3 [using Eqs. (iv) and (v)]


Now, 2x + 3 ¹ 0 Þ x ¹ -
2 Now, verify each alternative. Of course! g(b - ) and f (b + ) are
Hence, f ( x ) is continuous but not differential at undefined.
x = -3 / 2 . h(b - ) = f (b - ) = f (b ) = g(b ) = g(b + )
Also, f ( x ) is differentiable and continuous at x = 0.
and h(b + ) = g(b + ) = g(b ) = f (b ) = f (b - )
1/h 2
h ln (cos h ) ln (cos h )
59. f ¢( 0 + ) = lim = lim Hence, h(b - ) = h(b + ) = f (b ) = g(b )
h®0 h ln (1 + h 2 ) h ® 0 ln (1 + h 2 )
h2 and h (b ) is not defined.
1 1 1 65. (a) Domain is R - { - 1}; Range = R - { 0}
= lim 2 (cos h - 1 ) = - ; similarly f ¢( 0 - ) = -
h®0h 2 2 Y
Hence, f is continuous and derivable at x = 0.
ì 0, 0 < x < 1
ï X
60. f ( x ) = í 0, x = 0 or 1 or - 1 Þ f ( x ) = 0 for all in [ - 1, 1] –1 O
ï 0, - 1 < x < 0
î
ì 0, x = - 1
ï - 1, - 1 < x < 0
ï (b) Domain is x Î R; Range = ( 0, 1 ]
61. [| x| ] - |[ x ]| = í
ï 0, 0 £ x £ 1 (c) Domain is [ 0, ¥ ); Range = ( 0, 1 ]
ïî 0, 1 < x £ 2
(d) Domain is ( - ¥, 3 ); Range = ( 0, ¥ )
Þ Range is { 0, - 1 }. The graph is 2 (sin 2 x + cos 2 x )
66. f ( x ) = sec 2x + cosec 2x =
2 cos 2 x sin 2 x
Y 2 (sin 2 x + cos 2 x )
=
sin 4 x
–2 –1
X′ X np
O 1 is discontinuous, where 4x = np, n Î I or x = .
4
–1
Options (a) and (b) also satisfy the condition, since they are
Y′
subsets of option (c).
1 1 æ 1 ö
62. f ¢( 0 + ) = , f ¢( 0 - ) = - ; 67. lim f ( x ) = lim xn × sin ç 2 ÷ = 0, if n > 0
2 2 x®0 x®0 èx ø
x2 | x| and hence true for n > 1.
f (x ) = =
1+ 1-x 2
1+ 1-x 2 Since, f ( 0 ) = 0, f ( x ) is continuous at x = 0.
1
+
63. Range is R È { 0} Þ Option (b) is not correct. xn sin 2
f (x ) - f (0) x
Now, lim = lim
f is not differentiable at x = - 1 x®0 x-0 x®0 x
ì x 3 + 1, if x ³ - 1
f (x ) = í æ 1 ö
As, = lim xn - 1 sin ç 2 ÷ = 0, if n > 1.
î - ( x + 1 ), if x < - 1
3
x®0 èx ø
ì 3 x 2, if x > -1 Hence, f ( x ) is differentiable at x = 0, if n > 1.
Þ f ¢( x ) = í
î - 3 x , if x < -1 ì ex , x £ 0
2

ï
f ¢( - 1 + ) = 3, f ¢( - 1 - ) = - 3 Þ f is not differentiable at x = - 1. 68. f ( x ) = í1 - x , 0 < x < 1
Also, f is not bijective, hence it has no inverse. ï x - 1, x ³ 1
î
64. Given, f is continuous in [a, b ] …(i) Þ lim f ( x ) = 1, lim f ( x ) = 1
x ® 0- x ® 0+
Þ g is continuous in [b, c ] …(ii)
Þ f (b ) = g(b ) …(iii) and lim f ( x ) = 0, lim f ( x ) = 0
x ® 1- x ® 1+
h( x ) = f ( x ) for x Î [a, b )ü
Þ ý …(iv) Hence, f ( x ) is continuous at x = 0 and 1.
= g( x ) for x Î (b, c ]þ
f ¢( 0 - ) = 1 and f ¢( 0 + ) = - 1.
Þ h( x ) is continuous in [a, b ) È (b, c ] [using Eqs. (i) and (ii)]
Hence, f ( x ) is not differentiable at x = 1.
Chap 06 Continuity and Differentiability 387

-1
x
69. f ( x ) = ò | t + 1 | dt = - ò (t + 1) dt +
x
So, [ x ] is discontinuous at x = 0.
-2 -2 ò-1(t + 1) dt We know that, sum of continuous and discontinuous functions
x
1 æt2 ö x2 is discontinuous.
= + ç + t÷ = + x + 1, for x ³ - 1
2 è2 ø -1 2 74. We have,
ì2 sin (a cos- 1 x ), if x Î ( 0, 1 )
f ( x ) is a quadratic polynomial. ï
\ f ( x ) is continuous as well as differentiable in [ - 1, 1 ]. f (x ) = í 3, if x = 0
ï + if x < 0
Also, f ¢( x ) is continuous as well as differentiable in [ - 1, 1 ] . î ax b ,
70. We have, f ( x + y ) = f ( x ) + f (y ) + xy ( x + y ) Continuity at x = 0
f (0) = 0 (LHL at x = 0) = lim ax + b
x ® 0-
f (h ) = lim a( 0 - h ) + b = lim - ah + b
\ lim = -1 h®0 h®0
h®0 h

f (x + h ) - f (x ) =b
\ lim
h®0 h (RHL at x = 0) = lim 2 sin (a cos- 1 x )
x ® 0+
f ( x ) + f (h ) + xh ( x + h ) - f ( x )
= lim = lim 2 sin (a cos- 1 ( 0 + h ))
h®0 h h®0
f (h )
= lim + lim x ( x + h ) = - 1 + x 2 = lim 2 sin a cos- 1 h
h®0 h h®0 h®0

\ f ¢( x ) = - 1 + x 2 = 2 sin a cos- 1 h = 2 sin a cos- 1 0


x3 ap
\ f (x ) = -x+c = 2 sin
3 2
\ f ( x ) is a polynomial function, f (0) = 3
f ( x ) is twice differentiable for all x Î R and For f ( x ) to be continuous at x = 0,
f ¢(3 ) = 3 2 - 1 = 8 . LHL = RHL = f ( 0 )
ap
71. We have, f ¢( x ) = 6x + 12 \ b = 2 sin = 3
a
For f ( x ) is increasing, f ¢( x ) ³ 0 Þ x ³ - 2
ap 3
Hence, f ( x ) is increasing in [ - 1, 2 ] \ b = 3 and sin =
2 2
lim f ( x ) = 35, lim f ( x ) = 35 and f (2 ) = 35
x ® 2+ x ® 2- ap p
Þ b = 3 and =
Hence, f ( x ) is continuous on [ - 1, 3 ], f ¢(2 - ) = 24 and 2 3
f ¢(2 + ) = - 1 . Þ b = 3 and a =
2
Hence, f ¢(2 ) doesn’t exist for maximum, f (2 ) = 35 3
f ( - 1 ) = - 10, f (3 ) = 34 So, Statement I is correct.
Hence, f ( x ) has maximum value at x = 2. Since, for x < 0, f ( x ) = ax + b
72. Since, f ( x ) = 0, x < 0 and differentiable at x = 0, LHD = 0 which is a polynomial function and will be continuous for
( - ¥, 0 ).
(function is on X -axis for x < 0). If f ( x ),
Again, for x Î( 0, 1 ),
(a) x 2, x > 0
f ( x ) = 2 sin (a cos- 1 x ), which is trigonometric function will be
RHD at x = 0, continuous for x Î( 0, 1 ).
f ¢( 0 ) = 2 ´ 0 = 0 (possible) Q f ( x ) is continuous in ( - ¥, 1 ).
(d) - x 3/2, x > 0 \ Statement II is also correct.
RHD at x = 0, ì x2
-
f ¢( 0 ) = - 3 / 2 x1/2 = - 3 / 2 ´ 0 = 0 (possible) ï cos x - e 2
75. We have, f ( x ) = í , x¹0
73. We know that, sin x is periodic function in [ 0, 2p]. ï x3
î 0, x=0
\ sin x is continuous at x = 0
Clearly, f ( 0 ) = 0
Now, lim [ x ] x2
x®0 -
RHL = lim [ x ] = lim [ x + h ] = x = 0 [Q [ x + h ] = x ] cos x - e 2
é0 ù
x ® 0+ h®0 Now consider, lim f ( x ) = lim êë 0 form úû
x®0 x®0 x3
LHL = lim [ x ] = lim [ x - h ] = ( x - 1 ) [Q [ x - h ] = ( x - 1 )]
x ® 0- h®0 x2
-
= 0 -1 = -1 - sin x + e 2 ×x é0 ù
= lim êë 0 form úû
\ RHL ¹ LHL x®0 3x 2
388 Textbook of Differential Calculus

-
x2
-
x2 80. Consider g( x ) = x 3 at x = 0, g( 0) = 0
- cos x + e 2 -e 2 ×x 2
é0 ù
= lim | g( x )| is derivable at x = 0.
x®0 6x êë 0 form úû
Actually nothing definite can be said. Also, for g( x ) = x - 1
x2 x2 x2 with g (1 ) = 0.
- - -
sin x - e 2 × x - 2 xe 2 +e 2
Then, | g( x )| is not derivable at x = 1.
= lim
x®0 6
81. Y
=0 y = x – x2
Thus, lim f ( x ) = f ( 0 )
x®0 X
Þ f is continuous at x = 0 0 1
Hence, option (c) is correct.
Y
x3 2 y = f (g (x))
76. Let f ( x ) = - sin px + . Then, f ( x ) will be continuous
4 3
function. (Q Sum and difference of two continuous function is X
continuous) –1 0 1 2 3
8 4
Here, f (2 ) = and f ( - 2 ) = - [Q sin np = 0, " n Î Z ] 82. f ( x ) is discontinuous at x = 0 and f ( x ) < 0, " x Î [ - a, 0) and
3 3
f ( x ) > 0, " x Î [ 0, a ] .
Now, as f ( - 2 ) < 0 < f (2 ), therefore by intermediate value
theorem we can say that there exists atleast one point Sol. (Q. Nos. 83 to 85)
n Î [ - 2, 2 ]. Such that f (n ) = 0 ì x + 1, 0 £ x £2
f (x ) = í
Hence, f ( x ) = 0, i.e.
x3 2
- sin px + = 0 has atleast one î- x + 3 , 2 < x < 3
4 3 Y
solution in [ - 2, 2 ]. 3
Clearly, Statement II is wrong. Because for this to be true, f ( x ) 2
should be a continuous function (by intermediate value 1
theorem). X
0 1 2 3
Hence, option (c) is correct.
77. We have, \ f ( x ) is discontinuous at x = 2.
æ e 2x - e - 2x ö ì x + 2, 0 £ x £1
f ( x ) = x ç 2x ÷ + x2 + x4 ï
( fof ) ( x ) = í - x + 2 , 1 < x £ 2
è e + e - 2x ø
ï- x + 4 , 2 < x < 3
î
æ e 2( - x ) - e - 2( - x ) ö
\ f ( - x ) = ( - x ) ç 2( - x ) ÷ + (- x )2 + (- x )4 Þ( fof ) ( x ) is discontinuous at x = 1, 2
èe + e - 2( - x ) ø
and f (19 ) = f (3 ´ 6 + 1 ) = f (1 ) = 2
æ e - 2x - e + 2x ö
= - x ç - 2x ÷ + x2 + x4 83. (a) 84. (b, c) 85. (a)
èe + e + 2x ø Sol. (Q. Nos. 86 to 89)
é æ e 2x - e - 2x ö ù 1
= - x ê - ç 2x ÷ + x2 + x4 Since, f ( - x ) =
- 2x ú f (x )
êë è e + e ø úû
-
\ At x = 0
æe - e
2 x 2 x ö
= x ç 2x - 2x
÷ + x2 + x4 Þ f (0) =
1
Þ f 2( 0 ) = 1 Þ f ( 0 ) = + 1 , as f ( x ) > 0.
èe + e ø f (0)
= f (x ) f (x + h ) - f (x )
Þ f ¢( x ) = lim
\ f ( x ) is even function and even function can’t has range equal h®0 h
to R. f ( x ) × f (h ) - f ( x )
f ¢( x ) = lim
78. y = | ln x | not differentiable at x = 1. h®0 h
p 3p æ f (h ) - 1 ö
y = | cos| x || is not differentiable at x =
, × \ f ¢( x ) = f ( x ) × ç lim ÷ = f ( x ) × f ¢( 0 )
2 2 èh ® 0 h ø
-1 -1
y = cos (sgn x ) = cos (1 ) = 0 differentiable, " x Î ( 0, 2p ). f ¢( x )
h sin h - 0
Þ ò f (x ) = ò - 1 dx Þ log f (x ) = - x + c
79. f ¢ ( 0 + ) = lim =0
h®0 h f ( x ) = e - x × l at x = 0, l = 1 Þ f ( x ) = e - x
h sin ( - h ) - 0 Þ Range of f ( x ) Î R + . Þ Range of f (| x| ) is [0, 1].
f ¢ ( 0 - ) = lim =0
h®0 -h Þ f ( x ) is decreasing function
f ( x ) is differentiable at x = 0. and f ¢( x ) = - e - x = - f ( x ).
e.g. x | x | is derivable at x = 0.
86. (c) 87. (a) 88. (d) 89. (a)
Chap 06 Continuity and Differentiability 389

ì sin x , x£0 Now, f ( 0 ) = 0 Þ c = 0 and f (1 ) = 1 Þ k = 1


ï tan x , 0 < x < 2 p \ f ( x ) = x Þ f ¢( x ) = 1, for all x Î R
ï
90. f ( x ) = í
\ f ( x ) = x is an odd function.
ï cos x , 2 p £ x < 3 p
ïî 3 p , x = 3 p 96. (b) 97. (c) 98. (d) 99. (a)
p 3p f (x + h ) - f (x )
f ( x ) is discontinuous at , , 2p. We know, f ¢( x ) = lim
h®0 h
2 2
91. f ( x ) = [2x - 5]
3 f (x - h ) - f (x )
and f ¢( x ) = lim
Since 1 £ x < 2 Þ 1 £ x 3 < 8
h®0 -h
-3 £ 2 x 3 - 5 < 11 On adding, we get
Now, 2 x 3 - 5 is discontinuous at integer points. f (x + h ) - f (x ) f (x - h ) - f (x )
2 f ¢( x ) = lim + lim …(i)
\ -2, - 1, 0, 1, 2, 3, 4, 5, 6, 7, 8, 9, 10 . h®0 h h®0 -h
Hence, number of points of discontinuity = 13 f (x + h ) - f (x - h )
Þ 2 f ¢( x ) = lim
h®0 h
92. Max ([ x ], | x| ), hence discontinuity at x = f.
{ f (h ) - f ( -h )}
Y Þ 2 f ¢( x ) = lim f ( x ) ×
h®0 h
y = [x] [using f ( x + y ) - f ( x - y ) = f ( x ) { f (y ) - f ( -y )}]…(ii)
æ f (h ) - f ( 0 ) f ( -h ) - f ( 0 ) ö
Also, 2 f ¢( 0 ) = lim ç + ÷
O X h®0 è h -h ø
[using Eq. (i)]
93. From figure, option (c) is correct. f (h ) - f ( - h )
Y = lim …(iii)
h®0 h
From Eqs. (ii) and (iii), we get
f ¢( x )
O
X 2 f ¢( x ) = f ( x ) × 2 f ¢( 0 ) Þ = f ¢( 0 )
f (x )
π/2
f ¢( x )
\ = 2010 …(iv)
f (x )
94. From figure, option (b) is correct. On integrating both sides, we get
Y log( f ( x )) = 2010 x + c, as f ( 0 ) = 1
\ c = 0 Þ f ( x ) = e 2010x
Thus, { f ( x )} is non-periodic. …(v)
X
O 100. (b) 101. (d)
π/2
102. (A) lim sin {1 - h } = cos1 + a
h®0
Þ lim sin(1 - h ) - cos1 = a
95. From figure, option (c) is correct. h®0

Y Þ a = sin 1 - cos1
sin 1 - cos1
Now, | K | = =1 Þ K = ± 1
æ 1 1 ö
2 çsin 1 × - cos1 × ÷
X è 2 2ø
O
π/2 æ sin x ö
2 sin 2 ç ÷
è 2 ø æ sin x ö 2 1
(B) f ( 0 ) = lim ×ç
2 è ÷ =
x®0
2 æ sin x ö
2 ø 2
x ×ç ÷
Sol. (Q. Nos. 96 to 99) è 2 ø
Here, f ( x + y 2n + 1 ) = f ( x ) + ( f (y )) 2n + 1 (C) Function should have same rule for q and q¢ Þ x = 1 - x
On differentiating, we get 1
Þ x=
f ¢( x + y 2n + 1 ) = f ¢( x ) 2
é dy ù (D) f ( x ) = x + { - x } + [ x ]
êëQ x and y are independent, so dx = 0 úû x is continuous at x Î R.
Þ f ¢( x ) is constant, say f ¢( x ) = k. Check at x = I (where I is integer)
On integrating, we get f ( x ) = kx + c f ( I + ) = 2 I + 1 or f ( I - ) = 2 I - 1
So, f ( x ) is discontinuous at every integer, i.e. {1, 0, - 1 }.
390 Textbook of Differential Calculus

ì 1 - 1, 1 < x £ 2 ì 1 + x 3, x£0
ï x =1 ï
ï 0, \ f (x ) = í 1, 0 £ x £ 1 or x ³ 2
103. (A) f ( x ) í
ï x 2 - 3 x + 3, 1 £ x £ 2
ï 1 - x, 0 £ x < 1 î
ïî - sin px, -1 £ x < 0
Clearly, f ( x ) is not differentiable at 2 points.
At x = 0, f ( x ) is not continuous and not differentiable. At 1
x = 1, f ( x ) is continuous and non-differentiable. At x = 2 (C) f ( x ) = ( x + 4 ) 3
and -1, f ( x ) is continuous and differentiable.
1
1 Þ f ¢( x ) =
-
- h2 2
(B) f ( 0 ) = lim h e 2 h = lim 1/h = 0
h®0 h ® 0e 3( x + 4 ) 3
x \Functions non-derivable at x = - 4, i.e., at one point.
(C) f ( x ) = , not defined at x = - 1.
x+1 ì p æ 2x ö p p
f (x ) ï - 2 × log çè p ÷ø + 2 , 0 < x £ 2
g( x ) = (D) f ( x ) = í
f (x ) + 2 ï p 3p
p - x, <x<
g( x ) is not defined at f ( x ) = - 2 î 2 2
x 2 ì p p
Þ
x+1
= -2 Þ x = - ï- , 0 < x < 2
3 f ¢( x ) = í 2 x
p 3p
Also, x = 0 is not in domain. ï -1, <x<
î 2 2
\ f ( x ) is not differentiable at 3 points.
ì 1, x 2 - 1 > 0 ì 1, | x | > 1 æ p- ö æ p+ ö
f ¢ç ÷ = f ¢ç ÷ = -1
ï ï è 2 ø è 2 ø
(D) y = sgn( x 2 - 1 ) = í 0, x 2 - 1 = 0 = í 0, | x | = 1
ï -1, x 2 - 1 < 0 ï -1, | x | < 1 æ 3p ö
î î \ f ( x ) is differentiable for all x Î ç 0, ÷.
è 2 ø
\ Tangent exists for all x.
\ Number of points of non-differentiable is 0.
\ Number of points where tangent does not exists is 0.
cosh - 1 –h + 1 – 1
tan x + cot x tan x - cot x 105. (A) Rf ¢( 0) = lim = 0 and Lf ¢( 0 ) = lim = –1
104. (A) f ( x ) = – h®0 h h®0 –h
2 2 Obviously,
ì cot x, tan x ³ cot x f (0) = f (0- ) = f (0+ ) = 1

î tan x, cot x > tan x Hence, continuous and not derivable.
Y (B) g( x ) = 0 for all x, hence continuous and derivable.
(C) As 0 £ { x } < 1, hence h( x ) = { x } 2 = { x } which is
discontinuous, hence non-derivable all x Î I .
1

(D) lim x ln x
= lim x log x e = e = f (1 )
x ®1 x ®1
X′ X
–π/2 O π/2 π 3π/2 2π
π/4

o X
1
Y′
Hence, k( x ) is constant for all x > 0, hence continuous and
There are 4 points where the function is continuous but not differentiable at x = 1.
differentiable in ( 0, 2p ).
2
+ 1é x 4 + 1 - ( x 2 + 1) ù
(B) f ( x ) = min {1, 1 + x 3, x 2 - 3 x + 3 } can be shown as 106. (A) l = lim e x êëe - 1ú
x®¥ û
Y 1+x3
x2–3x+3 Consider, lim [ x 4 + 1 - ( x 2 + 1 )]
x®¥

x 4 + 1 - (x 4 + 1 + 2x 2 )
(0, 1) = lim
y=1 x®¥ x 4 + 1 + (x 2 + 1)
O X -2 x 2
= lim = -1
x®¥ x ( 1 + (1 / x 4 ) + 1 + 1/x 2 )
2
Chap 06 Continuity and Differentiability 391

Now, as x ® ¥, x 4 + 1 - ( x 2 + 1 ) ® - 1 On substituting the value of f ¢(2 ) = 7 + 2 f ¢( 4 ) in Eq. (i), we get


f ¢(1 ) - 2[7 + 2 f ¢( 4 )] = 5
æ1 ö
¥ ´ ç - 1 ÷ ® - ¥ and hence limit doesn’t exist. f ¢(1 ) - 4 f ¢( 4 ) = 19
èe ø
2 Þ f ¢(1 ) - 4 f ¢( 4 ) - 10 = 9
a 2h - 2ah + 1 æ ah - 1 ö
(B) f ( 0 + ) = lim = lim ç ÷ = ln 2 a
h®0 h 2 h®0è h ø 113. A, B, C are in AP.
-
f ( 0 ) = lim 3 ln (a + h ) - 2 = 3 ln a - 2 = f ( 0 ) \ 2B = A + C and A + B + C = 180°
h®0 \ B = 60°
For continuous 1 a2 + c2 - b2
\ cos B = =
ln 2 a = 3 ln a - 2 ln 2 a - 3 ln a + 2 = 0 2 2ac
Þ a 2 + c 2 = b 2 + ac
Þ (ln a - 2 ) (ln a - 1 ) = 0 ; a = e 2 or a = e
Þ (a - c ) 2 = b 2 - ac
(C) L = aa ln ae
or | sin A - sin C | = sin 2 B - sin A sin C
Þ M = aa
\ aa ln ae = 2aa æA + Cö æA -Cö 3
Þ 2 cos ç ÷ sin ç ÷ = - sin A sin C
è 2 ø è 2 ø 4
\ ln ae = 2 Þ ae = e 2 Þ a = e
æA -Cö
p 3p Þ 2 sin ç ÷ = 3 – 4 sin A sin C
107. tan x is discontinuous at x = ,
2 è 2 ø
2 2
p 3p æA -Cö
2 sin ç ÷
sec x is discontinuous at x = ,
2
3 - 4 sin A sin C è 2 ø
2 2 \ lim = lim
Þ Number of discontinuities = 2
A ®C | A - C| A ®C | A - C|
1 - ln x æA -Cö
108. Let g( x ) = x1/x , g ¢( x ) = x1/x + – sin ç ÷
x2 è 2 ø
e = lim
Þ g max = e Î (1, 2 )
1/e A ®C æA -Cö
ç ÷
è 2 ø
Þ lim x1/x = 0, lim x1/x = 1
x®0 x®¥ = | 1| = 1 Þ f (x ) = 1
ì 0, 0 < x < 1 \ f ¢( x ) = 0
So, f (x ) = í
î 1, 1 £ x < ¥ æ æ 3 ö 1/h æ 3 ö -1/h ö
çç ç ÷ - ç ÷ ÷÷ - 0
109. f ( x ) = x + cos x + 2, f ( 0) = 3 Þ g (3) = 0 èè 4ø è 4ø ø
g( f ( x )) = x Þ g ¢ ( f ( x )) × f ¢( x ) = 1, putting x = 0 114. RHD = lim h = -1
h®0 æ æ 3 ö 1/h æ 3 ö -1/h ö
g ¢(3 ) × f ¢( 0 ) = 1 çç ç ÷ + ç ÷ ÷÷ × h
èè 4ø è 4ø ø
Now, f ¢( x ) = 1 – sin x Þ f ¢( 0 ) = 1 Þ g ¢(3 ) = 1
-
æ æ 3 ö 1/h æ 3 ö 1/h ö
110. Let g( x ) = x tan -1 ( x 2 ). It is an odd function. çç ç ÷ - ç ÷ ÷÷ - 0
g 2m ( 0 ) = 0. èè 4ø è 4ø ø
So, LHD = lim( -h ) =1
h®0 æ æ 3 ö -1/h æ 3 ö 1/h ö
Let h( x ) = x 4 çç ç ÷ + ç ÷ ÷÷ × ( -h )
So, f ( x ) = g( x ) + h( x ) èè 4ø è 4ø ø

Þ f 2m ( 0 ) = g 2m ( 0 ) + h 2m ( 0 ) \ P = f ¢( 0 - ) - f ¢( 0 + ) = 1 - ( - 1 ) = 2
æ [ x + 1] ö
= h 2m ( 0 ) ¹ 0
ç (exp (( x + 2 ) log 4 ) 4 - 16 ÷
It happens when 2m = 4 Þ m = 2 Now, 4 × ç lim ÷
ç x ®2
-
4 x - 16 ÷
111. F ( x ) = 3e x and G( x ) = e -x è ø
The equation 9 x 4 = ( F ( x )) 2 G( x ) becomes x 4 = e x é [ x + 1] ù
ê ( 4 x + 2 ) 4 - 16 ú 1
Hence, number of solutions = 2 = 4 × ê lim ú = 4´2 =2
x ® 2- 4 x - 16
112. y ¢ = f ¢( x ) - 2 f ¢(2x ) ê ú
ë û
y ¢(1 ) = f ¢(1 ) - 2 f ¢(2 ) = 5 ...(i)
115. Here, f ( x ) = -x 3 + x 2 - x + 1 Þ f ¢( x ) = -3x 2 + 2x - 1
and y ¢(2 ) = f ¢(2 ) - 2 f ¢( 4 ) = 7 ...(ii)
Þ D = 4 - 12 = - 8 < 0
Now, let y = f ( x ) - f ( 4 x ) - 10 x
\ f ( x ) is decreasing.
y¢ = f ¢( x ) - 4 f ¢( 4 x ) - 10
Thus, min f (t ) = f ( x ), as f ( x ) is decreasing and 0 £ t £ x.
y ¢(1 ) = f ¢(1 ) - 4 f ¢( 4 ) - 10 ...(iii)
392 Textbook of Differential Calculus

ì -x 3 + x 2 - x + 1, 0 < x £ 1 which shows the graph of f ( x ) as shown in figure.


\ g (x ) = í
îx - 1 , 1 < x £ 2 Y

Þ lim g( g( x )) = lim g( 0 + ) = 1
x ®1+ x ®1+

Þ lim g( g( x )) = lim g( 0 + ) = 1 XN X
x ®1- x ®1- –2 –1 O 1 2 3
\ lim g ( g( x )) = 1 –1
x ®1
116. We have, f ( x ) = ( x - | x | ) 2 (1 - x +| x| ) 2 YN
ì(2 x ) 2 (1 - 2 x ) 2, x<0 Thus, f ( x ) is discontinuous at x Î integers.
Þ f (x ) = í
î 0, x³0 ì–1, | x | < 1
x 2n – 1 ï
ì16 x 4 - 16 x 3 + 4 x 2, (ii) g ( x ) = lim 2n = í 0, | x | = 1
x<0 n®¥ x +1 ï
=í | x | >1
x³0 î1,
î 0,
which can be shown as in the figure.
Clearly, f ( x ) is continuous as well as derivable for all x Î R.
Y
\ Number of points of non-differentiable = 0.
1
æp -1 2 ö -1
ç - sin (1 - h ) ÷ sin (1 - h )
è2 ø XN X
117. RHL = lim –2 –1 O 1 2
h®0 2(h - h 3 )
–1
cos-1(1 - h 2 )sin -1(1 - h )
= lim
h®0 2h (1 - h 2 ) YN
Thus, g ( x ) is discontinuous at x = ± 1.
sin -1 2h 2 - h 4 × sin -1(1 - h )
= lim ì x [ x ], 0 £ x <2
h®0 2h (1 + h )(1 - h ) 120. f ( x ) = í
î ( x – 1 ) [ x ], 2 £ x £3
sin -1(h 2 - h 2 )sin -1(1 - h ) To check continuity at x = 1
= lim
h®0 2h (1 + h )(1 - h ) RHL (at x = 1) = lim (1 + h ) [1 + h ] = 1
h®0
1 sin -1(h 2 - h 2 ) 2 - h 2 sin -1(1 - h ) LHL (at x = 1) = lim (1 – h ) [1 – h ] = 0
= lim × × × h®0
h®0 2 h × 2 - h2 1+h (1 - h)
Hence, the function is discontinuous at x = 1.
1 sin -1(1 ) p p To check continuity at x = 2
= × 2× = Þ k= …(i)
2 1 2 2 RHL (at x = 2) = lim (2 + h – 1 ) [2 + h ] = 2
h®0
A sin -1(1 –(1 - h )) × cos-1(1 - (1 - h ))
LHL = lim LHL (at x = 2) = lim (2 – h ) [2 – h ] = 2 Þ f (2 ) = (2 – 1 ) [2 ] = 2
h®0 2 (1 - h ) × (1 - (1 - h)) h®0

-1 -1
Hence, the function is continuous at x = 2.
A × sin (h ) × cos (h ) A × p /2 Ap
= lim = = To check differentiability at x = 2
h®0 2(1 - h ) × h 2 2 2 f (2 + h ) – f (2 )
Ap p RHD (at x = 2) = lim
Þ = ÞA = 2 h®0 h
2 2 2 (2 + h – 1 ) [2 + h ]– 2
æ Ap ö = lim
2æ p ö
Hence, sin 2 k + cos2 ç ÷ = sin ç ÷ + cos ( p ) = 1 + 1 = 2.
2 h®0 h
è 2ø è2ø (1 + h ) 2 – 2
= lim =2
118. f ( x ) = [[ x ]] – [ x – 1 ] h®0 h
or f (x ) = [ x ] – [ x ] + 1 LHD (at x = 2) = lim
(2 – h ) [2 – h ]– 2
or f ( x ) = 1, which is constant function and which is h®0 –h
continuous for all real numbers. 2–h–2
Þ lim =1
ì x – x, x Î integers h®0 –h
119. (i) f ( x ) = [ x ] + [– x ] = í
î [ x ]– 1 –[ x ], x Ï integers which shows f ( x ) is not differentiable at x = 2.
ì 0, x Î integers Also, f ( x ) is not differentiable at x = 1, as f ( x ) at x = 1 is not
\ f (x ) = í continuous.
î–1, x Ï integers
Chap 06 Continuity and Differentiability 393

121. Differentiating both the sides, we get \ f ( x ) = px + q


h ¢( x ) = 2 f ( x ) × f ¢( x ) + 2 g ( x ) × g ¢( x ) which shows f ( x ) is differentiable for all x in R.
h ¢( x ) = 2 f ( x ) × g ( x ) + 2 g ( x ) × f ¢¢( x ) 124. We have,
[as g ( x ) = f ¢( x ), g ¢( x ) = f ¢¢( x )] f (x + h ) – f (x )
f ¢( x ) = lim
h®0 h
h ¢( x ) = 2 f ( x ) × g ( x ) – 2 g ( x ) × f ( x ) [Q f ¢¢( x ) = – f ( x )]
h ¢( x ) = 0 f ( x ) + f (h ) + 2hx – 1 – f ( x )
= lim
h®0
\ h ( x ) must be constant function. h
Given, h (5 ) = 11 [using given definition]
Hence, h (10 ) = 11 ì f (h ) – 1ü
= lim í2 x + ý
h®0î h þ
122. Given that,
f ( x + y ) = f ( x ) × f (y ) for all x Î R ...(i) Now, substituting x = y = 0 in the given functional relation,
On putting x = y = 0 in Eq. (i), we get we get
f (0) { f (0) – 1 } = 0 f (0) = f (0) + f (0) + 0 – 1
Þ f ( 0 ) = 0 or f ( 0 ) = 1 Þ f (0) = 1
If f ( 0 ) = 0, then f (h ) – f ( 0 )
\ f ¢( x ) = 2 x + lim
h®0
f (x ) = f (x + 0) = f (x ) × f (0) h
Þ f ( x ) = 0 for all x Î R = 2 x + f ¢( 0 )
which is not true, so f ( 0 ) = 1 Þ f ¢( x ) = 2 x + cos a
f (x + h ) – f (x ) On integrating, f ( x ) = x 2 + x cos a + C
\ f ¢( x ) = lim
h®0 h Here, at x = 0, f ( 0 ) = 1
f ( x ) × f (h ) – f ( x ) \ 1 = C Þ f ( x ) = x 2 + x cos a + 1
= lim [from Eq. (i)]
h®0 h
It is a quadratic in x with discriminant,
ì f (h ) – 1ü
= lim f ( x )í ý D = cos2 a – 4 < 0
h®0 î h þ
= f ( x ) × f ¢( 0 ) and coefficient of x 2 = 1 > 0
= 2 f (x ) [given f ¢( 0 ) = 2] \ f ( x ) > 0, " x Î R
f ¢( x ) 125. Plan If a continuous function has values of opposite sign
or =2
f (x ) inside an interval, then it has a root in that interval.
On integrating both the sides, we get loge f ( x ) = 2 x + C f , g : [ 0, 1 ] ® R
At x = 0, f ( 0 ) = 1 We take two cases.
Hence, log f (1 ) = 2 ( 0 ) + C Case I Let f and g attain their common maximum value at p.
Þ log 1 = 0 + C Þ f ( p ) = g( p ),
Þ C =0 where p Î[ 0,1 ]
Þ loge f ( x ) = 2 x + 0 Case II Let f and g attain their common maximum value at
\ f ( x ) = e 2x different points.
123. Since, f ( x ) is differentiable at x = 0. Þ f (a ) = M and g(b ) = M
f (0 + h ) – f (0) Þ f (a ) - g(a ) > 0 and f (b ) - g(b ) < 0
Þ lim = p (say) ...(i) Þ f (c ) - g(c ) = 0 for some c Î[ 0, 1 ] as f and g are continuous
h®0 h
functions.
f (x + h ) – f (x )
Then, f ¢( x ) = lim Þ f (c ) - g(c ) = 0 for some c Î[ 0, 1 ] for all cases. ...(i)
h®0 h
Option (a) Þ f 2(c ) - g 2(c ) + 3 [ f (c ) - g(c )] = 0
ì 3 x + 3h ü æ 3x + 3 × 0 ö
f í ý – f çè ÷
ø which is true from Eq. (i).
or f ¢( x ) = lim î 3 þ 3
h®0 h Option (d) Þ f 2(c ) - g 2(c ) = 0, which is true from Eq. (i).
f (3 x ) + f (3h ) + f ( 0 )– f (3 x )– f ( 0 )– f ( 0 ) Now, if we take f ( x ) = 1 and g( x ) = 1, "x Î [ 0, 1 ]
or f ¢( x ) = lim
h®0 3h Options (b) and (c) does not hold.
f (3h ) – f ( 0 ) Hence, options (a) and (d) are correct.
or f ¢( x ) = lim
h®0 3h 126. Plan
é æ x + y ö f ( x ) + f (y ) + f ( 0 ) ù (i) In these type of questions, we draw the graph of the function.
ê using f çè 3 ÷ø = ú
ë 3 û (ii) The points at which the curve taken a sharp turn, are the
or f ¢( x ) = f ¢( 0 ) or f ¢( x ) = p (let) [from Eq. (i)] points of non-differentiability. Curve of f ( x ) and g( x ) are
394 Textbook of Differential Calculus

Y æ ph ö
x2 + 1 (2 - h ) 2 ×sin ç - ÷
è 2 (2 - h ) ø -p
|x| + 1 = lim ´ =-p
h®0 -p 2 (2 -h )

2 (2 - h )
1 Thus, f ( x ) is differentiable at x = 0 but not differentiable at x =2.
X
–1 O 1 128. f (2n ) = an , f (2n + ) = an
f (2n - ) = bn + 1 Þ an - bn = 1
h( x ) is not differentiable at x = ± 1 and 0.
As, h( x ) take sharp turns at x = ± 1 and 0. f (2n + 1 ) = an Þ f {(2n + 1 ) - } = an
Hence, number of points of non-differentiability of h ( x ) is 3. f {(2n + 1 ) + } = bn + 1 - 1
127. Plan To check differentiability at a point we use RHD and Þ an = bn + 1 - 1 or an - bn + 1 = - 1
LHD at a point and if RHD = LHD, then f ( x ) is differentiable at or an - 1 - bn = - 1
the point.
129. f ( x + y ) = f ( x ) + f (y ), as f ( x ) is differentiable at x = 0.
Description of Situation
f (x + h ) - f (x ) Þ f ¢ (0) = k …(i)
As, R{ f ¢( x )} = lim f (x + h ) - f (x )
h®0 h Now, f ¢( x ) = lim
h®0 h
f (x -h ) - f (x )
and L{ f ¢( x )} = lim f ( x ) + f (h ) - f ( x ) f (h ) é0 ù
h®0 -h = lim = lim
h®0 h h ® 0 h êë 0 form úû
To check differentiable at x = 0,
f (0 + h ) - f (0) é given, f ( x + y ) = f ( x ) + f (y ), " x, y ù
R { f ¢( 0 )} = lim ê\ ú
h®0 h f ( 0 ) = f ( 0 ) + f ( 0 ),
ê ú
p êë when x = y = 0 Þ f (0) = 0 úû
h 2 cos - 0
h p Using L’Hospital’s rule,
= lim = lim h × cos = 0
h®0 h h®0 h f ¢(h )
lim = f ¢( 0 ) = k …(ii)
æ pö h®0 1
h 2 cos ç - ÷ - 0
f (0 -h ) - f (0) è hø Þ f ¢( x ) = k, integrating both sides, we get
L {f ¢( 0 )} = lim = lim =0
h®0 -h h®0 -h f ( x ) = kx + C , as f ( 0 ) = 0 Þ C = 0
So, f ( x ) is differentiable at x = 0. \ f ( x ) = kx
To check differentiability at x =2, \ f ( x ) is continuous for all x Î R and f ¢ ( x ) = k, i.e. constant
for all x Î R. Hence, (b) and (c) are correct.
f (2 + h ) - f (2 )
R{f ¢(2 )} = lim ì p p
h®0 h ï- x - 2 , x£-
2
æ p ö æ p ö ïï p
(2 + h ) 2 cosç ÷ -0 (2 + h ) 2 × cos ç ÷ 130. We have, f ( x ) = í - cos x, - < x £ 0
è2+hø è2+hø 2
= lim = lim ï x - 1, 0 < x £1
h®0 h h®0 h ï
ïî log x, x >1
æ p p ö
(2 + h ) 2 ×sin ç - ÷ p æ pö æ pö p
è 2 2+hø Continuity at x = - , f ç- ÷ = - ç- ÷ - = 0
= lim 2 è 2 ø è 2ø 2
h®0 h
æ p ö
æ ph ö RHL = lim - cos ç - + h ÷ = 0
(2 + h ) 2 ×sin ç h®0 è 2 ø
÷
è 2 (2 + h ) ø p æ pö
= lim × =p \ Continuous at x = ç - ÷ .
h®0 p 2 (2 + h) è 2ø
h× Continuity at x = 0 Þ f ( 0 ) = - 1
2 (2 + h )
RHL = lim ( 0 + h ) - 1 = - 1
f (2 - h ) - f (2 ) h®0
and L { f ¢(2 )} = lim \ Continuous at x = 0. Continuity at x = 1, f (1 ) = 0
h®0 -h
RHL = lim log (1 + h ) = 0
p p h®0
(2 - h ) 2 × cos - 2 2 × cos
2 -h 2 \ Continuous at x = 1
= lim
h®0 -h ì p
æ p ö ï - 1, x£-
2
(2 - h ) 2 ç - cos ÷ -0 ï
è 2 -h ø p
= lim ïsin x, - < x £ 0
f ¢( x ) = í 2
h®0 -h
ï 1, 0 < x £1
æp p ö ï 1
- (2 - h ) 2 ×sin ç - ÷ ï , x >1
è 2 2 -h ø î x
= lim
h®0 h
Chap 06 Continuity and Differentiability 395

Differentiable at x = 0, LHD = 0, RHD = 1 Þ n = 2 and


n
=1
\ Not differentiable at x = 0 m
Differentiable at x = 1, LHD = 1, RHD = 1 \ m =n =2
\ Differentiable at x = 1. g( x ) cos x - g( 0 ) é0 ù
3 133. We have, lim êë 0 form úû
Also, for x = - Þ f ( x ) = - cos x x ®0 sin x
2
3 g ¢( x ) cos x - g( x )sin x
\ Differentiable at x = - = lim =0
2 x ®0 cos x
1 Statement I
131. Given, f ( x ) = x cos , x ³ 1
x Since, f ( x ) = g( x )sin x
1 1 1
Þ f ¢( x ) = sin + cos f ¢( x ) = g ¢( x ) sin x + g( x ) cos x
x x x
and f ¢¢( x ) = g ¢¢( x ) sin x + 2 g ¢( x ) cos x - g( x ) sin x
1 æ1ö
Þ f ¢¢( x ) = - 3 cos ç ÷ Þ f ¢¢( 0 ) = 0
x èxø
Thus, lim [ g( x ) cos x - g ( 0 )] cosec x = 0 = f ¢¢( 0 )
Now lim f ¢( x ) = 0 + 1 = 1 Þ Option (b) is correct. x ®0
x®¥ Þ Statement I is true.
1
Now, x Î [1, ¥ ) Þ Î ( 0, 1 ] Þ f ¢¢( x ) < 0 Statement II f ¢( x ) = g ¢( x ) sin x + g( x ) cos x Þ f ¢( 0 ) = g ( 0 )
x
Option (d) is correct. Statement II is not a correct explanation of Statement I.
As, f ¢(1 ) = sin 1 + cos1 > 1 134. Match the Columns
f ¢( x ) is strictly decreasing and lim f ¢( x ) = 1 A. x | x | is continuous, differentiable and strictly
x®¥ increasing in ( -1, 1 ) .
So, graph of f ¢( x ) is shown as below B. | x | is continuous in ( -1, 1 ) and not differentiable at x = 0.
Y
C. x + [ x ] is strictly increasing in ( -1, 1 ) and discontinuous at
(1, sin1+cos1) x=0
Þ not differentiable at x = 0.
1 D. | x - 1 | + | x + 1 | = 2 in ( -1, 1 )
X Þ The function is continuous and differentiable in ( -1, 1 ).
0 1
135. Here, f ( x ) = min { 1, x 2, x 3 } which could be graphically shown as
Y y = x2
Now, in [ x , x + 2 ], x Î [1, ¥ ), f ( x ) is continuous and y = x3
f (x + 2) - f (x )
differentiable so by LMVT, f ¢( x ) =
2 y=1
As, f ¢( x ) > 1
X′ X
For all x Î [1, ¥ ) O 1
f (x + 2) - f (x )
Þ > 1 Þ f (x + 2) - f (x ) > 2
2 Y′
For all x Î [1, ¥ ).
Þ f ( x ) is continuous for x Î R and not differentiable at x = 1
( x - 1 )n
132. Given, g( x ) = ; 0 < x < 2, m ¹ 0, n are integers due to sharp edge. Hence, (a) and (d) are correct answers.
log cosm ( x - 1 )
136. Using graphical transformation. As, we know that, the
ì x - 1, x ³ 1 function is not differentiable at sharp edges.
and | x - 1 | = í
î 1 - x, x < 1 Y Y
The left hand derivative of | x - 1 | at x = 1 is p = - 1. y = |x| – 1
O
Also, lim g( x ) = p = - 1 O 1
X
–1 1
X
x ® 1+
(1 + h - 1 )n –1 –1
Þ lim = -1
h ® 0 log cosm (1 + h - 1 )
(i) y = x – 1 (ii) y = |x| – 1
hn hn Y
Þ lim m
= - 1 Þ lim = -1
h ® 0 log cos h h ® 0 m log cos h

n × hn - 1 –1 O1
X
Þ lim = - 1 [using L’Hospital’s rule]
h®0 1
m ( - sin h )
cosh (iii) y = ||x| – 1|
n-2
æ nö h ænö hn - 2
Þ lim ç - ÷ × = - 1 Þ ç ÷ lim =1
h ® 0 è m ø æ tan h ö è m ø h ® 0 æ tan h ö In function, y = || x | - 1|, we have 3 sharp edges at x = - 1, 0, 1.
ç ÷ ç ÷
è h ø è h ø Hence, f ( x ) is not differentiable at { 0, ± 1 }.
396 Textbook of Differential Calculus

æ1ö æ1ö æ1ö and at x = 3, LHL = 2k


137. Given, f (1) = f ç ÷ = f ç ÷ = K = lim f ç ÷ = 0
è2ø è3ø n®¥ ènø \ 2k = 3m + 2 …(i)
æ1ö ì k
as f ç ÷ = 0; n Î integers and n ³ 1. ï , 0 £ x <3
ènø Also, g ¢( x ) = í 2 x + 1
æ1ö
Þ lim f ç ÷ = 0 ïî m, 3 < x £ 5
n®¥ ènø
k
Þ f (0) = 0 \ L { g ¢(3 )} = and R { g ¢(3 )} = m
4
Since, there are infinitely many points in neighbourhood of x = 0.
k
\ f (x ) = 0 Þ = m, i.e. k = 4m …(ii)
4
Þ f ¢( x ) = 0 Þ f ¢( 0 ) = 0
On solving Eqs. (i) and (ii), we get
Hence, f ( 0 ) = f ¢( 0 ) = 0 8 2
k = ,m =
ì1 5 5
ï 2 (– x - 1 ), if x < - 1
Þ k + m =2
ï
138. Given, f ( x ) = í tan -1 x, if - 1 £ x £ 1 143. Given, f ( 0) = 2 = g(1), g( 0) = 0 and f (1) = 6
ï1
ï 2 ( x - 1 ), if x > 1 f and g are differentiable in (0, 1).
î Let h( x ) = f ( x ) - 2 g( x ) …(i)
f ( x ) is discontinuous at x = - 1 and x = 1. h( 0 ) = f ( 0 ) - 2 g( 0 ) = 2 - 0 = 2
\ Domain of f ¢ ( x ) Î R - { -1, 1 }
and h(1 ) = f (1 ) - 2 g(1 ) = 6 - 2(2 ) = 2
139. Given, f ( x ) = [ x ] sin p x Þ h( 0 ) = h(1 ) = 2
If x is just less than k, [ x ] = k - 1 Hence, using Rolle’s theorem,
Þ f ( x ) = (k - 1 ) sin p x. h ¢(c ) = 0, such that c Î( 0, 1 )
(k - 1 ) sin p x - k sin p k
LHD of f ( x ) = lim Differentiating Eq. (i) at c , we get
x ®k x -k Þ f ¢(c ) - 2 g ¢(c ) = 0
(k - 1 ) sin p x Þ f ¢(c ) = 2 g ¢(c )
= lim ,
x ®k x -k
144. Given A function f : R ® R defined by
(k - 1 ) sin p (k - h ) æ 1ö
= lim [where x = k - h ] f ( x ) = [ x ] cos p ç x - ÷ , where [] denotes the greatest integer
h®0 -h è 2ø
k -1
(k - 1 ) ( -1 ) × sin h p function.
= lim = ( -1 )k (k - 1 ) p
h®0 -h To discuss The continuity of function f .
sin h - h Now, cos x is continuous, "x Î R.
140. RHD of sin (| x | ) - | x | = hlim® 0 = 1 - 1 = 0[Q f ( 0 ) = 0 ]
h æ 1ö
LHD of sin (| x | ) - | x | Þ cos p ç x - ÷ is also continuous, "x Î R.
è 2ø
sin | - h | - | - h |
= lim Hence, the continuity of f depends upon the continuity of [ x ].
h®0 -h
sin h - h Since, [ x ] is discontinuous, "x Î I .
= lim =0 So, we should check the conitnuity of f at x = n, "n Î I .
h®0 -h
Therefore, (d) is the answer. LHL at x = n is given by
f (n - ) = lim f ( x )
141. We have, f ( x ) = | log 2 - sin x | and g( x ) = f ( f ( x )), x Î R x ® n-
Note that, for x ® 0, log 2 > sin x æ 1ö (2n - 1 ) p
\ f ( x ) = log 2 - sin x = lim [ x ] cos p ç x - ÷ = (n - 1 ) cos =0
x ® n- è 2ø 2
Þ g( x ) = log 2 - sin ( f ( x ))
RHL at x = n is given by
= log 2 - sin ( log 2 - sin x )
æ 1ö
Clearly, g( x ) is differentiable at x = 0 as sin x is diferentiable. f (n + ) = lim f ( x ) = lim [ x ] cos p ç x - ÷
x ® n+ x ® n+ è 2ø
Now, g ¢( x ) = - cos (log 2 - sin x ) ( - cos x )
(2n - 1 ) p
= cos x × cos (log 2 - sin x ) = (n ) cos =0
2
Þ g ¢( 0 ) = 1 × cos (log 2 )
Also, value of the function at x = n is
142. Since, g( x ) is differentiable Þ g( x ) must be continuous. æ 1ö (2n - 1 ) p
f (n ) = [n ] cos p çn - ÷ = (n ) cos =0
ìk x + 1 , 0 £ x £ 3 è 2ø 2
\ g( x ) = í
î mx + 2, 3 < x £ 5 \ f (n + ) = f (n - ) = f (n )
At x = 3, RHL = 3m + 2 Hence, f is continuous at x = n, "n Î I .
CHAPTER

07
dy/dx as a Rate
Measurer and
Tangents, Normals
Learning Part
Session 1
● Derivative as the Rate of Change
● Velocity and Acceleration in Rectilinear Motion
Session 2
● Differential and Approximation
● Geometrical Meaning of Dx, Dy , dx and dy
Session 3
● Slope of Tangent and Normal
● Equation of Tangent
● Equation of Normal
Session 4
● Angle of Intersection of Two Curves
● Length of Tangent, Subtangent, Normal and Subnormal
Session 5
● Rolle’s Theorem
● Lagrange’s Mean Value Theorem
Session 6
● Application of Cubic Functions

Practice Part
● JEE Type Examples
● Chapter Exercises

Arihant on Your Mobile !


Exercises with this #L
symbol can be practised on your mobile. See title inside to activate for free.
Session 1
Derivative as the Rate of Change,
Velocity and Acceleration in Rectilinear Motion
Derivative as the Rate of Change In the interval, at which the abscissa changes at a faster rate
If a variable quantity y is some function of time t, i.e. than the ordinate, we must have
y = f (t ), then small change in time Dt have a dx
> 1 or
12
>1
corresponding change Dy in y. dy 3x 2
Dy
Thus, the average rate of change = 4 4 –x 2
Dt or >1 Þ > 0, when x ¹ 0
x2 x2
When limit D t ® 0 is applied, the rate of change becomes
instantaneous and we get the rate of change of y with Þ 4 – x 2 > 0 ; x ¹ 0 Þ x 2 – 4 < 0; x ¹ 0
respect to the instant t. Þ ( x – 2) ( x + 2) < 0 ; x ¹ 0
Dy dy + +
i.e. lim =
D t ® 0 Dt dt –2 – 2

Hence, it is clear that the rate of change of any Using number line rule, we have -2 < x < 2; x ¹ 0
variable with respect to some other variable is the Thus, x Î ( - 2, 2) - {0} is the required interval at which
derivative of first variable with respect to other abscissa changes at a faster rate than the ordinate.
variable.
dy
The differential coefficient of y with respect to x i.e. is
nothing but the rate of increase of y relative to x. dx Velocity and Acceleration in
y Example 1 If the radius of a circle is increasing at a Rectilinear Motion
uniform rate of 2 cm/s, then find the rate of increase of The velocity of a moving particle is defined as the rate of
area of circle, at the instant when the radius is 20 cm. change of its displacement with respect to time and the
dr acceleration is defined as the rate of change of its velocity
Sol. Given, = 2 cm/s [where, r is radius and t is time] with respect to time.
dt
v ∆s v+∆v
Now, area of circle is given by A = pr 2 s
Differentiating it with respect to time t, we get O A (t) B (t+∆t)
dA dr dA Figure 7.1
= 2pr Þ = 2p × 20 × 2 cm 2 /s
dt dt dt Let a particle A move rectilinearly as shown in figure. Let s
dA be the displacement from a fixed point O along the path at
Þ = 80 p cm 2 /s
dt time t ; s is considered to be positive on right of the point
Thus, the rate of change of area of circle with respect to O and negative on the left of it.
time is 80p cm 2 / s. Also, Ds is positive when s increases, i.e. when the particle
moves towards right.
y Example 2 On the curve x 3 = 12y , find the interval at
Thus, if Ds be the increment in s in time Dt, then average
which the abscissa changes at a faster rate than the velocity in this interval is
ordinate. Ds
Average velocity =
Sol. Given, x 3 = 12y Dt
Differentiating it with respect to y, we get and the instantaneous velocity, i.e. velocity at time t is
Ds ds
3x 2
dx
= 12 Þ
dx
=
12 v = lim =
dy dy 3x 2 Dt ® 0 D t dt
Chap 07 dy/dx as a Rate Measurer and Tangents, Normals 399

If the velocity varies, then there is change of velocity Dv (ii) We know, S = 4 pr 2 Þ


dS
= 8pr
dr
...(ii)
in time Dt. Hence, the acceleration at time t is dt dt
Dv dv dV dV/dt 1
= = ×r
Acceleration (a) = lim = Thus, [from Eqs. (i) and (ii)]
Dt ® 0 D t dt dS dS/dt 2
dV 1 dV
Þ = r or µr
y Example 3 If the displacement of a particle is given dS 2 dS
æ1 ö
by s = ç t 2 + 4 t ÷ m. Find the velocity and y Example 6 A man who is 1.6 m tall walks away from
è2 ø
a lamp which is 4 m above ground at the rate of
acceleration at t = 4 seconds.
30 m/min. How fast is the man’s shadow lengthening?
Sol. We have, the displacement of particle is given by,
Sol. Let PQ = 4 m be the height of lamp and AB = 1.6 m be
æ1 ö
s = ç t2 + 4 t ÷ m height of man. Let the end of shadow is R and it is at a
è2 ø distance of l from A when the man is at a distance of x
ds æ 2 ö from PQ, at some instance.
As, we know velocity v = = çt + ÷ m/s ...(i)
dt è tø Q
d 2s
dv æ 1 ö
and acceleration a = 2 = = ç1 – 3 / 2 ÷ m/s ...(ii)
dt dt è t ø 4m
B

æ 2 ö 1.6 m
Now, velocity when t = 4, is v = ç 4 + ÷ m/s [from Eq. (i)]
è 4ø
Þ v = 5 m/s P x A l R
æ 1 ö
and acceleration when t = 4, is a = ç1 – 3 / 2 ÷ m / s 2 Since, D PQR and D ABR are similar, we have
è 4 ø
PQ PR 4 x +l
æ 1ö 7 [from Eq. (i)] = Þ =
Þ a = ç1 – ÷ m / s 2 Þ a = m /s 2 AB AR 1.6 l
è 8ø 8
dx dl é dx ù
Þ 2x = 3l Þ 2 =3 êë given dt = 30 m /min úû
1 dt dt
y Example 4 If s = t 3 – 6t, then find the
2 dl 2
\ = × 30 m/min = 20 m/min (lengthening)
acceleration at time when the velocity tends to dt 3
zero.
1 y Example 7 If x and y are the sides of two squares
Sol. We have the displacement s is given by, s = t 3 – 6t
2 such that y = x – x 2 , find the rate of change of the
ds æ 3t 2 ö
Now, velocity v = =ç – 6÷ ...(i) area of the second square with respect to the first
dt è 2 ø
square.
dv d 2s Sol. Given, x and y are sides of two squares, thus the area of two
and acceleration, a = = = (3t ) ...(ii)
dt dt 2 squares are x 2 and y 2 .
To find acceleration, when velocity ® 0 dy
2y
3t 2 d (y 2 ) y dy
Now, when velocit y ® 0, then –6 = 0 [from Eq. (i)] We have to obtain 2
= dx = × ...(i)
2 d (x ) 2x x dx
Þ t2 = 4 Þ t = 2 The given curve is y = x – x2
Thus, acceleration when velocity tends to zero, is dy
Þ = 1 – 2x ...(ii)
a = 3t = 6 [from Eq. (ii)] dx
d (y 2 ) y
Thus, = ( 1 – 2x ) [from Eqs. (i) and (ii)]
y Example 5 If r is the radius, S the surface area d (x 2 ) x
and V the volume of a spherical bubble, prove that
d (y 2 ) ( x – x 2 ) ( 1 – 2x )
dV dr dV or =
(i) = 4pr 2 (ii) µ r. 2
d (x ) x
dt dt dS
4 3 d (y 2 )
Sol. (i) Since, V =
pr Þ = 2x 2 – 3x + 1
3 d (x 2 )
dV 4 dr dr
\ = p × 3r 2 = 4 pr 2 ...(i)
dt 3 dt dt
400 Textbook of Differential Calculus

Exercise for Session 1


1. The surface area of a spherical bubble is increasing at the rate of 2 cm 2/s. Find the rate at which the volume of
the bubble is increasing at the instant, if its radius is 6 cm.
2 3
2. A particle moves along the curve y = x + 1. Find the point on the curve at which y - coordinate is changing
3
twice as fast as x-coordinate.
3. The area of an expanding rectangle is increasing at the rate of 48 cm2/s. The length of rectangle is always
equal to square of the breadth. At which rate the length is increasing at the instant when the breadth is 4 cm?

4. An edge of a variable cube is increasing at the rate of 10 cm/s. How fast the volume of the cube is increasing
when the edge is 5 cm long?

5. An air-force plane is ascending vertically at the rate of 100 km/h. If the radius of the earth is r km, how fast is
the area of the earth, visible from the plane, increasing at 3 min after it started ascending?
2pr 2h
Note Visible area A = , where h is the height of the plane above the earth.
r +h

6. Sand is pouring from the pipe at the rate of 12 cm 3/s. The falling sand forms a cone on the ground in such a
way that the height of the cone is always one-sixth of the radius of the base. How fast is the height of the sand
cone increasing when the height is 4 cm?

7. Water is dripping out from a conical funnel, at the uniform rate of 2 cm3/s, through a tiny hole at the vertex of
the bottom. When the slant height of the water is 4 cm, find the rate of decrease of the slant height of the water,
if the vertical angle of funnel is 120°.

8. From a cylindrical drum containing oil and kept vertical, the oil leaking at the rate of 10 cm 3/s. If the radius
of the drum is 10 cm and height is 50 cm, then find the rate at which level of oil is changing when oil level is
20 cm.

9. A kite is 120 m high and 130 m of string is out. If the kite is moving away horizontally at the rate of 52 m/s, find the
rate at which the string is being paid out.

10. A ladder 13 m long leans against a wall. The foot of the ladder is pulled along the ground away from the wall, at
the rate of 1.5 m/s. How fast is the angle q between the ladder and the ground changing when the foot of the
ladder is 12 m away from the wall?

11. Water is running into a conical vessel, 15 cm deep and 5 cm in radius, at the rate of 0.1 cm 3/s. When the water
is 6 cm deep, find at what rate is
(a) the water level rising?
(b) the water surface area increasing?
(c) the wetted surface of the vessel increasing?
12. Height of a tank in the form of an inverted cone is 10 m and radius of its circular base is 2 m. The tank contains
water and it is leaking through a hole at its vertex at the rate of 0.02 m3/s. Find the rate at which the water level
changes and the rate at which the radius of water surface changes when height of water level is 5 m.

13. At what points of the ellipse 16x 2 + 9y 2 = 400 does the ordinate decreases at the same rate at which the
abscissa increases?

14. A satellite travels in a circular orbit of radius R. If its x-coordinate decreases at the rate of 2 units/s at the point
(a, b ), how fast is the y -coordinate changing?
15. The ends of a rod AB which is 5 m long moves along two grooves OX , OY which are at right angles. If A moves
1
at a constant speed of m/s, what is the speed of B, when it is 4 m from O?
2
Session 2
Differential and Approximation, Geometrical
Meaning of ∆x, ∆y, dx and dy
Differential and Approximation Þ
dy
× dx = f ( x + Dx ) - f ( x ) (approx)
We are familiar with the symbol dy/dx representing dx
derivative of y w.r.t. x. f ( x + Dx ) - f ( x ) = f ¢ ( x ) × Dx (approx)
Now, we shall give meaning to the symbols dx and dy
such that the symbol dy /dx represents the quotient of dy
and dx.
Errors
Definition of Absolute Error
Let y = f ( x ) be a function of x.
Dx or dx is called absolute error in x.
Let Dx denotes small change in x and let the
Definition of Relative Error
corresponding change in y be Dy.
Dx dx
Dy dy Dy dy or is called relative error in x .
Then, lim = Þ = (approx) x x
Dx ® 0 D x dx Dx dx
Definition of Percentage Error
Dy dy
Þ = + e, where e ® 0 as Dx ® 0 æ Dx ö æ dx ö
Dx dx ç ÷ × 100 or ç ÷ × 100 is called percentage error in x.
è x ø è x ø
dy
Þ Dy = × Dx + e × Dx …(i)
dx
Now, Dy consists of two parts. The part e × Dx is very Geometrical Meaning of
small and hence negligible. The part
dy
× Dx is called Dx , Dy , dx and dy
dx
Let us take a point A ( x , y ) on the curve y = f ( x ), where
principal part of Dy and is denoted by dy and is called
f ( x ) is differentiable real function.
differential of y.
dy Let B ( x + Dx , y + Dy ) be a neighbouring point on the
Thus, dy = × Dx …(ii) curve, where Dx denotes a small change in x and Dy is the
dx
dy corresponding change in y.
Þ dx = × Dx From the figure, it is clear that if D x and Dy are
dy
sufficiently small quantities, then
= ( 1) D x = D x …(iii)
Y
From Eqs. (ii) and (iii), we get
B(x+∆x, y+∆y)
dy
dy = × dx
dx
Þ (Differential of y) = (Differential coefficient of A (x, y)
y w.r.t. x) × (Differential of x)
y=f(x)
Differential of y ψ
Þ = (Differential coefficient of y w.r.t. x) X
Differential of x O x x + ∆x
dy Figure 7.2
From Eq. (i), Dy = × Dx (approx)
dx Dy dy
= tan y – = f ¢ (x )
Þ Dy = dy (approx) Dx dx
Hence, approximately Hence, approximate change in the value of y, called its
dy = Dy = (y + Dy ) - y = f ( x + Dx ) - f ( x ) differential, is given by D y = f ¢ ( x ) × Dx
402 Textbook of Differential Calculus

y Example 8 Use differential to approximate 101. 1


Þ tan -1(0999
. ) = tan -1(1) - (0001
. )×
1 1 + 12
Sol. Let f ( x ) = x , then f ¢ ( x ) = .
2 x p 1 22
= - 00005
. = ´ - 00005
. = 07852
.
Now, let x = 100 and Dx = 1, so that x + Dx = 101 4 4 7

Then, by definition f ( x + Dx ) - f ( x ) = f ¢ ( x ) × Dx , we get y Example 12 The time T of oscillation of a simple


101 - 100 = f ¢ ( 100 ) × 1
l
pendulum of length l is given by T = 2p × .
g
1
101 = 100 + ×1
2 100 Find percentage error in T corresponding to
1 (i) an increase of 2% in the value of l.
= 10 + = 1005
.
20 (ii) decrease of 2% in the value of l.
l
Sol. (i) We have, T = 2p ×
y Example 9 Use differential to approximate (66 )1/ 3 . g
Sol. Let f ( x ) = ( x )1/ 3 1 1
Þ log T = log 2 + log p + log l - log g
1 2 2
Then, f ¢( x ) = Differentiating w.r.t. l, we get
3x 2 / 3
1 dT 1 dT 1 dl
Now, let x = 64, Dx = 2, so that x + Dx = 66 =0+0+ -0 Þ =
T dl 2l T 2 l
Then, by definition f ( x + Dx ) - f ( x ) = f ¢ ( x ) × Dx
dT 1 æ dl ö 1
Þ f (66) - f (64 ) = f ¢ (64 ) × (2) \ ´ 100 = ç ´ 100÷ = (2)% = 1%
T 2è l ø 2
1
Þ (66)1/ 3 - (64 )1/ 3 = ´2 é dl ù
3(64 )2 / 3 êëQ l ´ 100 = 2% úû
Percentage error in T = 1%.
2 97
Þ (66)1/ 3 = 4 + = = 4.0416 dT 1 dl
48 24 (ii) We have, =
T 2 l
y Example 10 If the radius of a circle increases from dT 1 æ dl ö 1
Þ ´ 100 = ç ´ 100÷ = ( -2)% = - 1%
5 cm to 5.1 cm, find the increase in area. T 2è l ø 2
Sol. Let r be the radius of the circle. Then, r = 5 cm and [Q decrease in l = 2% ]
r + dr = 51
. cm, so that dr = 01. cm. Percentage error in T = - 1%.
Let A denotes the areas of the circle.
dA y Example 13 In an acute DABC, if sides a and b
Then, A = pr 2 \ = 2pr
dr are constants and the base angles A and B vary,
dA
\ At r = 5, = 10p show
dr
dA dB
Now, dA =
dA
× dr = 10p ´ (01. ) = p cm 2 that = .
dr a - b sin A
2 2 2
b - a 2 sin 2B
2

\ Increase in area = p cm 2 a b
Sol. = or b sin A = a sin B
sin A sin B
y Example 11 Find the approximate value of Þ b cos A dA = a cos B dB
tan -1 (0999
. ) using differential. dA dB
Þ =
Sol. Let f ( x ) = tan -1 x a cos B b cos A
1 dA dB
Then, f ¢( x ) = Þ =
1+x 2
a 1 - sin B 2
b 1 - sin 2 A
Now, let x = 1 and Dx = -0001 . . So that x + Dx = 0999 . dA dB
Þ =
Then, by definition, 2
b sin A 2
a 2 sin 2 B
a 1- b 1-
f ( x + Dx ) - f ( x ) = f ¢ ( x ) × Dx , we get a 2
b2
. ) - f (1) = f ¢ (1) × ( - 0001
f (0999 . ) dA dB
Þ =
a - b sin A
2 2 2
b - a 2 sin 2 B
2
Exercise for Session 2
1. Use differential to approximate 51 .

2. Use differential to approximate log (9.01). (Given, log 3 = 10986


. )

3. If the error committed in measuring the radius of a circle is 0.01%, find the corresponding error in calculating the
area.

4. The pressure p and the volume V of a gas are connected by the relation pV1/ 4 = a (constant). Find the
æ 1ö
percentage increase in pressure corresponding to a decrease of ç ÷% in volume.
è2ø

5. If in a DABC, the side c and the angle C remain constant, while the remaining elements are changed slightly.
da db
Using differential, show that + = 0.
cos A cos B
6. If a DABC, inscribed in a fixed circle, is slightly varied in such a way as to have its vertices always on the circle,
da db dc
show that + + = 0.
cos A cos B cos C
Session 3
Slope of Tangent and Normal, Equation
of Tangent, Equation of Normal
Slope of Tangent and Normal Þ Slope of normal at P ( x 1 , y 1 ) = –
1
æ dy ö
Slope of Tangent ç ÷
è dx ø ( x1 , y 1 )
Let y = f ( x ) be a continuous curve and let P ( x 1 , y 1 ) be a
æ dx ö
point on it. or Slope of normal at P ( x 1 , y 1 ) = – ç ÷
Y
è dy ø ( x1 , y 1 )
Tangent
Remarks
P (x1, y1) (i) Horizontal normal If normal is parallel to X-axis, then
θ æ dx ö æ dx ö
X -ç ÷ = 0 or ç ÷ = 0
O è dy ø( x 1 , y1 ) è dy ø( x 1 , y1 )
Normal
(ii) Vertical normal If normal is perpendicular to X-axis
or parallel to Y-axis, then - æç ö÷
Figure 7.3 dy
=0
è dx ø( x 1 , y1 )
æ dy ö
Then, ç ÷ is the slope of tangent to the
è dx ø ( x1 , y1 ) y Example 14 Find the slopes of the tangent and
normal to the curve x 3 + 3xy + y 3 = 2 at (1, 1).
curve y = f ( x ) at a point P( x 1 , y 1 ).
æ dy ö Sol. Given equation of curve is x 3 + 3xy + y 3 = 2.
Þ ç ÷ = tan q = Slope of tangent at P
è dx ø P Differentiating it w.r.t. x, we get
dy dy
where, q is the angle which the tangent at P ( x 1 , y 1 ) forms 3x 2 + 3x + 3y + 3y 2 =0
dx dx
with the positive direction of X-axis as shown in the figure.
dy (3x 2 + 3y ) dy (x 2 + y )
Remarks Þ =– Þ = –
dx (3x + 3y 2 ) dx (x + y 2 )
(i) Horizontal tangent If tangent is parallel to X-axis, then
q = 0 ° Þ tan q = 0 æ dy ö æ2ö
Þ ç ÷ = –ç ÷ = – 1
æ dy ö è dx ø (1, 1) è2ø
\ ç ÷ =0
è dx ø ( x 1 , y1 )
æ dy ö
(ii) Vertical tangent If tangent is perpendicular to X-axis or \ Slope of tangent at (1,1) = ç ÷ =–1
è dx ø (1, 1)
parallel to Y-axis, then
q = 90° Þ tan q = ¥ or cot q = 0 1 –1
and slope of normal at (1,1) = – = =1
æ dx ö æ dy ö –1
\ ç ÷ =0 ç ÷
è dy ø ( x 1 , y1 ) è dx ø (1, 1)

y Example 15 Find the point on the curve y = x 3 – 3x


Slope of Normal at which tangent is parallel to X-axis.
We know that the normal to the curve at P ( x 1 , y 1 ) is a Sol. Let the point at which tangent is parallel to X -axis be
line perpendicular to tangent at P ( x 1 , y 1 ) and passes P ( x 1, y1 ).
Then, it must lie on curve.
through P.
Therefore, we have y1 = x 13 – 3x 1 ...(i)
\ Slope of the normal at Differentiating y = x 3 - 3x w.r.t. x, we get
1
P =- dy æ dy ö
Slope of the tangent at P = 3x 2 – 3 Þ ç ÷ = 3x 12 – 3
dx è dx ø (x
1 , y1 )
Chap 07 dy/dx as a Rate Measurer and Tangents, Normals 405

Since, the tangent is parallel to X -axis. f (x 0 + h) - f (x 0 )


lim = ¥ or - ¥ but not both.
æ dy ö h ®0 h
\ ç ÷ = 0 Þ 3x 12 – 3 = 0
è dx ø (x , y ) Here, the functions f ( x ) = x 1/ 3 and f ( x ) = sgn x both have
1 1

Þ x1 = ± 1 ...(ii) a vertical tangent at x = 0, but f ( x ) = x 2 / 3 , f ( x ) = | x | and


From Eqs. (i) and (ii), we get ì0, if x < 0
f (x ) = í have no vertical tangent.
When x 1 = 1, then y1 = 1 – 3 = – 2 î1, if x ³ 0
When x 1 = – 1, then y1 = – 1 + 3 = 2 Explanation
\ Points at which tangent is parallel to X -axis are (1, – 2) (i) f ( x ) = x 1/ 3
and (–1, 2). h 1/ 3 1 ü
f ¢ (0+ ) = lim = 2/3 ® ¥ ïï
h ®0 h h
y Example 16 Find the point on the curve ý
(- h) 1/ 3
1 1
y = x 3 – 2x 2 – x at which the tangent line is parallel to f ¢ (0- ) = lim = = ® ¥ ï
h ®0 -h ( -h ) 2 / 3 h 2 / 3 ïþ
the line y = 3x – 2.
Þ f ( x ) has a vertical tangent at x = 0.
Sol. Let P ( x 1, y1 ) be the required point.
Y
Then, we have y1 = x 13 – 2x 12 – x 1 ...(i)
Differentiating the curve y = x 3 – 2x 2 – x w.r.t. x, we get
X′ X
O
dy æ dy ö
= 3x 2 – 4 x – 1 Þ ç ÷ = 3x 12 – 4 x 1 – 1
dx è dx ø (x , y )
1 1
Y′
Since, tangent at ( x 1, y1 ) is parallel to the line y = 3x – 2.
ì 1, if x > 0
\ Slope of the tangent at P ( x 1, y1 ) = Slope of the line ï
(ii) f ( x ) = sgn x = í 0, if x = 0
æ dy ö ï - 1, if x < 0
y = 3x – 2 Þ ç ÷ =3 î
è dx ø (x , y )
1 1
Y
Þ 3x 12 – 4 x1 – 1 = 3 Þ 3x 12 – 4 x1 – 4 = 0
1
Þ ( x 1 – 2) (3x 1 + 2) = 0 Þ x 1 = 2, – 2/ 3 ...(ii) X′ X
O
From Eqs. (i) and (ii), we get
When x 1 = 2, then −1
Y′
y1 = 8 – 8 – 2 Þ y1 = – 2
1-0 ü
When x 1 = – 2/ 3, then y1 = x 13 – 2x 12 – x 1 f ¢ (0+ ) = lim ® ¥ï
h ®0 h
–8 8 2 –14 -1 ý
Þ y1 =
– + Þ y1 = f ¢ (0- ) = lim ®¥ ï
27 9 3 27 h ®0 - h þ
Thus, the point at which tangent is parallel to y = 3x – 2 are
Þ f ( x ) has a vertical tangent at x = 0.
æ 2 14 ö
(2, – 2) and ç– , – ÷.
è 3 27 ø (iii) f ( x ) = x 2 / 3

y Example 17 In which of the following cases, the h 2/3 ü


f ¢ (0+ ) = lim ®¥ ïï
function f ( x ) has a vertical tangent at x = 0? h ®0 h
ý
( -h ) 2/3
1
(i) f (x ) = x 1/ 3 (ii) f (x ) = sgn x f ¢ (0- ) = lim = - 1/ 3 ® - ¥ï
h ®0 -h h ïþ
(iii) f (x ) = x 2/ 3 (iv) f (x ) = | x | Þ No vertical tangent at x = 0.
ì0, if x < 0 Y
(v) f (x ) = í
î1, if x ³ 0
Sol. Vertical Tangent X′ X
O
Concept y = f ( x ) has a vertical tangent at the points
x = x 0 , if
Y′
406 Textbook of Differential Calculus

é x , if x ³ 0 passing through (a, b ), can be found by solving for the


(iv) f ( x ) = | x | = ê
ë - x , if x < 0
point of contact Q.
Y
Y

Q(h, f(h))
X′ X
O
P(a, b)

Y′ y = f(x)
X
h -0 ü O
f ¢ (0+ ) = lim ® ¥ï
h ®0h ï
ý
- h Figure 7.5
f ¢ (0 ) = lim ®-¥ ï
h ®0 - h ïþ
f (h ) - b
Then, f ¢ (h ) =
Þ No vertical tangent at x = 0. h -a
and equation of tangent is
ì0, if x < 0
(v) f ( x ) = í Y f (h ) - b
î1, if x ³ 0 y -b = × (x - a)
h -a
1-1 ü
f ¢ (0+ ) = lim =0 ï X′ X
h ®0 h O y Example 18 Find value of c such that line joining
0-1 ý
-
f ¢ (0 ) = lim ® - ¥ï c
h ®0 h þ points (0, 3) and ( 5, - 2) becomes tangent to y = .
Y′ x+1
Þ No vertical tangent at x = 0.
Sol. Equation of line joining A(0, 3) and B(5, - 2) is x + y = 3.

Equation of Tangent Solving the line and curve, we get


c
3- x =
Let y = f ( x ) be the equation of curve and point ( x 1 , y 1 ) be x +1
any point on the curve. Let PT be the tangent at point
Þ x 2 - 2x + ( c - 3) = 0 …(i)
( x 1 , y 1 ).
Since, tangent is a line passing through the point P ( x 1 , y 1 ) For tangency, roots of this equation must be equal.
æ dy ö Hence, discriminant of quadratic equation = 0.
and having slopem = ç ÷ , therefore by coordinate Þ 4 = 4 ( c - 3) Þ c = 4
è dx ø ( x1 ,y 1 )
Hence, required value of c is 4.
geometry, the equation of tangent is
æ dy ö
y – y 1 = m (x – x 1 ) Þ y – y 1 = ç ÷ × ( x – x 1 ).
è dx ø ( x1 ,y1 ) Equation of Normal
We know that normal to curve at any point is a straight
Y T line passes through that point and is perpendicular to the
tangent to the curve at that point.
y=f(x) æ dy ö
Since, the slope of tangent at P ( x 1 , y 1 ) = ç ÷
è dx ø ( x1 , y 1 )
nt
nge
Ta

Normal T
θ Y
X¢ X
O

P(x1, y1)
Y′
nt
ge
n

Figure 7.4
Ta

( 90°+ θ)
θ
X′ X
Tangent from External Point O
If a point P (a, b ) does not lie on the curve y = f ( x ), then
the equation of all possible tangents to the curve y = f ( x ), Y′
Figure 7.6
Chap 07 dy/dx as a Rate Measurer and Tangents, Normals 407

Now, as normal is perpendicular to the tangent. and the equation of normal at (at 2 , 2at ) is,
-1 y – 2at 1
\ Slope of normal at P ( x 1 , y 1 ) = =– =–t [using Eq. (ii)]
æ dy ö x – at 2
æ ö
dy
ç ÷ ç ÷
è dx ø ( x1 , y 1 ) è dx ø (at 2 , 2at )
Þ y – 2at = – xt + at 3
Hence, from coordinate geometry equation of normal is
1 Þ y + xt = 2at + at 3 [required equation of normal]
y – y1 = – (x – x 1 )
æ dy ö y Example 21 Find the point on the curve
ç ÷
è dx ø ( x1 , y1 ) y – e xy + x = 0 at which we have vertical tangent.
æ dx ö Sol. The equation of given curve is,
or y – y1 = – ç ÷ (x – x 1 )
è dy ø ( x1 , y1 ) y – e xy + x = 0 ...(i)
Differentiating Eq. (i) w.r.t. x, we get
y Example 19 Find the equation of tangent and normal dy ì dy ü
– e xy í1 × y + x × ý + 1 = 0
to the curve 2y = 3 – x 2 at (1, 1). dx î dx þ
dy
Sol. The equation of given curve is, Þ (1 – xe xy ) = – 1 + y × e xy
dx
2y = 3 – x 2 ...(i)
dy –1 + y × e xy
Differentiating Eq. (i) w.r.t. x, we get Þ = ...(ii)
dx 1 – xe xy
æ dy ö dy
2 ç ÷ = – 2x Þ = –x
è dx ø dx
Let at point ( x 1, y1 ) on the curve, we have a vertical tangent
æ dy ö (i.e. a tangent parallel to Y -axis). Then,
Þ ç ÷ = –1 ...(ii)
è dx ø (1, 1) æ dy ö æ dx ö
ç ÷ = ¥ or ç ÷ =0
è dx ø (x , y ) è dy ø (x , y )
Now, the equation of tangent at (1, 1) is, 1 1 1 1

y – 1 æ dy ö y -1 1 – x 1e x 1y1
Þ =ç ÷ Þ = –1 Þ = 0 Þ 1 – x 1e x 1y 1
=0
x – 1 è dx ø (1, 1) x -1 –1 + y1e x 1y1
Þ x 1e x 1y1 = 1,
Þ y –1 = –x + 1
Þ y + x =2 [required equation of tangent] which is possible only if x 1 = 1 and y1 = 0.
and the equation of the normal at (1, 1) is, Thus, the required point is (1, 0).
y –1 1
=– =1
x –1 (dy / dx )(1, 1) Remarks
For standard curves students are advised to use direct method of
\ y–x =0 [required equation of normal] finding equation of tangent.
y Example 20 Find the equation of tangent and normal For the curve of the form
ax 2 + 2hxy + by 2 + 2gx + 2fy + c = 0, replace
to the parabola y 2 = 4ax at the point (at 2, 2at ).
x 2 by xx1; 2x by x + x1;
Sol. The equation of given curve is, 2
y by yy1 ; 2 y by y + y1
y 2 = 4ax ...(i) xy1 + x1 y
and xy by
dy 2
Differentiating Eq. (i) w.r.t. x, we get 2y = 4a
dx Then, equation of tangent is,
æ dy ö 4a 1 axx1 + h ( xy1 + yx1 ) + byy1 + g ( x + x1 ) + f ( y + y1 ) + c = 0
Þ ç ÷ = = ...(ii)
è dx ø (at 2 , 2at ) 4at t e.g.
(i) Find equation of tangent to the curve 2 y = x 2 + 3 at ( x1, y1 ).
Now, the equation of tangent at (at 2 , 2at ) is
Sol. On replacing 2 y by y + y1 and x 2 by xx1, we get
y – 2at æ dy ö 1 ( y + y1 ) = xx1 + 3, which is the required equation of tangent.
=ç ÷ = [using Eq. (ii)]
x – at 2 è dx ø (at 2 , 2at ) t (ii) Find equation of tangent to the curve y 2 = 4 ax at ( at 2, 2at ) .
Þ (y – 2at )t = x – at 2 Sol. Clearly, the equation of curve y 2 = 4 ax is a standard
equation of curve, therefore on replacing y 2 by yy1 and
Þ yt – 2at 2 = x – at 2 Þ yt = x + at 2
2x by x + x1,
[required equation of tangent]
408 Textbook of Differential Calculus

we can get the equation of tangent. Thus, the required (ii) Equation of tangent at origin to the curve
equation is given by
x 3 + y 3 - 3 x 2 y + 3 xy 2 + x 2 - y 2 = 0 is
yy1 = 2a ( x + x1 )
where ( x1, y1 ) = ( at 2, 2at )
x 2 - y 2 = 0.
Hence, the required equation of tangent is (iii) Equation of tangent at origin, to the curve
y ( 2at ) = 2a ( x + at 2 ) Þ yt = x + at 2 x 3 + y 3 - 3 xy = 0 is xy = 0.
(iii) Find the equation of tangent, at point P( x1, y1 ), to the curve 2. If the curve is x 4 + y 4 = x 2 + y 2 , then the equation of
x2 y2
+ 2 =1 the tangent would be x 2 + y 2 = 0 which would indicate
2
a b that the origin is an isolated point on the graph.
x2 y2 Y
Sol. Clearly, the equation of curve 2 + 2 = 1 is a standard
a b
equation of curve, therefore equation of the tangent can
be obtained by replacing x 2 by xx1 and y 2 by yy1. X′ X
–1 O 1
xx1 yy
i.e. 2
+ 21 = 1
a b
Y′
Some Important Points Regarding Figure 7.8

Tangent and Normal 3. Same line could be the tangent as well as normal to a
1. If a curve passes through the origin, then the equation given curve at a given point.
of the tangent at the origin can be directly written by e.g. In x 3 + y 3 - 3 xy = 0 [folium of descartes]
equating the lowest degree terms appearing in the Y
equation of the curve to zero.
Y X′ X
(x1, y1) O

Y′
X′ X Figure 7.9
O
The line pair xy = 0 is both the tangent as well as
normal at x = 0.
Y′
4. Some common parametric coordinates on a curve
Figure 7.7
(i) For x 2 / 3 + y 2 / 3 = a 2 / 3 , take parametric coordinate
Proof Let the equation of the curve be
x = a cos 3 q and y = a sin 3 q.
a 1 x + b 1 y + a 2 x 2 + b 2 xy + c 2 y 2 = 0 …(i)
(ii) For x + y = a , take x = a cos 4 q
y
Tangent y - 0 = lim 1 ( x - 0 ) and y = a sin 4 q.
x1 ® 0 x 1
y1 ® 0
xn yn
Now, Eq. (i) becomes (iii) For + = 1, take x = a (cos q ) 2 /n
n n
a b
y y y and y = b (sin q ) 2 /n .
a1 + b1 1 + a2 x 1 + b2 1 × x 1 + c 2 1 × y 1 = 0 …(ii)
x1 x1 x1
(iv) For c 2 ( x 2 + y 2 ) = x 2 y 2 , take x = c sec q
y and y = c cosec q.
at x 1 ® 0 and y 1 ® 0, 1 ® m
x1 (v) For y 2 = x 3 , take x = t 2 and y = t 3 .
é a1 ù
From Eq. (ii), a 1 + b 1m = 0 êQ m = - b ú
ë 1û y Example 22 Find the sum of the intercepts on the
a axes of coordinates by any tangent to the curve
Hence, tangent is y = - 1 x
b1 x + y = 2.
Þ a 1 x + b 1y = 0
e.g. Sol. Here, equation of curve is x + y =2.
(i) Equation of tangent at origin, to the curve Whose parametric coordinates are given by,
x 2 + y 2 + 2 gx + 2 fy = 0 is gx + fy = 0. x = 2 cos 2 q
Chap 07 dy/dx as a Rate Measurer and Tangents, Normals 409

and y = 2 sin 2 q PN = length of normal


1 y
i.e. x = 4 cos 4 q Now, tan f = – and PN =
æ dy ö sin f
y = 4 sin q 4 ç ÷
and è dx ø
dy 4 ´ 4 sin 3 q × cos q It is given OP = PN
\ = = – tan 2 q
dx 4 ´ 4 cos 3 q (–sin q ) æ dy ö
2
Þ x2 + y2 = y 1 + ç ÷
y – 4 sin 4 q è dx ø
Now, equation of tangent is = – tan 2 q
x – 4 cos 4 q Y

½ 4 sin q ½ 4
nt
\ x-intercept, ½4 cos 4 q + ½= 4 cos 2 q
n ge
½ tan 2 q ½ Ta
P (x, y)
½ 4 sin 4 q ½
and y-intercept, ½4 sin 4 q + ½= 4 sin 2 q y
½ tan 2
q ½ al
rm
No
Hence, the sum of intercepts made on the axes of φ
X′ X
coordinates is, N O
4 cos 2 q + 4 sin 2 q = 4
Y′
Figure 7.10
y Example 23 The tangent, represented by the graph of
the function y = f ( x ), at the point with abscissa x = 1 é æ dy ö ù
2
Þ x 2 + y 2 = y 2 ê1 + ç ÷ ú
form an angle of p/ 6, at the point x = 2 form an angle êë è dx ø ú
û
of p/ 3 and at the point x = 3 form an angle of p/ 4 . 2
Then, find the value of, æ dy ö dy x
Þ x2 = y2 ç ÷ Þ =±
3 3 è dx ø dx y
ò1 f ¢ ( x ) f ¢ ¢ ( x ) dx + ò 2 f ¢ ¢ ( x ) dx. or y dy = ± x dx
dy Integrating both the sides, we get
Sol. Given, at x = 1, = tan p/ 6 = 1/ 3
dx y 2 = ± x 2 + C , which is the required family of curves.
or at x = 1, f ¢ (1) = tan p/ 6 = 1/ 3
y Example 25 Find the condition that the line
Also, at x = 2, f ¢ (2) = tan p/ 3 = 3 x cos a + y sin a = p may touch the curve
and at x = 3, f ¢ (3) = tan p/ 4 = 1 æxö
m
æy ö
m
3 3 ç ÷ + ç ÷ =1 .
Then, ò1 f ¢ ( x ) f ¢ ¢ ( x ) dx + ò 2 f ¢ ¢ ( x ) dx èaø èb ø
m m
f ¢ (3 ) æx ö æy ö
Sol. Given equation of curve is ç ÷ + ç ÷ =1
= òf ¢ (1) t dt + ( f ¢ ( x ))32
èa ø èb ø
[putting f ¢ ( x ) = t Þ f ¢ ¢ ( x ) dx = dt ] Differentiating the equation of curve w.r.t. x, we get
1 2 f ¢ (3 ) m –1 m –1
= (t ) f ¢ (1) + { f ¢ (3) – f ¢ (2)} æx ö 1 æy ö 1 dy
2 mç ÷ × +m ç ÷ × × =0
èa ø a èb ø b dx
1
= {( f ¢ (3))2 – ( f ¢ (1))2 } + { f ¢ (3) – f ¢ (2)} dy –bm x m – 1
2 On simplifying, we get = m m –1
dx
1 ìï 2 æ 1 ö üï
2 a y
1 æ 1ö
= í(1) – ç ÷ ý + {1– 3 } = ç1 – ÷ + (1 – 3 )
2ï è 3ø ï 2 è 3ø Now, at any point P ( x 1, y1 ) on the curve,
î þ
æ dy ö –bm x 1m – 1
4 4 –3 3 slope of tangent = ç ÷ =
= – 3= è dx ø (x amy1m – 1
3 3 1, y1 )

–bm x 1m – 1
y Example 24 Find the equation for family of curves \ Equation of tangent at P is, y – y1 = ( x – x1 )
amy1m – 1
for which the length of normal is equal to the radius
yy1m – 1 y1m xx 1m – 1 x 1m
vector. Þ – =– +
bm bm am am
Sol. Let P ( x , y ) be the point on the curve.
m –1 m –1 m m
x æ x1 ö y æ y1 ö æx ö æy ö
OP = radius vector = x + y 2 2 i.e. ç ÷ + ç ÷ = ç 1 ÷ + ç 1 ÷ =1
a èaø bèbø èaø èbø
410 Textbook of Differential Calculus

Since, P lies on the curve, therefore the equation of tangent y Example 27 The maximum value of the sum of the
at P ( x 1, y1 ) on the curve is,
m –1 m –1
intercepts made by any tangent to the curve
x æ x1 ö y æy ö (a sin 2 q , 2a sin q ) with the axes is
ç ÷ + ç 1÷ =1 ...(i)
a èaø bèbø
(a) 2a (b) a/4 (c) a/2 (d) a
Also, we have x cos a + y sin a = p ...(ii)
If Eq. (ii) is the equation of tangent, then coefficients of Sol. The curve is (a sin 2 q , 2a sin q ) Þ x = a sin 2 q , y = 2a sin q
Eqs. (i) and (ii) must be proportional for point ( x 1, y1 ). Now, the equation of any tangent to the curve is,
cos a sin a p y - 2a sin q 1
i.e. m –1
= m –1
= = Þ y sin q = x + a sin 2 q
1 æ x1 ö 1 æ y1 ö 1 x - a sin q
2
sin q
ç ÷ ç ÷
aè a ø bèb ø x y
Þ + =1
1 1
- a sin q
2
a sin q
x 1 æ a cos a öm – 1 y1 æ b sin a öm – 1
This gives =ç ÷ , =ç ÷
a è p ø b è p ø Þ Sum of intercepts = a (sin 2 q + sin q )

ïì æ 1 ïü
2
Since, point P ( x 1, y1 ) lies on the curve. 1ö
= a í çsinq + ÷ - ý
æx ö æy ö
m m
ïî è 2 ø 4ï
Therefore, we have ç 1 ÷ + ç 1 ÷ = 1 þ
èaø èbø Which is maximum, when sinq = 1
m m i.e. (Sum of intercept) max = 2a
æ a cos a öm – 1 æ b sin a öm – 1
i.e. ç ÷ +ç ÷ =1 Hence, (a) is the correct answer.
è p ø è p ø
m m m
y Example 28 If g ( x ) is a curve which is obtained by
i.e. (a cos a ) m –1
+ (b sin a ) m –1
=p m –1
,
e x - e -x
which is the required condition. the reflection of f ( x ) = by the line y = x ,
then 2
y Example 26 If tangent and normal to the curve
pö (a) g(x) has more than one tangent parallel to X-axis
æ
y = 2 sin x + sin 2x are drawn at P ç x = ÷ , then area of (b) g(x) has more than one tangent parallel to Y-axis
è 3ø
(c) y = - x is a tangent to g(x) at (0, 0)
the quadrilateral formed by the tangent, the normal at
P and the coordinate axes is (d) g(x) has no extremum
p Sol. As g ( x ) is a curve, obtained by the reflection of
(a) (b) 3p
3 e x -e -x
f (x ) = on y = x .
p 3 2
(c) (d) None of these
2 so g ( x ) is inverse of f ( x )
-1
dy æ p 3 3ö \ g ( x ) = log ( x + 1 + x 2 ) = f (x )
Sol. Here, = 0 atçx = , y = ÷
dx è 3 2 ø æ ö
1 2x
p Þ g ¢( x ) = × ç1 + ÷
Þ Tangent at x = is parallel to X-axis. x + 1 + x2 ç 2 1+ x2 ÷
3 è ø
3 3 1
Þ Equation of tangent is, y = = ¹ 0," x ÎR
2 1 + x2
p
Also, equation of normal is, x = Þ g ( x ) has no tangent parallel to X-axis. Also, g ¢( x ) is
3 always defined, " x ÎR.
p 3 3 p 3 Þ g ( x ) has no tangent parallel to Y-axis.
Now, area of quadrilateral = × = sq unit
3 2 2 Since g ¢ ( x ) > 0 "x Î R,
Hence, (c) is the correct answer. therefore g ( x ) doesn’t have any extremum.
Hence, (d) is the correct answer.
Chap 07 dy/dx as a Rate Measurer and Tangents, Normals 411

Exercise for Session 3


1. If the line ax + by + c = 0 is normal to the xy + 5 = 0, then a and b have
(a) same sign (b) opposite sign
(c) cannot be discussed (d) None of these

2. The equation of tangent drawn to the curve y 2 - 2x 3 - 4y + 8 = 0 from the point (1, 2) is given by
(a) y - 2 (1 ± 2 ) = ± 2 3 (x - 2) (b) y - 2 (1 ± 3 ) = ± 2 2 (x - 2)
(c) y - 2 (1 ± 3 ) = ± 2 3 (x - 2) (d) None of these

3. The equation of the tangents to the curve (1 + x 2 )y = 1 at the points of its intersection with the curve ( x + 1)y = 1,
is given by
(a) x + y = 1, y = 1 (b) x + 2y = 2, y = 1
(c) x - y = 1, y = 1 (d) None of these
x
4. The tangent lines for the curve y = ò 2 | t | dt which are parallel to the bisector of the first coordinate angle, is
0
given by
3 1 1 3
(a) y = x + ,y = x- (b) y = - x + ,y = - x +
4 4 4 4
(c) x + y = 2, x - y = 1 (d) None of these

5. The equation of normal to x + y = x y , where it intersects X-axis, is given by


(a) x + y = 1 (b) x - y - 1 = 0
(c) x - y + 1 = 0 (d) None of these

6. The equation of normal at any point q to the curve


x = a cos q + aq sin q, y = a sin q - aq cos q is always at a distance of
(a) 2a unit from origin (b) a unit from origin
1
(c) a unit from origin (d) None of these
2
x1 y
7. If the tangent at ( x 0, y 0 ) to the curve x 3 + y 3 = a 3 meets the curve again at ( x1, y1), then + 1 is equal to
x0 y0
(a) a (b) 2a
(c) 1 (d) None of these

8. The area bounded by the axes of reference and the normal to y = loge x at (1, 0), is
(a) 1 sq unit (b) 2 sq units
1
(c) sq unit (d) None of these
2
x y xn yn
9. If + = 2 touches the curve n + n = 2 at the point ( a, b ), then
a b a b
(a) a = a 2 , b = b 2 (b) a = a, b = b
(c) a = - 2a, b = 4b (d) a = 3a, b = - 2b

10. The equation of tangents to the curve y = cos ( x + y ), -2p £ x £ 2p that are parallel to the line x + 2y = 0, is
p 3p p 3p
(a) x + 2y = and x + 2y = - (b) x + 2y = and x + 2y =
2 2 2 2
(c) x + 2y = 0 and x + 2y = p (d) None of these
Session 4
Angle of Intersection of Two Curves, Length of
Tangent, Subtangent, Normal and Subnormal
Angle of Intersection Orthogonal Curves If the angle of intersection of two
curves is a right angle, then the two curves are said to be
of Two Curves orthogonal and the curves are called orthogonal curves.
The angle of intersection of two curves is defined as the \ If the curves are orthogonal, then q = p/ 2
angle between the tangents to the two curves at their æ dy ö æ dy ö
point of intersection. Þ 1+ ç ÷ ç ÷ =0
è dx ø C 1 è dx ø C 2
Y
C1 æ dy ö æ dy ö
Þ ç ÷ ç ÷ =– 1
Tangent è dx ø C 1 è dx ø C 2
P

θ C2
Condition for Two Curves to Touch
θ1 θ2 If two curves touch each other, then
X′ X
O
T1
T2 q =0
Y′ Y
C1
Figure 7.11

Let C 1 and C 2 be two curves having equations P C2

y = f ( x ) and y = g( x ), respectively.
Let PT1 and PT2 be tangents to the curves C 1 and C 2 at X
O
their point of intersection.
Let q be the angle between the two tangents PT1 and PT2 Figure 7.12
and q 1 and q 2 are the angles made by tangents with the Þ tan q = 0
positive direction of X-axis in anti-clockwise sense.
æ ö
dy æ ö
dy
æ dy ö Þ ç ÷ - ç ÷ =0
Then, m 1 = tan q 1 = ç ÷ è dx ø C 1 è dx ø C 2
è dx ø C 1
æ dy ö æ dy ö
æ dy ö Þ ç ÷ =ç ÷
m 2 = tan q 2 = ç ÷ è dx ø C 1 è dx ø C 2
è dx ø C 2
From the figure it follows, q = q 2 – q 1 y Example 29 Find the angle of intersection of the
tan q 2 – tan q 1 curves y = x 2 and y = 4 – x 2 .
Þ tan q = tan (q 2 – q 1 ) =
1 + tan q 2 tan q 1 Sol. For the intersection points of the given curves, consider
æ dy ö æ dy ö x2 = 4 – x2
ç ÷ –ç ÷
è dx ø C1 è dx ø C2 Þ x=± 2
Þ tan q =
æ dy ö æ dy ö Now, at x = 2,
dy
for first curve = 2x | at x = =2 2
1+ç ÷ ç ÷ 2
è dx ø C1 è dx ø C2 dx
While at x = 2,
Angle of intersection of these curves is defined as dy
acute angle between the tangents. for second curve = -2x |at x= 2 = -2 2
dx
Chap 07 dy/dx as a Rate Measurer and Tangents, Normals 413

Hence, if q 1 is the acute angle of intersection of the curves, y Example 31 Find the angle of intersection of curves,
then
y = [| sin x | + | cos x | ] and x 2 + y 2 = 5, where [× ]
2 2 – (–2 2 ) 4 2
tan q 1 = = denotes the greatest integral function.
1 + 2 2 (–2 2 ) –7
Sol. We know that, 1 £ | sin x | + | cos x | £ 2

–1 æ 4
\ q 1 = tan ç ÷ ...(i) \ y = [| sin x | + | cos x | ] = 1
è 7 ø
Let P and Q be the points of intersection of given curves.
Again, at x =– 2,
Clearly, the given curves meet at points where y = 1, so we
æ dy ö get x 2 + 1 = 5
ç ÷ for first curve = –2 2
è dx ø
Þ x = ±2
æ dy ö Now, we have P (2, 1) and Q (–2, 1)
and ç ÷ for second curve = 2 2
è dx ø Y
Hence, if q 2 is the acute angle of intersection of the curves,
then 5
Q P
–2 2 –2 2 y=1
tan q 2 =
1 + (–2 2 )(2 2 ) X′
O 5
X
– 5
æ4 2ö x2 + y2 = 5
\ q 2 = tan –1 ç ÷ ...(ii)
è 7 ø – 5
Y′
From Eqs. (i) and (ii) the two acute angles are equal.
Differentiating x 2 + y 2 = 5 w.r.t. x, we get
y Example 30 Find the acute angle between the
dy
curves 2x + 2y =0
dx
y = | x 2 – 1 | and y = | x 2 – 3 | at their points of dy x
Þ =–
intersection when x > 0. dx y
æ dy ö
Sol. For the intersection of the given curves, Þ ç ÷ =–2
è dx ø (2 , 1)
| x 2 – 1 | = | x 2 – 3 | Þ ( x 2 – 1) 2 = ( x 2 – 3) 2
æ dy ö
Þ ( x 2 – 1)2 –( x 2 – 3)2 = 0 and ç ÷ =2
è dx ø (–2 , 1)
Þ [( x 2 – 1)–( x 2 – 3)][( x 2 – 1) + ( x 2 – 3)] = 0
Clearly, the slope of line y = 1 is zero and the slope of the
Þ 2 [2x 2 – 4 ] = 0 Þ 2x 2 = 4 Þ x = ± 2 tangents at P and Q are (– 2) and (2), respectively.
Neglecting x = – 2, as x > 0 Thus, the angle of intersection is tan –1(2).
Now, we have, point of intersection as x = 2.
Here, y = | x 2 – 1 | = ( x 2 – 1) in the neighbourhood of
x = 2 and y = –( x 2 – 3) in the neighbourhood of x = 2. Length of Tangent, Subtangent,
Now, at x = 2,
dy
dx
for first curve Normal and Subnormal
Length of Tangent The length of the segment PT, i.e.
æ dy ö the portion of the tangent intercepted between the point
Þ ç ÷ = 2x = 2 2
è dx øC of contact and X-axis is called the length of tangent.
1

dy æ dy ö Subtangent and its Length The projection of the


and for second curve ç ÷ = –2x = – 2 2
dx è dx øC segment PT along X-axis is called the subtangent.
2
Here, ST is the length of subtangent.
Hence, if q is angle between them, then
2 2 – (–2 2 ) 4 2 æ4 2ö Length of Normal The length of segment PN , i.e. the
tan q = = = ç ÷ portion of the normal intercepted between the point on
1 + 2 2 (–2 2 ) –7 è 7 ø
the curve and X-axis is called the length of normal.
æ4 2ö
\ q = tan –1 ç ÷
è 7 ø
414 Textbook of Differential Calculus

Subnormal and its Length y Example 32 Show that for the curve y = be x/a , the
The projection of the segment PN along X-axis is called subtangent is of constant length and the subnormal
the subnormal. Here, SN is the length of sub normal.
varies as the square of ordinate.
Y
Sol. The equation of given curve is y = be x/a
Let us consider a point ( x 1, y1 ) on the curve.
P(x1,y 1) Then, we have y1 = be x 1/a ...(i)
Differentiating the curve y = be x/a
w.r.t. x, we get
dy 1
X′
O T
X = be x/a ×
S N dx a
Y′ æ dy ö b
\ ç ÷ = e x 1/a ...(ii)
Figure 7.13 è dx ø (x , y ) a
1 1
From the figure, if PT makes an angle q with X-axis, then
½ æ dx ö ½
æ dy ö Thus, the length of subtangent =½y1 ç ÷ ½
tan q = ç ÷
è dx ø ( x1 , y 1 ) ½ è dy ø (x 1 , y1 ½
)

Y a a
= y1 × x 1/a
= be x 1/a ×
be be x 1/a
= a (constant) [using Eqs. (i) and (ii)]
(x1,y1)
P Þ Subtangent is of constant length a.
θ (90°– θ)
X′ X æ dy ö
O T S N Again, length of subnormal = y1 ç ÷
Tangent
Normal è dx ø (x
1, y1 )
Y′
x 1/a
Figure 7.14 be 1 1
= be x 1/a × = (be x 1/a )2 = y 2 [using Eq. (i)]
ST a a a
Also, we have = cot q Þ ST = PS cot q
PS Therefore, subnormal varies as the square of ordinate.
æ dx ö
or subtangent = ST = y 1 ç ÷ y Example 33 Find the length of tangent, subtangent,
è dy ø ( x1 , y1 )
normal and subnormal to y 2 = 4ax at (at 2 , 2at ) .
SN
Similarly, = cot (90° – q ) Sol. The equation of given curve is
PS
y 2 = 4ax ...(i)
Þ Subnormal = SN = PS tan q
Differentiating Eq. (i) w.r.t. x, we get
æ dy ö dy
Þ Subnormal = SN = y 1 ç ÷ 2y = 4a
è dx ø ( x1 , y 1 ) dx
é dy ù 4a 1
Now, length of tangent Þ êë dx úû 2 = = ...(ii)
(at , 2at ) 4 at t
2
æ dx ö Now, the length of tangent at (at 2 , 2at ) is
= PT = y 12 + y 12 ç ÷
è dy ø ( x1 , y 1 ) æ dx ö
2
= y1 1 + ç ÷ = 2at 1 + t2 [using Eq. (ii)]
2 è dy ø (x
1 , y1 )
æ dx ö
Þ PT = y 1 1 + ç ÷ length of normal at (at 2 , 2at ) is
è dy ø ( x1 , y 1 )
2
æ dy ö
2 = y1 1 + ç ÷ = 2at 1 + 1/t 2 = 2a t 2 + 1
æ dy ö è dx ø (x
and length of normal = PN = y 12 + y 12 ç ÷ 1
, y1 )
è dx ø ( x1 , y1 ) length of subtangent is
y1 2at
2 = = 2at 2
æ dy ö é dy ù 1/t
Þ PN = y 1 1+ç ÷
è dx ø ( x1 , y 1 ) êë dx úû
( x ,y )
1 1
Chap 07 dy/dx as a Rate Measurer and Tangents, Normals 415

and length of subnormal is Sol. The equation of given curve isby 2 = ( x + a )3


é dy ù 1 Differentiating both the sides w.r.t. x, we get
y1 ê ú = 2at × = 2a
ë dx û (x , y )1 1
t
2 by
dy
= 3 ( x + a )2 × 1
dx
y Example 34 Find the equation of tangent and dy 3 ( x + a )2
Þ =
normal, the length of subtangent and subnormal of the dx 2 by
circle x 2 + y 2 = a 2 at the point ( x 1 , y 1 ).
\ Length of subnormal
Sol. The equation of given curve is,
dy 3 ( x + a )2
x 2 + y 2 = a2 ...(i) Þ SN = y = × ...(i)
dx 2 b
Differentiating Eq. (i) w.r.t. x, we get and Length of subtangent
dy
2x + 2y =0 dx 2 by 2
dx Þ ST = y = ...(ii)
dy 3 ( x + a )2
æ dy ö x
Þ ç ÷ =- 1 ...(ii) p (ST )2
è dx ø (x , y ) y 1 Q = [given]
1 1
q (SN )
Thus, the equation of tangent is,
p (2 by 2 )2 × 2b
é dy ù \ = [using Eqs. (i) and (ii)]
y – y1 = ê ú ( x – x1 ) q {3 ( x + a )2 } 2 × 3 ( x + a )2
ë dx û (x , y ) 1 1

x 8b {( x + a )3 } 2 8b
Þ y – y1 = – 1 ( x – x 1 ) [from Eq. (ii)] = × = [using, by 2 = ( x + a )3 ]
y1 27 ( x + a )6 27

Þ yy1 – y12 = – xx 1 + x 12 \
p 8b
=
q 27
Þ xx 1 + yy1 = x 12 + y12
Þ xx 1 + yy1 = a 2 [using Eq. (i) as ( x 1, y1 ) lies on y Example 36 If the length of subnormal is equal to
x 2 + y 2 = a 2 Þ x 12 + y12 = a 2 ] length of subtangent at any point ( 3, 4 ) on the
While the equation of normal is,
curve y = f ( x ) and the tangent at (3, 4) to y = f ( x )
y
y – y1 = 1 ( x – x 1 )
x1 meets the coordinate axes at A and B, then
Þ x 1y – x 1y1 = xy1 – x 1y1 Þ xy1 – x 1y = 0 maximum area of the D OAB where O is origin, is
45 49 25 81
æ dx ö (a) (b) (c) (d)
The length of subtangent = y1 × ç ÷ 2 2 2 2
è dy ø (x , y1 )
1
Sol. Length of subnormal = Length of subtangent
æ –y ö
= y1 × ç 1 ÷ [using Eq. (ii)] Þ
dy
= ±1
è x1 ø dx
y12 dy
Þ The length of subtangent = If = 1, then equation of tangent is,
x1 dx
y - 4 = x -3 Þ y - x =1
æ dy ö
While the length of subnormal = y1 × ç ÷ 1
\ Area of DOAB = ´ 1 ´ 1 =
1
…(i)
è dx ø (x , y1 ) 2 2
1

æ x ö dy
= y1 ç– 1 ÷ = | x 1| If = - 1, then equation of tangent is,
è y1 ø dx
y - 4 = - x +3 Þ x +y =7
y Example 35 If the relation between subnormal SN 1 49
\ Area of DOAB = ´ 7 ´ 7 = …(ii)
and subtangent ST on the curve, by 2 = ( x + a ) 3 is 2 2
p 49
p (SN ) = q (ST ) 2 , then find the value of × Clearly, maximum area =
2
q
Hence, (b) is the correct answer.
416 Textbook of Differential Calculus

Exercise for Session 4


1. The angle of intersection of y = a x and y = b x, is given by
log (ab ) log(a / b )
(a) tan q = (b) tan q =
1 - log(ab ) 1 + loga logb
½ log(a / b ) ½
(c) tan q = ½ ½ (d) None of these
½ 1 - log(a / b )½

2. The angle between the curves x 2 + 4y 2 = 32 and x 2 - y 2 = 12, is


p p p p
(a) (b) (c) (d)
3 4 6 2

3. If ax 2 + by 2 = 1 cuts a ¢ x 2 + b ¢ y 2 = 1orthogonally, then


1 1 1 1 1 1 1 1
(a) - = - (b) + = +
a a¢ b b ¢ a a¢ b b ¢
1 1 1 1
(c) + = + (d) None of these
a b a¢ b ¢

4. The length of subtangent to the curve, y = e x / a, is


(a) 2a (b) a (c) a / 2 (d) a / 4
p
5. The length of normal to the curve x = a( q + sin q), y = a (1 - cos q), at q = is
2
(a) 2a (b) a (c) 2a (d) 2 2a

6. The length of tangent to the curve y = x + 3x + 4x - 1at a point x = 0, is


3 2

1 17 3 15
(a) (b) (c) (d)
4 4 4 4

7. The possible values of p such that the equation px 2 = log x has exactly one solution, are

(c) (-¥, 0] È ìí üý
1
(a) R (b) R + (d) [0, ¥)
î 2e þ

8. If curve y = 1 - ax 2 and y = x 2 intersect orthogonally then a is


1 1
(a) (b) (c) 2 (d) 3
2 3
æ tö
9. The length of tangent to the curve x = a çcos t + log tan ÷ , y = a(sin t ), is
è 2ø
(a) ax (b) ay (c) a (d) xy

10. The lines tangent to the curves y 3 -`x 2y + 5y - 2x = 0 and x 4 - x 3y 2 + 5x + 2y = 0 at the origin intersect at an
angle q equal to
p p p p
(a) (b) (c) (d)
6 4 3 2
Session 5
Rolle’s Theorem, Lagrange’s Mean Value Theorem
Rolle’s Theorem The function should either increase or decrease when x
The theorem was named after the French Mathematician assumes values slightly greater than a.
Michel Rolle (1652–1719). Who first gave it in his book Now, let f ( x ) increases for x > a.
Methode pour resoudre lesegalites (1691). Since, f (a ) = f (b ), hence the function increase at some
Statement value x = c and decrease upto x = b.
Let f be a real-valued function defined on the closed Clearly, at x = c function has maximum value.
interval [a, b ] such that Now, let h be a small positive quantity, then from
(i) f ( x ) is continuous in the closed interval[a, b ] definition of maximum value of the function,
(ii) f ( x ) is differentiable in the open interval ]a, b [ and f (c + h ) – f (c ) < 0
(iii) f (a ) = f (b ) and f (c – h ) – f (c ) <0
f (c + h ) – f (c )
Then, there is atleast one value c of x in open interval \ <0
]a, b [ for which f ¢ (c ) = 0. h
f (c – h ) – f (c )
and >0
Analytical Proof –h
Now, Rolle’s theorem is valid for a function such that f (c + h ) – f (c )
(i) f ( x ) is continuous in the closed interval[a, b ] So, lim £0
h®0 h
(ii) f ( x ) is differentiable in open interval ] a, b [ and f (c – h ) – f (c )
and lim ³0 ...(i)
(iii) f (a ) = f (b ) h®0 –h
So, generally two cases arise in such circumstances. f (c + h ) – f (c ) f (c – h ) – f (c )
But, if lim ¹ lim
Case I f ( x ) is constant in the interval [a, b ], then h®0 h h ® 0 –h
f ¢ ( x ) = 0 for all x Î[a, b ].
Then, Rolle’s theorem cannot be applied because in such
Y
case,
y=c
RHD at x = c ¹ LHD at x = c .
Hence, f ( x ) is not differentiable at x = c , which
X′ X contradicts the condition of Rolle’s theorem.
O a b

Y′ \ Only one possible solution arises, when


Figure 7.15 f (c + h ) – f (c ) f (c – h ) – f (c )
lim = lim =0
h®0 h h ® 0 –h
Hence, Rolle’s theorem follows and we can say, f ¢ (c ) = 0,
where a < c < b which implies that, f ¢ (c ) = 0 where a < c < b.
Case II f ( x ) is not constant in the interval [a, b ] and since Hence, Rolle’s theorem is proved.
f (a ) = f (b ). Y
Y
D A B
De se
I cre rea
as
e Inc Minimum

X′ X X′ X
O x=a x=c x=b O x=a x=c x=b

Y′ Y′
Figure 7.16 Figure 7.17
418 Textbook of Differential Calculus

Similarly, the case where f ( x ) decreases in the interval (vi) If f ¢ ( x ) tends to ± ¥ as x ® k, then f ( x ) is not
a < x < c and then increases in the interval c < x < b, differentiable at x = k.
f ¢ (c ) = 0, but when x = c , the minimum value of f ( x ) 1
e.g. If f ( x ) = (2 x – 1) 1 / 2 , then f ¢ ( x ) = is such
exists in the interval [a, b ] . 2 x –1
that as
+
Geometrical Proof æ 1ö
Consider the portion AB of the curve y = f ( x ), lying x ®ç ÷ Þ f ¢( x ) ® ¥
è2ø
between x = a and x = b, such that
So, f ( x ) is not differentiable at x = 1/ 2.
(i) it goes continuously from A to B
(ii) it has tangent at every point between A and B y Example 37 Verify Rolle’s theorem for the function
(iii) ordinate of A = ordinate of B f ( x ) = x 3 – 3x 2 + 2x in the interval [0, 2] .
From the figure, it is clear Y Sol. Here, we observe that
that f ( x ) increases in the C1 C2 (a) f ( x ) is polynomial and since polynomials are always
interval AC 1 , which implies continuous, so f ( x ) is continuous in the interval [0, 2].
A
that f ¢ ( x ) > 0 in this region B D (b) f ¢( x ) = 3x 2 – 6x + 2, which exists for all x Î(0, 2). So,
and decreases in the interval X′ X f ( x ) is differentiable for all x Î(0, 2) and
O x=a x=cx=b
C 1 B which implies f ¢ ( x ) < 0 (c) f (0) = 0, f (2) = 23 –3 × (2)2 + 2(2) = 0
in this region. Now, since Y′
\ f ( 0) = f ( 2)
there is unique tangent to Figure 7.18 Thus, all the conditions of Rolle’s theorem are satisfied.
be drawn on the curve
lying in between A and B and since each of them has So, there must exists some c Î(0, 2) such that f ¢ (c ) = 0
a unique slope, i.e. unique value of f ¢ ( x ). Þ f ¢ (c ) = 3c 2 – 6c + 2 = 0 Þ c = 1 ±
1
.
So, due to continuity and differentiability of the function 3
1
Where, c = 1 ± Î (0, 2), thus Rolle’s theorem is
f ( x ) in the region A to B, there is a point x = c , where 3
verified.
f ¢ (c ) should be zero.
Hence, f ¢ (c ) = 0, where a < c < b y Example 38 If ax 2 + bx + c = 0, a, b , c ÎR, then find
Thus, Rolle’s theorem is proved. the condition that this equation would have at least
one root in (0, 1).
Remarks
Sol. Let f ¢ ( x ) = ax 2 + bx + c
Generally, two types of problems are formulated on Rolle’s
theorem. Integrating both sides, we get
(i) To check the applicability of Rolle’s theorem to a given ax 3 bx 2
function on a given interval f (x ) = + + cx + d
3 2
(ii) To verify Rolle’s theorem for a given function in a given interval.
a b
In both the types of problems, first we check whether f ( x ) Þ f (0) = d and f (1) = + + c + d
satisfies the conditions of Rolle’s theorem or not. 3 2
Now, for Rolle’s theorem to be applicable, we should have
Some Important Results Regarding f ( 0) = f ( 1)
Rolle’s Theorem a b
(i) A polynomial function is everywhere continuous and Þ d = + +c +d
3 2
differentiable.
Þ 2a + 3b + 6c = 0
(ii) The exponential function, sine and cosine functions Here, the required condition is 2a + 3b + 6c = 0.
are everywhere continuous and differentiable.
(iii) Logarithmic functions is continuous and y Example 39 If f ( x ) and g ( x ) are continuous
differentiable in its domain. functions in [a, b ] and they are differentiable in (a, b ),
(iv) tan x is not continuous and differentiable at then prove that there exists c Î(a, b ) such that
x = ± p/ 2, ± 3 p/ 2 , ± 5 p/ 2 ,......... f (a ) f (b ) f (a ) f ¢(c )
= (b – a )
(v) | x | is not differentiable at x = 0. g (a ) g (b ) g (a ) g ¢(c )
Chap 07 dy/dx as a Rate Measurer and Tangents, Normals 419

Sol. Consider a function, f ( x ) = f (a ) g ( x ) – f ( x ) g (a ) for all y Example 41 Let f ( x ) and g ( x ) be differentiable


x Î[a, b ].
As, f( x ) is continuous and differentiable on (a, b ).
functions such that f ¢ ( x )g ( x ) ¹ f ( x )g ¢ ( x ) for any real
f (b ) – f (a )
x. Show that between any two real solutions of
\ f¢ (c ) = for some c Î(a, b ). ...(i) f ( x ) = 0, there is atleast one real solution of g ( x ) = 0 .
b –a
Now, as f ( x ) = f (a ) g ( x ) – f ( x ) g (a ) Sol. Let a, b be the solutions of f ( x ) = 0.
\ f¢ ( x ) = f (a ) g ¢( x ) – f ¢( x ) g (a ) Suppose g ( x ) is not equal to zero for any x belonging to [a, b].
Þ f¢ (c ) = f (a ) g ¢(c ) – f ¢(c ) g (a ) Now, consider h( x ) = f ( x ) / g ( x )
f (a ) f ¢(c ) Since, g ( x ) is not equal to zero, therefore
Þ f¢ (c ) =
g (a ) g ¢(c ) h ( x ) is differentiable and continuous in [a, b ].
f (a ) f (b ) Also, h (a ) = h (b ) = 0
Also, f (b ) = f (a ) g (b ) – f (b ) g (a ) = [as f (a ) = 0 and f (b ) = 0 but g (a ) and g (b ) ¹ 0]
g (a ) g (b )
Applying Rolle’s theorem for h ( x ) in [a, b ] , we get
and f (a ) = f (a ) g (a ) – f (a ) g (a ) = 0 h ¢ (c ) = 0 for some c belonging to (a, b )
\ Eq. (i) reduces to; Þ f (c )g ¢ (c ) = f ¢ (c )g (c ), for some c Î(a, b ).
f (a ) f ¢ (c ) 1 f (a ) f (b )
= This gives the contradiction. Hence proved.
g (a ) g ¢ (c ) (b – a ) g (a ) g (b )
y Example 42 Consider the function
f (a ) f (b ) f (a ) f ¢ (c ) p
= (b – a ) ìï
or
g ¢ (c ) x sin , for x > 0
g (a ) g (b ) g (a ) f (x ) = í x , then the number of points
ïî 0 , for x = 0
y Example 40 Use Rolle’s theorem to find the
in (0, 1), where the derivative f ¢( x ) tends to zero is
condition for the polynomial equation f ( x ) = 0 to have
a repeated real roots. Hence, or otherwise prove that (a) 0 (b) 1 (c) 2 (d) infinite [IIT 2010]
p
the equation; Sol. f ( x ) tends to zero at points, where sin = 0
x
x x2 xn p
1+ + + K+ = 0, cannot have repeated roots. i.e. = kp, k = 1, 2, 3, 4, K
1! 2 ! n! x
1
Sol. By Rolle’s theorem, we can say that between any two Hence, x = ×
roots of a polynomial there is always a root of its deriva- k
tive. Thus, if a is a repeated root of a polynomial f ( x ), p p p
Also, f ¢ ( x ) = sin - cos , if x ¹ 0
then there must be a root of f ¢( x ) in the interval. x x x
Þ f ¢ (a ) = 0 Since, the function has a derivative at any interior point of
i.e. f (a ) = f ¢ (a ) = 0, for a to be a repeated root. the interval (0, 1), also continuous in [0, 1] and f (0) = f (1),
therefore Rolle’s theorem is applicable to any one of the
x x2 xn
Let f (x ) = 1 + + +K+ has a repeated root a. é1 ù é1 1ù é 1 1ù
1! 2! n! intervals ê , 1ú, ê , ú, K, ê , ú.
Þ f (a ) = 0 ë2 û ë3 2û ëk + 1 k û
and f¢ (a ) = 0 Hence, there exists some c in each of these intervals, where
f ¢ (c ) = 0 Þ Infinite points.
a a a2 n
Þ 1+ + +K+ =0 Hence, (d) is the correct answer.
1! 2! n!
a2 an –1
1+a + +K+ =0
and
2! ( n – 1) ! Lagrange’s Mean Value Theorem
Solving above equations, we get First Form
an If a function f ( x ),
=0
n! (i) is continuous in the closed interval [a, b ] and
or a = 0, thus 0 is the repeated root of f ( x ) = 0. (ii) is differentiable in the open interval ]a, b [
But, 0 doesn’t satisfy f( x ). Then, there is at least one value c Î(a, b ), such that
\ There is no repeated root of f ( x ) = 0. f (b ) – f (a )
f ¢(c ) =
b –a
420 Textbook of Differential Calculus

Proof Consider the function, Hence, the Lagrange’s mean value theorem asserts that if
f (b ) – f (a ) a curve has a tangent at each of its points, then there is a
f (x ) = f (x ) – x point c on this curve in between A and B, the tangent at
b –a
which is parallel to the chord AB.
Since, f ( x ) is continuous in [a, b ] .
\ f ( x ) is also continuous in[a, b ]. y Example 43 Find c of the Lagrange’s mean value
Since, f ¢ ( x ) exists in (a, b ), therefore f¢ ( x ) also exists in theorem for which f ( x ) = 25 – x 2 in [1, 5] .
(a, b ) and
f (b ) – f (a ) Sol. It is clear that f ( x ) has a definite and unique value for
f¢ ( x ) = f ¢ ( x ) – ...(i) each x Î[1, 5] .
b –a
Thus, for every point in the interval [1, 5] the value of f ( x )
Also, we have f (a ) = f (b ). is equal to the limit of f ( x ).
Thus, f ( x ) satisfies all the conditions of Rolle’s theorem. So, f ( x ) is continuous in the interval [1, 5].
\ There is atleast one value of c of x between a and b, –x
Also, f ¢( x ) = , which clearly exists for all x in
such that f¢ (c ) = 0 25 – x 2
On substituting x = c in Eq. (i), we get open interval (1, 5)
f (b ) – f (a ) Hence, f ( x ) is differentiable in (1, 5).
f ¢ (c ) = , which proves the theorem.
b –a So, there must be a value c Î(1, 5) such that,
f (5) – f (1) 0 – 24 – 6
f ¢(c ) = = =
Second Form 5–1 4 2
If we write b = a + h, then c = a + qh , where 0 < q < 1. But f ¢(c ) =
–c
[Q a < c < b ] 25 – c 2
Thus, the mean value theorem can be stated as follows: –c – 6
\ = Þ 4c 2 = 6 (25 – c 2 )
If (i) f ( x ) is continuous in closed interval [a, a + h ]. 25 – c 2 2

(ii) f ¢ ( x ) exists in the open interval ]a, a + h [, then there Þ c = ± 15


exists at least one number q (0 < q < 1), such that Clearly, c = 15 Î (1, 5) . So, for c = 15 Lagrange’s
f (a + h ) = f (a ) + hf ¢ (a + qh ) theorem is satisfied.

y Example 44 Let f ( x ) and g ( x ) be differentiable for


Geometrical Interpretation of 0 £ x £ 2 such that f (0) = 2, g (0) = 1 and f (2) = 8. Let
Lagrange’s Theorem there exists a real number c in [0, 2] such that
Let A, B be the points on the curve y = f ( x ) corresponding f ¢(c ) = 3 g ¢(c ), then the value of g (2) must be
to x = a and x = b, so that A =[a, f (a )] and B =[b, f (b )]
(a) 2 (b) 3 (c) 4 (d) 5
f (b ) – f (a )
Now, slope of chord AB = Sol. As f ( x ) and g ( x ) are continuous and differentiable in
b –a [0, 2], therefore there exists atleast one value ‘c’ such that
The slope of the chord AB = f ¢ (c ), the slope of the tangent f ¢ ( c ) f ( 2) - f ( 0)
= Þ
8-2
=3
to the curve at x = c . g ¢ ( c ) g ( 2) - g ( 0) g(2) - 1
Y Þ g ( 2) - 1 = 2 Þ g ( 2) = 3
C Hence, (b) is the correct answer.
(b, f (b))
B y Example 45 If f ( x ) = log e x , g ( x ) = x 2 and
(a))

æ 4 25 ö
f

A c Î(4, 5), then c log ç 16 ÷ is equal to


(a,

O a c b
X è5 ø
(a) c log e 5 - 8 (b) 2 (c 2 log e 4 - 8)
Figure 7.19
(c) 2 (c 2 log e 5 - 8) (d) c log e 4 - 8
Chap 07 dy/dx as a Rate Measurer and Tangents, Normals 421

Sol. Let f ( x ) = x 2 (loge 4 ) - 16 loge x , which is continuous in Sol. For the function f ( x ) given in option (a), we have (LHD
[4, 5] and differentiable in (4, 5). at x = 1 / 2) = -1 and (RHD at x = 1/2) = 0.
Then, by Lagrange’s theorem, we get So, it is not differentiable at x = 1 /2 Î (0, 1).
f ( 5) - f ( 4 ) \ Lagrange’s mean value theorem is not applicable.
= f¢ (c ), for some c Î( 4, 5)
5- 4 Hence, (a) is the correct answer.
æ 4 25 ö
Now, f (5) - f ( 4 ) = loge ç 16 ÷ y Example 48 Let f ( x ) satisfy the requirements of
è5 ø
2 2 Lagrange’s mean value theorem in [0, 2]. If f (0) = 0
Also, f¢ (c ) = (c loge 4 - 8) and | f ¢ ( x )| £ 1 / 2 for all x Î[0, 2] , then
c
f ( 5) - f ( 4 ) (a) f (x) £ 2 (b) | f (x) | £ 2x
\ f¢ (c ) =
5- 4 (c) | f (x) | £ 1 (d) f (x) = 3,
2 2 æ 4 25 ö for atleast one x Î[0 , 2]
Þ (c loge 4 - 8) = loge ç 16 ÷ Sol. Let x Î(0, 2). Since, f ( x ) satisfies the requirements of
c è5 ø
Lagrange’s mean value theorem in [0, 2]. So, it also satis-
æ 4 25 ö fies in [0, x ]. Consequently, there exists c Î(0, x ) such that
or c log ç 16 ÷ = 2 (c 2 log 4 - 8)
è5 ø f ( x ) - f ( 0) f (x )
f ¢(c ) = Þ f ¢(c ) =
Hence, (b) is the correct answer. x -0 x

Þ ½ f ( x )½ = | f ¢(c )| £ 1 / 2 [Q | f ¢( x )| £ 1 / 2]
p ½ x ½
y Example 46 If 0 < a < b < and
2 x
tan b - tan a Þ | f (x ) | £
f (a, b ) = , then 2
b -a Þ | f (x ) | £ 1 [Q x Î (0, 2), \ x £ 2]
(a) f (a , b) ³ 2 (b) f (a , b) > 1
Hence, (c) is the correct answer.
(c) f (a , b) £ 1 (d) None of these
Sol. Consider the function f ( x ) = tan x , defined on [a, b] such y Example 49 Let f :[2, 7 ] ® [0, ¥ ) be a continuous and
æ pö differentiable function. Then, the value of
that a, b Î ç0, ÷.
è 2ø ( f (7 )) 2 + ( f (2)) 2 + f (2) × f (7 )
( f (7 ) - f (2)) is
Applying Lagrange’s mean value theorem, we have 3
f ¢(c ) =
f (b ) - f (a )
for some c Î(a, b )
(where c Î(2, 7 ))
b-a (a) 3 f 2 (c ) f ¢(c ) (b) 5 f 2 (c ) × f (c )
tan b - tan a
Þ sec 2 c = (c) 5 f 2 (c ) × f ¢(c ) (d) None of these
b-a
Sol. Let g ( x ) = f ( x ) Þ g ¢ ( x ) = 3 f 2 ( x ) × f ¢ ( x )
3
Þ f (a, b ) = sec 2 c
Þ f (a, b ) > 1 [Qsec 2 c > 1 as c Î(0, p / 2)] Q f : [2, 7 ] ® [0, ¥ ) \ g : [2, 7 ] ® [0, ¥ )
Using Lagrange’s mean value theorem on g ( x ), we get
Hence, (b) is the correct answer.
g ( 7 ) - g ( 2)
g ¢ (c ) = , c Î (2, 7 )
y Example 47 In [0, 1] Lagrange’s mean value theorem 5
is not applicable to [IIT JEE 2003] f 3 ( 7 ) - f 3 ( 2)
Þ 3 f 2 (c ) f ¢ (c ) =
ì 1 1 5
ïï 2 - x , x<
2
(a) f (x) = í 2
Þ 5 f 2 (c ) × f ¢ (c )
ï æç 1 - x ö÷ , x ³ 1 ( f (7 ) - f (2)) ( f 2 (7 ) + f 2 (2) + f (7 ) × f (2))
ïî è 2 ø 2 =
3
ì sin x
ï , x ¹0 Hence, (c) is the correct answer.
(b) f (x) = í x
îï 1, x =0 y Example 50 The equation sin x + x cos x = 0 has
atleast one root in the interval .
(c) f (x) = x |x |
æ p ö æ p 3p ö
(d) f (x) = | x | (a) ç - , 0 ÷ (b) (0 , p) (c) ç - , ÷ (d) None of these
è 2 ø è 2 2ø
422 Textbook of Differential Calculus

Sol. Consider the function given by, Sol. Let f ( x ) = e - x - sin x and a and b be two roots of the
f ( x ) = ò (sin x + x cos x ) dx = x sin x equation e x sin x - 1 = 0 such that a < b.
we observe that f (0) = f ( p ) = 0 Then, e a sin a = 1 and e b sin b = 1
\ 0 and p are two roots of f ( x ) = 0. Þ e - a - sin a = 0 and e -b - sin b = 0 …(i)
Consequently, f ¢ ( x ) = 0, i.e. sin x + x cos x = 0 has at least Clearly, f ( x ) is continuous on [a , b ] and differentiable on
one root in (0, p). (a , b ) . Also, f (a ) = f (b ) = 0 [using Eq. (i)]
Hence, (b) is the correct answer. \By Rolle’s theorem there exists c Î(a , b ) such that
y Example 51 Between any two real roots of the f ¢(c ) = 0
equation e x sin x - 1 = 0, the equation e x cos x + 1 = 0 Þ - e -c - cos c = 0
has Þ e c cos c + 1 = 0
(a) at least one root (b) at most one root Þ x = c is root of e x cos x + 1 = 0 ; where c Î(a , b )
(c) exactly one root (d) no root Hence, (a) is the correct answer.

Exercise for Session 5


1. If f ( x ) = x a log x and f (0) = 0, then the value of ‘a’ for which Rolle’s theorem can be applied in [0, 1,] is
(a) - 2 (b) - 1
1
(c) 0 (d)
2
1
2. If a, b , c are non-zero real numbers such that ò (1 + cos 8 x ) (ax 2 + bx + c ) dx
0
2
= ò (1 + cos 8 x ) (ax 2 + bx + c ) dx = 0, then the equation ax 2 + bx + c = 0 will have
0
(a) one root between 0 and 1 and another between 1 and 2
(b) both the roots between 0 and 1
(c) both the roots between 1 and 2
(d) None of the above
3
3. Let n Î N, if the value of c prescribed in Rolle’s theorem for the function f ( x ) = 2x ( x - 3)n on [0, 3] is , then n is
4
equal to
(a) 1 (b) 3
(c) 5 (d) 7

4. If f ( x ) > x ; " x Î R. Then, the equation f (f ( x )) - x = 0, has [IIT]


(a) atleast one real root
(b) more than one real root
(c) no real root if f (x ) is a polynomial and one real root if f (x ) is not a polynomial
(d) no real root

5. If f ( x ) is twice differentiable function such that f (a ) = 0, f (b ) = 2, f (c ) = - 1, f (d ) = 2, f (e ) = 0, where


a < b < c < d < e, then the minimum number of zeroes of g( x ) = (f ¢ ( x ))2 + f ¢¢ ( x ) × f ( x ) in the interval [a, e], is [IIT]
(a) 4 (b) 5
(c) 6 (d) 7
Session 6
Application of Cubic Functions
Application of Cubic Functions 4. All three distinct real roots
Y Y
The cubic function y = ax 3 + bx 2 + cx + d will have
x2
1. One real and two imaginary roots x1 X
(always monotonic), " x Î R x2
X
x1
Condition f ¢ ( x ) ³ 0 or f ¢ ( x ) £ 0 together with either
f ¢ ( x ) = 0 has no root (i.e. D < 0) or f ¢ ( x ) = 0 has a Figure 7.22
f ( x1 ) × f ( x 2 ) < 0
root x = a, then f (a ) = 0.
e.g. y = x 3 - 2x 2 + 5x + 4 where x 1 and x 2 are the roots of f ¢ ( x ) = 0
5. All the three roots are coincident
y ¢ = 3x 2 - 4x + 5 [ D < 0]
Y
y = ( x - 2) 3 Þ y ¢ = 3 ( x - 2) 2 = 0 Inflection
point
Þ x = 2 , also f (2 ) = 0 gives x = 2 , y (2 ) = 0
In this case, if f ¢ ( x ) = 0 has a root x = a and f (a ) = 0 X
this would mean f ( x ) = 0 has repeated roots, which is
dealt with separately.
Figure 7.23
2. Exactly one root and non-monotonic
f ¢ ( x ) ³ 0 or f ¢ ( x ) £ 0
Y
f (x) and f (a ) = 0
where a is a root of f ¢ ( x ) = 0
e.g. y = ( x - 1) 3
X′ x1 x2 X
Y′ Remarks
Y (i) Graph of every cubic polynomial must have exactly one point
of inflection.
X′ X (ii) In case (4),if f ( a), f ( b), f ( c ) and f ( d ) alternatively change sign.
x1
y Example 52 If the cubic function f ( x ) = x 3 + px + q
Y′ has 3 distinct real roots, then prove that 4p 3 + 27q 2 < 0 .
x2
Figure 7.20 Sol. f ¢ ( x ) = 3x 2 + p = 3x 2 + 0 × x + p
p
f (x 1 ) × f (x 2 ) > 0 Now, we have x 1 + x 2 = 0 and x 1x 2 = , where x 1, x 2
are roots of f ¢ ( x ) = 0 3
where x 1 and x 2 are the roots of f ¢ ( x ) = 0
According to given condition, we have
3. Two coincident and One different f ( x 1 ) × f ( x 2 ) = 0 ( x 13 + px 1 + q ) ( x 23 + px 2 + q ) < 0
Y
Y Þ x 13 × x 23 + px 13 x 2 + qx 13
x1 x2 + p 2 x 1x 2 + px 1x 23 + qx 23 + pqx 1 + q 2 + pqx 2 < 0
X′ X
Þ ( x 1x 2 )3 + px 1x 2 ( x 12 + x 22 ) + q ( x 13 + x 23 ) + pq ( x 1 + x 2 )
X′ x1 x2
X + p 2 x 1x 2 + q 2 < 0
Y′
Y′ Þ ( x 1x 2 )3 + px 1x 2 {( x 1 + x 2 )2 - 2x 1x 2 } + q {( x 1 + x 2 )3
Figure 7.21
- 3( x 1x 2 ) ( x 1 + x 2 )} + pq ( x 1 + x 2 ) + p 2 x 1x 2 + q 2 < 0
where x 1 and x 2 are the roots of f ¢ ( x ) = 0
424 Textbook of Differential Calculus

p 3 p 2 ì 2p ü 2ì p ü Sol. Take y = x 3 - 9 x 2 + 24 x
Þ + í- ý + p í ý + q < 0
2
27 3 î 3þ î 3þ dy
Þ = 3x 2 - 18x + 24 = 3 ( x 2 - 6x + 8)
dx
Þ 4 p 3 + 27q 2 < 0
= 3 ( x - 2) ( x - 4 )
y Example 53 If f ( x ) is a polynomial of degree 5 + +

with real coefficients such that f (| x | ) = 0 has 8 real 2 4

roots, then f ( x ) = 0 has for three real roots of


f ( x ) = x 3 - 9 x 2 + 24 x + c , f (2) f ( 4 ) < 0
(a) 4 real roots
\ (c + 20) (c + 16) < 0
(b) 5 real roots
c Î(–20, – 16)
(c) 3 real roots
Now, if c Î ( -20, - 18), a Î (1, 2), b Î (2, 3), g Î( 4, 5)
(d) nothing can be said
y=x 3– 9x2+24x
Sol. Given that f (| x | ) = 0 has 8 real roots.
Y
Þ f ( x ) = 0 has 4 positive roots. 20 (2, 20)
Since, f ( x ) is a polynomial of degree 5, f ( x ) cannot have (3, 18)
even number of real roots. 18
Þ f ( x ) has all the five roots real, in which four positive
and one root is negative. 16 (1, 16) (4, 16)
Hence, (b) is the correct answer. 0
X
1 2 3 4
y Example 54 If the function f ( x ) = x 3 - 9 x 2 + 24 x + c
has three real and distinct roots a , b and g, then the
value of [a ] + [b ] + [ g ] are Þ [a ] + [b ] + [ g ] =7
(a) 5, 6 If c Î ( -18, - 16)
(b) 6, 7 Þ a Î (1, 2),b Î (3, 4 ), g Î ( 4, 5)
(c) 7, 8 Þ [a ] + [b ] + [ g ] = 8
(d) None of the above Hence, (c) is the correct answer.

Exercise for Session 6


1. Find all the possible values of the parameter a, so that x 3 – 3x + a = 0 has three real and distinct roots.

2. f ( x ) is a polynomial of degree 4 with real coefficients such that f ( x ) = 0 is satisfied by x = 1, 2, 3 only, then
f ¢(1) × f ¢ (2) × f ¢ (3) is equal to
(a) 0 (b) 2
(c) –1 (d) None of these

3. If the function f ( x ) = | x 2 + a | x | + b | has exactly three points of non-differentiability, then which of the following
can be true?
(a) b = 0,a < 0 (b) b < 0, a ÎR
(c) b > 0, a ÎR (d) All of these

4. If the equation e| | x | - 2| + b = 2 has four solutions, then b lies in


(a) (log 2, - log 2) (b) (log 2 - 2 , log 2)
(c) (- 2, log 2) (d) (0, log 2)
JEE Type Solved Examples :
Single Option Correct Type Questions
l Ex. 1 Number of integral value(s) of k for which the If we put a = e , then loge x = x /e will have only one
equation 4 x 2 - 16 x + k = 0 has one root lie between 1 and 2 solution at x = e .
and other root lies between 2 and 3, is x
[Q the line y = is tangent to the curve y = loge x at x = e ]
(a) 1 (b) 2 (c) 3 (d) 4 e
Sol. (c) Here, f (1) × f (2) < 0
x
Þ loge x = has more than one solution for a Î (e , ¥ ).
Þ (k - 12) (k - 16) < 0 a
− x +9
+ + l Ex. 4 The tangent to the hyperbola y = passing
12 16 through the origin is x + 5
Þ 12 < k < 16 ...(i) (a) x + 25y = 0 (b) 5x + y = 0
Also, f ( 2) × f ( 3) < 0 (c) 5x - y = 0 (d) x - 25y = 0
Þ (k - 16) (k - 12) < 0 Þ 12 < k < 16 ...(ii) 4 4
Sol. (a) Here, y = 1 + Þ (dy /dx ) at ( x 1, y1 ) = -
From Eqs. (i) and (ii), we get x +5 ( x 1 + 5) 2
k = {13, 14, 15} Now, equation of tangent, is
\ Number of integral values of k = 3 æ 4 ö 4
y - ç1 + ÷=- ( x - x1 )
è x1 + 5 ø ( x 1 + 5) 2
l Ex. 2 Let a, b, c Î R such that two of them are equal and
2a b c Since, it passes through (0, 0), therefore
4 4 x1
satisfy b c 2a = 0, then equation 24ax 2 + 4bx + c = 0 -1 - =
x 1 + 5 ( x 1 + 5) 2
c 2a b
Þ - ( x 1 + 5) 2 - 4 ( x 1 + 5) = 4 x 1
has
Þ ( x 1 + 5) 2 + 4 ( x 1 + 5) + 4 x 1 = 0
(a) at least one root in (0, 1/ 2)
(b) at least one root in ( -1/ 2, 1/ 2) Þ x 12 + 18x 1 + 45 = 0
(c) at least one root in ( -1, 0) Þ ( x 1 + 15)( x 1 + 3) = 0 Þ x 1 = -15 or - 3
(d) at least two roots in (0, 2)
So, equation of tangents are
Sol. (a) Given determinant is
x + 25y = 0 or x + y = 0
2a (bc - 4a 2 ) - b (b 2 - 2ac ) + c (2ab - c 2 ) = 0
l Ex. 5 The tangent to the curve y = e x drawn at the point
Þ 6abc - 8a 3 - b 3 - c 3 = 0
(c , e c ) intersects the line joining the points (c - 1 , e c - 1 ) and
or (2a + b + c ) [(2a - b )2 + (b - c )2 + (c - 2a )2 ] = 0 (c + 1 , e c + 1 ) [IIT JEE 2007]
Þ 2a + b + c = 0 ...(i) (a) on the left of x = c (b) on the right of x = c
[Qb ¹ c ] (c) at no point (d) at all points
Let f ( x ) = 8ax 3 + 2bx 2 + c x Þ f ¢( x ) = 24a x 2 + 4bx + c Sol. (a) Slope of the line joining the points (c - 1, e c - 1 )
2a + b + c ec + 1 - ec - 1
Here, f (0) = 0 and f (1 /2) = a + b /2 + c /2 = and (c + 1, e c + 1 ) is equal to > ec
2 2
\ f ( 1 / 2) = 0 [using Eq. (i)]
y = ex
So, f ( x ) satisfy Rolle’s theorem and hence f ¢( x ) = 0 must
Y B
have at least one root in (0, 1 /2).

l Ex. 3 The set of values of ‘a’ for which the equation


log e (a log e x ) = log e x has more than one solution is
A
(a) (1, ¥ ) (b) (e , ¥ ) (c) (0, e ) (d) (1, e ) (c, ec)
X′ X
Sol. (b) Here, loge (a loge x ) = loge x O
Y′
Þ a loge x = e loge x Þ a loge x = x
426 Textbook of Differential Calculus

Þ Tangent to the curve y = e x will intersect the given line e c ( e - e -1 )


y - ec - 1 = [ x - (c - 1)] …(ii)
to the left of the line x = c . 2
Aliter Eliminating y from Eqs. (i) and (ii), we get
The equation of the tangent to the curve y = e x at (c , e c ) is [ x - (c - 1)] [2 - (e - e -1 )] = 2e -1
y - ec = ec ( x - c ) …(i) e + e -1 - 2
or x -c = <0
Equation of the line joining the given points is 2 - ( e - e -1 )
Þ x <c

JEE Type Solved Examples :


More than One Correct Option Type Questions
l Ex. 6 The coordinate of the point(s) on the graph of the l Ex. 8 If f ( x ) is continuous and derivable, " x Î R and
x 3 5x 2 f ¢(c ) = 0 for exactly 2 real values of ‘c’, then the number of
function, f ( x ) = - + 7 x - 4 where the tangent drawn
3 2 real and distinct values of ‘d’ for which f (d ) = 0 can be
cuts-off intercepts from the coordinate axes which are equal (a) 1 (b) 2 (c) 3 (d) 4
in magnitude but opposite in sign, is Sol. (a, b, c ) If f ¢( x ) = 0 has n real roots, then
æ 8ö æ 7ö f ( x ) = 0 has atmost (n + 1) real roots.
(a) ç 2, ÷ (b) ç3, ÷
è 3ø è 2ø Now, if f ¢(c ) = 0 for exactly 2 real values of c.
æ 5ö Then, following cases may arise
(c) ç1, ÷ (d) None of these
è 6ø (a)
Y
Sol. (a, b ) Since, intercepts are equal in magnitude but oppo-
site in sign, therefore
é dy ù
êë dx úû = 1 X′ X
P
O
Y′
dy
Now, = x 2 - 5x + 7 = 1
dx Þ f ( x ) = 0 have 1 real root.
Þ x - 5x + 6 = 0
2 (b)
Y
\ x = 2 or 3

l Ex. 7 Let f :[0, 1 ] ® R be a differentiable function with


non-increasing derivative such that f (0 ) = 0, f ¢(1) > 0, then X′
O
X
(a) f (1) ³ f ¢(1)
(b) f ¢(c ) ¹ 0 for any c Î(0, 1) Y′
(c) f (1/ 2) > f (1) Þ f ( x ) = 0 has 2 real roots.
(d) None of the above (c)
Sol. (a, b ) By Lagrange’s mean value theorem, Y
f ( 1) - f ( 0)
= f ¢(c ), c Î(0, 1)
1-0
X′ X
Q f ¢( x ) is non-increasing. O
\ f ¢(c ) ³ f ¢(1)
Þ f ¢ (c ) > 0 [Q f ¢ (1) > 0] Y′
Also, f (1) = f (1) - f (0) = f ¢ (c ) ³ f ¢(1)
Þ f ( x ) = 0 has 3 real roots.
\ f (1) ³ f ¢ (1).
Chap 07 dy/dx as a Rate Measurer and Tangents, Normals 427

l Ex. 9 Which of the following statements are true, where l Ex. 10 Let f ( x ) be twice differentiable function such
f( x ) is a polynomial? that f ¢¢( x ) < 0 in [0, 2]. Then,
(a) Between any two roots of f( x ) = 0 there exists at least (a) f (0) + f ( 2) = 2 f (c ), 0 < c < 2
one root of f ¢( x ) + l f ( x ) = 0 (b) f (0) + f ( 2) = 2 f (1)
(b) Between any two roots of f( x ) = 0 there exists at least (c) f (0) + f ( 2) > 2 f (1)
one root of x f ¢ ( x ) + l f( x ) = 0
(d) f (0) + f ( 2) < 2 f (1)
(c) Between any two roots of f( x ) = 0 there exists at least
Sol. (a, d ) By intermediate mean value theorem, we get
one root of ( x 2 + 1) f ¢ ( x ) + f ( x ) = 0
f ( 0) + f ( 2)
(d) Between any two roots of f( x ) = 0 there exists at least = f (c ), 0 < c < 2 ...(i)
2
one root of f ¢ ( x ) + x f ( x ) = 0
By Lagrange’s mean value theorem, we get
Sol. (c , d ) If a and b are the consecutive roots of f( x ) = 0, then
f (1) - f (0) = f ¢(c 1 ), 0 < c 1 < 1 ...(ii)
f ¢(a ) × f ¢(b ) £ 0
f (2) - f (1) = f ¢(c 2 ), 1 < c 2 < 2 ...(iii)
Let f( x ) = ( x - 1)( x - 2) On subtracting Eq. (ii) from Eq. (iii), we get
f ¢( x ) = 2x - 3 f (2) + f (0) - 2 f (1) = f ¢(c 2 ) - f ¢(c 1 ) ...(iv)
\ f ¢( x ) + l f( x ) = 0 Again, by Lagrange’s mean value theorem, we get
Þ (2x - 3) + l ( x - 1)( x - 2) = 0 f ¢(c 2 ) - f ¢(c 1 )
f ¢¢(c 3 ) = , for some c 3 Î(c 1, c 2 )
c 2 - c1
Must have at least one root Î(a , b ).
Þ f ¢(c 2 ) - f ¢ (c 1 ) = (c 2 - c 1 ) f ¢¢(c 3 ) < 0 [f ¢ ¢ ( x ) < 0] ...(v)
Similarly, ( x 2 + 1) f ¢( x ) + f ( x ) = 0
From Eqs. (iv) and (v), we get
and f ¢( x ) + x f ( x ) = 0 has at least one root Î(a , b ). f ( 2) + f ( 0) - 2 f ( 1) < 0 Þ f ( 0) + f ( 2) < 2 f ( 1)

JEE Type Solved Examples :


Statements I and II Type Questions
n
Directions (Q. Nos. 11 to 13) For the following questions,
choose the correct answers from the option (a), (b), (c) and \ The tangent meets the curve again at x = - 1.
(d) defined as follows
\ Statement I is false and Statement II is true.
(a) Statement I is true, Statement II is also true;
Statement II is the correct explanation
of Statement I
l Ex. 12 Statement I The ratio of length of tangent to
length of normal is inversely proportional to the ordinate of
(b) Statement I is true, Statement II is also true;
Statement II is not the correct explanation the point of tengency at the curve y 2 = 4ax .
of Statement I Statement II Length of normal and tangent to a curve
(c) Statement I is true, Statement II is false
(d) Statement I is false, Statement II is true y 1 + m2
y = f ( x ) is y 1 + m 2 and ,
m
l Ex. 11 Statement I The tangent at x =1 to the curve
y = x 3 - x 2 - x + 2 again meets the curve at x = - 2 . dy
where m = .
Statement II When an equation of a tangent solved with dx
the curve, repeated roots are obtained at the point of Sol. (a) Let the slope of the tangent be denoted by tan y and
tangency. the length of tangent = y cosec y.
Sol. (d) When x = 1, then y = 1. Also, y ¢ = 3x 2 - 2x - 1 Then, length of normal = y sec y
Length of tangent 1
Þ y¢ = 0 Þ equation of tangent is y = 1. \ = cot y µ
x =1 Length of normal y
Solving x 3 - x 2 - x + 2 = 1
\ Statement I is true.
or x 3 - x 2 - x + 1 = 0, we get x = - 1, 1 2
æ dy ö
Length of normal = y 1 + ç ÷
è dx ø
428 Textbook of Differential Calculus

= y 1 + m2 æ dy ö
Statement I ç ÷ = -3
è dx ø at (0 , 1)
2
æ dx ö y 1 + m2 \ Equation of the tangent is y - 1 = - 3 ( x - 0)
Length of tangent = y 1 + ç ÷ = i.e. y = - 3x + 1
è dy ø m
Now, - 3x + 1 = x 3 - 3x + 1 Þ x = 0
\ Statement II is true and explain Statement I. \ The tangent meets the curve at one point only.
\ Statement I is true.
l Ex. 13 Statement I Tangent drawn at the point (0, 1) æ dy ö
Statement II ç ÷ =0
to the curve y = x 3 - 3 x + 1 meets the curve thrice at one è dx ø at (1, -1)
point only. \ Equation of the tangent is y + 1 = 0 ( x - 1)
Statement II Tangent drawn at the point (1, -1) to the i.e. y = -1
curve y = x 3 - 3 x + 1 meets the curve at one point only. Now, - 1 = x 3 - 3x + 1

dy Þ x 3 - 3x + 2 = 0
Sol. (c) = 3x 2 - 3
dx Þ ( x - 1)( x 2 + x - 2) = 0
Þ The tangent meets the curve at two points.
\ Statement II is false.

JEE Type Solved Examples :


Passage Based Questions
Passage I (Ex. Nos. 14 to 18) l Ex. 17 If D = 4(a 2 - 3b ) > 0 and f ( x 1 ) × f ( x 2 ) = 0, where
Let f ( x ) = x 3 + ax 2 + bx + c be the given cubic polynomial x 1 , x 2 are the roots of f ( x ), then
and f ( x ) = 0 be the corresponding cubic equation, where (a) f ( x) has all real and distinct roots
a , b, c Î R. Now, f ¢ ( x ) = 3 x 2 + 2ax + b (b) f ( x) has three real roots but one of the roots would be
repeated
Let D = 4a 2 - 12b = 4 (a 2 - 3b ) be the discriminant of the
(c) f ( x) would have just one real root
equation f ¢ ( x ) = 0.
(d) f ( x) has three real roots but all are same
l Ex. 14 If D = 4(a 2 - 3b ) < 0, then l Ex. 18 If D = 4(a 2 - 3b ) = 0, then
(a) f ( x) has all real roots
(a) f ( x) has all real and distinct roots
(b) f ( x) has one real and two imaginary roots
(b) f ( x) has three real roots but one of the roots would be
(c) f ( x) has repeated roots repeated
(d) None of the above (c) f ( x) would have just one real root
l Ex. 15 If D = 4(a 2 - 3b ) > 0 and f ( x 1 ) × f ( x 2 ) > 0, where (d) None of the above

x 1 , x 2 are the roots of f ( x ), then n


Sol. (Ex. Nos. 14 to 18)
(a) f ( x) has all real and distinct roots Case I If D < 0 Þ f ¢ ( x ) > 0, " x Î R
(b) f ( x) has three real roots but one of the roots would That means f ( x ) would be an increasing function of x.
be repeated
Also, lim f ( - x ) = - ¥ and lim f ( x ) = ¥
(c) f ( x) would have just one real root x ®¥ x ®¥
(d) None of the above Y

lEx. 16 If D = 4 (a 2 - 3b ) > 0 and f ( x 1 ) × f ( x 2 ) < 0,


where x 1 , x 2 are the roots of f ( x ), then X′ X
C
(a) f ( x) has all real and distinct roots
(b) f ( x) has three real roots but one of the roots would be
repeated Y′
(c) f ( x) would have just one real root
The graph of f ( x ) would look like as shown in this figure.
(d) None of the above
Chap 07 dy/dx as a Rate Measurer and Tangents, Normals 429

It is clear that the graph of f ( x ) would intersect the X -axis 16. (a) Here, f ( x ) = 0 with three distinct root a , b, g .
only once. Y
f(x1) > 0
That means we would have just one real root (say x 0 ). f(x2) > 0
Case II Clearly, x 0 > 0, if c < 0 and x 0 < 0, if c > 0.
α x1 x2
If D > 0, f ¢ ( x ) = 0 would have two real roots (say x 1 and x 2 , X′ γ X
0 β
let x 1 < x 2 ).
Þ f ¢ ( x ) = 3 ( x - x1 ) ( x - x 2 )
Clearly, f ¢ ( x ) < 0, x Î ( x 1, x 2 ) Y′

f ¢ ( x ) > 0, x Î ( - ¥, x 1 ) ( x 2 , ¥ ) From graph, f ( x 1 ) f ( x 2 ) < 0


That means f ( x ) would increase in ( - ¥, x 1 ) and ( x 2 , ¥ ) \ f ( x ) has real and distinct roots.
and would decrease in ( x 1, x 2 ). Hence, x = x 1 would be a
point of local maxima and x = x 2 would be a point of local
17. (b) Here, f ( x ) = 0 with three roots x = a, x 2 (x 2 being
minima. Thus, the graph of y = f ( x ) could have these five repeated root).
possibilities. Y

Thus, the following results are obtained from the above f(x1) > 0, f(x2) = 0
graphs
(a) f ( x 1 ) × f ( x 2 ) > 0, f ( x ) = 0 would have just one real root.
X′ X
(b) f ( x 1 ) × f ( x 2 ) < 0, f ( x ) = 0 would have three real and 0 x1 x2
distinct roots.
(c) f ( x 1 ) × f ( x 2 ) = 0, f ( x ) = 0 would have three real roots
about one of the roots would be repeated. Y′

Case III If D = 0, f ¢ ( x ) = 3 ( x - x 1 )2 , where x 1 is root of 18. (b) Here, f ( x ) = 0 with three real roots x = x 1 , a ( x 1
being repeated root).
f ¢( x ) = 0
Y f(x1) = 0, f(x2) < 0
Þ f ( x ) = ( x - x 1 )3 + k
\ f ( x ) = 0 has three real roots, if k = 0 X′ 0 x1 x2 X
f ( x ) = 0 have one real root, if k ¹ 0.
14. (b) Here, f ( x ) = 0 and D < 0
Þ f ¢ ( x ) > 0, "x Î R Y′
Y
Passage II (Ex. Nos. 19 to 21)
n
If y = f ( x ) is a curve and if there exists two points
X′ X A( x1, f ( x1 )) and B( x2, f ( x2 )) on it such that
0
1 f ( x2 ) - f ( x1 )
f ¢ ( x1 ) = - = , then the tangent at x1 is
f ¢ ( x2 ) x2 - x1
Y′
normal at x2 for that curve. Now, answer the following
15. (c) Here, f ( x ) = 0 with one real root x = a and other two questions.
imaginary roots. l Ex. 19 Number of such lines on the curve y = sin x is
Y f (x1) < 0
Y f(x1) > 0, f(x2) > 0 (a) 1 (b) 0 (c) 2 (d) infinite
f (x2) < 0
Sol. (b) Let f ( x ) = y = sin x
dy
or Then, f ¢( x ) = = cos x
X′ X dx
1 sin x 2 - sin x 1
0 α x1 x2 X \ cos x 1 = - =
cos x 2 x 2 - x1
Y′ i.e. cos x 1 cos x 2 = - 1
From both graph, f ( x 1 ) f ( x 2 ) > 0 \ sin x 1 = sin x 2 = 0
\ There is no solution.
\ f ( x ) would have one real roots.
430 Textbook of Differential Calculus

l Ex. 20 Number of such lines on the curve y = |ln x | is Gives t =±


4
9
(a) 1 (b) 2 (c) 0 (d) infinite
Sol. (c) f ( x ) = y = | ln x | æ 4ö
h ç- ÷ = 0
1 f ( x 2 ) - f ( x1 ) è 9ø
\ f ¢ ( x1 ) = - =
f ¢( x 2 ) x 2 - x1 \ There are two distinct solutions of
ln x 1 x | ln x 2 | | ln x 2 | - | ln x 1| 729 x 13 - 432x 1 - 128 = 0.
Þ =- 2 = …(i)
x 1 | ln x 1| ln x 2 x 2 - x1
Þ ln x 1 × ln x 2 < 0
Passage III (Ex. Nos. 22 and 23)
x
Let 0 < x 1 < 1, then 1 < x 2 and x 1 × x 2 = 1 n Let f ( x ) = ò (|t - 1|-|t + 2|+ t - 2) dt, such that f ¢¢( a ) ¹ 1. If
0
From Eq. (i), we get vectors a$i - b2$j and $i + 3 b$j are parallel for at least one a,
1 ln x 2 + ln x 1 ln x 1 x 2 then
- = - x2 = = = 0,
x1 x 2 - x1 x 2 - x1
which is not possible.
l Ex. 22 Number of integral values of ‘b’ can be
(a) 5 (b) 10
\There is no solution.
(c) 11 (d) 13
l Ex. 21 Number of such lines on the curve y 2 = x 3 , is
l Ex. 23 Maximum value of (1 - 8b - b 2 ) is
(a) 1 (b) 2
(c) 3 (d) 0 (a) 4 (b) 8
Sol. (b) We know that, y = x 2 3 (c) 12 (d) 16
x
\ y = x 3 or - x 3 Sol. (Ex. Nos. 22 and 23) Here, f ( x ) = ò0 (|t - 1| - |t + 2| + t - 2) dt
dy Þ f ¢( x ) = | x - 1| - | x + 2| + x - 2
2y = 3x 2
dx ì x + 1, x < -2
ï
3x 12 2y 9 or f ¢( x ) = í - x - 3, - 2 £ x £ 1
= - 22 Þ x 12 x 22 = - y1y 2 ï x - 5,
2y1 3x 2 4 î x >1

\ y1y 2 < 0 Clearly, f ¢¢(a ) ¹ 1 only when a Î [ - 2, 1] ...(i)


Since, vectors are parallel.
Let y1 = x 13 and y 2 = - x 23
a -b 2
3 \ = Þ b 2 + 3ab = 0 ...(ii)
3x 12 2 x 23 - x 2 - x 13 1 3b
Thus, =- =
2 x 13 3x 22 x 2 - x1 Let g (a ) = b 2 + 3ab must have atleast one root in [ - 2, 1].

3 x1 2 x 23 + x 13 \ g ( - 2) × g ( 1) £ 0
Þ = =
2 3 x2 x1 - x 2 Þ (b 2 - 6b )(b 2 + 3b ) £ 0
4 Þ b 2 (b - 6)(b + 3) £ 0
Þ x1 x 2 =
9 + – +
–3 6
and 3x 1 x 1 - 3 x 1 x 2 = 2 x 23 +2 x 13
\ b Î [ - 3, 6] È {0}
3 ´ 16 64
Þ 3( x 1 )3 - = 2× + 2 x 13 Þ Number of integral values of ‘b’ can be 11.
81 x 1 3
729 x 1
Now, let h (b ) = 1 - 8b - b 2
16 128 h ¢ (b ) = -8 - 2b = 0
Þ 3x 13 - x1 = + 2x 13
27 729 Þ b = -4
Þ x 13 -
16
x1 =
128 \ h (b ) is decreasing, when b > -4
27 729 Þ (h(b )max ) at b = -3
Þ 739 x 13 - 432x 1 - 12 = 0 \ (h( -3))max = 16
22. (c)
Consider, h (t ) = 729 x 3 - 432t - 128
23. (d)
h ¢ (t ) = 3 ´ 729t 2 - 432 = 0
JEE Type Solved Examples :
Matching Type Questions
l Ex. 24 Match the statements of Column I with values of

Column II.
Column I Column II (C) We have y = 2e 2 x , which intersect Y -axis at (0, 2)
(A) The sides of a triangle vary slightly in such (p) 1 dy
Now, = 4e 2 x
a way that its circumradius remains dx
constant, if
æ dy ö
da
+
db
+
dc
+ 1 = | m |, then the \ ç ÷ =4
cos A cos B cos C è dx ø (0 , 2 )
value of m is \ Angle of intersection with Y-axis
(B) If the length of subtangent to the curve (q) –1 p
x 2 y2 = 16 at the point (–2, 2) is | k |, then the = - tan -1 4 = cot -1 4
2
value of k is
8n - 4
(C) If the curve y = 2e2x intersects the Y-axis at (r) 2 Hence = 4 , n = 2 or – 1
3
an angle cot -1 |(8n - 4 )/ 3 | , then the value of
n is (D) We have, x = e sin y
(D) If the area of a triangle formed by normal at (s) –2 dx
Þ = cos y e sin y
the point (1, 0) on the curve x = esin y with dy
axes is |2t + 1 |/6 sq units, then the value
Slope of normal at (1, 0) = - 1
of t is
Equation of normal is x + y = 1
Sol. (A) ® (p, q); (B) ® (r, s); (C) ® (r, q); (D) ® (p, s) 1
Area of triangle =
(A) We know that in any DABC, 2
a b c |2t + 1| 1
= = = 2R \ =
sin A sin B sin C 6 2
Þ a = 2 R sin A , b = 2 R sin B, c = 2 R sin C Þ t = 1, - 2
da db dc
Þ = 2R cos A , = 2R cos B, = 2R cos C
dA dB dC lEx. 25 Match the statements of Column I with values of
da db dc Column II.
Þ + + = 2R(dA + dB + dC )
cos A cos B cos C Column I Column II
Also, A + B + C = p (A) Rolle’s theorem in [–1, 1] ìæ 1 2
\ dA + dB + dC = 0 ö 1
does not hold, if ï ç - x÷ , x >
ïè 2 ø 2
da db dc (p) f (x ) = í
Hence, + + =0
ï æ - xö , x £
1 1
ïî çè 2 ÷
cos A cos B cos C
ø 2
da db dc
or + + +1 = 1
cos A cos B cos C (B) LMVT in [ 0, 1] does not (q) f (x ) = | x |
hold, if
Þ | m | = 1 Þ m = ±1
(C) Rolle’s theorem in [ 0, 1] ì 1
(B) We have, x 2y 2 = 16 does not hold, if ïï1, 0 £ x < 2
f (x ) = í
dy dy (r)
Þ 2x 2y + 2xy 2 = 0 Þ = - y /x ï 2, 1 £ x £ 1
dx dx ïî 2
dx x (s) f (x ) = log | x |
Length of subtangent = y = -y = | x |
dy y (t) f (x ) = x 2 + x + 1
Þ Length of subtangent = 2
\ k = ±2
432 Textbook of Differential Calculus

Sol. (A) ® (p,q,r,s,t); (B) ® (p, r, s); (C) ® (p,r,s,t) Y


ìæ 1 ö
2
1 2
ïç - x ÷ , x >
ïè 2 ø 2
(p) f (x ) = í
ï æ 1 - x ö, x £ 1
ïî çè 2 ÷ 1
ø 2
ì æ1 ö 1
ïï -2 çè 2 - x ÷ø, x > 2 X′ X
\ f ¢( x ) = í O 1/2 1
ï -1, 1

ïî 2
Y′
æ1+ ö æ1- ö
f ¢ç ÷ ¹f ¢ç ÷ Þ f ( x ) is discontinuous at x =
1
.
è2 ø è2 ø 2
1 Y
Þ f ( x ) is not differentiable at x = .
2
(q) f ( x ) = | x | is not differential at x = 0.
X′ X
y (–1, 0) O (1, 0)
f(x)= x

x′ x
O Y′
f ( x ) = log| x | is discontinuous at x = 0 .
y′
(t) f ( x ) = x 2 + x + 1
ì 1 Þ f ( - 1) = 1
ï1, 0 £ x < 2
(r) f (x ) = í and f ( 1) = 3
1
ï2, £ x <1 Þ f ( - 1) ¹ f ( 1)
î 2

JEE Type Solved Examples :


Single Integer Answer Type Questions
l Ex. 26 Suppose that f (0 ) = - 3 and f ¢ ( x ) £ 5 for all
values of x. Then, the largest value which f ( 2 ) can assume is T 3 - t3
………… . \ 3t 2 = = T 2 + Tt + t 2
T -t
Sol. (7) Using LMVT in [0, 2]
f ( 2) - f ( 0) Þ T 2 + Tt - 2t 2 = 0
= f ¢ (c ) [where c Î(0, 2)]
2-0 Þ (T - t ) (T + 2t ) = 0
f ( 2) + 3 Þ T = t or T = - 2t [T = t is not possible]
Þ £5
2
B (T, T3)
Þ f ( 2) £ 7
(t, t3) A
l Ex. 27 Let C be the curve y = x 3 (where x assumes all
real values). The tangent at A meets the curve again at B. If
the gradient at B is k times the gradient at A, then k is equal
to ………… . t3
Now, mA = = t 2; mB = T 2
dy t
Sol. (4) = 3x 2 = 3t 2 at ‘A’
dx
Chap 07 dy/dx as a Rate Measurer and Tangents, Normals 433

m B T 2 4t 2 Sol. (3) Let f ( x ) = tan -1 x , 0 < a < b < 3.


Þ = 2 = 2 [using T = - 2t ]
mA t t tan -1 b - tan -1 a 1
Then, by using LMVT, = ,
\ k =4 b -a 1+c2
where 0 < a < c < b < 3
l Ex. 28 Consider the two graphs y = 2 x and
1 1 1
x - xy + 2y 2 = 28. The absolute value of the tangent of the
2 So, < <
4 1+c2 1
angle between the two curves at the points where they meet, 1 tan -1 b - tan -1 a
is ……… . Þ < <1
4 b -a
Sol. (2) y = 2x , x 2 - xy + 2y 2 = 28 b -a
or 1< <4
Solving the point of intersection are (2, 4) and ( - 2, - 4 ). tan b - tan -1 a
-1

dy
For 1st curve, = 2 = m1 …(i) \ Maximum possible integral value is 3.
dx
dy y - 2x
For 2nd curve, = Þ m2 = 0 …(ii) l Ex. 31 Let P be a point on the curve c 1 : y = 2 - x 2 and
dx 4y - x
Q be a point on the curve c 2 : xy = 9, both P and Q be in the
\ tan q = 2 first quadrant. If d denotes the minimum distance between
P and Q, then d 2 is, ……
l Ex. 29 At the point P (a , a n ) on the graph of
Sol. (8) Here, c 1 is a semi-circle and c 2 is a rectangular
y = x n (n Î N ) in the first quadrant a normal is drawn. The
hyperbola. PQ will be minimum if the normal at P on
1
normal intersects the Y-axis at the point (0, b). If lim b = , semi-circle is also a normal at Q on xy = 9.
a ®0
Let the normal at P be y = mx (m > 0).
2 ...(i)
then n equals ......... .
Solving with xy = 9
dy
Sol. (2) y = x n Þ = n x n - 1 = nan - 1 at p (a, an ) Y
dx c2
1 y=mx
Slope of normal = - n - 1
na Q
xy=9
c1
Y
P
(0, b) X′ X
–√2 O √2
(a, an) y = xn
Y′
P
mx = 9 Þ x = 3/ m
2

X Þ y = 9 m /3
O
\ Q (3 / m , 3 m )
1
Equation of normal is y - a = - n
( x - a) Differentiating xy = 9 Þ x dy /dx + y = 0
nan - 1
1 Þ dy /dx = -y / x
Put x = 0 to get y-intercept , then y = an + n-2
na 3 m m
\ | dy /dx |Q = - = -m
1 3
Hence, b = an +
nan - 2 Now, tangent at P and Q must be parallel.
é 0, if n < 2 \ - m = -1/m Þ m 2 = 1 Þ m = 1
ê1 1
Now, lim b = ê , if n = 2 Þ lim b = , if n = 2 \ Normal at P and Q is y = x
a®0 2 a®0 2
ê ¥ , if n > 2 Now, we get P(1, 1) and Q(3, 3).
ë
\ ( PQ )2 = d 2 = 4 + 4 = 8
l Ex. 30 The maximum possible integral value of
b -a
, where 0 < a < b < 3 is
tan b - tan -1 a
-1
Subjective Type Questions
l Ex. 32 If the line x cos a + y sin a = P is the normal to Required points are (–6, 3) and (6, – 3).
the curve ( x + a )y = c 2 , then show that Distance of (–6, 3) from the given line
| – 18 + 6 + 1 | 11
æ p ö æ 3p ö = =
a Î ç 2np + , ( 2n +1) p ÷ È ç 2np + , ( 2n + 2 ) p ÷ . 13 13
è 2 ø è 2 ø
and distance of (6, - 3) from the given line
c2 dy c2
Sol. Here, y= Þ =– | 18 – 6 + 1 | 13
x +a dx ( x + a )2 = = = 13
13 13
( x + a )2 Thus, (–6, 3) is the required point.
Slope of normal is > 0 for all x.
c2
Q x cos a + y sin a = P is normal
l Ex. 35 Tangent at a point P1 (other than (0, 0 )) on the
cos a curve y = x 3 meets the curve again at P2 . The tangent at P2
\ – >0 Þ cota < 0
sin a meets the curve at P3 and so on, show that the abscissa of
i.e. a lies in II or IV quadrant. P1 , P2 , P3 ,..., Pn , form a GP. Also, find the ratio of
æ p ö æ 3p ö ar ( D P1P2 P3 )
So, a Î ç2np + , (2n + 1)p ÷ È ç2np + , (2n + 2)p ÷ .
è 2 ø è 2 ø ar ( D P2 P3 P4 )
l Ex. 33 Find the total number of parallel tangents of Sol. Let P1( x 1, y1 ) be a point on the curve.
f 1 ( x ) = x 2 – x + 1 and f 2 ( x ) = x 3 – x 2 – 2 x + 1 . y = x3 ...(i)
Sol. Here, f 1( x ) = x 2 – x + 1 Then, we have y1 = x 13 ...(ii)
and f 2 ( x ) = x – x – 2x + 1
3 2
y = x3
Y
Þ f 1¢ ( x 1 ) = 2x 1 – 1
f 2¢ ( x 2 ) = 3x 2 – 2x 2 – 2
2 P1 (x1 , y1)
and
Let the tangents drawn to the curves y = f 1( x ) and X¢ O X
y = f 2 ( x ) at ( x 1, f 1( x 1 )) and ( x 2 , f 2 ( x 2 )) respectively are
parallel.
2 2
\ 2x 1 – 1 = 3x 2 – 2x 2 – 2 or 2x 1 = 3x 2 – 2x 2 – 1 Y¢
Which is possible for infinite numbers of ordered pairs. dy
\It will have infinite number of parallel tangents. Now, = 3x 2
dx
lEx. 34 Find the point on the curve 3 x 2 - 4y 2 = 72 , \ Slope of the tangent at P1 = m1 = 3x 12
which is nearest to the line 3 x + 2y + 1 = 0. \ Equation of the tangent at P1( x 1, y1 ) is
Sol. Slope of the given line 3x + 2y + 1 = 0 is (–3/ 2) . y – x 13 = 3x 12 ( x – x 1 ) or y = 3x 12 x – 2x 13 ...(iii)
Let us locate the point ( x 1, y1 ) on the curve at which the Solving Eqs. (i) and (iii), we get
tangent is parallel to the given line. x 3 – 3x 12 x + 2x 13 = 0
Differentiating the curve both sides with respect to x, we get i.e. ( x – x 1 ) × ( x 2 + xx 1 – 2x 12 ) = 0
dy ( x – x 1 )( x – x 1 )( x + 2x 1 ) = 0
6x - 8y =0 i.e.
dx
\ x = x 1 (neglecting) or x = –2x 1
æ dy ö 3x 1 – 3
\ x 2 = – 2x 1, y 2 = x 23 = – 8x 13
Þ ç ÷ = = [Q parallel to 3x + 2y = 1]
è dx ø (x 4y 1 2
1 , y1 )
\ P2 ( x 2 , y 2 ) = (–2x 1, – 8x 13 )
Also, the point ( x 1, y1 ) lies on 3x - 4y = 72 2 2
Now, we find P3 , the point where the tangent, at P2 meets the
\ 3x 12 - 4y12 = 72 curve.
æ dy ö
x 12 72 72 é x1 ù Slope of the tangent at P2 = ç ÷
Þ -4= Þ 3( 4 ) - 4 = è dx ø (x , y )
3
y12 y12 y12 ê as y = – 2ú 2 2
ë 1 û
= 3x 22 = 3 × 4 x 12 = 12x 12
Þ y12 = 9 Þ y1 = ± 3
Chap 07 dy/dx as a Rate Measurer and Tangents, Normals 435

\ Equation of tangent at P2 is, æ – x + 3 – 3x 2 ö


As, P( x ) = P ç ÷
y– x 23 = 3x 22 ( x – x2) ...(iv) ç 2 ÷
è ø
To get, P3 = ( x 3 , y 3 ), solve Eqs. (i) and (iv), we get
æ – x + 3 – 3x 2 ö æ – x + 3 – 3x 2 ö
P3 = ( x 3 , y 3 ) = (–2x 2 , – 8x 23 ) = ( 4 x 1, 64 x 13 ) and so on. Þ 3ç ÷–4 ç ÷ = 3x – 4 x 3
ç 2 ÷ ç 2 ÷
è ø è ø
\Abscissa of P1, P2 , P3 , .... are given by x 1, – 2x 1, 4 x 1, – 8x 1, ....,
which forms an GP with common ratio = – 2. æ – x + 3 – 3x 2 ö
\ P1( x ) = P1 ç ÷
x 1 y1 1 ç 2 ÷
1 è ø
Now, a ( DP1P2 P3 ) = x2 y2 1
2 \ P1( x ) = P1(0) + (3x – 4 x 3 ) P2 ( x ) [using Eq. (ii)]
x3 y3 1
Þ P ( x ) = P (0) + (3x – 4 x 3 ) ( P1(0)) + (3x – 4 x 3 )P2 ( x ))
x1 x 13 1 = P (0) + (3x – 4 x 3 ) P1(0) + (3x – 4 x 3 )2 P2 ( x )
1
Þ ar ( DP1P2 P3 ) = –2x 1 –8x 13 1 Thus, in general,
2 P ( x ) = a 0 + a1(3x – 4 x 3 ) + a 2 (3x – 4 x 3 )2
4 x 1 64 x 13 1
+ K + (3x – 4 x 3 )k × k ( x )
1 1 1
x 14 Where k ( x ) is a polynomial with rational coefficient.
Þ ar ( DP1P2 P3 ) = –2 –8 1
2 l Ex. 37 Let f ( x ) = ( x – a ) ( x – b ) ( x – c ), a < b < c . Show
4 64 1
that f ¢( x ) = 0 has two roots one belonging to (a , b ) and other
x2 y2 1
1 belonging to (b, c ) .
Similarly, ar ( D P2 P3 P4 ) = x3 y3 1
2 Sol. Here, f ( x ) being a polynomial, is continuous and
x4 y4 1 differentiable for all real values of x.
–2x 1 – 8x 13 1 We also have, f (a ) = f (b ) = f (c )
1 If we apply Rolle’s theorem to f ( x ) in [a, b ] and [b, c ] we
Þ ar ( D P2 P3 P4 ) = 4 x1 64 x 13 1
2 observe that f ¢( x ) = 0 would have atleast one root in (a, b )
–8x 1 -512x 13 1 and atleast one root in (b, c ).
1 1 1 But f ¢( x ) = 0 is a polynomial of degree two, hence
Þ ar ( D P2 P3 P4 ) = 8x 14 –2 –8 1 f ¢( x ) = 0 cannot have more than two roots.
4 64 1 It implies that exactly one root of f ¢ ( x ) = 0 would lie in
(a, b ) and exactly one root of f ¢ ( x ) = 0 would lie in (b, c ).
ar ( D P1P2 P3 ) 1
\ = l Ex. 38 If at each point of the curve y = x 3 – ax 2 + x + 1 ,
ar ( D P2 P3 P4 ) 16
the tangents is inclined at an acute angle with the positive
l Ex. 36 Determine all polynomial P ( x ) with rational direction of the X-axis, then find the interval of a.
coefficient so that for all x with | x | £1 ; Sol. As, y = x 3 – ax 2 + x + 1 and the tangent is inclined at an
æ – x + 3 – 3x 2 ö acute angle with the positive direction of X -axis.
P(x ) = P ç ÷.
ç ÷ \
dy
³ 0 Þ 3x 2 – 2ax + 1 ³ 0, for all x Î R
è 2 ø
dx
æ – x + 3 – 3x 2 ö [we know, ax 2 + bx + c ³ 0 for all x Î R
Sol. Here, P ( x ) = P ç ÷ for | x | £ 1 ...(i)
ç 2 ÷
è ø Þ a > 0 and D £ 0]
Let x = 0 Þ P (0) = P ( 3/ 2) Þ (2a ) – 4(3)(1) £ 0 Þ 4 (a – 3) £ 0
2 2

æ 3ö Þ (a – 3 )(a + 3 ) £ 0 \ – 3 £a£ 3
Which shows, P ( x ) – P (0) is divisible by x ç x – ÷.
è 2 ø
l Ex. 39 Show that there is no cubic curve for which the
3
Since, P ( x ) – P (0) has rational coefficients and is one of tangent lines at two distinct points coincide.
the roots. 2
3 Sol. Suppose y º ax 3 + bx 2 + cx + d = 0 (a ¹ 0) be a cubic curve.
\ – is also a root of P ( x ) – P (0). We assume that ( x 1, y1 ) and ( x 2 , y 2 ), ( x 1 < x 2 ) are two
2
æ 3öæ 3ö 3 4 x 3 – 3x distinct points on the curve at which tangents coincide.
÷ çx + ÷=x – x=
3
Thus, x ç x – is Then, by mean value theorem, there exists
è 2 øè 2 ø 4 4
x 3 ( x 1 < x 3 < x 2 ) such that
factor of P ( x ) – P (0).
\ P ( x ) = P (0) + (3x – 4 x 3 ) P1( x ) for | x | £ 1 y 2 – y1
...(ii) = y ¢( x 3 )
x 2 – x1
436 Textbook of Differential Calculus

Since, tangent x 1, x 2 , x 3 are solutions of equation l Ex. 41 If the function of f :[0, 4 ] ® R is differentiable,
3ax + 2bx + c = M
2 then show that
But, it is a quadratic and thus cannot have more than two (i) (f (4))2 – (f (0))2 = 8 f ¢(a ) f (b ) for some a, b Î (0, 4).
roots. Therefore, no such cubic is possible. 4
ò 0 f (t ) dt = 2 {af ( a ) + b f (b 2 )} for some 0 < a, b < 2 .
2
(ii)
l Ex. 40 The tangent at a point P to the rectangular [IIT JEE 2003]
hyperbola xy = c 2 meets the lines x – y = 0, x + y = 0 at Q Sol. (i) Here, f is differentiable Þ f is also continuous.
and R, D 1 is the area of the D OQR, where O is the origin. The Now, by Lagrange’s mean value theorem, there exist
normal at P meets the X-axis at M and Y -axis at N. If D 2 is a Î(0, 4 ) such that
the area of the DOMN, show that D 2 varies inversely as the f ¢(a ) =
f ( 4 ) – f ( 0) f ( 4 ) – f ( 0)
= ...(i)
square of D 1 . 4 –0 4
Sol. Tangent at P ( x 1, y1 ) is xy1 + yx 1 = 2c 2 Also, by intermediate mean value theorem, there exists
b Î(0, 4 ) such that
The point of intersection of this line with x – y = 0 is given by,
f ( 4 ) + f ( 0)
x ( x 1 + y1 ) = 2c 2 , i.e. x = 2c 2 / ( x 1 + y1 ) f (b ) = ...(ii)
2
æ 2c 2 2c 2 ö From Eqs. (i) and (ii), we get
\ Q is ç , ÷
è x 1 + y1 x 1 + y1 ø f ( 4 ) – f ( 0) f ( 4 ) + f ( 0)
f ¢(a ) f (b ) = ´
4 2
The point of intersection of the tangent with x + y = 0 is
( f ( 4 ))2 – ( f (0))2
given by, f ¢(a ) f (b ) =
2c 2 8
x (y1 – x 1 ) = 2c 2 , x =
y1 – x 1 Þ ( f ( 4 ))2 – ( f (0))2 = 8 f ¢ (a ) f (b ) for some, a, b Î (0, 4 )
æ 2c 2 2c 2 ö (ii) Putting t = z 2 , we have
\ R is ç , ÷
è y1 – x 1 x 1 – y1 ø 4 2
ò0 f (t ) dt = ò0 2z f (z
2
) dz
1 1
Now, D1 = {a1b 2 – a 2b1 } = {–a1a 2 – a 2a1 } = – a1a 2
2 2 Consider the function f (t ) given by,
4c 4 t
f (t ) = ò 0 2z f ( z ) dz ; t Î[0, 2]
2
D1 = 2
x 1 – y12
x Clearly, f(t ) being an integral function of a continuous
The normal at ( x 1, y1 ) is y – y1 = 1 ( x – x 1 )
y1 function, is continuous and differentiable on [0, 2].
y12 \ By Lagrange’s mean value theorem, there exists
Intersection with y = 0 is given by, x – x 1 = – f ( 2) – f ( 0)
x1 c Î(0, 2) such that = f¢ (c )
2–0
x 12 – y12 æ x 2 - y 22 ö
Þ x= \ M is ç 1 , 0÷ 2 0
ò0 2z f (z ò0 2z f (z
2 2
) dz – ) dz
x1 è x1 ø Þ = 2cf (c 2 )
x 2 2–0
intersection with x = 0 is given by y – y1 = – 1
y1 [using f ¢ (t ) = 2t f (t 2 )]
2
y – x1 2 æ 2
y - x1
2ö 2
Þ ò0 2z f (z ) dz = 4cf (c 2 )
2
Þ y= 1 . \ N is ç0, 1 ÷ ...(i)
y1 è y1 ø
1 ( x 12 – y12 ) (y12 – x 12 ) Also, by intermediate mean value theorem for c Î(0, 2),
Now, D 2 = × there exist a , b Î (0, 2) such that
2 x1 y1
f¢ (a ) + f¢ (b )
1 ( x 12 – y12 )2 1 ( x 12 – y12 )2 = f¢ (c ), where 0 < a < c < b < 2
=– =– 2
2 x 1y1 2 c2
Þ 2af (a 2 ) + 2b f (b 2 ) = 2 { 2c f (c 2 )} ...(ii)
16c 8 2 2 2
æ 1 ö ( x 1 – y1 )
\ D 21 D 2 = 2 ç – ÷ = - 8c 6 = constant From Eqs. (i) and (ii), we get
( x 1 – y12 )2 è 2 ø c2 2
ò0 2z f (z ) dz = 2a f (a 2 ) + 2b f (b 2 ),
2
1
Þ D2 µ 2
D1 for some 0 < a , b < 2
or D 2 varies inversely as the square of D1. 4
Þ ò0 f (t ) dt = 2(a f (a ) + b f (b 2 )),
2

for some 0 < a , b < 2


dy/dx as Rate Measurer and Tangents, Normals Exercise 1 :
Single Option Correct Type Questions
1. Consider the cubic equation f ( x ) = x 3 - nx + 1 = 0 where 9. The curve x + y - ln ( x + y ) = 2x + 5 has a vertical
n ³ 3, n Î N , then f ( x ) = 0 has tangent at the point (a , b ). Then, a + b is equal to
(a) at least one root in ( 0, 1 ) (b) at least one root in (1, 2 ) (a) - 1 (b) 1 (c) 2 (d) - 2
(c) at least one root in ( -1, 0 ) (d) data insufficient 10. Let y = f ( x ), f : R ® R be an odd differentiable function
2. If the normal to y = f ( x ) at (0, 0) is given by y - x = 0, such that f ¢¢¢( x ) > 0 and
x2 g(a , b ) = sin 8 a + cos 8 b + 2 - 4 sin 2 a cos 2 b.
then lim is
x ®0 f ( x ) - 20 f (9 x ) + 2 f (99 x )
2 2 2
If f ¢ ¢ ( g (a , b )) = 0, then sin 2 a + sin 2 b is equal to
(a) 1 / 19 (b) - 1 / 19 (c) 1 / 2 (d) does not exist (a) 0 (b) 1 (c) 2 (d) 3
3. Tangent to a curve intersects the Y-axis at a point. A line 11. A polynomial of 6th degree f ( x ) satisfies
perpendicular to this tangent through P passes through f ( x ) = f (2 - x ), " x Î R, if f ( x ) = 0 has 4 distinct and
another point (1, 0). The differential equation of the 2 equal roots, then sum of the roots of f ( x ) = 0 is
curve is (a) 4 (b) 5 (c) 6 (d) 7
2 2
dy æ dy ö d 2y æ dy ö 12. Let a curve y = f ( x ), f ( x ) ³ 0, " x Î R has property that
(a) y -xç ÷ =1 (b) x 2 + ç ÷ = 1
dx è dx ø dx è dx ø
for every point P on the curve length of subnormal is
dy
(c) y + x =1 (d) None of these equal to abscissa of P. If f (1) = 3, then f ( 4 ) is equal to
dx
(a) - 2 6 (b) 2 6 (c) 3 5 (d) None of these
4. The number of points in the rectangle
13. If a variable tangent to the curve x y = c 3 makes2
{( x , y ):| x | £ 9, | y | £ 3} which lie on the curve
intercepts, a, b on X and Y-axes respectively, then the
y 2 = x + sin x and at which the tangent to the curve is
value of a 2 b is
parallel to X-axis, is 4 3
(a) 3 (b) 2 (c) 4 (d) None of these (a) 27 c 3 (b) c
27
a + 2c 4 27 3 4
(d) c 3
5. If a, b, c , d Î R such that = - , then the equation (c) c
b + 3d 3 4 9
ax + bx + cx + d = 0 has
3 2
1 1 1
(a) at least one root in ( -1, 0 ) (b) at least one root in ( 0, 1 ) 14. Let f ( x ) = 3 - x 5 - 3x 2
3x - 1 . Then, the
3

(c) no root in ( -1, 1 ) (d) no root in ( 0, 2 ) 2x - 1 3x - 1 7 x 8 - 1


2 5

6. If 3 (a + 2c ) = 4 (b + 3d ) ¹ 0, then the equation equation f ( x ) = 0 has


ax 3 + bx 2 + cx + d = 0 will have (a) no real root
(b) atmost one real root
(a) no real solution
(c) at least two real roots
(b) at least one real root in ( - 1, 0 )
(d) exactly one real root in (0, 1) and no other real root
(c) at least one real root in (0, 1)
(d) None of the above 15. The graphs y = 2x 3 - 4 x + 2 and y = x 3 + 2x - 1
7. Let f ( x ) be a differentiable function in the interval (0, 2), intersect at exactly 3 distinct points. The slope of the
2 line passing through two of these points is
then the value of ò f ( x ) dx (a) equal to 4 (b) equal to 6
0
(a) f (c ) where c Î( 0, 2 ) (b) 2 f (c ), where c Î( 0, 2 ) (c) equal to 8 (d) not unique
(c) f ¢(c ), where c Î( 0, 2 ) (d) None of these 16. In which of the following functions Rolle’s theorem is
8. Let f ( x ) be a fourth differentiable function such that applicable?
f (2x - 1) = 2xf ( x ), " x Î R, then f
2 iv
(0) is equal to ì x, 0 £ x < 1
(a) f ( x ) = í on [0, 1]
iv î 0, x =1
(where f (0) represents fourth derivative of f ( x ) at
ì sin x , - p £ x < 0
x = 0) ï
(b) f ( x ) = í x on [ - p , 0 ]
(a) 0 (b) 1 îï 0 , x=0
(c) - 1 (d) data insufficient
438 Textbook of Differential Calculus

x2 - x - 6 23. Water runs into an inverted conical tent at the rate of


(c) f ( x ) = on [ - 2, 3 ]
x -1 20 cu ft/min and leaks out at the rate of 5 cu ft/min. The
ì x 3 - 2x 2 - 5x + 6 height of the water is three times the radius of the
ïï , if x ¹ 1
x -1 water’s surface. The radius of the water surface is
(d) f ( x ) = í on [ - 2, 3 ]
ï increasing when the radius is 5 ft, is
ïî - 6, if x = 1 1 1 1
(a) ft/min (b) ft/min (c) ft/min (d) None of these
5p 10 p 15 p
17. The figure shows a right triangle with its hypotenuse OB
along the Y-axis and its vertex A on the parabola y = x 2 . 24. Let f ( x ) = x 3 - 3x 2 + 2x . If the equation f ( x ) = k has
exactly one positive and one negative solution, then the
Y
B value of k equals
2 3 2
(a) - (b) -
h 9 9
A (t, t 2 ) 2 1
(c) (d)
X′ X
3 3 3 3
O
25. The x-intercept of the tangent at any arbitrary point of
Let h represents the length of the hypotenuse which depends
a b
on the x-coordinate of the point A. The value of lim (h ) is
t ®0
the curve 2
+ = 1 is proportional to
equal to x y2
(a) 0 (b) 1/2 (c) 1 (d) 2
(a) square of the abscissa of the point of tangency
18. Number of positive integral value(s) of ‘a’ for which the (b) square root of the abscissa of the point of tangency
curve y = a x intersects the line y = x is (c) cube of the abscissa of the point of tangency
(a) 0 (b) 1 (c) 2 (d) more than 2 (d) cube root of the abscissa of the point of tangency

2/ 3 ì tan [x ] x ¹ 0 26. If f ( x ) is continuous and differentiable over [- 2, 5] and


ï
19. If f ( x ) = 4 - æç - x ö÷
1 ,
, g (x ) = í x - 4 £ f ¢ ( x ) £ 3 for all x in ( - 2, 5), then the greatest
è2 ø ïî 1 , x = 0 possible value of f (5) - f ( - 2) is
log 2 (x + 3 ) (a) 7 (b) 9
h ( x ) = {x }, k ( x ) = 5 , where [x] and {x} denote (c) 15 (d) 21
the greatest integer and fraction part function, then in
27. A curve is represented parametrically by the equations
[0, 1], Lagrange’s mean value theorem is not applicable to
x = t + e at and y = - t + e at , where t Î R and a > 0. If the
(a) f , g, h (b) h, k (c) f , g (d) g, h, k
curve touches the axis of x at the point A, then the
20. If the function f ( x ) = x 4 + bx 2 + 8x + 1 has a horizontal
coordinates of the point A are
tangent and a point of inflection for the same value of x,
(a) (1, 0) (b) (1 / e, 0 )
then the value of b is equal to (c) (e, 0 ) (d) (2e, 0 )
(a) - 1 (b) 1 (c) 6 (d) - 6
28. At any two points of the curve represented
21. Coffee is coming out from a conical filter, with height parametrically by x = a (2 cos t - cos 2t ),
and diameter both are 15 cm into a cylindrical coffee pot y = a (2 sin t - sin 2t ), the tangents are parallel to the axis
with a diameter 15 cm. The rate at which coffee comes of x corresponding to the values of the parameter t
out from the filter into the pot is 100 cu cm/min. differing from each other by
The rate (in cm/min) at which the level in the pot is (a) 2 p / 3 (b) 3 p / 4 (c) p / 2 (d) p / 3
rising at the instance when the coffee in the pot is
10 cm, is cos x (1 + sin –1 (t ))2 é pù
9 25 5 16 29. Let F ( x ) = ò e dt on ê0, ú, then
(a) (b) (c) (d) sin x ë 2û
16 p 9p 3p 9p
æ pö
(a) F ¢¢(c ) = 0 for all c Î ç 0, ÷
22. A horse runs along a circle with a speed of 20 km/h. è 2ø
A lantern is at the centre of the circle. A fence is there æ pö
(b) F ¢¢(c ) = 0 for some c Î ç 0, ÷
along the tangent to the circle at the point at which the è 2ø
horse starts. The speed with which the shadow of the æ pö
(c) F ¢(c ) = 0 for some c Î ç 0, ÷
horse moves along the fence at the moment when it è 2ø
covers 1/8 of the circle (in km/h) is æ pö
(d) F (c ) ¹ 0 for all c Î ç 0, ÷
(a) 20 (b) 40 (c) 30 (d) 60 è 2ø
Chap 07 dy/dx as a Rate Measurer and Tangents, Normals 439

d 33. Let S be a square with sides of length x. If we


30. If f ¢ (1) = 1 and ( f (2x )) = f ¢ ( x ), " x > 0. If f ¢ ( x ) is
dx approximate the change in size of the area of S by
differentiable, then there exists a number c Î(2, 4 ) such dA
h× , when the sides are changed from x 0 to
that f ¢ ¢ (c ) is equal to dx x = x 0
(a) - 1 /4
(b) - 1 /8 x 0 + h, then the absolute value of the error in our
(c) 1/4 approximation, is
(d) 1/8 (a) h 2 (b) 2hx 0 (c) x 02 (d) h

34. Consider f ( x ) = ò æçt + ö÷ dt and g ( x ) = f ¢ ( x ) for


31. Let f ( x ) and g ( x ) be two functions which are defined x 1
1 è ø
and differentiable for all x ³ x 0 . If f ( x 0 ) = g ( x 0 ) and t
f ¢ ( x ) > g ¢ ( x ) for all x > x 0 , then é 1 ù
x Î ê , 3ú . If P is a point on the curve y = g ( x ) such that
(a) f ( x ) < g ( x ) for some x > x 0 ë2 û
(b) f ( x ) = g ( x ) for some x > x 0 the tangent to this curve at P is parallel to a chord
(c) f ( x ) > g ( x ) only for some x > x 0 æ1 æ1öö
(d) f ( x ) > g ( x ) for all x > x 0 joining the points ç , g ç ÷ ÷ and (3, g (3)) of the curve,
è2 è2øø
32. The range of values of m for which the line y = mx and then the coordinates of the point P are
x æ 7 65 ö
the curve y = enclose a region, is (a) can’t be found out (b) ç , ÷
x2 +1 è 4 28 ø

(a) ( - 1, 1 ) (b) ( 0, 1 ) æ 3 5 ö
(c) (1, 2) (d) ç , ÷
(c) [ 0, 1 ] (d) (1, ¥ ) è 2 6ø

dy/dx as Rate Measurer & Tangents, Normals Exercise 2 :


More than One Option Correct Type Questions
35. For the curve represented parametrically by the 39. If f (0) = f (1) = f (2) = 0 and function f ( x ) is twice
equation, x = 2 log(cot t ) + 1 and y = tan t + cot t , then differentiable in (0, 2) and continuous in [0, 2], then
p which of the following is/are definitely true?
(a) tangent at t = is parallel to X -axis
4 (a) f ¢¢ (c ) = 0 ; " c Î ( 0, 2 )
p f ¢(c ) = 0 ; for at least two c Î( 0, 2 )
(b) normal at t = is parallel to Y -axis (b)
4 (c) f ¢(c ) = 0 ; for exactly one c Î( 0, 2 )
p (d) f ¢¢(c ) = 0 ; for at least one c Î( 0, 2 )
(c) tangent at t = is parallel to y = x
4 1
p 40. Equation - 3x + sin x = 0 has
(d) normal at t = is parallel to y = x ( x + 1) 3
4
(a) no real root
36. Consider the curve f ( x ) = x 1/ 3 , then (b) two real and distinct roots
(a) the equation of tangent at (0, 0) is x = 0 (c) exactly one negative root
(b) the equation of normal at (0, 0) is y = 0 (d) exactly one root between - 1 and ¥
(c) normal to the curve does not exist at (0, 0)
(d) f ( x ) and its inverse meet at exactly 3 points 41. If f is an odd continuous function in [- 1, 1] and
differentiable in ( - 1, 1), then
37. The angle at which the curve y = ke kx intersects Y -axis (a) f ¢( A ) = f (1 ) for some A Î ( - 1, 0 )
is (b) f ¢( B ) = f (1 ) for some B Î( 0, 1 )
(a) tan -1(k 2 ) (b) cot -1(k 2 ) (c) n ( f ( A )) n - 1 f ¢ ( A ) = ( f (1 ))n for some A Î ( - 1, 0 ), n Î N
æ 1 ö (d) n ( f ( B )) n - 1 f ¢ ( B ) = ( f (1 ))n for some B Î ( 0, 1 ), n Î N
(c) sin -1 ç ÷ (d) sec -1 ( 1 + k 4 )
ç 1 + k4 ÷
è ø 42. The parabola y = x 2 + px + q intersects the straight line
38. Let f ( x ) = 8x 3 - 6x 2 - 2x + 1, then y = 2x - 3 at a point with abscissa 1. If the distance
between the vertex of the parabola and the X -axis is
(a) f ( x ) = 0 has no root in ( 0, 1 )
(b) f ( x ) = 0 has at least one root in (0, 1)
least, then
(c) f ¢(c ) vanishes for some c Î( 0, 1 ) (a) p = 0 and q = - 2
(d) None of the above (b) p = - 2 and q = 0
440 Textbook of Differential Calculus

(c) least distance between the vertex of the parabola and 47. Equation of a line which is tangent to both the curves
X -axis is 2 y = x 2 + 1 and y = - x 2 is
(d) least distance between the vertex of the parabola and
1 1
X -axis is 1 (a) y = 2 x + (b) y = 2 x -
2 2
43. The abscissa of the point on the curve xy = a + x , the 1 1
(c) y = - 2 x + (d) y = - 2 x -
tangent at which cuts off equal intersects from the 2 2
coordinate axes, is (a > 0) 48. Let F ( x ) = ( f ( x )) 2 + ( f ¢( x )) 2 , F(0) = 6 where f ( x ) is
a a
(a) (b) - (c) a 2 (d) - a 2 thrice differentiable function such that | f ( x )| £ 1 for all
2 2
x Î [-1, 1], then choose the correct statement(s).
44. If the side of a triangle vary slightly in such a way that (a) There is at least one point in each of the intervals ( -1, 0 )
its circumradius remains constant, then and ( 0, 1 ) where | f ¢( x )| £ 2
da db dc
+ + is equal to (b) There is at least one point in each of the intervals ( -1, 0 )
cos A cos B cos C and ( 0, 1 ) where F ( x ) £ 5
(a) 6R (b) 2R (c) There is no point of local maxima of F ( x ) in ( -1, 1 )
(c) 0 (d) 2R (dA + dB + dC ) (d) For some c Î ( -1, 1 ), F (c ) ³ 6, F ¢(c ) = 0 and F ¢¢(c ) £ 0
45. Let f ( x ) satisfy the requirements of Lagrange’s mean 49. If the Rolle’s theorem is applicable to the function f
value theorem in [0, 1], f (0) = 0 and defined by
f ¢ ( x ) £ 1 - x , " x Î (0, 1), then ì 2
(a) f ( x ) ³ x (b) | f ( x )| ³ 1 ïax + b, | x | £ 1
(c) f ( x ) £ x (1 - x ) (d) f ( x ) £ 1 / 4 ï
f ( x ) = í 1, |x|=1
ln x ï c
46. For function f ( x ) = , which of the following ï |x|, |x|>1
x î
statements are true?
in the interval [-3, 3] , then which of the following
(a) f ( x ) has horizontal tangent at x = e
(b) f ( x ) cuts the X -axis only at one point
alternative(s) is/are correct?
(c) f ( x ) is many-one function (a) a + b + c = 2 (b) | a | + | b | + | c | = 3
(d) f ( x ) has one vertical tangent (c) 2a + 4b + 3c = 8 (d) 4a 2 + 4b 2 + 5c 2 = 15

dy/dx as Rate Measurer & Tangents, Normals Exercise 3 :


Statements I and II Type Questions
n
Directions (Q. Nos. 50 to 56) For the following questions, 51. Statement I Shortest distance between | x | + | y | = 2 and
choose the correct answer from the options (a), (b), (c) and x 2 + y 2 = 16 is 4 - 2 .
(d) defined as follows
(a) Statement I is true, Statement II is also true; Statement II Statement II Shortest distance between the two smooth
is the correct explanation of Statement I curves lies along the common normal.
(b) Statement I is true, Statement II is also true; Statement II 52. Statement I If a 1 , a 2 , a 3 , K , a n are the n real roots of
is not the correct explanation of Statement I
a polynomial equation of nth degree with real
(c) Statement I is true, Statement II is false
coefficients such that sum of the roots taken r (1 £ r £ n )
(d) Statement I is false, Statement II is true
at a time is positive, then all the roots are positive.
50. Statement I If g ( x ) is a differentiable function Statement II The number of times sign of coefficients
g (1) ¹ 0, g ( - 1) ¹ 0 and Rolle’s theorem is not applicable change while going left to right of a polynomial
x2 -1 equation is the number of maximum positive roots.
to f ( x ) = in [- 1, 1], then g ( x ) has atleast one root
g( x ) 53. Statement I Tangents at two distinct points of a cubic
in ( - 1, 1). polynomial cannot coincide.
Statement II If P( x ) is a polynomial of degree
Statement II If f (a ) = f (b ), then Rolle’s theorem is
n (n ³ 2), then P ¢ ( x ) + k cannot hold for n or more
applicable for x Î(a, b ).
distinct values of x.
Chap 07 dy/dx as a Rate Measurer and Tangents, Normals 441

ì 1 1 Statement II If f ( x ) is continuous in [a, b ] and is strictly


ïï 2 - x , x < 2 monotonic in (a, b ), then f has a unique root in (a, b ).
54. Statement I For f ( x ) = í 2 , mean value
ï æç 1 - x ö÷ , x ³ 1 56. Consider the polynomial function
ïî è 2 ø 2 x7 x6 x5 x4 x3 x2
theorem is applicable in the interval [0, 1]. f (x ) =- + - + - +x
7 6 5 4 3 2
Statement II For application of mean value theorem, Statement I The equation f ( x ) = 0 cannot have two or
f ( x ) must be continuous in [0, 1] and differentiable in (0, 1). more roots.
2x + 2
55. Let f ( x ) = ln (2 + x ) - . Statement II Rolle’s theorem is not applicable for
x +3 y = f ( x ) on any interval [a, b ] , where a, b Î R.
Statement I The function f ( x ) = 0 has a unique
solution in the domain of f ( x ).

dy/dx as Rate Measurer & Tangents, Normals Exercise 4 :


Passage Based Questions
Passage I (Q. Nos. 57 to 59) 61. The equation |ln mx | = px , where m is a positive
We say an equation f ( x ) = g ( x ) is consistent, if the curves constant, has exactly two roots for
y = f ( x ) and y = g ( x ) touch or intersect at at least one point. If m e
(a) p = (b) p =
the curves y = f ( x ) and y = g ( x ) do not intersect or touch, then e m
the equation f ( x ) = g ( x ) is said to be inconsistent, i.e. has no e m
(c) 0 < p £ (d) 0 < p £
solution. m e
57. The equation cos x + cos -1 x = sin x + sin -1 x is 62. The equation |ln mx | = px , where m is a positive
(a) consistent and has infinite number of solutions constant, has exactly three roots for
(b) consistent and has finite number of solutions m m
(a) p < (b) 0 < p <
(c) inconsistent e e
(d) None of the above e e
(c) 0 < p < (d) p <
m m
58. The equation sin x = x + x + 1 is
2

(a) consistent and has infinite number of solutions Passage III (Q. Nos. 63 to 64)
(b) consistent and has finite number of solutions Consider the family of circles: x 2 + y 2 - 3x - 4y - c i = 0, c i Î N
(c) inconsistent
(i = 1, 2, 3,...,n ) .
(d) consistent and has unique solution
Also, let all circles intersects X -axis at integral points only and
59. Among the following equations, which is consistent in c 1 < c 2 < c 3 < c 4 ... < c n . A point ( x , y ) is said to be integral point,
(0, p / 2)? if both coordinates x and y are integers.
(a) sin x + x 2 = 0 (b) cos x = x 63. If circle x 2 + y 2 - 3x - 4y - (c 2 - c 1 ) = 0 and circle
(c) tan x = x (d) All of these
x 2 + y 2 = r 2 have only one common tangent, then
Passage II (Q. Nos. 60 to 62) (a) r = 1 / 2
To find the point of contact P º ( x 1, y1 ) of a tangent to the graph (b) tangent passes through (10, 0 )
of y = f ( x ) passing through origin O, we equate the slope of (c) (3, 4) lies outside the circle x 2 + y 2 = r 2
tangent to y = f ( x ) at P to the slope of OP. Hence, we solve the (d) c 2 = 2r + c1
f ( x1 )
equation f ¢ ( x 1 ) = to get x 1 and y1. 64. The ellipse 4 x 2 + 9y 2 = 36 and hyperbola a 2 x 2 - y 2 = 4
x1
intersect orthogonally, then the equation of circle
60. The equation |ln mx | = px , where m is a positive through the points of intersection of two conics is
constant, has a single root for c4
m e (a) x 2 + y 2 = (c 5 ) 2 (b) x 2 + y 2 =
(a) 0 < p < (b) p < 7
e m (c) x 2 + y 2 = c 3 - 2 c1 (d) x 2 + y 2 = c 7 / 14
e m
(c) 0 < p < (d) p >
m e
442 Textbook of Differential Calculus

dy/dx as Rate Measurer & Tangents, Normals Exercise 5 :


Matching Type Questions
65. Match the statements of Column I with values of 66. Match the statements of Column I with values of
Column II. Column II.
Column I Column II Column I Column II
(A) The equation x log x = 3 - x has (p) (0, 1) (A) A circular plate is expanded by heat (p) 4
at least one root in from radius 5 cm to 5.06 cm.
(B) If 27a + 9b + 3c + d = 0, then the (q) (1, 3) Approximate increase in area is
equation (B) If an edge of a cube increases by 1%, (q) 0.6 p
4 ax 3 + 3bx 2 + 2cx + d = 0 has at then percentage increase in volume is
least one root in x2
(C) If the rate of decrease of - 2x + 5 (r) 3
1 2
(C) If c = 3 and f (x ) = x + , then (r) (0, 3)
x is twice the rate of decrease of x, then
interval of x in which LMVT is x is equal to (rate of decrease is
applicable for f (x ) is non-zero)
1 (D) Rate of increase in area of equilateral (s)
(D) If c = and f (x ) = 2x - x 2, then (s) (- 1, 1) 3 3
2 triangle of side 15 cm, when each
4
the interval of x in which LMVT is side is increasing at the rate of
applicable for f (x ), is 0.1 cm/s, is

dy/dx as Rate Measurer & Tangents, Normals Exercise 6 :


Single Integer Answer Type Questions
67. A point is moving along the curve y 3 = 27 x . The p b

interval in which the abscissa changes at slower rate


angle , then the value of
4 òa f ¢ ( x ) × f ¢¢ ( x ) dx is …… .

than ordinate, is (a, b). Then, (a + b ) is ……… . 72. Two curves C 1 : y = x 2 - 3 and C 2 : y = kx 2 , k Î R
68. The slope of the curve 2y = ax + b at (1, - 1) is - 1.
2 2
intersect each other at two different points. The tangent
Then, (a - b ) is ……… . drawn to C 2 at one of the points of intersection
A(a, y 1 ), (a > 0) meets C 1 again at B (1, y 2 ).
69. Let f (1) = - 2, f ¢ ( x ) ³ 4.2 for 1 £ x £ 6. The smallest
The value of a is ……… .
possible value of f (6) - 16 is ……… .
ì - x 2 , for x < 0 73. Consider the function f ( x ) = 8x 2 - 7 x + 5 on the
70. Let f ( x ) = í 2 . Then, the absolute interval [- 6, 6]. The value of c that satisfies the
î x + 8 , for x ³ 0 conclusion of the mean value theorem, is ……… .
value of x-intercept of the line that is tangent to the
graph of f ( x ), is ……… .
74. Suppose that f is differentiable for all x and that
f ¢ ( x ) £ 2 for all x. If f (1) = 2 and f ( 4 ) = 8, then f (2) has
71. The tangent to the graph of the function y = f ( x ) at that the value equal to ……… .
point with abscissa, x = a forms with the X - axis at an
p 75. Suppose a, b and c are positive integers with a < b < c
angle of and the point with abscissa at x = b at an such that 1/a + 1/b + 1/c = 1. The value of (a + b + c - 5)
3
is ……… .
Chap 07 dy/dx as a Rate Measurer and Tangents, Normals 443

Subjective Type Questions


76. Show that a tangent to an ellipse whose segment 82. If the equation of two curves are y 2 = 4ax and x 2 = 4ay
intercepted by the axes is the shortest, is divided at the (i) Find the angle of intersection of two curves.
point of tangency into two parts respectively, is equal to (ii) Find the equation of common tangents to these curves.
the semi-axes of the ellipse.
83. A straight line intersects the three concentric circles at
77. Tangents are drawn from the origin to the curve A, B, C . If the distance of the line from the centre of the
x2 circles is ‘ P’, prove that the area of the triangle formed by
y = sin x . Prove that points of contact lie on y 2 = .
1+ x2 æ1 ö
the tangents to the circle at A, B, C is ç × AB × BC × CA ÷.
x
è 2P ø
78. If f is a continuous function with ò f (t ) dt ® ¥ as
0 84. Find the equation of all possible curves such that length
| x | ® ¥, then show that every line y = mx intersects the of intercept made by any tangent on X-axis is equal to
x
curve y + ò f (t ) dt = 2 .
2 the square of x-coordinate of the point of tangency.
0 Given that the curve passes through (2, 1) .
79. Find the equation of the straight line which is a tangent 85. The tangent to the curve y = x – x 3 at a point P meets
at one point and normal at another point to the curve the curve again at Q. Prove that one point of trisection of
y = 8 t 3 – 1, x = 4t 2 + 3. PQ lies on the Y-axis. Find the locus of the other points
80. Let a curve y = f ( x ) passes through (1, 1), at any point P of trisection.
on the curve tangent and normal are drawn to intersect 86. Determine all the curves for which the ratio of the
the X -axis at Q and R, respectively. If QR = 2, find the length of the segment intercepted by any tangent on the
equation of all such possible curves. Y-axis to the length of the radius vector is constant.
81. Show that the angle between the tangent at any point P 87. If t be a real number satisfying 2t 3 – 9t 2 + 30 – a = 0,
and the line joining P to the origin ‘O’ is the same at all then the values of the parameter a for which the
points of the curve log ( x 2 + y 2 ) = c tan –1 (y/x ), where 1
equation x + = t gives six real and distinct values of x.
c is constant. x

dy/dx as Rate Measurer & Tangents, Normals Exercise 8 :


Questions Asked in Previous 10 Years’ Exams
(i) JEE Advanced & IIT-JEE
88. The slope of the tangent to the curve (y - x 5 ) 2 = x (1 + x 2 ) 2 at the point (1, 3) is [Integer Type Question 2014]

89. Let f ( x ) = 2 + cos x , for all real x.


Statement I For each real t, there exists a point c in [t , t + p ], such that f ¢ (c ) = 0. Because
Statement II f (t ) = f (t + 2p ) for each real t. [Assertion and Reason 2007]
(a) Statement I is correct, Statement II is also correct; Statement II is the correct explanation of Statement I
(b) Statement I is correct, Statement II is also correct; Statement II is not the correct explanation of Statement I
(c) Statement I is correct; Statement II is incorrect
(d) Statement I is incorrect; Statement II is correct

90. If | f ( x 1 ) - f ( x 2 ) | £ ( x 1 - x 2 ) 2 , "x 1 , x 2 Î R. Find the equation of tangent to the curve y = f ( x ) at the point (1, 2).
[Analytical Descriptive 2005]
91. The point(s) on the curve y 3 + 3x 2 = 12y, where the tangent is vertical, is (are) [One Correct Option 2002]
æ 4 ö æ 11 ö æ 4 ö
(a) ç ± , - 2÷ (b) ç ± , 0÷ (c) (0, 0) (d) ç ± , 2÷
è 3 ø è 3 ø è 3 ø
444 Textbook of Differential Calculus

3p
92. If the normal to the curve y = f ( x ) at the point (3, 4) makes an angle with the positive X-axis, then f ¢(3) is equal to
4
[One Correct Option 2000]
(a) –1 (b) –3/4 (c) 4/3 (d) 1

(ii) JEE Main & AIEEE


93. The normal to the curve y ( x - 2) ( x - 3) = x + 6 at the point where the curve intersects the Y-axis passes through the
point [2017 JEE Main]
æ1 1ö æ 1 1ö æ1 1ö æ1 1ö
(a) ç , ÷ (b) ç - , - ÷ (c) ç , ÷ (d) ç , - ÷
è2 3ø è 2 2ø è2 2ø è2 3ø

94. The normal to the curve x 2 + 2xy - 3y 2 = 0 at (1,1) [2015 JEE Main]
(a) does not meet the curve again (b) meets in the curve again the second quadrant
(c) meets the curve again in the third quadrant. (d) meets the curve again in the fourth quadrant

Answers
Exercise for Session 1 Exercise for Session 6
1. 6 cm /s 3
2. (1, 5 / 3) and (-1, 1/ 3) 1. Real and distinct roots if a Î (-2,2).
200 pr 3 2. (a) 3. (a) 4. (b)
3 2
3. 8 cm/s 4. 750 cm /s 5. km /h
(r + 5 ) 2 Chapter Exercises
1 1 1
6. cm/s 7. - cm/s 8. cm/s 1. (a) 2. (b) 3. (a) 4. (b) 5. (b)
48 p 3p 10p 6. (b) 7. (b) 8. (a) 9. (b) 10. (b)
11. (c) 12. (b) 13. (c) 14. (c) 15. (c)
-3 1
9. 20 m/s 10. rad /s 11. (a) cm/s 16. (d) 17. (c) 18. (b) 19. (a) 20. (d)
10 40p 21. (d) 22. (b) 23. (a) 24. (a) 25. (c)
1 10 26. (d) 27. (d) 28. (a) 29. (b) 30. (b)
(b) cm 2/s (c) cm 2/s
30 30 31. (d) 32. (b) 33. (a) 34. (d)
-16 ö 35. (a, b) 36. (a, b, d) 37. (b, c) 38. (b, c)
13. æç 3,
0.004 0.02 16 ö æ
12. m/s and m/s ÷, ç - 3, ÷
p p è 3ø è 3 ø 39. (b, d) 40. (b, c, d)
41. (a, b, d) 42. (b, d) 43. (a, b) 44. (c, d)
14. æç ö÷ units/s
2a 3
15. - m/s 45. (c, d) 46. (a, b, c) 47. (a, c)
èbø 8 48. (a, b, d) 49. (a, b, c, d) 50. (c)
51. (d) 52. (a) 53. (d) 54. (d) 55. (c)
Exercise for Session 2 56. (a) 57. (b) 58. (c) 59. (b) 60. (d) 61. (a)
50 62. (b) 63. (b) 64. (d)
1. 2. 2.1983 3. 0.02%
7 65. (A) ® (q); (B) ® (r); (C) ® (q); (D) ® (p)
66. (A) ® (q); (B) ® (r); (C) ® (p); (D) ® (s)
æ
4. ç ö÷ %
1
è8ø 67. (0) 68. (2) 69. (3) 70. (1) 71. (1)
72. (3) 73. (0) 74. (4) 75. (6)
Exercise for Session 3 79. ± 2 (27x - 105)
æ 1- 1 - y2 ö
1. (a) 2. (c) 3. (b) 4. (a) 5. (b) 80. log y - x = ± ç log - + 1 - y2 ÷ - 1
6. (b) 7. (d) 8. (c) 9. (b) 10. (a) ç y ÷
è ø
-1 æ 3 ö
Exercise for Session 4 82. (i) Q = tan ç ÷ (ii) x + y = a = 0
è4ø
1. (b) 2. (d) 3. (a) 4. (b) 5. (c) x 3x
6. (b) 7. (c) 8. (b) 9. (c) 10. (d) 84. Possible curve are y = or y = .
2(x - 1) 2(1 + x)
Exercise for Session 5 85. y = x - 5x3 86. ( y + x2 + y2 ) xk - 1 = c1
1. (d) 2. (a) 3. (b) 4. (d) 5. (c) 87. No real value 88. 8 89. (b) 90. y - 2 = 0
91. (d) 92. (d) 93. (c) 94. (d)
4. Here, y 2 = x + sin x

Solutions Þ

\
Since,
2y dy / dx = 1 + cos x
For horizontal tangent, dy / dx = 0
cos x = -1 Þ x = (2n + 1 ) p
y 2 = x + sin x and | y | £ 3
Þ 0 £ x £9
1. Here, f ( x ) = x - nx + 1
3 Þ 0 £ (2n + 1 ) p £ 9
\ n=0 Þ x=p
f ( 0 ) = 1 and f (1 ) = 1 - n + 1 = 2 - n
Q n ³3 Þ y 2 = p or y = ± p
\ f (1 ) < 0 and also we have, f ( 0 ) > 0 Thus, points are ( p , p ) and ( p , - p ).
\ f ( x ) must have at least one real root in ( 0, 1 ). \ Number of points is 2.
2. Given, slope of normal to y = f ( x ) is 1. a + 2c 4
5. Here, =-
æ 1 ö b + 3d 3
Þ ç- ÷ =1 Þ 3a + 6c = -4b - 12d
è f ¢( x ) ø ( 0, 0)
Þ 3a + 4b + 6c + 12d = 0
Þ f ¢( 0 ) = -1 ...(i) a b c
or + + +d = 0 ...(i)
x2 4 3 2
Now, lim
x ® 0 f ( x ) - 20 f (9 x 2 ) + 2 f (99 x 2 )
2
ax 4 bx 3 cx 2
Consider, f (x ) = + + + dx
2x 4 3 2
= lim
x ® 0 f ¢( x 2 ) × 2 x - 20 f ¢(9 x 2 ) × 18 x + 2 f ¢(99 x 2 ) × (198 x ) Then, f ( 0 ) = 0 = f (1 ) [using Eq. (i)] ...(ii)
[using L’ Hospital’s rule] \ f ( x ) satisfy the condition of Rolle’s theorem in [ 0, 1 ].
1 Hence, f ¢( x ) = 0 has at least one solution in ( 0, 1 ).
= lim
x ® 0 f ¢( x 2 ) - 180 f ¢(9 x 2 ) + f ¢(99 x 2 ) × (198 ) ax 4 bx 3 cx 2
6. Let f ( x ) = + + + dx,
1 4 3 2
=
f ¢( 0 ) - 180 × f ¢( 0 ) + 198 × f ¢( 0 ) which is continuous and differentiable.
1 a b c
= Also, f ( 0 ) = 0, f ( - 1 ) = - + - d
-1 + 180 - 198 4 3 2
1 1
=-
1 = (a + 2c ) - (b + 3d ) = 0
19 4 3
So, according to Rolle’s theorem, there exists at least one root
3. The equation of the tangent at the point R( x, f ( x )) is of f ¢( x ) = 0 in ( - 1, 0 ).
Y - f ( x ) = f ¢( x )( X - x ) ...(i) t
7. Let us consider g(t ) = ò f ( x ) dx
The point of intersection on Y-axis, is P ( 0, f ( x ) - x f ¢( x )). 0
The slope of line perpendicular to tangent at R, is Applying LMVT in (0, 2), we get
( f ( x ) - xf ¢( x )) - 0 g(2 ) - g( 0 )
mPQ = ...(ii) = g ¢(c ), where c Î( 0, 2 )
0 -1 2-0
2
Þ ò 0 f (x ) dx = 2 f (c ), where c Î(0, 2)
))

Y
(x
f
x,
R(

8. Replace x by - x
P Þ x[ f ( x ) + f ( - x )] = 0 Þ f ( x ) is an odd function.
Þ f iv ( x ) is also odd Þ f iv ( 0 ) = 0
(1, 0) 9. Given, x + y - ln ( x + y ) = 2x + 5
X′ X
O Q
dy 1 æ dy ö
Þ 1+ - ç1 + ÷ =2
Y′
dx x + y è dx ø
\ f ¢( x ) × mPQ = -1 dy x + y + 1
Þ =
( f ( x ) - xf ¢( x )) dx x + y - 1
Þ f ¢( x ) × = -1
-1 æ dy ö a + b+1
Þ f ( x ) f ¢( x ) - x ( f ¢( x )) 2 = 1 Þ ç ÷ = = ¥, when a + b = 1
è dx ø ( a , b ) a + b - 1
446 Textbook of Differential Calculus

10. Since, f ¢¢( x ) is odd function. Similarly, y 2 = 8x 2 - 4 …(iv)


\ g ( a, b ) = 0 Now, from Eqs. (iii) and (iv), we get y 2 - y1 = 8 ( x 2 - x1 )
y 2 - y1
Þ (sin a - 1 ) + (cos4 b - 1 ) 2 + 2 (sin 2 a - cos2 b ) 2 = 0
4 2 \ =8
x 2 - x1
Þ sin 2 a + sin 2 b = 1 16. (a) Discontinuous at x = 1, therefore not applicable.
11. Let a be the root of f ( x ) = 0. (b) Discontinuous at x = 0, therefore not applicable.
Þ f ( a ) = f (2 - a ) = 0 (c) Discontinuity at x = 1 Þ not applicable.
\ f ( x ) has 4 distinct and 2 equal roots. (d) Note that x 3 - 2 x 2 - 5 x + 6 = ( x - 1 ) ( x 2 - x - 6 ).
\ Sum of the roots = 6 Hence, f ( x ) = x 2 - x - 6, if x ¹ 1 and f (1 ) = - 6
dy Þ f is continuous at x = 1.
12. Given, y =x
dx So, f ( x ) = x 2 - x - 6 is continuous throughout the interval
Þ y dy = x dx Þ y 2 = x 2 + c
[ - 2, 3 ].
Q f (1 ) = 3 Also, note that f ( - 2 ) = f (3 ) = 0. Hence, Rolle’s theorem is
\ 9 = 1 + c Þ c = 8 Þ y 2 = x2 + 8 applicable.
1
Þ f (x ) = x 2 + 8 17. Let A = (t, t 2 ). Then, mOA = t, mAB = -
t
Þ f ( 4 ) = 16 + 8 = 2 6
Y
13. Given, x y = c 3
2
B
dy
Þ x2 + 2 xy = 0
dx h
dy 2y A (t, t2)
Þ =-
dx x
2y X′ X
Equation of tangent at ( x, y ) , is Y - y = - (X - x ) O
x Y′
3x
Y = 0, gives X = =a Equation of AB, y - t 2 = -
1
(x - t )
2 t
and X = 0, gives Y = 3y = b Putting x=0
9x 2 27 2 27 Þ h = t2 + 1
Now, a 2b = × 3y = x y = c3
4 4 4
Now, lim (h ) = lim (1 + t 2 ) = 1
14. Clearly, f ( 0) = f (1) = 0 and f ( x ) is a polynomial of degree 10. t ®0 t ®0

Therefore, by LMVT, we must have at least one root in (0, 1). 18. For 0 < a £ 1, the line always intersects y = a x
Since, the degree of f ( x ) is even. Y
\ It has at least two real roots.
15. Let ( x1, y1 ) and ( x 2, y 2 ) be two intersect points.
Given, y = x 3 + 2 x - 1 and y = 2 x 3 - 4 x + 2
y = ax, a ∈(0, 1)
\ y1 = 2 x13 - 4 x1 + 2 …(i)
and 2y1 = 2 x13 + 4 x1 - 2 …(ii) X′
O
X
Y′
Y
R
Y
Q
a=1

P
O X′ X
X′ X O

Y′
Y′
For a > 1, say a = e. Consider f ( x ) = e x - x, f ¢( x ) = e x - 1
Subtracting Eq. (i) from Eq. (ii), we get
y1 = 8 x1 - 4 …(iii) f ¢( x ) > 0 for x > 0 and f ¢( x ) < 0 for x < 0
Chap 07 dy/dx as a Rate Measurer and Tangents, Normals 447

\ f ( x ) is increasing ( ­ ) for x > 0 22. tan q = x / r Þ x = r tan q


and decreasing ( ¯ ) for x < 0 Y

y
x

θ
1 X′ r X
x′ x O
O

y′ Y′

y = e x always lies above y = x, i.e. e x - x ³ 1 for a > 1 Þ dx / dt = r sec q (dq / dt ) = r w sec 2 q = v sec 2 q
2

Hence, the line y = x intersect when a Î( 0, 1 ] . when, q = 2 p / 8, dx / dt = v sec 2 ( p / 4 ) = 2 v = 40 km/h.


1 dV 1 dx
19. Q f is not differentiable at x = , 23. = 15, h = 3r , V = px 2y , = ?, when x = 5
2 dt 3 dt
g is not continuous in [0, 1] at x = 0 and 1 , x r 1 2 15 cm
= = Þ V = p r 23 x = p x 3
and h is not continuous in [0, 1] at x = 1 y h 3 3 r
k ( x ) = ( x + 3 ) ln 2 5 = ( x + 3 ) p , where 2 < p < 3 dV
= 3 px 2 dx

\None of these, f , g, h follows Lagrange’s mean value dt dt


dx x h
theorem. Þ 15 = 3 p × 25
dt
20. f ¢( x ) = 0 and f ¢¢( x ) = 0 for the same x = x1 [say] y
dx 1
Now, f ¢ ( x ) = 4 x + 2bx + 8
3 Þ =
dt 5 p
f ¢ ( x1 ) = 2 [2 x13 + bx1 + 4 ] = 0 …(i)
24. Given, f ( x ) = x ( x 2 - 3x + 2)
f ¢¢ ( x1 ) = 2 [6 x12 + b ] = 0 …(ii)
Þ f ( x ) = x ( x - 2 )( x - 1 )
From Eq. (ii), b = - 6 x12 Graph of y = f ( x ) is shown as
Substituting this value of b in Eq. (i), we get
Y
2 x13 + ( - 6 x13 ) + 4 = 0
Þ 4 x13 = 4
Þ x1 = 1 1 + 1/√3
X′ X
Hence, b = -6 O 1 – 1/√3 1 2
21. For a cylindrical pot, V = pr h 2
y=k
dV é 2 dh dr ù é dr ù
Þ =p êër dt + h × 2r dt úû êër = constant, dt = 0 úû Y′
dt

15 cm
Now, for exactly one positive and one negative solution of the
equation f ( x ) = K
æ 1 ö
We should have, k = f ç1 + ÷
15 cm
è 3ø
é 1 1 ù
êëQ - 3 and 1 + 3 are the roots of f ¢( x ) = 0 ú
û
æ 1 öæ 1 öæ ö1
\ k = ç1 + ÷ç - 1÷ ç ÷
è 3ø è 3 ø è 3ø
1424 3 1424 3 123
x x-2 x -1

æ1 öæ 1 ö 2
= ç - 1÷ ç ÷ = -
è3 ø è 3ø 3 3
dh 225 dh é 15 ù
Þ 100 = pr 2 Þ 100 = p × × êëQ r = 2 cm úû 2 3
dt 4 dt =-
dh 400 16 3
Þ = = cm/min
dt 225 p 9 p
448 Textbook of Differential Calculus

a b 1 æpö 0
25. We have, + =1 Also, F ( 0 ) = ò f (t ) dt and F ç ÷ = ò f (t ) dt
x2 y 2 0 è2ø 1

2a 2b dy dy ay 3 æpö
Þ - - 3 =0 Þ =- 3 …(i) Hence, F ( 0 ) and F ç ÷ have opposite signs.
3 è2ø
x y dx dx bx
æ pö
ay 3 Þ F (c ) = 0 for some c Î ç 0, ÷
Equation of tangent is Y - y = - (X - x ) è 2ø
bx 3
30. Given, f ¢(1) = 1, 2 × f ¢(2x ) = f ¢( x )
For x-intercept, put Y = 0
f ¢(1 ) 1
bx 3 Put x = 1, f ¢(2 ) = =
\ X = +x 2 2
ay 2 1 1
and f ¢( 4 ) = f ¢(2 ) =
é bx 2 + ay 2 ù é x 2y 2 ù x 3 2 4
Þ X =xê ú=x ê 2ú=
ë ay
2
û ë ay û a Applying LMVT for y = f ¢( x ) in [2, 4], we get
1 1
[using Eq. (i)] -
f ¢( 4 ) - f ¢(2 ) 4 2 1
Þ x-intercept is proportional to cube of abscissa. f ¢¢(c ) = = =-
2 2 8
26. Using LMVT in [- 2, 5], we get 31. Consider f ( x ) = f ( x ) - g( x ) Þ f¢ ( x ) = f ¢( x ) - g ¢( x ) > 0.
f (5 ) - f ( - 2 )
-4£ £3 Clearly, f ( x ) is also continuous and derivable in [ x 0, x ].
7 Using LMVT for f ( x ) in [ x 0, x ] , we get
Þ - 28 £ f (5 ) - f ( - 2 ) £ 21 f (x ) - f (x 0 )
f¢ (c ) =
27. Given, x = t + eat, y = - t + eat x - x0
dx dy dy - 1 + aeat Since, f¢ ( x ) = f ¢( x ) - g ¢( x ) > 0 for all x > x 0
Þ = 1 + aeat
, = - 1 + aeat
, =
dt dt dx 1 + aeat \ f¢ (c ) > 0
dy Hence, f (x ) - f (x 0 ) > 0
At the point A, y = 0 and = 0 for some t = t1
dx Þ f (x ) > f (x 0 )
\ aeat1 = 1 …(i) Þ f( x ) > 0 [Q f ( x 0 ) = f ( x 0 ) - g( x 0 ) = 0 ]
Also, 0 = - t1 + e at1 Þ f (x ) - g (x ) > 0
x 1
\ eat1 = t1 …(ii) 32. Solving, mx = 2 Þ x2 + 1 = or x = 0
x +1 m
Putting this value in Eq. (i), we get Y y = mx
1
at1 = 1 Þ t1 =
a 1/2
Now from Eq. (i), ae = 1 y = x/(1 + x2)
1 –1
Þ a= X′ X
e O 1
Hence, x A = t1 + eat1 = e + e = 2e
–1/2
Þ A º (2e, 0 )
Y′
28. Given, x = a (2 cost - cos2t ) , y = a (2 sin t - sin 2t ) 1
dy cos t - cos 2t Þ x =
2
- 1 > 0 for a region
\ = =0 m
dx sin 2t - sin t m -1
Þ < 0 Þ m Î( 0, 1 )
Þ cos 2t = cos t m
Þ cos 2t = cos (2 p - t )
Þ t = 2p / 3 ? Remark
(1 + sin -1 (cos x )) 2 (1 + x ) 2 For m = 0 or 1, the line does not enclose a region.
29. F ¢( x ) = e × ( - sin x ) - e × cos x
\ F ¢( 0 ) = 0 - e = - e 33. Q Side = x
æpö dA ìï æ dA ö ïü
and F¢ç ÷ = -e - 0 = -e \Area A = x 2 Þ = 2 x. So, í ç ÷ ´ hý = 2 x 0h
è2ø è ø
dx îï dx x = x0 ïþ
Hence, Rolle’s theorem is applicable for the function F ¢( x ).
The exact change in the area of S when x is changed from x 0 to
æ pö x 0 + h, is
So, there exists c in ç 0, ÷ for which F ¢¢(c ) = 0 as
è 2ø
( x 0 + h ) 2 - x 02 = x 02 + 2 x 0h + h 2 - x 02 = 2 x 0h + h 2
é pù
Rolle’s theorem is applicable for F ¢( x ) in ê 0, ú . The difference between the exact change and the approximate
ë 2û change is 2 x 0h + h 2 - 2 x 0h = h 2
Chap 07 dy/dx as a Rate Measurer and Tangents, Normals 449

x æ 1ö 1 Equation of tangent at ( 0, 0 ) is x = 0.
34. f ( x ) = ò çt + ÷ dt Þ f ¢( x ) = x +
1 è tø x Equation of normal is y = 0.
\
1
g( x ) = x + Now, f ( x ) = f -1( x ) Þ x1/3 = x 3
x
é1 ù Þ x 9 = x Þ x = 0, 1, - 1
For x Î ê , 3ú,
ë2 û 37. Here,
dy
= k 2 ekx
æ1ö 1 5 1 10 dx
g ç ÷ = 2 + = , g (3 ) = 3 + =
è2ø 2 2 3 3 æ dy ö
Þ ç ÷ = k 2 = tan q, where q is angle made by X - axis.
é1 ù è dx ø x = 0
Let P º (c, g (c )), c Î ê , 3 ú
ë2 û Let f be the angle made by Y -axis.
Y
æ 3p ö
\ tan q = tan ç - f ÷ = cot f
è 2 ø
Þ cot f = k 2 Þ f = cot -1 k 2
æ 1 ö
Þ f = sin -1 ç ÷
ç 1 + k4 ÷
P è ø
X′ X
O Y

Y′
φ
æ1ö
g(3 ) - g ç ÷
è2ø π−φ
By LMVT, g ¢ (c ) =
1
3-
2
10 5 θ
- X′ X
1
\ 1- 2 = 3 2 O
c 1
3- Y′
2
3 38. As, f ( x ) is continuous in [0, 1] and differentiable in (0, 1) and
Þ c =
2
f ( 0 ) = f (1 ) = 1.
2
3 \By Rolle’s theorem, there must exist at least one x = c Î( 0, 1 )
Þ c= such that f ¢(c ) = 0
2
\ f ¢(c ) vanishes for some c Î( 0, 1 ).
3 1 5
\ g (c ) = + = 1
Now, f ( 0 ) = 1, f (1 / 2 ) = - and f (1 ) = 1
2 3 6 2
2 \By intermediate value theorem, f ( x ) must have one root
æ 3 5 ö æ 1ö æ1 ö
Thus, P ºç , ÷ belongs to ç 0, ÷ and other in the interval ç , 1 ÷ .
è 2 6ø è 2ø è2 ø

dx 2( - cosec 2 t ) 39. Here, f ( 0) = f (1) and f is continuous in [0, 1] and derivable in


35. Here, = (0, 1).
dt cot t
dx \ f ¢(c1 ) = 0 for at least one c1 Î( 0, 1 )
Þ p= - 4 Similarly, as f (1 ) = f (2 )
dt at t = 4
\ f ¢(c 2 ) = 0 for at least one c 2 Î(1, 2 ). Þ f ¢(c1 ) = f ¢(c 2 )
dy
and = sec 2 t - cosec 2 t Þ f ¢¢(c ) = 0 for at least one c Î(c1, c 2 ).
dt 1
dy 40. Let f ( x ) = - 3 x + sin x
Þ = 0 (x + 1)3
dt p
at t =
4 Domain of f is ( -¥, - 1 ) È ( - 1, ¥ ).
dy é 1 ù
\ =0 f ¢( x ) = - 3 ê + 1 ú + cos x Þ f ¢( x ) < 0
dx ë ( x + 1 ) 4
û
Hence, tangent is parallel to X -axis and its normal is parallel Þ f is decreasing.
to Y -axis.
1 Also, lim f ( x ) ® ¥, lim f ( x ) ® - ¥
36. Here, f ¢( x ) = 2/3 x ® -1+ x ® -1-
3x
Þ f ¢( 0 ) ® ¥ and tangent is vertical at x = 0. and lim f ( x ) ® - ¥, lim f ( x ) ® ¥
x®¥ x®- ¥
450 Textbook of Differential Calculus

Þ f ( x ) = 0 has exactly two roots. Also, A+B+C =p


Y So, dA + dB + dC = 0 …(ii)
From Eqs. (i) and (ii), we get
da db dc
+ + =0
X
cos A cos B cos C
X′
–1 O f (x ) - f (0)
45. = f ¢(c ) £ 1 - x for some c Î( 0, 1 ).
x-0
Y′ Þ f ( x ) £ x (1 - x ) £ 1 / 4
41. Clearly, f ( - 1) = - f (1) and f ( 0) = 0. For (a) and (b) apply ln x
46. Here, f ( x ) = …(i)
LMVT for the function f ( x ) in ( - 1, 0 ) and ( 0, 1 ), respectively. x
For (d) apply LMVT for ( f ( x ))n in ( 0, 1 ). \ Domain is R + .
1
42. When x = 1, y = - 1 [from the line] x × - ln x
x 1 - ln x
Thus, it must lie on the parabola y = x + px + q 2 Q f ¢( x ) = 2
=
x x2
Þ -1 =1 + p + q Þ p + q = -2 (a) For horizontal tangent, f ¢( x ) = 0
\ Now, distance of the vertex of the parabola from the X -axis Þ ln x = 1 Þ x = e [true]
is
2
(b) If f ( x ) intersects the X -axis.
æ pö p p2 p2
d = f ç- ÷ = - + q =q - ln x
= 0 Þ x =1
è 2ø 4 2 4 [true]
x
Y (c) f ¢( x ) is positive, if x Î( 0, e ) and f ¢( x ) is negative, if
y = 2x – 3
1 x Î (e, ¥ )
\ f ( x ) is not monotonic.
\ f ( x ) is many-one.
X′ X
–1 O 1 2 (d) For vertical tangent, f ¢( x ) = ¥ [true]
1 - ln x x2
–1 Þ = ¥ Þ =0
x2 1 - ln x
Y′ Þ x = 0 which is not in the domain of f ( x ). [false]
Substituting q = - 2 - p, we get 47. Let the tangent line be y = ax + b
p2 The equation for its intersection with the upper parabola is
d = -2 - p -
4 x 2 + 1 = ax + b
p2 Þ x 2 - ax + (1 - b ) = 0
Now, take g( p ) = -2 - p -
4
p This has a double root when a 2 - 4 (1 - b ) = 0
So, g ¢( p ) = - 1 - = 0 Þ p = - 2
2 or a 2 + 4b = 4
Hence, q=0
For the lower parabola, ax + b = - x 2
Note that least distance of the vertex from X -axis is 1.
Þ x 2 + ax + b = 0
43. Given, xy = a + x Þ xy = a 2 + x 2 + 2ax
a2 This has a double root when a 2 - 4b = 0
Þ y = + x + 2a
x 1
On subtracting these two equations, we get 8b = 4 or b =
dy a2 2
Þ = - 2 + 1 = -1
dx x On adding these equations, we get 2a 2 = 4 or a = ± 2
Þ 2x 2 = a 2 The tangent lines are y = 2 x +
1
and y = - 2 x +
1
a 2 2
Þ x=± f (1 ) - f ( 0 )
2 48. For some a Î( 0, 1), | f ¢( a )| = £ | f (1 )| + | f ( 0 )|
a b c 1-0
44. Given, = = = 2R [say]
sin A sin B sin C Þ | f ¢( a )| £ 1 + 1 = 2
\ da = 2 R cos A dA, db = 2 R cos B dB, Similarly, for some b Î ( -1, 0 ), | f ¢(b )| £ 2
dc = 2 R cos C dC Also, F ( x ) = ( f ( x )) 2 + ( f ¢( x )) 2
da db dc Þ F ( a ) = ( f ( a )) 2 + ( f ¢( a )) 2 £ 1 + 4 £ 5
\ + + = 2 R (dA + dB + dC ) …(i)
cos A cos B cos C Similarly, F (b ) £ 5 for some b Î ( -1, 0 )
Chap 07 dy/dx as a Rate Measurer and Tangents, Normals 451

As, F ( 0 ) = 6, so there must be a point of local maxima for F ( x ) 53. Let A(a, P (a )), B(b, P (b )), then slope of AB = P ¢(a ) = P ¢(b ) from
in ( -1, 1 ) and at the point of maxima, say x = c, LMVT there exists c Î(a, b ), where P ¢(c ) = slope of AB.
F (c ) ³ 6 Þ F ¢(c ) = 0 and F ¢¢ (c ) £ 0 ì 1 1
49. As, Rolle’s theorem is applicable, the function should be ïï 2 - x, x < 2
continuous and differentiable in [ -3, 3 ]. 54. f (x ) = í 2
ï æç - x ö÷ , x ³
1 1
So, at x = 1 it is continuous ïî è 2 ø 2
Þ lim f ( x ) = lim f ( x ) = f (1 ) ì 1
x ®1- x ®1+ - 1, x <
ï 2
Þ a +b =c =1 ...(i) Þ f ¢( x ) = í
æ1 ö 1
Since, differentiable at x = 1, therefore ï2 ç - x ÷ ( - 1 ), x >
î è 2 ø 2
f ¢(1 + ) = f ¢(1 - )
Left hand derivative at x = 1 / 2 is ( - 1 ) and right hand
c derivative at x = 1 / 2 is 0, so the function is not differentiable
-1
Þ lim 1 + h = 2a at x = 1 / 2.
h®0 h 2x + 2
c -1 -h 55. f ( x ) = ln (2 + x ) - is continuous in [ - 2, ¥ ).
Þ 2a = lim exists only when c = 1 x+3
h ® 0 h(h + 1 )
1 4 (x + 3)2 - 4 (x + 2)
-1 f ¢( x ) = - =
Þ 2a = lim = -1 x + 2 (x + 3) 2
(x + 2) (x + 3)2
h ® 0 (1 + h )
x + 2x + 1
2
(x + 1)2
\ a = -1 / 2 and c = 1 ...(ii) = = >0
(x + 2) (x + 3)2 (x + 2) (x + 3)2
From Eqs. (i) and (ii), we get
[ f ¢( x ) = 0 at x = - 1]
b = 3 /2 ...(iii)
Þ f is increasing in ( - 2, ¥ ).
\ a +b +c =2 ...(iv)
Also, lim f ( x ) ® - ¥
|a | + |b | + |c | = 3 ...(v) x ® - 2+
2a + 4b + 3c = 8 ...(vi) and lim f ( x ) ® ¥ Þ unique root.
4a 2 + 4b 2 + 5c 2 = 15 ...(vii) x®¥
56. Let f ( x ) = 0 has two roots say x = r1 and x = r2 , where
50. Statement I As, f ( - 1) = f (1) and Rolle’s theorem is not r1, r2 Î [a, b ].
applicable, then it implies that f ( x ) is either discontinuous or Þ f (r1 ) = f (r2 )
f ¢( x ) does not exist at at least one point in ( - 1, 1 ).
Hence, there must exist some c Î(r1, r2 ), where f ¢(c ) = 0
Þ g( x ) = 0 for at at least one value of x in ( -1, 1 ).
But f ¢( x ) = x 6 - x 5 + x 4 - x 3 + x 2 - x + 1
Statement II is false. Consider the example in Statement I.
for x ³ 1, f ¢( x ) = ( x 6 - x 5 ) + ( x 4 - x 3 ) + ( x 2 - x ) + 1 > 0
51. Common normal is y = x
Solving, y = x with x + y = 2 , we get A(1, 1 ) for x £ 1, f ¢( x ) = (1 - x ) + ( x 2 - x 3 ) + ( x 4 - x 5 ) + x 6 > 0
and with x 2 + y 2 = 16, we get B(2 2, 2 2 ) Hence, f ¢( x ) > 0 for all x.
Y \ Rolle’s theorem fails.
Þ f ( x ) = 0 cannot have two or more roots.
B
57. Let f ( x ) = cos x - sin x + cos-1 x - sin -1 x, x Î [ - 1, 1]
A
(1, 1)
Q f ( - 1 ) f (1 ) < 0
X′
\There exists at least one c Î ( - 1, 1 ) such that f (c ) = 0.
X
Hence, the curves y = cos x + cos-1 x and y = sin x + sin -1 x
intersect each other at at least one point.
2
æ 1ö 3
58. Given, sin x = x 2 + x + 1 = ç x + ÷ +
è 2ø 4
Y′
The shortest distance between the given curves is AB = 4 - 2. Y
But as the curves are not smooth, check at slope points. The   y = x2 + x + 1
coordinates in 1st quadrant are (2, 0) and (4, 0) and here –1 , 3
distance = 2.  2 4 (0, 1) Inconsistent
\ 4 - 2 is not the shortest.
X′ X
52. If P ( x ) = 0 is a polynomial equation, then P ( - x ) = 0 has no (0, 0)
positive root. y = sin x
Þ P ( x ) = 0 cannot have negative roots. Y′
452 Textbook of Differential Calculus

59. Y Y 64. An ellipse and hyperbola intersect orthogonally.


y=x
y = sin x They must be confocal Þ a = 2
y = cos x
Let point P( a, b ) lies on both the curves, then
Inconsistent X′
X′ X Consistent 4 a 2 + 9b 2 = 36 ...(i)
π/2 X O π/2
and 4a - b = 4
2 2
...(ii)
Y′ Y′ On adding Eqs. (i) and (ii), we get
(a) (b) 8 a 2 + 8b 2 = 40
Y y = tan x Þ a2 + b2 = 5
y=x
c
Inconsistent or x 2 + y 2 = 7 , as c 7 = 70.
X′ X 14
3
65. (A) f ¢( x ) = log x - + 1
Y′ x
(c)
Þ f ( x ) = ( x - 3 ) log x + c
Sol. (Q. Nos. 60 to 62) Þ f (1 ) = f (3 )
Slope of tangent at P = Slope of OP (B) f ¢( x ) = 4ax 3 + 3bx 2 + 2cx + d
1 ln mt
Þ = Þ f ( x ) = ax 4 + bx 3 + cx 2 + dx + e
t t
e Þ f ( 0 ) = f (3 ) [Q 27a + 9b + 3c + d = 0 ]
Þ t=
m f (b ) - f (a ) 2
(C) = f ¢( 3 ) =
Y b -a 3
px ab - 1 2
y= Þ =
ab 3
P(t, ln mt) f (b ) - f (a ) æ1ö
(D) = f ¢ç ÷ =1
b -a è2ø
Þ a + b =1
66. (A) Here, r = 5 cm, Dr = 0.06
α A = p r 2,
X Q
O 1/m
\ dA = 2pr dr = 2 pr × Dr = 10 p ´ 0.06 = 0.6 p
æe ö (B) v = x 3, dv = 3 x 2 dx
Þ P º ç , 1÷
èm ø
dv dx
m ´ 100 = 3 ´ 100 = 3 ´ 1 = 3
Þ tan a = p = v x
e
dx dx
60. p > m / e 61. p = m / e (C) ( x - 2 ) =2 Þ x=4
dt dt
62. 0 < p < m / e 3 2
(D) A = x
63. Putting y = 0, we get x 2 - 3x - ci = 0 4
As roots are integers and D = 9 + 4ci must be perfect square, dA
=
3
×x×
dx
therefore 9 + 4ci = (2 l + 1 ) 2, l Î I . dt 2 dt
Þ ci = l2 + l - 2 =
3 1 3 3
× 15 × = cm 2 / s
Þ ck = k(k + 3 ), k = 1, 2, 3,... 2 10 4
\ c1 = 4, c 2 = 10,...
dx dy dy
Þ x 2 + y 2 - 3 x - 4y - (c 2 - c1 ) = 0 67. Q < Þ >1
dt dt dx
or x 2 + y 2 - 3 x - 4y - 6 = 0 dy dy 9
and 3y 2 × = 27 or =
and x 2 + y 2 = r 2 will touch each other, if dx dx y 2
9 / 4 + 4 = | r - 9 / 4 + 10| \
9
>1 Þ y2 <9
Þ |r - 7 /2| = 5 /2 y2
Þ r =6 Þ - 3 < y < 3 Þ - 27 < y 3 < 27
\ Common tangent is 3 x + 4y - 30 = 0 and passes through Þ - 27 < 27 x < 27 Þ - 1 < x < 1
(10, 0). \ x Î ( - 1, 1 ) Þ a + b = 0
Chap 07 dy/dx as a Rate Measurer and Tangents, Normals 453

dy 72. Point A(a , y1 ) lies on C1 and C 2.


68. Here, 4y = 2ax
dx \ y1 = a 2 - 3 and y1 = ka 2.
dy -a
\ = = -1 Þ a 2 - 3 = ka 2 …(i)
dx (1, -1) 2
Y
Þ a =2 y= x2 – 3
Also, 2y 2 = ax 2 + b at (1, - 1 ) is B
(1,y2 )
2 =a + b
Þ b=0 O
X′ X
\ a -b =2 A (a,, y1 )
69. Using LMVT, for some c Î(1, 6) , we get (0,–3) y= kx2
f (6 ) - f (1 )
f ¢(c ) =
5 Y′
f (6 ) + 2
= ³ 4.2 Now, y = kx 2
5
dy
Þ f (6 ) ³ 19 Þ = 2kx
Þ f (6 ) - 16 ³ 3 dx
\ Least value of f (6 ) - 16 = 3 æ dy ö y - y1
\ ç ÷ = 2ka = 2
è dx ø (a , y ) 1 -a
70. Let y = mx + c be a tangent to f ( x ). 1

Y But, y2 = 1 - 3 = - 2
- 2 - (a 2 - 3 )
\ 2ka =
1 -a
X′ X 1 - a2
O Þ 2ka = =1 + a
1 -a
Þ 2ak = 1 + a …(ii)
a2 - 3
Y′ Substituting k = from Eq. (i) in Eq. (ii), we get
a2
For, x ³ 0, intersection point is given by
2a(a 2 - 3 )
mx + c = x 2 + 8 [Q y = x 2 + 8, for x ³ 0 ] =1 + a
a2
Þ x 2 - mx + (8 - c ) = 0 Þ 2a 2 - 6 = a + a 2
For line to be tangent, D = 0
Þ a2 - a - 6 = 0
\ m 2 = 4(8 - c ) …(i)
Þ a = 3, -2
Again, for x < 0
\ a =3 [Q -2 is rejected as a > 0]
mx + c = - x 2 [Q y = - x 2, for x < 0 ] f (6 ) - f ( - 6 )
73. f ¢(c ) = 16c - 7 =
Þ x + mx + c = 0
2
12
D=0 (8 × 36 - 7 × 6 + 5 ) - (8 × 36 + 7 × 6 + 5 )
Now, =
Þ m 2 = 4c …(ii) 12
2 ×7 ×6
From Eqs. (i) and (ii), we get =- = -7
12
c = 4, m = 4
Þ 16 c = 0 Þ c = 0
\Tangent is y = 4 x + 4
74. Using LMVT for f in [1, 2], we get, for some c Î(1, 2)
Putting y = 0, we get
x = -1 f (2 ) - f (1 )
\Absolute value of x-intercept is 1. = f ¢(c ) £ 2
2 -1
71. Here, f ¢(a ) = 3 and f ¢(b ) = 1
f (2 ) - f (1 ) £ 2
b
b æ ( f ¢( x )) 2 ö 1 Þ f (2 ) £ 4 …(i)
\ òa f ¢(x ) × f ¢¢(x ) dx = çè 2 ÷ø = 2 ( f ¢(b )) - ( f ¢(a ))
2 2
Again, using LMVT in [2, 4], we get, for some d Î(2, 4 )
a
f ( 4 ) - f (2 )
1 Þ = f ¢(d ) £ 2
= 1 -3 =1 4 -2
2
454 Textbook of Differential Calculus

\ f ( 4 ) - f (2 ) £ 4, 8 - f (2 ) £ 4, 4 £ f (2 ) 77. Let ( x1, y1 ) be a point of contact of tangents from the origin (0, 0)
Þ f (2 ) ³ 4 …(ii) to the curve y = sin x.
From Eqs. (i) and (ii), we get f (2 ) = 4 Here, y = sin x
dy
75. As, a, b and c are positive integers. \ = cos x
dx
1 é 1 1 ù
We must have < 1, so a > 1. êësimilarly, b < 1, c < 1 úû æ dy ö
a Þ ç ÷ = cos x1
è dx ø ( x
1 , y1 )
1 1 1
Since, > > Now, equation of tangent at ( x1, y1 ) is
a b c
æ dy ö
Þ
1 1
> Þ a < 3 Þa = 2 y – y1 = ç ÷ ( x – x1 )
è dx ø ( x , y )
a 3 1 1

1 1 1 1 1 1 Þ y – y1 = (cos x1 ) ( x – x1 )
\ + + = 1 Þ + = , where 2 < b < c.
a b c b c 2 Q It passes through (0, 0).
Similarly,
1 1
> , so b < 4 Þ b = 3 \ –y1 = (cos x1 ) (– x1 ) ...(i)
b 4 Also, ( x1, y1 ) lies on the curve.
1 1 1 So, y1 = sin x1 ...(ii)
Now, + + =1
a b c Squaring and adding Eqs. (i) and (ii), we get
1 1 1 2
Þ + + =1 æ y1 ö
ç ÷ + y1 = cos x1 + sin x1 = 1
2 2 2
2 3 c
è x1 ø
Þ c =6
\ (a + b + c - 5 ) = 2 + 3 + 6 - 5 = 6 or y12 + x12y12 = x12
x2 y 2 i.e. ( x12 + 1 ) y12 = x12
76. Equation of ellipse is + =1
a2 b2 x2
\ The point of contact ( x1, y1 ) lies on the curve y 2 = .
Equation of tangent at (a cos q, b sin q) is x +1
2

x y
cos q + sin q = 1 78. We must show that for a given m Î R these exists x ÎR such
a b
that,
Intercept on the X-axis = (a sec q) x
Intercept on the Y-axis = (b cosec q) m 2x 2 + ò 0 f (t ) dt = 2
Length of intercept of the tangent by the axes x
Let f ( x ) = ò [2 m 2t + f (t )] dt – 2, x ÎR
= a sec q + b cosec q
2 2 2 2 0

Since, f ( x ) is continuous and 2 m 2t 2 is continuous, therefore


Let l = a 2 sec 2q + b 2 cosec 2q
x
ò 0 [2m t +
2
dl f (t )] dt continuous on R,
Þ = 2a 2 sec 2q tan q – 2b 2 cosec 2q cot q
dq
\ f is continuous on R, also
dl
Now, =0 0
dq f ( 0 ) = ò [2 m 2t + f (t )] dt – 2 = 0 – 2 = – 2
0
a
Þ a 2 sin 4 q = b 2 cos4 q Þ = cot 2q and
x
f ( x ) = ò [2 m 2t + f (t )] dt – 2
b 0
b a x
Þ sin q = , cos q =
ò 0 f (t ) dt – 2 ® ¥
2 2
where, f ( x ) = m 2x 2 +
a+b a+b
Distance between (a sec q , 0 ) and point of tangency éQ x f (t ) dt ® ¥ ù
(a cos q , b sin q) is êë ò 0 úû
As, |x|® ¥
= a (sec q – cos q) + b sin q
2 2 2 2 Thus, these exists some a ÎR such that;
f ( x ) > 1, for | x | > a
= a 2 cos2 q (sec 2 q – 1 ) 2 + b 2 sin 2 q
Note that f is a continuous on [ 0, a + 1 ] and f ( 0 ) f (a + 1 ) < 0.
By the intermediate value theorem of continuous functions, we
2
æa + b ö have that there exists some b Î( 0, a + 1 ) such that f (b ) = 0, i.e.
= a 2 cos2 q ç – 1 ÷ + b 2 sin 2 q there exists a real b which satisfies the equation
è a ø
x

b2
m 2x 2 + ò 0 f (t ) dt = 2
= a cos q 2 + b 2 sin 2 q =b
2 2
a 79. Tangent at any point P (t1 ), i.e. ( 4 t12 + 3, 8 t13 – 1) be normal to
Similarly, distance between ( 0, b cosec q) = a the curve at Q (t 2 ), i.e. ( 4 t 22 + 3, 8 t 23 – 1 ).
Chap 07 dy/dx as a Rate Measurer and Tangents, Normals 455

The equation of the tangent at t1 is On integrating both the sides, we get


æ dy ö æ 1– 1 – y 2 ö
y – (8 t13 – 1 ) = ç ÷ × { x – ( 4 t12 + 3 )}
è dx ø t Þ log y m ç log + 1 – y 2 ÷ =x +c
ç y ÷
1
è ø
æ dy/dt ö
or y – (8 t13 – 1) = ç ÷ × { x – ( 4 t1 + 3 )}
2
The curve passes through (1, 1 ), so c = - 1
è dx/dt ø t
1
Hence, the possible curves
24 t12 æ ö
or y – (8 t13 – 1 ) = × { x – ( 4 t12 + 3 )} 1– 1 – y 2
8 t1 log y – x = ± ç log + 1 – y 2 ÷ -1
ç y ÷
è ø
or y – (8 t13 – 1 ) = 3t1 { x – ( 4 t12 + 3 )} ...(i)
Clearly, slope of tangent at t1 = slope of tangent at t 2. 81. Let the point P ( x, y ) be on the curve,
æ dy ö –1 –1 æy ö
\ ç ÷ = i . e. 3t1 = ...(ii) log ( x 2 + y 2 ) = c tan –1 ç ÷
è dx ø t æ dy ö 3t2 èxø
1 ç ÷
è dx ø t Differentiating both the sides w.r.t. ‘ x’, we get
2
2 x + 2yy ¢ c ( xy ¢ – y )
Þ Equation of normal at t 2 is y - (8t 23 - 1 ) = 3t1 { x - ( 4t 23 + 3 )} = 2
(x 2 + y 2 ) (x + y 2 )
…(iii)
On subtracting Eq. (iii) from Eq. (i), we get 2 x + cy
Þ y¢ = = m1 (say)
(8 t 23 – 1 ) – (8 t13 – 1 ) = 3 t1 {( 4 t 22 + 3 ) – ( 4 t12 + 3 )} cx – 2y
y
Þ 2 t 22 = t 1 t 2 + t12 Slope of OP = = m2 (say)
x
2
æ –1 ö 1 Let the angle between the tangent at P and OP be q.
Þ 2 × ç ÷ = – + t12 [using Eq. (ii)]
è 9 t1 ø 9 2 x + cy y

Þ 2 = – 9 t12 + 81 t14 m1 – m2 cx – 2y x 2
Then, tan q = = =
\ 81 t14 – 9 t12 – 2 = 0 1 + m1 m2 2 xy + cy 2
c
1+
x (cx – 2y )
2
Þ t1 = ±
3 æ2ö
\ q = tan –1 ç ÷ = constant.
Putting in Eq. (i), the equation is èc ø
27 (y + 1 ) m 16 2 = ± 2 (27 x – 105 ) Hence, the angle between the tangent at any point P and the
line joining P to the origin O is the same.
80. Equation of tangent at ( x, y ) is
82. (i) The given curves are
dx y 2 = 4 ax
Y –y = (X – x ) ...(i)
dy
and x 2 = 4 ay ...(ii)
æ dx ö
\ Q = çx – y , 0÷ Point of intersection of Eqs. (i) and (ii) are ( 0, 0 ) and ( 4 a, 4 a ).
è dy ø dy 2a
From Eq. (i), = = m1 (say)
Equation of normal at ( x, y ) is dx y
dx dy x
Y –y =– (X – x ) From Eq. (ii), = = m2 (say)
dy dx 2a
æ dy ö Let the angle of intersection of two curves is q, then
\ R = çx + y , 0÷
è dx ø 2a x

QR = 2 m – m2 y 2a 4 a 2 – xy
Given, tan q = 1 = =
dy dx 1 + m1 m2 1+
x 2a (x + y )
Þ y +y =2 y
dx dy
2 \ (tan q) ( 0,0) = ¥ or q = 90°
æ dy ö æ dy ö
Þ y ç ÷ –2 ç ÷ + y = 0 1
è dx ø è dx ø –2
3 3
and (tan q) ( 4a , 4a ) = 2 =– =
dy 2 ± 4 – 4y 1 ± 1 – y 1+1
2 2
4 4
Þ = =
dx 2y y
æ3ö
y dy 1 m 1–y2 Hence, q = tan –1 ç ÷
Þ = dx or dy = dx è 4ø
1 ± 1–y2 y
456 Textbook of Differential Calculus

(ii) The given curves are y 2 = 4 ax ...(i) 84. Let the curve be y = f ( x ) and tangent drawn at P ( x, y ) meets
and x = 4 ay
2
...(ii) the X-axis at T .
We have, OT = x 2
Tangents of Eq. (i) in terms of slope m is
a Equation of tangent at P ( x, y );
y = mx + ...(iii)
m Y – y = f ¢( x ) ( X – x )
Now, Eq. (iii) is also tangent of Eq. (ii). æ y ö
Þ T ºç x – , 0÷
Eliminating y from Eqs. (ii) and (iii), we have è f ¢( x ) ø
æ aö
x 2 = 4 a çmx + ÷ Þ
y
=x2
è mø x–
f ¢( x )
4a 2 f (x )
Þ x 2 – 4 amx – =0 ...(iv) Þ x– = ± x2
m f ¢( x )
Þ B 2 – 4 AC = 0 x f ¢ ( x )– f ( x )
Þ = ± f ¢( x )
æ 4a 2 ö x2
Þ 16 a 2m 2 – 4 ç– ÷ =0
è m ø d æ f (x ) ö
Þ ç ÷ = ± f ¢( x )
Þ m 3 = – 1 or m = – 1 dx è x ø
f (x )
From Eq. (iii) common tangent is On integrating both the sides, we get = ± f (x ) + c
y = – x – a or y + x + a = 0 x
Hence, the common tangent is x + y + a = 0. Since, the curve passes through (2, 1).
1
83. Let the coordinate system be chosen such that the given \ =±1+c
straight line is x = p and the equations of the circles are 2
x 2 + y 2 = a 2, x 2 + y 2 = b 2 , x 2 + y 2 = c 2 . 1 3
Þ c =– ,
The line x = p cuts these circles at A, B and C, respectively. 2 2
x 3x
The coordinates of these points are A( p, a 2 – p 2 ), Þ f (x ) = or f ( x ) =
2 (x – 1) 2 (1 + x )
B ( p, b 2 – p 2 ) and C ( p, c 2 – p 2 ).
x 3x
Equations of the tangents at these points are Hence, possible curves are y = and y = .
2 (x – 1) 2 (1 + x )
px + a 2 – p 2y = a 2, px + b 2 – p 2 y = b 2 dy
85. For y = x – x 3, = 1 – 3x 2
and px + c 2 – p 2 y = c 2. dx
These tangents intersect at Therefore, the equation of the tangent at the point P ( x1, y1 ) is
y – y1 = (1 – 3 x12 ) ( x – x1 )
é p2 – a2 – p2 b2 – p2 ù
ê , a 2 – p 2 + b 2 – p 2 ú, It meets the curve again at Q ( x 2, y 2 ).
êë p úû Hence, x 2 – x 23 – ( x1 – x13 ) = (1 – 3 x12 ) ( x 2 – x1 )
é p2 – b2 – p2 c2 – p2 ù Þ ( x 2 – x1 ) [1 – ( x 22 + x1 x 2 + x12 )] = ( x 2 – x1 ) (1 – 3 x12 )
ê , b 2 – p 2 + c 2 – p 2 ú,
êë p úû Þ 1 – x 22 – x1 x 2 – x12 = 1 – 3 x12
é p2 – c2 – p2 c2 – p2 ù Þ x 22 + x1x 2 – 2 x12 = 0
ê , c 2 – p 2 + a 2 – p 2 ú. 2
êë p úû æ x1 ö 9 x12 x 3x
Þ ç x2 + ÷ = Þ x2 + 1 = ± 1
Area of D formed by the tangents at A, B, C is è 2ø 4 2 2

p2 – a2 – p2 b2 – p2 Since, x1 ¹ x 2, we have x 2 = – 2 x1
a2 – p2 + b2 – p2 1 Þ Q is (–2 x1 , – 2 x1 + 8 x13 ).
p
2 2
1 p – b –p
2
c2 – p2 If L1 ( a, b ) is the point of trisection of PQ, then
D= b2 – p2 + c2 – p2 1 2 x – 2 x1
2 p a= 1 = 0. Hence, L1 lies on the Y -axis. If L2 (h, k ) is the
3
p2 – c2 – p2 c2 – p2 x – 4 x1
c2 – p2 + a2 – p2 1 other point of trisection, then h = 1 = – x1 and
p 3
y – 4 x1 + 16 x13
( a 2 – p 2 – c 2 – p 2 )( c 2 – p 2 – b 2 – p 2 ) k= 1
3
( b2 – p2 – a2 – p2 ) x1 – x13 – 4 x1 + 16 x13
= i.e. k= = – x1 + 5 x13
2p 3
CA × BC × AB Þ k = h – 5h 3
=
2p \ Locus of (h, k ) is y = x – 5 x 3.
Chap 07 dy/dx as a Rate Measurer and Tangents, Normals 457

86. Let the curve be y = f ( x ) æ dy ö


88. Slope of tangent at the point ( x1, y1 ) is given by ç ÷ .
The equation of the tangent at any point ( x, y ) is è dx ø ( x
1 , y1 )

Y –y =
dy
(X – x ) Given curve, (y - x 5 ) 2 = x (1 + x 2 ) 2
dx
æ dy ö
Its intercept on the y-axis is given by ( X = 0 ) Þ 2 (y - x 5 ) ç - 5 x 4 ÷ = (1 + x 2 ) 2 + 2 x (1 + x 2 ) × 2 x
è dx ø
dy
Y =y – x = k x2 + y 2 Put x = 1 and y = 3, then
dx
dy / dx = 8
dy
So, x – y + k x 2 + y 2 = 0 is the differential equation 89. Given, f ( x ) = 2 + cos x,"x Î R
dx
governing the curve. This can be written as Statement I There exists a point Î [t, t + p ], where f ¢(c ) = 0
dy y – k x + y
2 2
Hence, Statement I is true.
=
dx x Statement II f (t ) = f (t + 2p ) is true. But Statement II is not
dy dv correct explanation for Statement I.
Let y = vx, so that =v + x
dx dx 90. As, | f ( x1 ) - f ( x 2 )| £ ( x1 - x 2 ) 2, "x1, x 2 Î R
The differential equation becomes Þ | f ( x1 ) - f ( x 2 )| £ | x1 - x 2| 2 [as x 2 = | x | 2 ]
dv
v+x = v – k 1 + v2 f ( x1 ) - f ( x 2 )
dx \ £ | x1 - x 2|
dv dx x1 - x 2
Þ +k =0
1+v 2 x ½ f ( x1 ) - f ( x 2 )½
Þ lim ½ ½£ lim | x1 - x 2|
Þ log | v + 1 + v | + k log | x | = c (on integrating)
2 x1 ® x 2 ½ x1 - x 2 ½ x1 ® x 2

y y2 Þ | f ¢( x1 )| £ 0, "x1 Î R
Þ log + 1 + 2 + k log | x | = c \ | f ¢( x )| £ 0, which shows | f ¢( x )| = 0
x x
[as modulus is non-negative or | f ¢( x )| ³ 0 ]
Þ log | y + x 2 + y 2 | + (k – 1 ) log | x | = c \ f ¢( x ) = 0 or f ( x ) is constant function.
Þ (y + x 2 + y 2 ) xk – 1 = c1 Þ Equation of tangent at (1, 2) is
y -2
87. We have, 2 t 3 – 9 t 2 + 30 – a = 0 = f ¢( x )
x -1
Any real root t 0 of this equation gives two real and distinct
or y -2 = 0 [Q f ¢( x ) = 0 ]
values of x if | t 0| > 2.
\ y -2 = 0 is required equation of tangent.
Thus, we need to find the condition for the equation in t to
have three real and distinct roots none of which lies in [–2, 2 ]. 91. Given, y 3 + 3 x 2 = 12y ...(i)
On differentiating w.r.t. x, we get
dy dy
3y 2 + 6 x = 12
–2 0 2 3 dx dx
dy 6x
Þ =
Let f (t ) = 2 t 3 – 9 t 2 + 30 – a dx 12 - 3y 2
Þ f ¢(t ) = 6 t 2 – 8 t = 0 Þ t = 0, 3 dx 12 - 3y 2
Þ =
So, the equation f (t ) = 0 has three real and distinct roots, if dy 6x
f ( 0 ) × f (3 ) < 0 For vertical tangent,
Þ (30 – a ) (54 – 81 + 30 – a ) < 0 dx
Þ (30 – a ) (3 – a ) < 0 =0
dy
Þ (a – 3 ) (a – 30 ) < 0, using number line rule i.e.,
Þ 12 - 3y 2 = 0
Þ a Î(3, 30 ) ...(i)
+ – + Þ y =±2
4
3 30 On putting, y = 2 in Eq. (i), we get x = ± and again putting
3
Also, none of the roots lie in [–2,2 ]
y = - 2 in Eq. (i), we get 3 x 2 = - 16, no real solution.
if f (–2 ) > 0 and f (2 ) > 0
æ 4 ö
Þ (–16 – 36 + 30 – a ) > 0 and (16 – 36 + 30 – a ) > 0 So, the required point is ç ± , 2÷ .
è 3 ø
Þ a + 22 < 0 and a – 10 < 0
92. Slope of tangent to the curve, y = f ( x ) is
Þ a <– 22 and a < 10 dy
Þ a < – 22 ...(ii) = f ¢( x ) ( 3, 4 )
dx ( 3, 4 )
From Eqs. (i) and (ii) no real value of a exists.
458 Textbook of Differential Calculus

Therefore, slope of normal = -


1
=-
1 Þ x 2 + 4 x - 2 x 2 - 3( 4 + x 2 - 4 x ) = 0
f ¢( x ) ( 3, 4 ) f ¢(3 )
Þ - x 2 + 4 x - 12 - 3 x 2 + 12 x = 0
1 æ 3p ö
But - = tan ç ÷ [given] Þ - 4 x 2 + 16 x - 12 = 0
f ¢(3 ) è 4 ø
Þ 4 x 2 - 16 x + 12 = 0
-1 æp pö
Þ = tan ç + ÷ = - 1
f ¢(3 ) è2 4ø Þ x 2 - 4x + 3 = 0
f ¢(3 ) = 1 Þ ( x - 1 )( x - 3 ) = 0
x+6 \ x = 1, 3
93. We have, y =
(x - 2) (x - 3) Now, when x = 1, then y = 1
Point of intersection with Y -axis (0, 1) and when x = 3, then y = - 1.
( x 2 - 5 x + 6 ) (1 ) - ( x + 6 ) (2 x - 5 ) \ P = (1, 1 ) and Q = (3, - 1 )
y¢ = Hence, normal meets the curve again at (3, –1) in fourth
(x 2 - 5x + 6)2
quadrant.
Þ y ¢ = 1 at point (0, 1). Aliter
\Slope of normal is - 1.
Given, x 2 + 2 xy - 3y 2 = 0
Hence, equation of normal is x + y = 1.
æ1 1ö Þ ( x - y )( x + 3y ) = 0
\ ç , ÷ satisfy it.
è2 2ø Þ x - y = 0 or x + 3y = 0
Equation of normal at (1, 1) is
94. Given equation of curve is
x 2 + 2 xy - 3y 2 = 0 …(i) y - 1 = - 1( x - 1 )
Þ x + y -2 = 0
On differentiating w.r.t. x, we get
It intersects x + 3y = 0 at (3, –1) and hence normal meets the
x +y
2 x + 2 xy ¢ + 2y - 6yy ¢ = 0 Þ y ¢ = curve again in fourth quadrant.
3y - x
æ dy ö Y
At x = 1, y = 1, y ¢ = 1 i.e. ç ÷ =1
è dx ø (1, 1) x+y=2
y=x
x + 3y = 0
Equation of normal at (1, 1) is
1 (1,1)
y - 1 = - (x - 1) Þ y - 1 = - (x - 1)
1 X′ X
O
Þ x + y =2 …(ii)
(3,–1)
On solving Eqs. (i) and (ii) simultaneously, we get
Þ x 2 + 2 x(2 - x ) - 3 (2 - x ) 2 = 0 Y′
CHAPTER

08
Monotonicity,
Maxima and Minima
Learning Part
Session 1
● Monotonicity

Session 2
● Critical Points

Session 3
● Comparison of Functions Using Calculus

Session 4
● Introduction to Maxima and Minima

● Methods of Finding Extrema of Continuous Functions

● Convexity/Concavity and Point of Inflection

● Concept of Global Maximum/Minimum

Session 5
● Maxima and Minima of Discontinuous Functions

Practice Part
● JEE Type Examples
● Chapter Exercises

Arihant on Your Mobile !


Exercises with this #L
symbol can be practised on your mobile. See title inside to activate for free.
Session 1
Monotonicity
Monotonicity Strictly Decreasing
A function is said to be monotonic if it is either Now, we consider a function represented by the following graph.
increasing or decreasing in its domain. y=f(x) Y

Strictly Increasing
Consider a function represented by the following y1

graph.
y2
y=f(x)
Y
X
x1 x2

y2 Figure 8.2
For two different input arguments x 1 and x 2 , where
y1 x 1 < x 2 , y 1 = f ( x 1 ) will always be greater than y 2 = f ( x 2 ).
i.e. x1 < x 2 Þ f (x1 ) > f (x 2 )
x1 x2 X
Such a function is called a strictly decreasing function or a
Figure 8.1
monotonically decreasing function. e.g. consider f ( x ) = -[ x ].
For this function x 1 < x 2 does not implyf ( x 1 ) > f ( x 2 ).
i.e. x1 < x 2
However, x 1 < x 2 Þ f (x 1 ) ³ f (x 2 )
implies f (x 1 ) < f (x 2 )
In other words, f ( x ) = - [ x ] is not strictly decreasing. It would
Such a function is called a strictly increasing
only be termed decreasing. The following table list down a few
function or a monotonically increasing
Examples of functions and their behaviour in different intervals.
function.
Function Behaviour
For two different input arguments x 1 and x 2 ,
where x 1 < x 2 , y 1 = f ( x 1 ) will always be less f (x ) = x Strictly increasing on R
than y 2 = f ( x 2 ). f (x ) = x 2
Strictly decreasing on ( - ¥, 0 ]
The word ‘monotonically’ apparently has its origin Strictly increasing on [ 0, ¥ )
in the word monotonous. f (x ) = x Strictly increasing on [ 0, ¥ )

e.g. A monotonous routine is one in which one f (x ) = x 3


Strictly increasing on R
follows the same routine repeatedly or f (x ) = | x | Strictly decreasing on ( - ¥, 0 ]
continuously. Strictly increasing on [ 0, ¥ )
Similarly, a monotonically increasing function is f (x ) =
1 Neither decreasing nor increasing on R.
one that increases continuously. x Strictly decreasing on ( - ¥, 0 ), Strictly decreasing on ( 0, ¥ )
e.g. consider f ( x ) = [ x ]. f (x ) = [ x ] Increasing on R
For this function x 1 < x 2 does not always imply f (x ) = { x } Neither increasing nor decreasing on R.
f ( x 1 ) < f ( x 2 ). However, strictly increasing on [n, n + 1 ), where n ÎZ
However, x 1 < x 2 does imply f ( x 1 ) £ f ( x 2 ). f ( x ) = sin x Neither increasing nor decreasing on R.
In other words, f ( x ) = [ x ] is not strictly (or éæ 1ö æ 1ö ù
Strictly increasing on ê ç2n - ÷ p, ç2n + ÷ p ú; n ÎZ
monotonically) increases. It will nevertheless be ë è 2 ø è 2ø û
termed increasing. éæ 1ö æ 3ö ù
Strictly decreasing on ê ç2n + ÷ p, ç2n + ÷ p ú; n ÎZ
ëè 2ø è 2ø û
Chap 08 Monotonicity, Maxima and Minima 461

Function Behaviour Extending as above, we get the conditions for a strictly


f ( x ) = cos x Neither increasing nor decreasing on R. decreasing and decreasing functions
Strictly increasing on [(2n - 1 ) p, 2np ];n ÎZ Strictly decreasing : f ¢ ( x ) < 0 " x Î D
Strictly decreasing on [2np, (2n + 1 ) p ]; n Î Z
Decreasing : g ¢ (x ) £ 0 " x Î D
f ( x ) = tan x Neither increasing nor decreasing on R. Note
éæ 1ö æ 1ö ù
Strictly increasing on ê çn - ÷ p, çn + ÷ p ú; n Î Z (i) Above conditions on the derivatives to be applied, the
ëè 2ø è 2ø û function must be differentiable in the given domain.
f (x ) = e x Strictly increasing on R However, these conditions will hold good even if the
function is non differentiable, at a finite number (or
f (x ) = e -x Strictly decreasing on R
infinitely countable number) of points. e.g.
f ( x ) = ln x Strictly increasing on (0, ¥) f ( x ) = [ x ] + { x } is strictly increasing on R. However, f ( x )
is non-differentiable at all integers (a countable set).
(ii) A function must be continuous for above conditions to be
Monotonicity with applied. Consider y ={ x }. This is non-differentiable (due to
discontinuities) at all integers. At all other points, y¢ = 1 > 0.
the Help of Derivative However, we know that y = { x } is not strictly increasing.
Let us now deduce the condition(s) on the derivative of a 1
function f ( x ) which determines whether f ( x ) is Similarly, y = is non-differentiable (and non-continuous)
x
increasing/decreasing on a given interval. We are -1
at x = 0. At all other points, y ¢ = 2 < 0, so y should be
assuming that f ( x ) is everywhere differentiable. x
strictly decreasing on R \ {0} . However, it is not strictly
Y Y y=g(x) decreasing on R \ {0} although it is strictly decreasing in
y=f(x)
the separate intervals ( - ¥,0 ) and (0, ¥ ).
f′(x)>0 Therefore, we see that discontinuous functions cannot be
g′(x)>0 subjected to the derivative condition. Even though they may
be discontinuous only at finite (on infinitely countable)
X X number of points.
Strictly increasing function Increasing function

Figure 8.3 Figure 8.4 Properties


(i) If f ( x ) is strictly increasing, then f -1 ( x ) is also
In figure 8.3, the function y = f ( x ) is a strictly increasing strictly increasing. Similarly, if f ( x ) is strictly
function. Notice that the slope of the tangent drawn at any decreasing, then f -1 ( x ) is also strictly decreasing.
point on this curve is always positive. (ii) If f ( x ) and g( x ) have the same monotonicity (both
Hence, a sufficient condition for f ( x ) to be strictly increasing or decreasing) on [a, b ] , then f ( g ( x )) and
increasing on a given domain D is f ¢ ( x ) > 0 " x Î D g( f ( x )) are monotonically increasing on[a, b ].
(Later on, we will see that this is not a necessary condition (iii) If f ( x ) and g ( x ) have opposite monotonicity on [a, b ],
for a function to be strictly increasing). then f ( g ( x )) and g ( f ( x )) are strictly decreasing on [a, b ].
In figure 8.4, the function y = g( x ) is not strictly (iv) The inverse of a continuous function is continuous
increasing though it is increasing. Notice that g ¢ ( x ) > 0 or (v) If f ¢ ( x ) > 0, " x Î(a, b ) except for a finite (or an
g ¢ ( x ) = 0, "x. g ¢ ( x ) is never negative. infinitely countable) number of points, where
Hence, a sufficient condition for g( x ) to be increasing on f ¢ ( x ) = 0, f ( x ) is still strictly increasing on (a, b ). That
a given domain D is g ¢ ( x ) ³ 0, " x Î D. Now, consider is why, we said earlier that f ¢ ( x ) > 0, " x Î D is not
f ( x ) and g( x ) in given figure. necessary condition for strict increase. e.g. In a later
Example, we will consider the graph of the function
y=f(x) Y y=g(x) Y f ( x ) = x + cos x . We will see that f ¢ ( x ) = 1 - sin x is
æ p ö
not always positiveç at x = 2np + , n ÎZ , f ¢ ( x ) = 0 ÷ ;
f ′(x)<0 g′(x) ≤ 0 è 2 ø
even then, f ( x ) increases strictly because the points
X X at which f ¢ ( x ) = 0 are countable.
(vi) Similarly if f ¢ ( x ) < 0, " x Î(a, b ) except for a finite
Strictly decreasing function Decreasing function
(or an infinitely countable) number of points, where
Figure 8.5 f ¢ ( x ) = 0, f ( x ) is still strictly decreasing on (a, b ).
462 Textbook of Differential Calculus

Y
Classification of Strictly f(x)

Increasing Functions f ′(x)<0


Increasing functions can be classified as f ′′(x)>0
(i) Concave up When f ¢( x ) > 0 and f ¢¢ ( x ) > 0,
"x Î domain. O
X
Y
f(x)
Figure 8.9
f(x2) f ′ (x) >0
f ′′ (x) >0
From the figure 8.9, it is clear that the graph of the
function f ( x ) is concave up and decreasing as x increases.
f(x1) (ii) Concave down When f ¢ ( x ) < 0 and f ¢¢( x ) < 0,
O x1 x2 X "x Î domain.
Y
Figure 8.6
f(x)
From the figure 8.6, it is clear that the graph of f ( x ) is f ′ (x)<0
concave up and increasing as x increases. f ′′ (x)<0
(ii) Concave down When f ¢( x ) > 0 and f ¢¢ ( x ) < 0,
"x Î domain. O
X

Y
Figure 8.10
f(x) From the figure 8.10, it is clear that the graph of f ( x ) is
f ′ (x) > 0 concave down and f ( x ) is decreasing as x increases.
f ′′ (x) < 0
(iii) When f ¢ ( x ) < 0 and f ¢¢( x ) = 0, "x Î domain.
Y
X
O
f ′ (x)<0
Figure 8.7 f ′′ (x)=0
From the figure 8.7 it is clear that the graph of f ( x ) is f (x)
concave down and increasing as x increases. X
O
(iii) When f ¢ ( x ) > 0 and f ¢¢ ( x ) = 0, "x Î domain.
Figure 8.11
Y
f(x) From the figure 8.11, it is clear that the graph of f ( x )
f ′(x) > 0 is neither concave up nor concave down but f ( x ) is
f ′′(x) = 0 decreasing as x increases.

y Example 1 Find the interval in which


X f ( x ) = 2x 3 + 3x 2 – 12x + 1 is increasing.
O
Sol. Given, f ( x ) = 2x 3 + 3x 2 – 12x + 1
Figure 8.8
Differentiating both the sides, we have
From the figure 8.8, it is clear that the graph of f ( x ) is
f ¢( x ) = 6x 2 + 6x – 12
neither concave up nor concave down but still
increasing as x increases. Þ f ¢( x ) = 6 ( x 2 + x – 2)
Þ f ¢( x ) = 6( x + 2) ( x – 1)
Classification of Strictly Using number line rule, we have
+ – +
Decreasing Functions –∞
–2 1

Decreasing functions can be classified as follows :


Hence, f ¢( x ) ³ 0
(i) Concave up When f ¢( x ) < 0 and f ¢¢ ( x ) > 0, " x Î
when x Î (–¥, – 2] È [1, ¥ )
domain.
Þ f ( x ) is increasing when x Î (–¥, – 2] È [1, ¥)
Chap 08 Monotonicity, Maxima and Minima 463

y Example 2 Find the interval in which y Example 5 The function f ( x ) = sin 4 x + cos 4 x
f ( x ) = x 3 – 3x 2 – 9 x + 20 is strictly increasing or strictly increasing, if
decreasing. (a) 0 < x < p/ 8 (b) p/ 4 < x < 3 p/ 8
Sol. Given, f ( x ) = x – 3x – 9 x + 20
3 2
(c) 3 p/ 8 < x < 5p/ 8 (d) 5p/ 8 < x < 3 p/ 4
[IIT JEE 1999]
f ¢( x ) = 3x 2 – 6x – 9 Sol. Here, f ( x ) = sin x + cos x
4 4

Þ f ¢( x ) = 3( x 2 – 2x – 3) Þ f ¢( x ) = 4 sin 3 x × cos x + 4 cos 3 x (–sin x )


Þ f ¢( x ) = 3( x – 3) ( x + 1) f ¢( x ) = 4 sin x cos x (sin 2 x – cos 2 x )
Using number line method as shown in figure, f ¢( x ) = 2 (sin 2x ) (–cos 2x ) Þ f ¢( x ) = –sin 4 x
+ +
¢
Now, f ( x ) ³ 0, if sin 4 x £ 0 Þ p £ 4 x £ 2p

–∞ ∞
–1 3 Þ p/ 4 £ x £ p/ 2
æ p 3p ö ép pù
f ¢( x ) > 0 Here, ç , ÷ is only subset of ê , ú.
è4 8 ø ë4 2û
For x Î (–¥, – 1) È (3, ¥ ) and f ¢ ( x ) < 0, Hence, (b) is the correct answer.
For x Î(–1, 3)
x
Thus, f ( x ) is strictly increasing for x Î (–¥, – 1) È (3, ¥ ) and y Example 6 Let f ( x ) = ò e t (t – 1) (t – 2) dt. Then, f
0
strictly decreasing for x Î(–1, 3).
decreases in the interval [IIT JEE 2000]
y Example 3 Show that the function f ( x ) = x 2 is a (a) (– ¥ , – 2) (b) (–2, – 1)
strictly increasing function on (0, ¥). (c) [1, 2] (d) (2, ¥)
Sol. Given, f ( x ) = x 2 Þ f ¢( x ) = 2x x t
Sol. Here, f ( x ) = ò0 e (t – 1) (t – 2) dt
Þ f ¢( x ) > 0 for x Î (0, ¥ ) [Q x Î (0, ¥ ) Þ 2x > 0]
f ¢( x ) = e x ( x – 1) ( x – 2), [using Leibnitz rule]
Thus, f ( x ) is strictly increasing for x Î (0, ¥ ).
+ – +
y Example 4 Find the interval of increasing or –∞ ∞
x 1 2
decreasing of thef ( x ) = ò (t 2 + 2t ) (t 2 – 1) dt.
-1 Using number line rule for f ¢( x ), we get
x f ¢( x ) £ 0 when 1 £ x £ 2, as e x is always positive.
Sol. Given, f ( x ) = ò -1(t + 2t ) (t – 1) dt
2 2

\ f decreases when 1 £ x £ 2
On differentiating both the sides, we have
Hence, (c) is the correct answer.
ìd ü ìd ü
f ¢( x ) = ( x 2 + 2x ) ( x 2 – 1) í ( x )ý –(1 - 2) (1 – 1)í (–1)ý
î dx þ î dx þ y Example 7 If f ( x ) = x × e x (1 – x ) , then f ( x ) is
[using Leibnitz rule]
[IIT JEE 2000]
Þ f ¢( x ) = ( x 2 + 2x ) ( x 2 – 1)
é 1 ù
Þ f ¢( x ) = x ( x + 2) ( x + 1) ( x – 1) (a) increasing on ê – , 1ú (b) decreasing on R
ë 2 û
Using number line rule as shown in figure, é 1 ù
(c) increasing on R (d) decreasing on ê – , 1ú
– –
–∞
+ + +
∞ ë 2 û
–2 –1 0 1 Sol. Here, f ¢( x ) = x ×e x (1 – x ) ×(1 – 2x ) + 1×e x (1 – x )
Clearly, f ¢( x ) ³ 0 when x Î (–¥, – 2] È [–10 , ] È [1, ¥ ) f ¢( x ) = e x (1 – x ) [ x – 2x 2 + 1]
and f ¢( x ) £ 0 when x Î [–2, – 1] È [0, 1]. f ¢( x ) = –e x (1 – x )( x – 1) (2x + 1)
Hence, f ( x ) is increasing, when Using number line rule for f ¢( x ) we get, f ¢( x ) ³ 0, when
x Î (–¥, – 2] È [–1, 0] È [1, ¥ ) and f ( x ) is decreasing, when
é 1 ù
x Î [–2, – 1] È [0, 1]. x Î ê– , 1ú as shown in figure.
ë 2 û
Remark – + –
In above example, Leibnitz rule is used which is stated as –∞ ∞
–1/2 1
d é y( x )
f ( t ) dt ùú = f ( y( x )) ìí y( x ) üý – f ( f( x )) ìí f( x ) üý.
d d
dx êë òf( x ) û î dx þ î dx þ Hence, (a) is the correct answer.
464 Textbook of Differential Calculus

y Example 8 Find the interval for which f ( x ) = x – sin x (iv) f ( x ) = [ x ] can be graphically plotted as shown in the
following figure, which shows f ( x ) = [ x ] is increasing
is increasing or decreasing. but not strictly increasing i.e. non-decreasing in R.
Sol. Given, f ( x ) = x –sin x ,
Y
Differentiating both the sides w.r.t. x, we have
2
f ¢( x ) = 1 – cos x
1
We know, –1 £ cos x £ 1
or cos x £ 1 X′
0 1
X
–2 –1 2 3
Þ 1 – cos x ³ 0 –1
Therefore, f ¢ ( x ) ³ 0, " x Î R –2
y=[x]
Which shows f ( x ) is increasing for the entire number
scale. Y′

i.e. all real numbers. y Example 10 If H ( x 0 ) = 0 for some x = x 0 and


d
y Example 9 Discuss the nature of following H ( x ) > 2cx H ( x ) for all x ³ x 0 , where c > 0, then
functions graphically. dx
1 prove that H ( x ) cannot be zero for any x > x 0 .
(i) f (x) = x 3 (ii) f (x) =
|x | d
Sol. Given that, H ( x ) > 2 cx H ( x )
(iii) f (x) = e x (iv) f (x) = [ x ] dx
Sol. (i) f ( x ) = x 3 can be graphically plotted as shown in the d
Þ H ( x ) – 2 cx H ( x ) > 0
following figure, which shows f ( x ) = x 3 is strictly dx
d 2 2
increasing in R. Þ { H ( x )}e –c x – 2 cxe –c x × H ( x ) > 0
Y dx
ìd ü 2 ìd 2ü
y=x3 Þ í H ( x )ý e –c x + H ( x ) í e –c x ý > 0
î dx þ î dx þ
X′ X ìd –c x 2 ü
O Þ í H ( x ) ×e ý>0
î dx þ
2

Y′ \ H ( x ) e –cx is an strictly increasing function.


1 2
H ( x 0 ) = 0 and e –cx is always positive.
(ii) f ( x ) = can be graphically plotted as shown in the But
|x|
1 Þ H ( x 0 ) > 0 for all x > x 0
following figure, which shows f ( x ) = is strictly
|x| Þ H ( x ) cannot be zero for any x > x 0 .
increasing in ]– ¥, 0[ and strictly decreasing in ] 0, ¥ [.
Y y Example 11 If f ( x ) is a decreasing function and attain
y= 1 positive values, then the set of value of ‘k’, for which
|x|
x2 y2
the major axis of the ellipse + = 1,
X′ X f (k 2 + 2k + 5) f (k + 11)
O
is the X -axis, is (-a , b), then (a + b) is
Sol. Here, f ( x ) is decreasing function and major axis is X -axis
Y′
(iii) f ( x ) = e x can be graphically plotted as shown in the Þ f (k 2 + 2k + 5) > f (k + 11)
following figure, which shows f ( x ) = e x is strictly x2 y2
[Q for the ellipse + = 1 major axis lies
increasing in R. a2 b2
Y along X -axis, then a 2 > b 2 ]
As, f ( x ) is decreasing, therefore
y=e x k 2 + 2k + 5 < k + 11
Þ k2 +k -6 < 0
X′
Þ (k + 3)(k - 2) < 0
X
O
Þ k Î ( -3, 2) = ( -a, b )
Y′ \ a +b = 5
Chap 08 Monotonicity, Maxima and Minima 465

y Example 12 Let f ( x ) = 3x + 5, then show that f ( x ) is ì xe ax ; x £0


Sol. Given, f ( x ) = í
strictly increasing and f –1 ( x ) exists and is strictly î x + ax 2
– x 3
; x >0
increasing for x ÎR. Differentiating both the sides, we have
Sol. Here, f ( x ) = 3x + 5 ì axe ax + e ax ; x £ 0
f ¢( x ) = í
f ¢( x ) = 3 > 0, which is strictly increasing for x Î R. î1 + 2ax – 3x ; x > 0
2

Now, finding f –1
( x ), let f ( x ) = y , y = 3x + 5 Again, differentiating both sides, we have
y –5 ì2ae ax + a 2 xe ax ; x £ 0
or x= f ¢¢ ( x ) = í
3 î 2a – 6x ; x >0

or f (y ) =
–1 y –5
[as f ( x ) = y Þ x = f –1(y )] Now, f ¢¢ ( x ) = 0, then in the interval x £ 0 the root is
3 2 a
x = – and in interval x > 0 root is x =
x –5 a 3
f –1( x ) =
3 Using sign scheme or number line rule as shown in figure,
x – 5 we get
Which shows f –1( x ) = exists for all x Î R and is
3
d 1 – + –

strictly increasing as ( f –1( x )) = > 0 for all x ÎR. –∞
–2 a
dx 3 a 3

y Example 13 Let f ( x ) = sin (cos x ), then check whether


æ 2 ù éa ö
it is increasing or decreasing in [0, p/ 2]. f ¢( x ) decreases on ç–¥, – ú È ê , ¥ ÷ and increases on
è aû ë3 ø
Sol. l. Given, f ( x ) = sin (cos x )
é 2 aù
Þ f ¢( x ) = cos (cos x ) × (–sin x ) êë– a , 3 úû.
Þ f ¢( x ) = –cos (cos x ) × sin x
é æ p öù y Example 16 If a < 0 and f ( x ) = e ax + e –ax is
êQ cos(cos x ) > 0 and sin x > 0,"x Î çè0, 2 ÷ø ú
ë û monotonically decreasing. Find the interval to which
Therefore, it is clearly decreasing for x Î[0, p/ 2] as x belongs.
f¢ ( x ) £ 0. Sol. Given, a < 0 and f ( x ) = e ax + e –ax is monotonically
Aliter decreasing. ...(i)
Here, f ( x ) = sin x and g ( x ) = cos x are increasing and Þ f ¢( x ) < 0
decreasing in [0, p/ 2].
Þ ae ax – ae –ax < 0
Þ ( fog ) ( x ) = f ( x ) = sin (cos x ) is decreasing.
æ e 2ax – 1 ö
Þ a ç ax ÷ < 0 ...(ii)
y Example 14 Let f ( x ) = cos (cos x ), then check è e ø
whether it is increasing or decreasing in [0, p/ 2]. As from Eq. (i), a < 0
Sol. Given, f ( x ) = cos (cos x ) Þ (e 2ax – 1) > 0
Þ f ¢( x ) = –sin (cos x ) × (–sin x ) Þ e 2ax > 1
Þ f ¢( x ) = sin x sin (cos x ) Þ 2ax > 0
é æ p öù
êQ sin(cos x ) > 0 and sin x > 0,"x Î çè0, 2 ÷ø ú Þ ax > 0
ë û Þ x <0 [as a < 0]
Therefore, it is clearly increasing for x Î[0, p/ 2] as f ¢( x ) ³ 0. Thus, f ( x ) is monotonically decreasing, if x Î ( -¥,0).
Aliter
p
Here, f ( x ) = cos x and g ( x ) = cos x are decreasing in [0, p/ 2]. y Example 17 If 0 < a < , then the value of
Þ ( fog ) ( x ) = cos (cos x ) = f ( x ) is increasing. 6
(a cosec a) is
ì xe ax ; x £0 p p
y Example 15 Let f ( x ) = í where a (a) less than
3
(b) more than
3
î x + ax – x ; x > 0
2 3
p p
is positive constant. Find the interval in which f ¢( x ) (c) less than
6
(d) more than
6
is increasing. [IIT JEE 1996]
466 Textbook of Differential Calculus

Sol. From the figure, we can say y Example 19 If f :R ® R, f ( x ) is a differentiable


Slope of OA > Slope of OB
bijective function, then which of the following is true?
æp pö
(a) ( f (x) - x) f ¢¢ (x) < 0 , " x Î R
where A = (a , sin a ) and B ç , sin ÷ .
è6 6ø
(b) ( f (x) - x) f ¢¢ (x) > 0 , " x Î R
Y
B
(c) If ( f (x) - x) f ¢¢(x) > 0 , then f (x) = f -1 (x) has no solution
1
(d) If ( f (x) - x) f ¢¢(x) > 0 , then f (x) = f -1 (x)
A
has at least one real solution
y=sin x
Sol. As, ( f ( x ) - x ) f ¢¢( x ) < 0, " x Î R
Þ ( f ( x ) - x ) > 0 and f ¢¢( x ) < 0)
X
O α π π π or ( f ( x ) - x ) < 0 and f ¢¢( x ) > 0)
6 2
p Can’t be true as f ( x ) - x > 0 and f ¢( x ) are decreasing.
sin -0 Then, f ( x ) has to intersect the line y = x .
sin a - 0 6
Þ > Similarly, f ( x ) - x < 0 and f ¢( x ) is increasing, is not
a -0 p
-0 possible.
6
sina 3 a p p Also, f (x ) - x ¹ 0
Þ > or < Þ a cosec a <
a p sin a 3 3 Þ f (x ) = f -1
( x ) has no solution.
Hence, (a) is the correct answer.
Hence, (c) is the correct answer.
y Example 18 If f ( x ) = ax + bx + cx + d , where
3 2
y Example 20 If f ( x ) and g ( x ) are two positive
a, b , c and d are real numbers and 3b 2 < c 2 , is an
and increasing functions, then
increasing cubic function and
g ( x ) = af ¢( x ) + bf ¢¢ ( x ) + c 2 , then (a) ( f (x)) g( x ) is always increasing
x (b) if ( f (x)) g( x ) is decreasing, then f (x) < 1
(a) ò g(t ) dt is a decreasing function
a
x (c) if ( f (x)) g( x ) is increasing, then f (x) > 1
(b) ò g(t ) dt is an increasing function
a
(d) if f (x) > 1, then ( f (x)) g( x ) is increasing
x
(c) ò g(t ) dt is a neither increasing nor a decreasing function
a Sol. Let h ( x ) = ( f ( x ))g (x )
(d) None of the above Þ log (h ( x )) = g ( x ) {log f ( x )}
Sol. f ¢( x ) = 3ax 2 + 2bx + c > 0 [since, f ( x ) is increasing] 1 g(x ) ¢
Þ × h ¢( x ) = × f ( x ) + {log f ( x )} g ¢( x )
Þ a > 0 and b 2 - 3ac < 0 Þ a > 0 and b 2 < 3ac h( x ) f (x )

Also, g ( x ) = af ¢( x ) + bf ¢¢( x ) + c 2 and h ( x ) is increasing, if log ( f ( x )) > 0 Þ f ( x ) > 1


Hence, (d) is the correct answer.
g ( x ) = 3a 2 x 2 + 2abx + ac + 6abx + 2b 2 + c 2
g ( x ) = 3a 2 x 2 + 8abx + (2b 2 + c 2 + ac ) y Example 21 If the function y = sin ( f ( x )) is monotonic
where D = 64a b - 4 × 3a × (2b + c + ac )
2 2 2 2 2 for all values of x [where f ( x ) is continuous], then the
maximum value of the difference between the
= 4a 2 (16b 2 - 6b 2 - 3c 2 - 3ac )
maximum and the minimum value of f ( x ) , is
= 4a 2 (10b 2 - 3c 2 - 3ac ) < 4a 2 (10b 2 - 3c 2 - b 2 ) p
(a) p (b) 2p (c) (d) None of these
[as 3ac > b Þ - 3ac < - b ]
2 2 2
= 4a 2 (9b 2 - 3c 2 )= 12a 2 (3b 2 - c 2 ) [ given 3b 2 < c 2 ] Sol. As, y = sin ( f ( x )) is monotonic for
é p pù
\ D < 0 Þ g ( x ) > 0," x Î R f ( x ) Î ê2np - , 2np + ú
x ë 2 2û
\ òa g (t ) dt is an increasing function. é p 3p ù
é d æ x ö ù
or êë2np + 2 , 2np + 2 úû
êëQ dx çè òa g (t )dt ÷ø = g ( x ) > 0úû
\ The maximum value of difference is p.
Hence, (b) is the correct answer. Hence, (a) is the correct answer.
Chap 08 Monotonicity, Maxima and Minima 467

y Example 22 If f ¢¢( x ) > 0 and f ¢(1) = 0 such that for g ( x ) to be decreasing, g ¢ ( x ) < 0
g ( x ) = f (cot 2 x + 2 cot x + 2), where 0 < x < p, then the Þ f ¢{(cot x + 1)2 + 1} × ( -2 cosec 2 x ) (cot x + 1) < 0
interval in which g ( x ) is decreasing is
Þ f ¢{(cot x + 1)2 + 1} × (cot x + 1) > 0 …(i)
p
(a) (0 , p) (b) æç , p ö÷ As f ¢¢ ( x ) > 0 Þ f ¢( x ) is increasing, then
è2 ø
æ 3p ö æ 3p ö
3p 3p f ¢{(cot x + 1)2 + 1} > f ¢(1) = 0, " x Î ç0, ÷ È ç , p ÷
(c) æç , p ö÷ (d) æç0 , ö÷ è 4 ø è 4 ø
è 4 ø è 4ø
Thus, Eq. (i) holds, if cot x + 1 > 0
Sol. Here, g ( x ) = f (cot 2 x + 2 cot x + 2) æ 3p ö
Þ cot x > - 1," x Î ç0, ÷
Þ g ¢ ( x ) = f ¢(cot 2 x + 2 cot x + 2) × { -2 cot x cosec 2 x è 4 ø
- 2 cosec 2 x } Hence, (d) is the correct answer.

Exercise for Session 1


1. The curve y = f ( x ) which satisfies the condition f ¢( x ) > 0 and f ¢¢ ( x ) < 0 for all real x, is

Y Y Y Y

(a) X′ X (b) X′ X (c) X′ X (d) X′ X


O O O O
Y′ Y′ Y′ Y′

2. The interval in which f ( x ) = cot -1 x + x increases, is


(a) R (b) (0, ¥) (c) R - {np} (d) None of these

3. The interval in which f ( x ) = 3 cos x + 10 cos x + 6 cos x - 3 increase or decrease in (0, p )


4 3 2

p 2p ö æ p ö æ 2p ö æp ö æ pö
(a) decreases on æç , ÷ and increases on ç 0, ÷ È ç , p ÷ (b) decreases on ç , p ÷ and increases on ç 0, ÷
è2 3 ø è 2ø è 3 ø è2 ø è 2ø
p 2p p 2p ö æ pö æp ö
(c) decreases on æç 0, ö÷ È æç , p ö÷ and increases on æç , ÷ (d) decreases on ç 0, ÷ and increases on ç , p ÷
è 2ø è 3 ø è2 3 ø è 2ø è2 ø
x
4. The interval in which f ( x ) = ò {(t + 1) (et - 1) (t - 2) (t + 4)} dt increases and decreases
0
(a) increases on (-¥, - 4) È (-1, 0) È (2, ¥) and decreases on (-4, - 1) È (0, 2)
(b) increases on (-¥, - 4) È (-1, 2) and decreases on (-4, - 1) È (2, ¥)
(c) increases on (-¥, - 4) È (2, ¥) and decreases on (-4, 2)
(d) increases on (-4, - 1) È (0, 2) and decreases on (-¥, - 4) È (-1, 0) È (2, ¥)
x
5. The interval of monotonicity of the function f ( x ) = , is
loge x
(a) increases when x Î (e, ¥) and decreases when x Î(0, e )
(b) increases when x Î (e, ¥) and decreases when x Î (0, e ) - {1}
(c) increases when x Î(0, e ) and decreases when x Î (e, ¥)
(d) increases when x Î (0, e ) - {1 } and decreases when x Î (e, ¥)
6. Let f ( x ) = x 3 + ax 2 + bx + 5 sin2 x be an increasing function on the set R. Then,
(a) a 2 - 3 b + 15 > 0 (b) a 2 - 3 b + 5 < 0 (c) a 2 - 3 b + 15 < 0 (d) a 2 - 3 b + 5 > 0

7. Let g( x ) = f ( x ) + f (1 - x ) and f ¢¢ ( x ) > 0, " x Î (0, 1). Then, g( x ) is

(a) increasing on æç 0, ö÷ and decreasing on æç , 1ö÷ (b) increasing on æç , 1ö÷ and decreasing on æç 0,
1 1 1 1ö
÷
è 2ø è2 ø è2 ø è 2ø
(c) increasing on (0, 1) (d) decreasing on (0, 1)
Session 2
Critical Points
Critical Points Þ
é æa –2ö
( a – 1) ê1 – ç

÷ sin ú = 0
It is a collection of points for which, ë è 2 ø 2û
(i) f ( x ) does not exist (ii) f ¢( x ) does not exist Þ
æa –2ö
a = 1 and 1 – ç
x
÷ sin = 0,
(iii) f ¢( x ) = 0. è 2 ø 2

All the values of x obtained from above conditions are said but at a = 1 Þ f (x ) = 0
to be the critical points. æx ö 2
Thus, sin ç ÷ =
It should be noted that critical points are the interior è 2 ø a –2
points of an interval. 2
Þ £1
a –2
y Example 23 Find the critical points for
Þ | a – 2| ³ 2 Þ a –2 ³ 2
f ( x ) = ( x – 2) 2/ 3 (2x + 1).
or a – 2 £ –2
Sol. Given, f ( x ) = ( x – 2) 2 / 3 ( 2x + 1) a ³ 4 or a £ 0
2 a Î (–¥, 0] È [ 4, ¥ )
Þ f ¢( x ) = ( x – 2)–1/ 3 (2x + 1) + ( x – 2)2 / 3 × 2 Therefore,
3
é ( 2x + 1) ( x – 2) 2 / 3 ù y Example 25 The set of all values of ‘b’ for which the
or f ¢( x ) = 2 ê 1/ 3
+ ú function
ë 3( x – 2) 1 û f ( x ) = (b 2 - 3 b + 2) (cos 2 x - sin 2 x ) + (b - 1) x + sin 2
Clearly, f ¢( x ) is not defined at x = 2, so x = 2 is a critical point.
does not possess stationary points is
Another critical point is given by
(a) [1, ¥) (b) (0 , 1) È (1, 4)
f ¢( x ) = 0
(c) æç , ö÷
3 5
(d) None of these
é ( 2x + 1) + 3 ( x – 2) ù è 2 2ø
i.e. 2ê ú =0
ë ( x – 2)1/ 3 û Sol. Here, f ( x ) = (b 2 - 3b + 2)(cos 2 x - sin 2 x ) + (b - 1)x + sin 2
Þ 5x – 5 = 0 Þ x = 1 = (b 2 - 3b + 2)cos 2x + (b - 1)x + sin 2
Hence, x = 1 and x = 2 are two critical points of f ( x ).
Þ f ¢( x ) = (b 2 - 3b + 2) × ( -2sin 2x ) + (b - 1).
y Example 24 Find all the values of a for which the As, f ( x ) does not possess stationary points.
function Þ f ¢( x ) ¹ 0
æxö Þ (b - 1) (b - 2) ( -2sin 2x ) + (b - 1) ¹ 0,
f ( x ) = (a 2 – 3a + 2) cos ç ÷ + (a – 1) x , possess critical
è2ø for any x Î R
points. Þ (b - 1) {1 - 2 (b - 2) sin 2x } ¹ 0
æx ö ½
½ 1 ½
½ > 1 and b ¹ 1
Sol. Given, f ( x ) = (a 2 – 3a + 2) cos ç ÷ + (a – 1) x Þ
è2ø ½2 (b - 2) ½
1 æx ö 1 1
Þ f ¢( x ) = – (a – 1) (a – 2) sin ç ÷ + (a – 1) Þ - < b - 2 < and b ¹ 1
2 è2ø 2 2
é 1 æ x öù 3 5 æ3 5ö
Þ f ¢( x ) = (a – 1) ê1 – (a – 2) sin ç ÷ú Þ <b< Þ b Îç , ÷
è 2 øû 2 2 è2 2ø
ë 2
If f ( x ) possess critical points, then Hence, (c) is the correct answer.
f ¢( x ) = 0
Chap 08 Monotonicity, Maxima and Minima 469

y Example 26 Find the set of critical points of the 2p


or 4 x = 2np + , n ÎInteger
function 3
x æ1
np p
ö Þ x= ± ,n ÎI
f ( x ) = x – log x + ò ç – 2 – 2 cos 4z ÷ dz . 2 6
2 èz ø
But log x is defined for x > 0
æ1
x ö p
Sol. Here, f ( x ) = x – log x + ò2
ç – 2 – 2 cos 4z ÷ dz
èz ø
\ Forn = 0, x = ±
6
1 æ 1 ö p
Þ f ¢( x ) = 1 – + ç – 2 – 2 cos 4 x ÷ (1) – 0 Þ x= [neglecting x = – p/ 6 ]
x èx ø 6
= – 1 – 2 cos 4x [using Leibnitz rule] ì p np p ü
Put f ¢( x ) = 0, \ Set of critical points = í , ± ý, where n Î N .
1 æ pö î6 2 6þ
Þ cos 4 x = – or cos 4 x = cos ç p - ÷
2 è 3ø

Exercise for Session 2


1. Determine all the critical points for the function f ( x ) = 6x 5 + 33x 4 - 30x 3 + 100.

2. Find the critical points of f ( x ) = x 2/ 3(2x - 1).


2
3. Determine all the critical points for the function f ( x ) = xe x .

4. The number of critical points of f ( x ) = max {sin x , cos x }, " x Î ( -2p , 2p ) is


(a) 5 (b) 6
(c) 7 (d) 8
Session 3
Comparison of Functions Using Calculus
Comparison of Functions y Example 29 For all x Î(0, 1) [IIT JEE 2000]
Using Calculus (a) e < 1 + x
x
(b) log e (1 + x ) < x
If we want to compare f( x ) and g( x ) then we consider a (c) sin x > x (d) log e x > x
function f ( x ) = f( x ) – g ( x ) or f ( x ) = g( x ) – f( x ) and Sol. (a) Let f ( x ) = e –1 – x
x

check whether f ( x ) is increasing or decreasing in given


Þ f ¢( x ) = e x –1 > 0, "x Î (0, 1)
domain of f( x ) and g( x ). The procedure is illustrated by
following examples : So, f ( x ) is increasing, when 0 < x < 1
Þ f ( x ) > f (0) or e x – 1 – x > 0
y Example 27 Using calculus, find the order relation
Þ ex > 1+ x
between x and tan –1 x when x Î [0, ¥ ).
Hence, (a) is false.
Sol. Let f ( x ) = x – tan –1( x )
(b) Let g ( x ) = log (1 + x )– x
1
Þ f ¢( x ) = 1 – 1 –x
1+ x2 Þ g ¢( x ) = –1 = < 0, "x Î (0, 1)
1+ x 1+ x
x2
Þ f ¢( x ) = ³ 0, "x Î [0, ¥ ) So, g ( x ) is decreasing, when 0 < x < 1
1+ x2
Þ g (0) > g ( x ) Þ log (1 + x ) < x
Thus, f ( x ) is an increasing function.
Hence, (b) is true.
As, we know x 1 £ x 2 Þ f ( x 1 ) £ f ( x 2 ) for increasing function
(c) Let h ( x ) = sin x – x Þ h ¢( x ) = cos x – 1 < 0, "x Î (0, 1)
\ x ³ 0, "x Î [0, ¥ )
So, h ( x ) is decreasing, when 0 < x < 1 Þ h ( x ) < h (0)
Þ f ( x ) ³ f (0), "x Î [0, ¥ )
Þ sin x < x
Þ x – tan –1 x ³ 0 – tan –1 (0), "x Î [0, ¥ )
Hence, (c) is false.
Þ x ³ tan –1 x , "x Î [0, ¥ ) 1
(d) Let g ( x ) = log x – x Þ g ¢( x ) = –1
Thus, the above relation is the order relation between x and x
tan –1 x. \ g ¢ ( x ) > 0, "x Î (0, 1) or g ( x ) < g (1)
Þ log x – x < – 1
y Example 28 Using calculus, find the order relation Þ x – 1 > log x or x > log x
between x and tan –1 x when x Î (– ¥, 0]. Hence, (d) is false.
Thus, (b) is the correct answer.
Sol. Let f ( x ) = ( x )– tan –1 ( x )
1 y Example 30 Prove that
Þ f ¢( x ) = 1 – 2
1+ x2 æ –1 1 ö 2e 2
ç tan ÷ + < (tan –1 e ) 2 + .
x2 è e ø e +1
2
e +1
2
Þ f ¢( x ) = ³ 0, "x Î (–¥, 0]
1+ x2
Sol. Let us consider a function f ( x ),
Thus, f ( x ) is increasing function.
2
As, we know x 1 £ x 2 Þ f ( x 1 ) £ f ( x 2 ) for increasing i.e. f ( x ) = (tan –1 x )2 + for all x Î R
function. x +1
2

\ x £ 0, "x Î (–¥, 0] 2 tan –1 x 2x


\ f ¢( x ) = – 2
Þ f ( x ) £ f (0), "x Î (–¥, 0] 1 + x2 ( x + 1) 3 / 2
Þ x – tan –1 x £ 0, "x Î (–¥, 0]
2 é x ù
Þ x £ tan –1 x , "x Î (–¥, 0] = ê tan –1
x – ú
1 + x 2 êë x 2 + 1 úû
Thus, the above relation is the order relation between x and
tan –1 x.
Chap 08 Monotonicity, Maxima and Minima 471

2 Þ g ( x ) is increasing for x Î R
Þ f ¢( x ) = g ( x ),
1 + x2 Þ g ( x ) > g (0) for all x Î R
x
where g ( x ) = tan –1 x – ...(i) Þ tan –1 x –
x
>0 ...(ii)
x2 + 1 x +1
2

1 1 x2
\ g ¢( x ) = – + \ From Eqs. (i) and (ii), f ¢ ( x ) > 0 for all x > 0
1 + x2 x2 + 1 ( x 2 + 1) 3 / 2
1 1 \ f ( x ) is increasing for all x > 0
= –
1 + x2 ( x 2 + 1) 3 / 2 Þ f (1/e ) < f (e )
1 æ 1 ö æ –1 1 ö
2
2e 2
= ç1 – ÷ > 0 for all x Î R Þ ç tan ÷ + < (tan –1 e )2 +
1 + x çè2
( x 2 + 1) ÷ø è eø e +1
2
e +1
2

Exercise for Session 3


1. Show that sin x < x < tan x for 0 < x < p / 2.
x
2. Show that < log (1 + x ) < x for x > 0.
(1 + x )
x3 p
3. Show that x - < sin x for 0 < x < .
6 2
b
4. If ax 2 + ³ c for all positive x, where a, b > 0, then
x
(a) 27ab 2 ³ 4c 3 (b) 27ab 2 < 4c 3
(c) 4ab 2 ³ 27c 3 (d) None of these
b
5. If ax + ³ c for all positive x, where a, b > 0, then
x
c2 c2
(a) ab < (b) ab ³
4 4
c
(c) ab ³ (d) None of these
4
Session 4
Introduction to Maxima and Minima, Methods of Finding
Extrema of Continuous Functions, Convexity/Concavity
& Point of Inflection, Concept of Global Maximum/Minimum

Introduction to Methods of Finding Extrema


Maxima and Minima of Continuous Functions
By the maximum/minimum value of function f ( x ) we The following tests apply to a continuous function in order
mean local or regional maximum/minimum value and not to get the extrema
the greatest/least value attainable by the function. It is
also possible in a function that local maximum at one
point is smaller than local minimum at another point. First Derivative Test
Sometimes, we use the word extrema for maxima and As we know the function attains maximum, when it has
minima. assumed its maximum value and attains minimum, when it
Definition A function f ( x ) is said to have a maximum at has assumed its minimum value which could be shown as
x = a, if f (a ) is greatest of all values in the suitably small
(i) At a critical point x = a
neighbourhood of a, where x = a is an interior point in the
(a) When f ( x ) attains maximum at ( x = a )
domain of f ( x ).
Consider the following graph
Analytically, this means f (a ) ³ f (a + h ) and
f (a ) ³ f (a – h ), where h ³ 0 (very small quantity). Y

Y
f(a) ymax at x = a
f ( a –h ) f (a+h)
g
de

in
cre

s
ea
g

f (b)
as
sin

cr

θ2 > 90°
in
ing
rea

f (b+h) θ1 < 90°


inc

f(b–h) X
X O x=a
O a–h a a+h b b +h
b –h
Figure 8.12 Tangent at any point when x<a Tangent at any
Similarly, a function y = f ( x ) is said to have a minimum at point when x>a
x = b, if f (b ) is smallest of all values in the suitably small Figure 8.14
neighbourhood of b, where x = b is an interior point in the
domain of f ( x ). From above graph, we see that
Analytically, f (b ) £ f (b + h ) and f (b ) £ f (b – h ), where ìfor x < a, q 1 < 90° Þ tan q 1 > 0 or increasing for x < a
h ³ 0 (very small quantity). ïïfor x = a, tan q = 0 Þ neither increasing nor
Y í
f (a ) ï decreasing for x = a
f(a–h) f(a+h) ïîfor x > a, q 2 > 90° Þ tan q 2 < 0 or decreasing for x > a
s ing de
cr
ea ea
Thus, we can say,
cr s ing
in f (b )
f(b+h) f ( x ) is maximum at some point ( x = a ).
f(b–h)
ì f ( x ) is increasing for x < a
O a–h a a+h b–h b b+h
X Þ í
î f ( x ) is decreasing for x >a
Figure 8.13
Chap 08 Monotonicity, Maxima and Minima 473

(b) When f ( x ) attains minimum at ( x = a ) y Example 32 If f ¢( x ) changes from negative to


Consider the following graph
positive at x 0 while moving from left to right,
Y i.e. f ¢( x ) < 0, x < x 0
f ¢( x ) > 0, x > x 0 ,
then f ( x ) has local minimum value at x = x 0
Tangent at any
Sol. Consider the following graph
Tangent at θ1 > 90° point when x>a
Y Y
any point ymin at x = a
when x<a f ′(x) < 0 f ′(x) > 0
θ2 < 90°
X increasing
O x=a f ′(x)<0 f ′(x)>0
Local minima
Figure 8.15 Local minima
X X
O x0 O x0
From the above graph, we see that
f ′ (x0) doesn‘t exist
ìfor x < a, q 1 > 90° Þ tan q 1 < 0 or decreasing when x < a f ′ (x0) = 0
ïïfor x = a, tan q = 0 Þ neither increasing nor In both the graph we see that f ¢ ( x ) changes its sign from
í negative to positive when we move through x 0 and the
ïdecreasing for x = a curve has local minima at x = x 0 .
ïîfor x > a, q 2 < 90° Þ tan q 2 > 0 or increasing when x > a
Thus, we can say, y Example 33 If sign of f ¢( x ) doesn’t change at x 0 ,
f ( x ) is minimum at some points ‘x = a’, while moving from left to right, then f ( x ) has neither a
ì f ( x ) is decreasing for x < a maximum nor a minimum at x 0 .
Þ í Sol. Consider the following graph
î f ( x ) is increasing for x > a
Y Y
Here, some of the examples are given to make it more f ′(x)<0
clear.
f ′(x)>0
Remark (Ex. Nos. 31-33) f ′(x)>0
Statement of example Number 31, 32, 33 can be used directly
as result. f ′(x)<0
X X
O x0 O x0
y Example 31 If f ¢( x ) changes from positive to f ′(x0)=0 f ′(x0) doesn’t exist
negative at x 0 while moving from left to right, In both the graph we see that f ¢ ( x ) has same sign when
i.e. f ¢( x ) > 0, x < x 0 we move through x 0 , therefore f ( x ) has neither maxima
f ¢( x ) < 0, x > x 0 , then f ( x ) has local nor minima at x = x 0 .
maximum value at x = x 0 .
y Example 34 Let f ( x ) = x 3 – 3x 2 + 6 find the point at
Sol. Consider the following graph
Y Y which f ( x ) assumes local maximum and local minimum.
Local maxima Local maxima Sol. Given, f ( x ) = x 3 – 3x 2 + 6
f ′(x0)>0 f ′(x0)<0 f ′(x0)>0 f ′(x0)<0
Þ f ¢( x ) = 3x 2 – 6x = 3x ( x – 2)
Using number line rule,
f ¢( x ) changes sign from +ve to –ve at x = 0 and f ¢( x )
X X
changes sign from –ve to +ve at x = 2.
O x0 O x0
+ – +
f ′(x0)=0 {f ′(x0) exists} f ′(x0) doesn’t exist –∞ ∞
0 2
In both the graph we see that f ¢ ( x ) changes its sign from Therefore, at x = 0, we have local maxima and at x = 2 , we
positive to negative when we move through x 0 and the have local minima.
function has local maxima at x = x 0 .
474 Textbook of Differential Calculus

Y
y Example 35 Let f ( x ) = x 3 find the point at which local maxima
f ( x ) assumes local maximum and local minimum. f ′( x)<0, ∀x ∈ [a, b]
Sol. Given, f (x ) = x 3 local minima
Þ f ¢( x ) = 3x 2
X
Here, f ¢ ( x ) ³ 0 for all x, it does not change sign from O a b
negative to positive or positive to negative.
Thus, f ( x ), we has neither maximum nor minimum. Figure 8.16
Again, if f ¢( x ) > 0 for x > a, then f ( x ) has local minimum
1 at x = a and local maximum at x = b.
y Example 36 Let f ( x ) = x + , x ¹ 0. Discuss the
x Y
local maxima
maximum and minimum value of f ( x ).
1 local minima
Sol. Here, f (x ) = x + f ′( x)>0, ∀x ∈[a, b]
x
1
Þ f ¢( x ) = 1 –
x2 O a b
X

+ – +
–∞ ∞
Figure 8.17
–1 1
We can summarise above result as
x 2 – 1 ( x – 1)( x + 1)
f ¢( x ) = = (a) If a function is strictly increasing in[a, b ] , then
x2 x2
ì f (a ) is local minimum
Using number line rule, we have maximum at x = – 1 and í
minimum at x = 1 î f (b ) is local maximum
\ At x = –1, we have local maximum Þ f ( x )max = – 2 (b) If a function is strictly decreasing in[a, b ] , then
and at x = 1, we have local minimum Þ f ( x )min = 2 ì f (a ) is local maximum
í
î f (b ) is local minimum
y Example 37 The function
x
f ( x ) = ò t(e t – 1) (t – 1) (t – 2) 3 (t – 3) 5 dt has a local y Example 38 Find the local maximum and local minimum
-1
maximum at x equals [IIT JEE 1999]
of f ( x ) = x 3 + 3x in [–2, 4 ] .
(a) 0 (b) 1 Sol. Given, f ( x ) = x 3 + 3x
(c) 2 (d) 3 Þ f ¢( x ) = 3x 2 + 3 which is strictly increasing for all x Î R
x
ò-1t (e
Sol. Given, f ( x ) = t
– 1) (t – 1) (t – 2)3 (t – 3)5 dt and thus, increasing for [–2, 4 ].
Hence, local minimum is f (–2) = (–2)3 + 3 (–2) = –14
f ¢( x ) = x (e x – 1) ( x – 1) ( x – 2)3 ( x – 3)5 and local maximum is f ( 4 ) = ( 4 )3 + 3 ( 4 ) = 76
[using Leibnitz rule]
Using number line rule for f ¢( x ), we get the following ì 3x 2 + 12x – 1, – 1 £ x £ 2
figure which shows local maxima at x = 2 as f ¢( x ) changes y Example 39 If f ( x ) = í ,
from (+ve) to (–ve) and local minima at x = 1 and x = 3 as î 37 – x , 2 < x £ 3,
f ¢( x ) changes from (–ve) to (+ve). then
–∞ ∞ (a) f (x) is increasing on [–1, 2] [IIT JEE 1993]
0 1 2 3
(b) f (x) is continuous on [–1, 3 ]
\ Local minima at x =1,3 and local maxima at x =2. (c) f ¢(x) does not exist at x = 2
Hence, (c) is the correct answer. (d) f (x) has the maximum value at x = 2
ì3x 2 + 12x – 1, – 1 £ x £ 2
(ii) At left end point a and right end Sol. Given, f ( x ) = í
point b in [a, b] î 37 – x , 2 < x £ 3
Let f ( x ) be defined on [a, b ] and if f ¢( x ) < 0 for x > a, then Þ
ì6x + 12,
f ¢( x ) = í
–1 £ x < 2
f ( x ) has local maximum at x = a and local minimum at î – 1, 2< x £3
x = b. (a) Which shows f ¢( x ) > 0 for x Î[–1, 2]
Chap 08 Monotonicity, Maxima and Minima 475

So, f ( x ) is increasing on [–1, 2]. Hence, (a) is correct. If f ¢¢¢(a ) ¹ 0, then f ( x ) has neither maximum nor
(b) For continuity of f ( x ). [check at x = 2] minimum (inflexion point), at x = a.
Q f ( x ) = 3x 2 + 12x - 1,-1 £ x £ 2 But, if f ¢¢¢(a ) = 0, then find f iv (a )
It is a polynomial, therefore continuous in [– 1, 2].
If f iv (a ) = positive, then f ( x ) is minimum at x = a.
Also, for (2,3], f ( x ) = 37 - x which is again a If f iv (a ) = negative, then f ( x ) is maximum at x = a.
polynomial, therefore continuous in (2,3]. and so on, process is repeated till point is discussed.
Now, we have to check continuity of f ( x ) at x = 2
LHL = lim f (2 - h ) = lim 3(2 - h )2 + 12(2 - h ) - 1 = 35
h ®0 h ®0 Convexity/Concavity and
RHL = lim f (2 + h ) = lim 37 - (2 + h ) = 35
h ®0 h ®0 Point of Inflection
RHL = 35 , LHL = 35 and f (2) = 35
Observe the two graphs sketched in the figure below.
So, (b) is correct.
What is the difference between them? Although they are
(c) As discussed in previous chapter,
both increasing, the first graph’s rate of increase in itself
37 - (2 + h ) - 35
Rf ¢(2) = lim = -1 increasing whereas the rate of increase is decreasing in
h ®0 h case of the second graph.
3(2 - h )2 + 12(2 - h ) - 1 - 35
and Lf ¢(2) = lim = 24 Y Y
h ®0 -h
So, not differentiable at x = 2
Hence, (c) is correct.
(d) We know f ( x ) is increasing on [–1, 2] and decreasing X X
on (2, 3]. Thus, maximum at x = 2 O O
(i) (ii)
Hence, (d) is correct.
Figure 8.18
\ Hence, (a), (b), (c) and (d) all are correct answers.
On graph (i), if you draw a tangent any where, the entire
y Example 40 Let f ( x ) = sin x – x on [0, p/ 2] , find local curve will lie above this tangent. Such a curve is called a
maximum and local minimum. concave upwards curve.
Sol. Given, f ( x ) = sin x – x For graph (ii), the entire curve will lie below any tangent
drawn to itself. Such a curve is called a concave
f ¢( x ) = cos x –1, "x Î [0, p/ 2]
downwards curve.
(cos x – 1) £ 0, "x Î [0, p/ 2] , as cos x £ 1
The concavity’s nature can of course be restricted to
\ f ¢( x ) £ 0, "x Î [0, p/ 2] particular intervals. e.g. A graph might be concave
\ f ( x ) is decreasing for x Î[0, p/ 2]. upwards in some interval while concave downwards in
Hence, maximum value of f ( x ) is at x = 0 another shown in fig. 8.19.
i.e. f max (0) = sin 0 – 0 = 0 Y
and minimum value of f ( x ) is at x = p / 2
Concave
upwards

æpö p p p Concave
i.e. f min ç ÷ = sin – = 1 – downwards
è2ø 2 2 2

‘‘nth Derivative Test’’ O


X

Figure 8.19
First we find the root of f ¢( x ) = 0. Suppose x = a is one of
the roots of f ¢( x ) = 0.
Now, find f ¢¢ ( x ) at x = a Relation of Concavity y
(i) If f ¢¢(a ) = negative, then f ( x ) is maximum at x = a with the Derivative
(ii) If f ¢¢(a ) = positive, then f ( x ) is minimum at x = a Let us again consider a graph in figure.
(iii) If f ¢¢(a ) = zero This is a concave upwards curve. We see
x
that the rate of increase of the graph
Then, we find f ¢¢¢( x ) at x = a O
itself increases with increasing x, i.e. rate
of increase of slope is positive. Figure 8.20
476 Textbook of Differential Calculus

d æ dy ö d 2y y Example 41 Let f ( x ) = x ( x – 1) 2 , find the point at


ç ÷ > 0 Þ >0
dx è dx ø dx 2 which f ( x ) assumes maximum and minimum.
d 2y Sol. Given, f ( x ) = x ( x – 1) 2
Similarly, for a concave downwards curve, <0
dx 2 f ¢( x ) = 2x ( x – 1) + ( x – 1)2
The nature of concavity is simply related to the sign f ¢( x ) = ( x – 1) [2x + x – 1]
of the second derivative. f ¢( x ) = ( x – 1) (3x – 1)
We now finally come to what we mean by point of Using number line rule for f ¢( x ), we have the following
inflection. figure which shows f ¢( x ) changes sign from + ve to – ve at
Consider f ( x ) = x 3 again f ¢ (0 ) = f ¢¢(0 ) = 0 x =1/ 3. Hence, at x =1/ 3, we have maximum and f ¢( x )
changes sign from – ve to + ve at x =1.
f ¢ ¢ ( x ) < 0 for x < 0
+ +
and > 0 for x > 0 –∞
1

– 1
Þ f ¢ ¢ ( x ) changes sign as x crosses 0. 3
Þ f ( x ) changes the nature of its concavity as x crosses 0. Hence, f ( x ) is minimum at x =1.
Such a point is called a point of inflection, a point at Aliter
which the concavity of the graph changes. We have, f ¢( x ) = ( x –1)(3x –1) and f ¢¢( x ) = 6x – 4
Notice that f ¢¢(a ) = 0 alone is not sufficient to guarantee a Let f ¢( x ) = 0 Þ x =1,1/ 3 [critical points]
point of inflection at x = a. f ¢¢( x ) must also change sign as \ f ¢¢(1) = 2 > 0, i.e. minimum at x =1
x crosses a. and f ¢ ¢(1/ 3) = –2 < 0, i.e. maximum at x =1/ 3
e.g. In f ( x ) = x 4 , f ¢¢(0 ) = 0, but x = 0 is not a point of
inflection since f ¢¢( x ) does not change its sign as x y Example 42 Let f ( x ) = ( x – 1) 4 discuss the point at
crosses 0. From the higher order derivative test, we know which f ( x ) assumes maximum or minimum value.
that x = 0 is a local minimum for f ( x ) = x 4 .
Sol. Given, f ( x ) = ( x – 1) 4
A summary of results for maxima, f ¢( x ) = 4 ( x – 1)3
minima and point of Inflection Using number line rule.
+
First order Second order Higher order –∞ ∞
derivative test derivative test derivative test
– x=1
Shows f ¢( x ) changes sign from –ve to + ve.
Max f ¢(a ) = 0 f ¢(a ) = 0 f ¢(a ) = 0
. Therefore, f ( x ) assumes minimum at x = 1.
f ¢( x ) changes f ¢¢(a ) < 0 f ¢¢(a ) = 0
sign from +ve to Aliter
M
-ve as x crosses a Given, f ( x ) = ( x – 1)4 Þ f ¢( x ) = 4 ( x – 1)3
f n - 1(a ) = 0
f n (a ) < 0
Let f ¢( x ) = 0 Þ x = 1
Now, f ¢¢( x ) = 12 ( x – 1)2 which is zero at x = 1
where n is even
(If n is odd, x = a is not i.e. f ¢¢(1) = 0
an extremum point; it is
Thus, finding f ¢¢¢( x ) = 24 ( x – 1) which is again zero at
a point of inflextion)
x =1
Min. f ¢(a ) = 0 f ¢(a ) = 0 f ¢(a ) = 0 i.e. f ¢¢¢(1) = 0
f ¢( x ) changes f ¢¢(a ) > 0 f ¢¢(a ) = 0 Again, finding f iv ( x ) = 24 which is positive at x = 1,
sign from -ve to M
+ve as x crosses a f n - 1 (a ) = 0 i.e. f iv (1) > 0
f n (a ) > 0 Therefore, minimum at x = 1.
where n is even
(If n is odd, x = a is not y Example 43 Discuss the function
an extremum point; it is f ( x ) = x 6 - 3x 4 + 3x 2 - 5, and plot the graph.
a point of inflextion)
Sol. The domain is obviously R.
Point of inflection f ¢¢( x ) change sign f ( x ) is an even function.
at x = a
Since f ( x ) is a polynomial function, it is continuous and
differentiable on R.
Chap 08 Monotonicity, Maxima and Minima 477

f ¢ ( x ) = 6x 5 - 12x 3 + 6x = 6x ( x 4 - 2x 2 + 1) = 6x ( x 2 - 1)2 f ( x ) is continuous and differentiable on R.


f ¢ ( x ) = 0 Þ 0, ± 1 f ¢ ( x ) = cos 2 x - sin x
f ¢ ¢ ( x ) = 6( x 2 - 1)2 + 24 x 2 ( x 2 - 1) = 1 - 2sin 2 x - sin x
= ( x 2 - 1) {6( x 2 - 1) + 24 x 2 } = (1 + sin x ) (1 - 2sin x )
p 5p 3p
= ( x 2 - 1)(30x 2 - 6) …(i) This is 0 in [0, 2p] when x = , ,
6 6 2
= 6( 5x - 6x + 1)
4 2
f ¢¢( x ) = - 2sin 2x - cos x
f ¢ ¢ (0) = 6, f ¢ ¢ ( ± 1) = 0 æpö æ 5p ö æ 3p ö
Now, f ¢¢ ç ÷ < 0, f ¢¢ ç ÷ > 0 and f ¢¢ ç ÷ = 0
Þ x =0 is a point of local minimum and x = ±1 are points è6ø è 6 ø è 2 ø
of inflection
p æpö 3 3
(verify that f ¢¢( x ) does not change sign as x crosses ±1). Þx = is a local maximum for f ( x ); f ç ÷ =
6 è6ø 4
Now, f ¢ ( x ) > 0 if x > 0 andf ¢ ( x ) < 0 if x < 0.
Therefore, f ( x ) decreases on ( - ¥,0) and increases on 5p æ 5p ö - 3 3
x= is a local minimum for f ( x ); f ç ÷ =
(0, - ¥ ). 6 è 6 ø 4
There is one more important fact we must take into 3p æ 3p ö
1 x= is a point of inflection; f ç ÷ = 0
account. f ¢ ¢( x ) has roots ±1 and additionally, ± . 2 è 2 ø
5
We now need to analyse the sign of f ¢¢( x ).
[from Eq. (i)]
f ¢¢( x ) = - 2sin 2x - cos x
Therefore, at these four points the convexity of the graph
changes = - 4sin x cos x - cos x
æ -1 1 ö = - cos x (1 + 4 sin x )
Þ f ¢ ¢ ( x ) > 0" x Î( - ¥, - 1) È ç , ÷ È (1, ¥ ) so that
è 5 5ø This is 0 in [0, 2p] when
f ( x ) is concave upwards in these intervals. p 1 3p 1
x = , p + sin -1 , , 2p - sin -1
æ -1 ö æ 1 ö 2 4 2 4
Þ f ¢¢( x ) < 0 " x Îç - 1, ÷ È ç , 1÷ so that f ( x ) is
è 5ø è 5 ø We see that f ( x ) will change its convexity at four different
concave downwards in these intervals. points.
f (0) = - 5, f ( ±1) = - 4, f ( ± 2) = 23 æp 1ö æ 3p 1ö
Þ f ¢¢( x ) > 0 " x Îç , p + sin -1 ÷ È ç , 2p - sin -1 ÷
è2 4 ø è 2 4ø
Therefore one root each of f ( x ) lies in ( - 2, - 1) and (1, 2).
This information is sufficient to accurately draw the graph So that f ( x ) is concave upwards in these intervals
of the given function. æ pö
Þ f ¢ ¢ ( x ) < 0 " x Î ç0, ÷ È
–1 1 è 2ø
–1 √5 √5 1
æ -1 1 3p ö æ 1 ö
ç p + sin , ÷ È ç2p - sin -1 , 2p ÷
è 4 2 ø è 4 ø
So that, f ( x ) is concave downwards in these intervals.
æpö
f (0) = 1, f ç ÷ = 0, f (2p ) = 1
è2ø
x=–1, 1 are points The graph has been plotted below for [0, 2p]
of inflection
–4 Y

–5
1

y Example 44 Discuss the function X


0 π π 5π π 3π 2π
1 6 2
f ( x ) = sin 2x + cos x ., and plot its graph. –1
6 2
2 At the points marked
with squares, the convexity
Sol. The domain of f ( x ) is R. of the graph changes
f ( x ) is periodic with period 2p and therefore we need to
analyse it only in [0, 2p].
478 Textbook of Differential Calculus

y Example 45 Discuss the function


y = x + ln ( x 2 - 1) and plot its graph.
Global Maximum/Minimum in [a, b]
In order to find the global maximum and minimum of f ( x ) in
Sol. The domain is given by x 2 - 1 > 0 [a, b ], find out all critical points of f ( x ) in [a, b ] (i.e. all points
Þ D = R \ [ - 1,1] at which f ¢( x ) = 0).
f ( x ) is continuous and differentiable on D. Let c 1 , c 2 , c 3 , ..., c n be the points at which f ¢ ( x ) = 0
\ lim + y = - ¥; lim - y = - ¥ and let f (c 1 ), f (c 2 ), ...., f (c n ) be the values of the function at
x ®1 x ®1
these points,
Þ x = ± 1 are vertical asymptotes to the curve.
max { f (a ), f (c 1 ), f (c 2 ), ..., f (c n ), f (b )} = M 1 (say)
Verify that the graph has no other asymptotes.
2x min { f (a ), f (c 1 ), f (c 2 ), ..., f (c n ), f (b )} = M 2 (say)
y¢= 1 + 2 Then, M 1 is the greatest value or global maxima in[a, b ] and
x -1
M 2 is the least value or global minima in[a, b ].
Þ y ¢ = 0 when x 2 + 2x - 1 = 0
y Example 46 Let f ( x ) = 2x 3 – 9 x 2 + 12x + 6. Discuss the
Þ x = - 1± 2
global maxima and minima of f ( x ) in [0, 2].
x = - 1 + 2 does not belong to D.
Sol. Given, f ( x ) = 2x 3 – 9 x 2 + 12x + 6
x = - 1 - 2 is an extremum point.
Þ f ¢( x ) = 6x 2 – 18x + 12
2( x 2 + 1)
Þ y ¢¢ = - < 0, " x Þ f ¢( x ) = 6 ( x 2 – 3x + 2)
( x 2 - 1) 2
Þ f ¢( x ) = 6 ( x – 1) ( x – 2)
Þ The curve is always concave downwards so that
Put f ¢( x ) = 0
x = - 1 - 2 is a point of local maximum.
\ x = 1, 2 [say c 1 and c 2 ]
lim y = ¥
x ®+¥ Then, for global maximum or global minimum.
lim y = - ¥ We have, f (0) = 6, f (1) = 11, f (2) = 10,
x ®- ¥
\ Global maximum Þ M1 = max {6, 10, 11} = 11
\ Based on this data, the graph can be plotted as and global minimum Þ M 2 = min {6, 10, 11} = 6
shown below.
\ f (1) = 11 global maximum and f (0) = 6 global minimum.
Y

Global Maximum/Minimum in (a, b)


Method for obtaining the greatest and least values of f ( x ) in
–1–√2 (a, b) is almost the same as the method used for obtaining the
X′ X
–1 0 1 greatest and least values in [a, b].
–1
However a caution may be exercised
let M 1 = max { f (c 1 ), f (c 2 ), ..., f (c n )}
and M 2 = min { f (c 1 ), f (c 2 ), ..., f (c n )}
Y′ Now, M 1 and M 2 are global maximum and global minimum,
respectively.
But, if lim f ( x ) > M 1
Concept of Global x ®a+

lim f ( x ) < M 2
Maximum/Minimum or
x ®b–

Let y = f ( x ) be a given function with domain D. Þ f ( x ) would not possess global maximum or global minimum
Let [a, b ] Í D , then global maximum/minimum of f ( x ) in (a, b ).
in [a, b ] is basically the greatest/least value of f ( x ) in This means that if limiting values at the end points are
[a, b ]. greater than M 1 or less than M 2 , then global maximum
Global maxima/minima in [a, b ] would always occur at or global minimum does not exist in (a , b ).
critical points of f ( x ) within [a, b ] or at the end points
of the interval.
Chap 08 Monotonicity, Maxima and Minima 479

y Example 47 Let f ( x ) = 2x 3 – 9 x 2 + 12x + 6. Discuss the Thus, x = 2 is the point of global minima in (1, 3) and global
maxima does not exist in (1, 3).
global maxima and global minima of f ( x ) in (1, 3).
Sol. Given, f ( x ) = 2x 3 – 9 x 2 + 12x + 6 Remark
Based on the above discussion, we can summarize things in a
Þ f ¢( x ) = 6x 2 – 18x + 12 Þ f ¢( x ) = 6 ( x – 1) ( x – 2) single graph as given below.
Put f ¢( x ) = 0 Þ x = 1, 2
Y Global max.
\ f (1) = 11 and f (2) = 10 ...(i) f′=0 d
ec local max.
re
Let us consider the open interval (1, 3). local max as
i f′ does not exist
f ′< ng

g
f′=0

s in
Clearly, x = 2 is the only point in (1, 3).

0
f′ 0

rea
de < 0

f′ >
0

de
′ >
f (2) = 10 f cre f′=0 ea

inc

cr
And [from Eq. (i)] as sin
in local min g
g f′<
Now, lim f ( x ) = lim f (1 + h ) = lim 2(1 + h )3 - 9(1 + h )2 f′ =0 0 Global
x ® 1+ h ®0 h ®0 local min min
+12 (1 + h ) + 6 = 11 X
0 a x1 x2 x3 x4 x5 b
and lim – f ( x ) = lim f (3 - h ) = lim 2(3 - h )3 - 9(3 - h )2
x ®3 h ®0 h ®0 Figure 8.21
+12(3 - h ) + 6 = 15

Exercise for Session 4


1. The minimum value of x x is attained when x is equal to
(a) e (b) e -1 (c) 1 (d) e 2

2. The function f is defined by f ( x ) = x p (1 - x )q for all x ÎR, where p, q are positive integers, has a maximum
value, for x is equal to
pq p
(a) (b) 1 (c) 0 (d)
p+q p+q

3. The least area of the circle circumscribing any right triangle of area S is
(a) pS (b) 2pS (c) 2pS (d) 4pS

4. The coordinates of the point on the curve x = 4y , which is atleast distance from the line y = x - 4 is
2

(a) (2, 1) (b) (- 2, 1)


(c) (- 2, -1) (d) none of these

The largest area of a rectangle which has one side on the x-axis and the two vertices on the curve y = e - x is
2
5.
(a) 2 e -1/ 2 (b) 2 e -1/ 2
(c) e -1/ 2 (d) none of these
px
6. Let f ( x ) = ln (2x - x 2 ) + sin . Then,
2
(a) graph of f is symmetrical about the line x = 1 (b) graph of f is symmetrical about the line x = 2
(c) maximum value of f is 1 (d) minimum value of f does not exist

7. The sum of the legs of a right triangle is 9 cm. When the triangle rotates about one of the the legs, a cone
results which has the maximum volume. Then,
(a) slant height of such a cone is 3 5 (b) maximum volume of the cone is 32 p
(c) curved surface of the cone is 18 5 p (d) semi vertical angle of cone is tan-1 2
2 2
8. Least value of the function f ( x ) = 2x - 1 + 2
is
2x + 1
3 2
(a) 0 (b) (c) (d) 1
2 3
480 Textbook of Differential Calculus

9. The greatest and the least value of the function, f ( x ) = 1 - 2x + x 2 - 1 + 2x + x 2 , x Î ( - ¥, ¥ ) are


(a) 2, - 2 (b) 2,- 1
(c) 2, 0 (d) none

10. The minimum value of the polynomial x ( x + 1) ( x + 2) ( x + 3) is


9
(a) 0 (b)
16
3
(c) - 1 (d) -
2
1 1
11. The difference between the greatest and least values of the functions, f ( x ) = cos x + cos 2x - cos 3x is
2 3
4
(a) (b) 1
3
9 1
(c) (d)
4 6

12. The point at which the slope of the tangent of the function, f ( x ) = e x × cos x attains minima, when x Î[0, 2p ] is
p p
(a) (b)
4 2
3p
(c) (d) p
4

13. If l, m are real numbers such that, x 3 - lx 2 + mx - 6 = 0 has its real roots and positive, then the minimum value
of m is
(a) 3 (6)1/ 3 (b) 3 (6)2 / 3
(c) (6)1/ 3 (d) (6)2 / 3
x
14. The points for which the function f ( x ) = ò { 2 (t - 1) (t - 2)3 + 3 (t - 1)2(t - 2)2} dt attains maxima and minima, is
1

7
(a) maximum when x = and minimum when x = 1 (b) maximum when x = 1and minimum when x = 0
5
7
(c) maximum when x = 1and minimum when x = 2 (d) maximum when x = 1and minimum when x =
5
ax 3
15. The set of values of ‘a’ for which the function f ( x ) = + (a + 2)x 2 + (a - 1)x + 2 possess a negative point of
3
inflection.
(a) (- ¥, - 2) È (0, ¥) (b) {- 4 / 5 }
(c) (- 2, 0) (d) empty set
Session 5
Maxima and Minima of Discontinuous Functions
Maxima and Minima of from all the four above mentioned cases for minimum of
Discontinuous Functions discontinuous functions, we have
f (a ) £ f (a + h ) and f (a ) £ f (a – h )
In discontinuous functions we don’t apply any of the
methods discussed earlier but we observe certain
y Example 48 Discuss the minima of f ( x ) = { x },
conditions and their graphs which would give you more
clear picture. (where {.} denotes the fractional part of x) for x = 6.
Sol. As we have discussed for discontinuous functions, mini-
mum at x = a is attained when,
Minimum of Discontinuous Functions f (a ) £ f (a + h ) and f (a ) £ f (a – h ) Þ f ( x ) is minimum at
Let f ( x ) be a function discontinuous or not differentiable x =a
at x = a, then the following four cases arise for minimum Here, f ( x ) = { x } is discontinuous function at x = 6
at x = a where, f (6) = 0
f (6 + h ) > 0 and f (6 – h ) > 0
Case (i) From figure 8.22 So, f (6) < f (6 + h ) and f (6) < f (6 – h )
f (a ) < f (a + h ) f (a ) < f (a – h ) Þ f ( x ) is minimum at x = 6.
Case (ii) From figure 8.23 \ f ( x ) attains local minima for x =6.
f (a ) < f (a + h ) f (a ) < f (a – h ) ì| x – 2| + a 2 – 9a – 9, if x < 2
y Example 49 Let f ( x ) = í .
Y Y î 2x – 3, if x ³ 2
f (a – h ) f (a+h) Then, find the value of ‘a’ for which f ( x ) has local
minimum at x = 2.
f (a)
ì| x – 2| + a 2 – 9a – 9, if x < 2
Sol. We have, f ( x ) = í
f (a – h ) f (a + h ) î 2x – 3, if x ³ 2
f (a )
f ( x ) has local minima at x = 2.
X X
O a–h a a+h O a–h aa+h Since, f ( x ) = 2x – 3 for x ³ 2 [is strictly increasing]
Figure 8.22 Figure 8.23 \ lim – f ( x ) ³ f (2) or lim f (2–h ) ³ f (2)
x ®2 h ®0

Case (iii) From figure 8.24 Þ lim {| 2 – h – 2| + a – 9a – 9 } ³ 1 [Q f (2) = 2 ´ 2 - 3 = 1]


2
h ®0
f (a ) < f (a + h ) f (a ) £ f (a – h ) Þ a 2 – 9a – 10 ³ 0
Case (iv) From figure 8.25 Þ (a + 1)(a – 10) ³ 0
f (a ) £ f (a + h ) f (a ) < f (a – h ) + – +
–1 10
Y Y
Þ a £ –1 or a ³ 10

Maximum of Discontinuous Functions


f (a + h ) f (a – h)
f (a) f (a) Let f ( x ) be a function discontinuous or not differentiable
f (a – h ) f (a + h)
at x = a, then the following four cases arise for maximum
X X at x = a.
O a–h aa+h O a–h aa+h
(i) From figure 8.26 (ii) From figure 8.27
Figure 8.24 Figure 8.25 f (a ) > f (a + h ) f (a ) > f (a + h )
f (a ) > f (a – h ) f (a ) > f (a – h )
482 Textbook of Differential Calculus

Y Y
Y f (a ) Y
f (a)
f (a + h )
f (a – h ) f (a + h)
f (a ) f (a )
f (a – h ) f (a – h )
f (a – h ) f (a + h) f (a + h )

X X
X X O a–h aa+h O a–h aa+h
O a–ha a+h O a–haa+h
Figure 8.32 Figure 8.33
Figure 8.26 Figure 8.27
(v) f (a ) < f (a – h ) (vi) f (a ) < f (a – h )
(iii) From figure 8.28 (iv) From figure 8.29
f (a ) > f (a + h ) f (a ) ³ f (a + h )
f (a ) > f (a + h ) f (a ) ³ f (a + h )
Y Y
f (a ) ³ f (a – h ) f (a ) > f (a – h )
Y Y f (a – h)
f (a – h)
f (a) f (a)
f (a ) f (a )
f (a + h)
f (a – h) f (a + h) f (a + h)
f (a + h ) f (a – h )

X X
O a–h aa+h O a–h aa+h
Figure 8.34 Figure 8.35

O a–h a a+h
X
O a–h a a+h
X In all above cases no extremum exist.
Figure 8.28 Figure 8.29 ì 6, x £ 1
y Example 50 Let f ( x ) = í , then for f ( x ) at
From all the above four mentioned cases for maximum of î7 – x , x > 1
discontinuous functions, we have x = 1 discuss maxima and minima.
f (a ) ³ f (a + h ), f (a ) ³ f (a – h ) ì 6, x £ 1
Sol. Here, f ( x ) = í
î7 – x , x > 1
Neither Maximum Nor Minimum for Y

Discontinuous Functions 6
Let f ( x ) be a function discontinuous or not differentiable 7–x
at x = a, then the following cases arise for neither
maximum nor minimum at x = a 0 7
X
1
(i) f (a ) < f (a – h ) (ii) f (a ) £ f (a + h )
Clearly, f ( x ) is not differentiable at x = 1
f (a ) ³ f (a + h ) f (a ) > f (a – h )
Þ f ( 1) = 6 Þ f ( 1 – h ) = 6
Y Y and f (1 + h ) = 7 – (1 + h ) = 6 – h < 6
Thus, x = 1 is a point of maxima.
f (a – h )
f (a ) f (a) f (a + h)
f (a + h)
f (a – h )
y Example 51 Find the values of ‘a’ for which,
ì4 x – x 3 + log (a 2 – 3a + 3), 0 £ x < 3
f (x ) = í
î x – 18, x³3
O a – haa + h
X
O a – haa + h
X has a local maxima at x = 3 .
ì 4 x - x 3 + log(a 2 - 3a + 3), 0 £ x < 3
Figure 8.30 Figure 8.31 Sol. Given, f ( x ) = í
î x - 18, x ³3
(iii) f (a ) < f (a + h ) (iv) f (a ) £ f (a – h )
Since, function attains maxima at x = 3
f (a ) ³ f (a – h ) f (a ) > f (a + h ) Þ f ( 3) ³ f ( 3 – 0)
Þ –15 ³ 12 – 27 + log (a 2 – 3a + 3)
Chap 08 Monotonicity, Maxima and Minima 483

+ – +
Þ log (a 2 – 3a + 3) £ 0 α β
Where for ‘log’ to exists, Here, x = a, will be point of local maxima and x = b
a 2 – 3a + 3 > 0 and log (a 2 – 3a + 3) £ 0 will be point of local minima.
Þ 0 < a 2 – 3a + 3 £ 1 Case III If D = 0, f ¢( x ) = 3a( x - a ) 2 , where a is a root of
i.e. ( a – 2) ( a – 1) £ 0 f ¢( x ) = 0, then f ( x ) = a( x - a ) 3 + K . If K = 0, then
Using number line rule, we have f ( x ) = 0 has three equal real roots. if K ¹ 0, then
+ – + f ( x ) = 0 has one real root.
1 2
i.e. 1£a£2
Graphical Representation of
\ f ( x ) attains local maxima at x = 3, when a Î[1, 2]. Roots of Cubic Polynomials
Y Y

Nature of Roots of Cubic Polynomials f(a)>0


f(a)>0
Let f ( x ) = ax + bx + cx + d be the given cubic
3 2

polynomial and f ( x ) = 0 be the corresponding cubic (i) (ii) f(b)>0


equations, where a, b, c , d Î R and a > 0. b
a X a X
b
Now, f ¢( x ) = 3ax + 2bx + c
2
f(b)<0
f(x) has three distinct roots f(x) has one real root
Let D = 4b 2 - 12ac = 4(b 2 - 3ac ) be the discriminant of the Figure 8.37 Figure 8.38
equations f ¢( x ) = 0.
Y
Now, we have the following three cases. Y

Case I If D < 0 Þ f '( x ) > 0, "x Î R, i.e. f ( x ) would be an f(a)>0


f(a)<0
increasing function of x. Also, lim f ( x ) = ¥ and (iii)
a b (iv)
x ®¥
lim f ( x ) = -¥. X
x ®– ¥ f(b)=0
Since, f ( x ) is an increasing function, so it will f(b)<0 ab
X
intersect the X-axis at only once. f(x) has one real root f(x) has three real roots
Y (b being repeated)
Figure 8.39
(0, d) Figure 8.40
Y

(v)
X′ X a b
x0 O x0 X
O

f(a)>0 f(b)<0
(0, –d)
f(x) has three real roots
Y′ (a being repeated)
Figure 8.36 Figure 8.41
Clearly, x 0 > 0 if d < 0 and x 0 < 0, if d > 0 Conclusions
Case II If D > 0, f ¢( x ) = 0 would have two real roots (say) (i) If f (a ) × f (b ) < 0, then f ( x ) has three distinct roots
a and b (where a < b) (see figure 8.37).
Þ f ¢( x ) = 3a( x - a )( x - b) (ii) If f (a ) × f (b ) > 0, then f ( x ) has one real and two
imaginary roots (see figure 8.38 and 8.39).
ì f '( x ) < 0, x Î (a, b)
ï (iii) If f (a ) × f (b ) = 0, then f ( x ) has three roots but two
Þ í f '( x ) > 0, x Î ( -¥, a ) È (b, ¥) roots are identical (see figure 8.40 and 8.41).
ï f '( x ) = 0, x Î {a, b}
î
484 Textbook of Differential Calculus

y Example 52 If 4 x 3 - 3x - p = 0, where -1 £ p £ 1 has 1


Þ q = cos -1 ( p )
3
é1 ù
unique root in ê , 1ú, then the root is æ1 ö
ë2 û [IIT JEE 2005] \ Root is cos ç cos –1 p ÷
è3 ø
-1
(b) cos æç cos -1 p ö÷
cos p 1
(a) Hence, (b) is the correct answer.
3 è3 ø
-1 y Example 53 The number of distinct real roots of
(c) cos (cos p ) (d) None of these
Sol. Given that, -1 £ p £ 1
x 4 - 4 x 3 + 12x 2 + x - 1 = 0 [IIT JEE 2011]
Sol. Given, f ( x ) = x 4 - 4 x 3 + 12x 2 + x - 1
Let f ( x ) = 4 x 3 - 3x - p
Þ f ¢( x ) = 4 x 3 - 12x 2 + 24 x + 1
æ1ö 1 3
Now, f ç ÷ = - - p = -1 - p £ 0 [Q p ³ -1]
è2ø 2 2 Þ f ¢¢( x ) = 12x 2 - 24 x + 24
and f ( 1) = 4 - 3 - p = 1 - p ³ 0 [Q p £ 1] Þ f ¢¢( x ) = 12( x 2 - 2x + 2)
é1 ù Þ f ¢¢( x ) = 12(( x - 1)2 + 1) Þ f ¢¢( x ) > 0
\ f ( x ) has atleast one real root between ê , 1ú.
ë2 û Þ f ¢( x ) is an increasing cubic function.
To find a root we observe f ( x ) contains 4 x 3 - 3x , which is Þ f ¢( x ) has only one real root and two imaginary root.
multiple angle formula for cos3q.
\ f ( x ) cannot have all distinct roots, atmost two real roots.
\ We put x = cos q , then
Now, f ( -1) = 15, f (0) = -1, f (1) = 9
4 cos 3 q - 3 cos q - p = 0 \ f ( x ) must have one root in (–1, 0) and other in (0,1).
Þ p = cos 3 q Hence, there are 2 real roots.

Exercise for Session 5


ì x 3 - x 2 + 10x - 5, x £ 1
1. Let f ( x ) = í the set of values of b for which f ( x ) have greatest value at x = 1is
î - 2x + log2 (b - 2), x > 1
2

given by
(a) 1 < b £ 2 (b) b = {12
, } (c) b Î(- ¥, - 1) (d) [- 130, - 2 ) È ( 2, 130 ]
2. Number of solution(s) satisfying the equations, 3x 2 - 2x 3 = log2 ( x 2 + 1) - log2 x is
(a) 1 (b) 2 (c) 3 (d) None of these
3. Let f ( x ) = cos 2px + x - [ x ]([. ] denotes the greatest integer function). Then, number of points in [0, 10] at which
f ( x ) assumes its local maximum value, is
(a) 0 (b) 10 (c) 9 (d) infinite
4. If f ( x ) = | x | + | x - 1| + | x - 2 |, then
(a) f (x ) has minima at x = 1 (b) f (x ) has maxima at x = 0
(c) f (x ) has neither maxima nor minima at x = 3 (d) None of these
p
5. The function f ( x ) = 1 + [cos x ] x , in 0 < x £
2
(a) has a minimum value 0 (b) has a maximum value 2
p p
(c) is continuous in éê 0, ùú (d) is not differentiable at x =
ë 2û 2
6. If lim f ( x ) = lim [f ( x )]([. ] denotes the greater integer function) and f ( x ) is non-constant continuous function,
x ®a x ®a
then
(a) lim f (x ) is irrational (b) lim f (x ) is non-integer
x ®a x ®a
(c) f (x ) has local maxima at x = a (d) f (x ) has local minima at x = a
7. Find the value of a if x 3 - 3x + a = 0 has three distinct real roots.

8. Prove that there exist exactly three non- similar isosceles DABC such that tan A + tan B + tan C = 100.
JEE Type Solved Examples :
Single Option Correct Type Questions
l Ex. 1 The values of ‘K ’ for which the point of minimum (c) when a = - 2 Þ b < -
1
and when a = 3 Þ b < -
11
of the function f ( x ) = 1 + K 2 x - x 3 satisfy the inequality 2 27
(d) None of the above
(x 2 + x + 2)
< 0, belongs Sol. Given, f ( x ) = a 2 x 3 - 05
. ax 2 - 2x - b
( x 2 + 5x + 6) Þ f ¢ ( x ) = 3a x - ax - 2
2 2

(a) ( -3 3 , ¥ ) For extrema, f ¢( x ) = 0


(b) ( -3 3 , - 2 3 ) È (0, ¥ ) 1
Þ 3a 2 x 2 - ax - 2 = 0 at x =
(c) ( -3 3 , - 2 3 ) È ( 2 3 , 3 3 ) 3
(d) (0, ¥ ) é 1 ù
êë as at x = 3 function is minimum úû
2
æ 1ö 7
çx + ÷ +
x + x +2
2
è 2 ø 4
Sol. Here, 2 <0 Þ <0 2
x + 5x + 6 ( x + 2) ( x + 3) æ1ö æ1ö a2 a
\ 3a 2 ç ÷ - a ç ÷ -2=0 Þ - -2=0
+ – + è3ø è3ø 3 3
–3 –2 Þ a 2 - a - 6 = 0 or a = - 2 , 3
2
æ 1ö 7
where ç x + ÷ + is always positive. So, their arise two cases as
è 2ø 4 Case I At a = 3, if function attains minimum and is
\ Using number line rule for ( x + 2) ( x + 3) as shown positive.
3 2
above, we get æ1ö æ1ö æ1ö
Then, 9 ç ÷ - (0.5) (3) ç ÷ - 2 ç ÷ - b > 0
x2 + x + 2 è3ø è3ø è3ø
< 0 Þ -3 < x < - 2 …(i)
x 2 + 5x + 6 é ù
1 æ1ö
Now, consider f ( x ) = 1 + K 2 x - x 3 êQ minimum at x = 3 and a = 3 Þ f çè 3 ÷ø > 0 when a = 3ú
ë û
f ¢( x ) = K 2 - 3x 2 , f ¢¢( x ) = - 6x 1 15 . 2 1
Þ b< - - Þ b<-
For maximum/minimum let f ¢( x ) = 0, 3 9 3 2
|K | Case II At a = - 2, if function attains minimum and is
Þ x=± positive.
3
3 2
|K | |K | æ1ö æ1ö æ1ö
Let x1 = and x 2 = - Then, ( -2)2 ç ÷ - (0.5) ( -2) ç ÷ - 2 ç ÷ - b > 0
3 3 è3ø è3ø è3ø
\ f ¢¢( x 1 ) < 0 and f ¢¢( x 2 ) > 0 é 1 ù
Þ f ( x )is maximum at x = x 1 and f ( x )is minimum at x = x 2 . êsince minimum at x = 3 , when a = - 2ú
ê ú
\ -3 < x 2 < - 2
ê Þ f æç 1 ö÷ > 0, when a = - 2 ú
-| K | êë è3ø úû
Þ -3 < < -2 [from Eq. (i)]
3 4 1 2 11
Þ 3 3 > |K | > 2 3 Þ b< + - or b < -
27 9 3 27
Þ K Î ( - 3 3, - 2 3 ) È ( 2 3, 3 3 ) 1 11
\ When a = 3 Þ b < - and when a = - 2 Þ b < -
Hence, (c) is the correct answer. 2 27
Hence, (a) is the correct answer.
l Ex. 2 The values of a and b for which all the extrema of
the function, f ( x ) = a 2 x 3 - 0.5 ax 2 - 2 x - b, is positive and l Ex. 3 If f ¢¢( x ) + f ¢( x ) + f 2 ( x ) = x 2 is the differential
the minimum is at the point x 0 = , are
1 equation of a curve and let P be the point of maxima, then
3 number of tangents which can be drawn from P to
11 1
(a) when a = - 2 Þ b < - and when a = 3 Þ b < - x 2 - y 2 = a 2 is/are
27 2
11 1 (a) 2 (b) 1
(b) when a = 3 Þ b < - and when a = 2 Þ b < -
27 2 (c) 0 (d) either 1 or 2
486 Textbook of Differential Calculus

Sol. At point of maxima f ¢( x ) = 0 and f ¢¢( x ) < 0 l Ex. 6 Let g:[1, 6 ] ® [0, ¥) be a real valued differentiable
Þ f ¢¢( x ) = x 2 - f 2 ( x ) £ 0 2
function satisfying g ¢ ( x ) = and g(1) = 0, then the
(Since, the curve is x 2 - y 2 = a 2 and x 2 - f 2 ( x ) £ 0 x + g( x )
\x 12 - y12 < a 2 Þ point lies outside the hyperbola) maximum value of g cannot exceed
(a) log 2 (b) log6 (c) 6 log 2 (d) 2log 6
Þ Point P ( x , f ( x )) lies outside x 2 - y 2 = a 2
2
Sol. Here, g ¢ ( x ) = > 0, for all x Î[1, 6]
\ Two tangents can be drawn. x + g(x )
Hence, (a) is the correct answer.
Þ g ( x ) is increasing function on [1, 6]
x æ t 2 + 2 t + 1ö 62
Ex. 4 Let f ( x ) = ò
6
l cos ç ÷ dt, 0 < x < 2, then \ ò1 g ¢ ( x ) dx £ ò1 x dx
f (x )
0
è 5 ø
(a) increases monotonically Þ ( g ( x ))16 £ 2 (loge x )16
(b) decreasing monotonically Þ g (6) - g (1) £ 2 log 6
(c) has one point of local maximum \ g(6) £ 2 log 6, as g(1) = 0
(d) has one point of local minima Þ g cannot exceed 2 log 6 .
æ x 2 + 2x + 1 ö æ ( x + 1) 2 ö Hence, (d) is the correct answer.
Sol. We have, f ¢( x ) = cos ç ÷ ´ 1 = cos ç ÷
è 5 ø è 5 ø
l Ex. 7 The minimum value of the function,
( x + 1) 2 æ 1ö
Since, 0 < x < 2 Þ 1 /5 < > 9 /5 f ( x ) = x 3 / 2 + x -3 / 2 - 4 ç x + ÷ . for all permissible real
5 è xø
æ ( x + 1) 2 ö values of x, is
Now, cos ç ÷ =0
è 5 ø (a) -10 (b) -6 (c) -7 (d) -8
( x + 1) 2
p æ 1 ö
Þ = Sol. Here, f ( x ) = x 3 / 2 + x -3 / 2 - 4 ç x + ÷
è xø
5 2
f ¢ ( x ) > 0, if 0 < x < ( 5p /2 - 1) æ 1 ö
3
æ 1 ö ææ 1 ö
2 ö
f (x ) = ç x + ÷ - 3ç x + ÷ - 4 çç ç x + ÷ - 2÷÷
and f ¢ ( x ) < 0, if ( 5p /2 - 1) < x < 2 è xø è xø èè xø ø
\ x = ( 5p /2 - 1) is a point of local maxima Let x+
1
= t, [ x > 0]
x
Hence, (c) is the correct answer.
\ g (t ) = t 3 - 3t - 4(t 2 - 2)
l Ex. 5 As ‘x’ ranges over the interval (0, ¥), the function Now, g (t ) = t 3 - 4t 2 - 3t + 8, where t Î [2, ¥ )

f ( x ) = 9 x 2 + 173 x + 900 - 9 x 2 + 77 x + 900 , ranges over g ¢ (t ) = 3t 2 - 8t - 3 = (t - 3)(3t + 1)


(a) (0, 4) (b) (0, 8) g ¢ (t ) = 0 Þ t = 3 (t ¹ -1 /3)
(c) (0, 12) (d) (0, 16) g ¢¢(t ) = 6t - 8
Þ g ¢¢(3) = 10 > 0
Sol. Q 9 x 2 + 173x + 900 > 9 x 2 + 77 x + 900, " x Î (0, ¥ ) \ Minimum value is
Þ f ( x ) > 0, " x Î (0, ¥ ) g(3) = 27 - 36 - 9 + 8 = -10
96 x Hence, (a) is the correct answer.
Now, f (x ) =
9 x 2 + 173x + 900 + 9 x 2 + 77 x + 900
[rationalise]
l Ex. 8 The tangent to the curve y = 1 - x 2 at x = a , where
96 0 < a < 1, meets the axes at P and Q. If a varies and the
\ lim f ( x ) = lim
x ®¥ x ®¥ 173 900 77 900 minimum value of the area of the DOPQ is k times the area
9+ + 2 + 9+ + 2
x x x x bounded by the axis and the part of the curve for 0 < x < 1,
96 then k is equal to
= = 16 (a) 2 / 3 (b) 75 /16 (c) 25 /18 (d) 2 /3
6
\ Range of f ( x ) Î (0, 16) Sol. Here, equation of tangent
Hence, (d) is the correct answer. y + 2ax = a 2 + 1 …(i)
Chap 08 Monotonicity, Maxima and Minima 487

From Eq. (i) intersect the axes at f ¢ ( x ) = 3x 2 - 4 x = 0


æa 2 + 1 ö 4
Pç , 0÷ and Q(0, a 2 + 1) Þ x = 0,
è 2a ø 3
Y 4
\ f ( x ) is minimum at x =
Q
3
æ4ö 32
(α, 1–α2) f ç ÷ >0 Þ a>
è3ø 27
P Hence, (b) is the correct answer.
X
–1 O
1 æ pö
l Ex. 10 If k sin 2 x + cosec 2 x = 2 , x Î ç 0, ÷ ,
k è 2ø
then cos x + 5 sin x cos x + 6 sin x is equal to
2 2

1 ( a 2 + 1) 2 ( a 2 + 1) 2 k 2 + 5k + 6 k 2 - 5k + 6
\ Area of DOPQ = × = (a) (b)
2 2a 4a k2 k2
1 (c) 6 (d) None of these
f (a ) = (a 3 + 2a + 1 /a ) 1
4 Sol. Given, k sin x +
2
=2
k sin 2 x
1 1 (3a + 2a - 1) 4 2
f ¢ (a ) = (3a 2 + 2 - 1 /a 2 ) = 2
4 4 a2 æ 1 ö
Þ ç k sin x - ÷ =0
1 (a 2 + 1)(3a 2 - 1) è k sin x ø
= ×
4 a2 1
Þ sin 2 x =
1 (a + 1)( 3a - 1)( 3a + 1)
2 k
=
4 a2 So, cos 2 x + 5sin x cos x + 6sin 2 x
Using number line rule, we have k -1 5 k -1 6
= + +
+ – + k k k
–1/√3 1/√3 k +5 + 5 k - 1
=
\ Minimum when a = 1 / 3 k
Thus, minimum area of triangle = 4 /3 3 Hence, (d) is the correct answer.
1
4 1 æ x 3 ö 2k Ex. 11 The least value of the expression
= k ò (1 - x ) dx = k ç x -
l
÷ =
2
Given,
3 3 0 è 3 ø0 3 x 2 + 4y 2 + 3 z 2 - 2 x - 12y - 6 z + 14 is
Þ k = 2/ 3 (a) 0
(b) 1
l Ex. 9 Least natural number a for which (c) no least value
x + ax -2 > 2 , " x Î[0 , ¥) is (d) None of the above
(a) 1 (b) 2 (c) 5 (d) None of these Sol. Let f ( x , y , z ) = x 2 + 4y 2 + 3z 2 - 2x - 12y - 6z + 14

Sol. Here, x + ax -2 > 2 = ( x - 1)2 + (2y - 3)2 + 3(z - 1)2 + 1


Þ x 3 - 2x 2 + a > 0 For least value of f ( x , y , z )
x - 1 = 0, 2y - 3 = 0 and z - 1 = 0
Let f ( x ) = x 3 - 2x 2 + a
3
Since, f ( x ) > 0, " x Î [0, ¥ ), f (0) > 0 \ x = 1, y = , z = 1
2
and min f ( x ) > 0
æ 3 ö
Þ a > 0 and for minimum f ( x ) Hence, least value of f ( x , y , z ) is f ç1, , 1÷ = 1
è 2 ø
– +
Hence, (b) is the correct answer.
4/3
488 Textbook of Differential Calculus

é 5p 4p ù Þ
2 1
= Þp -1=8 Þ p =9
l Ex. 12 On the interval ê , ú , the least value of the p -1 4
ë 4 3 û
x Hence, (d) is the correct answer.
function f ( x ) = ò (3 sin t + 4 cos t ) dt is
5p / 4
3 1 3 1 l Ex. 15 The coordinates of the point on the curve
(a) + -2 3 (b) - +2 3
2 2 2 2 x = y ( x - a ) 2 , a > 0, where the ordinate is minimum
3

3
(c) -
1
-2 3 æ - 8a ö
(d) None of these (a) ( 2a, 8a ) (b) ç - 2a, ÷
2 2 è 9 ø
x é 5p 4 p ù æ 27a ö æ - 27a ö
Sol. Given, f (x ) = ò5 p/ 4 (3 sin t + 4 cos t ) dt , x Îê ,
ë 4 3 úû
. (c) ç3a,
è 4 ø
÷ (d) ç - 3a,
è 16 ø
÷

f ¢ ( x ) = 3sin x + 4 cos x Sol. The ordinates of any point on the curve is given by
é 5p 4 p ù
f ¢ ( x ) < 0 as sin x, cos x are negative for x Î ê , . x3
ë 4 3 úû y=
æ 4p ö 4 p/ 3 ( x - a )2
Þ f ( x ) | min = f ç ÷ = ò (3sin t + 4 cos t ) dt
è 3 ø 5 p/ 4 dy x 2 ( x - 3a )
=
3 1 dx ( x - a )3
= + -2 3
2 2 dy
Now, =0
Hence, (a) is the correct answer. dx
Þ x = 0 or x = 3a
l Ex. 13 For any real q, the maximum value of é d 2y ù é d 2y ù 72a 5
cos 2 (cos q ) + sin 2 (sin q ) is ê 2ú = 0 and ê 2 ú = >0
ë dx û x =0 ë dx û x = 3a
(2a )6
(a) 1 (b) 1 + sin2 1 (c) 1 + cos 2 1 (d) does not exist
27a
Sol. The maximum value of cos 2 (cos q ) is 1 and that of So, y is minimum at x = 3a and is equal to ×
4
p Hence, (c) is the correct answer.
sin 2 (sin q ) is sin 2 1, both exists for q = ×
2
So, maximum value is 1 + sin 2 1. lEx. 16 If x 2 + y 2 + z 2 = 1 for x , y , z Î R, then the
Hence, (b) is the correct answer. maximum value of x 3 + y 3 + z 3 - 3 xyz is
1
l Ex. 14 If sin q + cos q =1, then the minimum value of (a)
2
(b) 1 (c) 2 (d) 3

(1 + cosec q ) (1 + sec q ) is 1
Sol. Let t = xy + yz + zx , so - £t £1
(a) 3 (b) 4 (c) 6 (d) 9 2
Sol. We know that, AM ³ GM \ x 3 + y 3 + z 3 - 3xyz
sin q + cos q 1
³ sin q cos q Þ sin q cos q £ = ( x + y + z ) ( x 2 + y 2 + z 2 - xy - yz - zx )
2 4
Now, let sin q = x , cos q = y = ( 1 + 2t ) ( 1 - t )
and (1 + cosec q ) (1 + sec q ) ³ p Let f (t ) = (1 + 2t ) (1 - t )2
æ 1ö æ 1ö f ¢ (t ) = 6t (t - 1) = 0
Þ ç1 + ÷ ç1 + ÷ ³ p
è xø è yø
+ – +
æ1 + x ö æ1 + y ö 0 1
Þ ç ÷ç ÷³p
è x øè y ø Clearly, t max = f (0) = 1
Þ xy + x + y + 1 ³ pxy Hence, (b) is the correct answer.
Þ x + y + 1 ³ ( p - 1) xy
Þ 2 ³ ( p - 1) xy
l Ex. 17 If a , b Î R distinct numbers satisfying
[since, x + y = 1] | a - 1 | + | b - 1 | = | a | + | b | = | a + 1 | + | b + 1 |, then the
2 minimum value of | a - b | is
Þ xy £
p -1 (a) 3 (b) 0
(c) 1 (d) 2
Chap 08 Monotonicity, Maxima and Minima 489

Sol. Let a < b and f ( x ) = | x - a | + | x - b |, " x Î R (a 2 - b 2 ) cos q


So, f ( x ) is decreasing in ( - ¥, a ] constant in [a, b ] and
l Ex. 20 If a > b > 0 and f (q ) = , then the
a - b sin q
increasing in [b, ¥ ), we have
maximum value of f (q ), is
f ( 0) = f ( 1) = f ( - 1)
Þ { - 1, 0, 1} Î [a, b ] (a) 2 a 2 + b 2 (b) a 2 + b 2
\ | a - b | min = 2 (c) a 2 - b 2 (d) b 2 - a 2
Hence, (d) is the correct answer.
(a 2 - b 2 ) cos q (a 2 - b 2 )
Sol. Here, f (q ) = =
l Ex. 18 Let f ( x ) = x 4 + ax 3 + 3 x 2 + bx + 1, a , b Î R. If a - b sin q asec q - b tan q
f ( x ) ³ 0, " x Î R, then the maximum value of a 2 + b 2 is a2 - b2
or f (q ) = , where h (q ) = a sec q - b tan q
equal to h (q )
(a) 10 (b) 12 \ f (q ) is maximum and minimum as h(q ) is minimum and
(c) 16 (d) None of these maximum respectively.
Sol. Given, f ( x ) = x 4 + ax 3 + 3x 2 + bx + 1 and f ( x ) ³ 0 Þ h (q ) = a sec q - b tan q
Þ h ¢(q ) = sec q (a tan q - b sec q )
æ a ö
2
æ a2 + b2 ö 2 æ b ö
2
Þ çx 2 + x ÷ + ç3 - ÷ x + ç x + 1÷ ³ 0 For maximum and minimum put h ¢ (q ) = 0
è 2 ø è 4 ø è2 ø
b
Þ sinq = [sec q ¹ 0] …(i)
a2 + b2 a
which holds only when 3 - ³0
4 Also, h ¢¢(q ) = a sec 3q + a sec q × tan 2 q - 2b sec 2 q tan q
Þ a + b £ 12
2 2
a + a sin 2 q - 2b sin q
or h ¢¢(q ) =
Hence, (b) is the correct answer. cos 3 q
b
l Ex. 19 The maximum value of When sin q =
a
( -3 + 4 x - x 2 + 4 ) 2 + ( x - 5 ) 2 (where 1 £ x £ 3 ) is a + a×
b2
- 2b ×
b
a2 a2 - b2
(a) 34 (b) 36 (h ¢¢(q )) = 3/2
a =
3/2
>0
(c) 32 (d) 20 æ b2 ö æ a2 - b2 ö
ç1 - 2 ÷ aç ÷
Sol. Here, ( -3 + 4 x - x 2 + 4 )2 + ( x - 5)2 , represents the è a ø è a2 ø
[ as a > b ]
square of the distance between circle y = -3 + 4 x - x 2
b
and point (5, - 4 ). Þ h(q ) is minimum, when sinq =
a
Clearly, the centre and radius of the circle respectively are \ f (q ) is maximum, when
(2, 0) and 1.
b
i.e. Maximum distance between x 2 + y 2 - 4 x + 3 = 0 and sin q = Þ f max (q ) = a 2 - b 2
a
(5, - 4 ), Hence, (c) is the correct answer.
l Ex. 21 A solid cylinder of height H has a conical portion
of same height and radius 1/3rd of height removed from it.
Q
C r =1 P Rain water is accumulating in it, at the rate equal to p times
(2, 0) (5, – 4) the instantaneous radius of the water surface inside the hole,
the time after which hole will filled with water is
H2 H2 H2
(a) (b) H 2 (c) (d)
3 4 6
Þ PQ 2 = ( PC + radius )2 H x y
Sol. Here radius, r = , = Þ 3x = y Þ
dv
= px
3 r H dt
= ( ( 5 - 2) 2 + ( 4 - 0) 2 + 1) 2
d æ1 2 ö r t
Þ ç px y ÷ = px Þ 3 ò 0 x dx = ò 0 dt
= 62 = 36 dt è 3 ø
Hence, (b) is the correct answer. r2 3 H2 H2
Þ =t Þ ×
3 =t Þ t =
2 2 9 6
Hence, (c) is the correct answer.
490 Textbook of Differential Calculus

l Ex. 22 If composite function f 1 ( f 2 ( f 3 (....( f n ( x ))))) n Sol. r must be an even integer because two decreasing are
required to make it increasing function.
times, is an increasing function and if r of f i ’s are decreasing
Let y = r (n - r ),
function while rest are increasing, then maximum value When n is odd
of function is n -1 n +1
r = or for maximum value of y
n2 - 1 2 2
(a) when n is an even number When n is even
4 n
r = for maximum value of y
n2 2
(b) when n is an odd number
n2 - 1 n2
4 \ Maximum (y ) = , when n is odd and
n2 - 1 4 4
(c) when n is an odd number
4 When n is even.
(d) None of the above Hence, (c) is the correct answer.

JEE Type Solved Examples :


More than One Correct Option Type Questions
l Ex. 23 Let f ( x ) = sin x + ax + b. Then, f ( x ) = 0 has lEx. 25 Let f ( x , y ) = x 2 + 2 xy + 3y 2 - 6 x - 2y ,
(a) only one real root which is positive, if a > 1, b < 0 where x , y Î R, then
(b) only one real root which is negative, if a > 1, b > 0 (a) f ( x , y ) ³ - 11 (b) f ( x , y ) ³ - 10
(c) only one real root which is negative, if a < - 1, b < 0 (c) f ( x , y ) > - 11 (d) f ( x , y ) > - 12
(d) None of the above Sol. Let z = x 2 + 2xy + 3y 2 - 6x - 2y
Sol. Given, f ( x ) = sin x + ax + b
Þ x 2 + 2xy + 3y 2 - 6x - 2y - z = 0 as x Î R
Þ f ¢ ( x ) = - cos x + a, if a > 1, then f ( x ) is entirely
\ D ³0
increasing. So, f ( x ) = 0 has only one real root, which is
positive, if f ( x ) < 0 and negative, if f (0) > 0. Similarly, Þ 4 (y - 3) - 4(3y 2 - 2y - z ) ³ 0
2

when a < - 1 then, f ( x ) entirely decreasing. Therefore, Þ y 2 + 9 - 6y - 3y 2 + 2y + z ³ 0


f ( x ) has only one real root which is positive, if f (0) < 0 and
Þ - 2y 2 - 4y + 9 + z ³ 0
negative, if f (0) > 0.
Hence, (a), (b) and (c) are the correct answers. Þ z ³ 2 (y 2 + 2y + 1) - 11 = 2 (y + 1)2 - 11
Þ z ³ - 11
l Ex. 24 If a > 0, b > 0, c > 0 and a + b + c = abc , then atleast Hence, (a), (c) and (d) are the correct answers.
one of the numbers a , b, c exceeds
æp ö
3 17 l Ex. 26 Let g ( x ) = f (tan x ) + f (cot x ), " x Î ç , p ÷ . If
è2 ø
(a) (b)
2 10
13 æp ö
(c) 2 (d) f ¢ ¢ ( x ) < 0, " x Î ç , p ÷ , then
10 è2 ø
Sol. We may suppose a ³ b ³ c æ p 3p ö
(a) g ( x ) is increasing in ç , ÷
abc = a + b + c ³ 3c è2 4 ø
17 3p
So, ab ³ 3, a ³ b and a ³ 3 > (b) g ( x ) has local minimum at x =
10 4
Aliter (a + b + c )3 ³ 27 abc ³ 27 (a + b + c ) æ3p ö
(c) g ( x ) is decreasing in ç , p ÷
è 4 ø
\ a+b+c ³3 3 3p
(d) g ( x ) has local maximum at x =
Þ Atleast one of them ³ 3 4
Hence, (a), (b) and (d) are the correct answers. Sol. Given, g ( x ) = f (tan x ) + f (cot x )
g ¢ ( x ) = f ¢ (tan x ) sec 2 x - f ¢ (cot x ) cosec 2 x
Chap 08 Monotonicity, Maxima and Minima 491

For increasing g ¢ ( x ) > 0 l Ex. 29 Let F ( x ) = 1 + f ( x ) + ( f ( x )) 2 + ( f ( x )) 3 , where


f ¢ (tan x ) > f ¢ (cot x ) f ( x ) is an increasing differentiable function and F ( x ) = 0 has
æp pö a positive root, then
[Q f ¢ ¢ ( x ) < 0 and tan x < cot x , " x ç , ÷ ]
è4 2 ø (a) F ( x ) is an increasing function (b) F (0) £ 0
æ p 3p ö (c) f (0) £ - 1 (d) F ¢ (0) > 0
Also, sec x > cosec x , x Î ç , ÷
2 2
è2 4 ø Sol. Given, F ( x ) = 1 + f ( x ) + ( f ( x ))2 + ( f ( x ))3
æ p 3p ö
g ¢ ( x ) > 0 Þ g ( x ) is increasing in ç , ÷ F ¢ ( x ) = (1 + 2 f ( x ) + 3 ( f ( x ))2 ) f ¢ ( x ) > 0, so F ( x ) is
è2 4 ø
increasing.
æ 3p ö
Similarly, g ( x ) is decreasing in ç , p ÷ So, F(0) < 0 Þ (1 + f (0)) (1 + f (0))2 ) < 0 Þ f (0) < - 1
è 4 ø
3p Hence, (a), (b), (c) and (d) are the correct answers.
Also, g ( x ) has local maximum at x = × Hence, (a), (c) and
4
(d) are the correct answers.
l Ex. 30 The extremum values of the function
1 1
f (x ) = - , where x Î R is
l Ex. 27 The function f ( x ) = ò
x
1 - t 4 dt is such that sin x + 4 cos x - 4
0
4 2 2 2 2 4 2
(a) it is defined on the interval [ - 1, 1] (a) (b) (c) (d)
8- 2 8- 2 4 2 +1 8+ 2
(b) it is an increasing function
1 1
(c) it is an odd function Sol. Given, f (x ) = -
sin x + 4 cos x - 4
(d) the point (0, 0) is the point of inflection
- cos x sin x
Sol. f ¢ ( x ) = 1 - x 4 > 0 in ( - 1, 1) Þ f is increasing Þ f ¢( x ) = +
(sin x + 4 ) 2
(cos x - 4 )2
x -x
Now, f ( x ) + f ( - x ) = ò0 1 - t 4 dt + ò0 1 - t 4 dt Þ f ¢ ( x ) = 0 Þ (sin x + cos x ) (non-zero quantity)
= 0 Þ tan x = - 1
1 - t 4 dt + æç - 1 - y 4 dy ö÷ (t = - y ) = 0
x x
Þ ò0 è ò0 ø Þ x=
3p
or
7p
4 4
Þ f ( x ) is odd
- 4x 3 æ 3p ö
Again, f ¢ ¢( x ) = which vanished at x = 0 Global minimum = x = 2np + ç ÷
è 4 ø
2 1 - x4
æ7p ö
and changes sign Þ (0, 0) is point of inflection since, f is Global maximum = x = 2np + ç ÷
è 4 ø
well defined in [ - 1, 1].
4 4 2 2
Hence, (a), (b), (c) and (d) are the correct answers. M= ,m = =
8- 2 8+ 2 4 2 +1
sin( x + a ) Hence, (a) and (c) are the correct answers.
l Ex. 28 The function has no maxima or
sin( x + b )
minima, if
l Ex. 31 The function f ( x ) = x 1/3 ( x - 1)
(a) b - a = np, n Î I (b) b - a = ( 2n + 1) p, n Î I (a) has 2 inflection points
(c) b - a = 2np, n Î I (d) None of these (b) is strictly increasing for x >
1
and strictly decreasing
sin( x + a ) 4
Sol. Given, f ( x ) = 1
sin ( x + b ) for x <
4
sin ( x + b ) ´ cos ( x + a ) - sin ( x + a ) cos ( x + b ) æ 1 ö
f ¢( x ) = (c) is concave down in ç - , 0÷
sin 2 ( x + b ) è 2 ø
sin(b - a )
= (d) area enclosed by the curve lying in the fourth quadrant
sin 2 ( x + b ) is
9
If sin(b - a ) = 0, then f ¢ ( x ) = 0 28
Þ f ( x ) will be constant. Sol. y = x 1/ 3 ( x - 1)
i.e. b - a = np or b - a = (2n + 1) p dy 4 1/ 3 1 1 1
or b - a = 2np, = x - × 2 / 3 = 2 / 3 [ 4 x - 1]
dx 3 3 x 3x
then f ( x ) has no minima. 1
Hence, (a), (b) and (c) are the correct answers. x 2/3 is always positive and x = has a local minima.
4
492 Textbook of Differential Calculus

1 1
Hence, f is increasing for x > é3 3 ù 3 3 4 -7 9
4 = ê x 3/7 - x 4/3 ú = - =3 =
1 ë7 4 û0 7 4 28 28
and f is decreasing for x <
4 Hence, (a), (b), (c) and (d) are the correct answers.
4 1/ 3 1 -2 / 3
Now, f ¢( x ) = x - × x
3 3 l Ex. 32 Assume that inverse of the function f is denoted
[non-existence at x = 0, vertical tangent] by g, then which of the following statement hold good?
4 1 1 2 1 2 é 1ù (a) If f is increasing, then g is also increasing
f ¢ ¢( x ) = × 2/ 3 + × × 5/ 3 = 2/3 ê
2+ ú (b) If f is decreasing, then g is increasing
9 x 3 3 x 9x ë xû
(c) The function f is injective
2 é 2x + 1 ù
= 2/3 êë x úû
(d) The function g is onto
9x Sol. If f and g are inverse, then ( fog ) ( x ) = x
\ f ¢ ¢ ( x ) = 0 at x = -
1
[inflection point] f ¢ [ g ( x )]g ¢ ( x ) = 1
2 If f is increasing Þ f ¢ > 0 Þ Sign of g ¢ is also positive.
Y Therefore, option (a) is correct.
If f is decreasing Þ f ¢ < 0 Þ Sign of g ¢ is negative.
Therefore, option (b) is false.
Since, f has an inverse.
Þ f is bijective Þ f is injective
1/4 Therefore, option (c) is correct.
X
0 Inverse of a bijective mapping is bijective.
1/2 (1, 0)
Þ g is also bijective Þ g is onto
Therefore, option (d) is correct.
1 Hence, (a), (c) and (d) are the correct answers
Graph of f ( x ) is as A = ò0 (x - x 1/ 3 ) dx
4/3

JEE Type Solved Examples :


Statements I and II Type Questions
n
Directions (Ex. Nos. 33 to 39) For the following Þ y=x
questions, choose the correct answers from the codes (a), k k
(b), (c) and (d) defined as follows x +y = x + = f ( x ), f ¢ ( x ) = 1 - 2 = 0
x x
(a) Statement I is true, Statement II is also true, Statement II
is the correct explanation of Statement I. Þ x = k,y = k
(b) Statement I is true, Statement II is also true, Statement II
So, Statement II is true and it explains Statement I.
is not the correct explanation of Statement I
(c) Statement I is true, Statement II is false Hence, (a) is the correct answer.
(d) Statement I is false, Statement II is true
l Ex. 34 Statement I The function f ( x ) = ( x 3 + 3 x - 4 )
l Ex. 33 Statement I Among all the rectangles of the given ( x 2 + 4 x - 5 ) has local extremum at x =1.
perimeter, the square has the largest area. Also, among all the Statement II f ( x ) is continuous and differentiable and
rectangles of given area, the square has the least perimeter.
f ¢ (1) = 0 .
Statement II For x > 0, y > 0, if x + y = constant, then xy
Sol. Statement I is correct because f (1)- > f (1) < f (1)+ .
will be maximum for y = x and if xy = constant, then x + y
Statement II is correct as f ( x ) has a repeated root at x = 1.
will be minimum for y = x .
Statement II is not the correct explanation of Statement I as
Sol. Statement II x + y = k
f ¢ (c ) = 0 doesn’t imply that f has an extrema at x = c .
then xy = x (k - x ) = f ( x ), f ¢ ( x ) = k - 2x = 0
Hence, (b) is the correct answer.
k k
Þ x = ,y =
2 2
Chap 08 Monotonicity, Maxima and Minima 493

l Ex. 35 Statement I If f ( x ) is increasing function with l Ex. 37 Let f : R ® R is differentiable and strictly increas-
upward concavity, then concavity of f -1 ( x ) is also upwards. ing function throughout its domain.
Statement II If f ( x ) is decreasing function with upwards Statement I If | f ( x ) | is also strictly increasing function,
concavity, then concavity of f -1 ( x ) is also upwards. then f ( x ) = 0 has no real roots.
Sol. Let g ( x ) be the inverse function of f ( x ). Statement II At ¥ or - ¥, f ( x ) may approach to 0, but
Then, f ( g ( x )) = x cannot be equal to zero.
\ f ¢ ( g ( x )) × g ¢ ( x ) = 1 Sol. Suppose f ( x ) = 0 has a real root say x = a, then f ( x ) < 0
1 for x < a. Thus, | f ( x ) | becomes strictly decreasing on
i.e. g ¢( x ) = ( - ¥, a ), which is a contradiction.
f ¢ ( g ( x ))
Hence, (a) is the correct answer.
1
\ g ¢¢ = - × f ¢¢ ( g ( x )) × g ¢ ( x )
( f ¢ ( g ( x )))2 l Ex. 38 Statement I f ( x ) = x + cos x is strictly increasing.
In Statement I f ¢¢( g ( x )) > 0
Statement II If f ( x ) is strictly increasing, then f ¢ ( x ) may
and g ¢( x ) > 0
tend to zero at some finite number of points.
Þ g ¢¢ ( x ) < 0
-1
Sol. Given, f ( x ) = x + cos x
Þ Concavity of f ( x ) is downwards.
\ f ¢ ( x ) = 1 - sin x > 0, " x Î R,
\ Statement I is false. p
except at x = 2np +
In Statement II f ¢¢( g ( x )) > 0 2
and g ¢( x ) < 0 p
and f ¢ ( x ) = 0 at x = 2np +
Þ g ¢¢( x ) > 0 2
-1
Þ Concavity of f ( x ) is upwards. \ f ( x ) is strictly increasing.
\ Statement II is true. Statement II is true but does not explain Statement I.
Hence, (d) is the correct answer. Statement II gives f ¢( x ) may tend to zero at finite number
of points but in Statement I f ¢( x ) tend to zero at infinite
number of points.
l Ex. 36 Statement I The minimum distance of the fixed
1 Hence, (b) is the correct answer.
point (0, y 0 ), where 0 £ y 0 £ , from the curve y = x 2 is y 0 .
2 l Ex. 39 Statement I The largest term in the sequence
Statement II Maxima and minima of a function is always
a root of the equation f ¢ ( x ) = 0. n2 ( 400 ) 2 /3
an = , n Î N is
Sol. Let the point on the parabola be (t , t 2 ). n 3 + 200 600
x2
Let d be the distance between (t , t 2 ) and (0, y 0 ), Statement II f ( x ) = , x > 0, then at x = ( 400 ) 1/3 ,
f ( x ) is maximum. x + 200
3
then d 2 = t 2 + (t 2 - y 0 )2 = t 4 + (1 - 2y 0 ) t 2 + y 02
x2
= z 2 + (1 - 2y 0 ) z + y 02 , z ³ 0 Sol. Statement II Given, f ( x ) =
x + 200
3
1
Its vertex is at x = y 0 - <0
2 ( x + 200) 2x - 3x 2 x 2
3
- x 4 + 400 x
f ¢( x ) = =
\ The minimum value of d 2 is at z = 0, i.e. t 2 = 0 ( x 3 + 200)2 ( x 3 + 200)2
4002 / 3
\ d = y0 x ® 0+ f ( x ) = 0+ Þ x = 4001/ 3 f (x ) =
600
Statement I is true. Statement II is false because extremum x ® ¥ f (x ) ® 0
can occur at a point where f ¢( x ) does not exist.
So, Statement II is true. But Statement I is false as x Î N .
Hence, (c) is the correct answer. Hence, (d) is the correct answer.
494 Textbook of Differential Calculus

JEE Type Solved Examples :


Passage Based Questions
Passage I Passage II
(Ex. Nos. 40 to 42) (Ex. Nos. 43 to 45)
Let x 1 , x 2 , x 3 , x 4 be the roots (real or complex) of the Consider a D OAB formed by the point
equation x 4 + ax 3 + bx 2 + cx + d = 0. If x 1 + x 2 = x 3 + x 4 O (0, 0 ), A( 2, 0 ), B(1, 3 ). P ( x , y ) is an arbitrary interior point
and a , b, c , d Î R, then of triangle moving in such a way that
d (P , OA ) + d (P , AB ) + d (P , OB ) = 3,
l Ex. 40 If a = 2, then the value of b - c is where d (P , OA ), d (P , AB ), d (P , OB ) represent the distance of
(a) - 1 (b) 1 (c) - 2 (d) 2 P from the sides OA, AB and OB , respectively.

lEx. 41 If b < 0, then how many different values of a, we l Ex. 43 Area of region representing all possible position of
may have point P is equal to
(a) 3 (b) 2 (c) 1 (d) 0 (a) 2 3 (b) 6 (c) 3 (d) None of these

l Ex. 42 If b + c =1 and a ¹ - 2, then for real values of a , c Î


l Ex. 44 If the point P moves in such a way that
d (P , OA ) £ min (d (P , OB ), d (P , AB )), then area of region
æ 1ö
(a) ç - ¥, ÷ (b) ( - ¥, 3 ) representing all possible position of point P is equal to
è 4ø 1 1
(c) ( - ¥, 1) (d) ( - ¥, 4 ) (a) 3 (b) 6 (c) (d)
3 6
n
Sol. (Ex. Nos. 40 to 42)
l Ex. 45 If the point P moves in such a way that
Let x 4 + ax 3 + bx 2 + cx + d
= ( x - x1 ) ( x - x 2 ) ( x - x 3 ) ( x - x 4 ) d (P , OA ) ³ min (d (P , OB ), d (P , AB )), then area of region
Let ( x - x 1 ) ( x - x 2 ) = x 2 + px + q
representing all possible position of point P is equal to
1
and ( x - x 3 ) ( x - x 4 ) = x 2 + px + r (a) 3 (b) 6 (c) 1/ 3 (d)
6
\ q = x 1x 2 and r = x 3 x 4
x 4 + ax 3 + bx 2 + cx + d n
Sol. (Ex. Nos. 43 to 45)
= x + 2px + ( p + q + r ) x + p (q + r ) x + qr
4 3 2 2 43. DOAB is clearly equilateral
\ a = 2p , b = p + q + r , c = p (q + r ), d = qr
2
D OAB = D OPA + D APB + D OPB =
3 1
´ 4 = ×2
Clearly, a - 4ab + 8c = 0
3
…(i) 4 2
(d ( P , OA ) + ( P , AB ) + ( P , OB ))
40. If a = 2 Þ b - c = 1 4
d ( P , OA ) + d ( P , AB ) + d ( P , OB ) =
Hence, (b) is the correct answer. 3
41. Investigating the nature of the cubic equation of a. Hence, (c) is the correct answer.
Let f (a ) = a 3 - 4ab + 8c 44. We must have, d ( P , OA ) £ d ( P , OB ) as well as
f ¢ (a ) = 3a 2 - 4b d ( P , OA ) £ d ( P , AB ), then P lies either on or below the
If b < 0 Þ f ¢ (a ) > 0 angle bisector of Ð BOA and Ð BAO area
1 1
\The equation a 3 - 4ab + 8c = 0, has only one real root. = D OAB = ×
3 3
Hence, (c) is the correct answer.
Hence, (c) is the correct asnwer.
42. Substituting c = 1 - b in Eq. (i), we have
45. We must have d ( P , OA ) ³ d ( P , OB ) as well as
(a + 2) [(a - 1)2 + 3 - 4b ] = 0 d ( P , OA ) ³ d ( P , AB ), then P must be above bisector of
Þ 4b - 3 > 0 Ð BOA and Ð BAO.
3 1 1 1
Þ b> Þ c < × Area of triangle = D OAB =
4 4 3 3
Hence, (a) is the correct answer. Hence, (c) is the correct answer.
Chap 08 Monotonicity, Maxima and Minima 495

Passage III l Ex. 49 The function x 4 - 4 x + 1 will have


(Ex. Nos. 46 to 48) (a) absolute maximum value
Let f ( x ) = ax + bx + c , a , b, c Î R. It is given
2 (b) absolute minimum value
| f ( x ) | £ 1, | x | £ 1 (c) both absolute maximum and minimum values
(d) None of the above
8
lEx. 46 The possible value of | a + c |, if a 2 + 2b 2 is
3 l Ex. 50 The absolute minimum value of the function
maximum, is given by
x -2
(a) 1 (b) 0 (c) 2 (d) 3 is
x 2 +1
8
lEx. 47 The possible value of | a + b |, if a 2 + 2b 2 is (a) - 1 (b)
1
(c) - 5 (d) None of these
maximum, is given by 3 2
(a) 1 (b) 0 (c) 2 (d) 3
l Ex. 51 The absolute minimum and maximum values of
8
l Ex. 48 The possible maximum value of a 2 + 2b 2 is x 2 - x +1
3 the function is
given by x 2 + x +1
32
(a) 32 (b) 1
3 (a) 1 and 3 (b) and 3
2 16 2
(c) (d) 1
3 3 (c) and 3 (d) None of these
3
n Sol. (Ex. Nos. 46 to 48)
We know that for | u | £ 1 ; | v | £ 1, then | u - v | £ 2
4 9
l Ex. 52 The function f ( x ) = - will
Now, | f ( 1) - f ( 0) | £ 2 Þ | a + b | £ 2 x -1 x +1
Þ (a + b )2 £ 4
1
…(i) (a) have absolute maximum value -
2
Also, | f ( - 1) - f ( 0) | £ 2 Þ | a - b | £ 2 25
(b) have absolute minimum value -
Þ (a - b )2 £ 4 …(ii) 2
25 1
Now, 4a + 3b = 2(a + b ) + 2(a - b ) - b £ 16
2 2 2 2 2 (c) not lie between - and -
2 2
Þ ( 4a 2 + 3b 2 )max = 16 when b = 0 (d) always be negative
Þ |a + b | = |a - b | = |a | = 2
l Ex. 53 Which of the following functions will have abso-
Also, | f (1) - f (0) | = |(a + c ) - c | = | a | = 2
lute minimum value?
Þ |a + c | = |c | = 1
(a) cot (sin x ) (b) tan (log x )
The possible ordered triplet (a, b, c ) are (2, 0, - 1) or
( - 2, 0, 1). (c) x 2005
-x 1947
+1 (d) x 2006 + x 1947 + 1
8 2 2 2
Also, a + 2b 2 = ( 4a 2 + 3b 2 ) £ + 16 n
Sol. (Ex. Nos. 49 to 53)
3 3 3
49. Since, f ¢ ( x ) = 4 x 3 - 4 = 0 Þ x = 1
Ans. 46. (a) 47. (c) 48. (b)
Þ There is only one extrema which is minima.
Passage IV Þ 1 is a point of absolute minima.
(Ex. Nos. 49 to 53) Hence, (b) is the correct answer.
1 + 2x
The absolute maximum and minimum values of functions 50. Since, f ¢ ( x ) = 2
can be found by their monotonic and asymptotic behaviour ( x + 1) 3 / 2
provided they exist. We may agree that finite limiting values æ 1 ö
Þ f ( x ) is increasing in ç - , ¥ ÷ and decreasing in
may be regarded as absolute maximum or minimum. For è 2 ø
1 æ 1ö
instance the absolute maximum value of is unity. It is ç - ¥, - ÷
1+ x2 è 2ø
1
attained at x = 0 while absolute minimum value of the same Þ Absolute minimum occurs at x = - , we have
function is zero which is a limiting value of (x ® ¥ or 2
=- 5
x ® - ¥)
Hence, (c) is the correct answer.
496 Textbook of Differential Calculus

51. yx 2 + yx + y = x 2 - x + 1 x
Þ ò - ¥ f (t ) = cf ( x ) …(iii)
x 2 (1 - y ) - x (1 + y ) + (1 - y ) = 0 Q D ³ 0
0 1 1
Þ (1 + y )2 - 4 (1 - y )2 ³ 0 Þ - (3y - 1) (y - 3) ³ 0 As, f ( 0) = 1 Þ ò- ¥ f (t ) dt = 2 = c ´ 1 Þ c = 2
1
Þ £y £3 x 1
3 Þ ò- ¥ f (t ) dt = 2 f ( x ), differentiating both the sides and
Hence, (c) is the correct answer. on integrating and using boundary condition, we get
52. Should be the correct choice (we can prove by using f ( x ) = e 2 x , y = 2ex is tangent to y = e 2 x
25 1
monotonically that f cannot lie between - and - × This Þ Number of solutions = 1
2 2
is an Ex. of a function whose maximum (local) value is Clearly, f ( x ) is increasing for all x.
smaller than minimum value). -2 x
lim (e 2 x )e =1 [¥ 0 form]
Hence, (c) is the correct answer. x ®¥

53. As even degree polynomial will have absolute minimum Ans. 54. (b) 55. (c) 56. (a)
essentially.
Hence, (d) is the correct answer. Passage VI
(Ex. Nos. 57 to 59)
Passage V x
æ 1ö
(Ex. Nos. 54 to 56) Let f ( x ) = ç1 + ÷ ( x > 0 ) and
x
è xø
We are given the curves y = ò f (t ) dt through the point
-¥ é x ln (1 + (1 / x )), if 0 < x £ 1
g( x ) = ê
æ 1ö ë if x = 0
ç 0, ÷ and y = f ( x ), where f ( x ) > 0 and f ( x ) is
0,
è 2ø
differentiable, " x Î R through (0, 1). If tangents drawn to l Ex. 57 lim g ( x )
both the curves at the points with equal abscissae intersect x ® 0+
on the same point on the X-axis, then (a) is equal to 0 (b) is equal to 1
(c) is equal to e (d) is non-existent
l Ex. 54 Number of solutions f ( x ) = 2ex is equal to
(a) 0 (b) 1
l Ex. 58 The function f
(c) 2 (d) None of these (a) has a maxima but not minima
(b) has a minima but not maxima
l Ex. 55 lim ( f ( x )) f ( - x ) is (c) has both of maxima and minima
x ®¥ (d) is a monotonic
(a) 3 (b) 6
ì æ1ö æ n öü
1/n
(c) 1 (d) None of these æ 2ö æ3ö
l Ex. 59 lim í f ç ÷ × fç ÷× f ç ÷ .... f ç ÷ý equals
n ® ¥ î ènø ènø ènø è n øþ
l Ex. 56 The function f ( x ) is
(a) 2 e (b) 2e
(a) increasing for all x (b) non-monotonic
(c) decreasing for all x (d) None of these (c) 2 e (d) e

n
Sol. (Ex. Nos. 54 to 56) n
Sol. (Ex. Nos. 57 to 59)
We have, the equations of the tangents to the curves æ x + 1ö
ln ç ÷
x æ 1ö è x ø é¥ ù
y= ò- ¥ f (t ) dt and y = f ( x ) at arbitrary points on them are 57. lim x ln ç1 + ÷ = lim
x ® 0+ è x ø x ® 0+ 1 êë ¥ form úû
x
Y - ò- ¥ f (t ) dt = f ( x ) ( X - x) …(i) x
Using L’Hospital’s rule,
and Y - f ( x ) = f ¢( x ) ( X - X ) …(ii)
æ 1 1ö æ1 1 ö 2
As Eqs. (i) and (ii), intersect at the same point on X -axis. l = lim - ç - ÷ x 2 = lim ç - ÷ ×x
x ®0 èx + 1 x ø x ®0 èx x + 1ø
On putting Y = 0 and equating x-coordinates, we have
1 x
x
= lim × x 2 = lim =0
x-
f (x )
=x-
ò-¥ f (t ) dt Þ
f (x )
=
f ¢( x ) x ®0 x ( x + 1) x ® 0 ( x + 1)
f ¢( x ) x
f (x )
ò- ¥ f (t ) dt f (x ) Hence, (a) is the correct answer.
Chap 08 Monotonicity, Maxima and Minima 497

Y
58. lim f ( x ) = 1 (can be verified) l Ex. 62 If the sum of the base 2 logarithms of the roots of
x ®0
lim f ( x ) = e e
the cubic f ( x ) = 0 is 5, then the value of a is
x ®¥
Also, f is increasing for all (a) - 64 (b) - 8 (c) - 128 (d) - 256
(0, 1)
X
x >0 O n Sol. (Ex. Nos. 60 to 62)
Þ (d) (can be verified) 60. a = 1
Hence, (d) is the correct answer.
f ( x ) = 8x 3 + 4 x 2 + 2bx + 1
1/n
æ æ1ö æ2ö æn öö
59. Given, lim ç f ç ÷ f ç ÷ ... f ç ÷ ÷ f ¢ ( x ) = 24 x 2 + 8x + 2b = 2 (12x 2 + 4 x + b )
n ®¥ è èn ø èn ø èn øø
For increasing function f ¢ ( x ) ³ 0, " x Î R
1/n
æ n æ n ök /n ö \ D £0
l = çç P ç1 + ÷ ÷÷ 1
èk = 1è k ø ø Þ 16 - 48b £ 0 Þ b ³
3
k /n
æ nö Hence, (c) is the correct answer.
[given f ( x ) = (1 + 1 / x )x and f (k /n ) = ç1 + ÷ ]
è kø 61. If b = 1
Taking log, f ( x ) = 8x 3 + 4ax 2 + 2x + a
k /n
1 n
æ nö
ln l = lim × S ln ç1 + ÷ f ¢ ( x ) = 24 x 2 + 8ax + 2 or 2(12x 2 + 4ax + 1)
n ®¥ n k = 1 è kø
For non-monotonic f ¢ ( x ) = 0 must have distinct roots
1 n k æ 1 ö
= lim × S ln ç1 + ÷
hence,
n ® ¥ n k = 1n è k /n ø D > 0 , i.e. 16a 2 - 48 > 0
1
1 æ 1ö é æ 1 ö x2 ù Þ a2 > 3
= ò0 x ln ç1 + ÷ dx = ê ln
{ è xø
ç1 + ÷ × ú
è xø 2 û0 \ a > 3 or a < - 3
II 1424 3 ë
I \ a Î2, 3, 4, ...,
1æ 11 ö x2
+òç - ÷ × dx Sum = 5050 - 1 = 5049
0èx x + 1ø 2
Hence, (b) is the correct answer.
æ1 ö 1 1x + 1 - 1
= ç ln 2 - 0÷ + ò dx 62. If x 1, x 2 and x 3 are the roots, then
è2 ø 2 0 x +1
log 2 x 1 + log 2 x 2 + log 2 x 3 = 5
1 1
= ln 2 + [ x - ln ( x + 1)] 10 log 2 ( x 1x 2 x 3 ) = 5
2 2
1 1 1 x 1x 2 x 3 = 32
= ln 2 + [(1 - ln 2) - 0] = ; l = e a
2 2 2 - = 32 Þ a = - 256
Hence, (d) is the correct answer. 8
Hence, (d) is the correct answer.
Passage VII
(Ex. Nos. 60 to 62) Passage VIII
(Ex. Nos. 63 to 65)
Consider the cubic f ( x ) = 8 x 3 + 4ax 2 + 2bx + a, where
a , b Î R. ìmax {t - t 2 + t + 1, 0 £ t £ x }, 0 £ x £ 1
3
Let f ( x ) = í and
îmin {3 - t , 1 < t £ x }, 1 < x £ 2
l Ex. 60 For a =1, if y = f ( x ) is strictly increasing, " x Î R,
ìmax {3 /8 t 4 + 1/ 2 t 3 - 3 / 2 t 2 + 1, 0 £ t £ x }, 0 £ x £1
then maximum range of the values of b is g( x ) = í
îmin {3 /8 t + 1/32 sin p t + 5 /8, 1 £ t £ x }, 1 £ x £ 2
2
æ 1ù æ1 ö
(a) ç - ¥, ú (b) ç , ¥ ÷
è 3û è3 ø Define f ( x ) and g ( x ) expilicitly and then answer the
é1 ö following questions.
(c) ê , ¥ ÷ (d) ( - ¥, ¥ )
ë3 ø
l Ex. 63 The function f ( x ), " x Î[0, 2 ] is
l Ex. 61 For b =1, if y = f ( x ) is non-monotonic, then the (a) continuous and differentiable
sum of the integral values of a Î[1, 100 ], is (b) continuous but not differentiable
(a) 4950 (b) 5049 (c) discontinuous and not differentiable
(c) 5050 (d) 5047 (d) None of the above
498 Textbook of Differential Calculus

l Ex. 64 Which of the following is true. Passage IX


(a) lim ( fog )( x ) > lim ( gof )( x ) (Ex. Nos. 66 to 69)
x ® 1- x ® 1+
The graph of derivative of a function f ( x ) is given (i.e.
(b) lim ( fog )( x ) < lim ( gof )( x )
x ®1-
x ®1 + y = f ¢ ( x )). Analyse the graph in the given domain and
answer the following questions, if it is given that f (0 ) = 0
(c) lim ( fog )( x ) = lim ( gof )( x )
x ® 1- x ® 1+
Y
(d) None of the above
–a o a
X
l Ex. 65 Let z ( x ) =
d g d
f ( x ) ( x ) and y ( x ) = g( x ) f (x )
,
dx dx
then z ( x ) and y ( x ) vanish simultaneously at
(a) x = -1/3 (b) x = 0
(c) x = 1 (d) No real value of x l Ex. 66 The function f ¢ ( x ) is
n Sol. (Ex. Nos. 63 to 65) (a) even function (b) odd function
Consider f ( x ) in [0, 1] (c) neither even nor odd (d) indefinite

f ¢ (t ) = 3t 2 - 2t + 1 > 0, " t Î (0, 1) l Ex. 67 The function f ( x ) is


Þ f (t ) is increasing on (0, 1) (a) even function (b) odd function
(c) neither even nor odd (d) indefinite
Maximum occurs at t = x
ì x 3 - x 2 + x + 1, 0 £ x £ 1
l Ex. 68 The graph of y = f ( x ) has
and f (x ) = í
î 3 - x, 1 < x £ 2 (a) no inflexion point (b) one point of inflexion
(c) one extreme point (d) two extreme points
Again, consider g ( x ) in (0, 1)
l Ex. 69 The function f ( x ) for -a £ x £ a, is
g ( t ) = 3 / 8t 4 + 1 / 2t 3 - 3 / 2t 2 + 1
(a) always decreasing
g ¢ (t ) = 3 /2t 3 + 3 /2t 2 - 3t = 3 /2t (t 2 + t - 2) (b) always increasing
= 3 /2t (t - 1)(t + 2) (c) increasing for ( -a, 0) and decreasing for (0, a )
g (t ) decreases in [0, 1) Þ maximum occurs when t = 0 and (d) increasing for (0, a ) and decreasing for ( -a, 0)
g(0) = 1, again consider g ( x ) function in [1, 2]. n
Sol. (Ex. Nos 66 to 69)
g (t ) = 3 /8t + 1 /32sin pt + 5 /8
2
66. The graph of y = f ¢( x ) is symmetrical about Y -axis, so f ( x )
g ¢ (t ) = 3 /8 + p /32sin(2pt ) > 0, " t Î R is an even function.
\ g (t ) is an increasing function in [1, 2]. 67. f (0) = 0, so f ( x ) is an odd function (i.e. derivative of an odd
function is an even function)
Þ Minimum occurs when t = 1 and g(1) = 1
68. Here, f ¢¢( x ) > 0, as x < 0 and f ¢¢( x ) < 0, as x > 0
ì1, 0 £ x £ 1
Hence, g(x ) = í =1 \ x = 0, is the point of inflexion
î1, 1 < x £ 2 69. f ¢( x ) £ 0, " x
Þ g ( x ) = 1, " x Î [0, 2] So, f ( x ) is always decreasing.
63. f ( x ) is continuous but not differentiable at x = 1
Passage X
64. lim fog ( x ) = f (1) and lim gof ( x ) = 1, also f (1) > 1 (Ex. Nos. 70 to 72)
x ® 1- x ® 1+
d d
65. z ( x ) = f ( x )g ( x ) = ( f ( x ))1, " x Î [0, 1) È (1, 2] If a function (continuous and twice differentiable) is always
dx dx concave upward in an interval, then its graph lies always
= f ¢( x ), " x Î [0, 1) È (1, 2]
below the segment joining extremities of the graph in that
d
and y( x ) = ( g ( x )) f (x ) interval and vice-versa.
dx
d l Ex. 70 If sin x + x ³ | k | x 2 , " x Î[0, p / 2 ], then the great-
= (1) f (x ) = 0,
dx est value of k, is
" x Î[0, 1) È (1,2] -2( 2 + p ) 2( 2 + p )
(a) (b)
Hence, the functions y ( x ) and z ( x ) can vanish p 2
p2
simultaneously at f ¢ ( x ) = 0, which is not possible for any (c) can’t be determined finitely
real x. (d) zero
Chap 08 Monotonicity, Maxima and Minima 499

l Ex. 71 Let f ( x ), f ¢( x ) and f ¢¢( x ) are all positive, So, f ¢( x ) must have one root x = 1
-1
" x Î[0, 7 ]. If f ( x ) exists, then Y
-1 -1 -1
3f ( 4) - f (2) - 2 f (5 ), is
(a) always positive (b) always negative
(c) non-negative (d) non-positive

l Ex. 72 Let f : R + ® R + is such that f ¢¢( x ) ³ 0, X


b O
" x Î[a , b ], then value of ò f ( x )dx , cannot exceed x=α x=1 x=β
a
( f (a ) + f (b ))(b - a ) ( f (b ) - f (a ))(b - a ) \ f ¢ ( x ) = a ( x - 1)( x - a )( x - b )
(a) (b)
3 2 = a ( x - 1)[ x 2 - (a + b ) x + ab ]
( f (b ) + f (a ))(b - a )
(c) (d) None of these a +b
2 = a ( x - 1)[ x 2 - 2x + ab ], as =1
2
n Sol. (Ex. Nos. 70 to 72)
\ f ¢( x ) = a ( x 3 - 2x 2 + abx - x 2 + 2x - ab )
70. f ( x ) = sin x + x Þ f ¢( x ) = cos x + 1
f ¢¢( x ) = a (3x 2 - 6x + ab + 2)
Þ f ¢¢( x ) = - sin x
f ¢¢(2) = a (12 - 12 + ab + 2) = a(ab + 2) = 0
f is concave downward for x Î[0, p /2]
\ ab = -2 and a + b = 2
Irrespective of k, g ( x ) = | k | x 2 is concave upward
2
So, if g ( p /2) £ f ( p /2), then f ( x ) ³ g ( x ) Þ a - = 2 Þ a 2 - 2a - 2 = 0
a
p2 é ( - 2p + 4 ) ( 2p + 4 ) ù \ a = 1 - 3, b = 1 + 3
Þ 1 + p /2 ³ | k | Þ k Îê ,
4 ë p2 p 2 úû
So, sum = 1 + a + b = 3
71. f ¢¢( x ) > 0
-1 f ¢ ( x ) = a ( x - 1)( x 2 - 2x - 2) = a ( x - 1)[( x - 1)2 - 3]
Þ f is concave upwards Þ f is concave downwards
On integrating, we get
Consider point dividing the join of this segment in
æ 2 f -1(5) + f -1(2) ö é ( x - 1)4 3( x - 1)2 ù
ratio 2:1 is given as ç 4, ÷ when upon a f (x ) = a ê - ú +C
è 3 ø ë 4 2 û
point ( 4, f -1
( 4 )) on graph of f -1
( x ) is always above it. {Q f (1) = 0, so C = 0 and f (2) = 1, so a = -4 /5}
Þ 3f -1
(4) - f -1
( 2) - 2 f -1
( 5) > 0 é ( x - 1)4 3( x - 1)2 ù
Þ f ( x ) = - 4 / 5ê - ú
b
ë 4 2 û
72. òa f ( x )dx , is area bounded by curve in first quadrant.
( f (a ) + f (b ))(b - a ) 4 é 16 3 ù
is area of trapezium ABCD, which is f ( 3) = - ê - ( 2) 2 ú = 8 / 5
2 5ë 4 2 û
b
always more than or equal to ò f ( x )dx . Passage XII
a
(Ex. Nos. 75 to 76)
Passage XI
(Ex. Nos. 73 to 74) Let y = f ( x ) and y = g ( x ) be the two functions, then the
number of solutions of these two functions means the
Consider a twice differentiable function f ( x ) of degree four number of values of x for which these two function gives
symmetrical to line x =1, defined as f : R ® R and f ¢¢( 2 ) = 0, same of y as it is shown below.
then
Y
l Ex. 73 The sum of roots of the cubic, f ¢ ( x ) = 0, is f(x)
(a) 0 (b) 1 (c) 2 (d) 9/5
l Ex. 74 If f (1) = 0, f ( 2 ) = 1, then the value of f (3 ), is
(a) 6/7 (b) 7/5 (c) 8/5 (d) 9/5
n
Sol. (Ex. Nos. 73 to 74) x1 x2 x3 X
O
Since, f ( x ) is symmetric about x = 1 and it is twice
differentiable. g(x)
500 Textbook of Differential Calculus

l Ex. 75 Let max {| x + y |, | x - y | } =1 and y = x -[ x ] be So, graph is,


Y 1
two equations which x and y satisfy, then the number of
ordered pairs ( x , y ) is
(a) 6 (b) 8 (c) 0 (d) ¥ –1 0 1
X

ì1 ü –1
l Ex. 76 If f ( x ) = í ý and g ( x ) = { x 2 }, then the number of
îx þ Combining both graph,
positive roots satisfying the equations f ( x ) = g ( x ) such that when -1 £ x < 0, then both graph coincide.
2 < x2 <3 Y 2
1
(a) 1 (b) 0 (c) 3 (d) 2
n Sol. (Ex. Nos. 75 to 76) O
X
–2 –1 1 2
75. Here, y = x - [ x ] = { x } and we know graph of
fractional part of x. –1
–2
Y
So, number of solutions is infinite.
1 76. If 2 < x 2 < 3 Þ 2 < x < 3 [as x > 0]
1 1 1
So, if 2 < x < 3, then < <
X 3 x 2
–2 –1 0 1 2
ì ü
1 1
Þ í ý= ...(i)
îxþ x
Also, we have max {| x + y |, | x - y | } = 1 and {x 2 } = x 2 - 2 ...(ii)
If | x + y | ³ | x - y |, then | x + y | = 1 So, from Eqs. (i) and (ii), we get
On squaring above inequality, we get
ì 1ü
í ý = {x }
2
x 2 + y 2 + 2xy ³ x 2 + y 2 - 2xy
î þx
Þ 4 xy ³ 0, then | x + y | = 1, i.e. if function lies in I or 1
III quadrant, \ = x 2 - 2 Þ x 3 - 2x - 1 = 0
x
| x + y | = 1 Þ x + y = ±1 ...(i) Þ ( x + 1)( x 2 - x - 1) = 0
and of | x + y | £ | x - y |, then | x - y | = 1
1± 5
On squaring both sides 4 xy £ 0 means x and y lies in So, x = -1, , but as x > 0
2
II or IV quadrant. So, if curves lies in II or IV quadrant,
then 1+ 5 æ1 - 5 ö
| x - y | = 1 Þ x - y = ±1 ...(ii) \ x= ç and - 1 are neglected ÷
2 è 2 ø

JEE Type Solved Examples :


Matching Type Questions
l Ex. 77 Match the statements of Column I with values of Column II.
Column I Column II
æ pö (p) 0
(A) Number of the values of x lying in ç 0, ÷ at which f (x ) = ln (sin x ) is not monotonic, is
è 2ø
(B) If the greatest interval of decrease of the function f (x ) = x 3 - 3x + 2 is [ a, b ] , then a + b equals (q) 2

x2 + 2 (r) -3
(C) Let f (x ) = , 1 £ x £ 3, where [.] greatest integer function, then least value of f (x ) is
[x ]
(D) Set of all possible values of a such that f (x ) = e2x - (a + 1) ex + 2x is monotonically increasing for all x Î R is (s) 3
(- ¥ , a ], then a equals
Chap 08 Monotonicity, Maxima and Minima 501

Sol. (A) ® (p), (B) ® (p), (C) ® (s), (D) ® (s) (C) If x1 and x2 are abscissae of two points on (r) 4
cos x the curve f (x ) = x - x 2 in the interval
(A) f ( x ) = ln (sin x ) Þ f ¢ ( x ) = >0 [ 0, 1], then maximum value of expression
sin x
(x1 + x2 ) - (x12 + x22 ) is
\ Required number of values of x is 0.
(D) The number of non-zero integral values of a (s) 1
(B) f ¢ ( x ) = 3x 2 - 3 £ 0, if - 1 £ x £ 1 for which the function 2
\ a = - 1, b = 1 \ a + b = 0 3x 2
f (x ) = x 4 + ax 3 + + 1is concave
ì 2
ï x 2 + 2, 1 £ x < 2 upward along the entire real line is
ïï x 2 + 2 ì2x , 1 < x < 2
(C) f ( x ) = í , 2 £ x < 3 Þ f ¢( x ) = í Sol. (A) ® (p, q), (B) ® (r), (C) ® (s), (D) ® (r)
ï 2 2 î x, 2 < x < 3
ï x + 2 (A) Perimeter of the rectangle is 36 cm.
, x =3
ïî 3 If one side is x, then the other side = 18 - x
\Least value of f ( x ) in [1, 2] is 3. If the rectangle is revolved around the side x, then
Least value of f ( x ) in [2, 3) is 3. volume swept out V = px (18 - x )2
11 dV
f ( 3) = = p [(18 - x )2 - 2x (18 - x )]
3 dx
\Least value of f ( x ) is 3. = p (18 - x ) (18 - x - 2 x )
(D) f ( x ) = e 2 x - (a + 1) e x + 2x \ x = 6 and y = 12
f ¢ ( x ) = 2e 2 x - (a + 1) e x + 2 (B) A ( - 1, 2), B(2, 3) and P in a point on y = x . Perimeter of
Now, 2e 2 x - (a + 1)e x + 2 ³ 0, for all x Î R DPAB is minimum when PA + PB is minimum image of
A( - 1, 2) in the line y = x is A ¢ (2, - 1). Equation of A ¢ B
æ 1 ö
Þ 2 çe x + x ÷ - (a + 1) ³ 0, for all x Î R is x = 2, hence P is (2, 2).
è e ø
A′
Þ 4 - ( a + 1) ³ 0 Þ a £ 3 \ a = 3 B

y=x
lEx. 78 Match the statements of Column I with values of P
Column II.
A
Column I Column II (C) Let ( x 1, y1 ) and ( x 2 , y 2 ) are two points.
(A) The dimensions of the rectangle of (p) 6 \ y1 + y 2 = ( x 1 + x 2 ) - ( x 12 + x 22 )
perimeter 36 cm, which sweeps out the 2
æ 1ö 1
largest volume when revolved about one of Now, y = x - x 2 = - çx - ÷ +
its sides, are è 2ø 4
1 1
(B) Let A(- 1, 2) and B(2, 3) be two fixed (q) 12 \ (y1 + y 2 )max = 2 ´ =
points, A point P lying on y = x such that 4 2
perimeter of D PAB is minimum, then sum (D) f ¢¢ ( x ) = 12x 2 + 6ax + 3 ³ 0, " x Î R Þ a Î [ - 2, 2]
of the abscissa and ordinate of point P, is Þ Number of non-zero integer values of a is 4.

JEE Type Solved Examples :


Single Integer Answer Type Questions
x æpt2 ö px 2
l Ex. 79 The function S( x ) = ò sin ç ÷ dt has two Þ = np Þ x 2 = 2n [1 £ x 2 £ 5.76 as is given]
0
è 2 ø 2
Hence, n = 1 or 2
critical points in the interval [1 , 2.4 ]. One of the critical æ px 2 ö
points is a local minimum and the other is a local maximum. x = 2 or x = 2, S ¢¢( x ) = cos ç ÷ × px
The local minimum occurs at x equals ……… . è 2 ø
x æ pt 2 ö æ px 2 ö S ¢¢( 2 ) < 0 and S ¢¢(2) > 0
Sol. Given, S( x ) = ò0 sin ç
è 2 ø
÷ dt , S ¢ ( x ) = sin ç
è 2 ø
÷ =0 Þ minimum at x = 2
502 Textbook of Differential Calculus

l Ex. 80 The radius of a right circular cylinder increases at æ æx2 ö ö


f ¢ ( x ) = 2x ç f ¢ ç ÷ - f ¢ ( 6 - x 2 ) ÷
a constant rate. Its altitude is a linear function of the radius è è 2 ø ø
and increases three times as fast as radius. When the radius æx2 ö x2
is 1 cm the altitude is 6 cm. When the radius is 6 cm, the f ¢ ç ÷ > f ¢ (6 - x )2 , if > 6 - x2
è 2 ø 2
volume is increasing at the rate of 1 cu cm/s. When the
[Q f ¢ ( x ) is increasing]
radius is 36 cm, the volume is increasing
x2
at a rate of n cu cm/s. The value of n/11 is equal to ……… . >6- x 2
Þ x >4 2
2
dr
Sol. Here, = c and h = ar + b æx2 ö
dt Þ f ¢ ç ÷ - f ¢ (6 - x 2 ) > 0
dh dr è 2 ø
Also, =3 [given]
dt dt when x < - 2 or x > 2 Þ f ¢ ( x ) > 0
dr dr
\ a =3 Þ a=3 when x ( - 2, 0) È (2, ¥ )
dt dt \ a+b+c =0
Hence, h = 3r + b Þ [a + b + c ] = 0
when r = 1, h = 6 Þ 6 = 3 + b Þ b = 3 l Ex. 82 The graphs y = 2 x 3 - 4 x + 2 and y = x 3 + 2 x - 1
intersect at exactly 3 distinct points. The slope of the line
passing through two of these points is equal to……… .
Sol. Let ( x 1, y1 ) and ( x 2 , y 2 ) be two of these points. Given,
y = x 3 + 2x - 1 and y = 2x 3 - 4 x + 2
\ y1 = 2x 13 - 4 x 1 + 2 …(i)
and 2y1 = 2x 13 + 4 x1 - 2 …(ii)

\ h = 3 ( r + 1) Subtracting Eq. (i) from Eq. (ii), we get


Y
V = pr 2h = 3pr 2 (r + 1) = 3p (r 3 + r 2 ) R
Q
dV dr
= 3p (3r 2 + 2r )
dt dt P
dV
where r = 6, = 1 cc/s X
dt
dr dr
\ 1 = 3p (108 + 12) Þ 360 p =1
dt dt y 1 = 8x 1 - 4 …(iii)
dV Similarly, y 2 = 8x 2 - 4 …(iv)
Again when r = 23, =n
dt y 2 - y 1 = 8( x 2 - x 1 )
dr y 2 - y1
n = 3p ((3.36)2 + 2.36) =8
dt x 2 - x1
1
n = 3p × 36 (110) × Þ n = 33
360 p
l Ex. 83 The length of the shortest path that begins at the
n 33 point (2, 5), touches the X-axis and then ends at a point on
Þ = =3
11 11 the circle x 2 + y 2 + 12 x - 20y + 120 = 0, is……… .
l Ex. 81 The set of all points where f ( x ) is increasing is Sol. Circles with centre ( - 6, 10) and radius
(a , b ) È (c , ¥), then find [a + b + c ] {where [.] denotes GIF }. = 36 + 100 - 120 = 4
Given that
æx2 ö Now, let (a, 0) be a point on the X -axis. If y is the distance
f ( x ) = 2 f ç ÷ + f (6 - x 2 ), " x Î R from A to P and P to M
è 2 ø
y = (a - 2)2 + 25 + (a + 6)2 + 100 - 4
and f ¢¢( x ) > 0, " x Î R.
æx2 ö dy 2(a - 2) 2(a + 6)
= +
Sol. Here, f ( x ) = 2 f ç ÷ + f (6 - x 2 ) dx 2 (a - 2) + 25 2 (a + 6)2 + 100
2
è 2 ø
æx2 ö dy
can be zero only if a - 2 > 0 and a + 6 < 0 not possible or
f ¢ ( x ) = 2 f ¢ ç ÷ × x - 2x f ¢ ( 6 - x 2 ) da
è 2 ø a - 2 < 0 and a + 6 > 0, hence a Î ( - 6, 2)
Chap 08 Monotonicity, Maxima and Minima 503

Solving
dy
= 0, gives a = 10 (rejected) or a = -
2 Þ ( 4 + 8 + a - 2)( 4 - 8 + a - 2) ³ 0
da 3 Þ (a + 10)(a - 6) ³ 0 ...(ii)
Y + +
–10 – 6
10
C(– 6,10) 8 From Eqs. (i) and (ii), we get a Î ( -¥, - 10] È {6}
M 6
A (2,5) when a = 6, y 2 + 4y + 4 = 0 Þ y = -2
4
2 \ a Î ( -¥, - 10] È {6}
P (a,0) O 2
X a 2 = -10, a 3 = 6 Þ | a 2 + a 3 | = 4

64 256 l Ex. 86 Let f ( x ) be a cubic polynomial defined by


Hence, y min = + 25 + + 100 - 4
9 9 x3
f (x ) = + (a - 3 ) x 2 + x - 13 . Then, the sum of all possible
17 1156 17 34 3
= + -4= + - 4 = 17 - 4 = 13
3 3 3 3 value (s) of ‘a’ for which f ( x ) has negative point of local
minimum in the interval [1, 5 ], is
l Ex. 84 If f :[1, ¥) ® R : f ( x ) is monotonic and differen- x3
Sol. We have, f ( x ) = + (a - 3)x 2 + x - 13
tiable function and f (1) = 1 , then number of solutions of the 3
1 \ For f ( x ) have negative point of local minimum, the
equation f ( f ( x )) = is/are .... equation f ¢ ( x ) = 0 must have two distinct negative roots.
x - 2x + 2
2
Now, f ¢ ( x ) = x 2 + 2(a - 3)x + 1
Sol. Let g ( x ) = f ( f ( x )), given f ( x ) is monotonic and
differentiable \ Following condition(s) must be satisfied simultaneously
\ g ¢ ( x ) = f ¢ ( f ( x )) × f ¢ ( x ), since f ( x ) is monotonic (i) D > 0 (ii) a + b < 0 (iii) ab > 0
Þ f ¢ ( x ) × f ¢ ( f ( x )) > 0 for all x ³ 1 Now, D > 0 Þ 4(a - 3)2 - 4 > 0 Þ (a - 2)(a - 4 ) > 0
Here, g (1) = f ( f (1)) + +
\ g ( 1) = f ( 1) = 1 [given, f (1) = 1] ...(i) 2 – 4
Since, g ( x ) is increasing for x ³ 1. \ a Î ( -¥, 2) È ( 4, ¥ ) ...(i)
1 Also, -2(a - 3) < 0 Þ a - 3 > 0 Þ a > 3 ...(ii)
\ g ( x ) ³ g ( 1) Þ 2 ³1
x - 2x + 2 and product of root(s) = 1 > 0, " a Î R
1 \ ( i ) Ç ( ii ) Ç ( iii ) Þ a Î ( 4, ¥ ) ...(iii)
Þ ³ 1, i.e. only possible when x = 1.
( x - 1) 2 + 1 \ a = 5, as a Î[1, 5]
\ Number of solutions of f ( f ( x )) is 1. Thus, sum of all possible values is 5
l Ex. 87 Consider a polynomial P ( x ) of the least degree
l Ex. 85 The set of values of ‘a’ for which the equation
that has a maximum equal to 6 at x =1 and a minimum
x + 4 x 3 + ax 2 + 4 x + 1 = 0 has all its roots real is given by
4
equal to 2 at x = 3. Then, the value of P ( 2 ) + P ¢ (0 ) - 7, is
(a 1 , a 2 ] È {a 3 }. Then, | a 3 + a 2 | is
Sol. The polynomial is an everywhere differentiable function.
Sol. Here, x 4 + 4 x 3 + ax 2 + 4 x + 1 = 0 Therefore, the points of extremum can only be roots of
æ 2 1 ö æ 1ö derivative. Further more, the derivative of a polynomial is
çx + 2 ÷ + 4çx + ÷ + a = 0
2
dividing by x a polynomial.
è x ø è xø
The polynomial of the least degree with roots x 1 = 1 and
2
æ 1ö æ 1ö x 2 = 3 has the form a( x - 1)( x - 3).
Þ ç x + ÷ + 4 ç x + ÷ + ( a - 2) = 0
è xø è xø Hence, P ¢ ( x ) = a( x - 1)( x - 3) = a ( x 2 - 4 x + 3)
1 Since, at x = 1, there must be P(1) = 6, we have
Put x + = y , (y ³ 2 or y £ -2) x x
x P( x ) = ò1 P ¢ ( x )dx + 6 = a ò ( x 2 - 4 x + 3)dx + 6
1
\ y 2 + 4y + ( a - 2) = 0
æx3 4ö
For y to be real D ³ 0. = aç - 2x 2 + 3x - ÷ + 6
è 3 3ø
16 - 4 (a - 2) ³ 0 Þ a £ 6 ...(i)
Also, if f (y ) = y 2 + 4y + (a - 2) Þ f ( -2) × f (2) ³ 0 Also, P(3) = 2 Þ a = 3
Hence, P ( x ) = x 3 - 6x 2 + 9 x + 2
–2 2 Now, P(2) = 8 - 24 + 18 + 2 = 4, also
P ¢ ( x ) = 3 ( x 2 - 4 x + 3) Þ P ¢ ( 0) = 9
\ P ( 2) + P ¢ ( 0) - 7 = 6
504 Textbook of Differential Calculus

Subjective Type Questions


l Ex. 88 Let g ¢( x ) > 0 and f ¢( x ) < 0, " x Î R, then show and
æp pö
g ¢( x ) > 0, when x Î ç , ÷
(i) g ( f ( x + 1)) < g ( f ( x – 1)) (ii) f ( g ( x + 1)) < f ( g ( x – 1)) è4 2ø
g ¢ ( x ) > 0 and f ¢( x ) < 0, " x Î R æ pö
Sol. Here, g ¢( x ) < 0, when x Î ç0, ÷
è 4ø
i.e. g ( x ) is increasing [or if x 1 > x 2 Þ g ( x 1 ) > g ( x 2 )]
æp pö
and f ( x ) is decreasing \ g ( x ) is increasing, when x Î ç , ÷
è4 2ø
[or if x 1 > x 2 Þ f ( x 1 ) < f ( x 2 ), " x Î R] æ pö
g ( x ) is decreasing, when x Î ç0, ÷.
\ f ( x + 1) < f ( x – 1) …(i) è 4ø
and g ( x + 1) > g ( x – 1)
[as ( x + 1) > ( x – 1)] x x
…(ii) l Ex. 90 If f ( x ) = and g ( x ) = , where 0 < x £ 1,
Case I As, g ( x ) is increasing (so greater input gives greater sin x tan x
output) then in this interval find that f ( x ) and g ( x ) are increasing or
Þ g ( f ( x – 1)) > g ( f ( x + 1)) [using Eq. (i)] decreasing.
x
or g ( f ( x + 1)) < g ( f ( x – 1)) Sol. Here, f (x ) =
sin x
Case II f ( x ) is decreasing (so greater input gives smaller
sin x ×1 – x × cos x
output) Þ f ¢( x ) = ...(i)
sin 2 x
Þ f ( g ( x + 1)) < f ( g ( x – 1)) [using Eq. (ii)]
where sin 2 x is always positive when 0 < x £ 1.
æ pö But to check numerator we again let, h ( x ) = sin x – x cos x
l Ex. 89 Let f ¢(sin x ) < 0 and f ¢ ¢(sin x ) > 0, " x Î ç 0, ÷
è 2ø Þ h ¢( x ) = cos x – 1× cos x + x sin x = x sin x , which is
+ve for 0 < x £ 1
and g ( x ) = f (sin x ) + f (cos x ), then find the interval in \ h ¢( x ) > 0
which g ( x ) is increasing and decreasing. Þ h ( x ) is increasing, when 0 < x £ 1
æ pö Þ h ( 0) < h ( x )
Sol. Here, f ¢(sin x ) < 0 and f ¢¢(sin x ) > 0," x Î ç0, ÷ ...(i)
è 2ø Þ 0 < sin x – x cos x
and g ( x ) = f (sin x ) + f (cos x )
\In Eq. (i), (sin x – x cos x ) > 0,
Þ g ¢( x ) = f ¢(sin x ) × cos x + f ¢(cos x )(–sin x )
Þ f ¢( x ) > 0, when 0 < x £ 1
Þ g ¢¢( x ) = {– f ¢(sin x ) × sin x + f ¢¢(sin x ) × cos 2 x }
Hence, f ( x ) is increasing, when 0 < x £ 1
– { f ¢(cos x ) × cos x – f ¢¢(cos x ) × sin 2 x } ...(ii) x
Again, g (x ) = [given]
As, f ¢(sin x ) < 0, f ¢¢ (sin x ) > 0, tan x
æ pö
sin x > 0, cos x > 0, " x Î ç0, ÷ ...(iii) tan x ×1 – x ×sec 2 x
è 2ø Þ g ¢( x ) = ...(ii)
\ From Eqs. (ii) and (iii), we can say tan 2 x
g ¢¢ ( x ) = {– f ¢(sin x )× cos x } + { f ¢¢ (sin x )× cos 2 x } where tan 2 x > 0
144 42444 3 144 42444 3
+ ve + ve we let f ( x ) = tan x – x sec 2 x
+ { f ¢¢(cos x )×sin 2 x } + {– f ¢(cos x )× cos x } Þ f ¢( x ) = sec 2 x –sec 2 x – x (2sec x )×(sec x tan x )
144 42444 3 144 42444 3
+ ve + ve f ¢( x ) = –2x sec 2 x tan x
æ pö f ¢( x ) < 0, "0 < x £ 1
Þ g ¢¢ ( x ) > 0, " x Î ç0, ÷ ...(iv) As
è 2ø
[Q x Î (01
, ], sec 2 x > 0,tan x > 0]
æ pö
Þ g ¢( x ) is increasing in ç0, ÷ ...(v) \ f ( x ) is decreasing, when 0 < x £ 1.
è 2ø
Now, putting g ¢ ( x ) = 0, Þ f (0) > f ( x ) or 0 > tan x – x sec 2 x
g ¢( x ) = f ¢(sin x )× cos x – f ¢(cos x ) × sin x = 0 \ In Eq. (ii), (tan x – x sec 2 x ) < 0
p Þ g ¢( x ) < 0, when 0 < x £ 1
Þ x=
4 \ g ( x ) is decreasing, when 0 < x £ 1.
Chap 08 Monotonicity, Maxima and Minima 505

l Ex. 91 Let f :[0, ¥) ® [0, ¥) and g :[0, ¥) ® [0, ¥) be l Ex. 93 Given that, S = | x 2 + 4 x + 5 - x 2 + 2 x + 5 | for
non-increasing and non-decreasing function and all real x, then find the maximum value of S 4 .
h( x ) = g ( f ( x )) and if f and g are differentiable for all points
Sol. Here, S= ( x + 2) 2 + 1 - ( x + 1) 2 + 4
in their respective domains and h(0 ) = 0. Then, show h( x ) is
always identically zero.
B(–1, 2)
Sol. Here, h ( x ) = g ( f ( x )), since, g ( x ) Î [0, ¥ ) 2
h ( x ) ³ 0, " x Î domain
Also, h ¢( x ) = g ¢( f ( x )) × f ¢( x ) £ 0, as g ¢ ( x ) ³ 0 A(–2, 1)
1
P
and h ( x ) £ 0, " x Îdomain as h(0) = 0 0
–2 –1 (x, 0)
Hence, h ( x ) = 0," x Î domain
l Ex. 92 A cubic function f ( x ) tends to zero at x = –2 and
has relative maximum/ minimum at x = –1 and x = . If
1 \ S= ( x - ( -2))2 + (1 - 0)2 - ( x - ( -1))2 + (2 - 0)2
3
= PA - PB ...(i)
ò-1 f ( x ) dx = 3 . Find the cubic function f ( x ). [IIT JEE 1992]
1 14
Since, PA - PB £ AB, using triangle law.
Sol. f ( x ) is a cubic polynomial. Therefore, f ¢( x ) is a quadratic \ PA - PB max = 1 + 1 = 2
polynomial and f ( x ) has relative maximum and minimum
1
at x = and x = –1, respectively. [Q AB = -1 + 22 + (2 - 1)2 = 2] ...(ii)
3
1 \ S max = 2
\ –1 and are roots of f ¢( x ) = 0
3 Þ S4 = 4
æ 1ö æ 2 1ö
Þ f ¢( x ) = a ( x + 1) ç x – ÷ = a ç x 2 + x – ÷
è 3ø è 3 3ø l Ex. 94 If f ( x ) = max 2 sin y - x , (where y Î R), then
Now, integrating w.r.t. x, we get
find the minimum value of f ( x ).
éx3 x2 x ù
f (x ) = a ê + – ú + c , c is constant of integration. ì2 - x , x £ 0
Sol. Here, f ( x ) = max 2sin y - x = í
ë 3 3 3û
î2 + x , x > 0
Again, f (–2) = 0 [given]
æ –8 4 2 ö y
\ f (–2) = a ç + + ÷ + c = 0 y=2–x 2+x= y
è 3 3 3ø
(0, 2)
–2a 2a
Þ + c = 0 or c =
3 3 0
æx 3
x 2
x ö 2a
Þ f (x ) = a ç + – ÷+
è 3 3 3ø 3
æ x 3 x 2 x 2ö \ Minimum value of f ( x ) = 2
Þ f (x ) = a ç + – + ÷
è 3 3 3 3ø
l Ex. 95 Find the maximum and minimum value of
1 14
Again, ò-1 f ( x ) dx = 3 [given] f (x ) =
40
.
3 x + 8 x – 18 x 2 + 60
4 3
a 1 3 14
Þ
3 ò- 1
( x + x 2 – x + 2) dx =
3 40
Sol. Given, f ( x ) = is maximum or
Because, y = x 3 and y = –x are odd functions. 3x + 8x – 18x 2 + 60
4 3

1 1
minimum according by 3x 4 + 8x 3 – 18x 2 + 60 is minimum
ò-1 x dx = ò-1 x dx = 0
3
So,
1
or maximum.
a é 2x 3 ù 14 f max , if 3x 4 + 8x 3 – 18x 2 + 60 is minimum.
\ ê + 4x ú = Þ a=3 Then,
3ë 3 û0 3
f min , if 3x 4 + 8x 3 – 18x 2 + 60 is maximum.
\ f (x ) = x 3 + x 2 – x + 2 Let g ( x ) = 3x 4 + 8x 3 – 18x 2 + 60
506 Textbook of Differential Calculus

Þ g ¢ ( x ) = 12x ( x 2 + 2x – 3) But x = e is the only extreme value.


Þ g ¢ ( x ) = 12x ( x + 3) ( x – 1) \ f has the greatest value at x = e
Using number line rule, Þ f (e ) > f ( p ), for all x > 0
+ + 1 1
– –
–3 0 1 Þ (e )e > ( p ) p
which indicates g ¢( x ) changes sign from –ve to +ve at Þ e p > pe
x = –3, 1.
\ Local minimum at x = –3, 1 l Ex. 97 Using the relation 2 (1 – cos x ) < x 2 , x ¹ 0 or
and local maximum at x = 0 [as changes from +ve to –ve] é pù
otherwise prove that sin (tan x ) ³ x for all x Î ê0, ú .
Þ g ( x ) is maximum at x = 0
ë 4û
i.e. g max (0) = 60 [IIT JEE 2003]
and for g ( x ) to be minimum, Sol. Let f ( x ) = sin (tan x ) – x
g min (–3) = 243 – 216 – 162 + 60 = –75
Then, f ¢( x ) = cos (tan x ) × sec 2 x – 1
g min (1) = 3 + 8 – 18 + 60 = 53
Þ f ¢( x ) = cos (tan x ){1 + tan 2 x } – 1
Substituting these values in Eq. (i), we get
40 40 Þ f ¢( x ) = tan 2 x cos (tan x ) – {1 – cos (tan x )}
f max ( x ) when g min , i.e. f ( x ) = and
–75 53
[using 2 (1 – cos x ) < x 2 ]
40 –8
Maximum value = , 1
53 15 f ¢( x ) > tan 2 x cos (tan x ) – tan 2 x
40 2 2
f min ( x ) when g max i.e. f ( x )= = 1
60 3 Þ f ¢( x ) > tan 2 x {2 cos (tan x ) – 1}
2 2
Minimum value =
3 [again, using 2 (1 – cos x ) < x 2 ]
1
l Ex. 96 Use the function f ( x ) = x 1/x , x > 0 and determine f ¢( x ) > tan 2 x (1 – tan 2 x )
2
the bigger of the two numbers e p and p e . é pù
Þ f ¢( x ) ³ 0, " x Î ê0, ú
Sol. Given, f ( x ) = x 1/x , x > 0 ë 4û
Let y = f ( x ) = x 1/x é pù
Þ f ( x ) is increasing function for all x Î ê0, ú.
1 ë 4û
Taking log on both sides, we have log y = log x
x é pù
\ f ( x ) ³ f (0), for all x Î ê0, ú
Differentiating both the sides, we have ë 4û
1 dy 1 æ 1 ö Þ sin (tan x ) – x > sin (tan 0) – 0
× = + log x ç– 2 ÷
y dx x 2 è x ø
é pù
dy é 1 – log x ù Þ sin (tan x ) ³ x , for all x Î ê0, ú
or =y ê ë 4û
dx ë x
2 úû
d
1/x
l Ex. 98 If P (1) = 0 and {P ( x )} > P ( x ) for all x ³1,
x dx
\ f ¢( x ) = 2 [1 – log x ]
x then prove that P ( x ) > 0 for all x >1. [IIT JEE 2003]
Let ¢
f ( x ) = 0, Sol. Here,
d
{ P ( x )} > P ( x ), for all x ³ 1
Þ log x = 1 or x = e dx
d
x 1/x é 1ù d æ x 1/x ö or { P ( x )}e –x > P ( x )e –x , for all x ³ 1
Again, f ¢¢( x ) = 2 êë 0 – + ( 1 – log x ) ç ÷
x úû
dx
x dx è x 2 ø
d
e 1/e æ 1 ö Þ { P ( x )}e –x – P ( x )e –x > 0, for all x ³ 1
\ f ¢¢(e ) = 2 ç– ÷ + 0 dx
e è eø d –x
Þ {e P ( x )} > 0, for all x ³ 1
Þ f ¢¢ (e ) < 0 dx
\ ‘f ’ has a maximum at x = e Þ P ( x )e –x is an increasing function for all x ³ 1
Chap 08 Monotonicity, Maxima and Minima 507

Þ P ( x ) e –x > P (1) e –1, for all x > 1 Differentiating w.r.t. x, we have

Þ –x
> 0, for all x > 1 [as P(1) = 0, given] d ( PQ 2 ) é9 1ù
P (x ) e = x 2 ê × 2 (1 + loge x ) × ú
dx ë2 xû
Thus, P ( x ) > 0, for all x > 1 [as e –x > 0, for all x]
é 9 ù
+ ê1 + (1 + loge x )2 ú 2x
3 ë 2 û
l Ex. 99 In the graph of the function y = x log x , where
2 = x [9 (1 + loge x ) + 2 + 9 (1 + loge x )2 ]
x Î[e –1. 5 , ¥[; find the point P ( x , y ) such that the segment of d ( PQ 2 )
For extremum =0
the tangent to the graph of the function at the point, inter- dx
+
cepted between the point P and Y -axis, is shortest.
3 e–3/2 – e–4/3
Sol. Given, y= x log x
2 Since, x ¹ 0, 9 (1 + log x )2 + 9 (1 + log x ) + 2 = 0
Differentiating w.r.t. x, the given function, we have Þ (1 + log x ) = – 1/ 3,–2/ 3
dy 3 æ 1 ö 3 Þ log x = – 4 / 3,–5/ 3
= ç x × + loge x ÷ = (1 + loge x )
dx 2 è x ø 2 Þ x = e –4/ 3, e –5/ 3
æ 3 ö x = e – 5 / 3 lie outside the interval (e –1. 5 , ¥ ).
Let the point P ( x , y ) Þ P çx, x loge x ÷
è 2 ø
d ( PQ 2 )
The sign scheme for is shown in figure, which
Hence, the equation of tangent to the curve at the point dx
æ 3 ö shows that PQ 2 is minimum.
P çx, x loge x ÷ is
è 2 ø
Therefore, PQ is minimum when x = e – 4 / 3 .
3 3
Y– x loge x = (1 + loge x ) ( X – x )
2 2
l Ex. 100 John has x children from his first wife. Mary has
When it cuts Y-axis, X =0
( x +1) children from her first husband. They marry and have
3 3
Thus, y= x loge x – (1 + loge x ) x children of their own. The whole family has 24 children.
2 2 Assuming that the two children of same parents do not fight,
Y
then find the maximum possible number of fights that can
take place.
Sol. Since, the whole family has 24 children, those of John and
Mary are
X 24 – x – ( x + 1)
O
i.e. (23 – 2x )
P
Now, F = Total number of fights.
Q (X ,Y)
= (number of fights when a John’s child fights a
3 Mary’s child) + (number of fights when a John child
So, y =– x
2 fights a John-Mary’s child) + (number of fights
Hence, the given tangent intersects axis at when a Mary’s child fights a John-Mary’s child)
æ 3 ö = x ( x + 1) + x (23 – 2x ) + ( x + 1)(23 – 2x )
Q ç0, – x÷
è 2 ø = 23 + 45x – 3x 2
2 dF
æ 3 ö For maximum, =0
Now, PQ 2 = x 2 + çy + x÷ dx
è 2 ø
2 Þ 45 – 6x = 0 or x = 7. 5
æ 3 ö
= x 2 + ç x (1 + loge x )÷ d 2F
è 2 ø Þ = 45 > 0
dx 2
é 9 ù
PQ 2 = x 2 ê1 + (1 + loge x )2 ú \ f ( x ) is minimum when x = 7. 5
ë 2 û
508 Textbook of Differential Calculus

But in this case fractional value is not possible. The nearest æ 2 f( x ) ö


integral values are x = 7 and x = 8. Sol. Here, f ( x ) = sin –1 ç ÷
è 1 + f2 ( x ) ø
In either case the total number of fights = 23 + 45 ´ 7 – 3(7 )2
Case I | f ( x )| < 1
= 191
Let f ( x ) = tan q
l Ex. 101 Find the point on the curve 4 x 2 + a 2 y 2 = 4a 2 ; æ 2 tan q ö
\ f ( x ) = sin –1 ç ÷ = sin (sin 2 q ) = 2 q
–1

4 < a 2 < 8 that is farthest from the point (0, – 2 ). è 1 + tan 2 q ø


Sol. The equation of given curve can be expressed as shown in Þ f ( x ) = 2 tan –1 { f ( x )}
the figure 2
Þ f ¢( x ) = × f¢ ( x )
x2 y2 1 + { f ( x )2 }
+ = 1, where 4 < a 2 < 8 which is equation of ellipse.
a2 4 where f ¢( x ) > 0 Þ f ¢( x ) > 0 Þ f ( x ) is increasing
Y [since f ( x ) = tan q , which is increasing]
½ 1 ½
Case II When | f ( x )| > 1 or ½ ½< 1
P(a cos θ, 2 sin θ) ½f( x )½
1
X′ X Now, put = tan q
O (a, 0) f( x )
(0, –2) æ ö
ç 2× 1 ÷
ç f( x ) ÷
Y′ \ f ( x ) = sin –1 ç 2÷
ç1 + æ 1 ö ÷
Hence, let us consider a point P (a cos q , 2 sin q ) on the ç ç ÷
è è f ( x ) ø ÷ø
ellipse.
Let the distance of P (a cos q , 2 sin q ) from (0, – 2) be L. æ 2 tan q ö
Þ f ( x ) = sin –1 ç ÷
Then, L2 = (a cos q – 0)2 + (2 sin q + 2)2 è 1 + tan 2 q ø
Differentiating w.r.t. q, we have æ 1 ö
f ( x ) = sin –1(sin 2q ) = 2q = 2 tan –1 ç ÷
2
d (L ) è f( x ) ø
= a 2 × 2 cos q (–sin q ) + 4 × 2 (sin q + 1) × cos q
dq = 2 cot –1( f ( x ))
= cos q [–2a sin q + 8 sin q + 8] = 0
2
1
Þ f ¢( x ) = – 2 × × f ¢( x )
Þ Either cos q = 0 or (8 – 2a ) sin q + 8 = 0 2
1 + ( f ( x ))2

i.e. q = p/ 2 or sin q =
4 where f ¢( x ) > 0
f ¢( x ) < 0 for all | f ( x )| > 1
2
a –4 Þ
Since, a <82 Hence, f ( x ) is increasing, when | f ( x )| < 1 and f ( x ) is
decreasing, when | f ( x )| > 1.
Þ a –4 < 4
2

4 Ex. 103 Find the minimum value of


Þ > 1 Þ sin q > 1, which is not possible. l

a2 – 4 f ( x ) = | x + 2 | – 2 | x – 2 | + | x |.
d 2 ( L2 )
Further, = cos q [–2a 2 cos q + 8 cos q ] Sol. Here, f ( x ) = | x + 2| – 2 | x – 2| + | x |, which gives rise to
dq 2 four cases as
+ (–sin q )[–2a 2 sin q + 8 sin q + 8] Case I x < – 2
d 2 ( L2 ) f ( x ) = |– ( x + 2) + 2 ( x – 2)| – x
At q = p/ 2, = 0 –[16 – 2a 2 ] = 2 (a 2 – 8) < 0 [as a 2 < 8]
dq 2 = |–x – 2 + 2x – 4 | – x
Hence, L is maximum at q = p/ 2 and the farthest point is (0, 2). = | x – 6 | – x = – ( x – 6) – x
Þ f ( x ) = – 2x + 6 ...(i)
æ 2 f(x ) ö Case II –2 £ x < 0
l Ex. 102 Let f ( x ) = sin –1 ç ÷ , then find the
è1 + f2 (x ) ø f ( x ) = |( x + 2) + 2 ( x – 2)| – x
interval in which f ( x ) is increasing or decreasing. = | 3x – 2| – x = –(3x – 2) – x
Þ f (x ) = – 4 x + 2 ...(ii)
Chap 08 Monotonicity, Maxima and Minima 509

Case III 0 £ x < 2 Suppose Eq. (i) meets the curve again at B (t 1, t 12 ), then
f ( x ) = | x + 2 + 2( x – 2)| + x 1 1
t 12 – t 2 = –
(t 1 – t ) Þ t 1 + t = –
= | 3x – 2| + x 2t 2t
ì 2 1
Þ t1 = – t – ...(ii)
ï–(3x – 2) + x , for 0 £ x < 3 2t

Let L be the length of the chord AB (as normal)
ï (3x –2) + x , for 2 £ x < 2
î 3 L = AB 2 = (t – t 1 )2 + (t 2 – t 12 )2
ì–2x + 2, 0 £ x < 2/ 3 = (t – t 1 )2 [1 + (t + t 1 )2 ]
Þ f (x ) = í ...(iii)
î 4 x – 2, 2/ 3 £ x < 2
1ö é 1ö ù
2 2
æ æ
Case IV x ³ 2 = çt + t + ÷ ê1 + ç t - t - ÷ ú [using Eq. (ii)]
è 2t ø ê è 2t ø ú
ë û
f ( x ) = | x + 2 – 2( x – 2)| + x = | – x + 6| + x = | x – 6 | + x
2
ì–( x –6) + x , 2 £ x < 6 æ 1ö æ 1 ö
=í = ç2t + ÷ ç1 + 2 ÷
è 2t ø è 4t ø
î( x – 6) + x , x ³ 6
3
ì 6, 2 £ x < 6 æ 1 ö
f (x ) = í ...(iv) L = 4 t 2 ç1 + 2 ÷
î2x – 6, x ³ 6 è 4t ø
From Eqs. (i), (ii), (iii) and (iv), we have the following figure. Y
y = x2
Y
B
10
A
8 X′ X
6

4
Y′
2 3 2
2/3 dL æ 1 ö æ 1 ö æ 2 ö
X Þ = 8t ç1 + 2 ÷ + 12t 2 ç1 + 2 ÷ × ç– 3 ÷
–4 –2 0 2/3 2 4 6
dt è 4t ø è 4t ø è 4t ø
2
dL æ 1 ö é æ 1 ö 3ù
Þ = 2 ç1 + 2 ÷ ê 4 t ç1 + 2 ÷ – ú
2 dt è 4t ø ë è 4t ø t û
From the graph, minimum value of f ( x ) = .
3 2 2
dL æ 1 ö æ 2ö æ 1 ö æ 1ö
Þ = 2 ç1 + 2 ÷ ç 4 t – ÷ = 4 ç1 + 2 ÷ ç2t – ÷
l Ex. 104 Which normal to the curve y = x 2 forms the dt è 4t ø è tø è 4t ø è tø

shortest chord? dL
[IIT JEE 1992] For extremum, let =0
dt
Sol. Let (t , t 2 ) be any point on the parabola y = x 2 1
Þ t =±
dy æ dy ö 2
Now, = 2x Þ ç ÷ = 2t , which is the slope of
dx è dx ø (t , t 2 ) Again,
2
tangent. d 2L æ 1 öæ 1 öæ 1ö æ 1 ö æ 1ö
= 8 ç1 + 2 ÷ ç– 2 ÷ ç2t – ÷ + 4 ç1 + 2 ÷ ç2 + 2 ÷
æ 1ö dt 2 è 4 t ø è 2t ø è t ø è 4t ø è t ø
So, the slope of the normal to y = x 2 at (t , t 2 ) is ç– ÷.
è 2t ø æd 2L ö
Þ ç 2÷ >0
\ The equation of the normal to y = x at (t , t ) is
2 2
è dt øt = ± 1

æ 1ö 2
y – t 2 = ç– ÷ ( x – t ) ...(i) 1
è 2t ø \ Minimum when t = ±
2
510 Textbook of Differential Calculus

æ 1 1ö
Thus, points are A ç ± , ÷ and B( m 2, 2). For extrema let f ¢( x ) = 0
è 2 2ø Þ 16x – 8bx + 1 = 0
2

Þ Equation of normal AB is 2x + 2y – 2 = 0 and 8b ± 64 (b 2 – 1)


2x – 2y + 2 = 0. So, x=
2 ´ 16
b ± b2 –1
l Ex. 105 Let f ( x ) = sin 3 x + l sin 2 x where or x=
4
–p/ 2 < x < p/ 2. Find the intervals in which l should lie in Obviously the roots are real, if b 2 – 1 ³ 0
order that f ( x ) has exactly one minimum. Þ b>1 [as b > 0]
Sol. Given, f ( x ) = sin 3 x + l sin 2 x Hence, when b > 1, then using number line rule for f ¢( x ) as
shown in given figure.
\ f ¢( x ) = 3 sin 2 x (cos x ) + l × 2 sin x (cos x )
We know f ¢( x ) changes sign from +ve to –ve at
= sin x cos x (3 sin x + 2l )
b – b2 –1
For extremum, let f ¢( x ) = 0 x=
4
\ sin x = 0, cos x = 0,
b – b2 –1
sin x = – 2l/ 3 \ f ( x )max at x =
4
Since, –p/ 2 < x < p/ 2
\ cos x ¹ 0 and f ¢( x ) changes sign from –ve to +ve at
Þ sin x = 0 Þ x =0 b + b2 –1
x=
– 2l æ – 2l ö 4
and sin x = Þ x = sin –1 ç ÷ ...(i)
3 è 3 ø + +

One of these from Eq. (i) will give maximum and one b –√ b2–1 b+√ b2–1
minimum, provided 4 4

– 2l b + b2 –1
–1 < sin x = <1 \ f ( x )min at x =
3 4
– 2l
i.e. –1 < <1 16x 2 – 8x + 1 ( 4 x – 1)2
3 Also, if b = 1 f ¢( x ) = = no change in
8x 8x
Þ –3 < – 2l < 3 Þ –3 < 2l < 3 sign.
i.e. –3/ 2 < l < 3/ 2 \ Neither maximum nor minimum, if b = 1
But, if l = 0, then sin x = 0 has only one solution. Thus,
\ l Î(–3/ 2, 3/ 2) – {0 } ì b– b2 - 1
ï f ( x )0max when x = and b > 1
Þ l Î (–3/ 2, 0) È (0,3/ 2) ï 4
For this value of l there are two distinct solutions. ï b+ b2 - 1
f (x ) = í f ( x )min when x = and b > 1
Since, f ( x ) is continuous, these solutions give one ï 4
maximum and one minimum because for a continuous ï f ( x ) neither maximum nor minimum when b = 1
function, between two maxima there must lie one minima ï
and vice-versa. î
l Ex. 107 Find the points on the curve ax 2 + 2bxy + ay 2 = c ,
l Ex. 106 Determine the points of maxima and minima of 0 < a < b < c, whose distance from the origin is minimum.
1
the function, f ( x ) = log x – bx + x 2 , x > 0 when b ³ 0 is a
8 Sol. Let P ( x , y ) be a point on the curve ax 2 + 2bxy + ay 2 = c ,
constant. [IIT JEE 1996] whose distance from the origin is r .
1
Sol. Here, f ( x ) = log x – bx + x 2 is defined and continuous \ x = r cos q and y = r sin q
8 As, r cos q and r sin q lies on ax 2 + 2bxy + ay 2 = c
for all x > 0.
1 Þ ar 2 cos 2 q + 2br 2 sin q cos q + ar 2 sin 2 q = c
Then, f ¢( x ) = – b + 2x
8x Þ (a + b sin 2q )r 2 = c
16x 2 – 8bx + 1 c
or f ¢( x ) = Þ r2 = ...(i)
8x a + b sin 2q
Chap 08 Monotonicity, Maxima and Minima 511

From Eq. (i) r is minimum when (a + b sin 2q ) is maximum, 21 – 4 b – b 2


i.e. sin 2q is maximum. \ 1– >0
b +1
p 5p p 5p
i.e. 2q = or Þ q= , æ 21 – 4 b – b 2 ö
2 2 4 4 and ( 0) 2 – 4 ´ 3 ç1 – ÷5<0
ç b +1 ÷
For q, maximum value of (a + b sin 2q ) = a + b è ø
21 – 4b – b 2
c Þ 1– >0
\ r min = b +1
a+b
The above inequality holds, when, (i) b + 1 < 0 and (ii)
p æ r r ö æ c c ö 21 – 4b – b 2 > 0
Also, when q = ,P ç , ÷=ç , ÷
4 è 2 2 ø è 2 (a + b ) 2 (a + b ) ø
\ b < –1 and b 2 + 4b – 21 < 0
5p Þ b < –1 and (b + 7 ) (b – 3) < 0
Again, when q = ,
4 Þ b < –1 and –7 < b < 3 [using number line rule]
æ –r –r ö æ c c ö \ b Î(–7, – 1) ...(i)
Pç , ÷ = ç– ,– ÷
è 2 2 ø è 2 (a + b ) 2 (a + b ) ø Again, when b + 1 > 0, f ( x ) will be increasing for all x, if
æ c c ö 21 – 4 b – b 2
Thus, the required points are ± ç , ÷× 21 – 4 b – b 2 > 0 and 1 >
è 2 (a + b ) 2 (a + b ) ø b +1
Þ b 2 + 4 b – 21 < 0
l Ex. 108 Find the value of n, for which
and (b + 1)2 > (21 – 4b – b 2 ) [as b + 1 > 0 ]
f ( x ) = ( x 2 – 4 ) n ( x 2 – x + 1), n Î N assumes a local minima
at x = 2. Þ (b + 7 ) (b – 3) < 0 and b + 3b – 10 > 0
2

Sol. Here, f ( x ) = ( x 2 – 4 )n ( x 2 – x + 1) assumes local minima at Þ (–7 < b < 3) and (b < – 5 or b > 2)
x =2 Þ 2<b<3 ...(ii)
Þ f (2) < f (2 – h ) and f (2) < f (2 + h ), where h > 0 From Eqs. (i) and (ii), we have concluded that.
where f ( 2) = 0 b Î (–7, – 1) È (2, 3)
Þ f (2 – h ) > 0 and f (2 + h ) > 0, " h > 0
Þ (–h )n ( 4 – h )n × {h 2 – 3h + 1} > 0
l Ex. 110 Find the set of all values of 'a' for which
æ a +4 ö 5
and h n ( 4 + h )n (h 2 + 5h + 1) > 0 f (x ) = ç – 1÷ x – 3 x + log 5 monotonically decreases
è 1– a ø
i.e. (–h )n > 0
for all x.
[Q( 4 – h ) > 0, h 2 – 3h + 1 > 0, 4 + h > 0,
h 2 + 5h + 1 > 0, " h > 0] æ a+4 ö 5
Sol. Given, f ( x ) = ç – 1÷ x – 3x + log 5 decreases for
Þ n Îeven number. è 1–a ø
all x.
l Ex. 109 The interval to which b may belong so that the
æ a+4 ö 4
function, Then, f ¢( x ) = ç – 1÷ 5x – 3 < 0, " x Î R
è 1–a ø
æ 21 – 4 b – b 2 ö÷ 3
f ( x ) = ç1 – x + 5 x + 16 , æ a+4 ö 4
ç b + ÷ i.e. 5ç – 1÷ x – 3 < 0, " x Î R
è 1 ø è 1–a ø
increases for all x. æ a+4 ö
æ 21 – 4 b – b 2 ö 3 Þ ç – 1÷ < 0, " x Î R
Sol. If f ( x ) = ç1 – ÷ x + 5x + 16, increases we è 1–a ø
ç b +1 ÷
è ø [as ax 2 + bx + c < 0 Þ a < 0 and D < 0] ...(i)
must have f ¢( x ) > 0, " x Î real number.
æ Now, two cases arise
21 – 4 b – b 2 ö 2
Then, f ¢( x ) = ç1 – ÷ 3x + 5 > 0, " x Î R Case I If 1 – a < 0 Þ a > 1,
ç b +1 ÷
è ø then a + 4 > (1 – a ) and a + 4 > 0
[as we know ax 2 + bx + c > 0, " x Î R, we must have a > 0 Which is always true as LHS > 0 and RHS < 0
and D < 0 ] \ Above inequality is true for all a > 1 ...(ii)
512 Textbook of Differential Calculus

Case II If 1 –a > 0 Þ a<1 1é æ4 +t ö æ 4 –t öù


Then, f ¢(t ) = ê g ç ÷–g ç ÷ >0 [using Eq. (iii)]
Þ a + 4 < 1– a and a > –4 2ë è 2 ø è 2 ø úû
Þ a 2 – 3a – 3 > 0 and a > –4 Þ f ¢(t ) > 0
3 – 21 Hence, f (t ) increases as t increases.
Þ a< a b
2 or ò g ( x ) dx + ò g ( x ) dx is increasing as (b – a ) increases.
0 0
3 + 21
or a> and a > –4
2 æxö
l Ex. 112 Let g ( x ) = 2 f ç ÷ + f ( 2 – x ) and f ¢¢( x ) < 0,
Þ –4 < a <
3 – 21
...(iii)
è 2ø
2 " x Î (0, 2 ). Find the intervals of increase and decrease of
From Eqs. (ii) and (iii), we conclude g ( x ).
æ 3 - 21 ö æx ö
a Î ç– 4, ÷ È (1, ¥ ) Sol. We have, g ( x ) = 2 f ç ÷ + f (2 – x )
è 2 ø è2ø
æ x ö æ1ö
Þ g ¢( x ) = 2 f ¢ç ÷ × ç ÷ + f ¢(2 – x ) ×(–1)
l Ex. 111 Let a + b = 4, where a < 2 and let g ( x ) be a è 2 ø è2ø
dg æx ö
differentiable function. If >0, " x prove that Þ g ¢( x ) = f ¢ ç ÷ – f ¢ (2 – x ) ...(i)
dx è2ø
b a
ò0 g ( x ) dx + ò0 g ( x ) dx increases as (b – a ) increases. We are given that f ¢¢( x ) < 0, " x Î (0, 2)
[IIT JEE 1997] It means that f ¢( x ) would be decreasing on (0, 2).
4 –t
Sol. Let (b –a ) = t and since a + b = 4, we have a = Now, two cases arise
2
x
t +4 Case I > (2 – x ) and f ¢( x ) is decreasing.
and b= ...(i) 2
2
æx ö 4
where t > 0 (as a < 2 and b > 2) Þ f ¢ ç ÷ < f ¢ (2 – x ), " x >
è2ø 3
a b
Let ò0 g ( x ) dx + ò0 g ( x ) dx = f (t ) é x 4ù
4 -t 4 +t
êë As 2 > 2 - x Þ x > 3 úû
Þ f (t ) = ò0 2 g ( x ) dx + ò
0
2 g ( x ) dx
or
æx ö 4
g ¢( x ) = f ¢ ç ÷ – f ¢(2 – x ) < 0, " < x < 2
æ 4 – t ö æ –1 ö æ4 +t ö æ1ö è2ø 3
\ f ¢(t ) = g ç ÷.ç ÷+g ç ÷ ç ÷
è 2 ø è2ø è 2 ø è2ø æ4 ö
\ g ( x ) is decreasing in ç , 2÷. ...(ii)
[using Leibnitz rule] è3 ø
1é æ4 +t ö æ 4 –t öù x
< (2 – x ) and f ¢( x ) is decreasing.
Þ f ¢(t ) = ê g ç ÷–g ç ÷ ...(ii) Case II
2ë è 2 ø è 2 ø úû 2
æx ö 4 é x 4ù
Since, g ( x ) is increasing and we know Þ f ç ÷ > f (2 – x ), " x < êë As 2 < 2 - x Þ x < 3 úû
è2ø 3
x1 > x 2
æx ö 4
é dg ù or g ¢( x ) = f ç ÷ – f (2– x ) > 0, " 0 < x <
Þ g ( x 1 ) > g ( x 2 ) êQ > 0ú è2ø 3
ë dx û
æ 4ö
4 + t 4 –t \ g ( x ) is increasing in ç0, ÷. ...(iii)
Here, > and g ( x ) is increasing. è 3ø
2 2
From Eqs. (ii) and (iii), we conclude g ( x ) is increasing in
æ4 +t ö æ 4 –t ö æ 4ö æ4 ö
\ gç ÷>gç ÷ ...(iii)
è 2 ø è 2 ø ç0, ÷ and decreasing in ç , 2÷.
è 3ø è3 ø
Chap 08 Monotonicity, Maxima and Minima 513

l Ex. 113 Let f ¢( x ) > 0 and f ¢¢ ( x ) > 0 where x 1 < x 2 . æ x + x 2 f ( x1 ) + f ( x 2 ) ö


and ç 1 , ÷ is the mid-point of chord
f (x1 ) + f (x 2 ) è 2 2 ø
æ x + x2 ö
Then, show f ç 1 ÷ < × joining A and B.
è 2 ø 2
Thus, it can be expressed as shown in figure
Sol. As we have discussed in theory, if f ¢( x ) > 0 and  
f ¢¢ ( x ) > 0, then graphically it can be expressed as shown

x1+x2 , x1+ x2
f  2 
in the following figure  2
Y B (x2) f(x2)
D  
x1+x2 f(x1)+f(x2)
Y ,
C  2 2 
B (x1) f(x1)
f (x2) [(x2) f(x2)] A

f ′ (x) > 0
f ′′ (x) > 0
x1+x2 x2 X
f (x1) O x1
[(x1) f(x1)] 2
A
X
O x1 x2 æ x + x 2 ö f ( x1 ) + f ( x 2 )
From the adjacent figure, f ç 1 ÷> .
è 2 ø 2
x1 + x 2
We know, x1 < < x2
2 l Ex. 115 If f ( x ) is monotonically increasing function for
æ x + x 2 f ( x1 ) + f ( x 2 ) ö all x Î R, such that f ¢ ¢( x ) > 0 and f –1 ( x ) exists, then prove
and ç 1 , ÷ is mid-point of the chord
è 2 2 ø f –1 ( x 1 ) + f –1 ( x 2 ) + f –1 ( x 3 ) æ x + x 2 + x3 ö
joining A and B. that < f –1 ç 1 ÷
3 è 3 ø
Y B [x , f(x )]
2 2 Sol. Let g ( x ) = f –1
(x )
  Since, g is the inverse of f .
x1+x2 f (x1)+f (x2) D

,
  
2 2 x1 + x2 1
C f
 2  Þ fog ( x ) = gof ( x ) = x Þ g ¢( x ) =
[x1, f (x1)] ¢
f ( g ( x ))
A
x2 Þ g ¢( x ) > 0 [as f ( x ) is increasing] ...(i)
X
O x1 x1 + x2 Þ g ( x ) is increasing for all x Î R.
2
Þ f –1( x ) is increasing for all x Î R.
1
Thus, it can be expressed, from the figure as Again, g ¢( x ) =
f ¢( g ( x ))
æ x + x 2 ö f ( x1 ) + f ( x 2 )
f ç 1 ÷< 1
è 2 ø 2 Þ g ¢¢( x ) = – f ¢¢( g ( x )) g ¢( x ), for all x Î R
( f ¢ ( g ( x )))2
l Ex. 114 Let f ¢( x ) > 0 and f ¢¢( x ) < 0 where x 1 < x 2 . Þ g ¢¢( x ) < 0,
{as g ¢ ( x ) > 0, from relation
æ x + x 2 ö f (x1 ) + f (x 2 )
Then, show f ç 1 ÷> . (i) f ¢¢ ( x ) > 0, given
è 2 ø 2
Þ g ¢( x ) is decreasing for all x Î R.
Sol. As we know, if f ¢( x ) > 0 and f ¢¢( x ) < 0. Then, it could be d
expressed graphically as shown in the following figure Þ f –1( x ) is increasing and { f –1( x )} is decreasing.
dx
Y
B Thus, the graph for f –1( x ) could be plotted as
f (x2)
f ′(x) > 0 Y
f (x1) f ′′(x) < 0 B
N C
A y = f –1(x)
M
(x1) f (x1) A L

x1 x2 X
O

x1 + x 2
We know, x1 < < x2 0 x1 x2 x1+x2+x3 x3
X
2 3
514 Textbook of Differential Calculus

In above figure, we have taken three points A , B, C as; l Ex. 117 One corner of a long rectangular sheet of paper
A ( x 1, f –1( x 1 )), B ( x 2 , f –1( x 2 )), C ( x 3 , f –1( x 3 )). of width 1 unit is folded over so as to reach the opposite edge
Also, M is the mid-point of AB as of the sheet. Find the minimum length of the crease.
æ x 1 + x 2 f –1( x 1 ) + f –1( x 2 ) ö Sol. Let ABCD be the rectangular sheet whose corner C is
ç , ÷ folded over along EF so as to reach the edge AB at C ¢.
è 2 2 ø
and L as the centroid of D ABC, Let EF = x
æ x + x 2 + x 3 f –1( x 1 ) + f –1( x 2 ) + f –1( x 3 ) ö ÐFEC = q = ÐFEC ¢
i.e. L = ç 1 , ÷
è 3 3 ø \ EC = x cos q = EC ¢
Correspondingly a point N lies on the curve; A D
æ x + x 2 + x 3 –1 æ x 1 + x 2 + x 3 ö ö
N =ç 1 ,f ç ÷÷
è 3 è 3 øø
Also, from above figure it is clear that ordinate of
N > ordinate of L F
æ x + x 2 + x 3 ö f ( x1 ) + f ( x 2 ) + f ( x 3 )
–1 –1 –1
Þ f –1 ç 1 ÷> C'
è 3 ø 3
x
l Ex. 116 A box of maximum volume with top open is to
be made by cutting out four equal squares from four corners
θ
of a square tin sheet of side length a feet and then folding up π –2θ θ
the flaps. Find the side of the square cut-off. B E C
Sol. Volume of the box is, V = (a – 2x ) × x i.e. squares of side
2
From D BEC ¢, we have BE = C ¢ E cos ( p – 2q )
x are cut out, then we will get a box with a square base
Þ BE = – x cos q × cos 2q
of side (a – 2x ) and height x.
\ BC = BE + EC
1 = – x cos q cos 2q + x cos q
x x
a 1
x Þ x= ...(i)
x x cos q (1 – cos 2q )
x x
Let Z = cos q (1 - cos 2q ) x to be minimum, Z has to be
Removed a maximum.
x x a – 2x
Z = cos q (1 – cos 2q ) ...(ii)
Differentiating Eq. (ii) w.r.t. q, we get
dZ
x = cos q (0 + 2 sin 2q ) – sin q (1 – cos 2q )
dq
d 2Z
a – 2x and = cos q ( 4 cos 4q )– 2 sin 2q × sin q
dq 2
– sin q (2 sin 2q )–cos q (1 – cos 2q )
Volume of box (V ) = (2a - x )2 × x
For maximum/minimum,
dV dZ
\ = (a – 2x )2 + x × 2 (a – 2x ) × (–2) = (a – 2x )(a – 6x ) =0
dx dq
dV
For V to be extremum, =0 Þ 2 sin q (2 – 3 sin 2 q ) = 0
dx
\ sin q = + 2/ 3 [Qsin q ¹ 0]
Þ x = a /2, a /6
But when x = a /2; V = 0 (minimum) and we know 2
When sin q =
minimum and maximum occurs simultaneously in a 3
continuous function. d 2Z 5 16 1 8
Hence, V is maximum when x = a /6. Þ =– – – =– <0
dq 2
5 3 3 3 3 3
Chap 08 Monotonicity, Maxima and Minima 515

Hence, Z is maximum. Sol. Let 2b be the diameter of the circular portion and a be the
1 lengths of the other side of the rectangle.
Þ x = is minimum [from Eq. (i)]
Z Total perimeter = 2a + 4b + pb = K [say]...(i)
\ x is minimum. Now, let the light transmission rate (per square metre) of
the coloured glass be L and Q be the total amount of
1 1 3 3
Þ x min Þ = = unit transmitted light.
Z æ 1ö 4
( 1/ 3 ) ç1 + ÷
è 3ø
Coloured
Glass
l Ex. 118 Find the volume of the greatest right circular
cone that can be described by the revolution about a side of
a right angled triangle of hypotenuse 1 ft.
Sol. Let ABC be right angled triangle . a Clear Glass a

Let the cone be revolved about AB.


AC = 1 ft [given]
2b
Let AB = a = height of cone
1
Then, Q = 2ab × (3L ) + pb 2 × ( L )
\ BC = 1 – a 2 = radius of cone 2
L
A Q = [ pb + 12ab ]
2
2
L
Q = [ pb 2 + 6b ( K – 4 b – pb )]
2
a l' L
Q = [6Kb – 24b 2 – 5pb 2 ]
2
dQ L
1 – a2 \ = [6K – 48b – 10pb ] = 0
B C db 2
6K
b= ...(ii)
1 48 + 10p
Volume of cone = p (1 – a 2 )a
3 d 2Q L
1 and 2
= [– 48 – 10p ] < 0
V = p (a – a 3 ) db 2
3
Thus, Q is maximum and from Eqs. (i) and (ii)
dV 1
Þ = p (1 – 3a 2 ) ( 48 + 10p ) b = 6K = 6 { 2a + 4b + pb }
da 3
2b 6
d 2V 1 Thus, the ratio = =
and = p (–6a ) < 0 a 6+ p
da 2 3
dV
\ Maximum volume when =0 l Ex. 120 Let S be a square of unit area. Consider any
da
1 quadrilateral which has one vertex on each side of S. If a , b, c
i.e. when a = and d denote the lengths of the sides of the quadrilateral,
3
1 2 3 prove that 2 £ a 2 + b 2 + c 2 + d 2 £ 4 [IIT JEE 1997]
Putting a = , we get Vmax = p cu ft
3 27 Sol. Let S be the square of unit area and ABCD be the
quadrilateral of sides a, b, c and d.
l Ex. 119 A window of fixed perimeter (including the base Y
of the arc) is in the form of a rectangle surmounted by a (1 – w) C (w,1)
(0, 1) (1, 1)
semi-circle. The semi-circular portion is fitted with coloured c
(1 – y) b (1 – z)
glass while the rectangular portion is fitted with clear glass.
The clear glass transmits three times as much light per (0, y) D B (1, z)
d a
square metre as the coloured glass does. What is the ratio of
the sides of the rectangle so that the window transmits the X
(0, 0) (x, 0) A (1 – x) (1, 0)
maximum light? [IIT JEE 1991]
516 Textbook of Differential Calculus

Here, a 2 = (1 – x 2 ) + z 2 , b 2 = w 2 + (1 – z )2 Þ Either cos 3a = 0 or cos a = 0


c = (1 – w ) + (1 – y ) , d = x + y
2 2 2 2 2 2 Þ a = p/ 6, p/ 2
But a = p/ 2 is not possible.
Adding all the above, a 2 + b 2 + c 2 + d 2
d 2A
= { x + (1 – x ) } + {y + (1 – y ) } + { z + (1 – z ) }
2 2 2 2 2 2 Now, = 2a 2 [– sin 2a – 4 sin 4a ] = –ve at a = p/ 6
da 2
+ { w 2 + (1 – w )2 } \ A is maximum when a = p/ 6.
where 0 £ x , y , z, w £ 1 Also, Ð A = p/ 3 and triangle is isosceles.
Let us consider a function, Hence, D ABC must be equilateral.
f ( x ) = x 2 + (1 – x )2 , 0 £ x £ 1
l Ex. 122 Show that the height of the cylinder of
Then, f ¢( x ) = 2x – 2(1 – x )
maximum volume that can be inscribed in a sphere of radius
Let f ¢( x ) = 0 for maximum/minimum. a is 2a/ 3 .
Þ 4 x – 2 = 0 Þ x = 1/ 2 Sol. Let h be the height and r be the radius of the cylinder. Let
Again, f ¢¢( x ) = 4 > 0 when x = 1/ 2 O be the centre of the sphere.
\ f ( x ) is minimum at x = 1/ 2 and maximum at x = 1 From the figure,
Þ 2 £ a2 + b2 + c 2 + d 2 £ 4 OA 2 = OP 2 + PA 2

Ex. 121 Show that a triangle of maximum area that can h2


l
i.e. a2 =
+r2 ...(i)
be inscribed in a circle of radius a is an equilateral triangle. 4
Sol. Let BC be one of the sides of the triangle and the third Now, volume of the cylinder,
vertex A should be in a position that the altitude AD is æ h2 ö
maximum (for area of the triangle to be maximum). V = pr 2h = ph ça 2 – ÷ [using Eq. (i)]
è 4 ø
For that the D ABC must be symmetric about AD.
i.e. D should be the mid-point of BC. D C

Let Ð A = 2a
\ ÐBOD = ÐCOD = 2a O
a
Thus, AD = AO + OD = a + a cos 2a
θ
and CD = a sin 2a A P B
A
Differentiating w.r.t. h, we have
aa
dV é 3h 2 ù
= p êa 2 – ú = 0 for extremum
a dh ë 4 û
O 2a
Þ h=
2α a 3
B D C d 2V 3p
Also, 2
=– h <0
dh 2
1 2a
Hence, area of D ABC = A = × AD × BC at h=
2 3
1 2a
Area, A = × a (1 + cos 2a ) × a sin 2a Thus, volume is maximum when h = .
2 3
a2 1
A= (sin 2a + sin 4a )
2 2 l Ex. 123 Let A ( p 2 , p ), B (q 2 , – q ) and C (r 2 , – r ) be the
Differentiating w.r.t. a, we get vertices of the D ABC. A parallelogram AFDE is drawn with
dA a 2 vertices D , E , F on the line segments BC , CA and AB, respec-
= [2 cos 2a + 2 cos 4a ] = 0
da 2 tively. Show that maximum area of such a parallelogram is
1
\ 2a 2 cos 3a × cos a = 0 ( p – q )(q – r )(r – p ).
4
Chap 08 Monotonicity, Maxima and Minima 517

Sol. Let AF = x = DE and AE = y = DF Sol. Let the double ordinate PP ¢ be drawn at a distance x = at 2
Since, D¢s CAB and CED are similar. from the origin. (vertex).
A Thus, the coordinate of P is (at 2 , 2at ) as shown in figure
M y Y
x

L
E (a, 2a)
P

F
x R
y X
O Q (at2, 0) (a, 0)

B D C
P'
CE DE
We have, = [as shown in figure] L'
CA AB
b –y x Hence, the area of trapezium is,
Þ = 1
b c A = ( PP ¢ + LL ¢)× QR
[ here, BC = a, AC = b and AB = c ] ...(i) 2
1
Now, area of parallelogram, = ( 4at + 4a ) × (a – at 2 )
S = AF × EM = xy sin A 2
æ xö A = 2a 2 (t + 1)(1 – t 2 )
S = x × b ç1 – ÷ sin A [from Eq. (i)] ...(ii)
è cø \
dA
= 2a 2 (t + 1)(1 - 3t ) = 0 [for extremum]
dt
Differentiating w.r.t. x, we have
Þ t = – 1, 1/ 3
dS b
= (c – 2x ) sin A [where sin A is constant] 2
d A
dx c Also, = 2a 2 (–6 t – 2)
dt 2
dS
For extremum, =0 d 2A
dx Thus, A is maximum when t = 1/ 3 as <0
c dt 2
Þ x= a
2 Hence, x = at 2 = is the point at which area of trapezium
d 2S 2b 9
Also, 2
=– <0 is maximum.
dx c
c
at x= l Ex. 125 The circle x 2 + y 2 = 1 intersects the X-axis at P
2
c and Q. Another circle with centre at Q and variable radius
Hence, S is maximum when x =
2 intersects the first circle at R, above the X-axis and the line
1 segment PQ at S. Find the maximum area of the D QSR.
Now, S max = bc sin A [from Eq. (ii)]
4 [IIT JEE 1994]
1 æ1 ö
S max = ç bc sin A ÷ Sol. The centre of the circle
è
2 2 ø
x2 + y2 = 1 ...(i)
1
S max = ( area of D ABC )
2 is (0, 0) and radius OP = 1 = OQ
p 2 –p 1 So, coordinates of Q are (1,0).
1 2
S max = q –q 1 Y
4 2
r –r 1
R
1
S max = ( p – q )(q – r )(r – p )
4 X
P O ST Q
(1, 0)
l Ex. 124 L L ¢ be latusrectum of the parabola y 2 = 4ax
and PP ¢ is a double ordinate between the vertex and the
latusrectum. Show that the area of trapezium PP ¢L ¢L is Let the radius of the variable circle be r .
a Hence, its equation is
maximum when the distance of PP ¢ from the vertex is .
9 ( x – 1) 2 + ( y ) 2 = r 2 ...(ii)
518 Textbook of Differential Calculus

Subtracting Eq. (ii) from Eq. (i), we get dT 1 ( 2x ) 1


Þ = × –
2x – 1 = 1 – r 2 dx 40 2 x 2 + 36 50
r2 For maximum and minimum value, we must have
x = 1– = OT ...(iii)
2 dT
2 =0
æ r2ö r4 dx
Now, RT = OR – OT 2 2
= 1 – ç1 – ÷ = r 2 - ...(iv) x 1
è 2ø 4 Þ =
40 x + 36
2 50
1
Now, the area of D QSR is, A = × QS × RT
2 5x
or = x 2 + 36
1 4
\ A 2 = (QS 2 ) × ( RT 2 )
4 25 x 2
or – x 2 = 36
1 æ r4ö 16
Þ A 2 = r 2 çr 2 – ÷
4 è 4ø 36 ´ 16
Þ x2 =
[using Eqs. (ii) and (iv)] 9
6´ 4
Þ A2 =
1
( 4r 4 – r 6 ) = f ( r ) (say) Þ x= = 8 km
16 3
df (r ) 1
Thus, = (16r 3 – 6r 5 ) = 0 l Ex. 127 From point A located on a highway a boy has to
dr 16
get his bus to his school B located in the field at a distance l
[for extremum]
from the highway in the least possible time. At what distance
2
Þ r =2 from D should the bus leave the highway when the bus
3 moves n times slower in the field than on the highway?
d 2 f (r ) 1 16 Sol. Let AD = s and CD = x , where C is a point where the bus
Also, =
( 48r 2 – 30r 4 ) = – < 0
dr 2
16 3 leaves the highway.
2 Also, let the speed of the bus is ‘ v ’ on the highway.
where r =2
3 \ Total time taken,
2 4 AC BC
Hence, area is maximum at r = 2 and A max = sq unit. t = +
3 3 3 v (v/n )

s – x n l2 + x2
l Ex. 126 A despatch rider is in open country at a distance Þ t = +
v v
of 6 km from the nearest point P of a straight road. He
s
wishes to proceed as quickly as possible to a point Q on the
A s–x C x D
road 20 km from P. If his maximum speed across country is
40 km/h, 50 km/h on road, then at what distance from P, he
should touch the road. l

Sol. Let A be the initial position of rider.


Then, PB = x Þ QB = 20 – x
B school
and AB = AP + BP 2 = 62 + x 2 km
2
Differentiating w.r.t. x, we have
1é ù
A
dt n
= ê– 1 + ( + 2x )ú = 0 [for extremum]
dx v ê
ë 2 l +x
2 2
úû
6 km
Þ n 2x 2 = l 2 + x 2
l
x
Þ x=
Q B P 2
n –1
20 +
\ Total time T , – l/ n2 –1
x + 36 20 – x
2
l
T = + Thus, ‘ t ’ is minimum when x = , as shown in figure.
40 50 2
n –1
Chap 08 Monotonicity, Maxima and Minima 519

l Ex. 128 Two men are walking on a path x 3 + y 3 = a 3 But,


x2 y2
¹
when the first man arrives at a point (x 1 , y 1 ), he finds the x 1 y1
second man in the direction of his own instantaneous motion. x2 y2
\ + +1=0
If the coordinates of the second man are (x 2 , y 2 ), then show x 1 y1
that
x2 y2
+ + 1 = 0. l Ex. 129 In still water a boat moves with a velocity which
x1 y 1 is K times less than velocity the river has current. At what
Sol. Since, ( x 1, y1 ) and ( x 2 , y 2 ) lies on the curve. angle to the stream direction must the boat move to
\ x 13 + y13 = a 3 ...(i) minimize drifting.
and x 23 + y 23 =a 3
...(ii) Sol. Let the flow velocity of river be u and the velocity of boat
Subtracting Eqs. (i) from (ii), we get in still water be v.
( x 23 – x 13 ) + (y 23 – y13 ) = 0 Thus, v = u/K
Also, let the boat moves at an angle q with direction of
or x 23 – x 13 = –(y 23 – y13 ) ...(iii)
stream.
Now, differentiating both sides of x + y = a w.r.t. x, we 3 3 3
Now, the velocity of boat in the river is vector resultant of
get the velocity of boat and velocity of following water or
dy water current, which can be written as,
3x 2 + 3y 2 =0
dx v = (u – v cos a ) i$ + (u sin a ) $j = (u + v cos q ) i$ + (u sin q )$j
B
2
x1 Hence, the time taken to cross the river
Slope of tangent at ( x 1, y1 ) = – 2
y1 d
= [where, d = width of river]
\The equation of tangent at ( x 1, y1 ) u sin q
x2 Thus, the drift s = (u + v cos q ) × d
y – y1 = – 12 ( x – x 1 )
y1 v
Þ s = d (cosec q + cot q )
It passes through ( x 2 , y 2 ). u
–x 12 Þ
ds v
= d (–cosec q cot q – cosec 2 q ) = 0
\ y 2 – y1 = ( x 2 – x1 )
y12 dq u
Y
or x 12 ( x 2 – x 1 ) = – y12 (y 2 – y1 ) ...(iv) s
u
Dividing Eq. (iii) by Eq. (iv), we get
d
x 23 – x 13 –(y 23 – y13 ) v
=
x 12 ( x 2 – x 1 ) –y12 (y 2 – y1 ) α θ
O X
x 22 + x 12 + x 1x 2 y 22 + y12 + y1y 2
Þ =
x 12 y12 v
2 2 Þ – cosec 2 q = cosec q cot q
æx2 ö æx2 ö æy2 ö æy2 ö u
Þ ç ÷ + ç ÷ +1= ç ÷ + ç ÷ +1
è x1 ø è x1 ø è y1 ø è y1 ø Þ cosec q = –1/K
2 2 Þ q = cos –1(–1/K )
æx2 ö æy2 ö æy2 ö æ x 2 ö
Þ ç ÷ –ç ÷ = ç ÷–ç ÷
è x1 ø è y1 ø è y1 ø è x 1 ø l Ex. 130 Consider a square with vertices at (1, 1) (–1, 1)
æ x2 y2 ö æ x2 y2 ö æ x2 y2 ö (–1, –1) and (1, –1). Let S be the region consisting of all
Þ ç – ÷ × ç + ÷ + ç – ÷ =0
è x 1 y1 ø è x 1 y1 ø è x 1 y1 ø points inside the square which are nearer to the origin than
æ x2 y2 ö æ x2 y2 ö to any edge. Sketch the region S and find its area.
Þ ç – ÷ × ç + + 1÷ = 0 [IIT JEE 1995]
è x 1 y1 ø è x 1 y1 ø
Sol. Let the square ABCD and the equations of the sides of the
x2 y2 square are as follows
i.e. either =
x 1 y1 AB : y = 1, BC : x = – 1, CD : y = – 1, DA : x = 1
x2 y2 Let the region be S and ( x , y ) is any point inside it. Then,
or + +1=0 according to given conditions,
x 1 y1
520 Textbook of Differential Calculus

x 2 + y 2 < | 1 – x |, | 1 + x |, | 1 – y |,| 1 + y | 1 1
= (3 – 2 2 ) + ( 2 – 1) 3
2 3
Þ ( x 2 + y 2 ) < x 2 – 2x + 1, x 2 + 2x + 1, y 2 – 2y 1 1 1
= (3 – 2 2 ) + ( 5 2 – 7 ) = [ 4 2 – 5]
+ 1, y 2 + 2y + 1 2 3 6
1
Þ y 2 < 1 – 2x , y 2 < 1 + 2x , x 2 < 1 – 2y and x 2 < 2y + 1 Similarly, area of region OEGO = ( 4 2 – 5)
6
2
Y So, area of S lying in first quadrant = ( 4 2 – 5)
B(–1, 1) A(1, 1) 6
4
Hence, S = ( 4 2 – 5)
(0, 1/2) S 3
(–1/2, 0)
X l Ex. 131 If x is increasing at the rate of 2 cm/s at the
O (1/2, 0)
(0, –1/2)
instant when x = 3 cm and y =1 cm, at what rate y must be
changing in order that the quantity ( 2 xy – 3 x 2 y ) shall be
C(–1, –1) D (1, –1)
neither increasing nor decreasing.
If S = {(a , b ) Î R ´ R : x = a , y = b, 2 xy – 3 x 2 y = constant
Y dy
Þ > 0}
A (1, 1) dx
(0, 1)
y2 =2x –1
S ¢ = {( x , y ) Î A ´ B : – 1 £ A £ 1 and – 1 £ B £ 1 }, then find the
G
E x 2 = 2y – 1 area S Ç S ¢.
I dx
Sol. Here, x = 3 cm, y = 1 cm and = 2 cm/s
2y – 1 dt
X
O HF (1, 0)
Let f = 2xy – 3x 2y
df dx dy dy dx
Þ = 2y + 2x – 3x 2 – 6xy
dt dt dt dt dt
Now, in y 2 = 1 – 2x and y 2 = 2x + 1, the first equation
2 dy dx
represents a parabola with vertex (1/ 2,0) and second = (2x - 3x ) + (2y - 6xy ) ...(i)
dx dt
equation represents a parabola with vertex (–1/ 2, 0) and in
Q f is neither increasing nor decreasing.
x 2 = 1 – 2y and x 2 = 1 + 2y , the first equation represents
æ df ö
parabola with vertex at (0, 1/ 2) and second equation \ ç ÷ =0
è dt ø (3 , 1)
represents a parabola with vertex at (0,– 1/ 2).
dy
So, the region S is the region lying inside the four parabolas. Þ (2 ´ 3 – 27 ) + (2 – 18)2 = 0
dt
y2 = 1 – 2x
Þ dy/dt = – 32/ 21
y2 = 1 + 2x
Now, for S, f = constant
x2 = 1 – 2y dy dx
x2 = 1 + 2y Þ 0 = ( 2x – 3x 2 ) + (2y – 6xy )
dt dt
dy 6xy – 2y
Now, S is symmetrical in all four quadrants. Þ =
\ S = 4 ´ area lying in first quadrant. dx 2x – 3x 2
dy 6xy – 2y
Now, y 2 = 1 – 2x Þ >0 Þ >0
dx 2x – 3x 2
and x 2 = 1 – 2y intersect on y = x y ( 3x – 1)
Þ < 0,
The point of intersection is E ( 2 – 1, 2 – 1). x ( 3x – 2)
\Area of region OEFO = Area of D OEH + Thus, there arise two cases
Area of region HEFH Case I y (3x – 1) > 0 and x (3x – 2) < 0
1 1/ 2 Þ (y > 0 and x > 1/ 3) or (y < 0 and x < 1/ 3)
= ( 2 – 1) 2 + ò 1 – 2x dx
2 2 –1 æ 2ö
and ç0 < x < ÷
1 2 è 3ø
= (2 + 1 – 2 2 ) + [(1 – 2x )3 / 2 ]1/ 22 – 1
2 3 æ 1 2ö æ 1ö
1 1 Þ çy > 0 and < x < ÷ or çy < 0 and 0 < x < ÷
= (3 – 2 2 ) + (3 – 2 2 )3 / 2 è 3 3ø è 3ø
2 3
Chap 08 Monotonicity, Maxima and Minima 521

Case II y (3x – 1) < 0 and x (3x – 2) > 0 l Ex. 132 In how many parts an integer N ³ 5 should be
Þ (y > 0 and x < 1/ 3) divide so that the product of the parts is maximized?
or (y < 0 and x > 1/ 3) Sol. Let x 1 + x 2 + x 3 + ...+ x n = N
and ( x < 0 or x > 2/ 3) \ To maximize the value of x 1x 2 x 3 K x n
Þ (y > 0 and x < 0) Using AM > GM
or (y < 0 and x > 2/ 3) x1 + x 2 + K + xn
> ( x 1x 2 K x n )1/n
Thus, S is shaded portion shown in figure (i). n
Also, S ¢ represents the portion shown in figure (ii). n
æ x + x 2 + K + xn ö
Þ x 1x 2 K x n £ ç 1 ÷
Y è n ø
Y
which shows maximum value of x 1x 2 K x n is obtained
1 when x 1 = x 2 = x 3 = K = x n
n
2/3 X æ x + x 2 + K + xn ö
1/3
X –1 1 Now, function to be maximized is ç 1 ÷
O è n ø
–1 which is discrete function of n. In order to arrive at some
possible neighbourhood first we make it continuous. Thus,
(ii) by changing the variable n to x.
(i)
x
æN ö
Thus, area for S Ç S ¢ is shown in figure (iii). We write f (x ) = ç ÷
èxø
Y
For maxima f ¢( x ) = 0
2/3 x
æN ö ì N ü N
–1 O 1/3 1
X i.e. ç ÷ í log – 1ý = 0 Þ =e
èxø î x þ x
N
or x = , now as x Î integer
(iii) e
Thus, the area of S Ç S ¢ = 2 éN ù éN ù
Þ The nearest integer is ê ú or ê ú + 1.
ëe û ëe û
Monotonicity, Maxima and Minima Exercise 1 :
Single Option Correct Type Questions
1. If f : [1, 10] ® [1, 10] is a non-decreasing function and 8. sin x + cos x = y 2 - y + a has no value of x for any y, if a
g : [1, 10] ® [1, 10] is a non-increasing function. Let belongs
h ( x ) = f ( g ( x )) with h(1) = 1 . Then, h(2) (a) ( 0, 3 ) (b) ( - 3, 0 )
(a) lies in (1, 2) (b) is more than two (c) ( - ¥ , - 3 ) (d) ( 3, ¥ )
(c) is equal to one (d) is not defined
9. If f : R ® R is the function defined by
2. P is a variable point on the curve y = f ( x ) and A is a 2 2
e x -e -x
fixed point in the plane not lying on the curve. If PA 2 is f (x ) = 2 2
, then
minimum, then the angle between PA and the tangent at e x + e -x
P is (a) f ( x ) is an increasing function
p p (b) f ( x ) is a decreasing function
(a) (b)
4 3 (c) f ( x ) is a onto
p (d) None of the above
(c) (d) None of these
2 10. Let f ( x ) be a quadratic expression positive for all real x.
ì1 + sin x , x < 0 If g ( x ) = f ( x ) - f ¢ ( x ) + f ¢ ¢( x ), then for any real x
3. Let f ( x ) = í 2 . Then,
î x - x + 1, x ³ 0 (a) g ( x ) > 0 (b) g ( x ) £ 0
(c) g ( x ) ³ 0 (d) g ( x ) < 0
(a) f has a local maximum at x = 0
(b) f has a local minimum at x = 0 11. f ( x ) = min {1, cos x , 1 - sin x }, - p £ x £ p , then
(c) f is increasing everywhere (a) f ( x ) is differentiable at 0
(d) f is decreasing everywhere p
(b) f ( x ) is differentiable at
4. If m and n are positive integers and 2
x (c) f ( x ) has local maxima at 0
f ( x ) = ò (t - a ) 2n (t - b ) 2m +1 dt , a ¹ b, then (d) None of the above
1
ì2 - | x 2 + 5x + 6 |, x ¹ -2
12. f ( x ) = ïí
(a) x = b is a point of local minimum
, then the range of a, so
(b) x = b is a point of local maximum ïî a 2 + 1, x = -2
(c) x = a is a point of local minimum
(d) x = a is a point of local maximum that f ( x ) has maxima at x = - 2 is
(a) | a | ³ 1 (b) | a | < 1 (c) a > 1 (d) a < 1
5. Let f ( x ) = x n +1 + a × x n , where a is a positive real
number. Then, x = 0 is a point of
13. Maximum number of real solution for the equation
(a) local minimum for any integer n
ax n + x 2 + bx + c = 0, where a , b , c Î R and n is an even
(b) local maximum for any integer n positive number, is
(c) local minimum if n is an even integer (a) 2 (b) 3 (c) 4 (d) infinite
(d) local minimum if n is an odd integer
14. Maximum area of rectangle whose two sides are
ì 2 2 x = x 0 , x = p - x 0 and which is inscribed in a region
2x + 2 , 0 < | x | £ 2
6. If f ( x ) = ïí x , then bounded by y = sin x and X-axis is obtained when x 0 Î
ï3, x >2
î æp pö æp pö
(a) x = 1, - 1 are the points of global minima (a) ç , ÷ (b) ç , ÷
è4 3ø è6 4ø
(b) x = 1, - 1 are the points of local minima
æ pö
(c) x = 0 is the points of local minima (c) ç 0, ÷ (d) None of these
è 6ø
(d) None of the above
15. f ( x ) = -1 + kx + k neither touches nor intersect the
7. Given a function y = x x , x > 0 and 0 < x < 1 . The values
curve f ( x ) = ln x , then minimum value of k Î
of x for which the function attain values exceeding the
æ1 1 ö
values of its inverse are (a) ç , ÷ (b) (e , e 2 )
èe e ø
(a) 0 < x < 1 (b) 1 < x < ¥
(c) 0 < x < 2 (d) None of these æ 1 ö
(c) ç , e ÷ (d) None of these
è e ø
Chap 08 Monotonicity, Maxima and Minima 523

16. Let f ( x ) be a polynomial with real coefficients satisfies 23. Analyze the following graph of f ¢( x )
f ( x ) = f ¢ ( x ) ´ f ¢ ¢ ¢ ( x ). If f ( x ) = 0 satisfies x = 1, 2, 3 Y
only, then the value of f ¢ (1) ´ f ¢ (2) ´ f ¢ (3) is equal to y = f ′(x)
(a) positive (b) negative α c d X
(c) 0 (d) inadequate data a b e β
17. A curve whose concavity is directly proportional to the
logarithm of its x-coordinates at any of the curve, is given
by which is incorrect about f ( x ) for a < x < b?
(a) c1 × x 2(2 log x - 3 ) + c 2x + c 3 (a) only three extreme points
(b) c1x 2(2 log x + 3 ) + c 2x + c 3 (b) two inflexion points
(c) c1x 2(2 log x ) + c 2 (c) f ¢¢¢( x ) > 0 for d < x < e
(d) None of the above (d) x = e is the point of local maxima
p 24. Let f ( x ) = x 2 - 2x and g ( x ) = f ( f ( x ) - 1) + f (5 - f ( x )),
18. f ( x ) = 4 tan x - tan 2 x + tan 3 x , " x ¹ np + , " n Î I ,
then 2 then
(a) f ( x ) is increasing for all x Î R (a) g ( x ) < 0, " x Î R
(b) f ( x ) is decreasing for all x Î R (b) g ( x ) < 0, for some x Î R
(c) f ( x ) is increasing in its domain (c) g ( x ) ³ 0, for some x Î R
(d) None of the above (d) g ( x ) ³ 0, " x Î R
ì3 + | x - k |, for x £ k 25. Let f : N ® N be such that f (n + 1) > f ( f (n )) for all
19. If f ( x ) = ïí sin ( x - k ) has minimum n Î N , then
a -2+ 2
, for x > k
ïî x -k (a) f ( x ) = x 2 - x + 1
at x = k , then (b) f ( x ) = x - 1
(a) a Î R (b) | a | < 2 (c) f ( x ) = x 2 + 1
(c) | a | > 2 (d) 1 < | a | < 2
(d) None of the above
20. Let f be a linear function with properties 1
f (1) £ f (2), f (3) ³ f ( 4 ) and f (5) = 5, then which of the 26. The equation | 2ax - 3 | + | ax + 1 | + | 5 - ax | = possesses
2
following is true?
(a) infinite number of real solutions for some a Î R
(a) f ( 0 ) < 0 (b) f ( 0 ) = 0 (b) finitely many real solutions for some a Î R
(c) f (1 ) < f ( 0 ) < f ( -1 ) (d) f ( 0 ) = 5 (c) no real solutions for some a Î R
21. If P ( x ) is polynomial satisfying P ( x 2 ) = x 2 P ( x ) and (d) no real solutions for all a Î R
P (0) = - 2, P ¢(3 / 2) = 0 and P (1) = 0. 2
27. If ò 2x f 2 (t ) dt = æç ò 2 f ( x - t ) dt ö÷ for f (1) = 1 and f ( x )
x x

The maximum value of P ( x ) is 0 è 0 ø


(a) -
1
(b) -
1 is continuous function for x > 0 and {an } is a sequence
3 4 such that an +1 = an + 1 + an2 for a = 0; if f ( x ) is an
1
(c) - (d) None of these a
2 increasing function, then lim nk-1 =
n ®¥ 2
22. If the curve x 2 = - 4 (y - a ) does not intersect the curve
( where k = f (n 2 -1 )) is
y = [x 2 - x + 1] (where [.] denotes the greatest integer
(a) p / 4 (b) 4/ p (c) p (d) p /2
é 1+ 5ù
function) in ê0, ú, then 28. A function f is defined by f ( x ) = | x | | x - 1 |n , " x Î R.
m
ë 2 û
The maximum value of the function is (m, n Î N )
1
(a) < a < 1 (b) -1 < a < 1 (a) 1 (b) mn × nm
3
m ×n
m n
(mn )mn
1 (c) (d)
(c) < a < 1 (d) None of these (m + n )m + n (m + n )m + n
4
524 Textbook of Differential Calculus

Monotonicity, Maxima and Minima Exercise 2 :


More than One Option Correct Type Questions
29. Which of the following is/are true? 34. If maximum and minimum values of the determinant
ln ( lnx ) 1 + sin 2 x cos 2 x sin 2x
(you may use f ( x ) = )
lnx sin 2 x 1 + cos 2 x sin 2x
(a) (ln 2.1 ) ln 2 .2 > (ln 2.2 ) ln 2 .1 sin 2 x cos 2 x 1 + sin 2x
(b) (ln 4 ) ln 5
< (ln 5 ) ln 4
are a and b, then
(c) (ln 30 ) ln 31 > (ln 31 ) ln 30 (a) a + b 99 = 4
(d) (ln 28 ) 30 < (ln 30 ) ln 28 (b) a 3 - b17 = 26
30. If lim f ( x ) = lim [ f ( x )] (where [×] denotes the greatest (c) ( a 2n - b 2n ) is always an even integer for n Î N
x ®a x ®a (d) a triangle can be drawn having it’s sides as a, b
integer function) and f ( x ) is non-constant continuous and a - b
function, then ì
(a) lim f ( x ) is an integer
x ®a
ïx 2 + 4 x , - 3 £ x £ 0
ï
p
(b) lim f ( x ) is non-integer
x ®a 35. Let f ( x ) = ïí - sin x , 0 < x £ . Then,
ï 2
(c) f ( x ) has local maximum at x = a
ï p
(d) f ( x ) has local minimum at x = a ïî - cos x - 1, 2 < x £ p
31. Let S be the set of real values of parameter l for which (a) x = -2 is the point of global minima
the equation f ( x ) = 2x - 3(2 + l )x + 12lx has exactly
3 2 (b) x = p is the point of global maxima
p
one local maximum and exactly one local minimum. (c) f ( x ) is non-differentiable at x =
2
Then, S is a subset of
(d) f ( x ) is discontinuous at x = 0
(a) ( - 4, ¥ ) (b) ( -3, 3 )
(c) (3, ¥ ) (d) ( - ¥ , 0 ) 36. Let f ( x ) = ab sin x + b 1 - a 2 cos x + c ,
æx 2 ö where | a | < 1, b > 0, then
32. h ( x ) = 3 f ç ÷ + f (3 - x 2 ), " x Î ( - 3, 4 ), where (a) maximum value of f ( x ) is b, if c = 0
è 3 ø (b) difference of maximum and minimum value of f ( x ) is 2b
f ¢ ¢( x ) > 0, " x Î ( - 3, 4 ), then h ( x ) is (c) f ( x ) = c, if x = - cos-1 a
æ3 ö (d) f ( x ) = c, if x = cos-1 a
(a) increasing in ç , 4 ÷
è2 ø
xn dt
æ 3 ö 37. If f ( x ) = ò , x > 0 and n > m , then
(b) increasing in ç - , 0 ÷ xm ln t
è 2 ø
x m - 1( x - 1 )
æ 3ö (a) f ¢( x ) =
(c) decreasing in ç - 3, - ÷ ln x
è 2ø (b) f ( x ) is decreasing for x > 1
æ 3ö (c) f ( x ) is increasing in ( 0, 1 )
(d) decreasing in ç 0, ÷
è 2ø (d) f ( x ) is increasing for x > 1
33. f ( x ) = tan -1 (sin x + cos x ), then f ( x ) is increasing in 38. f ( x ) = x - 1 + 2 - x and g ( x ) = x 2 + bx + c are two
æ p pö æ p pö given functions such that f ( x ) and g ( x ) attain their
(a) ç - , ÷ (b) ç - , ÷
è 2 4ø è 4 4ø maximum and minimum values respectively for same
æ 5p 3p ö æ 7p ö value of x, then
(c) ç , ÷ (d) ç - 2 p , - ÷ 1 3
è 4 2 ø è 4 ø (a) f ( x ) max at x = (b) f ( x ) max at x =
2 2
(c) b = 3 (d) b = -3
Chap 08 Monotonicity, Maxima and Minima 525

| x|
39. If f ( x ) = a {a
| x|
; g ( x ) = a [a sgn x ] for a > 0, a ¹ 1 and
sgn x } (b) Maximum value of f is p
(c) There exists atleast one c Î( 0, 2 p ) if f ¢(c ) = 0
x Î R, where {×} and [×] denote the fractional part and
integral part functions respectively, then which of the p
(d) Minimum value of f is -
following statements can hold good for the function 2
h( x )? where (ln a ) h( x ) = (ln f ( x ) + ln g ( x )) x -1
43. Let f ( x ) = , then which of the following is correct?
(a) h is even and increasing x2
(b) h is odd and decreasing (a) f ( x ) has minima but no maxima
(c) h is even and decreasing (b) f ( x ) increases in the interval ( 0, 2 ) and decreases in the
(d) h is odd and increasing interval ( - ¥, 0 ) È (2, ¥ )
(c) f ( x ) can come down in ( - ¥, 0 ) È (2, 3 )
40. For the function f ( x ) = ln (1 - ln x ) , which of the (d) x = 3 is the point of inflection
following do not hold good? 44. Let f ( x ) be differentiable function on the interval
(a) increasing in (0, 1) and decreasing in (1, e) af ( x ) - xf (a )
( -¥, ¥ ) such that f (1) = 5 and lim = 2, for
(b) decreasing in (0, 1) and increasing in (1, e) a ®x a-x
(c) x = 1 is the critical number for f ( x ) all x Î R. Then, which of the following alternative(s)
(d) f has two asymptotes is/are correct?
(a) f ( x ) has an inflection point
ì x + 2, if x < - 1
(b) f ¢( x ) = 3, " x Î R
41. The function f ( x ) = ïí x 2 , if - 1 £ x < 1 2
(c) ò f ( x ) dx = 10
ï( x - 2) 2 , if x ³ 1 0
î (d) Area bounded by f ( x ) with coordinate axes is (2 / 3 )
(a) is continuous for all x Î R
45. Let f :(0, ¥ ) ® (0, ¥ ) be a derivable function and F ( x ) is
(b) is continuous but not differentiable, " x Î R
the primitive of f ( x ) such that 2( F ( x ) - f ( x )) = f 2 ( x ) for
(c) is such that f ¢( x ) change its sign exactly twice
any real positive x
(d) has two local maxima and two local minima
f (x )
p (a) f is strictly increasing (b) lim =1
42. A function f is defined by f ( x ) = ò cos t cos ( x - t ) dt , x®¥x
0
(c) f is strictly decreasing (d) f is non-monotonic
0 £ x £ 2 p, then which of the following hold(s) good?
(a) f ( x ) is continuous but not differentiable in ( 0, 2 p )

Monotonicity, Maxima and Minima Exercise 3 :


Statements I and II Type Questions
Directions (Q. Nos. 46 to 54) For the following questions, 47. Statement I For the function
choose the correct answers from the codes (a), (b), (c) and (d) ì15 - x , x < 2
defined as follows. f (x ) = í x = 2 has neither a maximum nor a
(a) Statement I is true, Statement II is also true, Statement II î2x - 3, x ³ 2
is the correct explanation of Statement I minimum point.
(b) Statement I is true, Statement II is also true, Statement II Statement II f ¢ ( x ) does not exist at x = 2.
is not the correct explanation of Statement I 48. Statement I
(c) Statement I is true, Statement II is false x ép pù
f ( x ) = ò (3 sin t + 4 cos t ) dt , x Î ê , ú f ( x ) attains its
(d) Statement I is false, Statement II is true 0 ë6 3û
p
46. Statement I The equation 3x 2 + 4ax + b = 0 has atleast maximum value at x = .
one root in (0, 1), if 3 + 4a = 0 . 3
x
Statement II f ( x ) = 3x 2 + 4 x + b is continuous and Statement II f ( x ) = ò (3 sin t + 4 cos t ) dt , f ( x ) is
0
differentiable in (0, 1). ép pù
increasing function in ê , ú.
ë6 3û
526 Textbook of Differential Calculus

49. Let f ( x ) be a twice differentiable function in [a, b ] , 52. Statement I For any DABC
given that f ( x ) and f ¢ ¢( x ) has same sign in [a, b]. æ A + B + C ö sin A + sin B + sin C
sin ç ÷³
Statement I f ¢( x ) = 0 has at the most one real root in [a, b ]. è 3 ø 3
Statement II An increasing function can intersect the
Statement II y = sin x is concave downward for
X-axis at the most once.
x Î(0, p ].
50. Let u = c + 1 - c and v = c - c - 1, c > 1 and let 53. Statement I If f ( x ) = [x ] (sin x + cos x - 1)
f ( x ) = ln (1 + x ), " x Î ( - 1, ¥ ).
Statement I f (u ) > f (v ), " c > 1 because (where [ × ] denotes the greatest integer function),
Statement II f ( x ) is increasing function, hence for then f ¢( x ) = [x ](cos x - sin x ) for any x Îinteger.
u > v , f (u ) > f (v ). Statement II f ¢( x ) does not exist for any
p 3p x Îinteger.
51. Let f (0) = 0, f æç ö÷ = 1, f æç ö÷ = - 1 be a continuous and
è2ø è2ø 54. f ( x ) is a polynomial of degree 3 passing through origin
twice differentiable function. having local extrema at x = ± a .
æ 3p ö Statement I Ratio of areas in which f ( x ) cuts the
Statement I | f ¢ ¢( x )| £ 1 for atleast one x Î ç0, ÷
because è 2ø circle x 2 + y 2 = 36 is 1 : 1.
Statement II According to Rolle's theorem, if y = g ( x ) is Statement II Both y = f ( x ) and the circle are
continuous and differentiable, "x Î[a, b ]and g (a ) = g (b ), symmetric about origin.
then there exists atleast one c such that g ¢(c ) = 0.

Monotonicity, Maxima and Minima Exercise 4 :


Passage Based Questions
Passage I Passage II
(Q. Nos. 55 to 57) (Q. Nos. 58 to 60)
Let f ( x ) =
1
, let m be the slope, a be the x-intercept and b be Consider the function f ( x ) = max { x 2 ,
1+ x 2
(1 - x )2 , 2x (1 - x )}, x Î [0, 1]
the y-intercept of a tangent to y = f ( x ).
58. The interval in which f ( x ) is increasing, is
55. Abscissa of the point of contact of the tangent for which
æ1 2ö æ1 1ö
m is greatest, is (a) ç , ÷ (b) ç , ÷
è3 3ø è3 2ø
1
(a) (b) 1
3 æ1 1ö æ1 2ö æ1 1ö æ2 ö
(c) ç , ÷ È ç , ÷ (d) ç , ÷ È ç , 1 ÷
1 è3 2ø è2 3ø è3 2ø è3 ø
(c) – 1 (d) -
3
59. Let RMVT is applicable for f ( x ) on (a , b ), then a + b + c is
56. Value of b for the tangent drawn to the curve y = f ( x ) (where c is the point such that f ¢ (c ) = 0)
whose slope is greatest, is
2 1
9 3 (a) (b)
(a) (b) 3 3
8 8
1 5 1 3
(c) (d) (c) (d)
8 8 2 2
57. Value of a for the tangent drawn to the curve y = f ( x ) 60. The interval in which f ( x ) is decreasing, is
whose slope is greatest, is æ1 2ö æ1 1ö
(a) ç , ÷ (b) ç , ÷
(a) - 3 è3 3ø è3 2ø
(b) 1 æ 1ö æ1 2ö æ1 1ö æ2 ö
(c) ç 0, ÷ È ç , ÷ (d) ç , ÷ È ç , 1 ÷
(c) – 1 è 3ø è2 3ø è3 2ø è3 ø
(d) 3
Chap 08 Monotonicity, Maxima and Minima 527

Passage III 68. The value of a 0 is


(Q. Nos. 61 to 63) (a) equal to 50
f ( x ), g ( x ), h ( x ) all are continuous and differentiable functions in (b) greater than 54
[a, b ] also a < c < b and f (a ) = g (a ) = h (a ). Point of intersection of (c) less than 54
the tangent at x = c with chord joining x = a and x = b is on the (d) less than 50
left of c in y = f ( x ) and on the right in y = h ( x ). And tangent at 69. f ( -10) is defined for
x = c is parallel to the chord in case of y = g ( x ) . (a) a 0 > 830
Now, answer the following questions. (b) a 0 < 830
61. If f ¢ ( x ) > g ¢ ( x ) > h ¢ ( x ), then (c) a 0 = 830
(a) f (b ) < g (b ) < h (b ) (b) f (b ) > g (b ) > h (b ) (d) None of the above
(c) f (b ) £ g (b ) £ h (b ) (d) f (b ) ³ g (b ) ³ h (b )
Passage VI
62. If f (b ) = g (b ) = h (b ), then (Q. Nos. 70 to 74)
(a) f ¢(c ) = g ¢(c ) = h ¢(c ) (b) f ¢(c ) > g ¢(c ) > h ¢(c )
(c) f ¢(c ) < g ¢(c ) < h ¢(c )
x + bx + c
2
(d) None of these If f : D ® R, f ( x ) = , where a , b are the roots of the
a +b x 2 + b1x + c 1
63. If c = for each b, then equation x 2 + bx + c = 0 and a 1, b1 are the roots of
2
(a) g ( x ) = Ax 2 + Bx + C x 2 + b1x + c 1 = 0. Now, answer the following questions for f ( x ). A
(b) g ( x ) = log x combination of graphical and analytical approach may be helpful
(c) g ( x ) = sin x in solving these problems. (If a 1 and b1 are real, then f ( x ) has
(d) g (x ) = e x vertical asymptote at x = a 1, b1 ) . Then,
70. If a 1 < a < b 1 < b , then
Passage IV
(a) f ( x ) is increasing in ( a1, b1 )
(Q. Nos. 64 to 66) (b) f ( x ) is decreasing in ( a, b )
In the non-decreasing sequence of odd integers (c) f ( x ) is decreasing in (b1, b )
(a1, a 2 , a 3 , ...) = {1, 3, 3, 3, 5, 5, 5, 5, 5, ...} each positive odd integer k (d) f ( x ) is decreasing in ( - ¥, a )
appears k times. It is a fact that there are integers b, c and d such
that for all positive integer n , an = b [ n + c ] + d (where [.] 71. If a 1 < b 1 < a < b , then
denotes greatest integer function). (a) f ( x ) has a maxima in [ a1, b1 ] and a minima is [ a, b ]
(b) f ( x ) has a minima in ( a1, b1 ) and a maxima in ( a, b )
64. The possible value of b + c + d is (c) f ¢( x ) > 0 where ever defined
(a) 0 (b) 1 (c) 2 (d) 4 (d) f ¢( x ) < 0 where ever defined
b - 2d
65. The possible value of is 72. If the equations x 2 + bx + c = 0 and x 2 + b1 x + c 1 = 0 do
8
not have real roots, then
(a) 0 (b) 1
(c) 2 (d) 4 (a) f ¢( x ) = 0 has real and distinct roots
c +d (b) f ¢( x ) = 0 has real and equal roots
66. The possible value of is (c) f ¢( x ) = 0 has imaginary roots
2b
(d) nothing can be said
(a) 0 (b) 1
(c) 2 (d) 4 73. In the above problem, lim [ f ( x )]× lim [ f ( x )] (where [.]
x ®¥ x ®¥
Passage V denotes the greatest integer function) is equal to
(Q. Nos. 67 to 69) (a) 1
(b) 0
Let g ( x ) = a 0 + a1x + a 2 x 2 + a 3 x 3 and f ( x ) = g ( x ), f ( x ) have (c) – 1
its non-zero local minimum and maximum values at - 3 and 3, (d) does not exist
respectively. If a 3 Îthe domain of the function 74. In the last problem, if b > b1 , then
æ1 + x 2 ö
h ( x ) = sin -1 ç ÷ (a) x-coordinate of point of minima is greater than the
è 2x ø x-coordinate of point of maxima
(b) x-coordinate of point of minima is less than x-coordinate
67. The value of a1 + a 2 is equal to of point of maxima
(a) 30 (b) – 30 (c) it also depends upon c and c1
(c) 27 (d) – 27 (d) nothing can be said
528 Textbook of Differential Calculus

Passage VII 80. Possible graph of y = f ( x ) is


(Q. Nos. 75 to 77) Y
x2
Consider the function f ( x ) =
x2 - 1 (a)
75. The interval in which f is increasing is X
–5 0 2 4
(a) ( - 1, 1 )
(b) ( - ¥, - 1 ) È ( - 1, 0 )
Y
(c) ( - ¥, ¥ ) - { - 1, 1 }
(d) ( 0, 1 ) È (1, ¥ )
76. If f is defined from R - {- 1, 1} ® R, then f is (b)
(a) injective but not surjective X
(b) surjective but not inective –5 0 2 4
(c) injective as well as surjective
Y
(d) neither injective nor surjective
77. f has
(c)
(a) local maxima but not local minima
(b) local minima but not local maxima 0 X
–5 2 4
(c) both local maxima and local minima
(d) neither local maxima nor local minima Y

Passage VIII
(d)
(Q. Nos. 78 to 80)
Suppose you do not know the function f ( x ), however some X
–5 0 2 4
information about f ( x ) is listed below.
Read the following carefully before attempting the questions Passage IX
(i) f ( x ) is continuous and defined for all real numbers (Q. Nos. 81 to 83)
(ii) f ¢ ( - 5) = 0, f ¢ (2) is not defined and f ¢ ( 4 ) = 0
Let f ( x ) = e (p +1)x - e x for real number p > 0, then
(iii) ( - 5, 12) is a point which lies on the graph of f ( x )
(iv) f ² (2) is undefined, but f ² ( x ) is negative everywhere else 81. The value of x = s p for which f ( x ) is minimum, is
(v) The signs of f ¢( x ) is given below - loge ( p + 1 )
(a) (b) - loge ( p + 1 )
p
f′+ + + +++ æ p + 1ö
(c) - loge p (d) loge ç ÷
è p ø
t +1
–5 0 2 4 82. Let g (t ) = ò f ( x ) e t - x dx . The value of t = t p , for which
t
g (t ) is minimum, is
78. On the possible graph of y = f ( x ), we have æep -1ö
(e p - 1 ) 1
(a) x = - 5 is a point of relative minima (a) - loge (b) - loge ç ÷
p p è p ø
(b) x = 2 is a point of relative maxima
(c) x = 4 is a point of relative minima 1 æ ( p + 1 )(e p - 1 ) ö
(c) - loge ç ÷ (d) - loge (( p + 1 )(e p - 1 ))
(d) graph of y = f ( x ) must have a geometrically sharp corner p è p ø
79. From the possible graph of y = f ( x ), we can say that p e p -1 p
83. Use the fact that 1 + £ £ 1 + + p 2 (0 < p £ 1),
(a) there is exactly one point of inflection on the curve 2 p 2
(b) f ( x ) increases on - 5 < x < 2 and x > 4 and decreases on the value of lim (s p - t p ) is
- ¥ < x < - 5 and 2 < x < 4 p ®0+
(c) the curve is always concave down
1
(d) curve always concave up (a) 0 (b) (c) 1 (d) non-existent
2
Chap 08 Monotonicity, Maxima and Minima 529

Passage X 89. Range of function sin -1 ( fog( x )) is


(Q. Nos. 84 to 86) (a) ( 0, p / 2 ) (b) { 0, p / 2 }
Consider f , g and h be three real valued function defined on R. (c) { -p / 2, 0, p / 2 } (d) { p / 2 }
Let f ( x ) = sin 3x + cos x , g ( x ) = cos 3x + sin x and Passage XII
h ( x ) = f 2 ( x ) + g 2 ( x ).
(Q. Nos. 90 to 92)
84. The length of a longest interval in which the function Consider f, g and h be three real valued differentiable functions
g = h ( x ) is increasing, is defined on R.
(a) p/8 (b) p/4 (c) p/6 (d) p/2 Let g ( x ) = x 3 + g ¢¢(1)x 2 + (3 g ¢(1) - g ¢¢(1) - 1) x + 3 g ¢(1)
85. The general solution of the equation h( x ) = 4, is f ( x ) = x g ( x ) - 12x + 1
(a) ( 4n + 1 ) p / 8 (b) (8n + 1 ) p / 8 and f ( x ) = (h ( x ))2 , where h(0) = 1.
(c) (2n + 1 ) p / 4 (d) (7n + 1 ) p / 4 90. The function y = f ( x ) has
86. Number of point(s) where the graphs of the two (a) Exactly one local minima and no local maxima
function, y = f ( x ) and y = g ( x ) intersects in [0, p ], is (b) Exactly one local maxima and no local minima
(a) 2 (b) 3 (c) Exactly one local maxima and two local minima
(c) 4 (d) 5
(d) Exactly two local maxima and one local minima
Passage XI 91. Which of the following is/are true for the function
(Q. Nos. 87 to 89) y = g ( x )?
(a) g( x ) monotonically decreases inEREDEDDD
Consider f, g and h be three real valued functions defined on R.
æ 1 ö æ 1 ö
ì -1, x < 0 ç -¥, 2 - ÷ È ç2 + , ¥÷
è 3 ø è 3 ø
ï
Let f ( x ) = í 0, x = 0, g ( x ) = x (1 - x 2 ) and h ( x ) be such that
é 1 1 ö
ï 1, x > 0 (b) g( x ) monotonically increases in ê2 - ,2+ ÷
î ë 3 3ø
h ¢¢( x ) = 6x - 4. (c) There exists exactly one tangent to y = g( x ) which is
Also, h ( x ) has local minimum value 5 at x = 1. parallel to the chord joining the points (1, g(1 )) and (3, g(3 ))
(d) There exists exactly two distinct lagrange’s mean value in
87. The equation of tangent at m(2, 7 ) to the curve y = h( x ), ( 0, 4 ) for the function y = g( x )
is
92. Which one of the following does not hold good for
(a) 5 x + y = 17 (b) x + 5y = 37
y = h( x )?
(c) x - 5y + 33 = 0 (d) 5 x - y = 3
(a) Exactly one critical point
88. The area bounded by y = h( x ), y = g ( f ( x )) between x = 0 (b) No point of inflexion
and x = 2 equals (c) Exactly one real zero in ( 0, 3 )
(a) 23 / 2 (b) 20 / 3 (c) 32 / 3 (d) 40 / 3 (d) Exactly one tangent parallel to X -axis

Monotonicity, Maxima and Minima Exercise 5 :


Matching Type Questions
93. Match the following :
Column I Column II
(A) The maximum value attained by y = 10 - | x - 10 |, - 1 £ x £ 3 is (p) 3
(B) If P (t , 2t ), t Î [ 0, 2 ] is an arbitrary point on parabola y = 4 x, Q is foot of perpendicular from
2 2
(q) 1
focus S on the tangent at P, then maximum area of D PQS is 3
æ 1ö æ 1ö (r) 5
(C) If a + b = 1, a, b > 0, then maximum value of ç1 + ÷ ç1 + ÷ is
è aø è bø
x+2 (s) 1
(D) For real values of x, the greatest and least value of expression is -
2x + 3x + 6
2
13
530 Textbook of Differential Calculus

94. Match the entries of the following two columns. Column I Column II
(C) f (x ) = {2 x} denotes fractional part of x (r) continuous
Column I Column II
is/has in (0, 1)
(A) The least value of the function (p) 5
(D) f (x ) = ì || x - 2 | - 2 |, x < 2
f (x ) = 2 × 33x - 32x × 4 + 2 × 3x (s) non-differe
í ntiable
î[ x ], x³2
in [– 1, 1] is
(B) The minimum value of the polynomial (q) – 1 (where [ × ] denotes the greatest integer
f (x ) = (x - 1) x (x + 1) is function), then f (x ) is/has in (– 1, 4)
(C) The value of the polynWmial (r) 3
3 96. Match the statements of Column I with the values of
ò -1 (| x - 2 | -[ x ]) dx (where [ × ] denotes the Column II.
greatest integer function) is Column I Column II
(D) If period of the function (s) 0 (A) f (x ) = cos px + 10x + 3x + x , 2 3
(p) 3/4
f (x ) = sin 36 x tan 42x is p, then
- 2 £ x £ 3. The absolute minimum
18p
equals value of f (x ) is
p
(B) If x Î [ - 1, 1] , then the minimum value (q) 2
95. Match the following : of f (x ) = x 2 + x + 1is
(C) 4 3
Column I Column II Let f (x ) = x - 4 x , 0 £ x £ 2. Then, (r) – 15
2 3
2x (p) local
(A) f (x ) = ò (t - 2)(t + 1)3 (t - 3)2 dt is/has the global minimum value of the
0 maxima
function is
in (– 1, 1)
(D) Let f (x ) = 6 - 12x + 9x 2 - 2x 3 , (s) – 8/3
ì æ px ö (q) local
ï sin çè ÷ø , x £ 2 1 £ x £ 4 . Then, the absolute
(B) f (x ) = í 4 is/has in (0,2) minima
maximum value of f (x ) in the
ï9 - 4x , x > 2
î interval is

Monotonicity, Maxima and Minima Exercise 6 :


Single Integer Answer Type Questions
97. A particular substance is being cooled by a streem of 100. Number of positive integral values of a for which the
cold air (temperature of the air is constant and is 5°C) curve y = a x intersects the line y = x is …… .
where rate of cooling is directly proportional to square
of difference of temperature of the substance and the air. 101. The least value of a for which the equation,
4 1
If the substance is cooled from 40°C to 30°C in 15 min and + = a has atleast one solution in the
temperature after 1 hour is T°C, then find the value of sin x 1 - sin x
[T ]/2, where [. ] represents the greatest integer function. interval (0, p /2) is ……… .
æ pö ì x 3/ 5 if x £ 1
tan ç x + ÷ 102. Let f ( x ) = í , then the number of
è 6ø î - ( x - 2) if x > 1
3
98. The minimum value of is ……… .
tan x critical points on the graph of the function are …… .
99. The figure shows a right triangle with its hypotenuse OB
along the Y -axis and its vertex A on the parabola y = x 2 .
103. The graph of y = f ¢¢( x ) for a function f is shown.
Let h represents the length of the hypotenuse which Number of points of inflection for y = f ( x ) is ……… .
depends on the x-coordinate of the point A. Y
The value of lim (h ) equals ……… .
x ®0 y = f′′(x)
Y

h
A(t,t 2) X
0 A B C D E
X
O
Chap 08 Monotonicity, Maxima and Minima 531

104. Number of critical points of the function, 107. If f ( x ) = ( x - a )( x - b ) for a, b Î R, then minimum
2 x x æ1 1 ö
f (x ) = x 3 - + ò ç + cos 2t - t ÷dt which lie number of roots of equation
3 2 1 è2 2 ø p( f ¢ ( x )) 2 cos( pf ( x )) + sin( pf ( x )) × f ¢¢( x ) = 0 in (a , b ),
in the interval [- 2p, 2p ] is ……… .
where f (a ) = 3 = f (b ) is ……… . (here, a < a < b < b)
105. Let f ( x ) and g ( x ) be two continuous functions defined
108. If absolute maximum value of
from R ® R, such that f ( x 1 ) > f ( x 2 ) and 1 1 p
g ( x 1 ) < g ( x 2 ), " x 1 > x 2 , then the least integral value of f (x ) = + is , (p , q are coprime) the
a for which f ( g (a 2 - 2a )) > f ( g (3a - 4 )) is ……… . | x - 4 | + 1 | x + 8| + 1 q
( p - q ) is ……… .
t + 3x - x 2
106. If the function f ( x ) = , where t is a
x-4
parameter, has a minimum and a maximum, then the
greatest value of t is ……… .

Monotonicity, Maxima and Minima Exercise 7 :


Questions Asked in Previous 10 Years’ Exams
(i) JEE Advanced & IIT-JEE
1
109. The least value of a Î R for which 4ax 2 + ³ 1 , for all 113. The function f ( x ) = 2 | x | + | x + 2 | - || x + 2 | - 2 | x || has a
x > 0, is x
local minimum or a local maximum at x is equal to
[One Correct Option 2016 Adv.] [More than One Correct Option, 2013 Adv.]
1 1 -2
(a) (b) (a) -2 (b)
64 32 3
1 1
(c) (d) (c) 2 (d) 2/3
27 25
114. A rectangular sheet of fixed perimeter with sides having
110. The number of points in ( - ¥, ¥ ) for which their lengths in the ratio 8 : 15 is converted into an open
x 2 - x sin x - cos x = 0, is [One Correct Option, 2013 Adv.] rectangular box by folding after removing squares of
(a) 6 (b) 4 equal area from all four corners. If the total area of
(c) 2 (d) 0 removed squares is 100, the resulting box has maximum
111. Let f : R ® (0, ¥ ) and g : R ® R be twice differentiable volume. The lengths of the sides of the rectangular sheet
are [More than One Correct Option, 2013 Adv.]
functions such that f ¢ ¢ and g ¢¢ are continuous funcitons
(a) 24 (b) 32 (c) 45 (d) 60
of R. Suppose f ¢ (2) = g (2) = 0, f ¢¢(2) ¹ 0 and g ¢ (2) ¹ 0. If
f ( x )g ( x ) 115. A vertical line passing through the point (h, 0) intersects
lim = 1, then
x ®2 f ¢ ( x )g ¢ ( x ) x2 y2
the ellipse + = 1 at the points P and Q. If the
[More than One Correct option, 2016 Adv.] 4 3
(a) f has a local minimum at x = 2 tangents to the ellipse at P and Q meet at the point R.
(b) f has a local maximum at x = 2 If D(h ) = area of the D PQR, D 1 = max D(h ) and
1/ 2 £ h £ 1
(c) f ¢¢(2 ) > f (2 ) 8
(d) f ( x ) – f ¢¢( x ) = 0 for atleast one x Î R
D 2 = min D(h ), then D 1 - 8D 2 is equal to
1/ 2 £ h £ 1 5
æ 1ö
- çt + ÷ [Integer Type Question, 2013 Adv.]
x è tø dt
112. If f :(0, ¥ ) ® R be given by f ( x ) = ò e . 116. Let f , g and h be real-valued functions defined on the
1/ x t 2 2 2 2
Then, [More than One Correct Option, 2014 Adv.] interval [0, 1] by f ( x ) = e x + e - x , g ( x ) = x e x + e - x
2 2
(a) f ( x ) is monotonically increasing on [1, ¥ ) and h ( x ) = x 2 e x + e - x . If a, b and c denote
(b) f ( x ) is monotonically decreasing on [ 0, 1 )
respectively, the absolute maximum of f , g and h on
æ1ö
(c) f ( x ) + f ç ÷ = 0, "x Î ( 0, ¥ ) [0, 1], then [One Correct Option, 2010]
èxø
(a) a = b and c ¹ b (b) a = c and a ¹ b
(d) f (2 x ) is an odd function of x on R (c) a ¹ b and c ¹ b (d) a = b = c
532 Textbook of Differential Calculus

117. The total number of local maxima and local minima of (c) g( x ) has no local minima
ì(2 + x ) 3 , - 3 < x £ -1 (d) f ( x ) has no local maxima
ï
the function f ( x ) = í 2 is 125. If f ( x ) is a cubic polynomial which has local maximum
ïx 3 , - < <
î 1 x 2 at x = - 1. If f (2) = 18, f (1) = - 1 and f ¢ ( x ) has local
[One Correct Option, 2008] minimum at x = 0, then [More than One Correct Option, 2006]
(a) 0 (b) 1 (c) 2 (d) 3 (a) the distance between (– 1, 2) and (a, f (a )), where x = a is
p p
118. If the function g : ( -¥, ¥ ) ® æç - , ö÷ is given by the point of local minima, is 2 5
è 2 2ø (b) f ( x ) is increasing for x Î[1, 2 5 ]
p
g (u ) = 2 tan -1(e u )- . Then, g is [One Correct Option, 2008) (c) f ( x ) has local minima at x = 1
2 (d) the value of f ( 0 ) = 5
(a) even and is strictly increasing in ( 0, ¥ )
(b) odd and is strictly decreasing in ( -¥, ¥ ) 126. Consider the function f :( -¥, ¥ ) ® ( -¥, ¥ ) defined by
x 2 - ax + 1
(c) odd and is strictly increasing in ( -¥, ¥ ) f (x ) = ; 0 < a < 2. Which of the following is
(d) neither even nor odd but is strictly increasing in ( -¥, ¥ ) x 2 + ax + 1
true ? [Passage Based Question, 2008]
119. The second degree polynomial f ( x ), satisfying f (0) = 0, (a) (2 + a ) f ¢¢(1 ) + (2 - a ) f ¢¢( -1 ) = 0
2 2

f (1) = 1, f ¢ ( x ) > 0"x Î (0, 1) [One Correct Option, 2005] (b) (2 - a ) 2 f ¢¢(1 ) - (2 + a ) 2 f ¢¢ ( -1 ) = 0
(a) f ( x ) = f
(c) f ¢(1 ) f ¢( -1 ) = (2 - a ) 2
(b) f ( x ) = ax + (1 - a ) x 2, " a Î ( 0, ¥ )
(c) f ( x ) = ax + (1 - a ) x 2, a Î ( 0, 2 ) (d) f ¢(1 ) f ¢( -1 ) = - (2 + a ) 2
(d) No such polynomial 127. Which of the following is true?
120. If f ( x ) = x + bx + cx + d and 0 < b < c, then in
3 2 2 [Passage Based Question, 2008]
(a) f ( x ) is decreasing on ( -1, 1 ) and has a local minimum
( -¥, ¥ ) [One Correct Option, 2004]
at x = 1
(a) f ( x ) is strictly increasing function (b) f ( x ) is increasing on ( -1, 1 ) and has a local maximum at x = 1
(b) f ( x ) has a local maxima (c) f ( x ) is increasing on ( -1, 1 ) but has neither a local
(c) f ( x ) is strictly decreasing function
maximum nor a local minimum at x = 1
(d) f ( x ) is bounded
(d) f ( x ) is decreasing on ( -1, 1 ) but has neither a local
121. If f ( x ) = x 2 + 2bx + 2c 2 and g ( x ) = - x 2 - 2cx + b 2, maximum nor a local minimum at x = 1
such that min f ( x ) > max g ( x ), then the relation ex f ¢ (t )
between b and c, is [One Correct Option, 2003] 128. Let g ( x ) = ò dt . Which of the following is true?
0
1+t2
(a) No real value of b and c (b) 0 < c < b 2 [Passage Based Question, 2008]

(c) | c | < | b | 2 (d) | c | > | b | 2 (a) g ¢( x ) is positive on ( - ¥, 0 ) and negative on ( 0, ¥ )


(b) g ¢( x ) is negative on ( - ¥, 0 ) and positive on ( 0, ¥ )
122. The length of a longest interval in which the function
(c) g ¢( x ) changes sign on both ( - ¥, 0 ) and ( 0, ¥ )
3 sin x - 4 sin x is increasing, is [One Correct Option, 2002]
3
(d) g ¢( x ) does not change sign ( - ¥, ¥ )
p p 3p
(a) (b) (c) (d) p
3 2 2 129. For the circle x 2 + y 2 = r 2 , find the value of r for
123. The maximum value of (cos a 1 ). (cos a 2 ) . ... . (cos a n ), which the area enclosed by the tangents drawn from
p the point P(6, 8) to the circle and the chord of contact is
under the restrictions 0 £ a 1, a 2 ...., a n £ and maximum. [Subjective Type Question, 2003]
2
(cot a 1 ).(cot a 2 ) . K .(cot a n ) = 1 is 130. Find a point on the curve x 2 + 2y 2 = 6 whose distance
[One Correct Option, 2001] from the line x + y = 7, is minimum.
1 1 1 [Subjective Type Question, 2003]
(a) (b) n (c) (d) 1
2n/2 2 2n
131. Let f : R ® R be defined as f ( x ) = | x | + | x 2 - 1 |. Total
ì ex , 0£ x £1
ï x -1 x number of points at which f attains either a local
124. If f ( x ) = í2 - e , 1 < x £ 2 and g ( x ) = ò f (t ) dt , maximum or a local minimum is
0
ï x -e , 2< x £3 [Integer Type Question, 2012]
î
x Î[1, 3], then [More than One Correct Option, 2006] 132. Let p ( x ) be a real polynomial of least degree which has a
(a) g( x ) has local maxima at x = 1 + loge 2 and local minima local maximum at x = 1 and a local minimum at x = 3. If
at x = e p(1) = 6 and p(3) = 2, then p ¢(0) is equal to
(b) f ( x ) has local maxima at x = 1 and local minima at x = 2 [Integer Type Question, 2012]
Chap 08 Monotonicity, Maxima and Minima 533

133. Let f be a function defined on R (the set of all real 134. The maximum value of the expression
numbers) such that f ¢ ( x ) = 2010 ( x - 2009 ) ( x - 2010) 2 1
is …… .
( x - 2011) 3 ( x - 2012) 4 , "x Î R. If g is a function defined sin q + 3 sin q cos q + 5 cos 2 q
2
[Integer Type Question, 2010]

on R with values in the interval (0, ¥ ) such that 135. The maximum value of the function
f ( x ) = ln ( g ( x )), " x Î R, then the number of points in R f ( x ) = 2x 3 - 15x 2 + 36x - 48 on the set
at which g has a local maximum is …… . A = {x | x 2 + 20 £ 9 x } is … . [Integer Type Question, 2009]
[Integer Type Question, 2010]

(ii) JEE Main & AIEEE


136. A wire of length 2 units is cut into two parts which are 138. Let a, b Î R be such that the function f given by
bent respectively to form a square of side = x units and a f ( x ) = log | x | + bx 2 + ax , x ¹ 0 has extreme values at
circle of radius = r units. If the sum of the areas of the x = - 1 and x = 2.
square and the circle so formed is minimum, then Statement I f has local maximum at x = - 1 and at x = 2.
[2016 JEE Main] 1 -1
(a) 2 x = ( p + 4 )r (b) ( 4 - p ) x = pr Statement II a = and b =
2 4 [2012 AIEEE]
(c) x = 2r (d) 2x = r
(a) Statement I is false, Statement II is true
137. If x = - 1 and x = 2 are extreme points of (b) Statement I is true, Statement II is true; Statement II is a
f ( x ) = a log| x | + bx 2 + x , then [2014 JEE Main]
correct explanation of Statement I
1 1 (c) Statement I is true, Statement II is true; Statement II is not
(a) a = - 6, b = (b) a = - 6, b = - a correct explanation of Statement I
2 2
1 1 (d) Statement I is true, Statement II is false
(c) a = 2, b = - (d) a = 2, b =
2 2

Answers
Exercise for Session 1 36. (a,b,c) 37. (c,d) 38. (b,d)
1. (d) 2. (a) 3. (c) 4. (a) 5. (b) 6. (c) 7. (b) 39. (b,d) 40. (a,b,c) 41. (a,b,d)
42. (a,b) 43. (b,c,d) 44. (b,c,d)
Exercise for Session 2 45. (a,b) 46.(d) 47. (d) 48. (a) 49. (a)
3 1
1. -5, 0, 2. 0, 3. No critical points 4. (c) 50. (d) 51. (a) 52. (b) 53. (b) 54. (a)
5 5 55. (d) 56. (a) 57. (a) 58. (d) 59. (d)
Exercise for Session 3 60. (c) 61. (b) 62. (c) 63. (a) 64. (c)
4. (a) 5. (b) 65. (a) 66. (a) 67. (c) 68. (b) 69. (a)
70. (a) 71. (a) 72. (a) 73. (b) 74. (b)
Exercise for Session 4
75. (b) 76. (d) 77. (a) 78. (d) 79. (c)
1. (b) 2. (d) 3. (a) 4. (a) 5. (a)
80. (c) 81. (a) 82. (c) 83. (b) 84. (b)
6. (c, d) 7. (a,c) 8. (d) 9. (a) 10. (c)
85. (a) 86. (c) 87. (d) 88. (c) 89. (b)
11. (c) 12. (d) 13. (b) 14. (d) 15. (a)
90. (c) 91. (d) 92. (c)
Exercise for Session 5 93. (A) ® (p), (B) ® (r), (C)® (p), (D) ® (q, s)
1. (d) 2. (a) 3. (b) 4. (a, c) 5. (c) 6. (d) 94. (A) ® (s), (B) ® (q), (C) ® (r), (D) ® (r)
7. -2 < a < 2 95. (A) ® (p, r), (B) ® (p, r, s), (C) ® (p, s), (D) ® (q, s)
Chapter Exercises 96. (A) ® (r), (B) ® (p), (C) ® (s), (D) ® (q)
97. (9) 98. (3) 99. (1) 100. (1) 101. (9)
1. (c) 2. (c) 3. (a) 4. (a) 5. (c)
102. (3) 103. (2) 104. (4) 105. (2) 106. (3)
6. (b) 7. (a) 8. (d) 9. (d)
107. (4) 108. (1) 109. (c) 110. (c) 111. (a,d) 112. (a, c, d)
10. (a) 11. (c) 12. (a) 13. (d) 14. (b)
113. (a,b) 114. (a,c) 115. (9) 116. (d) 117. (a) 118. (c)
15. (a) 16. (c) 17. (a) 18. (c) 19. (c) 119. (c) 120. (a) 121. (d) 122. (a) 123. (a) 124. (a,b)
20. (d) 21. (b) 22. (c) 23. (c) 24. (d) 125. (b,c) 126. (a) 127. (d) 128. (b) 129. (5) 130. (2,1)
25. (d) 26. (d) 27. (b) 28. (c) 29. (b,c) 131. (5) 132. (9) 133. (1) 134. (2) 135. (7) 136. (c)
30. (a,d) 31. (c,d) 32. (a,b,c,d) 137. (c) 138. (c)
33. (a,b,c,d) 34. (a,b,c) 35. (a,b,c)
= ax 2 + (b - 2a ) x + (2a - b + c )

Solutions Þ

Þ
D = (b - 2a ) 2 - 4a (2a - b + c )
= (b 2 - 4ac ) - 4a 2 < 0
g (x ) > 0, " x Î R
11. We have, f ( x ) = min {1, cos x, 1 - sin x }
[using Eq. (i)]

ì ì
1. Since, f is non-decreasing and g is non-increasing, so h is a ï cos x, -p £ x £ 0 ï - sin x, - p £ x £ 0
non-increasing function. Now, h(1 ) = 1 ï ï
Þ h ( x ) is a constant function ï p ï p
f ( x ) = í1 - sin x, 0 < x £ Þ f ¢( x ) = í - cos x, 0 < x £
Þ h(2 ) = 1 ï 2 ï 2
ï p ï p
2. Since, maximum or minimum distance between two curve ïî cos x , <x£p ïî - sin x, 2 < x £ p
always measure along common normal. AP is perpendicular on 2
the tangent drawn to the curve. Therefore, f ¢( 0 ) = 0
3. f is continuous at 0 and f ¢( 0-) > 0 and f ¢( 0+ ) < 0 . Thus, f Hence, f ( x ) has local maximum at 0 and f ( x ) is not
has a local maximum at 0. p
differentiable at x = 0 and
x 2
4. Here, f ( x ) = ò (t - a ) 2n (t - b ) 2m + 1dt, a ¹ b
1 Aliter
f ¢( x ) = ( x - a ) 2n ( x - b ) 2m + 1. Y
y=2
Obviously, f ¢(a - ), f ¢(a + ) > 0
while f ¢(b - ) < 0 and f ¢(b + ) > 0 y=1
Hence, x = b is a point of local minima.
5. f ¢( x ) = xn - 1[(n + 1) x + an ]. 1 – sinx
X′ –π π π π X
If n is even then, f ¢( 0 - ) < 0 and f ¢( 0 + ) > 0 – O
2 2
Hence, 0 is a point of minimum when n is even.
4 4 4
6. f ¢( x ) = 4 x - 3 = 3 ( x 4 - 1 ) = 3 ( x - 1 )( x + 1 )( x 2 + 1 )
x x x Y′ cos x
Þ f ¢( x ) = 0 at x = 1 and -1 From the graph it is clear that f ( x ) is not differentiable at
12
f ²( x ) = 4 + 4 > 0 for x = ± 1 p
x = 0, and f ( x ) has occurs local maximum at x = 0.
x 2
Þ x = 1 and -1 are points of local minima.
12. f ( x ) will have maxima only, if a 2 + 1 ³ 2 Þ | a | ³ 1
1 1
7. Since, y min = 1/e > . So, graph always lie above the line y = x. Y
e e
Hence, 0 < x < 1 is correct answer.
2 2
æ 1ö 1 2 – |x 2 + 5x + 6|
8. y - y + a = çy - ÷ + a -
2
è 2ø 4
X¢ X
Since, - 2 £ sin x + cos x £ 2, given equation will have no –3 –2 0
1
real value of x for any y, if a - > 2 Y¢
4
æ 1 ö 13. ax = - x - bx - c. For a = -1 and b = c = 0 and n = 2, it will
n 2

i.e. a Î ç 2 + , ¥ ÷ have infinite solutions.


è 4 ø
é 1 ù 14. A = Area = sin x ( p - 2x ) (as l = p - 2x, b = sin x)
Þ a Î ( 3, ¥) êë as 2 + 4 < 3 úû dA p
2 = ( p - 2 x ) cos x - 2 sin x = 0 Þ tan x = - x
2e - x 2 dx 2
9. f ( x ) = 1 - 2 =1- 2
p
-x 2
e +e
x
e 2x
+1 Let f ( x ) = tan x + x -
As x ® +¥, f ( x ) ® 1 2
Y
As x ® -¥, f ( x ) ® 1
Þ f ( x ) is increasing as well as decreasing in some intervals. sinx
Since, the range of f ( x ) is [ 0, 1 ) which does not coincide with
the co-domain R and hence f is not an onto function.
p - 2x X
O x
10. Let f ( x ) = ax 2 + bx + c > 0, " x Î R
Þ b 2 - 4ac < 0 and a > 0 ... (i) æpö æpö
f ç ÷ is negative, f ç ÷ is positive.
è6ø è 4ø
Now, g ( x ) = f ( x ) - f ¢( x ) + f ¢¢( x )
= (ax 2 + bx + c ) - (2ax + b ) + 2a æp pö
So, one root lies between ç , ÷ .
è6 4ø
Chap 08 Monotonicity, Maxima and Minima 535

15. f ( x ) + 1 = k ( x + 1) always passes through ( -1, - 1). Clearly, its 21. Let P ( x ) is of degree n, then
maximum slope can go upto ¥. For minimum slope this line Þ 2n = n + 2 [as P ( x 2 ) = x 2P ( x )]
should touch y = ln x .
dy 1 æ1 ö Þ n =2
= = k . So, ç , - ln k ÷ is point of tangency. P ( x ) = ax 2 + bx + c form
dx x èk ø
Y (1/k, -ln k) P( 0 ) = - 2
Þ c = -2 …(i)
P(1 ) = 0
Þ a+b+c=0 …(ii)
æ3ö
X¢ X P ¢ç ÷ = 0 Þ 3a + b = 0
è2ø
a -1
(–1, –1) Þ = Þ P (x ) = - x 2 + 3x - 2
Y¢ b 3
1
- lnk + 1 Hence, maximum P ( x ) = -
Now, =k Þ - lnk + 1 = k + 1 4
1
+1 22. In [0, 1], [ x 2 - x + 1] = 0
k
é 1+ 5ù
Þ - lnk = k ln ê1, ú , [ x - x + 1 ] = 1 Þ x = - 4 (y - a )
2 2

Let f ( x ) = k + ln k ë 2 û
æ1ö 1 Put y = 0, x 2 = 4 (a - 1 ) Þ 4a > 1
f ç ÷ = -1 [negative]
èe ø e Put y = 1, x 2 = 4 (a - 1 ) Þa < 1
æ 1 ö 1 1 1
fç ÷= - [positive] Þ < a < 1.
è eø e 2 4
1 1 23. Clearly, f ¢(a ) = 0, f ¢(c ) = 0, f ¢(e ) = 0, x = b and x = d two
So, one root must lie between and ×
e e points of inflexion f ¢¢¢( x ) > 0, d < x < e.
16. f ( x ) = f ¢( x ) ´ f ¢¢¢( x ) is satisfied only by the polynomial of x = e is point of local minima.
degree 4. Since, f ( x ) = 0 satisfies x = 1, 2, 3 only. It is clear that 24. g ( x ) = f ( x 2 - 2x - 1) + f (5 - x 2 + 2x )
one of the roots is repeated twice.
= 2 x 4 - 8 x 3 - 4 x 2 + 24 x + 18
Þ f ¢(1 ) f ¢(2 ) f ¢(3 ) = 0
g ¢( x ) = 8 x 3 - 24 x 2 - 8 x + 24
d 2y dy
17. 2
= k log x Þ = k ( x log x - x ) + A g ¢( x ) = 0 Þ x = - 1, 1, 3
dx dx
= kx (log x - 1 ) + A We observe that g ( x ) ³ min { g ( -1 ), g (1 ), g (3 )} = 0
é x2 x2 ù ∴ g ( x ) ³ 0, " x Î R
= k ê(log x - 1 ) - ú + Ax + B
ë 2 4û 25. Using the fact that every set of natural numbers have the
2 smallest element.
x
= k (2 log x - 3 ) + Ax + B ∴ f ( f (1 )) is the smallest element in { f ( f (1 )), f (2 ), f ( f (2 )), ... }
4
Þ y = c1(2 log x - 3 ) x 2 + c 2x + c 3 Same argument implies that f (1 ) = 1
Repeating the argument for f {n ³ 2 } ® {n ³ 2 }, we get f (2 ) = 2
éæ 1ö
2
11 ù
18. f ¢( x ) = 3 sec 2 ê ç tan x - ÷ + ú > 0 for all x in its domain. ∴ Clearly, f ( x ) = x
è 3ø
ëê 9 ûú
1
26. = | 2ax -3 | + | ax + 1 | + | 5 -ax | ³ | 2ax -3 –(ax + 1 ) +5 - ax | = 1
19. lim f ( x ) = lim 3 + h = 3 Þ f (k ) = 3 2
x ®k- h®0
which is impossible.
sin h
and lim f ( x ) = lim a 2 - 2 + = a2 - 2 + 1 2
27. Given, ò 2x f 2(t ) dt = æç ò 2 f ( x - t ) dt ö÷
x x
x ®k + h®0 h 0 è 0 ø
Since, f ( x ) has minimum at x = k 2
x × ò f 2(t ) dt = 2 æç ò f (t ) dt ö÷ , using ò f ( x ) dx
x x a
f (k - ) > f (k ) Þ
0 è 0 ø 0
and f (k + ) > f (k ) Þα 2 - 2 + 1 > 3 a
= ò f (a - x ) dx
|a | > 2 0
Differentiating, we get
20. Let f ( x ) = mx + b x x
Q f (1 ) £ f (2 ) Þ m ³ 0, similarly x × f 2( x ) + ò f 2(t ) dt = 4 ò f (t ) dt × f ( x )
0 0

f (3 ) ³ f ( 4 ) Þ m £ 0 Þm = 0 [using Leibnitz rule]


x x
∴ f ( 0 ) = f (5 ) = 5 Þ x f ( x ) + x × ò f (t ) dt = 4 x × f ( x ) × ò f (t ) dt
2 2 2
0 0
536 Textbook of Differential Calculus

2 ln(ln x ) 1 - ln(ln x )
x 2 f 2( x ) + 2 æç ò f (t ) dt ö÷ = 4 x f ( x ) × ò f (t ) dt
x x
Þ 29. Let f ( x ) = Þ f ¢( x ) = < 0, " x > ee
è 0 ø 0 ln x x(ln x ) 2
2 So, f ( x ) is increasing in (1, ee ) and decreasing in (ee , ¥ ).
Þ 2 æç ò f (t ) dt ö÷ - 4 x × f ( x ) × ò f (t ) dt + x 2 f 2( x ) = 0
x x
è 0 ø 0 Therefore, x > y
æQax 2 + bx + c = 0 ö Þ (ln x ) ln y < (ln y ) ln x , " x, y Î (ee , ¥ ) and x < y
x 4 x f ( x ) ± 16 x 2 f 2 ( x ) - 8 x 2 f 2 ( x ) ç ÷
ò0 f (t )dt = 4 ç
çÞ x =
-b ± d ÷
÷
Þ (ln x ) ln y < (ln y ) ln x , " x, y Î (1, ee )
è 2a ø
30. lim[ f ( x )] can exist only when f ( x ) either increases
x x ®a
Þ 2 ò f (t ) dt = ( 2 ± 1 ) x f ( x )
0 or decreases at both sides of the point x = a .
Again, differentiating, we get Since, lim f ( x ) = lim[ f ( x )]
x ®a x ®a
2 f ( x ) = ( 2 ± 1 )( f ( x ) + x f ¢( x ))
So, this can occur only when lim f ( x ) is an integer.
Þ 2 f ( x ) = ( 2 ± 1 ) f ( x ) + ( 2 ± 1 ) x × f ¢( x ) x ®a

Þ ± f ( x ) = ( 2 ± 1 ) × x f ¢( x ) 31. f ( x ) = 2x 3 - 3 (2 + l) x 2 + 12lx
1 1 Þ f ¢( x ) = 6 x 2 - 6 (2 + l ) x + 12 l
Þ ò ± 2 ± 1 × x dx Þ f ¢( x ) = 0 Þ x = 2, l
f ¢( x ) If f ( x ) has exactly one local maximum and exactly one local
=ò dx
f (x ) minimum, then l ¹ 2.1
Þ ± ( 2 ± 1 ) × log x = log| f ( x )| + C, as f (1 ) = 1 ÞC = 0 æ x2 ö æ æ x2 ö ö
32. h ( x ) = 3 f ç ÷ + f (3 - x 2 ) Þ h ¢( x ) = 2 x ç f ¢ ç ÷ - f ¢(3 - x 2 ) ÷
Þ f (x ) = x 2 ±1 è ø
3 è è ø 3 ø
2 +1 æ x2 ö x2 9
\ f (x ) = x [as f ¢( x ) > 0] Þ f ¢ ç ÷ > f ¢(3 - x 2 ), " x such that > 3 - x2 Þ x2 >
2 -1 è 3 ø 3 4
\ f (n ) =n ...(i)
æ x2 ö 9
Now, let an = cot qn Þ f ¢ ç ÷ < f ¢(3 - x 2 ), " x such that x 2 <
æq ö è 3 ø 4
Þ an + 1 = cot ç n ÷
è2ø - + - +
1 –3 –3/2 0 3/2 4
Þ qn + 1 = qn
2 using number line rule
Q q0 = p / 2 æ 3 ö æ3 ö
1æpö 1 æpö Þ h ( x ) increases in ç - , 0 ÷ È ç , 4 ÷
Þ q1 = ç ÷, q2 = 2 ç ÷ è 2 ø è2 ø
2è2ø 2 è2ø
æ 3ö æ 3ö
1 p æ p ö andh ( x ) decreases in ç - 3, - ÷ È ç 0, ÷
Þ qn = n ´ \ an = cot ç n + 1 ÷ è 2ø è 2ø
2 2 è2 ø
cos x - sin x
an 1 33. f ¢( x ) = ³ 0 Þ cos x ³ sin x
Þ lim = -1 = 4/ p 1 + (sin x + cos x ) 2
n ® ¥ 2n -1 2 × p /2
é 7p ù é 3p p ù é 5p ù
28. We have, f ( x ) = | x |m | x - 1 |n Þ x Î ê - 2 p, - ú È ê - , ú È ê , 2 p ú
ë 4 û ë 4 4 û ë 4 û
ì( -1 )m + n xm × ( x - 1 )n , if x<0
ï
f (x ) = í ( -1 )n xm ( x - 1 )n , if 0 £ x <1
ï xm ( x - 1 )n , if x ³1
î O
-2p p 2p
Let, g( x ) = xm ( x - 1 )n , then
g ¢( x ) = mxm -1 × ( x - 1 )n + n xm × ( x - 1 )n -1
= xm -1 × ( x - 1 )n -1 × (m( x - 1 ) + n × x ) = 0 cos2 x 2
sin 2 x
Now, f ¢( x ) = 0 34. Applying C1 ® C1 + C 2, we get 2 1 + cos2 x sin 2 x
Þ g ¢( x ) = 0 1 cos2 x 1 + sin 2 x
m
Þ x = 0 or or 1. Applying R2 ® R2 - R1 and R3 ® R3 - R1, we get
m+n
f ( 0 ) = 0, f (1 ) = 0 2 cos2 x sin 2 x
æ m ö n m × n × ( -1 )
m n n
é m ù 0 1 0 = 2 + sin 2 x
fç ÷ = ( -1 ) × êë as, 0 < m + 1 < 1 úû
èm + nø (m + n )m + n -1 0 1
m ×n
m n Since, the maximum value of sin2x is 1 and minimum value of
\ Maximum value = sin2x is (– 1). Therefore, a = 3, b = 1.
(m + n )m + n
Chap 08 Monotonicity, Maxima and Minima 537

35. Y Also, f (1 ) = 0, lim ® - ¥, lim f ( x ) ® ¥


x ®e x ® 0+
- ln x
f ¢¢( x ) = 2
1 π x (1 - ln x ) 2
O π
X¢ 2 X f ¢¢(1 ) = 0 which is a point of inflection as shown in graph
–4 –3 –2 –1
–1
Y -axis and x = e are two asymptotes.
–2 –sin x –(1+cos x) 41. f is obvious continuous, " x Î R and not derivable at - 1 and 1.
–3 f ¢( x ) changes sign 4 times at - 1, 0, 1, 2
–4 Y
x2 + 4x
Y′
From above figure clearly options (a), (b) and (c) are correct.
X
–2 –1 0 1 2
36. f ( x ) = a 2b 2 + b 2 - b 2a 2 sin( x + a ) + c
1 -a2 Local maxima at 1 and - 1.
= b sin( x + a ) + c, where tan a =
a Local minima at x = 0 and 2.
p
f max = b + c 42. f ( x ) = ò cost cos ( x - t ) dt …(i)
f min = -b + c 0
p
( f ( x )) max - ( f ( x )) min = 2b = ò - cost × cos ( x - p + t ) dt
0
Also, at x = - cos-1 a, f ( x ) = c. p
f ( x ) = ò - cost × cos ( x + t ) dt …(ii)
n × xn - 1 1 × mxm - 1
0
37. f ¢( x ) = - , On adding Eqs. (i) and (ii), we get
(ln xn ) (ln xm ) p
Clearly, (c) and (d) are the answers. 2 f ( x ) = ò cos t (2 cos x × cos t ) dt
0
3 p /2
38. f ( x ) occurs maximum at x =
2
2 f ( x ) = 2 cos x ò0 cos2 t dt
-b p cos x
g( x ) occurs maximum at x = f (x ) = Now, verify.
2 2
3 -b Only (a) and (b) are correct.
\ = Þ b = -3
2 2 Aliter Convert the integer and into sum of two cosine functions.
| x| | x|
ln( f ( x ) × g( x )) ln a {a sgn x } + [a × sgn x ] 2-x x -3
39. h( x ) = = 43. f ¢( x ) = 3
and f ¢¢( x ) = 4 Now, interpret
ln a ln a x x
= {a| x| × sgn x } + [a| x| × sgn x ] = a| x| sgn x Y
ì a for x > 0
x
ï 1/4
= í 0 for x = 0 [Q {y } + [y ] = y ]
ï - a - x for x < 0 0
X
î 1 2 3
Þ h( x ) is an odd function.
40. f ( x ) = ln (1 - ln x ), Domain ( 0, e )
1 1 a f ( x ) - x f (a )
f ¢( x ) = - × <0 é0 ù
(1 - ln x ) x 44. We have, lim êë 0 form úû
a®x a -x
Y x=e f ( x ) - x f ¢(a )
Þ lim =2
a®x 1
1 Þ f ( x ) - x f ¢( x ) = 2
X f ¢( x ) 1
(1, 0) Þ =
f (x ) - 2 x
On integrating both sides w.r.t. x, we get
Þ Decreasing, " x in its domain f ¢( x ) 1
Þ (a) and (b) are incorrect. ò f (x ) - 2 dx = ò x dx
f ¢(1 ) = - 1 Þ (c) is also incorrect.
538 Textbook of Differential Calculus

Þ log ( f ( x ) - 2 ) = log x + log c Hence, g ( x ) is a decreasing function.


Þ c +1 >c
x+
2 g (c + 1 ) < g (c )
=3 (2, 8)
f(x) Þ f (u ) < f (v )
B(0, 2) é pù
51. Let h ( x ) = f ( x ) - sin x or x Î ê 0, ú
ë 2û
X′ X æpö
0 2 Þ h( 0 ) = h ç ÷ = 0
–2
A ,0
3
è2ø
æ pö
According to Rolle's theorem for atleast one c Î ç 0, ÷ ×
è 2ø
Þ f ( x ) = cx + 2 h ¢(c ) = f ¢(c ) - sin c = 0
As, f (1 ) = 5, so 5 = c + 2 Þ c = 3 é p 3p ù æ p ö æ 5p ö
for x Îê , , hç ÷ = hç ÷ = 0
Hence, f (x ) = 3x + 2 ë 2 2 úû è 2 ø è 2 ø
1 æ2ö 2 æ p 3p ö
Clearly, area ( DOAB ) = ç ÷ (2 ) = According to Rolle's theorem for atleast one d Î ç , ÷×
è
2 3 ø 3 è2 2 ø
2 æ 3x 2 ö
2 h ¢(d ) = f ¢(d ) - cos d = 0
Also, ò0(3x + 2)dx = çè 2 + 2x ÷ø for x Î [c, d ], h ¢(c ) = h ¢(d ) = 0
0
According to Rolle's theorem for atleast one x Î(c, d ).
= 6 + 4 = 10
d F (x ) h ¢( x ) = f ¢¢( x ) + sin x = 0
45. Given, 2( F ( x ) - f ( x )) = f 2( x ) and = f (x ) ...(i) æ 3p ö
dx Þ | f ¢¢( x )| £ 1 for atleast one x Î ç 0, ÷.
è 2 ø
f 2( x )
\ F (x ) = + f (x )
2 52. Statements I and II both are true but Statement II does not
Þ F ¢( x ) = f ( x ) × f ¢( x ) + f ¢( x ) explain Statement I.
\ f ( x ) = f ( x ) × f ¢( x ) + f ¢( x ), using F ¢( x ) = f ( x ) 53. Statement II is true as f ¢(a ) + = f ¢(a ) -, " a Î I .
f (x ) Statement I is true and is obtained from differentiable rule.
Þ f ¢( x ) = , as f ( 0, ¥ ) ® ( 0, ¥ )
1 + f (x ) 54. Statement II is correct as y = f ( x ) is odd and hence Statement I
\ f ¢( x ) > 0 is correct.
f (x ) f ¢( x ) Sol. (Q. Nos. 55 to 57)
Hence, f is strictly increasing and lim = lim
x®¥ x x®¥ 1 -2 x 6x 2 - 2
Here, we have f ¢( x ) = and f ¢¢( x ) =
[using L’Hospital’s rule] (1 + x )
2 2
(1 + x 2 ) 3
æ 1 ö 1
= lim ç1 + ÷ , as x ® ¥ ; f ( x ) ® ¥ \ f ¢( x ) is maximum at x = -
x ® ¥è 1 + f (x ) ø 3
f (x ) 3 3
\ lim =1 If m is greatest, then m =
x®0 x 8
3
46. If b < 0, then f ( 0) = b < 0, f (1) = b < 0 y-coordinate of the point of contact is ×
4
\ 0, 1 lies between the roots, Statement I is false.
3 3 3æ 1 ö
47. x = 2 is a point of local minima. \ Equation of the tangent is y - = çx + ÷
4 8 è 3ø
48. f¢( x ) = 3 sin x + 4 cos x > 0 9
ép pù \ a = - 3 and b =
f( x ) is increasing in ê , ú . 8
ë6 3û
55. (d) 56. (a) 57. (a)
p
∴ f ( x ) attain maximum value at x = × Sol. (Q. Nos. 58 to 60)
3
We draw the graphs of f1( x ) = x 2, f 2( x ) = (1 - x ) 2 and
49. Let g ( x ) = f ( x ) × f ¢( x ) f 3( x ) = 2 x (1 - x ). Here, f ( x ) is redefined as
Þ g ¢( x ) > 0 in [a, b]. Y
50. Let g ( x ) = x - x - 1, x > 1 f(x) = (1 – x)2
1 1 1 æ x -1 - x ö f (x) = 2x (1 – x)
Þ g ¢( x ) = - = ç ÷ f(x) = x2
2 x 2 x - 1 2 è x (x - 1) ø
-1
= < 0, " x > 1
2 x x -1( x + x -1)

O X
Chap 08 Monotonicity, Maxima and Minima 539

ì 1 f ( -10 ) = a 0 + 270 - 1000


ï (1 - x ) , 0 £ x < 3
2
Clearly, f ( -10 ) is defined for a 0 > 830
ï
ï 1 2
f ( x ) = í 2 x (1 - x ), £ x £ 67. (c) 68. (b) 69. (a)
ï 3 3
Sol. (Q. Nos. 70 to 74)
ï 2
2
x , < x £1
îï 3
70. Graph of f ( x ) is shown.
Y
æ1 1ö æ2 ö
Interval of increase of f ( x ) is ç , ÷ È ç , 1 ÷ ×
è3 2ø è3 ø
asymptote
æ 1ö æ1 2ö
Interval of decrease of f ( x ) is ç 0, ÷ È ç , ÷ ×

asymptote

asymptote
è 3ø è2 3ø b1
a1 a X
b
é1 2ù
Clearly, Rolle’s theorem is applicable on ê , ú ,
ë3 3û
where f ( x ) = 2 x (1 - x ).
1
Þ f ¢(c ) = 2 - 4c = 0 Þ c =
2
1 2 1 3 Clearly, f ( x ) is increasing in ( a1, b1 ).
Þ a +b +c = + + =
3 3 2 2 71. Clearly, f ( x ) has a maximum in [ a1, b1 ] and a minima in [ a, b],
58. (d) 59. (d) 60. (c)
shown as.
Sol. (Q. Nos. 61 to 63)
According to paragraph
f (b ) - f (a ) g (b ) - g (a ) h (b ) - h (a )
> f ¢(c ), = g ¢(c ) and < h ¢ (c )
b -a b -a b -a y=1
As, f ¢( x ) > g ¢( x ) > h ¢ ( x ) a1 a b
y=0 b1
f (b ) - f (a ) g (b ) - g (a ) h (b ) - h (a )
Þ > >
b -a b -a b -a
If g ( x ) = Ax + Bx + C
2

g (b ) - g (a ) A (b 2 - a 2 ) + B (b - a )
Þ = 72. f ( x ) has one of the two graphs.
b -a b -a
⇒ f ¢( x ) = 0 has real and distinct roots.
(b + a ) æ b + a ö
Þ 2A + B = g ¢ç ÷ f ′(x) = 0
2 è 2 ø
61. (b) 62. (c) 63. (a)
Sol. (Q. Nos. 64 to 66) x1
y=1
an > an -1 iff n + c is a perfect square, since a 2 > a1 and a 5 > a 4 . x2
Þ 2 + c and 5 + c are perfect squares
Þ c = -1 ...(i)
y=0
Now, a2 = 3 = b [ 2 + c ] + d f ′(x) = 0
f ′(x) = 0
∴ b +d =2 ...(ii)
Also, a10 > a 9
Þ a10 = 7 = 3b + d ...(iii)
x2
On solving Eqs. (ii) and (iii), we get b = 2, d = 1 y=1
x1
64. (c) 65. (a) 66. (a)
Sol. (Q. Nos 67 to 69)
f ′(x) = 0
Dh = { -1, 1 }, as minimum occurs before maxima y=0
∴ a3 = - 1 73. Clearly, lim [ f ( x )] = 0 and lim [ f ( x )] = 1
x®¥ x® ¥
Now, g ( x ) = a 0 + a1x + a 2x 2 - x 3
g ¢( x ) = a1 + 2a 2x - 3 x 2 = - 3 ( x - 3 )( x + 3 ) = - 3 x 2 = 27 ∴ lim [ f ( x )] lim [ f ( x )] = 0
x ®¥ x® ¥

∴ a1 = 27, a 2 = 0
∴ a1 + a 2 = 27
Also, g ( -3 ) > 0 and g (3 ) > 0 Þ a 0 > 54 and a 0 < - 54
\ a 0 > 54
Now, g ( x ) = a 0 + 27 x - x 3 Þ f ( x ) = a 0 + 27 x - x 3
540 Textbook of Differential Calculus

x 2 + bx + c (b - b1 ) x + (c - c1 ) 80. Clearly, one of the graph of f ( x ) is


74. f ( x ) = =1+
x + b1x + c1
2
x 2 + b1x + c1 Y
(b - b1 ) (c - c1 ) (–5,12)
+
=1+ x x2 f ¢(–5) = 0 f ¢(4) = 0
b1 c1
1+ + 2
x x
For b > b1 , lim f ( x ) ® 1 + X
x®¥ 0
–5 2 4
⇒ Point of maxima is greater than point of minima.
Sol. (Q. Nos. 75 to 77)
x2 Sol. (Q. Nos. 81 to 83)
y = , not defined at x = ± 1 f ( x ) = e ( p +1) x - e x = e x (e px - 1 )
x2 - 1
1 2x f ¢( x ) = e x (e px - 1 ) + e x × pe px
=1 + 2 Þ y¢ = - 2
x -1 (x - 1)2 = e x [( p + 1 )e px - 1 ] = 0 [e x ¹ 0 ]
dy 1
=0 Þ e px = Þ px = - loge ( p + 1 )
dx p+1
Þ x=0 [point of maxima] log ( p + 1 )
x=- e ...(i)
as x ® 1 + , y ® ¥, x ® 1 - , y ® - ¥ p

Similarly, x ® - 1 + , y ® - ¥, x ® - 1 - , y ® ¥ Now, f ¢¢( x ) = ( p + 1 ) 2e ( p + 1) x - e x


= e x [( p + 1 ) 2e px - 1 ]
Y
æ log( p + 1 ) ö
\ f ¢¢ç - ÷ = e [( p + 1 ) - 1 ] > 0
x
è p ø
1 log ( p + 1 )
Hence, x = s p = - e
0 p
X
–1 1 t + 1 px t + 1 px
g(t ) = ò (e × e x - e x ) et × e - x dx = et òt (e - 1 ) dx
t
On integrating, we get
t +1
æ e px ö é e pt (e p - 1 ) ù
x2 g(t ) = et × ç - x÷ = et ê - 1ú
The graph of y = 2 is as shown è p øt p
x -1 ë û
verify all alternatives from the graph. (e p - 1 )e ( p + 1) t
\ g(t ) = - et
75. (b) 76. (d) 77. (a) p
Sol. (Q. Nos. 78 to 80) (e p - 1 ) ( p + 1)t t
g ¢(t ) = ( p + 1 ) e -e = 0
From given statements (i) to (v), one of the graph of f ( x ) can p
be plotted as ( p + 1 )(e p - 1 ) pt p
Þ e = 1 Þe pt =
f ¢(– 5) = 0 p ( p + 1 )(e p - 1 )
Y
(– 5,12)
12 æ p ö
pt = loge ç ÷
De

è ( p + 1 )(e p - 1 ) ø
g

cr
s in

ea

æ ( p + 1 )(e p - 1 ) ö
rea

1
ing

\ t = - log ç ÷ = tp
Inc

f ′(4) = 0
p è p ø
X 1 æ ( p + 1 )(e p - 1 ) ö log ( p + 1 )
–5 2 4 Also, s p - t p = log ç ÷-
p è p ø p
1 æep -1ö
=
log ç ÷
p è p ø
f ′(2) is not defined 1 æep -1ö
Hence, lim (s p - t p ) = lim log ç ÷
p ®0 +
p®0 + p
è p ø
78. Since, f ¢(2) is not defined and continuous for x Î R.
æ ep - p -1ö
Þy = f ( x ) must have a geometrically sharp corner at x = 2. log ç1 + ÷
è p ø æep - p -1ö æep - p -1ö 1
79. At x = - 5, f ¢( x ) changes from positive to negative and at = lim ×ç ÷ = lim ç ÷=
x = 4, f ¢( x ) change sign for positive to negative, hence maxima
p®0 æe - p -1ö
p
è p ø p ® 0+ è p2 ø 2
ç ÷×p
at x = - 5 and 4. f is continuous and f ¢(2 ) is not defined, hence è p ø
x = 2 must be geometrical sharp corner.
Chap 08 Monotonicity, Maxima and Minima 541

81. (c) 82. (c) 83. (b) 89. Also, range of f ( g( x )) = { -1, 0, 1}
Sol. (Q. Nos. 84 to 86) \ f ( g( x )) Î { 0, 1 }
84. Here, h( x ) = f 2( x ) + g 2( x ) Hence, range of sin -1 ( fog )( x ) Î { 0, p / 2 }
= (sin 3 x + cos x ) 2 + (cos3 x + sin x ) 2 = 2 + 2 sin 4 x
Sol. (Q. Nos. 90 to 92)
Graph of y = sin 4 x We have, g( x ) = x 3 + g ¢¢(1 ) x 2 + {3 g ¢(1 ) - g ¢¢(1 ) - 1 } x + 3 g ¢(1 )
Y Let g ¢(1 ) = a, g ¢¢(1 ) = b, then
g( x ) = x 3 + bx 2 + (3a - b - 1 ) x + 3a
1

Differentiating both sides w.r.t. x, we get
8
–π 0 π π π X g ¢( x ) = 3 x 2 + 2bx + (3a - b - 1 )
8 8 4 2 Put x = 1,
–1
Þ g ¢(1 ) = 3 + 2b + 3a - b - 1 [Q g ¢(1 ) = a ]
Þ a = b + 3a + 2 or 2a + b = -2 ...(i)
Clearly, h( x ) is periodic function with period p/2 and from
above graph, the length of a longest interval in which the Again differentiating, we get
function y = h( x ) is increasing = p / 8 - ( - p / 8 ) = p / 4 g ¢¢( x ) = 6 x + 2b
85. We have, h( x ) = 4 Put x = 1,
Þ 2 + 2 sin 4 x = 4 Þ sin 4 x = 1 Þ g ¢¢(1 ) = 6 + 2b [\ g ¢¢(1 ) = b ]
\ 4 x = 2np + p / 2 = ( 4n + 1 ) p / 2 Þ b = 6 + 2b
Þ x = ( 4n + 1 ) p / 8, n Î I Þ b = -6 ...(ii)
86. We have, f ( x ) = g( x ) From Eq. (i), a = 2
Þ sin 3 x + cos x = cos3 x + sin x \ g( x ) = x 3 - 6 x 2 + 11 x + 6
Þ sin 3 x - sin x = cos3 x - cos x Given, f ( x ) = x g( x ) - 12 x + 1
Þ 2 sin x(cos2 x + sin 2 x ) = 0 = x 4 - 6 x 3 + 11 x 2 + 6 x - 12 x + 1
Þ either sin x = 0 or tan2 x = -1 = x 4 - 6 x 3 + 11 x 2 - 6 x + 1
Þ x = 0, p, 3 p / 8, 7 p / 8 = ( x 2 + 1 ) 2 - 2 x 2 + 11 x 2 - 6 x 3 - 6 x
\ Number of solution = 4 = ( x 2 + 1 ) 2 - 6 x( x 2 + 1 ) + (3 x ) 2
Sol. (Q. Nos. 87 to 89) = ( x 2 + 1 - 3 x ) 2 = {h( x )} 2, given
87. We have, h ¢¢( x ) = 6 x - 4 Q h( 0 ) = 1
Þ h ¢( x ) = 3 x 2 - 4 x + c \ h( x ) = x 2 - 3 x + 1
As, h¢(1 ) = 0 Þ 0 = -1 + c or c = 1
90. f (x ) = (x 2 - 3x + 1)2
So, h ¢( x ) = 3 x 2 - 4 x + 1
Q f ¢( x ) = 2( x 2 - 3 x + 1 )(2 x - 3 ) = 0
Þ h( x ) = x 3 - 2 x 2 + x + k
3 3± 5
Also, h(1 ) = 5 Þ k = 5 Þ x= ,
2 2
\ h( x ) = x 3 - 2 x 2 + x + 5
+ +
Now, h¢(2 ) = 5 – –
3– 5 3/2 3+ 5
\ The equation of tangent at m(2, 7 ) to y = h( x ), is 2 2
(g - 7) = 5 (x - 2)
Þ 5x - y = 3 Clearly, f ( x ) has local maxima at x = 3 / 2 and local minima at
3± 5
88. Also, g( f ( x )) = 0, " x Î R x=
2
2
2 æ x 4 2x 3 x 2 ö \ f ( x ) has exactly one local maxima and two local minima
\ Required area = ò h( x ) dx = ç - + + 5x ÷
0 è 4 3 2 ø0 91. We have,
16 g( x ) = x 3 - 6 x 2 + 11 x + 6
= 4- + 2 + 10 = 32 / 3
3 g ¢( x ) = 3 x 2 - 12 x + 11
Y
= 3( x - 2 ) 2 - 1 = 3 [( x - 2 ) 2 - 1 / 3 ]
æ 1 ö æ 1 ö
\ g ¢( x ) > 0 Þ x Î ç -¥, 2 - ÷ È ç2 + , ¥÷
è 3ø è 3 ø
æ 1 1 ö
and g ¢( x ) < 0 Þ x Î ç 2 - ,2+ ÷
X è 3 3ø
0 1 x=1 x=2x
x=
3
542 Textbook of Differential Calculus

\ g( x ) monotonically increases for æ 1ö æ 1ö æ 1 öæ 1ö


æ 1 ö æ ö (C) Let y = ç1 + ÷ ç1 + ÷ = ç1 + ÷ ç1 + ÷
x Î ç -¥, 2 - ÷ È ç2 +
1
, ¥ ÷ and monotonically decreases è aø è bø è 1 -bø è bø
è 3 ø è 3 ø
Þb 2 (y - 1 ) + b (1 - y ) + 2 = 0
æ 1 1 ö
for x Î ç2 - ,2+ ÷ Then, (1 - y ) 2 - 8 (y - 1 ) £ 0
è 3 3ø
For x Î[1, 3 ] Þ 1 £ y £3
g( x ) = ( x - 1 )( x - 2 )( x - 3 ) + 12 x+2
(D) y = 2
Þ g(1 ) = 12 and g(3 ) = 12 2x + 3x + 6
\ By Rolle’s theorem in [1, 3], we have g ¢(c ) = 0 Þ 2yx 2 + (3y - 1 ) x + 6y - 2 = 0
1
Þ c =2± [both Î(1, 3 )] (3y - 1 ) 2 - 4 ´ 2y (6y - 2 ) ³ 0
3
\ There exists two distinct tangents to the curve y = g( x ) Þ (3y - 1 ) (13y + 1 ) £ 0
which are parallel to the chord joining (1, g(1 )) and (3, g(3 )). -1 1
Þ £y £
For x Î[ 0, 4 ] 13 3
g( 0 ) = 6 and g( 4 ) = 18 é1 ù
94. (A) 3 x = t Þ Q (t ) = 2t 3 - 4t 2 + 2t in t Î ê , 3ú
\ By LMVT ë3 û
g( 4 ) - g( 0 ) 18 - 6 æ1 ö
g ¢(c ) = = =3 ⇒ Q ¢(t ) < 0 in ç , 1 ÷ and Q ¢(t ) > 0 in (1, 3) ⇒ Q (t ) min = Q (1 ) = 0
4-0 4 è3 ø
\ 3c 2 - 12c + 11 = 3 (B) Take x 2 + x = t Þ Q (t ) = t (t - 2 ) Þ Q (t ) min = - 1
Þ 3c 2 - 12c + 8 = 0 3 3
(C) ò | x - 2 | dx = 5 and ò [ x ]dx = 2 Þ I = 3
Þ c = 2 ± 2/ 3 [both Î( 0, 4 )] -1 -1
p
\ There exists exactly two distinct lagrange’s mean value in (D) Period of sin36 x =
(0, 4) for y = g( x ). 18
p p
æ 3ö 5
2
Period of tan 42 x = ⇒ p=
92. We have, h( x ) = x 2 - 3x + 1 = ç x - ÷ - 42 6
è 2ø 4
The curve y = h( x ) is an upward parabola, intersecting X -axis 95. (A) f ¢( x ) = 8 x ( x - 1)( x + 1) [positive factor]
at two distinct points. ⇒ Local maxima at x = 0
\ h( x ) has exactly one critical point (i.e. the vertex) and no (B) Clearly, we see (p), (r), (s)
point of inflexion. Y
Also, h( x ) = 0
3± 5
Þ x= [both Î( 0, 3 )]
2
\ h( x ) = 0 has exactly two distinct zeroes in (0, 3). X
0 2
93. (A) y = 10 - (10 - x ), - 1 £ x £ 3
=x æ1ö
-
æ1ö æ1ö
+

\ Maximum value = 3 (C) f ç ÷ > f ç ÷ < f ç ÷


è2ø è2ø è2ø
(B) Equation of tangent at (t 2, 2t )
1
Also, f ( x ) is non-differentiable at x =
ty = x + t 2 2
this tangent means Y -axis at Q ( 0, t ) ⇒ Clearly, we see (p), (s).
æ -1 ö Y
Here, QS ´ PQ = t ´ ç ÷ = - 1
è t ø 3
t 2 2t 1 2
1
Area of triangle PQS , D = 0 t 1
2
1 0 1
X
0 2 3 4
1
Þ D = [3t 2 + t ] (D) Clearly, we see (q), (s).
2
dD
³0
for Î [ 0, 2 ] , 96.(A) f ¢( x ) = - p sin px + 10 + 6x + 3x 2
dt
D is maximum at t = 2 = 3 ( x + 1 ) 2 + 7 - p sin px > 0 for all x
1
\ max D = [2 3 + 2 ] = 5 ∴ f ( x ) is minimum in - 2 £ x £ 3 . So, the absolute minimum
2 = f ( - 2 ) = - 15
Chap 08 Monotonicity, Maxima and Minima 543

1 æ pö æ pö p
(B) f ¢( x ) = 2 x + 1 . Therefore, for -1 £ x < - , we get 2 sin ç x + ÷ cos x sin ç2 x + ÷ + sin
2 è 6ø è 6ø 6
98. y = =
1
f ¢( x ) < 0 and for - < x £ 1, we get f ¢( x ) > 0 æ pö æ pö p
2 sin x cos ç x + ÷ sin ç2 x + ÷ - sin
2 è 6ø è 6ø 6
é 1ù 1
∴ f ( x ) is minimum decrease in f ê - 1, - ú × =1 +
ë 2û æ pö p
sin ç2 x + ÷ - sin
æ 1 ù è 6ø 6
and minimum decrease in ç - , 1 ú ×
è 2 û Y
2
æ 1ö æ 1ö æ 1ö 3
∴ min f ( x ) = f ç - ÷ = ç - ÷ + ç - ÷ + 1 =
è 2ø è 2ø è 2ø 4 Local minima
(C) f ¢( x ) = 4 ( x 2 - 1 ). So, for 0 £ x < 1 , we get 3

f ¢( x ) < 0 , i.e. f ( x ) is monotonically decreasing and for


1 < x £ 2, we get f ¢( x ) > 0, f ( x ) is monotonically Local maxima
increasing. 1/3
4 8
∴ min f ( x ) = f (1 ) = - 4 = - × X
3 3 0 π/3 π/2 2π/3 5p/6 π
π/6
(D) f ¢( x ) = - 12 + 18 x - 6 x 2
= - 6 ( x 2 - 3 x + 2 ) = - 6 ( x - 1 )( x - 2 ) p p p
y is minimum, if 2 x + = Þ x=
∴ f ¢( x ) > 0, if 1 < x < 2 6 2 6
and f ¢( x ) < 0, if 2 < x £ 4 Þ y min = 1 + 2 = 3
∴ f ( x ) is monotonically increasing in 1 < x < 2 and 1
99. Let A = (t, t 2 ), mOA = t, mAB = -
monotonically decreasing in 2 < x £ 4 . t
1
∴ Absolute maximum = The greatest among { f (1 ), f (2 )} Equation of AB, y - t = - ( x - t )
2
t
= The greatest among {1, 2 } = 2
Put x = 0, h = t 2 + 1
dT
97. = k (T - 5 ) 2
dt Now, lim (h ) = lim (1 + t 2 ) = 1 [as x ® 0, then t ® 0]
x®0 t ®0
dT
= k dt
(T - 5 ) 2 100. For 0 < a £ 1 the line always intersects y = a x
Y
(T - 5 ) -1
Þ = kt + c
-1
1 y = ax,∈ a(0,1)
\ - ( 40 - 5 ) -1 = c i.e. c=- X
35 O
1 1
\ - = kt - Y
T -5 35
After 15 min a=1
1 1
- = 15k - X
30 - 5 35 0
1 æ1 1ö - 10 -2
\ k= ç - ÷= = For a > 1 say a = e consider f ( x ) = e x - x
15 è 35 25 ø 15 ´ 35 ´ 25 75 ´ 35
Temperature after 60 min is given by f ¢( x ) = e x - 1

æ -2 ö 1 f ¢( x ) > 0 for x > 0 and f ¢( x ) < 0 for x < 0


1
- = 60 ç ÷- \ f ( x ) is increasing (­) for x > 0 and decreasing (¯) for x < 0
T -5 è 75 ´ 35 ø 35
I I
1 120 + 75 195
Þ = +5=
T - 5 75 ´ 35 75 ´ 35
x
75 ´ 35 5 ´ 35 175 x=0
Þ T = +5= +5 = +5
195 13 13 y = e x always lies above y = x, i.e. e x - x ³ 1 for a > 1, hence
Þ [T ] = 18
never intersects Þ a Î ( 0, 1 ]
\ [T ]/ 2 = 18 / 2 = 9
544 Textbook of Differential Calculus

101.
dy
=-
4 cos x
+
cos x
= 0 gives sin x =
2 Similarly, for x Î ( 4, ¥ ) both g( x ) and g( x + 12 ) are decreasing,
dx sin 2 x (1 - sin x ) 2 3 hence maximum value can’t occur in this interval.
p - So, now for all values of x Î ( -8, 4 )
note that f ( x ) ® ¥ as x ® 0 + or x ® and between two 1 1
2 f (x ) = +
maxima, we have a minima. x +9 5-x

a=
4
+
1
=9 1 1 ( x + 9 ) 2 - (5 - x ) 2
So, Þ f ¢( x ) = - + =
2/3 1 -2/3 (x + 9) 2
(5 - x ) 2
( x + 9 ) 2(5 - x ) 2
102. Y
14(2 x + 4 )
A \ f ¢( x ) =
( x + 9 ) 2(5 - x ) 2
– +
B C –2=x
X
0 2
1 \ Minimum at x = -2, and maximum occurs at x = + 4 or
x = -8.
A, B, C are the 3 critical points of y = f ( x ). 14 p
Here, f ( 4 ) = f ( -8 ) = =
103. Only at A and E, f ( x ) = 0 but does not change sign. 13 q
104. Note that f is defined for x > 0 \ (p - q ) = 1
1 p 1
f ¢( x ) = cos 2 x = 0 Þ x = np ± 109. Here, to find the least value of a Î R, for which 4 ax 2 + ³ 1,
2 4 x
for all x > 0.
105. Obviously, f is increasing and g is decreasing in ( x1, x 2 ), hence 1
i.e. to find the minimum value of a when y = 4 ax 2 + ; x >0
f ( g( a 2 - 2 a )) > f ( g(3 a - 4 )) as f is increasing x
attains minimum value of a.
Þ g( a 2 - 2 a ) > g(3 a - 4 ) dy 1
\ = 8 ax - 2 …(i)
\ a 2 - 2 a < 3 a - 4 as g is decreasing. dx x
a2 - 5a + 4 < 0 d 2y 2
Now, = 8a + 3 …(ii)
Þ ( a - 1 ) ( a - 4 ) < 0 Þ a Î(1, 4 ) dx 2 x
dy
t + 3x - x 2 ( x - 4 ) (3 - 2 x ) - (t + 3 x - x 2 ) When = 0, then 8 x 3a = 1
106. f ( x ) = , f ¢( x ) = dx
x-4 (x - 4)2 21/ 3
æ 1 ö d y
For maximum or minimum, f ¢( x ) = 0 At x = ç ÷ , 2 = 8 a + 16 a = 24 a, Thus, y attains
è 8 a ø dx
- 2 x 2 + 11 x - 12 - t - 3 x + x 2 = 0 minimum when
1/ 3
- x 2 + 8 x - (12 + t ) = 0 æ 1 ö
x = ç ÷ ; a > 0.
For one M and m, D>0 è 8a ø
1/ 3
64 - 4 (12 + t ) > 0 æ 1 ö
\ y attains minimum when x = ç ÷ .
16 - 12 - t > 0 Þ 4 > t or t < 4 è 8a ø
2/ 3
Hence, the greatest value of t is 3. æ 1 ö
i.e. 4a ç ÷ + (8 a )1/3 ³ 1
107. Let g( x ) = f ¢( x )sin( p f ( x )) ...(i) è 8a ø
\ g ¢( x ) = p ( f ¢( x )) 2 cos [ p f ( x )] + sin( p f ( x )) × f ¢¢( x ) ...(ii) Þ a1/3 + 2 a1/3 ³ 1
Here, g(a ) = 0 = g(b )
g( a ) = g(b ) = 0 1
and Þ 3 a1/3 ³ 1 Þ a ³
æ a + bö 27
gç ÷=0 1
è 2 ø Hence, the least value of a is .
27
\ According to Rolle’s theorem,
æ a + b ö æa + b ö
110. Plan The given equation contains algebraic and trigonometric
g ¢( x ) = 0 has atleast one root in ( a, a ), ça, ÷, ç , b÷ functions called transcendental equation. To solve
è 2 ø è 2 ø
transcendental equations we should always plot the graph for
and (b, b ), i.e. minimum of 4 roots. LHS and RHS.
1 Here, x 2 = x sin x + cos x
108. Let g( x ) =
| x - 4| + 1 Y x2
1
Þ g( x + 12 ) =
| x + 8| + 1
X′ X
When, x < -8 both g( x ) and g( x + 12 ) are increasing, hence O
maximum value can’t occur in this interval. Y′
Chap 08 Monotonicity, Maxima and Minima 545

æ 1ö æ1 ö
Let f ( x ) = x 2 and g( x ) = x sin x + cos x - çx + ÷ - ç + x÷
è xø è ø
e æ -1 ö e x
We know that, the graph for f ( x ) = x 2 f ¢( x ) = 1 × -ç 2÷
x èx ø 1/x
To plot, g( x ) = x sin x + cos x æ 1ö æ 1ö æ 1ö
- çx + ÷ - çx + ÷ - çx + ÷
g ¢( x ) = x cos x + sin x - sin x e è xø
e 2e è xø è xø
g ¢( x ) = x cos x …(i) = + =
x x x
g ¢¢( x ) = - x sin x + cos x …(ii)
As f ¢( x ) > 0, "x Î ( 0, ¥ )
Put g ¢( x ) = 0 Þ x cos x = 0
\ f ( x ) is monotonically increasing on ( 0, ¥ ).
p 3p 5p 7p
\ x = 0, , , , Þ Option (a) is correct and option (b) is wrong.
2 2 2 2 æ 1ö æ 1ö
- çt + ÷ - çt + ÷
è tø 1/ x e è t ø
g(x) æ1ö x e
Now, f ( x ) + f ç ÷ = ò dt + ò dt
π/2 è x ø 1/x t x t
= 0, "x Î ( 0, ¥ )
æ 1ö
- çt + ÷
2x è tø
e
x Now, let g( x ) = f (2 x ) = ò - x dt
–5π/2 –π –π/2O π/2 π 5π/2 2 t
æ 1ö
- çt + ÷
2- x è tø
Y′ -x e
g ( - x ) = f (2 ) = ò x dt = -g ( x )
3p 7p t 2
At x = 0, , ,..., f ¢¢( x ) > 0, so minimum
2 2 \ f (2 x ) is an odd function of x on R.
p 5p 9p
At x = , , ,..., f ¢( x ) < 0, so maximum 113. Plan
2 2 2
ì x, if x ³ 0
So, graph of f ( x ) and g( x ) are shown as We know that, | x | = í
î - x, if x < 0
Y
f (x) ì x - a, if x ³ a
g(x) Þ | x - a| = í
î - ( x - a ), if x < a
and for non-differentiable continuous function, the maximum or
X′ minimum can be checked with graph as
X
–π/2 O π/2 Y Y
Y′

So, number of solutions are 2.


f ( x ) × g( x )
111. Here, lim =1
x ® 2 f ¢( x ) × g ¢( x )

f ( x ) g ¢( x ) + f ¢( x ) g ( x ) X X
Þ lim = 1 [using L’Hospital’s rule] O x=a x=a
x ® 2 f ¢¢( x ) g ¢( x ) + f ¢( x ) g ¢¢( x ) Minimum at x = a Maximum at x = a
f (2 ) g ¢(2 ) + f ¢(2 ) g(2 )
Þ =1 Y
f ¢¢(2 ) g ¢(2 ) + f ¢(2 ) g ¢¢(2 )
f (2 ) g ¢(2 )
Þ =1 [Q f ¢(2 ) = g(2 ) = 0]
f ¢¢(2 ) g ¢(2 )
Þ f (2 ) = f ¢¢(2 ) ...(i)
\ f ( x ) – f ¢¢( x ) = 0 for atleast one x Î R.
Þ Option (d) is correct. O
X
x=a
Also, f : R ® ( 0, ¥ )
Neither maximum
Þ f (2 ) > 0 nor minimum at x = a
\ f ¢¢(2 ) = f (2 ) > 0 [from Eq. (i)]
Since, f ¢(2 ) = 0 and f ¢¢(2 ) > 0 Here, f ( x ) = 2 | x | + | x + 2 | - || x + 2 | - 2 | x ||
\ f ( x ) attains local minimum at x = 2.
ì - 2 x - ( x + 2 ) + ( x - 2 ), if when x £ - 2
Þ Option (a) is correct. ï - 2 x + x + 2 + 3 x + 2, if when - 2 < x £ -2 / 3
æ 1ö ï
- çt + ÷
è tø ï 2
x e =í - 4 x, if when - < x £ 0
112. Given, f ( x ) = ò1 dt ï 3
x
t
ï 4 x, if when 0 < x £ 2
ïî 2 x + 4, if when x > 2
546 Textbook of Differential Calculus

ì - 2 x - 4, if x £ -2 æ d 2v ö
ï 2 x + 4, At x = 3, ç 2 ÷ = 2(30 - 69 ) < 0
if -2 < x £ - 2 / 3 è da ø
ï
ï 2
= í - 4 x, if - <x£0 5
3 \ Maximum when x = 3, also at x =
ï 6
ï 4 x, if 0 < x £2
ïî 2 x + 4, if x >2 æ d 2v ö
Þ ç 2÷ > 0
è da ø
Graph for y = f ( x ) is shown as
\ At x = 5 / 6, volume is minimum.
Thus, sides are 8 x = 24 and 15 x = 45
2x + 4
8 115. Plan As to maximise or minimise area of triangle, we should
find area in terms of parametric coordinates and use second
–2x– 4 derivative test.
4x
4 8/3 Here, tangent at P(2 cos q, 3 sin q) is
+
2x – 4x

–2 –2/3 0 2 P (2 cos θ, √3 sin θ)

114. Plan The problem is based on the concept to maximise volume O (h,0) R
of cuboid, i.e. to form a function of volume, say f ( x ) find f ¢( x ) (2 sec θ, 0)
Q
and f ¢¢( x ).
(2 sec θ, –√3 sin θ)
Put f ¢( x ) = 0 and check f ¢¢( x ) to be + ve or - ve for minimum
and maximum, respectively. x y
cos q + sin q = 1
Here, l = 15 x - 2a, b = 8 x - 2a and h = a 2 3
a \ R(2 sec q, 0 )
Þ D = Area of DPQR
a 1
= (2 3 sin q) (2 sec q - 2 cos q)
2
8x–2a 8x = 2 3 × sin 3 q/ cos q …(i)
15x – 2a 1
Since, £h £1
2
1
15x \ £ 2 cos q £ 1
2
\ Volume = (8 x - 2a ) (15 x - 2a ) a 1 1
Þ £ cos q £ …(ii)
V = 2a × ( 4 x - a ) (15 x - 2a ) …(i) 4 2
dD 2 3 {cos q × 3 sin 2 q cos q - sin 3 q ( - sin q)}
\ =
dq cos2 q
2 3 × sin 2 q 2 3 sin 2 q
a = [3 cos2 q + sin 2 q] = × [2 cos2 q + 1 ]
cos q2
cos2 q
8x – 2a
15x – 2a = 2 3 tan 2 q (2 cos2 q + 1 ) > 0
1 1
On differentiating Eq. (i) w.r.t. a, we get When £ cos q £ ,
4 2
dv
= 6a 2 - 46ax + 60 x 2 1 æ 2 3 × sin 3 q ö
da \ D1 = D max occurs at cos q = = ç ÷
4 è cos q ø
d 2v
Again, differentiating, we get 2 = 12a - 46 x 1 45 5
da When cos q = =
æ dv ö 4 8
Here, ç ÷=0 1 æ 2 3 sin 3 q ö
è da ø
D 2 = D min occurs at cos q = =ç ÷
Þ 6 x 2 - 23 x + 15 = 0 2 è cos q ø
5 1 9
At a = 5, x = 3, When cos q = =
6 2 2
æ d 2v ö 8
\ D1 - 8 D 2 = 45 - 36 = 9
Þ ç 2 ÷ = 2 (30 - 23 x ) 5
è da ø
Chap 08 Monotonicity, Maxima and Minima 547

f ( x ) = x 3 + bx 2 + cx + d
2 2 2 2
116. Given function, f ( x ) = e x + e - x , g ( x ) = xe x + e - x and 120. Given,
2 2
h ( x ) = x 2e x + e - x are strictly increasing on [0, 1]. Þ f ¢( x ) = 3 x 2 + 2bx + c
Hence, at x = 1, the given function attains absolute maximum As we know that, if ax 2 + bx + c > 0, "x, then a > 0 and D < 0.
all equal to e + 1 / e . Now, D = 4b 2 - 12c = 4(b 2 - c ) - 8c
Þ a =b =c [where, b 2 - c < 0 and c > 0]
ìï(2 + x ) 3 , if - 3 < x £ -1 \ D = (–ve) or D < 0
117. Given, f (x ) = í
îï x
2 /3
, if - 1 < x < 2 Þ f ¢( x ) = 3 x 2 + 2bx + c > 0 "x Î ( - ¥, ¥ ) [as D < 0 and a > 0]
ì3 ( x + 2 ) 2 , if - 3 < x < -1 Hence, f ( x ) is strictly increasing function.
ï 121. Given f ( x ) = x 2 + 2bx + 2c 2 and g( x ) = - x 2 - 2cx + b 2
Þ f ¢( x ) = í 2 - 1
ï x 3 , if - 1 < x < 2 Then, f ( x ) is minimum and g( x ) is maximum at
î3
y æ -b -D ö
çx = and f ( x ) = ÷ , respectively.
è 4a 4a ø
-( 4b 2 - 8c 2 )
\ min f ( x ) = = (2c 2 - b 2 )
x 4
–3 –2 –1 1 2 ( 4c 2 + 4b 2 )
and max g( x ) = - = (b 2 + c 2 )
4( -1 )
Now, min f ( x ) > max g( x )
Þ 2c 2 - b 2 > b 2 + c 2 Þ c 2 > 2b 2 Þ | c | > 2 | b |
Clearly, f ¢( x ) changes its sign at x = -1 from +ve to -ve and so
f ( x ) has local maxima at x = -1 . 122. Let f ( x ) = 3 sin x - 4 sin 3 x = sin 3x
Also, f ¢( 0 ) does not exist but f ¢( 0 - ) < 0 and f ¢( 0 + ) > 0. It can The longest interval in which sin x is increasing is of length p .
only be inferred that f ( x ) has a possibility of a minima at So, the length of largest interval in which f ( x ) = sin 3 x is
x = 0. Hence, the given function has one local maxima at p
increasing is .
x = -1 and one local minima at x = 0 . 3
p 123. Given, cot a1 . cot a 2. ... . cot an = 1
118. Given, g(u ) = 2 tan -1(eu ) - for u Î( -¥, ¥ )
2 cos a1 . cos a 2 . cos a 3 cos an
p p Þ ×…× =1
g( -u ) = 2 tan (e ) - = 2 (cot -1(eu )) -
-1 - u sin a1 sin a 2 sin a 3 sin an
2 2
Let cos a1 . cos a 2 . cos a 3 . … × cos an = k …(i)
æp ö p
= 2 ç - tan -1(eu ) ÷ - and sin a1 . sin a 2 . sin a 3. … ×sin an = k … (ii)
è2 ø 2
Again, on multiplying Eqs. (i) and (ii), we get
= p / 2 - 2 tan -1(eu ) = - g(u ) (cos a1. cos a 2. cos a 3. K . cos an )
\ g( -u ) = - g(u ) ´ (sin a1.sin a 2. sin a 3 . K . sin an ) = k 2
Þ g(u ) is an odd function. 1
We have, g(u ) = 2 tan -1(eu ) - p 2 k2 = (2 sin a1 cos a1 )
2 ´ 2 ´ K n times
2e u (2 sin a 2 cos a 2 )...(2 sin an cos an )
g ¢(u ) =
1 + e 2u 1
Þ k = n (sin 2 a1 )(sin 2 a 2 ) K (sin 2 an )
2

g ¢(u ) > 0, " x Î R [Q eu > 0 ] 2


1 1
So, g ¢(u ) is increasing for all x Î R. £ n sin 2 ai £ 1, "1 £ i < n Þ k £ n /2
2 2
119. Let f ( x ) = bx 2 + ax + c
ì ex, if 0 £ x £ 1
Q f (0) = 0 Þ c = 0 ï
and f (1 ) = 1 Þ a + b = 1 124. Given, f ( x ) = í2 - e x - 1 , if 1 < x £ 2
ï x - e, if 2 < x £ 3
\ f ( x ) = ax + (1 - a ) x 2 î
Also, f ¢( x ) > 0, for x Î ( 0, 1 ) and g( x ) = ò
x
f (t ) dt
Þ a + 2 (1 - a ) x > 0 0

Þ a (1 - 2 x ) + 2 x > 0 Þ g ¢( x ) = f ( x )
2x Put g ¢( x ) = 0 Þ x = 1 + loge 2 and x = e.
Þ a> Þ 0 < a < 2, since x Î( 0, 1 )
2x - 1 ìe x , if 0 £ x £ 1
ï
\ f ( x ) = ax + (1 - a ) x 2 , 0 < a < 2 Also, g ¢¢ ( x ) = í - e x - 1, if 1 < x £ 2
ï 1, if 2 < x £ 3
î
548 Textbook of Differential Calculus

At x = 1 + loge 2, g ¢¢ (1 + loge 2 ) = - e loge 2 < 0, g( x ) has a 4a (a + 2 ) 4a


Now, f ¢¢(1 ) = =
local maximum. (a + 2 ) 3 (a + 2 ) 2
Also, at x = e, 4a (a - 2 ) - 4a
and f ¢¢( -1 ) = =
g ¢¢ (e ) = 1 > 0, g( x ) has a local minima. (2 - a ) 3
(a - 2 ) 2
Q f ( x ) is discontinuous at x = 1, then we get local maxima at \ (2 + a ) 2 f ¢¢(1 ) + (2 - a ) 2 f ¢¢( -1 ) = 4a - 4a = 0
x = 1 and local minima at x = 2.
Hence, (a) and (b) are correct answers. 127. When x Î( -1,1),
125. Since, f ( x ) has local maxima at x = - 1 and f ¢( x ) has local x2 < 1
minima at x = 0. Þ x2 - 1 < 0
\ f ¢¢( x ) = lx \ f ¢( x ) < 0, f ( x ) is decreasing.
On integrating, we get 4a
Also, at x = 1, f ¢¢ (1 ) = >0 [Q0 < a < 2 ]
x2 (a + 2 ) 2
f ¢ (x ) = l +c [Q f ¢ ( - 1 ) = 0 ]
l 2 So, f ( x ) has local minimum at x = 1.
Þ + c = 0 Þ l = - 2c …(i)
2 f ¢(e x ) x é e 2x - 1 ù æ ex ö
Again, integrating on both sides, we get 128. g ¢( x ) = × e = 2a ê 2x 2ú
ç ÷
1 + (e )x 2
ë (e + ae + 1 ) û è 1 + e ø
x 2x
x3
f (x ) = l + cx + d g ¢( x ) = 0, if e 2x - 1 = 0,
6
æ8ö i.e. x=0
Þ f (2 ) = l ç ÷ + 2c + d = 18 …(ii)
è6ø If x < 0, e 2x < 1
l Þ g ¢( x ) < 0
and f (1 ) = + c + d = -1 …(iii)
6
129. To maximize area of D APB, we know that, OP = 10
From Eqs. (i), (ii) and (iii), we get
and sin q = r / 10 , where q Î( 0, p / 2 ) … (i)
1
f ( x ) = (19 x 3 - 57 x + 34 ) Y
4
1 57 P(6,8)
\ f ¢( x ) = (57 x 2 - 57 ) = ( x - 1 ) ( x + 1 )
4 4 θ
For maxima or minima, put f ¢( x ) = 0
A
Þ x = 1, - 1 θ
1 r
Now f ¢¢( x ) = (114 x ) X′ Q
4 O X
At x = 1 , f ¢¢( x ) > 0, minima
At x = - 1, f ¢¢( x ) < 0 , maxima B
\ f ( x ) is increasing for [1, 2 5 ] .
\ f ( x ) has local maxima at x = - 1 and f ( x ) has local minima Y′
at x = 1. 1
\ Area = (2 AQ ) ( PQ )
Also, f ( 0 ) = 34 / 4 2
Hence, (b) and (c) are the correct answers. = AQ . PQ = (r cos q) (10 - OQ )
( x 2 + ax + 1 ) - 2ax 2ax = (r cos q) (10 - r sin q)
126. f (x ) = =1 - 2
x 2 + ax + 1 x + ax + 1 = 10 sin q cos q (10 - 10 sin 2 q) [from Eq. (i)]
é ( x 2 + ax + 1 ) × 2a - 2ax(2 x + a ) ù Þ A = 100 cos3 q sin q
f ¢( x ) = - ê ú
ë ( x 2 + ax + 1 ) 2 û Þ
dA
= 100 cos4 q - 300 cos2 q × sin 2 q
é -2ax 2 + 2a ù é (x 2 - 1) ù dq
= –ê 2 2ú
= 2a ê 2 2ú
...(i) dA
=0
ë ( x + ax + a ) û ë ( x + ax + 1 ) û
Put
dq
é ( x 2 + ax + 1 ) 2 (2 x ) - 2( x 2 - 1 ) ù Þ cos2 q = 3 sin 2 q
ê ú
( x 2 + ax + 1 )(2 x + a ) ú Þ tan q = 1 / 3
f ¢¢( x ) = 2a êê ú
( x 2 + ax + 1 ) 4 Þ q = p /6
ê ú
êë úû dA
At which < 0, thus when q = p /6, area is maximum
dq
é 2 x ( x + ax + 1 ) - 2 ( x - 1 )(2 x + a ) ù
2 2
= 2a ê ú …(ii) p
( x 2 + ax + 1 ) 3 From Eq. (i), r = 10 sin = 5 units
ë û 6
Chap 08 Monotonicity, Maxima and Minima 549

x2 y 2 3 3
ò1p ¢(x )dx = ò1 l(x - 4 x + 3 ) dx
2
130. Let us take a point P( 6 cos q, 3 sin q) on + =1.
6 3 3
Now, to minimise the distance from P to given straight line æ x3 ö
Þ ( p( x ))13 = l ç - 2 x 2 + 3 x ÷
x + y = 7, shortest distance exists along the common normal. è 3 ø1
Y æ æ1 öö
Þ p (3 ) - p (1 ) = l ç(9 - 18 + 9 ) - ç - 2 + 3 ÷ ÷
y è è3 øø
N
x+y=7 ì ü- 4
P Þ 2 - 6 = lí ý Þ l = 3
X′
O
X î 3 þ
Þ p ¢( x ) = 3 ( x - 1 )( x - 3 )
\ p¢( 0 ) = 9
133. Let g ( x ) = e , " x Î R Þ g ¢( x ) = e f ( x ) × f ¢( x )
f (x )
Y′
a 2 / x1 6 sec q Þ f ¢( x ) changes its sign from positive to negative in the
Slope of normal at P = = = 2 tan q = 1 neighbourhood of x = 2009
b 2 / y1 6 cosec q
Þ f ( x ) has local maxima at x = 2009.
2 1
So, cos q =
and sin q = So, the number of local maximum is one.
3 3 1
Hence, required point is P(2, 1 ). 134. Let f ( q) =
sin 2 q + 3 sin q cos q + 5 cos2 q
131. Plan Again let, g ( q) = sin 2 q + 3 sin q cos q + 5 cos2 q
(i) Local maximum and local minimum are those points at 1 - cos2 q æ 1 + cos2 q ö 3
which f ¢( x ) = 0, when defined for all real numbers. = + 5ç ÷ + sin 2 q
2 è 2 ø 2
(ii) Local maximum and local minimum for piecewise 3
= 3 + 2 cos2 q + sin 2 q
functions are also been checked at sharp edges. 2
x, if x ³ 0
Description of Situation y = | x | = ìí - x, if x < 0
9 5 1
\ g ( q) min = 3 - 4 + = 3 - =
î 4 2 2
ì( x 2 - 1 ), if x £ - 1 or x ³ 1 1
Also, y = | x 2 - 1| = í \ Maximum value of f ( q) = =2
î (1 - x ), if - 1 £ x £ 1
2 12
ì - x + 1 - x 2, if x £ - 1 135. Given, A = { x | x 2 + 20 £ 9x } = { x | x Î [ 4,5]}
ï
ï - x + 1 - x , if - 1 £ x £ 0
2
y = | x | + | x 2 - 1| = í Y
ïx +1-x , if 0 £ x £1
2

ïî x + x 2 - 1, if x ³ 1
O 2 3 4 X
ì - x - x + 1, if x £ - 1
2
5
ï
ï - x - x + 1, if - 1 £ x £ 0
2
–16

ï - x + x + 1, if 0 £ x £ 1
2 –20
–21
ïî x 2 + x - 1, if x ³1
which could be graphically shown as
Y
Now, f ¢( x ) = 6( x 2 - 5 x + 6 )
– x 2– x + 1 – x 2– x + 1 – x 2+ x +1 x 2+x –1 Put f ¢( x ) = 0 Þ x = 2, 3
f (2 ) = - 20, f (3 ) = - 21, f ( 4 ) = - 16, f (5 ) = 7
1 From graph, maximum value of f ( x ) on set A is f (5 ) = 7.
X 136. According to given information, we have
–1 –1/2 O 1/2 1
1 -1 Perimeter of square + Perimeter of circle = 2 units
Thus, f ( x ) attains maximum at x = , and f ( x ) attains
2 2 Þ 4 x + 2 pr = 2
minimum at x = -1, 0,1. 1 - 2x
Þ r= ...(i)
Þ Total number of points = 5 p
Now, let A be the sum of the areas of the square and the circle.
132. Plan If f ( x ) is least degree polynomial having local maximum
and local minimum at a and b. (1 - 2 x ) 2
Then, A = x 2 + pr 2 = x 2 + p
Then, f ¢( x ) = l ( x - a )( x - b ) p2
Here, p ¢( x ) = l ( x - 1 )( x - 3 ) = l( x 2 - 4 x + 3 ) (1 - 2 x ) 2
Þ A( x ) = x 2 +
On integrating both sides between 1 to 3, we get p
550 Textbook of Differential Calculus

Now, for minimum value of A( x ),


dA
=0 (ii) The function f has extrema at x = - 1 and x = 2 , i.e.,
dx f ¢( -1 ) = f ¢(2 ) = 0 and f ¢¢( -1 ) ¹ 0 ¹ f ¢¢(2 ).
2 (1 - 2 x ) 2 - 4x Now, given function f is given by
Þ 2x + × (- 2) = 0 Þ x =
p p f ( x ) = log | x | + bx 2 + ax
Þ px + 4 x = 2 1 -1
2 Þ f ¢( x ) = + 2bx + a Þ f ¢¢( x ) = 2 + 2b
Þ x= ...(ii) x x
p+4
Since, f has extrema at x = - 1 and x = 2.
Now, from Eq. (i), we get Hence, f ¢( -1 ) = 0 = f ¢(2 )
2
1 -2× f ¢( - 1 ) = 0
p+4 p+4-4 1 Þ a - 2b = 1
r= = = ...(iii) …(i)
p p(p + 4 ) p + 4 and f ¢(2 ) = 0
From Eqs. (ii) and (iii), we get x = 2r -1
Þ a + 4b = …(ii)
137. Here, x = -1 and x = 2 are extreme points of 2
f ( x ) = a log| x | + b x 2 + x , then On solving Eqs. (i) and (ii), we get
a 1 -1
f ¢( x ) = + 2 b x + 1 a= and b =
x 2 4
\ f ¢( - 1 ) = - a - 2 b + 1 = 0 …(i) -1 -1 æ x2 + 2ö
Þ f ¢¢( x ) = 2 + =-ç ÷
[at extreme point, f ¢( x ) = 0] x 2 è 2x 2 ø
a
and f ¢( - 2 ) = + 4b + 1 = 0 …(ii) Þ f ¢¢( -1 ) < 0 and f ¢¢(2 ) < 0
2
Hence, f has local maxima at both x = - 1 and x = 2.
On solving Eqs. (i) and (ii), we get
Hence, Statement I is correct.
1
a = 2, b = - Also, while solving for Statement I, we found the values of a
2
138. Given and b, which justify that Statement II is also correct.
(i) A function f, such that However, Statement II does not explain Statement I in any
f ( x ) = log | x | + bx 2 + ax, x ¹ 0 way.

You might also like